You are on page 1of 2322

Introduction to Methods of Applied Mathematics

or
Advanced Mathematical Methods for Scientists and Engineers

Sean Mauch
http://www.its.caltech.edu/sean

January 24, 2004


Contents

Anti-Copyright xxiv

Preface xxv
0.1 Advice to Teachers . . . . . . . . . . . . . . . . . . . . . . . . . . . . . . . . . . . . . . . . . . . . . xxv
0.2 Acknowledgments . . . . . . . . . . . . . . . . . . . . . . . . . . . . . . . . . . . . . . . . . . . . . xxv
0.3 Warnings and Disclaimers . . . . . . . . . . . . . . . . . . . . . . . . . . . . . . . . . . . . . . . . . xxvi
0.4 Suggested Use . . . . . . . . . . . . . . . . . . . . . . . . . . . . . . . . . . . . . . . . . . . . . . . xxvii
0.5 About the Title . . . . . . . . . . . . . . . . . . . . . . . . . . . . . . . . . . . . . . . . . . . . . . xxvii

I Algebra 1
1 Sets and Functions 2
1.1 Sets . . . . . . . . . . . . . . . . . . . . . . . . . . . . . . . . . . . . . . . . . . . . . . . . . . . . 2
1.2 Single Valued Functions . . . . . . . . . . . . . . . . . . . . . . . . . . . . . . . . . . . . . . . . . . 4
1.3 Inverses and Multi-Valued Functions . . . . . . . . . . . . . . . . . . . . . . . . . . . . . . . . . . . . 6
1.4 Transforming Equations . . . . . . . . . . . . . . . . . . . . . . . . . . . . . . . . . . . . . . . . . . 9
1.5 Exercises . . . . . . . . . . . . . . . . . . . . . . . . . . . . . . . . . . . . . . . . . . . . . . . . . . 11
1.6 Hints . . . . . . . . . . . . . . . . . . . . . . . . . . . . . . . . . . . . . . . . . . . . . . . . . . . . 14
1.7 Solutions . . . . . . . . . . . . . . . . . . . . . . . . . . . . . . . . . . . . . . . . . . . . . . . . . . 16

i
2 Vectors 22
2.1 Vectors . . . . . . . . . . . . . . . . . . . . . . . . . . . . . . . . . . . . . . . . . . . . . . . . . . . 22
2.1.1 Scalars and Vectors . . . . . . . . . . . . . . . . . . . . . . . . . . . . . . . . . . . . . . . . 22
2.1.2 The Kronecker Delta and Einstein Summation Convention . . . . . . . . . . . . . . . . . . . . 25
2.1.3 The Dot and Cross Product . . . . . . . . . . . . . . . . . . . . . . . . . . . . . . . . . . . . 26
2.2 Sets of Vectors in n Dimensions . . . . . . . . . . . . . . . . . . . . . . . . . . . . . . . . . . . . . . 33
2.3 Exercises . . . . . . . . . . . . . . . . . . . . . . . . . . . . . . . . . . . . . . . . . . . . . . . . . . 36
2.4 Hints . . . . . . . . . . . . . . . . . . . . . . . . . . . . . . . . . . . . . . . . . . . . . . . . . . . . 38
2.5 Solutions . . . . . . . . . . . . . . . . . . . . . . . . . . . . . . . . . . . . . . . . . . . . . . . . . . 40

II Calculus 47
3 Differential Calculus 48
3.1 Limits of Functions . . . . . . . . . . . . . . . . . . . . . . . . . . . . . . . . . . . . . . . . . . . . . 48
3.2 Continuous Functions . . . . . . . . . . . . . . . . . . . . . . . . . . . . . . . . . . . . . . . . . . . 53
3.3 The Derivative . . . . . . . . . . . . . . . . . . . . . . . . . . . . . . . . . . . . . . . . . . . . . . . 56
3.4 Implicit Differentiation . . . . . . . . . . . . . . . . . . . . . . . . . . . . . . . . . . . . . . . . . . . 61
3.5 Maxima and Minima . . . . . . . . . . . . . . . . . . . . . . . . . . . . . . . . . . . . . . . . . . . . 62
3.6 Mean Value Theorems . . . . . . . . . . . . . . . . . . . . . . . . . . . . . . . . . . . . . . . . . . . 66
3.6.1 Application: Using Taylors Theorem to Approximate Functions. . . . . . . . . . . . . . . . . . 68
3.6.2 Application: Finite Difference Schemes . . . . . . . . . . . . . . . . . . . . . . . . . . . . . . 73
3.7 LHospitals Rule . . . . . . . . . . . . . . . . . . . . . . . . . . . . . . . . . . . . . . . . . . . . . . 75
3.8 Exercises . . . . . . . . . . . . . . . . . . . . . . . . . . . . . . . . . . . . . . . . . . . . . . . . . . 81
3.8.1 Limits of Functions . . . . . . . . . . . . . . . . . . . . . . . . . . . . . . . . . . . . . . . . 81
3.8.2 Continuous Functions . . . . . . . . . . . . . . . . . . . . . . . . . . . . . . . . . . . . . . . 81
3.8.3 The Derivative . . . . . . . . . . . . . . . . . . . . . . . . . . . . . . . . . . . . . . . . . . . 82
3.8.4 Implicit Differentiation . . . . . . . . . . . . . . . . . . . . . . . . . . . . . . . . . . . . . . . 84
3.8.5 Maxima and Minima . . . . . . . . . . . . . . . . . . . . . . . . . . . . . . . . . . . . . . . . 84
3.8.6 Mean Value Theorems . . . . . . . . . . . . . . . . . . . . . . . . . . . . . . . . . . . . . . . 85

ii
3.8.7 LHospitals Rule . . . . . . . . . . . . . . . . . . . . . . . . . . . . . . . . . . . . . . . . . . 85
3.9 Hints . . . . . . . . . . . . . . . . . . . . . . . . . . . . . . . . . . . . . . . . . . . . . . . . . . . . 87
3.10 Solutions . . . . . . . . . . . . . . . . . . . . . . . . . . . . . . . . . . . . . . . . . . . . . . . . . . 93
3.11 Quiz . . . . . . . . . . . . . . . . . . . . . . . . . . . . . . . . . . . . . . . . . . . . . . . . . . . . 113
3.12 Quiz Solutions . . . . . . . . . . . . . . . . . . . . . . . . . . . . . . . . . . . . . . . . . . . . . . . 114

4 Integral Calculus 116


4.1 The Indefinite Integral . . . . . . . . . . . . . . . . . . . . . . . . . . . . . . . . . . . . . . . . . . . 116
4.2 The Definite Integral . . . . . . . . . . . . . . . . . . . . . . . . . . . . . . . . . . . . . . . . . . . . 122
4.2.1 Definition . . . . . . . . . . . . . . . . . . . . . . . . . . . . . . . . . . . . . . . . . . . . . 122
4.2.2 Properties . . . . . . . . . . . . . . . . . . . . . . . . . . . . . . . . . . . . . . . . . . . . . 123
4.3 The Fundamental Theorem of Integral Calculus . . . . . . . . . . . . . . . . . . . . . . . . . . . . . . 125
4.4 Techniques of Integration . . . . . . . . . . . . . . . . . . . . . . . . . . . . . . . . . . . . . . . . . 127
4.4.1 Partial Fractions . . . . . . . . . . . . . . . . . . . . . . . . . . . . . . . . . . . . . . . . . . 127
4.5 Improper Integrals . . . . . . . . . . . . . . . . . . . . . . . . . . . . . . . . . . . . . . . . . . . . . 130
4.6 Exercises . . . . . . . . . . . . . . . . . . . . . . . . . . . . . . . . . . . . . . . . . . . . . . . . . . 134
4.6.1 The Indefinite Integral . . . . . . . . . . . . . . . . . . . . . . . . . . . . . . . . . . . . . . . 134
4.6.2 The Definite Integral . . . . . . . . . . . . . . . . . . . . . . . . . . . . . . . . . . . . . . . . 134
4.6.3 The Fundamental Theorem of Integration . . . . . . . . . . . . . . . . . . . . . . . . . . . . . 136
4.6.4 Techniques of Integration . . . . . . . . . . . . . . . . . . . . . . . . . . . . . . . . . . . . . 136
4.6.5 Improper Integrals . . . . . . . . . . . . . . . . . . . . . . . . . . . . . . . . . . . . . . . . . 137
4.7 Hints . . . . . . . . . . . . . . . . . . . . . . . . . . . . . . . . . . . . . . . . . . . . . . . . . . . . 138
4.8 Solutions . . . . . . . . . . . . . . . . . . . . . . . . . . . . . . . . . . . . . . . . . . . . . . . . . . 141
4.9 Quiz . . . . . . . . . . . . . . . . . . . . . . . . . . . . . . . . . . . . . . . . . . . . . . . . . . . . 150
4.10 Quiz Solutions . . . . . . . . . . . . . . . . . . . . . . . . . . . . . . . . . . . . . . . . . . . . . . . 151

5 Vector Calculus 154


5.1 Vector Functions . . . . . . . . . . . . . . . . . . . . . . . . . . . . . . . . . . . . . . . . . . . . . . 154
5.2 Gradient, Divergence and Curl . . . . . . . . . . . . . . . . . . . . . . . . . . . . . . . . . . . . . . . 155
5.3 Exercises . . . . . . . . . . . . . . . . . . . . . . . . . . . . . . . . . . . . . . . . . . . . . . . . . . 163

iii
5.4 Hints . . . . . . . . . . . . . . . . . . . . . . . . . . . . . . . . . . . . . . . . . . . . . . . . . . . . 166
5.5 Solutions . . . . . . . . . . . . . . . . . . . . . . . . . . . . . . . . . . . . . . . . . . . . . . . . . . 168
5.6 Quiz . . . . . . . . . . . . . . . . . . . . . . . . . . . . . . . . . . . . . . . . . . . . . . . . . . . . 177
5.7 Quiz Solutions . . . . . . . . . . . . . . . . . . . . . . . . . . . . . . . . . . . . . . . . . . . . . . . 178

III Functions of a Complex Variable 179


6 Complex Numbers 180
6.1 Complex Numbers . . . . . . . . . . . . . . . . . . . . . . . . . . . . . . . . . . . . . . . . . . . . . 180
6.2 The Complex Plane . . . . . . . . . . . . . . . . . . . . . . . . . . . . . . . . . . . . . . . . . . . . 184
6.3 Polar Form . . . . . . . . . . . . . . . . . . . . . . . . . . . . . . . . . . . . . . . . . . . . . . . . . 188
6.4 Arithmetic and Vectors . . . . . . . . . . . . . . . . . . . . . . . . . . . . . . . . . . . . . . . . . . 193
6.5 Integer Exponents . . . . . . . . . . . . . . . . . . . . . . . . . . . . . . . . . . . . . . . . . . . . . 195
6.6 Rational Exponents . . . . . . . . . . . . . . . . . . . . . . . . . . . . . . . . . . . . . . . . . . . . 197
6.7 Exercises . . . . . . . . . . . . . . . . . . . . . . . . . . . . . . . . . . . . . . . . . . . . . . . . . . 201
6.8 Hints . . . . . . . . . . . . . . . . . . . . . . . . . . . . . . . . . . . . . . . . . . . . . . . . . . . . 208
6.9 Solutions . . . . . . . . . . . . . . . . . . . . . . . . . . . . . . . . . . . . . . . . . . . . . . . . . . 211

7 Functions of a Complex Variable 239


7.1 Curves and Regions . . . . . . . . . . . . . . . . . . . . . . . . . . . . . . . . . . . . . . . . . . . . 239
7.2 The Point at Infinity and the Stereographic Projection . . . . . . . . . . . . . . . . . . . . . . . . . . 242
7.3 A Gentle Introduction to Branch Points . . . . . . . . . . . . . . . . . . . . . . . . . . . . . . . . . . 246
7.4 Cartesian and Modulus-Argument Form . . . . . . . . . . . . . . . . . . . . . . . . . . . . . . . . . . 246
7.5 Graphing Functions of a Complex Variable . . . . . . . . . . . . . . . . . . . . . . . . . . . . . . . . 249
7.6 Trigonometric Functions . . . . . . . . . . . . . . . . . . . . . . . . . . . . . . . . . . . . . . . . . . 252
7.7 Inverse Trigonometric Functions . . . . . . . . . . . . . . . . . . . . . . . . . . . . . . . . . . . . . . 259
7.8 Riemann Surfaces . . . . . . . . . . . . . . . . . . . . . . . . . . . . . . . . . . . . . . . . . . . . . 268
7.9 Branch Points . . . . . . . . . . . . . . . . . . . . . . . . . . . . . . . . . . . . . . . . . . . . . . . 270
7.10 Exercises . . . . . . . . . . . . . . . . . . . . . . . . . . . . . . . . . . . . . . . . . . . . . . . . . . 286

iv
7.11 Hints . . . . . . . . . . . . . . . . . . . . . . . . . . . . . . . . . . . . . . . . . . . . . . . . . . . . 297
7.12 Solutions . . . . . . . . . . . . . . . . . . . . . . . . . . . . . . . . . . . . . . . . . . . . . . . . . . 302

8 Analytic Functions 360


8.1 Complex Derivatives . . . . . . . . . . . . . . . . . . . . . . . . . . . . . . . . . . . . . . . . . . . . 360
8.2 Cauchy-Riemann Equations . . . . . . . . . . . . . . . . . . . . . . . . . . . . . . . . . . . . . . . . 367
8.3 Harmonic Functions . . . . . . . . . . . . . . . . . . . . . . . . . . . . . . . . . . . . . . . . . . . . 372
8.4 Singularities . . . . . . . . . . . . . . . . . . . . . . . . . . . . . . . . . . . . . . . . . . . . . . . . 377
8.4.1 Categorization of Singularities . . . . . . . . . . . . . . . . . . . . . . . . . . . . . . . . . . . 377
8.4.2 Isolated and Non-Isolated Singularities . . . . . . . . . . . . . . . . . . . . . . . . . . . . . . 381
8.5 Application: Potential Flow . . . . . . . . . . . . . . . . . . . . . . . . . . . . . . . . . . . . . . . . 383
8.6 Exercises . . . . . . . . . . . . . . . . . . . . . . . . . . . . . . . . . . . . . . . . . . . . . . . . . . 388
8.7 Hints . . . . . . . . . . . . . . . . . . . . . . . . . . . . . . . . . . . . . . . . . . . . . . . . . . . . 396
8.8 Solutions . . . . . . . . . . . . . . . . . . . . . . . . . . . . . . . . . . . . . . . . . . . . . . . . . . 399

9 Analytic Continuation 437


9.1 Analytic Continuation . . . . . . . . . . . . . . . . . . . . . . . . . . . . . . . . . . . . . . . . . . . 437
9.2 Analytic Continuation of Sums . . . . . . . . . . . . . . . . . . . . . . . . . . . . . . . . . . . . . . 440
9.3 Analytic Functions Defined in Terms of Real Variables . . . . . . . . . . . . . . . . . . . . . . . . . . 442
9.3.1 Polar Coordinates . . . . . . . . . . . . . . . . . . . . . . . . . . . . . . . . . . . . . . . . . 446
9.3.2 Analytic Functions Defined in Terms of Their Real or Imaginary Parts . . . . . . . . . . . . . . 450
9.4 Exercises . . . . . . . . . . . . . . . . . . . . . . . . . . . . . . . . . . . . . . . . . . . . . . . . . . 454
9.5 Hints . . . . . . . . . . . . . . . . . . . . . . . . . . . . . . . . . . . . . . . . . . . . . . . . . . . . 456
9.6 Solutions . . . . . . . . . . . . . . . . . . . . . . . . . . . . . . . . . . . . . . . . . . . . . . . . . . 457

10 Contour Integration and the Cauchy-Goursat Theorem 462


10.1 Line Integrals . . . . . . . . . . . . . . . . . . . . . . . . . . . . . . . . . . . . . . . . . . . . . . . . 462
10.2 Contour Integrals . . . . . . . . . . . . . . . . . . . . . . . . . . . . . . . . . . . . . . . . . . . . . . 464
10.2.1 Maximum Modulus Integral Bound . . . . . . . . . . . . . . . . . . . . . . . . . . . . . . . . 466
10.3 The Cauchy-Goursat Theorem . . . . . . . . . . . . . . . . . . . . . . . . . . . . . . . . . . . . . . . 467

v
10.4 Contour Deformation . . . . . . . . . . . . . . . . . . . . . . . . . . . . . . . . . . . . . . . . . . . 469
10.5 Moreras Theorem. . . . . . . . . . . . . . . . . . . . . . . . . . . . . . . . . . . . . . . . . . . . . . 471
10.6 Indefinite Integrals . . . . . . . . . . . . . . . . . . . . . . . . . . . . . . . . . . . . . . . . . . . . . 473
10.7 Fundamental Theorem of Calculus via Primitives . . . . . . . . . . . . . . . . . . . . . . . . . . . . . 474
10.7.1 Line Integrals and Primitives . . . . . . . . . . . . . . . . . . . . . . . . . . . . . . . . . . . . 474
10.7.2 Contour Integrals . . . . . . . . . . . . . . . . . . . . . . . . . . . . . . . . . . . . . . . . . 474
10.8 Fundamental Theorem of Calculus via Complex Calculus . . . . . . . . . . . . . . . . . . . . . . . . . 475
10.9 Exercises . . . . . . . . . . . . . . . . . . . . . . . . . . . . . . . . . . . . . . . . . . . . . . . . . . 478
10.10Hints . . . . . . . . . . . . . . . . . . . . . . . . . . . . . . . . . . . . . . . . . . . . . . . . . . . . 482
10.11Solutions . . . . . . . . . . . . . . . . . . . . . . . . . . . . . . . . . . . . . . . . . . . . . . . . . . 483

11 Cauchys Integral Formula 493


11.1 Cauchys Integral Formula . . . . . . . . . . . . . . . . . . . . . . . . . . . . . . . . . . . . . . . . . 494
11.2 The Argument Theorem . . . . . . . . . . . . . . . . . . . . . . . . . . . . . . . . . . . . . . . . . . 501
11.3 Rouches Theorem . . . . . . . . . . . . . . . . . . . . . . . . . . . . . . . . . . . . . . . . . . . . . 502
11.4 Exercises . . . . . . . . . . . . . . . . . . . . . . . . . . . . . . . . . . . . . . . . . . . . . . . . . . 505
11.5 Hints . . . . . . . . . . . . . . . . . . . . . . . . . . . . . . . . . . . . . . . . . . . . . . . . . . . . 509
11.6 Solutions . . . . . . . . . . . . . . . . . . . . . . . . . . . . . . . . . . . . . . . . . . . . . . . . . . 511

12 Series and Convergence 525


12.1 Series of Constants . . . . . . . . . . . . . . . . . . . . . . . . . . . . . . . . . . . . . . . . . . . . . 525
12.1.1 Definitions . . . . . . . . . . . . . . . . . . . . . . . . . . . . . . . . . . . . . . . . . . . . . 525
12.1.2 Special Series . . . . . . . . . . . . . . . . . . . . . . . . . . . . . . . . . . . . . . . . . . . 527
12.1.3 Convergence Tests . . . . . . . . . . . . . . . . . . . . . . . . . . . . . . . . . . . . . . . . . 529
12.2 Uniform Convergence . . . . . . . . . . . . . . . . . . . . . . . . . . . . . . . . . . . . . . . . . . . 536
12.2.1 Tests for Uniform Convergence . . . . . . . . . . . . . . . . . . . . . . . . . . . . . . . . . . 537
12.2.2 Uniform Convergence and Continuous Functions. . . . . . . . . . . . . . . . . . . . . . . . . . 539
12.3 Uniformly Convergent Power Series . . . . . . . . . . . . . . . . . . . . . . . . . . . . . . . . . . . . 539
12.4 Integration and Differentiation of Power Series . . . . . . . . . . . . . . . . . . . . . . . . . . . . . . 547
12.5 Taylor Series . . . . . . . . . . . . . . . . . . . . . . . . . . . . . . . . . . . . . . . . . . . . . . . . 550

vi
12.5.1 Newtons Binomial Formula. . . . . . . . . . . . . . . . . . . . . . . . . . . . . . . . . . . . . 553
12.6 Laurent Series . . . . . . . . . . . . . . . . . . . . . . . . . . . . . . . . . . . . . . . . . . . . . . . 555
12.7 Exercises . . . . . . . . . . . . . . . . . . . . . . . . . . . . . . . . . . . . . . . . . . . . . . . . . . 560
12.7.1 Series of Constants . . . . . . . . . . . . . . . . . . . . . . . . . . . . . . . . . . . . . . . . 560
12.7.2 Uniform Convergence . . . . . . . . . . . . . . . . . . . . . . . . . . . . . . . . . . . . . . . 566
12.7.3 Uniformly Convergent Power Series . . . . . . . . . . . . . . . . . . . . . . . . . . . . . . . . 566
12.7.4 Integration and Differentiation of Power Series . . . . . . . . . . . . . . . . . . . . . . . . . . 568
12.7.5 Taylor Series . . . . . . . . . . . . . . . . . . . . . . . . . . . . . . . . . . . . . . . . . . . . 569
12.7.6 Laurent Series . . . . . . . . . . . . . . . . . . . . . . . . . . . . . . . . . . . . . . . . . . . 571
12.8 Hints . . . . . . . . . . . . . . . . . . . . . . . . . . . . . . . . . . . . . . . . . . . . . . . . . . . . 574
12.9 Solutions . . . . . . . . . . . . . . . . . . . . . . . . . . . . . . . . . . . . . . . . . . . . . . . . . . 582

13 The Residue Theorem 626


13.1 The Residue Theorem . . . . . . . . . . . . . . . . . . . . . . . . . . . . . . . . . . . . . . . . . . . 626
13.2 Cauchy Principal Value for Real Integrals . . . . . . . . . . . . . . . . . . . . . . . . . . . . . . . . . 634
13.2.1 The Cauchy Principal Value . . . . . . . . . . . . . . . . . . . . . . . . . . . . . . . . . . . . 634
13.3 Cauchy Principal Value for Contour Integrals . . . . . . . . . . . . . . . . . . . . . . . . . . . . . . . 639
13.4 Integrals on the Real Axis . . . . . . . . . . . . . . . . . . . . . . . . . . . . . . . . . . . . . . . . . 643
13.5 Fourier Integrals . . . . . . . . . . . . . . . . . . . . . . . . . . . . . . . . . . . . . . . . . . . . . . 647
13.6 Fourier Cosine and Sine Integrals . . . . . . . . . . . . . . . . . . . . . . . . . . . . . . . . . . . . . 649
13.7 Contour Integration and Branch Cuts . . . . . . . . . . . . . . . . . . . . . . . . . . . . . . . . . . . 652
13.8 Exploiting Symmetry . . . . . . . . . . . . . . . . . . . . . . . . . . . . . . . . . . . . . . . . . . . . 655
13.8.1 Wedge Contours . . . . . . . . . . . . . . . . . . . . . . . . . . . . . . . . . . . . . . . . . . 655
13.8.2 Box Contours . . . . . . . . . . . . . . . . . . . . . . . . . . . . . . . . . . . . . . . . . . . 658
13.9 Definite Integrals Involving Sine and Cosine . . . . . . . . . . . . . . . . . . . . . . . . . . . . . . . . 659
13.10Infinite Sums . . . . . . . . . . . . . . . . . . . . . . . . . . . . . . . . . . . . . . . . . . . . . . . . 662
13.11Exercises . . . . . . . . . . . . . . . . . . . . . . . . . . . . . . . . . . . . . . . . . . . . . . . . . . 666
13.12Hints . . . . . . . . . . . . . . . . . . . . . . . . . . . . . . . . . . . . . . . . . . . . . . . . . . . . 680
13.13Solutions . . . . . . . . . . . . . . . . . . . . . . . . . . . . . . . . . . . . . . . . . . . . . . . . . . 686

vii
IV Ordinary Differential Equations 772
14 First Order Differential Equations 773
14.1 Notation . . . . . . . . . . . . . . . . . . . . . . . . . . . . . . . . . . . . . . . . . . . . . . . . . . 773
14.2 Example Problems . . . . . . . . . . . . . . . . . . . . . . . . . . . . . . . . . . . . . . . . . . . . . 775
14.2.1 Growth and Decay . . . . . . . . . . . . . . . . . . . . . . . . . . . . . . . . . . . . . . . . . 775
14.3 One Parameter Families of Functions . . . . . . . . . . . . . . . . . . . . . . . . . . . . . . . . . . . 777
14.4 Integrable Forms . . . . . . . . . . . . . . . . . . . . . . . . . . . . . . . . . . . . . . . . . . . . . . 779
14.4.1 Separable Equations . . . . . . . . . . . . . . . . . . . . . . . . . . . . . . . . . . . . . . . . 780
14.4.2 Exact Equations . . . . . . . . . . . . . . . . . . . . . . . . . . . . . . . . . . . . . . . . . . 782
14.4.3 Homogeneous Coefficient Equations . . . . . . . . . . . . . . . . . . . . . . . . . . . . . . . . 786
14.5 The First Order, Linear Differential Equation . . . . . . . . . . . . . . . . . . . . . . . . . . . . . . . 791
14.5.1 Homogeneous Equations . . . . . . . . . . . . . . . . . . . . . . . . . . . . . . . . . . . . . . 791
14.5.2 Inhomogeneous Equations . . . . . . . . . . . . . . . . . . . . . . . . . . . . . . . . . . . . . 792
14.5.3 Variation of Parameters. . . . . . . . . . . . . . . . . . . . . . . . . . . . . . . . . . . . . . . 795
14.6 Initial Conditions . . . . . . . . . . . . . . . . . . . . . . . . . . . . . . . . . . . . . . . . . . . . . . 796
14.6.1 Piecewise Continuous Coefficients and Inhomogeneities . . . . . . . . . . . . . . . . . . . . . . 797
14.7 Well-Posed Problems . . . . . . . . . . . . . . . . . . . . . . . . . . . . . . . . . . . . . . . . . . . . 801
14.8 Equations in the Complex Plane . . . . . . . . . . . . . . . . . . . . . . . . . . . . . . . . . . . . . . 803
14.8.1 Ordinary Points . . . . . . . . . . . . . . . . . . . . . . . . . . . . . . . . . . . . . . . . . . 803
14.8.2 Regular Singular Points . . . . . . . . . . . . . . . . . . . . . . . . . . . . . . . . . . . . . . 806
14.8.3 Irregular Singular Points . . . . . . . . . . . . . . . . . . . . . . . . . . . . . . . . . . . . . . 812
14.8.4 The Point at Infinity . . . . . . . . . . . . . . . . . . . . . . . . . . . . . . . . . . . . . . . . 814
14.9 Additional Exercises . . . . . . . . . . . . . . . . . . . . . . . . . . . . . . . . . . . . . . . . . . . . 816
14.10Hints . . . . . . . . . . . . . . . . . . . . . . . . . . . . . . . . . . . . . . . . . . . . . . . . . . . . 819
14.11Solutions . . . . . . . . . . . . . . . . . . . . . . . . . . . . . . . . . . . . . . . . . . . . . . . . . . 822
14.12Quiz . . . . . . . . . . . . . . . . . . . . . . . . . . . . . . . . . . . . . . . . . . . . . . . . . . . . 843
14.13Quiz Solutions . . . . . . . . . . . . . . . . . . . . . . . . . . . . . . . . . . . . . . . . . . . . . . . 844

viii
15 First Order Linear Systems of Differential Equations 846
15.1 Introduction . . . . . . . . . . . . . . . . . . . . . . . . . . . . . . . . . . . . . . . . . . . . . . . . 846
15.2 Using Eigenvalues and Eigenvectors to find Homogeneous Solutions . . . . . . . . . . . . . . . . . . . 847
15.3 Matrices and Jordan Canonical Form . . . . . . . . . . . . . . . . . . . . . . . . . . . . . . . . . . . 852
15.4 Using the Matrix Exponential . . . . . . . . . . . . . . . . . . . . . . . . . . . . . . . . . . . . . . . 860
15.5 Exercises . . . . . . . . . . . . . . . . . . . . . . . . . . . . . . . . . . . . . . . . . . . . . . . . . . 865
15.6 Hints . . . . . . . . . . . . . . . . . . . . . . . . . . . . . . . . . . . . . . . . . . . . . . . . . . . . 870
15.7 Solutions . . . . . . . . . . . . . . . . . . . . . . . . . . . . . . . . . . . . . . . . . . . . . . . . . . 872

16 Theory of Linear Ordinary Differential Equations 900


16.1 Exact Equations . . . . . . . . . . . . . . . . . . . . . . . . . . . . . . . . . . . . . . . . . . . . . . 900
16.2 Nature of Solutions . . . . . . . . . . . . . . . . . . . . . . . . . . . . . . . . . . . . . . . . . . . . 901
16.3 Transformation to a First Order System . . . . . . . . . . . . . . . . . . . . . . . . . . . . . . . . . . 905
16.4 The Wronskian . . . . . . . . . . . . . . . . . . . . . . . . . . . . . . . . . . . . . . . . . . . . . . . 905
16.4.1 Derivative of a Determinant. . . . . . . . . . . . . . . . . . . . . . . . . . . . . . . . . . . . 905
16.4.2 The Wronskian of a Set of Functions. . . . . . . . . . . . . . . . . . . . . . . . . . . . . . . . 906
16.4.3 The Wronskian of the Solutions to a Differential Equation . . . . . . . . . . . . . . . . . . . . 908
16.5 Well-Posed Problems . . . . . . . . . . . . . . . . . . . . . . . . . . . . . . . . . . . . . . . . . . . . 911
16.6 The Fundamental Set of Solutions . . . . . . . . . . . . . . . . . . . . . . . . . . . . . . . . . . . . . 913
16.7 Adjoint Equations . . . . . . . . . . . . . . . . . . . . . . . . . . . . . . . . . . . . . . . . . . . . . 915
16.8 Additional Exercises . . . . . . . . . . . . . . . . . . . . . . . . . . . . . . . . . . . . . . . . . . . . 919
16.9 Hints . . . . . . . . . . . . . . . . . . . . . . . . . . . . . . . . . . . . . . . . . . . . . . . . . . . . 920
16.10Solutions . . . . . . . . . . . . . . . . . . . . . . . . . . . . . . . . . . . . . . . . . . . . . . . . . . 922
16.11Quiz . . . . . . . . . . . . . . . . . . . . . . . . . . . . . . . . . . . . . . . . . . . . . . . . . . . . 928
16.12Quiz Solutions . . . . . . . . . . . . . . . . . . . . . . . . . . . . . . . . . . . . . . . . . . . . . . . 929

17 Techniques for Linear Differential Equations 930


17.1 Constant Coefficient Equations . . . . . . . . . . . . . . . . . . . . . . . . . . . . . . . . . . . . . . 930
17.1.1 Second Order Equations . . . . . . . . . . . . . . . . . . . . . . . . . . . . . . . . . . . . . . 931
17.1.2 Real-Valued Solutions . . . . . . . . . . . . . . . . . . . . . . . . . . . . . . . . . . . . . . . 935

ix
17.1.3 Higher Order Equations . . . . . . . . . . . . . . . . . . . . . . . . . . . . . . . . . . . . . . 937
17.2 Euler Equations . . . . . . . . . . . . . . . . . . . . . . . . . . . . . . . . . . . . . . . . . . . . . . 940
17.2.1 Real-Valued Solutions . . . . . . . . . . . . . . . . . . . . . . . . . . . . . . . . . . . . . . . 942
17.3 Exact Equations . . . . . . . . . . . . . . . . . . . . . . . . . . . . . . . . . . . . . . . . . . . . . . 945
17.4 Equations Without Explicit Dependence on y . . . . . . . . . . . . . . . . . . . . . . . . . . . . . . . 946
17.5 Reduction of Order . . . . . . . . . . . . . . . . . . . . . . . . . . . . . . . . . . . . . . . . . . . . . 947
17.6 *Reduction of Order and the Adjoint Equation . . . . . . . . . . . . . . . . . . . . . . . . . . . . . . 948
17.7 Additional Exercises . . . . . . . . . . . . . . . . . . . . . . . . . . . . . . . . . . . . . . . . . . . . 951
17.8 Hints . . . . . . . . . . . . . . . . . . . . . . . . . . . . . . . . . . . . . . . . . . . . . . . . . . . . 957
17.9 Solutions . . . . . . . . . . . . . . . . . . . . . . . . . . . . . . . . . . . . . . . . . . . . . . . . . . 960

18 Techniques for Nonlinear Differential Equations 984


18.1 Bernoulli Equations . . . . . . . . . . . . . . . . . . . . . . . . . . . . . . . . . . . . . . . . . . . . 984
18.2 Riccati Equations . . . . . . . . . . . . . . . . . . . . . . . . . . . . . . . . . . . . . . . . . . . . . 986
18.3 Exchanging the Dependent and Independent Variables . . . . . . . . . . . . . . . . . . . . . . . . . . 990
18.4 Autonomous Equations . . . . . . . . . . . . . . . . . . . . . . . . . . . . . . . . . . . . . . . . . . 992
18.5 *Equidimensional-in-x Equations . . . . . . . . . . . . . . . . . . . . . . . . . . . . . . . . . . . . . . 995
18.6 *Equidimensional-in-y Equations . . . . . . . . . . . . . . . . . . . . . . . . . . . . . . . . . . . . . . 997
18.7 *Scale-Invariant Equations . . . . . . . . . . . . . . . . . . . . . . . . . . . . . . . . . . . . . . . . . 1000
18.8 Exercises . . . . . . . . . . . . . . . . . . . . . . . . . . . . . . . . . . . . . . . . . . . . . . . . . . 1001
18.9 Hints . . . . . . . . . . . . . . . . . . . . . . . . . . . . . . . . . . . . . . . . . . . . . . . . . . . . 1004
18.10Solutions . . . . . . . . . . . . . . . . . . . . . . . . . . . . . . . . . . . . . . . . . . . . . . . . . . 1006

19 Transformations and Canonical Forms 1018


19.1 The Constant Coefficient Equation . . . . . . . . . . . . . . . . . . . . . . . . . . . . . . . . . . . . 1018
19.2 Normal Form . . . . . . . . . . . . . . . . . . . . . . . . . . . . . . . . . . . . . . . . . . . . . . . . 1021
19.2.1 Second Order Equations . . . . . . . . . . . . . . . . . . . . . . . . . . . . . . . . . . . . . . 1021
19.2.2 Higher Order Differential Equations . . . . . . . . . . . . . . . . . . . . . . . . . . . . . . . . 1022
19.3 Transformations of the Independent Variable . . . . . . . . . . . . . . . . . . . . . . . . . . . . . . . 1024
19.3.1 Transformation to the form u + a(x) u = 0 . . . . . . . . . . . . . . . . . . . . . . . . . . . 1024

x
19.3.2 Transformation to a Constant Coefficient Equation . . . . . . . . . . . . . . . . . . . . . . . . 1025
19.4 Integral Equations . . . . . . . . . . . . . . . . . . . . . . . . . . . . . . . . . . . . . . . . . . . . . 1027
19.4.1 Initial Value Problems . . . . . . . . . . . . . . . . . . . . . . . . . . . . . . . . . . . . . . . 1027
19.4.2 Boundary Value Problems . . . . . . . . . . . . . . . . . . . . . . . . . . . . . . . . . . . . . 1029
19.5 Exercises . . . . . . . . . . . . . . . . . . . . . . . . . . . . . . . . . . . . . . . . . . . . . . . . . . 1032
19.6 Hints . . . . . . . . . . . . . . . . . . . . . . . . . . . . . . . . . . . . . . . . . . . . . . . . . . . . 1034
19.7 Solutions . . . . . . . . . . . . . . . . . . . . . . . . . . . . . . . . . . . . . . . . . . . . . . . . . . 1035

20 The Dirac Delta Function 1041


20.1 Derivative of the Heaviside Function . . . . . . . . . . . . . . . . . . . . . . . . . . . . . . . . . . . 1041
20.2 The Delta Function as a Limit . . . . . . . . . . . . . . . . . . . . . . . . . . . . . . . . . . . . . . . 1043
20.3 Higher Dimensions . . . . . . . . . . . . . . . . . . . . . . . . . . . . . . . . . . . . . . . . . . . . . 1045
20.4 Non-Rectangular Coordinate Systems . . . . . . . . . . . . . . . . . . . . . . . . . . . . . . . . . . . 1046
20.5 Exercises . . . . . . . . . . . . . . . . . . . . . . . . . . . . . . . . . . . . . . . . . . . . . . . . . . 1048
20.6 Hints . . . . . . . . . . . . . . . . . . . . . . . . . . . . . . . . . . . . . . . . . . . . . . . . . . . . 1050
20.7 Solutions . . . . . . . . . . . . . . . . . . . . . . . . . . . . . . . . . . . . . . . . . . . . . . . . . . 1052

21 Inhomogeneous Differential Equations 1059


21.1 Particular Solutions . . . . . . . . . . . . . . . . . . . . . . . . . . . . . . . . . . . . . . . . . . . . 1059
21.2 Method of Undetermined Coefficients . . . . . . . . . . . . . . . . . . . . . . . . . . . . . . . . . . . 1061
21.3 Variation of Parameters . . . . . . . . . . . . . . . . . . . . . . . . . . . . . . . . . . . . . . . . . . 1065
21.3.1 Second Order Differential Equations . . . . . . . . . . . . . . . . . . . . . . . . . . . . . . . . 1065
21.3.2 Higher Order Differential Equations . . . . . . . . . . . . . . . . . . . . . . . . . . . . . . . . 1068
21.4 Piecewise Continuous Coefficients and Inhomogeneities . . . . . . . . . . . . . . . . . . . . . . . . . . 1071
21.5 Inhomogeneous Boundary Conditions . . . . . . . . . . . . . . . . . . . . . . . . . . . . . . . . . . . 1074
21.5.1 Eliminating Inhomogeneous Boundary Conditions . . . . . . . . . . . . . . . . . . . . . . . . . 1074
21.5.2 Separating Inhomogeneous Equations and Inhomogeneous Boundary Conditions . . . . . . . . . 1076
21.5.3 Existence of Solutions of Problems with Inhomogeneous Boundary Conditions . . . . . . . . . . 1077
21.6 Green Functions for First Order Equations . . . . . . . . . . . . . . . . . . . . . . . . . . . . . . . . 1079
21.7 Green Functions for Second Order Equations . . . . . . . . . . . . . . . . . . . . . . . . . . . . . . . 1082

xi
21.7.1 Green Functions for Sturm-Liouville Problems . . . . . . . . . . . . . . . . . . . . . . . . . . . 1092
21.7.2 Initial Value Problems . . . . . . . . . . . . . . . . . . . . . . . . . . . . . . . . . . . . . . . 1095
21.7.3 Problems with Unmixed Boundary Conditions . . . . . . . . . . . . . . . . . . . . . . . . . . . 1098
21.7.4 Problems with Mixed Boundary Conditions . . . . . . . . . . . . . . . . . . . . . . . . . . . . 1100
21.8 Green Functions for Higher Order Problems . . . . . . . . . . . . . . . . . . . . . . . . . . . . . . . . 1104
21.9 Fredholm Alternative Theorem . . . . . . . . . . . . . . . . . . . . . . . . . . . . . . . . . . . . . . 1109
21.10Exercises . . . . . . . . . . . . . . . . . . . . . . . . . . . . . . . . . . . . . . . . . . . . . . . . . . 1117
21.11Hints . . . . . . . . . . . . . . . . . . . . . . . . . . . . . . . . . . . . . . . . . . . . . . . . . . . . 1123
21.12Solutions . . . . . . . . . . . . . . . . . . . . . . . . . . . . . . . . . . . . . . . . . . . . . . . . . . 1126
21.13Quiz . . . . . . . . . . . . . . . . . . . . . . . . . . . . . . . . . . . . . . . . . . . . . . . . . . . . 1164
21.14Quiz Solutions . . . . . . . . . . . . . . . . . . . . . . . . . . . . . . . . . . . . . . . . . . . . . . . 1165

22 Difference Equations 1166


22.1 Introduction . . . . . . . . . . . . . . . . . . . . . . . . . . . . . . . . . . . . . . . . . . . . . . . . 1166
22.2 Exact Equations . . . . . . . . . . . . . . . . . . . . . . . . . . . . . . . . . . . . . . . . . . . . . . 1168
22.3 Homogeneous First Order . . . . . . . . . . . . . . . . . . . . . . . . . . . . . . . . . . . . . . . . . 1169
22.4 Inhomogeneous First Order . . . . . . . . . . . . . . . . . . . . . . . . . . . . . . . . . . . . . . . . 1171
22.5 Homogeneous Constant Coefficient Equations . . . . . . . . . . . . . . . . . . . . . . . . . . . . . . . 1174
22.6 Reduction of Order . . . . . . . . . . . . . . . . . . . . . . . . . . . . . . . . . . . . . . . . . . . . . 1177
22.7 Exercises . . . . . . . . . . . . . . . . . . . . . . . . . . . . . . . . . . . . . . . . . . . . . . . . . . 1179
22.8 Hints . . . . . . . . . . . . . . . . . . . . . . . . . . . . . . . . . . . . . . . . . . . . . . . . . . . . 1180
22.9 Solutions . . . . . . . . . . . . . . . . . . . . . . . . . . . . . . . . . . . . . . . . . . . . . . . . . . 1181

23 Series Solutions of Differential Equations 1184


23.1 Ordinary Points . . . . . . . . . . . . . . . . . . . . . . . . . . . . . . . . . . . . . . . . . . . . . . 1184
23.1.1 Taylor Series Expansion for a Second Order Differential Equation . . . . . . . . . . . . . . . . 1188
23.2 Regular Singular Points of Second Order Equations . . . . . . . . . . . . . . . . . . . . . . . . . . . . 1198
23.2.1 Indicial Equation . . . . . . . . . . . . . . . . . . . . . . . . . . . . . . . . . . . . . . . . . . 1201
23.2.2 The Case: Double Root . . . . . . . . . . . . . . . . . . . . . . . . . . . . . . . . . . . . . . 1203
23.2.3 The Case: Roots Differ by an Integer . . . . . . . . . . . . . . . . . . . . . . . . . . . . . . . 1206

xii
23.3 Irregular Singular Points . . . . . . . . . . . . . . . . . . . . . . . . . . . . . . . . . . . . . . . . . . 1216
23.4 The Point at Infinity . . . . . . . . . . . . . . . . . . . . . . . . . . . . . . . . . . . . . . . . . . . . 1216
23.5 Exercises . . . . . . . . . . . . . . . . . . . . . . . . . . . . . . . . . . . . . . . . . . . . . . . . . . 1219
23.6 Hints . . . . . . . . . . . . . . . . . . . . . . . . . . . . . . . . . . . . . . . . . . . . . . . . . . . . 1224
23.7 Solutions . . . . . . . . . . . . . . . . . . . . . . . . . . . . . . . . . . . . . . . . . . . . . . . . . . 1225
23.8 Quiz . . . . . . . . . . . . . . . . . . . . . . . . . . . . . . . . . . . . . . . . . . . . . . . . . . . . 1248
23.9 Quiz Solutions . . . . . . . . . . . . . . . . . . . . . . . . . . . . . . . . . . . . . . . . . . . . . . . 1249

24 Asymptotic Expansions 1251


24.1 Asymptotic Relations . . . . . . . . . . . . . . . . . . . . . . . . . . . . . . . . . . . . . . . . . . . 1251
24.2 Leading Order Behavior of Differential Equations . . . . . . . . . . . . . . . . . . . . . . . . . . . . . 1255
24.3 Integration by Parts . . . . . . . . . . . . . . . . . . . . . . . . . . . . . . . . . . . . . . . . . . . . 1263
24.4 Asymptotic Series . . . . . . . . . . . . . . . . . . . . . . . . . . . . . . . . . . . . . . . . . . . . . 1270
24.5 Asymptotic Expansions of Differential Equations . . . . . . . . . . . . . . . . . . . . . . . . . . . . . 1272
24.5.1 The Parabolic Cylinder Equation. . . . . . . . . . . . . . . . . . . . . . . . . . . . . . . . . . 1272

25 Hilbert Spaces 1278


25.1 Linear Spaces . . . . . . . . . . . . . . . . . . . . . . . . . . . . . . . . . . . . . . . . . . . . . . . 1278
25.2 Inner Products . . . . . . . . . . . . . . . . . . . . . . . . . . . . . . . . . . . . . . . . . . . . . . . 1280
25.3 Norms . . . . . . . . . . . . . . . . . . . . . . . . . . . . . . . . . . . . . . . . . . . . . . . . . . . 1281
25.4 Linear Independence. . . . . . . . . . . . . . . . . . . . . . . . . . . . . . . . . . . . . . . . . . . . . 1283
25.5 Orthogonality . . . . . . . . . . . . . . . . . . . . . . . . . . . . . . . . . . . . . . . . . . . . . . . 1283
25.6 Gramm-Schmidt Orthogonalization . . . . . . . . . . . . . . . . . . . . . . . . . . . . . . . . . . . . 1284
25.7 Orthonormal Function Expansion . . . . . . . . . . . . . . . . . . . . . . . . . . . . . . . . . . . . . 1287
25.8 Sets Of Functions . . . . . . . . . . . . . . . . . . . . . . . . . . . . . . . . . . . . . . . . . . . . . 1288
25.9 Least Squares Fit to a Function and Completeness . . . . . . . . . . . . . . . . . . . . . . . . . . . . 1294
25.10Closure Relation . . . . . . . . . . . . . . . . . . . . . . . . . . . . . . . . . . . . . . . . . . . . . . 1297
25.11Linear Operators . . . . . . . . . . . . . . . . . . . . . . . . . . . . . . . . . . . . . . . . . . . . . . 1302
25.12Exercises . . . . . . . . . . . . . . . . . . . . . . . . . . . . . . . . . . . . . . . . . . . . . . . . . . 1303
25.13Hints . . . . . . . . . . . . . . . . . . . . . . . . . . . . . . . . . . . . . . . . . . . . . . . . . . . . 1304

xiii
25.14Solutions . . . . . . . . . . . . . . . . . . . . . . . . . . . . . . . . . . . . . . . . . . . . . . . . . . 1305

26 Self Adjoint Linear Operators 1307


26.1 Adjoint Operators . . . . . . . . . . . . . . . . . . . . . . . . . . . . . . . . . . . . . . . . . . . . . 1307
26.2 Self-Adjoint Operators . . . . . . . . . . . . . . . . . . . . . . . . . . . . . . . . . . . . . . . . . . . 1308
26.3 Exercises . . . . . . . . . . . . . . . . . . . . . . . . . . . . . . . . . . . . . . . . . . . . . . . . . . 1311
26.4 Hints . . . . . . . . . . . . . . . . . . . . . . . . . . . . . . . . . . . . . . . . . . . . . . . . . . . . 1312
26.5 Solutions . . . . . . . . . . . . . . . . . . . . . . . . . . . . . . . . . . . . . . . . . . . . . . . . . . 1313

27 Self-Adjoint Boundary Value Problems 1314


27.1 Summary of Adjoint Operators . . . . . . . . . . . . . . . . . . . . . . . . . . . . . . . . . . . . . . 1314
27.2 Formally Self-Adjoint Operators . . . . . . . . . . . . . . . . . . . . . . . . . . . . . . . . . . . . . . 1315
27.3 Self-Adjoint Problems . . . . . . . . . . . . . . . . . . . . . . . . . . . . . . . . . . . . . . . . . . . 1318
27.4 Self-Adjoint Eigenvalue Problems . . . . . . . . . . . . . . . . . . . . . . . . . . . . . . . . . . . . . 1318
27.5 Inhomogeneous Equations . . . . . . . . . . . . . . . . . . . . . . . . . . . . . . . . . . . . . . . . . 1323
27.6 Exercises . . . . . . . . . . . . . . . . . . . . . . . . . . . . . . . . . . . . . . . . . . . . . . . . . . 1326
27.7 Hints . . . . . . . . . . . . . . . . . . . . . . . . . . . . . . . . . . . . . . . . . . . . . . . . . . . . 1327
27.8 Solutions . . . . . . . . . . . . . . . . . . . . . . . . . . . . . . . . . . . . . . . . . . . . . . . . . . 1328

28 Fourier Series 1330


28.1 An Eigenvalue Problem. . . . . . . . . . . . . . . . . . . . . . . . . . . . . . . . . . . . . . . . . . . 1330
28.2 Fourier Series. . . . . . . . . . . . . . . . . . . . . . . . . . . . . . . . . . . . . . . . . . . . . . . . 1333
28.3 Least Squares Fit . . . . . . . . . . . . . . . . . . . . . . . . . . . . . . . . . . . . . . . . . . . . . 1337
28.4 Fourier Series for Functions Defined on Arbitrary Ranges . . . . . . . . . . . . . . . . . . . . . . . . . 1341
28.5 Fourier Cosine Series . . . . . . . . . . . . . . . . . . . . . . . . . . . . . . . . . . . . . . . . . . . . 1344
28.6 Fourier Sine Series . . . . . . . . . . . . . . . . . . . . . . . . . . . . . . . . . . . . . . . . . . . . . 1345
28.7 Complex Fourier Series and Parsevals Theorem . . . . . . . . . . . . . . . . . . . . . . . . . . . . . . 1346
28.8 Behavior of Fourier Coefficients . . . . . . . . . . . . . . . . . . . . . . . . . . . . . . . . . . . . . . 1349
28.9 Gibbs Phenomenon . . . . . . . . . . . . . . . . . . . . . . . . . . . . . . . . . . . . . . . . . . . . 1358
28.10Integrating and Differentiating Fourier Series . . . . . . . . . . . . . . . . . . . . . . . . . . . . . . . 1358

xiv
28.11Exercises . . . . . . . . . . . . . . . . . . . . . . . . . . . . . . . . . . . . . . . . . . . . . . . . . . 1363
28.12Hints . . . . . . . . . . . . . . . . . . . . . . . . . . . . . . . . . . . . . . . . . . . . . . . . . . . . 1371
28.13Solutions . . . . . . . . . . . . . . . . . . . . . . . . . . . . . . . . . . . . . . . . . . . . . . . . . . 1373

29 Regular Sturm-Liouville Problems 1420


29.1 Derivation of the Sturm-Liouville Form . . . . . . . . . . . . . . . . . . . . . . . . . . . . . . . . . . 1420
29.2 Properties of Regular Sturm-Liouville Problems . . . . . . . . . . . . . . . . . . . . . . . . . . . . . . 1422
29.3 Solving Differential Equations With Eigenfunction Expansions . . . . . . . . . . . . . . . . . . . . . . 1433
29.4 Exercises . . . . . . . . . . . . . . . . . . . . . . . . . . . . . . . . . . . . . . . . . . . . . . . . . . 1439
29.5 Hints . . . . . . . . . . . . . . . . . . . . . . . . . . . . . . . . . . . . . . . . . . . . . . . . . . . . 1443
29.6 Solutions . . . . . . . . . . . . . . . . . . . . . . . . . . . . . . . . . . . . . . . . . . . . . . . . . . 1445

30 Integrals and Convergence 1470


30.1 Uniform Convergence of Integrals . . . . . . . . . . . . . . . . . . . . . . . . . . . . . . . . . . . . . 1470
30.2 The Riemann-Lebesgue Lemma . . . . . . . . . . . . . . . . . . . . . . . . . . . . . . . . . . . . . . 1471
30.3 Cauchy Principal Value . . . . . . . . . . . . . . . . . . . . . . . . . . . . . . . . . . . . . . . . . . 1472
30.3.1 Integrals on an Infinite Domain . . . . . . . . . . . . . . . . . . . . . . . . . . . . . . . . . . 1472
30.3.2 Singular Functions . . . . . . . . . . . . . . . . . . . . . . . . . . . . . . . . . . . . . . . . . 1473

31 The Laplace Transform 1475


31.1 The Laplace Transform . . . . . . . . . . . . . . . . . . . . . . . . . . . . . . . . . . . . . . . . . . 1475
31.2 The Inverse Laplace Transform . . . . . . . . . . . . . . . . . . . . . . . . . . . . . . . . . . . . . . 1477
31.2.1 f(s) with Poles . . . . . . . . . . . . . . . . . . . . . . . . . . . . . . . . . . . . . . . . . . 1480
31.2.2 f(s) with Branch Points . . . . . . . . . . . . . . . . . . . . . . . . . . . . . . . . . . . . . . 1484
31.2.3 Asymptotic Behavior of f(s) . . . . . . . . . . . . . . . . . . . . . . . . . . . . . . . . . . . 1488
31.3 Properties of the Laplace Transform . . . . . . . . . . . . . . . . . . . . . . . . . . . . . . . . . . . . 1489
31.4 Constant Coefficient Differential Equations . . . . . . . . . . . . . . . . . . . . . . . . . . . . . . . . 1492
31.5 Systems of Constant Coefficient Differential Equations . . . . . . . . . . . . . . . . . . . . . . . . . . 1495
31.6 Exercises . . . . . . . . . . . . . . . . . . . . . . . . . . . . . . . . . . . . . . . . . . . . . . . . . . 1497
31.7 Hints . . . . . . . . . . . . . . . . . . . . . . . . . . . . . . . . . . . . . . . . . . . . . . . . . . . . 1504

xv
31.8 Solutions . . . . . . . . . . . . . . . . . . . . . . . . . . . . . . . . . . . . . . . . . . . . . . . . . . 1507

32 The Fourier Transform 1539


32.1 Derivation from a Fourier Series . . . . . . . . . . . . . . . . . . . . . . . . . . . . . . . . . . . . . . 1539
32.2 The Fourier Transform . . . . . . . . . . . . . . . . . . . . . . . . . . . . . . . . . . . . . . . . . . . 1541
32.2.1 A Word of Caution . . . . . . . . . . . . . . . . . . . . . . . . . . . . . . . . . . . . . . . . . 1544
32.3 Evaluating Fourier Integrals . . . . . . . . . . . . . . . . . . . . . . . . . . . . . . . . . . . . . . . . 1545
32.3.1 Integrals that Converge . . . . . . . . . . . . . . . . . . . . . . . . . . . . . . . . . . . . . . 1545
32.3.2 Cauchy Principal Value and Integrals that are Not Absolutely Convergent. . . . . . . . . . . . . 1548
32.3.3 Analytic Continuation . . . . . . . . . . . . . . . . . . . . . . . . . . . . . . . . . . . . . . . 1550
32.4 Properties of the Fourier Transform . . . . . . . . . . . . . . . . . . . . . . . . . . . . . . . . . . . . 1552
32.4.1 Closure Relation. . . . . . . . . . . . . . . . . . . . . . . . . . . . . . . . . . . . . . . . . . . 1552
32.4.2 Fourier Transform of a Derivative. . . . . . . . . . . . . . . . . . . . . . . . . . . . . . . . . . 1553
32.4.3 Fourier Convolution Theorem. . . . . . . . . . . . . . . . . . . . . . . . . . . . . . . . . . . . 1554
32.4.4 Parsevals Theorem. . . . . . . . . . . . . . . . . . . . . . . . . . . . . . . . . . . . . . . . . 1557
32.4.5 Shift Property. . . . . . . . . . . . . . . . . . . . . . . . . . . . . . . . . . . . . . . . . . . . 1559
32.4.6 Fourier Transform of x f(x). . . . . . . . . . . . . . . . . . . . . . . . . . . . . . . . . . . . . 1559
32.5 Solving Differential Equations with the Fourier Transform . . . . . . . . . . . . . . . . . . . . . . . . 1560
32.6 The Fourier Cosine and Sine Transform . . . . . . . . . . . . . . . . . . . . . . . . . . . . . . . . . . 1562
32.6.1 The Fourier Cosine Transform . . . . . . . . . . . . . . . . . . . . . . . . . . . . . . . . . . . 1562
32.6.2 The Fourier Sine Transform . . . . . . . . . . . . . . . . . . . . . . . . . . . . . . . . . . . . 1563
32.7 Properties of the Fourier Cosine and Sine Transform . . . . . . . . . . . . . . . . . . . . . . . . . . . 1564
32.7.1 Transforms of Derivatives . . . . . . . . . . . . . . . . . . . . . . . . . . . . . . . . . . . . . 1564
32.7.2 Convolution Theorems . . . . . . . . . . . . . . . . . . . . . . . . . . . . . . . . . . . . . . . 1566
32.7.3 Cosine and Sine Transform in Terms of the Fourier Transform . . . . . . . . . . . . . . . . . . 1568
32.8 Solving Differential Equations with the Fourier Cosine and Sine Transforms . . . . . . . . . . . . . . . 1569
32.9 Exercises . . . . . . . . . . . . . . . . . . . . . . . . . . . . . . . . . . . . . . . . . . . . . . . . . . 1571
32.10Hints . . . . . . . . . . . . . . . . . . . . . . . . . . . . . . . . . . . . . . . . . . . . . . . . . . . . 1578
32.11Solutions . . . . . . . . . . . . . . . . . . . . . . . . . . . . . . . . . . . . . . . . . . . . . . . . . . 1581

xvi
33 The Gamma Function 1605
33.1 Eulers Formula . . . . . . . . . . . . . . . . . . . . . . . . . . . . . . . . . . . . . . . . . . . . . . 1605
33.2 Hankels Formula . . . . . . . . . . . . . . . . . . . . . . . . . . . . . . . . . . . . . . . . . . . . . . 1607
33.3 Gauss Formula . . . . . . . . . . . . . . . . . . . . . . . . . . . . . . . . . . . . . . . . . . . . . . . 1609
33.4 Weierstrass Formula . . . . . . . . . . . . . . . . . . . . . . . . . . . . . . . . . . . . . . . . . . . . 1611
33.5 Stirlings Approximation . . . . . . . . . . . . . . . . . . . . . . . . . . . . . . . . . . . . . . . . . . 1613
33.6 Exercises . . . . . . . . . . . . . . . . . . . . . . . . . . . . . . . . . . . . . . . . . . . . . . . . . . 1618
33.7 Hints . . . . . . . . . . . . . . . . . . . . . . . . . . . . . . . . . . . . . . . . . . . . . . . . . . . . 1619
33.8 Solutions . . . . . . . . . . . . . . . . . . . . . . . . . . . . . . . . . . . . . . . . . . . . . . . . . . 1620

34 Bessel Functions 1622


34.1 Bessels Equation . . . . . . . . . . . . . . . . . . . . . . . . . . . . . . . . . . . . . . . . . . . . . 1622
34.2 Frobeneius Series Solution about z = 0 . . . . . . . . . . . . . . . . . . . . . . . . . . . . . . . . . . 1623
34.2.1 Behavior at Infinity . . . . . . . . . . . . . . . . . . . . . . . . . . . . . . . . . . . . . . . . . 1626
34.3 Bessel Functions of the First Kind . . . . . . . . . . . . . . . . . . . . . . . . . . . . . . . . . . . . . 1628
34.3.1 The Bessel Function Satisfies Bessels Equation . . . . . . . . . . . . . . . . . . . . . . . . . . 1629
34.3.2 Series Expansion of the Bessel Function . . . . . . . . . . . . . . . . . . . . . . . . . . . . . . 1630
34.3.3 Bessel Functions of Non-Integer Order . . . . . . . . . . . . . . . . . . . . . . . . . . . . . . 1633
34.3.4 Recursion Formulas . . . . . . . . . . . . . . . . . . . . . . . . . . . . . . . . . . . . . . . . 1636
34.3.5 Bessel Functions of Half-Integer Order . . . . . . . . . . . . . . . . . . . . . . . . . . . . . . 1639
34.4 Neumann Expansions . . . . . . . . . . . . . . . . . . . . . . . . . . . . . . . . . . . . . . . . . . . 1640
34.5 Bessel Functions of the Second Kind . . . . . . . . . . . . . . . . . . . . . . . . . . . . . . . . . . . 1644
34.6 Hankel Functions . . . . . . . . . . . . . . . . . . . . . . . . . . . . . . . . . . . . . . . . . . . . . . 1646
34.7 The Modified Bessel Equation . . . . . . . . . . . . . . . . . . . . . . . . . . . . . . . . . . . . . . . 1646
34.8 Exercises . . . . . . . . . . . . . . . . . . . . . . . . . . . . . . . . . . . . . . . . . . . . . . . . . . 1650
34.9 Hints . . . . . . . . . . . . . . . . . . . . . . . . . . . . . . . . . . . . . . . . . . . . . . . . . . . . 1655
34.10Solutions . . . . . . . . . . . . . . . . . . . . . . . . . . . . . . . . . . . . . . . . . . . . . . . . . . 1657

xvii
V Partial Differential Equations 1680
35 Transforming Equations 1681
35.1 Exercises . . . . . . . . . . . . . . . . . . . . . . . . . . . . . . . . . . . . . . . . . . . . . . . . . . 1682
35.2 Hints . . . . . . . . . . . . . . . . . . . . . . . . . . . . . . . . . . . . . . . . . . . . . . . . . . . . 1683
35.3 Solutions . . . . . . . . . . . . . . . . . . . . . . . . . . . . . . . . . . . . . . . . . . . . . . . . . . 1684

36 Classification of Partial Differential Equations 1685


36.1 Classification of Second Order Quasi-Linear Equations . . . . . . . . . . . . . . . . . . . . . . . . . . 1685
36.1.1 Hyperbolic Equations . . . . . . . . . . . . . . . . . . . . . . . . . . . . . . . . . . . . . . . 1686
36.1.2 Parabolic equations . . . . . . . . . . . . . . . . . . . . . . . . . . . . . . . . . . . . . . . . 1691
36.1.3 Elliptic Equations . . . . . . . . . . . . . . . . . . . . . . . . . . . . . . . . . . . . . . . . . 1692
36.2 Equilibrium Solutions . . . . . . . . . . . . . . . . . . . . . . . . . . . . . . . . . . . . . . . . . . . 1694
36.3 Exercises . . . . . . . . . . . . . . . . . . . . . . . . . . . . . . . . . . . . . . . . . . . . . . . . . . 1696
36.4 Hints . . . . . . . . . . . . . . . . . . . . . . . . . . . . . . . . . . . . . . . . . . . . . . . . . . . . 1697
36.5 Solutions . . . . . . . . . . . . . . . . . . . . . . . . . . . . . . . . . . . . . . . . . . . . . . . . . . 1698

37 Separation of Variables 1704


37.1 Eigensolutions of Homogeneous Equations . . . . . . . . . . . . . . . . . . . . . . . . . . . . . . . . 1704
37.2 Homogeneous Equations with Homogeneous Boundary Conditions . . . . . . . . . . . . . . . . . . . . 1704
37.3 Time-Independent Sources and Boundary Conditions . . . . . . . . . . . . . . . . . . . . . . . . . . . 1706
37.4 Inhomogeneous Equations with Homogeneous Boundary Conditions . . . . . . . . . . . . . . . . . . . 1709
37.5 Inhomogeneous Boundary Conditions . . . . . . . . . . . . . . . . . . . . . . . . . . . . . . . . . . . 1710
37.6 The Wave Equation . . . . . . . . . . . . . . . . . . . . . . . . . . . . . . . . . . . . . . . . . . . . 1713
37.7 General Method . . . . . . . . . . . . . . . . . . . . . . . . . . . . . . . . . . . . . . . . . . . . . . 1716
37.8 Exercises . . . . . . . . . . . . . . . . . . . . . . . . . . . . . . . . . . . . . . . . . . . . . . . . . . 1718
37.9 Hints . . . . . . . . . . . . . . . . . . . . . . . . . . . . . . . . . . . . . . . . . . . . . . . . . . . . 1734
37.10Solutions . . . . . . . . . . . . . . . . . . . . . . . . . . . . . . . . . . . . . . . . . . . . . . . . . . 1739

xviii
38 Finite Transforms 1821
38.1 Exercises . . . . . . . . . . . . . . . . . . . . . . . . . . . . . . . . . . . . . . . . . . . . . . . . . . 1825
38.2 Hints . . . . . . . . . . . . . . . . . . . . . . . . . . . . . . . . . . . . . . . . . . . . . . . . . . . . 1826
38.3 Solutions . . . . . . . . . . . . . . . . . . . . . . . . . . . . . . . . . . . . . . . . . . . . . . . . . . 1827

39 The Diffusion Equation 1831


39.1 Exercises . . . . . . . . . . . . . . . . . . . . . . . . . . . . . . . . . . . . . . . . . . . . . . . . . . 1832
39.2 Hints . . . . . . . . . . . . . . . . . . . . . . . . . . . . . . . . . . . . . . . . . . . . . . . . . . . . 1834
39.3 Solutions . . . . . . . . . . . . . . . . . . . . . . . . . . . . . . . . . . . . . . . . . . . . . . . . . . 1835

40 Laplaces Equation 1841


40.1 Introduction . . . . . . . . . . . . . . . . . . . . . . . . . . . . . . . . . . . . . . . . . . . . . . . . 1841
40.2 Fundamental Solution . . . . . . . . . . . . . . . . . . . . . . . . . . . . . . . . . . . . . . . . . . . 1841
40.2.1 Two Dimensional Space . . . . . . . . . . . . . . . . . . . . . . . . . . . . . . . . . . . . . . 1842
40.3 Exercises . . . . . . . . . . . . . . . . . . . . . . . . . . . . . . . . . . . . . . . . . . . . . . . . . . 1843
40.4 Hints . . . . . . . . . . . . . . . . . . . . . . . . . . . . . . . . . . . . . . . . . . . . . . . . . . . . 1846
40.5 Solutions . . . . . . . . . . . . . . . . . . . . . . . . . . . . . . . . . . . . . . . . . . . . . . . . . . 1847

41 Waves 1859
41.1 Exercises . . . . . . . . . . . . . . . . . . . . . . . . . . . . . . . . . . . . . . . . . . . . . . . . . . 1860
41.2 Hints . . . . . . . . . . . . . . . . . . . . . . . . . . . . . . . . . . . . . . . . . . . . . . . . . . . . 1866
41.3 Solutions . . . . . . . . . . . . . . . . . . . . . . . . . . . . . . . . . . . . . . . . . . . . . . . . . . 1868

42 Similarity Methods 1888


42.1 Exercises . . . . . . . . . . . . . . . . . . . . . . . . . . . . . . . . . . . . . . . . . . . . . . . . . . 1892
42.2 Hints . . . . . . . . . . . . . . . . . . . . . . . . . . . . . . . . . . . . . . . . . . . . . . . . . . . . 1893
42.3 Solutions . . . . . . . . . . . . . . . . . . . . . . . . . . . . . . . . . . . . . . . . . . . . . . . . . . 1894

43 Method of Characteristics 1897


43.1 First Order Linear Equations . . . . . . . . . . . . . . . . . . . . . . . . . . . . . . . . . . . . . . . . 1897
43.2 First Order Quasi-Linear Equations . . . . . . . . . . . . . . . . . . . . . . . . . . . . . . . . . . . . 1898

xix
43.3 The Method of Characteristics and the Wave Equation . . . . . . . . . . . . . . . . . . . . . . . . . . 1900
43.4 The Wave Equation for an Infinite Domain . . . . . . . . . . . . . . . . . . . . . . . . . . . . . . . . 1901
43.5 The Wave Equation for a Semi-Infinite Domain . . . . . . . . . . . . . . . . . . . . . . . . . . . . . . 1902
43.6 The Wave Equation for a Finite Domain . . . . . . . . . . . . . . . . . . . . . . . . . . . . . . . . . 1904
43.7 Envelopes of Curves . . . . . . . . . . . . . . . . . . . . . . . . . . . . . . . . . . . . . . . . . . . . 1905
43.8 Exercises . . . . . . . . . . . . . . . . . . . . . . . . . . . . . . . . . . . . . . . . . . . . . . . . . . 1908
43.9 Hints . . . . . . . . . . . . . . . . . . . . . . . . . . . . . . . . . . . . . . . . . . . . . . . . . . . . 1910
43.10Solutions . . . . . . . . . . . . . . . . . . . . . . . . . . . . . . . . . . . . . . . . . . . . . . . . . . 1911

44 Transform Methods 1918


44.1 Fourier Transform for Partial Differential Equations . . . . . . . . . . . . . . . . . . . . . . . . . . . . 1918
44.2 The Fourier Sine Transform . . . . . . . . . . . . . . . . . . . . . . . . . . . . . . . . . . . . . . . . 1920
44.3 Fourier Transform . . . . . . . . . . . . . . . . . . . . . . . . . . . . . . . . . . . . . . . . . . . . . 1920
44.4 Exercises . . . . . . . . . . . . . . . . . . . . . . . . . . . . . . . . . . . . . . . . . . . . . . . . . . 1922
44.5 Hints . . . . . . . . . . . . . . . . . . . . . . . . . . . . . . . . . . . . . . . . . . . . . . . . . . . . 1926
44.6 Solutions . . . . . . . . . . . . . . . . . . . . . . . . . . . . . . . . . . . . . . . . . . . . . . . . . . 1928

45 Green Functions 1950


45.1 Inhomogeneous Equations and Homogeneous Boundary Conditions . . . . . . . . . . . . . . . . . . . 1950
45.2 Homogeneous Equations and Inhomogeneous Boundary Conditions . . . . . . . . . . . . . . . . . . . 1951
45.3 Eigenfunction Expansions for Elliptic Equations . . . . . . . . . . . . . . . . . . . . . . . . . . . . . . 1953
45.4 The Method of Images . . . . . . . . . . . . . . . . . . . . . . . . . . . . . . . . . . . . . . . . . . . 1958
45.5 Exercises . . . . . . . . . . . . . . . . . . . . . . . . . . . . . . . . . . . . . . . . . . . . . . . . . . 1960
45.6 Hints . . . . . . . . . . . . . . . . . . . . . . . . . . . . . . . . . . . . . . . . . . . . . . . . . . . . 1971
45.7 Solutions . . . . . . . . . . . . . . . . . . . . . . . . . . . . . . . . . . . . . . . . . . . . . . . . . . 1974

46 Conformal Mapping 2034


46.1 Exercises . . . . . . . . . . . . . . . . . . . . . . . . . . . . . . . . . . . . . . . . . . . . . . . . . . 2035
46.2 Hints . . . . . . . . . . . . . . . . . . . . . . . . . . . . . . . . . . . . . . . . . . . . . . . . . . . . 2038
46.3 Solutions . . . . . . . . . . . . . . . . . . . . . . . . . . . . . . . . . . . . . . . . . . . . . . . . . . 2039

xx
47 Non-Cartesian Coordinates 2051
47.1 Spherical Coordinates . . . . . . . . . . . . . . . . . . . . . . . . . . . . . . . . . . . . . . . . . . . 2051
47.2 Laplaces Equation in a Disk . . . . . . . . . . . . . . . . . . . . . . . . . . . . . . . . . . . . . . . . 2052
47.3 Laplaces Equation in an Annulus . . . . . . . . . . . . . . . . . . . . . . . . . . . . . . . . . . . . . 2055

VI Calculus of Variations 2059


48 Calculus of Variations 2060
48.1 Exercises . . . . . . . . . . . . . . . . . . . . . . . . . . . . . . . . . . . . . . . . . . . . . . . . . . 2061
48.2 Hints . . . . . . . . . . . . . . . . . . . . . . . . . . . . . . . . . . . . . . . . . . . . . . . . . . . . 2075
48.3 Solutions . . . . . . . . . . . . . . . . . . . . . . . . . . . . . . . . . . . . . . . . . . . . . . . . . . 2079

VII Nonlinear Differential Equations 2166


49 Nonlinear Ordinary Differential Equations 2167
49.1 Exercises . . . . . . . . . . . . . . . . . . . . . . . . . . . . . . . . . . . . . . . . . . . . . . . . . . 2168
49.2 Hints . . . . . . . . . . . . . . . . . . . . . . . . . . . . . . . . . . . . . . . . . . . . . . . . . . . . 2173
49.3 Solutions . . . . . . . . . . . . . . . . . . . . . . . . . . . . . . . . . . . . . . . . . . . . . . . . . . 2174

50 Nonlinear Partial Differential Equations 2196


50.1 Exercises . . . . . . . . . . . . . . . . . . . . . . . . . . . . . . . . . . . . . . . . . . . . . . . . . . 2197
50.2 Hints . . . . . . . . . . . . . . . . . . . . . . . . . . . . . . . . . . . . . . . . . . . . . . . . . . . . 2200
50.3 Solutions . . . . . . . . . . . . . . . . . . . . . . . . . . . . . . . . . . . . . . . . . . . . . . . . . . 2201

VIII Appendices 2220


A Greek Letters 2221

xxi
B Notation 2223

C Formulas from Complex Variables 2225

D Table of Derivatives 2228

E Table of Integrals 2232

F Definite Integrals 2236

G Table of Sums 2238

H Table of Taylor Series 2241

I Continuous Transforms 2244


I.1 Properties of Laplace Transforms . . . . . . . . . . . . . . . . . . . . . . . . . . . . . . . . . . . . . 2244
I.2 Table of Laplace Transforms . . . . . . . . . . . . . . . . . . . . . . . . . . . . . . . . . . . . . . . . 2247
I.3 Table of Fourier Transforms . . . . . . . . . . . . . . . . . . . . . . . . . . . . . . . . . . . . . . . . 2250
I.4 Table of Fourier Transforms in n Dimensions . . . . . . . . . . . . . . . . . . . . . . . . . . . . . . . 2253
I.5 Table of Fourier Cosine Transforms . . . . . . . . . . . . . . . . . . . . . . . . . . . . . . . . . . . . 2254
I.6 Table of Fourier Sine Transforms . . . . . . . . . . . . . . . . . . . . . . . . . . . . . . . . . . . . . 2255

J Table of Wronskians 2257

K Sturm-Liouville Eigenvalue Problems 2259

L Green Functions for Ordinary Differential Equations 2261

M Trigonometric Identities 2264


M.1 Circular Functions . . . . . . . . . . . . . . . . . . . . . . . . . . . . . . . . . . . . . . . . . . . . . 2264
M.2 Hyperbolic Functions . . . . . . . . . . . . . . . . . . . . . . . . . . . . . . . . . . . . . . . . . . . . 2266

xxii
N Bessel Functions 2269
N.1 Definite Integrals . . . . . . . . . . . . . . . . . . . . . . . . . . . . . . . . . . . . . . . . . . . . . . 2269

O Formulas from Linear Algebra 2270

P Vector Analysis 2271

Q Partial Fractions 2273

R Finite Math 2276

S Physics 2277

T Probability 2278
T.1 Independent Events . . . . . . . . . . . . . . . . . . . . . . . . . . . . . . . . . . . . . . . . . . . . 2278
T.2 Playing the Odds . . . . . . . . . . . . . . . . . . . . . . . . . . . . . . . . . . . . . . . . . . . . . . 2279

U Economics 2280

V Glossary 2281

W whoami 2283

xxiii
Anti-Copyright

Anti-Copyright @ 1995-2001 by Mauch Publishing Company, un-Incorporated.

No rights reserved. Any part of this publication may be reproduced, stored in a retrieval system, transmitted or
desecrated without permission.

xxiv
Preface

During the summer before my final undergraduate year at Caltech I set out to write a math text unlike any other,
namely, one written by me. In that respect I have succeeded beautifully. Unfortunately, the text is neither complete nor
polished. I have a Warnings and Disclaimers section below that is a little amusing, and an appendix on probability
that I feel concisesly captures the essence of the subject. However, all the material in between is in some stage of
development. I am currently working to improve and expand this text.
This text is freely available from my web set. Currently Im at http://www.its.caltech.edu/sean. I post new
versions a couple of times a year.

0.1 Advice to Teachers


If you have something worth saying, write it down.

0.2 Acknowledgments
I would like to thank Professor Saffman for advising me on this project and the Caltech SURF program for providing
the funding for me to write the first edition of this book.

xxv
0.3 Warnings and Disclaimers
This book is a work in progress. It contains quite a few mistakes and typos. I would greatly appreciate your
constructive criticism. You can reach me at sean@caltech.edu.

Reading this book impairs your ability to drive a car or operate machinery.

This book has been found to cause drowsiness in laboratory animals.

This book contains twenty-three times the US RDA of fiber.

Caution: FLAMMABLE - Do not read while smoking or near a fire.

If infection, rash, or irritation develops, discontinue use and consult a physician.

Warning: For external use only. Use only as directed. Intentional misuse by deliberately concentrating contents
can be harmful or fatal. KEEP OUT OF REACH OF CHILDREN.

In the unlikely event of a water landing do not use this book as a flotation device.

The material in this text is fiction; any resemblance to real theorems, living or dead, is purely coincidental.

This is by far the most amusing section of this book.

Finding the typos and mistakes in this book is left as an exercise for the reader. (Eye ewes a spelling chequer
from thyme too thyme, sew their should knot bee two many misspellings. Though I aint so sure the grammars
too good.)

The theorems and methods in this text are subject to change without notice.

This is a chain book. If you do not make seven copies and distribute them to your friends within ten days of
obtaining this text you will suffer great misfortune and other nastiness.

The surgeon general has determined that excessive studying is detrimental to your social life.

xxvi
This text has been buffered for your protection and ribbed for your pleasure.

Stop reading this rubbish and get back to work!

0.4 Suggested Use


This text is well suited to the student, professional or lay-person. It makes a superb gift. This text has a boquet that
is light and fruity, with some earthy undertones. It is ideal with dinner or as an apertif. Bon apetit!

0.5 About the Title


The title is only making light of naming conventions in the sciences and is not an insult to engineers. If you want to
learn about some mathematical subject, look for books with Introduction or Elementary in the title. If it is an
Intermediate text it will be incomprehensible. If it is Advanced then not only will it be incomprehensible, it will
have low production qualities, i.e. a crappy typewriter font, no graphics and no examples. There is an exception to this
rule: When the title also contains the word Scientists or Engineers the advanced book may be quite suitable for
actually learning the material.

xxvii
Part I

Algebra

1
Chapter 1

Sets and Functions

1.1 Sets
Definition. A set is a collection of objects. We call the objects, elements. A set is denoted by listingthe elements
between braces. For example: {e, , , 1} is the set of the integer 1, the pure imaginary number = 1 and the
transcendental numbers e = 2.7182818 . . . and = 3.1415926 . . .. For elements of a set, we do not count multiplicities.
We regard the set {1, 2, 2, 3, 3, 3} as identical to the set {1, 2, 3}. Order is not significant in sets. The set {1, 2, 3} is
equivalent to {3, 2, 1}.
In enumerating the elements of a set, we use ellipses to indicate patterns. We denote the set of positive integers as
{1, 2, 3, . . .}. We also denote sets with the notation {x|conditions on x} for sets that are more easily described than
enumerated. This is read as the set of elements x such that . . . . x S is the notation for x is an element of the
set S. To express the opposite we have x 6 S for x is not an element of the set S.

Examples. We have notations for denoting some of the commonly encountered sets.

= {} is the empty set, the set containing no elements.

Z = {. . . , 3, 2, 1, 0, 1, 2, 3 . . .} is the set of integers. (Z is for Zahlen, the German word for number.)

2
1
Q = {p/q|p, q Z, q 6= 0} is the set of rational numbers. (Q is for quotient.)
2
R = {x|x = a1 a2 an .b1 b2 } is the set of real numbers, i.e. the set of numbers with decimal expansions.

C = {a + b|a, b R, 2 = 1} is the set of complex numbers. is the square root of 1. (If you havent seen
complex numbers before, dont dismay. Well cover them later.)

Z+ , Q+ and R+ are the sets of positive integers, rationals and reals, respectively. For example, Z+ = {1, 2, 3, . . .}.
We use a superscript to denote the sets of negative numbers.

Z0+ , Q0+ and R0+ are the sets of non-negative integers, rationals and reals, respectively. For example, Z0+ =
{0, 1, 2, . . .}.

(a . . . b) denotes an open interval on the real axis. (a . . . b) {x|x R, a < x < b}

We use brackets to denote the closed interval. [a..b] {x|x R, a x b}


The cardinality or order of a set S is denoted |S|. For finite sets, the cardinality is the number of elements in the
set. The Cartesian product of two sets is the set of ordered pairs:

X Y {(x, y)|x X, y Y }.

The Cartesian product of n sets is the set of ordered n-tuples:

X1 X2 Xn {(x1 , x2 , . . . , xn )|x1 X1 , x2 X2 , . . . , xn Xn }.

Equality. Two sets S and T are equal if each element of S is an element of T and vice versa. This is denoted,
S = T . Inequality is S 6= T , of course. S is a subset of T , S T , if every element of S is an element of T . S is a
proper subset of T , S T , if S T and S 6= T . For example: The empty set is a subset of every set, S. The
rational numbers are a proper subset of the real numbers, Q R.
1
Note that with this description, we enumerate each rational number an infinite number of times. For example: 1/2 = 2/4 =
3/6 = (1)/(2) = . This does not pose a problem as we do not count multiplicities.
2
Guess what R is for.

3
Operations. The union of two sets, S T , is the set whose elements are in either of the two sets. The union of n
sets,
nj=1 Sj S1 S2 Sn
is the set whose elements are in any of the sets Sj . The intersection of two sets, S T , is the set whose elements are
in both of the two sets. In other words, the intersection of two sets in the set of elements that the two sets have in
common. The intersection of n sets,
nj=1 Sj S1 S2 Sn
is the set whose elements are in all of the sets Sj . If two sets have no elements in common, S T = , then the sets
are disjoint. If T S, then the difference between S and T , S \ T , is the set of elements in S which are not in T .

S \ T {x|x S, x 6 T }

The difference of sets is also denoted S T .

Properties. The following properties are easily verified from the above definitions.
S = S, S = , S \ = S, S \ S = .

Commutative. S T = T S, S T = T S.

Associative. (S T ) U = S (T U ) = S T U , (S T ) U = S (T U ) = S T U .

Distributive. S (T U ) = (S T ) (S U ), S (T U ) = (S T ) (S U ).

1.2 Single Valued Functions


Single-Valued Functions. A single-valued function or single-valued mapping is a mapping of the elements x X
f
into elements y Y . This is expressed as f : X Y or X Y . If such a function is well-defined, then for each
x X there exists a unique element of y such that f (x) = y. The set X is the domain of the function, Y is the
codomain, (not to be confused with the range, which we introduce shortly). To denote the value of a function on a

4
particular element we can use any of the notations: f (x) = y, f : x 7 y or simply x 7 y. f is the identity map on
X if f (x) = x for all x X.
Let f : X Y . The range or image of f is
f (X) = {y|y = f (x) for some x X}.
The range is a subset of the codomain. For each Z Y , the inverse image of Z is defined:
f 1 (Z) {x X|f (x) = z for some z Z}.

Examples.
Finite polynomials, f (x) = nk=0 ak xk , ak R, and the exponential function, f (x) = ex , are examples of single
P
valued functions which map real numbers to real numbers.
The greatest integer function, f (x) = bxc, is a mapping from R to Z. bxc is defined as the greatest integer less
than or equal to x. Likewise, the least integer function, f (x) = dxe, is the least integer greater than or equal to
x.

The -jectives. A function is injective if for each x1 6= x2 , f (x1 ) 6= f (x2 ). In other words, distinct elements are
mapped to distinct elements. f is surjective if for each y in the codomain, there is an x such that y = f (x). If a
function is both injective and surjective, then it is bijective. A bijective function is also called a one-to-one mapping.

Examples.
The exponential function f (x) = ex , considered as a mapping from R to R+ , is bijective, (a one-to-one mapping).
f (x) = x2 is a bijection from R+ to R+ . f is not injective from R to R+ . For each positive y in the range, there
are two values of x such that y = x2 .
f (x) = sin x is not injective from R to [1..1]. For each y [1..1] there exists an infinite number of values of
x such that y = sin x.

5
Injective Surjective Bijective

Figure 1.1: Depictions of Injective, Surjective and Bijective Functions

1.3 Inverses and Multi-Valued Functions


If y = f (x), then we can write x = f 1 (y) where f 1 is the inverse of f . If y = f (x) is a one-to-one function, then
f 1 (y) is also a one-to-one function. In this case, x = f 1 (f (x)) = f (f 1 (x)) for values of x where both f (x) and
f 1 (x) are defined. For example ln x, which maps R+ to R is the inverse of ex . x = eln x = ln(ex ) for all x R+ .
(Note the x R+ ensures that ln x is defined.)

If y = f (x) is a many-to-one function, then x = f 1 (y) is a one-to-many function. f 1 (y) is a multi-valued function.
We have x = f (f 1 (x)) for values of x where f 1 (x) is defined, however x 6= f 1 (f (x)). There are diagrams showing
one-to-one, many-to-one and one-to-many functions in Figure 1.2.

Example 1.3.1 y = x2 , a many-to-one function has the inverse x = y 1/2 . For each positive y, there are two values of
x such that x = y 1/2 . y = x2 and y = x1/2 are graphed in Figure 1.3.

there are two branches of y = x1/2


We say that : the positive
and the negative branch. We denote the
positive
1/2
branch as y = x; the negative branch is y = x. We call x the principal branch of x . Notethat x is a
one-to-one function. Finally, x = (x1/2 )2 since ( x)2 = x, but x 6= (x2 )1/2 since (x2 )1/2 = x. y = x is graphed
in Figure 1.4.

6
one-to-one many-to-one one-to-many

domain range domain range domain range

Figure 1.2: Diagrams of One-To-One, Many-To-One and One-To-Many Functions

Figure 1.3: y = x2 and y = x1/2


Figure 1.4: y = x

7
Now consider the many-to-one function y = sin x. The inverse is x = arcsin y. For each y [1..1] there are an
infinite number of values x such that x = arcsin y. In Figure 1.5 is a graph of y = sin x and a graph of a few branches
of y = arcsin x.

Figure 1.5: y = sin x and y = arcsin x

Example 1.3.2 arcsin x has an infinite number of branches. We will denote the principal branch by Arcsin x which
maps [1..1] to 2 .. 2 . Note that x = sin(arcsin x), but x 6= arcsin(sin x). y = Arcsin x in Figure 1.6.

Figure 1.6: y = Arcsin x

Example 1.3.3 Consider 11/3 . Since x3 is a one-to-one function, x1/3 is a single-valued function. (See Figure 1.7.)
11/3 = 1.

Example 1.3.4 Consider arccos(1/2). cos x and a portion of arccos x are graphed in Figure 1.8. The equation
cos x = 1/2 has the two solutions x = /3 in the range x (..]. We use the periodicity of the cosine,

8
Figure 1.7: y = x3 and y = x1/3

cos(x + 2) = cos x, to find the remaining solutions.

arccos(1/2) = {/3 + 2n}, n Z.

Figure 1.8: y = cos x and y = arccos x

1.4 Transforming Equations


Consider the equation g(x) = h(x) and the single-valued function f (x). A particular value of x is a solution of the
equation if substituting that value into the equation results in an identity. In determining the solutions of an equation,
we often apply functions to each side of the equation in order to simplify its form. We apply the function to obtain
a second equation, f (g(x)) = f (h(x)). If x = is a solution of the former equation, (let = g() = h()), then it

9
is necessarily a solution of latter. This is because f (g()) = f (h()) reduces to the identity f () = f (). If f (x) is
bijective, then the converse is true: any solution of the latter equation is a solution of the former equation. Suppose
that x = is a solution of the latter, f (g()) = f (h()). That f (x) is a one-to-one mapping implies that g() = h().
Thus x = is a solution of the former equation.
It is always safe to apply a one-to-one, (bijective), function to an equation, (provided it is defined for that domain).
For example, we can apply f (x) = x3 or f (x) = ex , considered as mappings on R, to the equation x = 1. The
equations x3 = 1 and ex = e each have the unique solution x = 1 for x R.

In general, we must take care in applying functions to equations. If we apply a many-to-one function, we may
2
introduce spurious solutions. Applying f (x) = x2 to the equation x = 2 results in x2 = 4 , which has the two solutions,
2
x = { 2 }. Applying f (x) = sin x results in x2 = 4 , which has an infinite number of solutions, x = { 2 +2n | n Z}.

We do not generally apply a one-to-many, (multi-valued), function to both sides of an equation as this rarely is useful.
Rather, we typically use the definition of the inverse function. Consider the equation
sin2 x = 1.
Applying the function f (x) = x1/2 to the equation would not get us anywhere.
1/2
sin2 x = 11/2
Since (sin2 x)1/2 6= sin x, we cannot simplify the left side of the equation. Instead we could use the definition of
f (x) = x1/2 as the inverse of the x2 function to obtain
sin x = 11/2 = 1.
Now note that we should not just apply arcsin to both sides of the equation as arcsin(sin x) 6= x. Instead we use the
definition of arcsin as the inverse of sin.
x = arcsin(1)
x = arcsin(1) has the solutions x = /2 + 2n and x = arcsin(1) has the solutions x = /2 + 2n. We enumerate
the solutions. n o
x= + n | n Z
2

10
1.5 Exercises
Exercise 1.1
The area of a circle is directly proportional to the square of its diameter. What is the constant of proportionality?
Hint, Solution
Exercise 1.2
Consider the equation
x+1 x2 1
= 2 .
y2 y 4
1. Why might one think that this is the equation of a line?
2. Graph the solutions of the equation to demonstrate that it is not the equation of a line.
Hint, Solution
Exercise 1.3
Consider the function of a real variable,
1
f (x) = .
x2 +2
What is the domain and range of the function?
Hint, Solution
Exercise 1.4
The temperature measured in degrees Celsius 3 is linearly related to the temperature measured in degrees Fahrenheit 4 .
Water freezes at 0 C = 32 F and boils at 100 C = 212 F . Write the temperature in degrees Celsius as a function
of degrees Fahrenheit.
3
Originally, it was called degrees Centigrade. centi because there are 100 degrees between the two calibration points. It is now
called degrees Celsius in honor of the inventor.
4
The Fahrenheit scale, named for Daniel Fahrenheit, was originally calibrated with the freezing point of salt-saturated water to
be 0 . Later, the calibration points became the freezing point of water, 32 , and body temperature, 96 . With this method, there are
64 divisions between the calibration points. Finally, the upper calibration point was changed to the boiling point of water at 212 .
This gave 180 divisions, (the number of degrees in a half circle), between the two calibration points.

11
Hint, Solution
Exercise 1.5
Consider the function graphed in Figure 1.9. Sketch graphs of f (x), f (x + 3), f (3 x) + 2, and f 1 (x). You may
use the blank grids in Figure 1.10.

Figure 1.9: Graph of the function.

Hint, Solution
Exercise 1.6
A culture of bacteria grows at the rate of 10% per minute. At 6:00 pm there are 1 billion bacteria. How many bacteria
are there at 7:00 pm? How many were there at 3:00 pm?
Hint, Solution
Exercise 1.7
The graph in Figure 1.11 shows an even function f (x) = p(x)/q(x) where p(x) and q(x) are rational quadratic
polynomials. Give possible formulas for p(x) and q(x).
Hint, Solution

12
Figure 1.10: Blank grids.

Exercise 1.8
Find a polynomial of degree 100 which is zero only at x = 2, 1, and is non-negative.
Hint, Solution

13
2 2

1 1

1 2 2 4 6 8 10

Figure 1.11: Plots of f (x) = p(x)/q(x).

1.6 Hints
Hint 1.1
area = constant diameter2 .

Hint 1.2
A pair (x, y) is a solution of the equation if it make the equation an identity.

Hint 1.3
The domain is the subset of R on which the function is defined.

Hint 1.4
Find the slope and x-intercept of the line.

Hint 1.5
The inverse of the function is the reflection of the function across the line y = x.

Hint 1.6
The formula for geometric growth/decay is x(t) = x0 rt , where r is the rate.

14
Hint 1.7
Note that p(x) and q(x) appear as a ratio, they are determined only up to a multiplicative constant. We may take the
leading coefficient of q(x) to be unity.
p(x) ax2 + bx + c
f (x) = = 2
q(x) x + x +
Use the properties of the function to solve for the unknown parameters.
Hint 1.8
Write the polynomial in factored form.

15
1.7 Solutions
Solution 1.1

area = radius2

area = diameter2
4
The constant of proportionality is 4 .

Solution 1.2
1. If we multiply the equation by y 2 4 and divide by x + 1, we obtain the equation of a line.
y+2=x1

2. We factor the quadratics on the right side of the equation.


x+1 (x + 1)(x 1)
= .
y2 (y 2)(y + 2)
We note that one or both sides of the equation are undefined at y = 2 because of division by zero. There are
no solutions for these two values of y and we assume from this point that y 6= 2. We multiply by (y 2)(y + 2).
(x + 1)(y + 2) = (x + 1)(x 1)

For x = 1, the equation becomes the identity 0 = 0. Now we consider x 6= 1. We divide by x + 1 to obtain
the equation of a line.
y+2=x1
y =x3
Now we collect the solutions we have found.
{(1, y) : y 6= 2} {(x, x 3) : x 6= 1, 5}
The solutions are depicted in Figure /reffig not a line.

16
6

-6 -4 -2 2 4 6

-2

-4

-6

x+1 x2 1
Figure 1.12: The solutions of y2
= y 2 4
.

Solution 1.3
The denominator is nonzero for all x R. Since we dont have any division by zero problems, the domain of the
function is R. For x R,
1
0< 2 2.
x +2
Consider
1
y= 2 . (1.1)
x +2
For any y (0 . . . 1/2], there is at least one value of x that satisfies Equation 1.1.

1
x2 + 2 =
y
r
1
x= 2
y

Thus the range of the function is (0 . . . 1/2]

17
Solution 1.4
Let c denote degrees Celsius and f denote degrees Fahrenheit. The line passes through the points (f, c) = (32, 0) and
(f, c) = (212, 100). The x-intercept is f = 32. We calculate the slope of the line.
100 0 100 5
slope = = =
212 32 180 9
The relationship between fahrenheit and celcius is

5
c = (f 32).
9
Solution 1.5
We plot the various transformations of f (x).

Solution 1.6
The formula for geometric growth/decay is x(t) = x0 rt , where r is the rate. Let t = 0 coincide with 6:00 pm. We
determine x0 .
 0
9 11
x(0) = 10 = x0 = x0
10
x0 = 109

At 7:00 pm the number of bacteria is


60
1160

9 11
10 = 51
3.04 1011
10 10

At 3:00 pm the number of bacteria was


180
10189

9 11
10 = 35.4
10 11180

18
Figure 1.13: Graphs of f (x), f (x + 3), f (3 x) + 2, and f 1 (x).

Solution 1.7
We write p(x) and q(x) as general quadratic polynomials.

p(x) ax2 + bx + c
f (x) = =
q(x) x2 + x +
We will use the properties of the function to solve for the unknown parameters.

19
Note that p(x) and q(x) appear as a ratio, they are determined only up to a multiplicative constant. We may take
the leading coefficient of q(x) to be unity.

p(x) ax2 + bx + c
f (x) = = 2
q(x) x + x +
f (x) has a second order zero at x = 0. This means that p(x) has a second order zero there and that 6= 0.

ax2
f (x) =
x2 + x +
We note that f (x) 2 as x . This determines the parameter a.

ax2
lim f (x) = lim
x x x2 + x +
2ax
= lim
x 2x +
2a
= lim
x 2
=a

2x2
f (x) =
x2 + x +
Now we use the fact that f (x) is even to conclude that q(x) is even and thus = 0.

2x2
f (x) =
x2 +
Finally, we use that f (1) = 1 to determine .
2x2
f (x) =
x2 + 1

20
Solution 1.8
Consider the polynomial
p(x) = (x + 2)40 (x 1)30 (x )30 .
It is of degree 100. Since the factors only vanish at x = 2, 1, , p(x) only vanishes there. Since factors are non-
negative, the polynomial is non-negative.

21
Chapter 2

Vectors

2.1 Vectors
2.1.1 Scalars and Vectors
A vector is a quantity having both a magnitude and a direction. Examples of vector quantities are velocity, force
and position. One can represent a vector in n-dimensional space with an arrow whose initial point is at the origin,
(Figure 2.1). The magnitude is the length of the vector. Typographically, variables representing vectors are often
written in capital letters, bold face or with a vector over-line, A, a, ~a. The magnitude of a vector is denoted |a|.
A scalar has only a magnitude. Examples of scalar quantities are mass, time and speed.

Vector Algebra. Two vectors are equal if they have the same magnitude and direction. The negative of a vector,
denoted a, is a vector of the same magnitude as a but in the opposite direction. We add two vectors a and b by
placing the tail of b at the head of a and defining a + b to be the vector with tail at the origin and head at the head
of b. (See Figure 2.2.)
The difference, a b, is defined as the sum of a and the negative of b, a + (b). The result of multiplying a by
a scalar is a vector of magnitude || |a| with the same/opposite direction if is positive/negative. (See Figure 2.2.)

22
z

Figure 2.1: Graphical representation of a vector in three dimensions.

2a
b
a
a
a+b -a

Figure 2.2: Vector arithmetic.

Here are the properties of adding vectors and multiplying them by a scalar. They are evident from geometric

23
considerations.

a+b=b+a a = a commutative laws


(a + b) + c = a + (b + c) (a) = ()a associative laws
(a + b) = a + b ( + )a = a + a distributive laws

Zero and Unit Vectors. The additive identity element for vectors is the zero vector or null vector. This is a vector
of magnitude zero which is denoted as 0. A unit vector is a vector of magnitude one. If a is nonzero then a/|a| is a
unit vector in the direction of a. Unit vectors are often denoted with a caret over-line, n.

Rectangular Unit Vectors. In n dimensional Cartesian space, Rn , the unit vectors in the directions of the
coordinates axes are e1 , . . . en . These are called the rectangular unit vectors. To cut down on subscripts, the unit
vectors in three dimensional space are often denoted with i, j and k. (Figure 2.3).

j
y

Figure 2.3: Rectangular unit vectors.

24
Components of a Vector. Consider a vector a with tail at the origin and head having the Cartesian coordinates
(a1 , . . . , an ). We can represent this vector as the sum of n rectangular component vectors, a = a1 e1 + + an en .
(See Figure 2.4.) Another p notation for the vector a is ha1 , . . . , an i. By the Pythagorean theorem, the magnitude of
2
the vector a is |a| = a1 + + a2n .
z

a
a3 k
y
a1 i

a2 j

Figure 2.4: Components of a vector.

2.1.2 The Kronecker Delta and Einstein Summation Convention


The Kronecker Delta tensor is defined (
1 if i = j,
ij =
0 if i 6= j.
This notation will be useful in our work with vectors.

Consider writing a vector in terms of its rectangular components. Instead of using ellipses: a = a1 e1 + + an en , we
could write the expression as a sum: a = ni=1 ai ei . We can shorten this notation by leaving out the sum: a = ai ei ,
P
where it is understood that whenever an index is repeated in a term we sum over that index from 1 to n. This is the

25
Einstein summation convention. A repeated index is called a summation index or a dummy index. Other indices can
take any value from 1 to n and are called free indices.

Example 2.1.1 Consider the matrix equation: A x = b. We can write out the matrix and vectors explicitly.

a11 a1n x1 b1
.. . . .. .. = ..
. . . . .
an1 ann xn bn
This takes much less space when we use the summation convention.
aij xj = bi
Here j is a summation index and i is a free index.

2.1.3 The Dot and Cross Product


Dot Product. The dot product or scalar product of two vectors is defined,
a b |a||b| cos ,
where is the angle from a to b. From this definition one can derive the following properties:
a b = b a, commutative.
(a b) = (a) b = a (b), associativity of scalar multiplication.
a (b + c) = a b + a c, distributive. (See Exercise 2.1.)
ei ej = ij . In three dimensions, this is
i i = j j = k k = 1, i j = j k = k i = 0.

a b = ai bi a1 b1 + + an bn , dot product in terms of rectangular components.


If a b = 0 then either a and b are orthogonal, (perpendicular), or one of a and b are zero.

26
The Angle Between Two Vectors. We can use the dot product to find the angle between two vectors, a and
b. From the definition of the dot product,
a b = |a||b| cos .
If the vectors are nonzero, then  
ab
= arccos .
|a||b|

Example 2.1.2 What is the angle between i and i + j?


 
i (i + j)
= arccos
|i||i + j|
 
1
= arccos
2

= .
4

Parametric Equation of a Line. Consider a line in Rn that passes through the point a and is parallel to the
vector t, (tangent). A parametric equation of the line is

x = a + ut, u R.

Implicit Equation of a Line In 2D. Consider a line in R2 that passes through the point a and is normal,
(orthogonal, perpendicular), to the vector n. All the lines that are normal to n have the property that x n is a
constant, where x is any point on the line. (See Figure 2.5.) x n = 0 is the line that is normal to n and passes
through the origin. The line that is normal to n and passes through the point a is

x n = a n.

The normal to a line determines an orientation of the line. The normal points in the direction that is above the
line. A point b is (above/on/below) the line if (b a) n is (positive/zero/negative). The signed distance of a point

27
x n=1 x n= a n

n a
x n=0

x n=-1

Figure 2.5: Equation for a line.

b from the line x n = a n is


n
(b a) .
|n|

Implicit Equation of a Hyperplane. A hyperplane in Rn is an n 1 dimensional sheet which passes through


a given point and is normal to a given direction. In R3 we call this a plane. Consider a hyperplane that passes through
the point a and is normal to the vector n. All the hyperplanes that are normal to n have the property that x n is a
constant, where x is any point in the hyperplane. x n = 0 is the hyperplane that is normal to n and passes through
the origin. The hyperplane that is normal to n and passes through the point a is
x n = a n.
The normal determines an orientation of the hyperplane. The normal points in the direction that is above the
hyperplane. A point b is (above/on/below) the hyperplane if (b a) n is (positive/zero/negative). The signed

28
distance of a point b from the hyperplane x n = a n is
n
(b a) .
|n|

Right and Left-Handed Coordinate Systems. Consider a rectangular coordinate system in two dimensions.
Angles are measured from the positive x axis in the direction of the positive y axis. There are two ways of labeling the
axes. (See Figure 2.6.) In one the angle increases in the counterclockwise direction and in the other the angle increases
in the clockwise direction. The former is the familiar Cartesian coordinate system.

y x


x y

Figure 2.6: There are two ways of labeling the axes in two dimensions.

There are also two ways of labeling the axes in a three-dimensional rectangular coordinate system. These are called
right-handed and left-handed coordinate systems. See Figure 2.7. Any other labelling of the axes could be rotated into
one of these configurations. The right-handed system is the one that is used by default. If you put your right thumb in
the direction of the z axis in a right-handed coordinate system, then your fingers curl in the direction from the x axis
to the y axis.

Cross Product. The cross product or vector product is defined,

a b = |a||b| sin n,

where is the angle from a to b and n is a unit vector that is orthogonal to a and b and in the direction such that
the ordered triple of vectors a, b and n form a right-handed system.

29
z z

k k
j y i x
i j
x y

Figure 2.7: Right and left handed coordinate systems.

You can visualize the direction of a b by applying the right hand rule. Curl the fingers of your right hand in the
direction from a to b. Your thumb points in the direction of a b. Warning: Unless you are a lefty, get in the habit
of putting down your pencil before applying the right hand rule.

The dot and cross products behave a little differently. First note that unlike the dot product, the cross product is not
commutative. The magnitudes of a b and b a are the same, but their directions are opposite. (See Figure 2.8.)
Let
a b = |a||b| sin n and b a = |b||a| sin m.
The angle from a to b is the same as the angle from b to a. Since {a, b, n} and {b, a, m} are right-handed systems,
m points in the opposite direction as n. Since a b = b a we say that the cross product is anti-commutative.

Next we note that since


|a b| = |a||b| sin ,
the magnitude of a b is the area of the parallelogram defined by the two vectors. (See Figure 2.9.) The area of the
triangle defined by two vectors is then 12 |a b|.

From the definition of the cross product, one can derive the following properties:

30
a b

a
b a

Figure 2.8: The cross product is anti-commutative.

b b
b sin

a a

Figure 2.9: The parallelogram and the triangle defined by two vectors.

a b = b a, anti-commutative.

(a b) = (a) b = a (b), associativity of scalar multiplication.

a (b + c) = a b + a c, distributive.

(a b) c 6= a (b c). The cross product is not associative.

i i = j j = k k = 0.

31
i j = k, j k = i, k i = j.


i j k

a b = (a2 b3 a3 b2 )i + (a3 b1 a1 b3 )j + (a1 b2 a2 b1 )k = a1 a2 a3 ,
b1 b2 b3
cross product in terms of rectangular components.

If a b = 0 then either a and b are parallel or one of a or b is zero.

Scalar Triple Product. Consider the volume of the parallelopiped defined by three vectors. (See Figure 2.10.)
The area of the base is ||b||c| sin |, where is the angle between b and c. The height is |a| cos , where is the angle
between b c and a. Thus the volume of the parallelopiped is |a||b||c| sin cos .

b c
a

c

Figure 2.10: The parallelopiped defined by three vectors.

Note that

|a (b c)| = |a (|b||c| sin n)|


= ||a||b||c| sin cos | .

32
Thus |a (b c)| is the volume of the parallelopiped. a (b c) is the volume or the negative of the volume depending
on whether {a, b, c} is a right or left-handed system.
Note that parentheses are unnecessary in a b c. There is only one way to interpret the expression. If you did the
dot product first then you would be left with the cross product of a scalar and a vector which is meaningless. a b c
is called the scalar triple product.

Plane Defined by Three Points. Three points which are not collinear define a plane. Consider a plane that
passes through the three points a, b and c. One way of expressing that the point x lies in the plane is that the vectors
x a, b a and c a are coplanar. (See Figure 2.11.) If the vectors are coplanar, then the parallelopiped defined by
these three vectors will have zero volume. We can express this in an equation using the scalar triple product,
(x a) (b a) (c a) = 0.

c
a
b

Figure 2.11: Three points define a plane.

2.2 Sets of Vectors in n Dimensions


Orthogonality. Consider two n-dimensional vectors
x = (x1 , x2 , . . . , xn ), y = (y1 , y2 , . . . , yn ).

33
The inner product of these vectors can be defined
n
X
hx|yi x y = xi yi .
i=1

The vectors are orthogonal if x y = 0. The norm of a vector is the length of the vector generalized to n dimensions.

kxk = x x

Consider a set of vectors


{x1 , x2 , . . . , xm }.
If each pair of vectors in the set is orthogonal, then the set is orthogonal.

xi xj = 0 if i 6= j

If in addition each vector in the set has norm 1, then the set is orthonormal.
(
1 if i = j
xi xj = ij =
0 if i 6= j

Here ij is known as the Kronecker delta function.

Completeness. A set of n, n-dimensional vectors

{x1 , x2 , . . . , xn }

is complete if any n-dimensional vector can be written as a linear combination of the vectors in the set. That is, any
vector y can be written
X n
y= ci xi .
i=1

34
Taking the inner product of each side of this equation with xm ,
n
!
X
y xm = ci xi xm
i=1
n
X
= ci xi xm
i=1
= cm xm xm
y xm
cm =
kxm k2

Thus y has the expansion


n
X y xi
y= xi .
i=1
kxi k2
If in addition the set is orthonormal, then
n
X
y= (y xi )xi .
i=1

35
2.3 Exercises
The Dot and Cross Product
Exercise 2.1
Prove the distributive law for the dot product,

a (b + c) = a b + a c.

Hint, Solution
Exercise 2.2
Prove that
a b = ai bi a1 b1 + + an bn .
Hint, Solution
Exercise 2.3
What is the angle between the vectors i + j and i + 3j?
Hint, Solution
Exercise 2.4
Prove the distributive law for the cross product,

a (b + c) = a b + a b.

Hint, Solution
Exercise 2.5
Show that
i j k

a b = a1 a2 a3
b1 b2 b3
Hint, Solution

36
Exercise 2.6
What is the area of the quadrilateral with vertices at (1, 1), (4, 2), (3, 7) and (2, 3)?
Hint, Solution
Exercise 2.7
What is the volume of the tetrahedron with vertices at (1, 1, 0), (3, 2, 1), (2, 4, 1) and (1, 2, 5)?
Hint, Solution
Exercise 2.8
What is the equation of the plane that passes through the points (1, 2, 3), (2, 3, 1) and (3, 1, 2)? What is the distance
from the point (2, 3, 5) to the plane?
Hint, Solution

37
2.4 Hints
The Dot and Cross Product
Hint 2.1
First prove the distributive law when the first vector is of unit length,
n (b + c) = n b + n c.
Then all the quantities in the equation are projections onto the unit vector n and you can use geometry.
Hint 2.2
First prove that the dot product of a rectangular unit vector with itself is one and the dot product of two distinct
rectangular unit vectors is zero. Then write a and b in rectangular components and use the distributive law.
Hint 2.3
Use a b = |a||b| cos .

Hint 2.4
First consider the case that both b and c are orthogonal to a. Prove the distributive law in this case from geometric
considerations.
Next consider two arbitrary vectors a and b. We can write b = b + bk where b is orthogonal to a and bk is
parallel to a. Show that
a b = a b .
Finally prove the distributive law for arbitrary b and c.
Hint 2.5
Write the vectors in their rectangular components and use,
i j = k, j k = i, k i = j,
and,
i i = j j = k k = 0.

38
Hint 2.6
The quadrilateral is composed of two triangles. The area of a triangle defined by the two vectors a and b is 21 |a b|.

Hint 2.7
Justify that the area of a tetrahedron determined by three vectors is one sixth the area of the parallelogram determined
by those three vectors. The area of a parallelogram determined by three vectors is the magnitude of the scalar triple
product of the vectors: a b c.
Hint 2.8
The equation of a line that is orthogonal to a and passes through the point b is a x = a b. The distance of a point
c from the plane is
(c b) a

|a|

39
2.5 Solutions
The Dot and Cross Product
Solution 2.1
First we prove the distributive law when the first vector is of unit length, i.e.,
n (b + c) = n b + n c. (2.1)
From Figure 2.12 we see that the projection of the vector b + c onto n is equal to the sum of the projections b n and
c n.
Now we extend the result to the case when the first vector has arbitrary length. We define a = |a|n and multiply
Equation 2.1 by the scalar, |a|.
|a|n (b + c) = |a|n b + |a|n c
a (b + c) = a b + a c.
Solution 2.2
First note that
ei ei = |ei ||ei | cos(0) = 1.
Then note that that dot product of any two distinct rectangular unit vectors is zero because they are orthogonal. Now
we write a and b in terms of their rectangular components and use the distributive law.
a b = ai e i b j e j
= ai b j e i e j
= ai bj ij
= ai b i
Solution 2.3
Since a b = |a||b| cos , we have  
ab
= arccos
|a||b|

40
c
n
b b+c

nc
nb n (b+c)

Figure 2.12: The distributive law for the dot product.

when a and b are nonzero.


    !
(i + j) (i + 3j) 4 2 5
= arccos = arccos = arccos 0.463648
|i + j||i + 3j| 2 10 5

Solution 2.4
First consider the case that both b and c are orthogonal to a. b + c is the diagonal of the parallelogram defined by
b and c, (see Figure 2.13). Since a is orthogonal to each of these vectors, taking the cross product of a with these
vectors has the effect of rotating the vectors through /2 radians about a and multiplying their length by |a|. Note

41
that a (b + c) is the diagonal of the parallelogram defined by a b and a c. Thus we see that the distributive law
holds when a is orthogonal to both b and c,

a (b + c) = a b + a c.

b a c

b+c c
a b

a (b+c)

Figure 2.13: The distributive law for the cross product.

Now consider two arbitrary vectors a and b. We can write b = b + bk where b is orthogonal to a and bk is
parallel to a, (see Figure 2.14).
By the definition of the cross product,
a b = |a||b| sin n.
Note that
|b | = |b| sin ,

42
a

b
b

Figure 2.14: The vector b written as a sum of components orthogonal and parallel to a.

and that a b is a vector in the same direction as a b. Thus we see that

a b = |a||b| sin n
= |a|(sin |b|)n
= |a||b |n = |a||b | sin(/2)n

a b = a b .
Now we are prepared to prove the distributive law for arbitrary b and c.

a (b + c) = a (b + bk + c + ck )
= a ((b + c) + (b + c)k )
= a ((b + c) )
= a b + a c
=ab+ac

a (b + c) = a b + a c

43
Solution 2.5
We know that
i j = k, j k = i, k i = j,
and that
i i = j j = k k = 0.
Now we write a and b in terms of their rectangular components and use the distributive law to expand the cross
product.

a b = (a1 i + a2 j + a3 k) (b1 i + b2 j + b3 k)
= a1 i (b1 i + b2 j + b3 k) + a2 j (b1 i + b2 j + b3 k) + a3 k (b1 i + b2 j + b3 k)
= a1 b2 k + a1 b3 (j) + a2 b1 (k) + a2 b3 i + a3 b1 j + a3 b2 (i)
= (a2 b3 a3 b2 )i (a1 b3 a3 b1 )j + (a1 b2 a2 b1 )k

Next we evaluate the determinant.



i j k
a1 a2 a3 = i a2 a3 j a1 a3 + k a1 a2

b2 b3 b1 b3 b1 b2
b1 b2 b3
= (a2 b3 a3 b2 )i (a1 b3 a3 b1 )j + (a1 b2 a2 b1 )k

Thus we see that,


i j k

a b = a1 a2 a3
b1 b2 b3

Solution 2.6
The area area of the quadrilateral is the area of two triangles. The first triangle is defined by the vector from (1, 1) to
(4, 2) and the vector from (1, 1) to (2, 3). The second triangle is defined by the vector from (3, 7) to (4, 2) and the
vector from (3, 7) to (2, 3). (See Figure 2.15.) The area of a triangle defined by the two vectors a and b is 12 |a b|.

44
The area of the quadrilateral is then,

1 1 1 1
|(3i + j) (i + 2j)| + |(i 5j) (i 4j)| = (5) + (19) = 12.
2 2 2 2

y (3,7)

(2,3)
(4,2)
(1,1) x

Figure 2.15: Quadrilateral.

Solution 2.7
The tetrahedron is determined by the three vectors with tail at (1, 1, 0) and heads at (3, 2, 1), (2, 4, 1) and (1, 2, 5).
These are h2, 1, 1i, h1, 3, 1i and h0, 1, 5i. The area of the tetrahedron is one sixth the area of the parallelogram
determined by these vectors. (This is because the area of a pyramid is 13 (base)(height). The base of the tetrahedron is
half the area of the parallelogram and the heights are the same. 21 31 = 16 ) Thus the area of a tetrahedron determined
by three vectors is 16 |a b c|. The area of the tetrahedron is

1 1 7
|h2, 1, 1i h1, 3, 1i h1, 2, 5i| = |h2, 1, 1i h13, 4, 1i| =
6 6 2

45
Solution 2.8
The two vectors with tails at (1, 2, 3) and heads at (2, 3, 1) and (3, 1, 2) are parallel to the plane. Taking the cross
product of these two vectors gives us a vector that is orthogonal to the plane.

h1, 1, 2i h2, 1, 1i = h3, 3, 3i

We see that the plane is orthogonal to the vector h1, 1, 1i and passes through the point (1, 2, 3). The equation of the
plane is
h1, 1, 1i hx, y, zi = h1, 1, 1i h1, 2, 3i,
x + y + z = 6.
Consider the vector with tail at (1, 2, 3) and head at (2, 3, 5). The magnitude of the dot product of this vector with
the unit normal vector gives the distance from the plane.


h1, 1, 2i h1, 1, 1i 4
= = 4 3
|h1, 1, 1i| 3 3

46
Part II

Calculus

47
Chapter 3

Differential Calculus

3.1 Limits of Functions


Definition of a Limit. If the value of the function y(x) gets arbitrarily close to as x approaches the point ,
then we say that the limit of the function as x approaches is equal to . This is written:

lim y(x) =
x

Now we make the notion of arbitrarily close precise. For any  > 0 there exists a > 0 such that |y(x) | < 
for all 0 < |x | < . That is, there is an interval surrounding the point x = for which the function is within  of
. See Figure 3.1. Note that the interval surrounding x = is a deleted neighborhood, that is it does not contain the
point x = . Thus the value of the function at x = need not be equal to for the limit to exist. Indeed the function
need not even be defined at x = .
To prove that a function has a limit at a point we first bound |y(x) | in terms of for values of x satisfying
0 < |x | < . Denote this upper bound by u(). Then for an arbitrary  > 0, we determine a > 0 such that the
the upper bound u() and hence |y(x) | is less than .

48
y

x
+

Figure 3.1: The neighborhood of x = such that |y(x) | < .

Example 3.1.1 Show that


lim x2 = 1.
x1

Consider any  > 0. We need to show that there exists a > 0 such that |x2 1| <  for all |x 1| < . First we
obtain a bound on |x2 1|.

|x2 1| = |(x 1)(x + 1)|


= |x 1||x + 1|
< |x + 1|
= |(x 1) + 2|
< ( + 2)

Now we choose a positive such that,


( + 2) = .
We see that
= 1 +  1,
is positive and satisfies the criterion that |x2 1| <  for all 0 < |x 1| < . Thus the limit exists.

49
Example 3.1.2 Recall that the value of the function y() need not be equal to limx y(x) for the limit to exist. We
show an example of this. Consider the function
(
1 for x Z,
y(x) =
0 for x
6 Z.

For what values of does limx y(x) exist?


First consider 6 Z. Then there exists an open neighborhood a < < b around such that y(x) is identically zero
for x (a, b). Then trivially, limx y(x) = 0.
Now consider Z. Consider any  > 0. Then if |x | < 1 then |y(x) 0| = 0 < . Thus we see that
limx y(x) = 0.
Thus, regardless of the value of , limx y(x) = 0.

Left and Right Limits. With the notation limx+ y(x) we denote the right limit of y(x). This is the limit as x
approaches from above. Mathematically: limx+ exists if for any  > 0 there exists a > 0 such that |y(x) | < 
for all 0 < x < . The left limit limx y(x) is defined analogously.
sin x
Example 3.1.3 Consider the function, |x|
, defined for x 6= 0. (See Figure 3.2.) The left and right limits exist as x
approaches zero.
sin x sin x
lim+ = 1, lim = 1
x0 |x| x0 |x|
However the limit,
sin x
lim ,
x0 |x|
does not exist.

Properties of Limits. Let lim f (x) and lim g(x) exist.


x x

lim (af (x) + bg(x)) = a lim f (x) + b lim g(x).


x x x

50
Figure 3.2: Plot of sin(x)/|x|.
  
lim (f (x)g(x)) = lim f (x) lim g(x) .
x x x
 
f (x) limx f (x)
lim = if lim g(x) 6= 0.
x g(x) limx g(x) x

Example 3.1.4 We prove that if limx f (x) = and limx g(x) = exist then
  
lim (f (x)g(x)) = lim f (x) lim g(x) .
x x x

Since the limit exists for f (x), we know that for all  > 0 there exists > 0 such that |f (x) | <  for |x | < .
Likewise for g(x). We seek to show that for all  > 0 there exists > 0 such that |f (x)g(x) | <  for |x | < .
We proceed by writing |f (x)g(x) |, in terms of |f (x) | and |g(x) |, which we know how to bound.

|f (x)g(x) | = |f (x)(g(x) ) + (f (x) )|


|f (x)||g(x) | + |f (x) |||

If we choose a such that |f (x)||g(x) | < /2 and |f (x) ||| < /2 then we will have the desired result:
|f (x)g(x) | < . Trying to ensure that |f (x)||g(x) | < /2 is hard because of the |f (x)| factor. We will replace
that factor with a constant. We want to write |f (x) ||| < /2 as |f (x) | < /(2||), but this is problematic for
the case = 0. We fix these two problems and then proceed. We choose 1 such that |f (x) | < 1 for |x | < 1 .

51
This gives us the desired form.

|f (x)g(x) | (|| + 1)|g(x) | + |f (x) |(|| + 1), for |x | < 1

Next we choose 2 such that |g(x)| < /(2(||+1)) for |x| < 2 and choose 3 such that |f (x)| < /(2(||+1))
for |x | < 3 . Let be the minimum of 1 , 2 and 3 .

 
|f (x)g(x) | (|| + 1)|g(x) | + |f (x) |(|| + 1) < + , for |x | <
2 2
|f (x)g(x) | < , for |x | <

We conclude that the limit of a product is the product of the limits.

  
lim (f (x)g(x)) = lim f (x) lim g(x) = .
x x x

52
Result 3.1.1 Definition of a Limit. The statement:

lim y(x) =
x

means that y(x) gets arbitrarily close to as x approaches . For any  > 0 there exists a
> 0 such that |y(x) | <  for all x in the neighborhood 0 < |x | < . The left and
right limits,
lim y(x) = and lim+ y(x) =
x x

denote the limiting value as x approaches respectively from below and above. The neigh-
borhoods are respectively < x < 0 and 0 < x < .
Properties of Limits. Let lim u(x) and lim v(x) exist.
x x

lim (au(x) + bv(x)) = a lim u(x) + b lim v(x).


x x x
  
lim (u(x)v(x)) = lim u(x) lim v(x) .
x x x
 
u(x) limx u(x)
lim = if lim v(x) 6= 0.
x v(x) limx v(x) x

3.2 Continuous Functions


Definition of Continuity. A function y(x) is said to be continuous at x = if the value of the function is
equal to its limit, that is, limx y(x) = y(). Note that this one condition is actually the three conditions: y() is

53
defined, limx y(x) exists and limx y(x) = y(). A function is continuous if it is continuous at each point in its
domain. A function is continuous on the closed interval [a, b] if the function is continuous for each point x (a, b) and
limxa+ y(x) = y(a) and limxb y(x) = y(b).

Discontinuous Functions. If a function is not continuous at a point it is called discontinuous at that point. If
limx y(x) exists but is not equal to y(), then the function has a removable discontinuity. It is thus named because
we could define a continuous function
(
y(x) for x 6= ,
z(x) =
limx y(x) for x = ,

to remove the discontinuity. If both the left and right limit of a function at a point exist, but are not equal, then the
function has a jump discontinuity at that point. If either the left or right limit of a function does not exist, then the
function is said to have an infinite discontinuity at that point.
sin x
Example 3.2.1 x
has a removable discontinuity at x = 0. The Heaviside function,

0
for x < 0,
H(x) = 1/2 for x = 0,

1 for x > 0,

1
has a jump discontinuity at x = 0. x
has an infinite discontinuity at x = 0. See Figure 3.3.

Properties of Continuous Functions.


u(x)
Arithmetic. If u(x) and v(x) are continuous at x = then u(x) v(x) and u(x)v(x) are continuous at x = . v(x)
is continuous at x = if v() 6= 0.

Function Composition. If u(x) is continuous at x = and v(x) is continuous at x = = u() then u(v(x)) is
continuous at x = . The composition of continuous functions is a continuous function.

54
Figure 3.3: A Removable discontinuity, a Jump Discontinuity and an Infinite Discontinuity

Boundedness. A function which is continuous on a closed interval is bounded in that closed interval.

Nonzero in a Neighborhood. If y() 6= 0 then there exists a neighborhood ( , + ),  > 0 of the point such
that y(x) 6= 0 for x ( , + ).

Intermediate Value Theorem. Let u(x) be continuous on [a, b]. If u(a) u(b) then there exists [a, b] such
that u() = . This is known as the intermediate value theorem. A corollary of this is that if u(a) and u(b) are
of opposite sign then u(x) has at least one zero on the interval (a, b).

Maxima and Minima. If u(x) is continuous on [a, b] then u(x) has a maximum and a minimum on [a, b]. That is, there
is at least one point [a, b] such that u() u(x) for all x [a, b] and there is at least one point [a, b]
such that u() u(x) for all x [a, b].

Piecewise Continuous Functions. A function is piecewise continuous on an interval if the function is bounded on
the interval and the interval can be divided into a finite number of intervals on each of which the function is continuous.
For example, the greatest integer function, bxc, is piecewise continuous. (bxc is defined to the the greatest integer less
than or equal to x.) See Figure 3.4 for graphs of two piecewise continuous functions.

Uniform Continuity. Consider a function f (x) that is continuous on an interval. This means that for any point
in the interval and any positive  there exists a > 0 such that |f (x) f ()| <  for all 0 < |x | < . In general,
this value of depends on both and . If can be chosen so it is a function of  alone and independent of then

55
Figure 3.4: Piecewise Continuous Functions

the function is said to be uniformly continuous on the interval. A sufficient condition for uniform continuity is that the
function is continuous on a closed interval.

3.3 The Derivative


Consider a function y(x) on the interval (x . . . x + x) for some x > 0. We define the increment y = y(x + x)
y
y(x). The average rate of change, (average velocity), of the function on the interval is x . The average rate of change
is the slope of the secant line that passes through the points (x, y(x)) and (x + x, y(x + x)). See Figure 3.5.

y
x
x

Figure 3.5: The increments x and y.

56
If the slope of the secant line has a limit as x approaches zero then we call this slope the derivative or instantaneous
dy
rate of change of the function at the point x. We denote the derivative by dx , which is a nice notation as the derivative
y
is the limit of x as x 0.
dy y(x + x) y(x)
lim .
dx x0 x
dy
x may approach zero from below or above. It is common to denote the derivative dx d
by dx y, y 0 (x), y 0 or Dy.
A function is said to be differentiable at a point if the derivative exists there. Note that differentiability implies
continuity, but not vice versa.

Example 3.3.1 Consider the derivative of y(x) = x2 at the point x = 1.

y(1 + x) y(1)
y 0 (1) lim
x0 x
(1 + x)2 1
= lim
x0 x
= lim (2 + x)
x0
=2

Figure 3.6 shows the secant lines approaching the tangent line as x approaches zero from above and below.

Example 3.3.2 We can compute the derivative of y(x) = x2 at an arbitrary point x.

d  2 (x + x)2 x2
x = lim
dx x0 x
= lim (2x + x)
x0
= 2x

57
4 4
3.5 3.5
3 3
2.5 2.5
2 2
1.5 1.5
1 1
0.5 0.5
0.5 1 1.5 2 0.5 1 1.5 2

Figure 3.6: Secant lines and the tangent to x2 at x = 1.

Properties. Let u(x) and v(x) be differentiable. Let a and b be constants. Some fundamental properties of
derivatives are:
d du dv
(au + bv) = a +b Linearity
dx dx dx
d du dv
(uv) = v+u Product Rule
dx dx dx
d  u  v du u dv
= dx 2 dx Quotient Rule
dx v v
d a du
(u ) = aua1 Power Rule
dx dx
d du dv
(u(v(x))) = = u0 (v(x))v 0 (x) Chain Rule
dx dv dx
These can be proved by using the definition of differentiation.

58
Example 3.3.3 Prove the quotient rule for derivatives.

u(x+x) u(x)
d u v(x+x)
v(x)
= lim
dx v x0 x
u(x + x)v(x) u(x)v(x + x)
= lim
x0 xv(x)v(x + x)
u(x + x)v(x) u(x)v(x) u(x)v(x + x) + u(x)v(x)
= lim
x0 xv(x)v(x)
(u(x + x) u(x))v(x) u(x)(v(x + x) v(x))
= lim
x0 xv 2 (x)
u(x+x)u(x) v(x+x)v(x)
limx0 x
v(x) u(x) limx0 x
=
v 2 (x)
v du
dx
dv
u dx
=
v2

59
Trigonometric Functions. Some derivatives of trigonometric functions are:
d d 1
sin x = cos x arcsin x =
dx dx (1 x2 )1/2
d d 1
cos x = sin x arccos x =
dx dx (1 x2 )1/2
d 1 d 1
tan x = 2
arctan x =
dx cos x dx 1 + x2
d x d 1
e = ex ln x =
dx dx x
d d 1
sinh x = cosh x arcsinh x = 2
dx dx (x + 1)1/2
d d 1
cosh x = sinh x arccosh x = 2
dx dx (x 1)1/2
d 1 d 1
tanh x = 2 arctanh x =
dx cosh x dx 1 x2
Example 3.3.4 We can evaluate the derivative of xx by using the identity ab = eb ln a .
d x d x ln x
x = e
dx dx
d
= ex ln x (x ln x)
dx
1
= xx (1 ln x + x )
x
x
= x (1 + ln x)

Inverse Functions. If we have a function y(x), we can consider x as a function of y, x(y). For example, if
x+2
y(x) = 8x3 then x(y) = 3 y/2; if y(x) = x+1 then x(y) = 2y
y1
. The derivative of an inverse function is
d 1
x(y) = dy .
dy dx

60
Example 3.3.5 The inverse function of y(x) = ex is x(y) = ln y. We can obtain the derivative of the logarithm from
the derivative of the exponential. The derivative of the exponential is
dy
= ex .
dx
Thus the derivative of the logarithm is
d d 1 1 1
ln y = x(y) = dy = x = .
dy dy dx
e y

3.4 Implicit Differentiation


An explicitly defined function has the form y = f (x). A implicitly defined function has the form f (x, y) = 0. A few
examples of implicit functions are x2 + y 2 1 = 0 and x + y + sin(xy) = 0. Often it is not possible to write an implicit
equation in explicit form. This is true of the latter example above. One can calculate the derivative of y(x) in terms
of x and y even when y(x) is defined by an implicit equation.

Example 3.4.1 Consider the implicit equation


x2 xy y 2 = 1.
This implicit equation can be solved for the dependent variable.
1 
y(x) = x 5x2 4 .
2
We can differentiate this expression to obtain
 
0 1 5x
y = 1 .
2 5x2 4
One can obtain the same result without first solving for y. If we differentiate the implicit equation, we obtain
dy dy
2x y x 2y = 0.
dx dx

61
dy
We can solve this equation for dx
.
dy 2x y
=
dx x + 2y
We can differentiate this expression to obtain the second derivative of y.

d2 y (x + 2y)(2 y 0 ) (2x y)(1 + 2y 0 )


=
dx2 (x + 2y)2
5(y xy 0 )
=
(x + 2y)2

Substitute in the expression for y 0 .

10(x2 xy y 2 )
=
(x + 2y)2

Use the original implicit equation.

10
=
(x + 2y)2

3.5 Maxima and Minima


A differentiable function is increasing where f 0 (x) > 0, decreasing where f 0 (x) < 0 and stationary where f 0 (x) = 0.
A function f (x) has a relative maxima at a point x = if there exists a neighborhood around such that f (x) f ()
for x (x , x + ), > 0. The relative minima is defined analogously. Note that this definition does not require
that the function be differentiable, or even continuous. We refer to relative maxima and minima collectively are relative
extrema.

62
Relative Extrema and Stationary Points. If f (x) is differentiable and f () is a relative extrema then x =
is a stationary point, f 0 () = 0. We can prove this using left and right limits. Assume that f () is a relative maxima.
Then there is a neighborhood (x , x + ), > 0 for which f (x) f (). Since f (x) is differentiable the derivative
at x = ,
f ( + x) f ()
f 0 () = lim ,
x0 x
exists. This in turn means that the left and right limits exist and are equal. Since f (x) f () for < x < the
left limit is non-positive,
f ( + x) f ()
f 0 () = lim 0.
x0 x
Since f (x) f () for < x < + the right limit is nonnegative,

f ( + x) f ()
f 0 () = lim + 0.
x0 x

Thus we have 0 f 0 () 0 which implies that f 0 () = 0.

It is not true that all stationary points are relative extrema. That is, f 0 () = 0 does not imply that x = is an
extrema. Consider the function f (x) = x3 . x = 0 is a stationary point since f 0 (x) = x2 , f 0 (0) = 0. However, x = 0 is
neither a relative maxima nor a relative minima.

It is also not true that all relative extrema are stationary points. Consider the function f (x) = |x|. The point x = 0
is a relative minima, but the derivative at that point is undefined.

First Derivative Test. Let f (x) be differentiable and f 0 () = 0.

If f 0 (x) changes sign from positive to negative as we pass through x = then the point is a relative maxima.

If f 0 (x) changes sign from negative to positive as we pass through x = then the point is a relative minima.

63
If f 0 (x) is not identically zero in a neighborhood of x = and it does not change sign as we pass through the
point then x = is not a relative extrema.

Example 3.5.1 Consider y = x2 and the point x = 0. The function is differentiable. The derivative, y 0 = 2x, vanishes
at x = 0. Since y 0 (x) is negative for x < 0 and positive for x > 0, the point x = 0 is a relative minima. See Figure 3.7.

Example 3.5.2 Consider y = cos x and the point x = 0. The function is differentiable. The derivative, y 0 = sin x
is positive for < x < 0 and negative for 0 < x < . Since the sign of y 0 goes from positive to negative, x = 0 is a
relative maxima. See Figure 3.7.

Example 3.5.3 Consider y = x3 and the point x = 0. The function is differentiable. The derivative, y 0 = 3x2 is
positive for x < 0 and positive for 0 < x. Since y 0 is not identically zero and the sign of y 0 does not change, x = 0 is
not a relative extrema. See Figure 3.7.

Figure 3.7: Graphs of x2 , cos x and x3 .

Concavity. If the portion of a curve in some neighborhood of a point lies above the tangent line through that point,
the curve is said to be concave upward. If it lies below the tangent it is concave downward. If a function is twice
differentiable then f 00 (x) > 0 where it is concave upward and f 00 (x) < 0 where it is concave downward. Note that
f 00 (x) > 0 is a sufficient, but not a necessary condition for a curve to be concave upward at a point. A curve may be
concave upward at a point where the second derivative vanishes. A point where the curve changes concavity is called
a point of inflection. At such a point the second derivative vanishes, f 00 (x) = 0. For twice continuously differentiable

64
functions, f 00 (x) = 0 is a necessary but not a sufficient condition for an inflection point. The second derivative may
vanish at places which are not inflection points. See Figure 3.8.

Figure 3.8: Concave Upward, Concave Downward and an Inflection Point.

Second Derivative Test. Let f (x) be twice differentiable and let x = be a stationary point, f 0 () = 0.

If f 00 () < 0 then the point is a relative maxima.

If f 00 () > 0 then the point is a relative minima.

If f 00 () = 0 then the test fails.

Example 3.5.4 Consider the function f (x) = cos x and the point x = 0. The derivatives of the function are
f 0 (x) = sin x, f 00 (x) = cos x. The point x = 0 is a stationary point, f 0 (0) = sin(0) = 0. Since the second
derivative is negative there, f 00 (0) = cos(0) = 1, the point is a relative maxima.

Example 3.5.5 Consider the function f (x) = x4 and the point x = 0. The derivatives of the function are f 0 (x) = 4x3 ,
f 00 (x) = 12x2 . The point x = 0 is a stationary point. Since the second derivative also vanishes at that point the
second derivative test fails. One must use the first derivative test to determine that x = 0 is a relative minima.

65
3.6 Mean Value Theorems
Rolles Theorem. If f (x) is continuous in [a, b], differentiable in (a, b) and f (a) = f (b) = 0 then there exists a
point (a, b) such that f 0 () = 0. See Figure 3.9.

Figure 3.9: Rolles Theorem.

To prove this we consider two cases. First we have the trivial case that f (x) 0. If f (x) is not identically zero
then continuity implies that it must have a nonzero relative maxima or minima in (a, b). Let x = be one of these
relative extrema. Since f (x) is differentiable, x = must be a stationary point, f 0 () = 0.

Theorem of the Mean. If f (x) is continuous in [a, b] and differentiable in (a, b) then there exists a point x =
such that
f (b) f (a)
f 0 () = .
ba
That is, there is a point where the instantaneous velocity is equal to the average velocity on the interval.
We prove this theorem by applying Rolles theorem. Consider the new function
f (b) f (a)
g(x) = f (x) f (a) (x a)
ba
Note that g(a) = g(b) = 0, so it satisfies the conditions of Rolles theorem. There is a point x = such that g 0 () = 0.
We differentiate the expression for g(x) and substitute in x = to obtain the result.
f (b) f (a)
g 0 (x) = f 0 (x)
ba

66
Figure 3.10: Theorem of the Mean.

f (b) f (a)
g 0 () = f 0 () =0
ba
f (b) f (a)
f 0 () =
ba

Generalized Theorem of the Mean. If f (x) and g(x) are continuous in [a, b] and differentiable in (a, b), then
there exists a point x = such that
f 0 () f (b) f (a)
0
= .
g () g(b) g(a)
We have assumed that g(a) 6= g(b) so that the denominator does not vanish and that f 0 (x) and g 0 (x) are not
simultaneously zero which would produce an indeterminate form. Note that this theorem reduces to the regular
theorem of the mean when g(x) = x. The proof of the theorem is similar to that for the theorem of the mean.

Taylors Theorem of the Mean. If f (x) is n + 1 times continuously differentiable in (a, b) then there exists a
point x = (a, b) such that

0 (b a)2 00 (b a)n (n) (b a)n+1 (n+1)


f (b) = f (a) + (b a)f (a) + f (a) + + f (a) + f (). (3.1)
2! n! (n + 1)!
For the case n = 0, the formula is
f (b) = f (a) + (b a)f 0 (),

67
which is just a rearrangement of the terms in the theorem of the mean,
f (b) f (a)
f 0 () = .
ba

3.6.1 Application: Using Taylors Theorem to Approximate Functions.


One can use Taylors theorem to approximate functions with polynomials. Consider an infinitely differentiable function
f (x) and a point x = a. Substituting x for b into Equation 3.1 we obtain,

(x a)2 00 (x a)n (n) (x a)n+1 (n+1)


f (x) = f (a) + (x a)f 0 (a) + f (a) + + f (a) + f ().
2! n! (n + 1)!

If the last term in the sum is small then we can approximate our function with an nth order polynomial.

(x a)2 00 (x a)n (n)


f (x) f (a) + (x a)f 0 (a) + f (a) + + f (a)
2! n!
The last term in Equation 3.6.1 is called the remainder or the error term,

(x a)n+1 (n+1)
Rn = f ().
(n + 1)!

Since the function is infinitely differentiable, f (n+1) () exists and is bounded. Therefore we note that the error must
vanish as x 0 because of the (x a)n+1 factor. We therefore suspect that our approximation would be a good one
if x is close to a. Also note that n! eventually grows faster than (x a)n ,

(x a)n
lim = 0.
n n!
So if the derivative term, f (n+1) (), does not grow to quickly, the error for a certain value of x will get smaller with
increasing n and the polynomial will become a better approximation of the function. (It is also possible that the
derivative factor grows very quickly and the approximation gets worse with increasing n.)

68
Example 3.6.1 Consider the function f (x) = ex . We want a polynomial approximation of this function near the point
x = 0. Since the derivative of ex is ex , the value of all the derivatives at x = 0 is f (n) (0) = e0 = 1. Taylors theorem
thus states that
x2 x3 xn xn+1
ex = 1 + x + + + + + e,
2! 3! n! (n + 1)!
for some (0, x). The first few polynomial approximations of the exponent about the point x = 0 are

f1 (x) = 1
f2 (x) = 1 + x
x2
f3 (x) = 1 + x +
2
x2 x3
f4 (x) = 1 + x + +
2 6

The four approximations are graphed in Figure 3.11.

2.5 2.5 2.5 2.5


2 2 2 2
1.5 1.5 1.5 1.5
1 1 1 1
0.5 0.5 0.5 0.5
-1 -0.5 0.5 1 -1 -0.5 0.5 1 -1 -0.5 0.5 1 -1 -0.5 0.5 1

Figure 3.11: Four Finite Taylor Series Approximations of ex

Note that for the range of x we are looking at, the approximations become more accurate as the number of terms
increases.

Example 3.6.2 Consider the function f (x) = cos x. We want a polynomial approximation of this function near the

69
point x = 0. The first few derivatives of f are
f (x) = cos x
f 0 (x) = sin x
f 00 (x) = cos x
f 000 (x) = sin x
f (4) (x) = cos x
Its easy to pick out the pattern here,
(
(n) (1)n/2 cos x for even n,
f (x) = (n+1)/2
(1) sin x for odd n.
Since cos(0) = 1 and sin(0) = 0 the n-term approximation of the cosine is,
x2 x4 x6 2(n1) x2(n1) x2n
cos x = 1 + + + (1) + cos .
2! 4! 6! (2(n 1))! (2n)!
Here are graphs of the one, two, three and four term approximations.

1 1 1 1
0.5 0.5 0.5 0.5
-3 -2 -1 1 2 3 -3 -2 -1 1 2 3 -3 -2 -1 1 2 3 -3 -2 -1 1 2 3
-0.5 -0.5 -0.5 -0.5
-1 -1 -1 -1

Figure 3.12: Taylor Series Approximations of cos x

Note that for the range of x we are looking at, the approximations become more accurate as the number of terms
increases. Consider the ten term approximation of the cosine about x = 0,
x2 x4 x18 x20
cos x = 1 + + cos .
2! 4! 18! 20!

70
Note that for any value of , | cos | 1. Therefore the absolute value of the error term satisfies,

20
|x|20

x
|R| = cos .
20! 20!

x20 /20! is plotted in Figure 3.13.

0.8

0.6

0.4

0.2

2 4 6 8 10

Figure 3.13: Plot of x20 /20!.

Note that the error is very small for x < 6, fairly small but non-negligible for x 7 and large for x > 8. The ten
term approximation of the cosine, plotted below, behaves just we would predict.
The error is very small until it becomes non-negligible at x 7 and large at x 8.

Example 3.6.3 Consider the function f (x) = ln x. We want a polynomial approximation of this function near the

71
1

0.5

-10 -5 5 10

-0.5

-1

-1.5

-2

Figure 3.14: Ten Term Taylor Series Approximation of cos x

point x = 1. The first few derivatives of f are

f (x) = ln x
1
f 0 (x) =
x
1
f 00 (x) = 2
x
2
f 000 (x) = 3
x
3
f (4) (x) = 4
x
The derivatives evaluated at x = 1 are

f (0) = 0, f (n) (0) = (1)n1 (n 1)!, for n 1.

By Taylors theorem of the mean we have,

(x 1)2 (x 1)3 (x 1)4 (x 1)n (x 1)n+1 1


ln x = (x 1) + + + (1)n1 + (1)n .
2 3 4 n n + 1 n+1

72
2 2 2 2
1 1 1 1
-1 0.5 1 1.5 2 2.5 3 -1 0.5 1 1.5 2 2.5 3 -1 0.5 1 1.5 2 2.5 3 -1 0.5 1 1.5 2 2.5 3
-2 -2 -2 -2
-3 -3 -3 -3
-4 -4 -4 -4
-5 -5 -5 -5
-6 -6 -6 -6

Figure 3.15: The 2, 4, 10 and 50 Term Approximations of ln x

Below are plots of the 2, 4, 10 and 50 term approximations.


Note that the approximation gets better on the interval (0, 2) and worse outside this interval as the number of terms
increases. The Taylor series converges to ln x only on this interval.

3.6.2 Application: Finite Difference Schemes


Example 3.6.4 Suppose you sample a function at the discrete points nx, n Z. In Figure 3.16 we sample the
function f (x) = sin x on the interval [4, 4] with x = 1/4 and plot the data points.
1

0.5

-4 -2 2 4

-0.5

-1

Figure 3.16: Sampling of sin x

We wish to approximate the derivative of the function on the grid points using only the value of the function on

73
those discrete points. From the definition of the derivative, one is lead to the formula
f (x + x) f (x)
f 0 (x) . (3.2)
x
Taylors theorem states that
x2 00
f (x + x) = f (x) + xf 0 (x) + f ().
2
Substituting this expression into our formula for approximating the derivative we obtain
2
f (x + x) f (x) f (x) + xf 0 (x) + x2 f 00 () f (x) x 00
= = f 0 (x) + f ().
x x 2
Thus we see that the error in our approximation of the first derivative is x 2
f 00 (). Since the error has a linear factor
of x, we call this a first order accurate method. Equation 3.2 is called the forward difference scheme for calculating
the first derivative. Figure 3.17 shows a plot of the value of this scheme for the function f (x) = sin x and x = 1/4.
The first derivative of the function f 0 (x) = cos x is shown for comparison.
1

0.5

-4 -2 2 4

-0.5

-1

Figure 3.17: The Forward Difference Scheme Approximation of the Derivative

Another scheme for approximating the first derivative is the centered difference scheme,
f (x + x) f (x x)
f 0 (x) .
2x

74
Expanding the numerator using Taylors theorem,
f (x + x) f (x x)
2x
x2 00 x3 000 x2 00 x3 000
f (x) + xf 0 (x) + 2
f (x) + 6
f () f (x) + xf 0 (x) 2
f (x) + 6
f ()
=
2x
x2 000
= f 0 (x) + (f () + f 000 ()).
12
The error in the approximation is quadratic in x. Therefore this is a second order accurate scheme. Below is a plot
of the derivative of the function and the value of this scheme for the function f (x) = sin x and x = 1/4.
1

0.5

-4 -2 2 4

-0.5

-1

Figure 3.18: Centered Difference Scheme Approximation of the Derivative

Notice how the centered difference scheme gives a better approximation of the derivative than the forward difference
scheme.

3.7 LHospitals Rule


Some singularities are easy to diagnose. Consider the function cosx x at the point x = 0. The function evaluates
to 10 and is thus discontinuous at that point. Since the numerator and denominator are continuous functions and the

75
denominator vanishes while the numerator does not, the left and right limits as x 0 do not exist. Thus the function
has an infinite discontinuity at the point x = 0. More generally, a function which is composed of continuous functions
and evaluates to a0 at a point where a 6= 0 must have an infinite discontinuity there.

Other singularities require more analysis to diagnose. Consider the functions sinx x , sin
|x|
x sin x
and 1cos x
at the point x = 0.
0
All three functions evaluate to 0 at that point, but have different kinds of singularities. The first has a removable
discontinuity, the second has a finite discontinuity and the third has an infinite discontinuity. See Figure 3.19.

sin x sin x sin x


Figure 3.19: The functions x
, |x| and 1cos x
.


An expression that evaluates to 00 , , 0 , , 1 , 00 or 0 is called an indeterminate. A function f (x) which
is indeterminate at the point x = is singular at that point. The singularity may be a removable discontinuity, a finite
discontinuity or an infinite discontinuity depending on the behavior of the function around that point. If limx f (x)
exists, then the function has a removable discontinuity. If the limit does not exist, but the left and right limits do exist,
then the function has a finite discontinuity. If either the left or right limit does not exist then the function has an infinite
discontinuity.

76
LHospitals Rule. Let f (x) and g(x) be differentiable and f () = g() = 0. Further, let g(x) be nonzero in a
deleted neighborhood of x = , (g(x) 6= 0 for x 0 < |x | < ). Then

f (x) f 0 (x)
lim = lim 0 .
x g(x) x g (x)

To prove this, we note that f () = g() = 0 and apply the generalized theorem of the mean. Note that

f (x) f (x) f () f 0 ()
= = 0
g(x) g(x) g() g ()

for some between and x. Thus


f (x) f 0 () f 0 (x)
lim = lim 0 = lim 0
x g(x) g () x g (x)

provided that the limits exist.


LHospitals Rule is also applicable when both functions tend to infinity instead of zero or when the limit point, ,
is at infinity. It is also valid for one-sided limits.
LHospitals rule is directly applicable to the indeterminate forms 00 and

.
sin x sin x sin x
Example 3.7.1 Consider the three functions x
, |x| and 1cos x
at the point x = 0.

sin x cos x
lim = lim =1
x0 x x0 1
sin x
Thus x
has a removable discontinuity at x = 0.

sin x sin x
lim+ = lim+ =1
x0 |x| x0 x

sin x sin x
lim = lim = 1
x0 |x| x0 x

77
sin x
Thus |x|
has a finite discontinuity at x = 0.

sin x cos x 1
lim = lim = =
x0 1 cos x x0 sin x 0

sin x
Thus 1cos x
has an infinite discontinuity at x = 0.

Example 3.7.2 Let a and d be nonzero.

ax2 + bx + c 2ax + b
lim 2
= lim
x dx + ex + f x 2dx + e
2a
= lim
x 2d
a
=
d

Example 3.7.3 Consider


cos x 1
lim .
x0 x sin x

This limit is an indeterminate of the form 00 . Applying LHospitals rule we see that limit is equal to

sin x
lim .
x0 x cos x + sin x

This limit is again an indeterminate of the form 00 . We apply LHospitals rule again.

cos x 1
lim =
x0 x sin x + 2 cos x 2

78
Thus the value of the original limit is 12 . We could also obtain this result by expanding the functions in Taylor series.
 
x2 x4
cos x 1 1 2
+ 24
1
lim = lim x3 x5

x0 x sin x x0 x x + 120
6
x2 4
2 + x24
= lim 4 6
x0 x2 x + x
6 120
2
12 + x24
= lim 2 4
x0 1 x + x
6 120
1
=
2

We can apply LHospitals Rule to the indeterminate forms 0 and by rewriting the expression in a
different form, (perhaps putting the expression over a common denominator). If at first you dont succeed, try, try
again. You may have to apply LHospitals rule several times to evaluate a limit.

Example 3.7.4
 
1 x cos x sin x
lim cot x = lim
x0 x x0 x sin x
cos x x sin x cos x
= lim
x0 sin x + x cos x
x sin x
= lim
x0 sin x + x cos x
x cos x sin x
= lim
x0 cos x + cos x x sin x

=0

You can apply LHospitals rule to the indeterminate forms 1 , 00 or 0 by taking the logarithm of the expression.

79
Example 3.7.5 Consider the limit,
lim xx ,
x0

which gives us the indeterminate form 00 . The logarithm of the expression is

ln(xx ) = x ln x.

As x 0 we now have the indeterminate form 0 . By rewriting the expression, we can apply LHospitals rule.

ln x 1/x
lim = lim
x0 1/x x0 1/x2

= lim (x)
x0
=0

Thus the original limit is


lim xx = e0 = 1.
x0

80
3.8 Exercises
3.8.1 Limits of Functions
Exercise 3.1
Does  
1
lim sin
x0 x
exist?
Hint, Solution
Exercise 3.2
Does  
1
lim x sin
x0 x
exist?
Hint, Solution
Exercise 3.3
Evaluate the limit:

n
lim 5.
n

Hint, Solution

3.8.2 Continuous Functions


Exercise 3.4
Is the function sin(1/x) continuous in the open interval (0, 1)? Is there a value of a such that the function defined by
(
sin(1/x) for x 6= 0,
f (x) =
a for x = 0

81
is continuous on the closed interval [0, 1]?
Hint, Solution
Exercise 3.5
Is the function sin(1/x) uniformly continuous in the open interval (0, 1)?
Hint, Solution
Exercise 3.6 1
Are the functions x and x
uniformly continuous on the interval (0, 1)?
Hint, Solution
Exercise 3.7
Prove that a function which is continuous on a closed interval is uniformly continuous on that interval.
Hint, Solution
Exercise 3.8
Prove or disprove each of the following.
1. If limn an = L then limn a2n = L2 .
2. If limn a2n = L2 then limn an = L.
3. If an > 0 for all n > 200, and limn an = L, then L > 0.
4. If f : R 7 R is continuous and limx f (x) = L, then for n Z, limn f (n) = L.
5. If f : R 7 R is continuous and limn f (n) = L, then for x R, limx f (x) = L.
Hint, Solution

3.8.3 The Derivative


Exercise 3.9 (mathematica/calculus/differential/definition.nb)
Use the definition of differentiation to prove the following identities where f (x) and g(x) are differentiable functions
and n is a positive integer.

82
d
1. dx
(xn ) = nxn1 , (I suggest that you use Newtons binomial formula.)
dg
2. d
dx
(f (x)g(x)) = f dx + g df
dx

d
3. dx
(sin x) = cos x. (Youll need to use some trig identities.)

4. d
dx
(f (g(x))) = f 0 (g(x))g 0 (x)

Hint, Solution
Exercise 3.10
Use the definition of differentiation to determine if the following functions differentiable at x = 0.

1. f (x) = x|x|
p
2. f (x) = 1 + |x|

Hint, Solution
Exercise 3.11 (mathematica/calculus/differential/rules.nb)
Find the first derivatives of the following:

a. x sin(cos x)

b. f (cos(g(x)))
1
c. f (ln x)

x
d. xx

e. |x| sin |x|

Hint, Solution

83
Exercise 3.12 (mathematica/calculus/differential/rules.nb)
Using
d d 1
sin x = cos x and tan x =
dx dx cos2 x
find the derivatives of arcsin x and arctan x.
Hint, Solution

3.8.4 Implicit Differentiation


Exercise 3.13 (mathematica/calculus/differential/implicit.nb)
Find y 0 (x), given that x2 + y 2 = 1. What is y 0 (1/2)?
Hint, Solution
Exercise 3.14 (mathematica/calculus/differential/implicit.nb)
Find y 0 (x) and y 00 (x), given that x2 xy + y 2 = 3.
Hint, Solution

3.8.5 Maxima and Minima


Exercise 3.15 (mathematica/calculus/differential/maxima.nb)
Identify any maxima and minima of the following functions.

a. f (x) = x(12 2x)2 .

b. f (x) = (x 2)2/3 .

Hint, Solution
Exercise 3.16 (mathematica/calculus/differential/maxima.nb)
A cylindrical container with a circular base and an open top is to hold 64 cm3 . Find its dimensions so that the surface
area of the cup is a minimum.
Hint, Solution

84
3.8.6 Mean Value Theorems
Exercise 3.17
Prove the generalized theorem of the mean. If f (x) and g(x) are continuous in [a, b] and differentiable in (a, b), then
there exists a point x = such that
f 0 () f (b) f (a)
0
= .
g () g(b) g(a)
Assume that g(a) 6= g(b) so that the denominator does not vanish and that f 0 (x) and g 0 (x) are not simultaneously
zero which would produce an indeterminate form.
Hint, Solution
Exercise 3.18 (mathematica/calculus/differential/taylor.nb)
1
Find a polynomial approximation of sin x on the interval [1, 1] that has a maximum error of 1000
. Dont use any more
terms that you need to. Prove the error bound. Use your polynomial to approximate sin 1.
Hint, Solution
Exercise 3.19 (mathematica/calculus/differential/taylor.nb)
You use the formula f (x+x)2f (x)+f (xx)
x2
to approximate f 00 (x). What is the error in this approximation?
Hint, Solution
Exercise 3.20
The formulas f (x+x)f
x
(x)
and f (x+x)f
2x
(xx)
are first and second order accurate schemes for approximating the first
0
derivative f (x). Find a couple other schemes that have successively higher orders of accuracy. Would these higher
order schemes actually give a better approximation of f 0 (x)? Remember that x is small, but not infinitesimal.
Hint, Solution

3.8.7 LHospitals Rule


Exercise 3.21 (mathematica/calculus/differential/lhospitals.nb)
Evaluate the following limits.
xsin x
a. limx0 x3

85
1

b. limx0 csc x x

1 x

c. limx+ 1 + x

d. limx0 csc2 x x12 . (First evaluate using LHospitals rule then using a Taylor series expansion. You will find


that the latter method is more convenient.)

Hint, Solution
Exercise 3.22 (mathematica/calculus/differential/lhospitals.nb)
Evaluate the following limits,
 a bx
lim xa/x , lim 1 + ,
x x x
where a and b are constants.
Hint, Solution

86
3.9 Hints
Hint 3.1
Apply the , definition of a limit.

Hint 3.2
Set y = 1/x. Consider limy .

Hint 3.3

Write n 5 in terms of the exponential function.

Hint 3.4
The composition of continuous functions is continuous. Apply the definition of continuity and look at the point x = 0.

Hint 3.5
1 1
Note that for x1 = (n1/2)
and x2 = (n+1/2)
where n Z we have | sin(1/x1 ) sin(1/x2 )| = 2.

Hint 3.6
Note that the function x + x is a decreasing function of x and an increasing function of for positive x and
. Bound this function for fixed .
Consider any positive and . For what values of x is

1 1
> .
x x+

Hint 3.7
Let the function f (x) be continuous on a closed interval. Consider the function

e(x, ) = sup |f () f (x)|.


|x|<

Bound e(x, ) with a function of alone.

87
Hint 3.8
CONTINUE

1. If limn an = L then limn a2n = L2 .

2. If limn a2n = L2 then limn an = L.

3. If an > 0 for all n > 200, and limn an = L, then L > 0.

4. If f : R 7 R is continuous and limx f (x) = L, then for n Z, limn f (n) = L.

5. If f : R 7 R is continuous and limn f (n) = L, then for x R, limx f (x) = L.

Hint 3.9
a. Newtons binomial formula is
n  
n
X n n(n 1) n2 2
(a + b) = ank bk = an + an1 b + a b + + nabn1 + bn .
k=0
k 2

Recall that the binomial coefficient is  


n n!
= .
k (n k)!k!

b. Note that  
d f (x + x)g(x + x) f (x)g(x)
(f (x)g(x)) = lim
dx x0 x
and    
0 0 f (x + x) f (x) g(x + x) g(x)
g(x)f (x) + f (x)g (x) = g(x) lim + f (x) lim .
x0 x x0 x
Fill in the blank.

88
c. First prove that
sin
lim = 1.
0

and  
cos 1
lim = 0.
0

d. Let u = g(x). Consider a nonzero increment x, which induces the increments u and f . By definition,

f = f (u + u) f (u), u = g(x + x) g(x),

and f, u 0 as x 0. If u 6= 0 then we have


f df
= 0 as u 0.
u du
If u = 0 for some values of x then f also vanishes and we define  = 0 for theses values. In either case,
df
y = u + u.
du
Continue from here.
Hint 3.10

Hint 3.11
a. Use the product rule and the chain rule.

b. Use the chain rule.

c. Use the quotient rule and the chain rule.

d. Use the identity ab = eb ln a .

89
e. For x > 0, the expression is x sin x; for x < 0, the expression is (x) sin(x) = x sin x. Do both cases.

Hint 3.12
Use that x0 (y) = 1/y 0 (x) and the identities cos x = (1 sin2 x)1/2 and cos(arctan x) = 1
(1+x2 )1/2
.

Hint 3.13
Differentiating the equation
x2 + [y(x)]2 = 1
yields
2x + 2y(x)y 0 (x) = 0.
Solve this equation for y 0 (x) and write y(x) in terms of x.

Hint 3.14
Differentiate the equation and solve for y 0 (x) in terms of x and y(x). Differentiate the expression for y 0 (x) to obtain
y 00 (x). Youll use that
x2 xy(x) + [y(x)]2 = 3

Hint 3.15
a. Use the second derivative test.

b. The function is not differentiable at the point x = 2 so you cant use a derivative test at that point.

Hint 3.16
Let r be the radius and h the height of the cylinder. The volume of the cup is r2 h = 64. The radius and height are
64 2 2 128
related by h = r 2 . The surface area of the cup is f (r) = r + 2rh = r + r . Use the second derivative test to

find the minimum of f (r).

Hint 3.17
The proof is analogous to the proof of the theorem of the mean.

90
Hint 3.18
The first few terms in the Taylor series of sin(x) about x = 0 are
x3 x5 x7 x9
sin(x) = x + + + .
6 120 5040 362880
When determining the error, use the fact that | cos x0 | 1 and |xn | 1 for x [1, 1].

Hint 3.19
The terms in the approximation have the Taylor series,
x2 00 x3 000 x4 0000
f (x + x) = f (x) + xf 0 (x) +
f (x) + f (x) + f (x1 ),
2 6 24
0 x2 00 x3 000 x4 0000
f (x x) = f (x) xf (x) + f (x) f (x) + f (x2 ),
2 6 24
where x x1 x + x and x x x2 x.
Hint 3.20

Hint 3.21
a. Apply LHospitals rule three times.
b. You can write the expression as
x sin x
.
x sin x
c. Find the limit of the logarithm of the expression.
d. It takes four successive applications of LHospitals rule to evaluate the limit.
For the Taylor series expansion method,
1 x2 sin2 x x2 (x x3 /6 + O(x5 ))2
csc2 x = =
x2 x2 sin2 x x2 (x + O(x3 ))2

91
Hint 3.22
To evaluate the limits use the identity ab = eb ln a and then apply LHospitals rule.

92
3.10 Solutions
Solution 3.1
Note that in any open neighborhood of zero, (, ), the function sin(1/x) takes on all values in the interval [1, 1].
Thus if we choose a positive  such that  < 1 then there is no value of for which | sin(1/x) | <  for all
x (, ). Thus the limit does not exist.

Solution 3.2
We make the change of variables y = 1/x and consider y . We use that sin(y) is bounded.
 
1 1
lim x sin = lim sin(y) = 0
x0 x y y

Solution
3.3
We write n 5 in terms of the exponential function and then evaluate the limit.


 
n ln 5
lim 5 = lim exp
n n n
 
ln 5
= exp lim
n n

= e0
=1

Solution 3.4
Since x1 is continuous in the interval (0, 1) and the function sin(x) is continuous everywhere, the composition sin(1/x)
is continuous in the interval (0, 1).
Since limx0 sin(1/x) does not exist, there is no way of defining sin(1/x) at x = 0 to produce a function that is
continuous in [0, 1].

Solution 3.5
1 1
Note that for x1 = (n1/2)
and x2 = (n+1/2)
where n Z we have | sin(1/x1 ) sin(1/x2 )| = 2. Thus for any

93
0 <  < 2 there is no value of > 0 such that | sin(1/x1 ) sin(1/x2 )| <  for all x1 , x2 (0, 1) and |x1 x2 | < .
Thus sin(1/x) is not uniformly continuous in the open interval (0, 1).

Solution 3.6
First consider the function x. Note that the function x + x is a decreasing function of x and an increasing

function of for positive x and . Thus for any fixed ,the maximum value of x + x is bounded
by .
2
Therefore on the interval (0, 1), a sufficient condition for | x | <  is |x | <  . The function x is uniformly
continuous on the interval (0, 1).
Consider any positive and . Note that
1 1
>
x x+
for r !
1 4
x< 2 + .
2 
Thus there is no value of such that
1 1
<
x
1
for all |x | < . The function x
is not uniformly continuous on the interval (0, 1).

Solution 3.7
Let the function f (x) be continuous on a closed interval. Consider the function
e(x, ) = sup |f () f (x)|.
|x|<

Since f (x) is continuous, e(x, ) is a continuous function of x on the same closed interval. Since continuous functions
on closed intervals are bounded, there is a continuous, increasing function () satisfying,
e(x, ) (),
for all x in the closed interval. Since () is continuous and increasing, it has an inverse (). Now note that
|f (x) f ()| <  for all x and in the closed interval satisfying |x | < (). Thus the function is uniformly
continuous in the closed interval.

94
Solution 3.8
1. The statement
lim an = L
n

is equivalent to
 > 0, N s.t. n > N |an L| < .
We want to show that
> 0, M s.t. m > M |a2n L2 | < .
Suppose that |an L| < . We obtain an upper bound on |a2n L2 |.

|a2n L2 | = |an L||an + L| < (|2L| + )

Now we choose a value of  such that |a2n L2 | <

(|2L| + ) =

 = L2 + |L|

Consider any fixed > 0. We see that since



for = L2 + |L|, N s.t. n > N |an L| < 

implies that
n > N |a2n L2 | < .
Therefore
> 0, M s.t. m > M |a2n L2 | < .
We conclude that limn a2n = L2 .

2. limn a2n = L2 does not imply that limn an = L. Consider an = 1. In this case limn a2n = 1 and
limn an = 1.

95
3. If an > 0 for all n > 200, and limn an = L, then L is not necessarily positive. Consider an = 1/n, which
satisfies the two constraints.
1
lim = 0
n n

4. The statement limx f (x) = L is equivalent to

 > 0, X s.t. x > X |f (x) L| < .

This implies that for n > dXe, |f (n) L| < .

 > 0, N s.t. n > N |f (n) L| < 


lim f (n) = L
n

5. If f : R 7 R is continuous and limn f (n) = L, then for x R, it is not necessarily true that limx f (x) = L.
Consider f (x) = sin(x).
lim sin(n) = lim 0 = 0
n n

limx sin(x) does not exist.

Solution 3.9
a.

(x + x)n xn
 
d n
(x ) = lim
dx x0 x
 n(n1) n2

xn + nxn1 x + 2
x x2 + + xn xn
= lim
x0 x
 
n1 n(n 1) n2 n1
= lim nx + x x + + x
x0 2
= nxn1

96
d n
(x ) = nxn1
dx

b.
 
d f (x + x)g(x + x) f (x)g(x)
(f (x)g(x)) = lim
dx x0 x
 
[f (x + x)g(x + x) f (x)g(x + x)] + [f (x)g(x + x) f (x)g(x)]
= lim
x0 x
   
f (x + x) f (x) g(x + x) g(x)
= lim [g(x + x)] lim + f (x) lim
x0 x0 x x0 x
= g(x)f 0 (x) + f (x)g 0 (x)

d
(f (x)g(x)) = f (x)g 0 (x) + f 0 (x)g(x)
dx

c. Consider a right triangle with hypotenuse of length 1 in the first quadrant of the plane. Label the vertices A, B,
C, in clockwise order, starting with the vertex at the origin. The angle of A is . The length of a circular arc of
radius cos that connects C to the hypotenuse is cos . The length of the side BC is sin . The length of a
circular arc of radius 1 that connects B to the x axis is . (See Figure 3.20.)
Considering the length of these three curves gives us the inequality:

cos sin .

Dividing by ,
sin
cos 1.

Taking the limit as 0 gives us
sin
lim = 1.
0

97
B

cos sin


A
C

Figure 3.20:

One more little tidbit well need to know is


   
cos 1 cos 1 cos + 1
lim = lim
0 0 cos + 1
2
 
cos 1
= lim
0 (cos + 1)

sin2
 
= lim
0 (cos + 1)
   
sin sin
= lim lim
0 0 (cos + 1)
 
0
= (1)
2
= 0.

98
Now were ready to find the derivative of sin x.
 
d sin(x + x) sin x
(sin x) = lim
dx x0 x
 
cos x sin x + sin x cos x sin x
= lim
x0 x
   
sin x cos x 1
= cos x lim + sin x lim
x0 x x0 x
= cos x

d
(sin x) = cos x
dx

d. Let u = g(x). Consider a nonzero increment x, which induces the increments u and f . By definition,

f = f (u + u) f (u), u = g(x + x) g(x),

and f, u 0 as x 0. If u 6= 0 then we have

f df
= 0 as u 0.
u du

If u = 0 for some values of x then f also vanishes and we define  = 0 for theses values. In either case,

df
y = u + u.
du

99
We divide this equation by x and take the limit as x 0.

df f
= lim
dx x0  x 
df u u
= lim +
x0 du x x
     
df f u
= lim + lim  lim
du x0 x x0 x0 x
 
df du du
= + (0)
du dx dx
df du
=
du dx

Thus we see that

d
(f (g(x))) = f 0 (g(x))g 0 (x).
dx

Solution 3.10
1.

|| 0
f 0 (0) = lim  0

= lim  0||
=0

The function is differentiable at x = 0.

100
2.
p
0 1 + || 1
f (0) = lim  0

1
(1 + ||)1/2 sign()
= lim  0 2
1
1
= lim  0 sign()
2
Since the limit does not exist, the function is not differentiable at x = 0.
Solution 3.11
a.
d d d
[x sin(cos x)] = [x] sin(cos x) + x [sin(cos x)]
dx dx dx
d
= sin(cos x) + x cos(cos x) [cos x]
dx
= sin(cos x) x cos(cos x) sin x

d
[x sin(cos x)] = sin(cos x) x cos(cos x) sin x
dx
b.
d d
[f (cos(g(x)))] = f 0 (cos(g(x))) [cos(g(x))]
dx dx
d
= f 0 (cos(g(x))) sin(g(x)) [g(x)]
dx
= f (cos(g(x))) sin(g(x))g 0 (x)
0

d
[f (cos(g(x)))] = f 0 (cos(g(x))) sin(g(x))g 0 (x)
dx

101
c.
  d
d 1 [f (ln x)]
= dx
dx f (ln x) [f (ln x)]2
f 0 (ln x) dx
d
[ln x]
=
[f (ln x)]2
f 0 (ln x)
=
x[f (ln x)]2

f 0 (ln x)
 
d 1
=
dx f (ln x) x[f (ln x)]2

d. First we write the expression in terms exponentials and logarithms,


x
xx = xexp(x ln x) = exp(exp(x ln x) ln x).

Then we differentiate using the chain rule and the product rule.
d d
exp(exp(x ln x) ln x) = exp(exp(x ln x) ln x) (exp(x ln x) ln x)
dx  dx 
xx d 1
=x exp(x ln x) (x ln x) ln x + exp(x ln x)
dx x
 
x 1
= xx xx (ln x + x ) ln x + x1 exp(x ln x)
x
xx
= x x (ln x + 1) ln x + x1 xx
x

x
= xx +x x1 + ln x + ln2 x


d xx x
x = xx +x x1 + ln x + ln2 x

dx

102
e. For x > 0, the expression is x sin x; for x < 0, the expression is (x) sin(x) = x sin x. Thus we see that

|x| sin |x| = x sin x.

The first derivative of this is

sin x + x cos x.

d
(|x| sin |x|) = sin x + x cos x
dx

Solution 3.12
Let y(x) = sin x. Then y 0 (x) = cos x.

d 1
arcsin y = 0
dy y (x)
1
=
cos x
1
=
(1 sin2 x)1/2
1
=
(1 y 2 )1/2

d 1
arcsin x =
dx (1 x2 )1/2

103
Let y(x) = tan x. Then y 0 (x) = 1/ cos2 x.

d 1
arctan y = 0
dy y (x)
= cos2 x
= cos2 (arctan y)
 
1
=
(1 + y 2 )1/2
1
=
1 + y2

d 1
arctan x =
dx 1 + x2

Solution 3.13
Differentiating the equation
x2 + [y(x)]2 = 1
yields
2x + 2y(x)y 0 (x) = 0.
We can solve this equation for y 0 (x).
x
y 0 (x) =
y(x)
To find y 0 (1/2) we need to find y(x) in terms of x.

y(x) = 1 x2

Thus y 0 (x) is
x
y 0 (x) = .
1 x2

104
y 0 (1/2) can have the two values:
 
0 1 1
y = .
2 3

Solution 3.14
Differentiating the equation
x2 xy(x) + [y(x)]2 = 3
yields
2x y(x) xy 0 (x) + 2y(x)y 0 (x) = 0.
Solving this equation for y 0 (x)
y(x) 2x
y 0 (x) = .
2y(x) x
Now we differentiate y 0 (x) to get y 00 (x).
(y 0 (x) 2)(2y(x) x) (y(x) 2x)(2y 0 (x) 1)
y 00 (x) = ,
(2y(x) x)2
xy 0 (x) y(x)
y 00 (x) = 3 ,
(2y(x) x)2
y(x)2x
00
x 2y(x)x y(x)
y (x) = 3 ,
(2y(x) x)2
x(y(x) 2x) y(x)(2y(x) x)
y 00 (x) = 3 ,
(2y(x) x)3
x2 xy(x) + [y(x)]2
y 00 (x) = 6 ,
(2y(x) x)3
18
y 00 (x) = ,
(2y(x) x)3

105
Solution 3.15
a.
f 0 (x) = (12 2x)2 + 2x(12 2x)(2)
= 4(x 6)2 + 8x(x 6)
= 12(x 2)(x 6)
There are critical points at x = 2 and x = 6.
f 00 (x) = 12(x 2) + 12(x 6) = 24(x 4)
Since f 00 (2) = 48 < 0, x = 2 is a local maximum. Since f 00 (6) = 48 > 0, x = 6 is a local minimum.
b.
2
f 0 (x) = (x 2)1/3
3
The first derivative exists and is nonzero for x 6= 2. At x = 2, the derivative does not exist and thus x = 2 is a
critical point. For x < 2, f 0 (x) < 0 and for x > 2, f 0 (x) > 0. x = 2 is a local minimum.

Solution 3.16
Let r be the radius and h the height of the cylinder. The volume of the cup is r2 h = 64. The radius and height are
64 2 2 128
related by h = r 2 . The surface area of the cup is f (r) = r + 2rh = r + r . The first derivative of the surface
0 128 0
area is f (r) = 2r r2 . Finding the zeros of f (r),
128
2r = 0,
r2
2r3 128 = 0,
4
r= .
3

The second derivative of the surface area is f 00 (r) = 2 + 256 r3
. Since f 00 ( 4
3 ) = 6, r =
4

3 is a local minimum of
f (r). Since this is the only critical point for r > 0, it must be a global minimum.
4 4
The cup has a radius of 3 cm and a height of 3 .

106
Solution 3.17
We define the function
f (b) f (a)
h(x) = f (x) f (a) (g(x) g(a)).
g(b) g(a)
Note that h(x) is differentiable and that h(a) = h(b) = 0. Thus h(x) satisfies the conditions of Rolles theorem and
there exists a point (a, b) such that
f (b) f (a) 0
h0 () = f 0 () g () = 0,
g(b) g(a)
f 0 () f (b) f (a)
0
= .
g () g(b) g(a)

Solution 3.18
The first few terms in the Taylor series of sin(x) about x = 0 are

x3 x5 x7 x9
sin(x) = x + + + .
6 120 5040 362880
The seventh derivative of sin x is cos x. Thus we have that
x3 x5 cos x0 7
sin(x) = x + x,
6 120 5040
where 0 x0 x. Since we are considering x [1, 1] and 1 cos(x0 ) 1, the approximation

x3 x5
sin x x +
6 120
1
has a maximum error of 5040
0.000198. Using this polynomial to approximate sin(1),

13 15
1 + 0.841667.
6 120

107
To see that this has the required accuracy,
sin(1) 0.841471.

Solution 3.19
Expanding the terms in the approximation in Taylor series,
x2 00 x3 000 x4 0000
f (x + x) = f (x) + xf 0 (x) +f (x) + f (x) + f (x1 ),
2 6 24
x2 00 x3 000 x4 0000
f (x x) = f (x) xf 0 (x) + f (x) f (x) + f (x2 ),
2 6 24
where x x1 x + x and x x x2 x. Substituting the expansions into the formula,
f (x + x) 2f (x) + f (x x) 00 x2 0000
= f (x) + [f (x1 ) + f 0000 (x2 )].
x2 24
Thus the error in the approximation is
x2 0000
[f (x1 ) + f 0000 (x2 )].
24

Solution 3.20

Solution 3.21
a.
   
x sin x 1 cos x
lim = lim
x0 x3 x0 3x2
 
sin x
= lim
x0 6x
h cos x i
= lim
x0 6
1
=
6

108
 
x sin x 1
lim 3
=
x0 x 6

b.

   
1 1 1
lim csc x = lim
x0 x x0 sin x x
 
x sin x
= lim
x0 x sin x
 
1 cos x
= lim
x0 x cos x + sin x
 
sin x
= lim
x0 x sin x + cos x + cos x
0
=
2
=0

 
1
lim csc x =0
x0 x

109
c.

  x    x 
1 1
ln lim 1+ = lim ln 1+
x+ x x+ x
  
1
= lim x ln 1 +
x+ x
" #
ln 1 + x1

= lim
x+ 1/x
" 1 #
1 + x1 x12
= lim
x+ 1/x2
" 1 #
1
= lim 1+
x+ x
=1

Thus we have

 x 
1
lim 1+ = e.
x+ x

110
d. It takes four successive applications of LHospitals rule to evaluate the limit.

x2 sin2 x
 
2 1
lim csc x 2 = lim
x0 x x0 x2 sin2 x

2x 2 cos x sin x
= lim 2
x0 2x cos x sin x + 2x sin2 x

2 2 cos2 x + 2 sin2 x
= lim 2
x0 2x cos2 x + 8x cos x sin x + 2 sin2 x 2x2 sin2 x
8 cos x sin x
= lim
x0 12x cos2 x + 12 cos x sin x 8x2 cos x sin x 12x sin2 x

8 cos2 x 8 sin2 x
= lim
x0 24 cos2 x 8x2 cos2 x 64x cos x sin x 24 sin2 x + 8x2 sin2 x
1
=
3

It is easier to use a Taylor series expansion.

x2 sin2 x
 
21
lim csc x 2 = lim
x0 x x0 x2 sin2 x

x2 (x x3 /6 + O(x5 ))2
= lim
x0 x2 (x + O(x3 ))2
x2 (x2 x4 /3 + O(x6 ))
= lim
x0 x4 + O(x6 )
 
1 2
= lim + O(x )
x0 3
1
=
3

111
Solution 3.22
To evaluate the first limit, we use the identity ab = eb ln a and then apply LHospitals rule.
a ln x
lim xa/x = lim e x
x x
 
a ln x
= exp lim
x x
 
a/x
= exp lim
x 1

= e0

lim xa/x = 1
x

We use the same method to evaluate the second limit.


 a bx   a 
lim 1 + = lim exp bx ln 1 +
x x x x 
  a
= exp lim bx ln 1 +
 x x 
ln(1 + a/x)
= exp lim b
x 1/x
2

a/x
1+a/x
= exp lim b
x 1/x2
 
a
= exp lim b
x 1 + a/x

 a bx
lim 1 + = eab
x x

112
3.11 Quiz
Problem 3.1
Define continuity.
Solution
Problem 3.2
Fill in the blank with necessary, sufficient or necessary and sufficient.
Continuity is a condition for differentiability.
Differentiability is a condition for continuity.
f (x+x)f (x)
Existence of limx0 x
is a condition for differentiability.
Solution
Problem 3.3
d
Evaluate dx f (g(x)h(x)).
Solution
Problem 3.4
d
Evaluate dx f (x)g(x) .
Solution
Problem 3.5
State the Theorem of the Mean. Interpret the theorem physically.
Solution
Problem 3.6
State Taylors Theorem of the Mean.
Solution
Problem 3.7
Evaluate limx0 (sin x)sin x .
Solution

113
3.12 Quiz Solutions
Solution 3.1
A function y(x) is said to be continuous at x = if limx y(x) = y().

Solution 3.2
Continuity is a necessary condition for differentiability.
Differentiability is a sufficient condition for continuity.
Existence of limx0 f (x+x)fx
(x)
is a necessary and sufficient condition for differentiability.

Solution 3.3

d d
f (g(x)h(x)) = f 0 (g(x)h(x)) (g(x)h(x)) = f 0 (g(x)h(x))(g 0 (x)h(x) + g(x)h0 (x))
dx dx
Solution 3.4

d d g(x) ln f (x)
f (x)g(x) = e
dx dx
d
= eg(x) ln f (x) (g(x) ln f (x))
 dx
f 0 (x)

g(x) 0
= f (x) g (x) ln f (x) + g(x)
f (x)

Solution 3.5
If f (x) is continuous in [a..b] and differentiable in (a..b) then there exists a point x = such that

f (b) f (a)
f 0 () = .
ba
That is, there is a point where the instantaneous velocity is equal to the average velocity on the interval.

114
Solution 3.6
If f (x) is n + 1 times continuously differentiable in (a..b) then there exists a point x = (a..b) such that

(b a)2 00 (b a)n (n) (b a)n+1 (n+1)


f (b) = f (a) + (b a)f 0 (a) + f (a) + + f (a) + f ().
2! n! (n + 1)!

Solution 3.7
Consider limx0 (sin x)sin x . This is an indeterminate of the form 00 . The limit of the logarithm of the expression
is limx0 sin x ln(sin x). This is an indeterminate of the form 0 . We can rearrange the expression to obtain an
indeterminate of the form
and then apply LHospitals rule.

ln(sin x) cos x/ sin x


lim = lim = lim ( sin x) = 0
x0 1/ sin x x0 cos x/ sin2 x x0

The original limit is


lim (sin x)sin x = e0 = 1.
x0

115
Chapter 4

Integral Calculus

4.1 The Indefinite Integral


The opposite of a derivative is the anti-derivative or the indefinite integral. The indefinite integral of a function f (x)
is denoted, Z
f (x) dx.

It is defined by the property that Z


d
f (x) dx = f (x).
dx
While a function f (x) has a unique derivative if it is differentiable, it has an infinite number of indefinite integrals, each
of which differ by an additive constant.

Zero Slope Implies a Constant Function. If the value of a functions derivative is identically zero, df dx
= 0,
then the function is a constant, f (x) = c. To prove this, we assume that there exists a non-constant differentiable
function whose derivative is zero and obtain a contradiction. Let f (x) be such a function. Since f (x) is non-constant,
there exist points a and b such that f (a) 6= f (b). By the Mean Value Theorem of differential calculus, there exists a

116
point (a, b) such that
f (b) f (a)
f 0 () = 6= 0,
ba
which contradicts that the derivative is everywhere zero.

Indefinite Integrals Differ by an Additive Constant. Suppose that F (x) and G(x) are indefinite integrals
of f (x). Then we have
d
(F (x) G(x)) = F 0 (x) G0 (x) = f (x) f (x) = 0.
dx
RThus we see that F (x) G(x) = c and the two indefinite integrals must differ by a constant. For example, we have
sin x dx = cos x + c. While every function that can be expressed in terms of elementary functions, (the exponent,
logarithm, trigonometric functions, etc.), has a derivative
R that can be written explicitly in terms of elementary functions,
the same is not true of integrals. For example, sin(sin x) dx cannot be written explicitly in terms of elementary
functions.

Properties. Since the derivative is linear, so is the indefinite integral. That is,
Z Z Z
(af (x) + bg(x)) dx = a f (x) dx + b g(x) dx.

d
For each derivative identity there is a corresponding integral identity. Consider the power law identity, dx
(f (x))a =
a(f (x))a1 f 0 (x). The corresponding integral identity is

(f (x))a+1
Z
(f (x))a f 0 (x) dx = + c, a 6= 1,
a+1
d f 0 (x)
where we require that a 6= 1 to avoid division by zero. From the derivative of a logarithm, dx
ln(f (x)) = f (x)
, we
obtain, Z 0
f (x)
dx = ln |f (x)| + c.
f (x)

117
Figure 4.1: Plot of ln |x| and 1/x.

d 1 1
Note the absolute value signs. This is because dx
ln |x| = x
for x 6= 0. In Figure 4.1 is a plot of ln |x| and x
to reinforce
this.

Example 4.1.1 Consider


Z
x
I= dx.
(x2 + 1)2
We evaluate the integral by choosing u = x2 + 1, du = 2x dx.
Z
1 2x
I= dx
2 (x + 1)2
2
Z
1 du
=
2 u2
1 1
=
2 u
1
= 2
.
2(x + 1)

Example 4.1.2 Consider


Z Z
sin x
I= tan x dx = dx.
cos x

118
By choosing f (x) = cos x, f 0 (x) = sin x, we see that the integral is

sin x
Z
I= dx = ln | cos x| + c.
cos x

Change of Variable. The differential of a function g(x) is dg = g 0 (x) dx. Thus one might suspect that for
= g(x),
Z Z
f () d = f (g(x))g 0 (x) dx, (4.1)

since d = dg = g 0 (x) dx. This turns out to be true. To prove it we will appeal to the the chain rule for differentiation.
Let be a function of x. The chain rule is
d
f () = f 0 () 0 (x),
dx

d df d
f () = .
dx d dx
We can also write this as
df dx df
= ,
d d dx
or in operator notation,
d dx d
= .
d d dx
Now were ready to start. The derivative of the left side of Equation 4.1 is
Z
d
f () d = f ().
d

119
Next we differentiate the right side,

Z Z
d 0 dx d
f (g(x))g (x) dx = f (g(x))g 0 (x) dx
d d dx
dx
= f (g(x))g 0 (x)
d
dx dg
= f (g(x))
dg dx
= f (g(x))
= f ()

to see that it is in fact an identity for = g(x).

Example 4.1.3 Consider


Z
x sin(x2 ) dx.

We choose = x2 , d = 2xdx to evaluate the integral.

Z Z
2 1
x sin(x ) dx = sin(x2 )2x dx
2
Z
1
= sin d
2
1
= ( cos ) + c
2
1
= cos(x2 ) + c
2

120
Integration by Parts. The product rule for differentiation gives us an identity called integration by parts. We start
with the product rule and then integrate both sides of the equation.
d
(u(x)v(x)) = u0 (x)v(x) + u(x)v 0 (x)
Z dx
(u0 (x)v(x) + u(x)v 0 (x)) dx = u(x)v(x) + c
Z Z
u (x)v(x) dx + u(x)v 0 (x)) dx = u(x)v(x)
0

Z Z
u(x)v (x)) dx = u(x)v(x) v(x)u0 (x) dx
0

The theorem is most often written in the form


Z Z
u dv = uv v du.

So what is the usefulness of this? Well, it may happen for some integrals and a good choice of u and v that the integral
on the right is easier to evaluate than the integral on the left.
R
Example 4.1.4 Consider x ex dx. If we choose u = x, dv = ex dx then integration by parts yields
Z Z
x x
x e dx = x e ex dx = (x 1) ex .

Now notice what happens when we choose u = ex , dv = x dx.


Z Z
x 1 2 x 1 2 x
e
x dx = x e x e dx
2 2
The integral gets harder instead of easier.

When applying integration by parts, one must choose u and dv wisely. As general rules of thumb:

121
Pick u so that u0 is simpler than u.
Pick dv so that v is not more complicated, (hopefully simpler), than dv.
Also note that you may have to apply integration by parts several times to evaluate some integrals.

4.2 The Definite Integral


4.2.1 Definition
The area bounded by the x axis, the vertical lines x = a and x = b and the function f (x) is denoted with a definite
integral, Z b
f (x) dx.
a
The area is signed, that is, if f (x) is negative, then the area is negative. We measure the area with a divide-and-conquer
strategy. First partition the interval (a, b) with a = x0 < x1 < < xn1 < xn = b. Note that the area under the
curve on the subinterval is approximately the area of a rectangle of base xi = xi+1 xi and height f (i ), where
i [xi , xi+1 ]. If we add up the areas of the rectangles, we get an approximation of the area under the curve. See
Figure 4.2
Z b n1
X
f (x) dx f (i )xi
a i=0
As the xi s get smaller, we expect the approximation of the area to get better. Let x = max0in1 xi . We
define the definite integral as the sum of the areas of the rectangles in the limit that x 0.
Z b n1
X
f (x) dx = lim f (i )xi
a x0
i=0

The integral is defined when the limit exists. This is known as the Riemann integral of f (x). f (x) is called the
integrand.

122
f(1 )

a x1 x2 x3 x n-2 x n-1 b
xi

Figure 4.2: Divide-and-Conquer Strategy for Approximating a Definite Integral.

4.2.2 Properties
Linearity and the Basics. Because summation is a linear operator, that is

n1
X n1
X n1
X
(cfi + dgi ) = c fi + d gi ,
i=0 i=0 i=0

definite integrals are linear,


Z b Z b Z b
(cf (x) + dg(x)) dx = c f (x) dx + d g(x) dx.
a a a

One can also divide the range of integration.


Z b Z c Z b
f (x) dx = f (x) dx + f (x) dx
a a c

123
We assume that each of the above integrals exist. If a b, and we integrate from b to a, then each of the xi will be
negative. From this observation, it is clear that
Z b Z a
f (x) dx = f (x) dx.
a b

If we integrate any function from a point a to that same point a, then all the xi are zero and
Z a
f (x) dx = 0.
a

Bounding the Integral. Recall that if fi gi , then


n1
X n1
X
fi gi .
i=0 i=0

Let m = minx[a,b] f (x) and M = maxx[a,b] f (x). Then


n1
X n1
X n1
X
(b a)m = mxi f (i )xi M xi = (b a)M
i=0 i=0 i=0

implies that Z b
(b a)m f (x) dx (b a)M.
a
Since n1 n1
X X
fi |fi |,



i=0 i=0

we have Z b Z b


f (x) dx |f (x)| dx.
a a

124
Mean Value Theorem of Integral Calculus. Let f (x) be continuous. We know from above that

Z b
(b a)m f (x) dx (b a)M.
a

Therefore there exists a constant c [m, M ] satisfying

Z b
f (x) dx = (b a)c.
a

Since f (x) is continuous, there is a point [a, b] such that f () = c. Thus we see that

Z b
f (x) dx = (b a)f (),
a

for some [a, b].

4.3 The Fundamental Theorem of Integral Calculus

Definite Integrals with Variable Limits of Integration. Consider a to be a constant and x variable, then
the function F (x) defined by
Z x
F (x) = f (t) dt (4.2)
a

125
is an anti-derivative of f (x), that is F 0 (x) = f (x). To show this we apply the definition of differentiation and the
integral mean value theorem.

F (x + x) F (x)
F 0 (x) = lim
x0 x
R x+x Rx
f (t) dt f (t) dt
= lim a a
x0 x
R x+x
f (t) dt
= lim x
x0 x
f ()x
= lim , [x, x + x]
x0 x
= f (x)

The Fundamental Theorem of Integral Calculus. Let F (x) be any anti-derivative of f (x). Noting that all
anti-derivatives of f (x) differ by a constant and replacing x by b in Equation 4.2, we see that there exists a constant c
such that Z b
f (x) dx = F (b) + c.
a
Now to find the constant. By plugging in b = a,
Z a
f (x) dx = F (a) + c = 0,
a

we see that c = F (a). This gives us a result known as the Fundamental Theorem of Integral Calculus.
Z b
f (x) dx = F (b) F (a).
a

We introduce the notation


[F (x)]ba F (b) F (a).

126
Example 4.3.1
Z
sin x dx = [ cos x]0 = cos() + cos(0) = 2
0

4.4 Techniques of Integration


4.4.1 Partial Fractions
A proper rational function
p(x) p(x)
=
q(x) (x a)n r(x)
Can be written in the form
 
p(x) a0 a1 an1
= + + + + ( )
(x )n r(x) (x )n (x )n1 x

where the ak s are constants and the last ellipses represents the partial fractions expansion of the roots of r(x). The
coefficients are
1 dk p(x)
 
ak = .
k! dxk r(x) x=

Example 4.4.1 Consider the partial fraction expansion of

1 + x + x2
.
(x 1)3

The expansion has the form


a0 a1 a2
3
+ 2
+ .
(x 1) (x 1) x1

127
The coefficients are
1
a0 = (1 + x + x2 )|x=1 = 3,
0!
1 d
a1 = (1 + x + x2 )|x=1 = (1 + 2x)|x=1 = 3,
1! dx
1 d2 1
a2 = 2
(1 + x + x2 )|x=1 = (2)|x=1 = 1.
2! dx 2
Thus we have
1 + x + x2 3 3 1
3
= 3
+ 2
+ .
(x 1) (x 1) (x 1) x1

Example 4.4.2 Suppose we want to evaluate

1 + x + x2
Z
dx.
(x 1)3

If we expand the integrand in a partial fraction expansion, then the integral becomes easy.

1 + x + x2
Z Z  
3 3 1
dx. = + + dx
(x 1)3 (x 1)3 (x 1)2 x 1
3 3
= 2
+ ln(x 1)
2(x 1) (x 1)

Example 4.4.3 Consider the partial fraction expansion of

1 + x + x2
.
x2 (x 1)2

The expansion has the form


a0 a1 b0 b1
2
+ + 2
+ .
x x (x 1) x1

128
The coefficients are
1 1 + x + x2
 
a0 = = 1,
0! (x 1)2
x=0
1 d 1 + x + x2 2(1 + x + x2 )
   
1 + 2x
a1 = = = 3,
1! dx (x 1)2
x=0 (x 1)2 (x 1)3
x=0
1 1 + x + x2
 
b0 = = 3,
0! x2
x=1
1 d 1 + x + x2 1 + 2x 2(1 + x + x2 )
   
b1 = = = 3,
1! dx x2
x=1 x2 x3
x=1

Thus we have
1 + x + x2 1 3 3 3
2 2
= 2+ + 2
.
x (x 1) x x (x 1) x1

If the rational function has real coefficients and the denominator has complex roots, then you can reduce the work
in finding the partial fraction expansion with the following trick: Let and be complex conjugate pairs of roots of
the denominator.
 
p(x) a0 a1 an1
= + + +
(x )n (x )n r(x) (x )n (x )n1 x
 
a0 a1 an1
+ + + + + ( )
(x )n (x )n1 x

Thus we dont have to calculate the coefficients for the root at . We just take the complex conjugate of the coefficients
for .

Example 4.4.4 Consider the partial fraction expansion of


1+x
.
x2 + 1

129
The expansion has the form
a0 a0
+
xi x+i
The coefficients are
 
1 1 + x 1
a0 = = (1 i),
0! x + i x=i 2

1 1
a0 = (1 i) = (1 + i)
2 2

Thus we have
1+x 1i 1+i
2
= + .
x +1 2(x i) 2(x + i)

4.5 Improper Integrals


Rb
If the range of integration is infinite or f (x) is discontinuous at some points then a
f (x) dx is called an improper
integral.

Discontinuous Functions. If f (x) is continuous on the interval a x b except at the point x = c where
a < c < b then
Z b Z c Z b
f (x) dx = lim+ f (x) dx + lim+ f (x) dx
a 0 a 0 c+

provided that both limits exist.

Example 4.5.1 Consider the integral of ln x on the interval [0, 1]. Since the logarithm has a singularity at x = 0, this

130
is an improper integral. We write the integral in terms of a limit and evaluate the limit with LHospitals rule.
Z 1 Z 1
ln x dx = lim ln x dx
0 0
= lim[x ln x x]1
0
= 1 ln(1) 1 lim( ln )
0
= 1 lim( ln )
0
 
ln
= 1 lim
0 1/
 
1/
= 1 lim
0 1/ 2
= 1
Example 4.5.2 Consider the integral of xa on the range [0, 1]. If a < 0 then there is a singularity at x = 0. First
assume that a 6= 1.
Z 1  a+1 1
a x
x dx = lim+
0 0 a+1
1 a+1
= lim
a + 1 0+ a + 1
This limit exists only for a > 1. Now consider the case that a = 1.
Z 1
x1 dx = lim+ [ln x]1
0 0

= ln(0) lim+ ln
0

This limit does not exist. We obtain the result,


Z 1
1
xa dx = , for a > 1.
0 a+1

131
Infinite Limits of Integration. If the range of integration is infinite, say [a, ) then we define the integral as
Z Z
f (x) dx = lim f (x) dx,
a a

provided that the limit exists. If the range of integration is (, ) then


Z Z a Z
f (x) dx = lim f (x) dx + lim f (x) dx.
+ a

Example 4.5.3
 
d 1
Z Z
ln x
dx = ln x dx
1 x2 1 dx x
  Z
1 1 1
= ln x dx
x 1 1 x x
   
ln x 1
= lim
x+ x x 1
 
1/x 1
= lim lim + 1
x+ 1 x x

=1

Example 4.5.4 Consider the integral of xa on [1, ). First assume that a 6= 1.


xa+1
Z 
a
x dx = lim
1 + a + 1
1
a+1 1
= lim
+ a + 1 a+1

132
The limit exists for < 1. Now consider the case a = 1.
Z
x1 dx = lim [ln x]1
1 +
1
= lim ln
+ a+1
This limit does not exist. Thus we have
Z
1
xa dx = , for a < 1.
1 a+1

133
4.6 Exercises
4.6.1 The Indefinite Integral
Exercise R4.1 (mathematica/calculus/integral/fundamental.nb)
Evaluate (2x + 3)10 dx.
Hint, Solution

Exercise R4.2 (mathematica/calculus/integral/fundamental.nb)


2
Evaluate (lnxx) dx.
Hint, Solution

Exercise R4.3(mathematica/calculus/integral/fundamental.nb)
Evaluate x x2 + 3 dx.
Hint, Solution

Exercise R4.4 (mathematica/calculus/integral/fundamental.nb)


Evaluate cos x
sin x
dx.
Hint, Solution

Exercise R4.5 (mathematica/calculus/integral/fundamental.nb)


2
Evaluate x3x5 dx.
Hint, Solution

4.6.2 The Definite Integral


Exercise 4.6 (mathematica/calculus/integral/definite.nb)
Use the result
Z b N
X 1
f (x) dx = lim f (xn )x
a N
n=0

134
ba
where x = N
and xn = a + nx, to show that
Z 1
1
x dx = .
0 2
Hint, Solution
Exercise 4.7 (mathematica/calculus/integral/definite.nb) R
Evaluate the following integral using integration by parts and the Pythagorean identity. 0 sin2 x dx
Hint, Solution
Exercise 4.8 (mathematica/calculus/integral/definite.nb)
Prove that
Z f (x)
d
h() d = h(f (x))f 0 (x) h(g(x))g 0 (x).
dx g(x)
(Dont use the limit definition of differentiation, use the Fundamental Theorem of Integral Calculus.)
Hint, Solution
Exercise 4.9 (mathematica/calculus/integral/definite.nb)
Let An be the area between the curves x and xn on the interval [0 . . . 1]. What is limn An ? Explain this result
geometrically.
Hint, Solution
Exercise 4.10 (mathematica/calculus/integral/taylor.nb)
a. Show that Z x
f (x) = f (0) + f 0 (x ) d.
0

b. From the above identity show that


Z x
0
f (x) = f (0) + xf (0) + f 00 (x ) d.
0

135
c. Using induction, show that
Z x
1 1 1 n (n+1)
f (x) = f (0) + xf (0) + x2 f 00 (0) + + xn f (n) (0) +
0
f (x ) d.
2 n! 0 n!

Hint, Solution
Exercise 4.11
Find a function f (x) whose arc length from 0 to x is 2x.
Hint, Solution
Exercise 4.12
Consider a curve C, bounded by 1 and 1, on the interval (1 . . . 1). Can the length of C be unbounded? What if we
change to the closed interval [1 . . . 1]?
Hint, Solution

4.6.3 The Fundamental Theorem of Integration


4.6.4 Techniques of Integration
Exercise R4.13 (mathematica/calculus/integral/parts.nb)
Evaluate x sin x dx.
Hint, Solution

Exercise R4.14 (mathematica/calculus/integral/parts.nb)


Evaluate x3 e2x dx.
Hint, Solution

Exercise R4.15 (mathematica/calculus/integral/partial.nb)


Evaluate x214 dx.
Hint, Solution

136
Exercise R4.16 (mathematica/calculus/integral/partial.nb)
x+1
Evaluate x3 +x 2 6x dx.

Hint, Solution

4.6.5 Improper Integrals


Exercise R4.17 (mathematica/calculus/integral/improper.nb)
4 1
Evaluate 0 (x1) 2 dx.

Hint, Solution
Exercise R4.18 (mathematica/calculus/integral/improper.nb)
1
Evaluate 0 1x dx.
Hint, Solution
Exercise R4.19 (mathematica/calculus/integral/improper.nb)

Evaluate 0 x21+4 dx.
Hint, Solution

137
4.7 Hints
Hint 4.1
Make the change of variables u = 2x + 3.

Hint 4.2
Make the change of variables u = ln x.

Hint 4.3
Make the change of variables u = x2 + 3.

Hint 4.4
Make the change of variables u = sin x.

Hint 4.5
Make the change of variables u = x3 5.

Hint 4.6

Z 1 N
X 1
x dx = lim xn x
0 N
n=0
N
X 1
= lim (nx)x
N
n=0

Hint 4.7 R
Let u = sin x and dv = sin x dx. Integration by parts will give you an equation for 0 sin2 x dx.

Hint 4.8
Let H 0 (x) = h(x) and evaluate the integral in terms of H(x).

138
Hint 4.9
CONTINUE

Hint 4.10
a. Evaluate the integral.
b. Use integration by parts to evaluate the integral.
1 n
c. Use integration by parts with u = f (n+1) (x ) and dv = n!
.

Hint 4.11
The arc length from 0 to x is Z x p
1 + (f 0 ())2 d (4.3)
0
First show that the arc length of f (x) from a to b is 2(b a). Then conclude that the integrand in Equation 4.3 must
everywhere be 2.
Hint 4.12
CONTINUE

Hint 4.13
Let u = x, and dv = sin x dx.

Hint 4.14
Perform integration by parts three successive times. For the first one let u = x3 and dv = e2x dx.

Hint 4.15
Expanding the integrand in partial fractions,
1 1 a b
= = +
x2 4 (x 2)(x + 2) (x 2) (x + 2)
1 = a(x + 2) + b(x 2)

139
Set x = 2 and x = 2 to solve for a and b.
Hint 4.16
Expanding the integral in partial fractions,

x+1 x+1 a b c
= = + +
x3 2
+ x 6x x(x 2)(x + 3) x x2 x+3

x + 1 = a(x 2)(x + 3) + bx(x + 3) + cx(x 2)


Set x = 0, x = 2 and x = 3 to solve for a, b and c.
Hint 4.17
Z 4 Z 1 Z 4
1 1 1
dx = lim+ dx + lim+ dx
0 (x 1)2 0 0 (x 1)2 0 1+ (x 1)2

Hint 4.18
Z 1 Z 1
1 1
dx = lim dx
0 x 0+  x

Hint 4.19
Z
1 1 x
dx = arctan
x 2 + a2 a a

140
4.8 Solutions
Solution 4.1
Z
(2x + 3)10 dx

Let u = 2x + 3, g(u) = x = u3
2
, g 0 (u) = 21 .
Z Z
10 1
(2x + 3) dx = u10 du
2
11
u 1
=
11 2
(2x + 3)11
=
22
Solution 4.2

(ln x)2
Z Z
d(ln x)
dx = (ln x)2 dx
x dx
(ln x)3
=
3
Solution 4.3

Z Z
1 d(x2 )
x x2 + 3 dx = x2 + 3 dx
2 dx
1 (x2 + 3)3/2
=
2 3/2
(x + 3)3/2
2
=
3

141
Solution 4.4

Z Z
cos x 1 d(sin x)
dx = dx
sin x sin x dx
= ln | sin x|
Solution 4.5

x2 1 1 d(x3 )
Z Z
dx = dx
x3 5 x3 5 3 dx
1
= ln |x3 5|
3
Solution 4.6

Z 1 N
X 1
x dx = lim xn x
0 N
n=0
N
X 1
= lim (nx)x
N
n=0
N
X 1
= lim x2 n
N
n=0
N (N 1)
= lim x2
N 2
N (N 1)
= lim
N 2N 2
1
=
2

142
Solution 4.7
Let u = sin x and dv = sin x dx. Then du = cos x dx and v = cos x.
Z Z
2

cos2 x dx

sin x dx = sin x cos x 0 +
0 0
Z
= cos2 x dx
Z0
= (1 sin2 x) dx
0
Z
= sin2 x dx
0
Z
2 sin2 x dx =
0
Z

sin2 x dx =
0 2
Solution 4.8
Let H 0 (x) = h(x).
Z f (x)
d d
h() d = (H(f (x)) H(g(x)))
dx g(x) dx
= H 0 (f (x))f 0 (x) H 0 (g(x))g 0 (x)
= h(f (x))f 0 (x) h(g(x))g 0 (x)

Solution 4.9
First we compute the area for positive integer n.
1 1
x2 xn+1
Z 
n 1 1
An = (x x ) dx = =
0 2 n+1 0 2 n+1

143
Then we consider the area in the limit as n .
 
1 1 1
lim An = lim =
n n 2 n+1 2

In Figure 4.3 we plot the functions x1 , x2 , x4 , x8 , . . . , x1024 . In the limit as n , xn on the interval [0 . . . 1] tends to
the function
(
0 0x<1
1 x=1

Thus the area tends to the area of the right triangle with unit base and height.

0.8

0.6

0.4

0.2

0.2 0.4 0.6 0.8 1

Figure 4.3: Plots of x1 , x2 , x4 , x8 , . . . , x1024 .

144
Solution 4.10
1.
Z x
f (0) + f 0 (x ) d = f (0) + [f (x )]x0
0
= f (0) f (0) + f (x)
= f (x)

2.
Z x Z x
0 00 0 0 x
f (0) + xf (0) + f (x ) d = f (0) + xf (0) + [f (x )]0 f 0 (x ) d
0 0
0 0
= f (0) + xf (0) xf (0) [f (x )]x0
= f (0) f (0) + f (x)
= f (x)

3. Above we showed that the hypothesis holds for n = 0 and n = 1. Assume that it holds for some n = m 0.
Z x
0 1 2 00 1 n (n) 1 n (n+1)
f (x) = f (0) + xf (0) + x f (0) + + x f (0) + f (x ) d
2 n! 0 n!
 x
0 1 2 00 1 n (n) 1 n+1 (n+1)
= f (0) + xf (0) + x f (0) + + x f (0) + f (x )
2 n! (n + 1)! 0
Z x
1 n+1 (n+2)
f (x ) d
0 (n + 1)!
1 1 1
= f (0) + xf 0 (0) + x2 f 00 (0) + + xn f (n) (0) + xn+1 f (n+1) (0)
2 n! (n + 1)!
Z x
1
+ n+1 f (n+2) (x ) d
0 (n + 1)!
This shows that the hypothesis holds for n = m + 1. By induction, the hypothesis hold for all n 0.

145
Solution 4.11
First note that the arc length from a to b is 2(b a).
Z bp Z bp Z a p
0 2
1 + (f (x)) dx = 0 2
1 + (f (x)) dx 1 + (f 0 (x))2 dx = 2b 2a
a 0 0

Since a and b are arbitrary, we conclude that the integrand must everywhere be 2.
p
1 + (f 0 (x))2 = 2

f 0 (x) = 3

f (x) is a continuous, piecewise differentiable function which satisfies f 0 (x) = 3 at the points where it is differentiable.
One example is

f (x) = 3x

Solution 4.12
CONTINUE

Solution 4.13
Let u = x, and dv = sin x dx. Then du = dx and v = cos x.
Z Z
x sin x dx = x cos x + cos x dx

= x cos x + sin x + C

Solution 4.14
Let u = x3 and dv = e2x dx. Then du = 3x2 dx and v = 12 e2x .
Z Z
3 2x 1 3
x e dx = x3 e2x x2 e2x dx
2 2

146
Let u = x2 and dv = e2x dx. Then du = 2x dx and v = 12 e2x .
Z  Z 
3 2x 1 3 2x 3 1 2 2x 2x
x e dx = x e x e x e dx
2 2 2
Z Z
3 2x 1 3 2x 3 2 2x 3
x e dx = x e x e + x e2x dx
2 4 2
Let u = x and dv = e2x dx. Then du = dx and v = 12 e2x .
Z  Z 
3 2x 1 3 3 1 2x 1
x e dx = x3 e2x x2 e2x + xe e 2x
dx
2 4 2 2 2
Z
1 3 3 3
x3 e2x dx = x3 e2x x2 e2x + x e2x e2x +C
2 4 4 8
Solution 4.15
Expanding the integrand in partial fractions,

1 1 A B
= = +
x2 4 (x 2)(x + 2) (x 2) (x + 2)

1 = A(x + 2) + B(x 2)
Setting x = 2 yields A = 41 . Setting x = 2 yields B = 14 . Now we can do the integral.
Z Z  
1 1 1
dx = dx
x2 4 4(x 2) 4(x + 2)
1 1
= ln |x 2| ln |x + 2| + C
4 4
1 x 2
= +C
4 x + 2

147
Solution 4.16
Expanding the integral in partial fractions,

x+1 x+1 A B C
= = + +
x3 2
+ x 6x x(x 2)(x + 3) x x2 x+3

x + 1 = A(x 2)(x + 3) + Bx(x + 3) + Cx(x 2)


Setting x = 0 yields A = 61 . Setting x = 2 yields B = 3
10
. 2
Setting x = 3 yields C = 15 .
Z Z  
x+1 1 3 2
dx = + dx
x + x2 6x
3 6x 10(x 2) 15(x + 3)
1 3 2
= ln |x| + ln |x 2| ln |x + 3| + C
6 10 15
|x 2|3/10
= ln 1/6 +C
|x| |x + 3|2/15

Solution 4.17

Z 4 Z 1 Z 4
1 1 1
dx = lim+ dx + lim+ dx
0 (x 1)2 0 0 (x 1) 2 0 1+ (x 1)
2
 1  4
1 1
= lim+ + lim+
0 x1 0 0 x 1 1+
   
1 1 1
= lim+ 1 + lim+ +
0 0 3 
=+

The integral diverges.

148
Solution 4.18

Z 1 Z 1
1 1
dx = lim dx
0 x 0+  x
 1
= lim+ 2 x 
0

= lim+ 2(1 )
0
=2

Solution 4.19

Z Z
1 1
2
dx = lim dx
0 x +4 +4 x2
0  x 
1
= lim arctan
2 2 0
1
 
= 0
2 2

=
4

149
4.9 Quiz
Problem 4.1 Rb
Write the limit-sum definition of a f (x) dx.
Solution
ProblemR4.2p
2
Evaluate 1 |x| dx.
Solution
Problem 4.3 R x2
d
Evaluate dx x
f () d.
Solution
ProblemR4.4
2
Evaluate 1+x+x
(x+1)3
dx.
Solution
Problem 4.5
State the integral mean value theorem.
Solution
Problem 4.6
1
What is the partial fraction expansion of x(x1)(x2)(x3)
?
Solution

150
4.10 Quiz Solutions
Solution 4.1
Let a = x0 < x1 < < xn1 < xn = b be a partition of the interval (a..b). We define xi = xi+1 xi and
x = maxi xi and choose i [xi ..xi+1 ].
Z b n1
X
f (x) dx = lim f (i )xi
a x0
i=0

Solution 4.2

Z 2 p Z 0 2Z
|x| dx = x dx + x dx
1 1 0
Z 1 Z 2

= x dx + x dx
0 0
 1  2
2 3/2 2 3/2
= x + x
3 0 3 0
2 2 3/2
= + 2
3 3
2
= (1 + 2 2)
3
Solution 4.3

Z x2
d d 2 d
f () d = f (x2 ) (x ) f (x) (x)
dx x dx dx
2
= 2xf (x ) f (x)

151
Solution 4.4
First we expand the integrand in partial fractions.
1 + x + x2 a b c
= + +
(x + 1)3 (x + 1)3 (x + 1)2 x + 1

a = (1 + x + x2 ) x=1 = 1
 
1 d 2

b= (1 + x + x ) = (1 + 2x) x=1 = 1
1! dx
 2  x=1

1 d 2
1
c= 2
(1 + x + x ) = (2) x=1 = 1
2! dx x=1 2
Then we can do the integration.
1 + x + x2
Z Z  
1 1 1
dx = + dx
(x + 1)3 (x + 1)3 (x + 1)2 x + 1
1 1
= + + ln |x + 1|
2(x + 1)2 x + 1
x + 1/2
= + ln |x + 1|
(x + 1)2
Solution 4.5
Let f (x) be continuous. Then
Z b
f (x) dx = (b a)f (),
a
for some [a..b].

Solution 4.6

1 a b c d
= + + +
x(x 1)(x 2)(x 3) x x1 x2 x3

152
1 1
a= =
(0 1)(0 2)(0 3) 6
1 1
b= =
(1)(1 2)(1 3) 2
1 1
c= =
(2)(2 1)(2 3) 2
1 1
d= =
(3)(3 1)(3 2) 6
1 1 1 1 1
= + +
x(x 1)(x 2)(x 3) 6x 2(x 1) 2(x 2) 6(x 3)

153
Chapter 5

Vector Calculus

5.1 Vector Functions


Vector-valued Functions. A vector-valued function, r(t), is a mapping r : R 7 Rn that assigns a vector to each
value of t.
r(t) = r1 (t)e1 + + rn (t)en .
An example of a vector-valued function is the position of an object in space as a function of time. The function is
continous at a point t = if
lim r(t) = r( ).
t
This occurs if and only if the component functions are continuous. The function is differentiable if
dr r(t + t) r(t)
lim
dt t0 t
exists. This occurs if and only if the component functions are differentiable.
If r(t) represents the position of a particle at time t, then the velocity and acceleration of the particle are
dr d2 r
and ,
dt dt2

154
respectively. The speed of the particle is |r0 (t)|.

Differentiation Formulas. Let f (t) and g(t) be vector functions and a(t) be a scalar function. By writing out
components you can verify the differentiation formulas:

d
(f g) = f 0 g + f g0
dt
d
(f g) = f 0 g + f g0
dt
d
(af ) = a0 f + af 0
dt

5.2 Gradient, Divergence and Curl


Scalar and Vector Fields. A scalar field is a function of position u(x) that assigns a scalar to each point in space.
A function that gives the temperature of a material is an example of a scalar field. In two dimensions, you can graph a
scalar field as a surface plot, (Figure 5.1), with the vertical axis for the value of the function.
A vector field is a function of position u(x) that assigns a vector to each point in space. Examples of vectors fields
are functions that give the acceleration due to gravity or the velocity of a fluid. You can graph a vector field in two or
three dimension by drawing vectors at regularly spaced points. (See Figure 5.1 for a vector field in two dimensions.)

Partial Derivatives of Scalar Fields. Consider a scalar field u(x). The partial derivative of u with respect to
xk is the derivative of u in which xk is considered to be a variable and the remaining arguments are considered to be
parameters. The partial derivative is denoted x k u(x), x
u
k
or uxk and is defined

u u(x1 , . . . , xk + x, . . . , xn ) u(x1 , . . . , xk , . . . , xn )
lim .
xk x0 x

Partial derivatives have the same differentiation formulas as ordinary derivatives.

155
1
0.5
6
0
-0.5
4
-1
0

2 2
4

6 0

Figure 5.1: A Scalar Field and a Vector Field

Consider a scalar field in R3 , u(x, y, z). Higher derivatives of u are denoted:

2u u
uxx 2
,
x x x
2u u
uxy ,
xy x y
4u 2 u
uxxyz .
x2 yz x2 y z

156
If uxy and uyx are continuous, then
2u 2u
= .
xy yx
This is referred to as the equality of mixed partial derivatives.

Partial Derivatives of Vector Fields. Consider a vector field u(x). The partial derivative of u with respect to
u
xk is denoted x k u(x), xk
or uxk and is defined

u u(x1 , . . . , xk + x, . . . , xn ) u(x1 , . . . , xk , . . . , xn )
lim .
xk x0 x

Partial derivatives of vector fields have the same differentiation formulas as ordinary derivatives.

Gradient. We introduce the vector differential operator,


e1 + + en ,
x1 xn

which is known as del or nabla. In R3 it is



i+ j + k.
x y z

Let u(x) be a differential scalar field. The gradient of u is,

u u
u e1 + + en ,
x1 xn

Directional Derivative. Suppose you are standing on some terrain. The slope of the ground in a particular
direction is the directional derivative of the elevation in that direction. Consider a differentiable scalar field, u(x). The

157
derivative of the function in the direction of the unit vector a is the rate of change of the function in that direction.
Thus the directional derivative, Da u, is defined:
u(x + a) u(x)
Da u(x) = lim
0 
u(x1 + a1 , . . . , xn + an ) u(x1 , . . . , xn )
= lim
0 
(u(x) + a1 ux1 (x) + + an uxn (x) + O(2 )) u(x)
= lim
0 
= a1 ux1 (x) + + an uxn (x)

Da u(x) = u(x) a.

Tangent to a Surface. The gradient, f , is orthogonal to the surface f (x) = 0. Consider a point on the
surface. Let the differential dr = dx1 e1 + dxn en lie in the tangent plane at . Then
f f
df = dx1 + + dxn = 0
x1 xn
since f (x) = 0 on the surface. Then
 
f f
f dr = e1 + + en (dx1 e1 + + dxn en )
x1 xn
f f
= dx1 + + dxn
x1 xn
=0

Thus f is orthogonal to the tangent plane and hence to the surface.

Example 5.2.1 Consider the paraboloid, x2 + y 2 z = 0. We want to find the tangent plane to the surface at the
point (1, 1, 2). The gradient is
f = 2xi + 2yj k.

158
At the point (1, 1, 2) this is
f (1, 1, 2) = 2i + 2j k.
We know a point on the tangent plane, (1, 1, 2), and the normal, f (1, 1, 2). The equation of the plane is

f (1, 1, 2) (x, y, z) = f (1, 1, 2) (1, 1, 2)


2x + 2y z = 2

The gradient of the function f (x) = 0, f (x), is in the direction of the maximum directional derivative. The
magnitude of the gradient, |f (x)|, is the value of the directional derivative in that direction. To derive this, note that

Da f = f a = |f | cos ,

where is the angle between f and a. Da f is maximum when = 0, i.e. when a is the same direction as f . In
this direction, Da f = |f |. To use the elevation example, f points in the uphill direction and |f | is the uphill
slope.

Example 5.2.2 Suppose that the two surfaces f (x) = 0 and g(x) = 0 intersect at the point x = . What is the angle
between their tangent planes at that point? First we note that the angle between the tangent planes is by definition the
angle between their normals. These normals are in the direction of f () and g(). (We assume these are nonzero.)
The angle, , between the tangent planes to the surfaces is
 
f () g()
= arccos .
|f ()| |g()|

Example 5.2.3 Let u be the distance from the origin:



u(x) = x x = xi xi .

In three dimensions, this is p


u(x, y, z) = x2 + y 2 + z 2 .

159
The gradient of u, (x), is a unit vector in the direction of x. The gradient is:
 
x1 xn x i ei
u(x) = ,..., = .
xx xx xj xj

In three dimensions, we have


* +
x y z
u(x, y, z) = p ,p ,p .
x2 + y 2 + z 2 x2 + y 2 + z 2 x2 + y 2 + z 2

This is a unit vector because the sum of the squared components sums to unity.
xi ei xk ek xi xi
u u = =1
xj xj xl xl xj xj

Figure 5.2 shows a plot of the vector field of u in two dimensions.

Example 5.2.4 Consider an ellipse. An implicit equation of an ellipse is


x2 y 2
+ 2 = 1.
a2 b
We can also express an ellipse as u(x, y) + v(x, y) = c where u and v are the distance from the two foci. That is, an
ellipse is the set of points such that the sum of the distances from the two foci is a constant. Let n = (u + v). This
is a vector which is orthogonal to the ellipse when evaluated on the surface. Let t be a unit tangent to the surface.
Since n and t are orthogonal,

nt=0
(u + v) t = 0
u t = v (t).

Since these are unit vectors, the angle between u and t is equal to the angle between v and t. In other words:
If we draw rays from the foci to a point on the ellipse, the rays make equal angles with the ellipse. If the ellipse were

160
Figure 5.2: The gradient of the distance from the origin.
n
v

-t u


t
u v

Figure 5.3: An ellipse and rays from the foci.

a reflective surface, a wave starting at one focus would be reflected from the ellipse and travel to the other focus. See
Figure 5.3. This result also holds for ellipsoids, u(x, y, z) + v(x, y, z) = c.

161
We see that an ellipsoidal dish could be used to collect spherical waves, (waves emanating from a point). If the
dish is shaped so that the source of the waves is located at one foci and a collector is placed at the second, then any
wave starting at the source and reflecting off the dish will travel to the collector. See Figure 5.4.

Figure 5.4: An elliptical dish.

162
5.3 Exercises
Vector Functions
Exercise 5.1
Consider the parametric curve
   
t t
r = cos i + sin j.
2 2
d2 r
Calculate dr
dt
and dt2
. Plot the position and some velocity and acceleration vectors.
Hint, Solution
Exercise 5.2
Let r(t) be the position of an object moving with constant speed. Show that the acceleration of the object is orthogonal
to the velocity of the object.
Hint, Solution

Vector Fields
Exercise 5.3
Consider the paraboloid x2 + y 2 z = 0. What is the angle between the two tangent planes that touch the surface at
(1, 1, 2) and (1, 1, 2)? What are the equations of the tangent planes at these points?
Hint, Solution
Exercise 5.4
Consider the paraboloid x2 + y 2 z = 0. What is the point on the paraboloid that is closest to (1, 0, 0)?
Hint, Solution
Exercise 5.5
Consider the region R defined by x2 + xy + y 2 9. What is the volume of the solid obtained by rotating R about the
y axis?
Is this the same as the volume of the solid obtained by rotating R about the x axis? Give geometric and algebraic
explanations of this.

163
Hint, Solution
Exercise 5.6
Two cylinders of unit radius intersect at right angles as shown in Figure 5.5. What is the volume of the solid enclosed
by the cylinders?

Figure 5.5: Two cylinders intersecting.

Hint, Solution
Exercise 5.7
Consider the curve f (x) = 1/x on the interval [1 . . . ). Let S be the solid obtained by rotating f (x) about the x
axis. (See Figure 5.6.) Show that the length of f (x) and the lateral area of S are infinite. Find the volume of S. 1
Hint, Solution
Exercise 5.8
Suppose that a deposit of oil looks like a cone in the ground as illustrated in Figure 5.7. Suppose that the oil has a
1
You could fill S with a finite amount of paint, but it would take an infinite amount of paint to cover its surface.

164
1

1 1-1
2
3 0
4
5 -1

Figure 5.6: The rotation of 1/x about the x axis.

density of 800kg/m3 and its vertical depth is 12m. How much work2 would it take to get the oil to the surface.
Hint, Solution
Exercise 5.9
Find the area and volume of a sphere of radius R by integrating in spherical coordinates.
Hint, Solution

2
Recall that work = force distance and force = mass acceleration.

165
surface
32 m

12 m

12 m

ground

Figure 5.7: The oil deposit.

5.4 Hints
Vector Functions
Hint 5.1
Plot the velocity and acceleration vectors at regular intervals along the path of motion.

Hint 5.2
If r(t) has constant speed, then |r0 (t)| = c. The condition that the acceleration is orthogonal to the velocity can be
stated mathematically in terms of the dot product, r00 (t) r0 (t) = 0. Write the condition of constant speed in terms of
a dot product and go from there.

Vector Fields
Hint 5.3
The angle between two planes is the angle between the vectors orthogonal to the planes. The angle between the two

166
vectors is  
h2, 2, 1i h2, 2, 1i
= arccos
|h2, 2, 1i||h2, 2, 1i|
The equation of a line orthogonal to a and passing through the point b is a x = a b.
Hint 5.4
Since the paraboloid is a differentiable surface, the normal to the surface at the closest point will be parallel to the
vector from the closest point to (1, 0, 0). We can express this using the gradient and the cross product. If (x, y, z) is
the closest point on the paraboloid, then a vector orthogonal to the surface there is f = h2x, 2y, 1i. The vector
from the surface to the point (1, 0, 0) is h1 x, y, zi. These two vectors are parallel if their cross product is zero.

Hint 5.5
CONTINUE

Hint 5.6
CONTINUE

Hint 5.7
CONTINUE

Hint 5.8
Start with the formula for the work required to move the oil to the surface. Integrate over the mass of the oil.
Z
Work = (acceleration) (distance) d(mass)

Here (distance) is the distance of the differential of mass from the surface. The acceleration is that of gravity, g.

Hint 5.9
CONTINUE

167
5.5 Solutions
Vector Functions
Solution 5.1
The velocity is    
0 1 t 1 t
r = sin i + cos j.
2 2 2 2
The acceleration is    
0 1 t 1 t
r = cos i sin j.
4 2 4 2
See Figure 5.8 for plots of position, velocity and acceleration.

Figure 5.8: A Graph of Position and Velocity and of Position and Acceleration

Solution 5.2
If r(t) has constant speed, then |r0 (t)| = c. The condition that the acceleration is orthogonal to the velocity can be
stated mathematically in terms of the dot product, r00 (t) r0 (t) = 0. Note that we can write the condition of constant

168
speed in terms of a dot product, p
r0 (t) r0 (t) = c,

r0 (t) r0 (t) = c2 .
Differentiating this equation yields,
r00 (t) r0 (t) + r0 (t) r00 (t) = 0

r00 (t) r0 (t) = 0.


This shows that the acceleration is orthogonal to the velocity.

Vector Fields
Solution 5.3
The gradient, which is orthogonal to the surface when evaluated there is f = 2xi+2yjk. 2i+2jk and 2i2jk
are orthogonal to the paraboloid, (and hence the tangent planes), at the points (1, 1, 2) and (1, 1, 2), respectively.
The angle between the tangent planes is the angle between the vectors orthogonal to the planes. The angle between
the two vectors is  
h2, 2, 1i h2, 2, 1i
= arccos
|h2, 2, 1i||h2, 2, 1i|
 
1
= arccos 1.45946.
9

Recall that the equation of a line orthogonal to a and passing through the point b is a x = a b. The equations of
the tangent planes are
h2, 2, 1i hx, y, zi = h2, 2, 1i h1, 1, 2i,

2x 2y z = 2.
The paraboloid and the tangent planes are shown in Figure 5.9.

169
-1
0
1

1
0
-1

Figure 5.9: Paraboloid and Two Tangent Planes

Solution 5.4
Since the paraboloid is a differentiable surface, the normal to the surface at the closest point will be parallel to the
vector from the closest point to (1, 0, 0). We can express this using the gradient and the cross product. If (x, y, z) is
the closest point on the paraboloid, then a vector orthogonal to the surface there is f = h2x, 2y, 1i. The vector
from the surface to the point (1, 0, 0) is h1 x, y, zi. These two vectors are parallel if their cross product is zero,
h2x, 2y, 1i h1 x, y, zi = hy 2yz, 1 + x + 2xz, 2yi = 0.
This gives us the three equations,
y 2yz = 0,
1 + x + 2xz = 0,
2y = 0.
The third equation requires that y = 0. The first equation then becomes trivial and we are left with the second equation,
1 + x + 2xz = 0.
Substituting z = x2 + y 2 into this equation yields,
2x3 + x 1 = 0.

170
The only real valued solution of this polynomial is
2/3
62/3 9 + 87 61/3
x= 1/3 0.589755.
9 + 87

Thus the closest point to (1, 0, 0) on the paraboloid is


2/3 2/3 !2
62/3 9 + 87 61/3 62/3 9 + 87 61/3
1/3 , 0, 1/3 (0.589755, 0, 0.34781).
9 + 87 9 + 87

The closest point is shown graphically in Figure 5.10.

1-1
1
0.5 -0.5
0 0
-0.5 0.5
-1 1
2

1.5

0.5

Figure 5.10: Paraboloid, Tangent Plane and Line Connecting (1, 0, 0) to Closest Point

171
Solution 5.5
We consider the region R defined by x2 + xy + y 2 9. The boundary of the region is an ellipse. (See Figure 5.11 for
the ellipse and the solid obtained by rotating the region.) Note that in rotating the region about the y axis, only the

2
3 0
-2
2

1
2

-3 -2 -1 1 2 3 0

-1
-2

-2
-2
-3 0
2

Figure 5.11: The curve x2 + xy + y 2 = 9.

portions in the second and fourth quadrants make a contribution. Since the solid is symmetric across the xz plane, we
will find the volume of the top half and then double this to get the volume of the whole solid. Now we consider rotating
the region in the second quadrant about the y axis. In the equation for the ellipse, x2 + xy + y 2 = 9, we solve for x.

1 p 
x= y 3 12 y 2
2
p
In the second quadrant, the curve (y 3 12 y 2 )/2 is defined on y [0 . . . 12] and the curve (y
p
2
3 12 y )/2 is defined on y [3 . . . 12]. (See Figure 5.12.) We find the volume obtained by rotating the

172
3.5

2.5

1.5

0.5

-3.5 -3 -2.5 -2 -1.5 -1 -0.5

p p
Figure 5.12: (y 3 12 y 2 )/2 in red and (y + 3 12 y 2 )/2 in green.

first curve and subtract the volume from rotating the second curve.

Z 12 p !2 Z 12 p !2
y 3 12 y 2 y + 3 12 y 2
V = 2 dy dy
0 2 3 2
Z 12  p Z 12  p
!
2 2
V = y + 3 12 y 2 dy y + 3 12 y 2 dy
2 0 3
!

Z 12  p  Z 12  p 
V = 2y 2 + 12y 12 y 2 + 36 dy 2y 2 12y 12 y 2 + 36 dy
2 0 3
 12  12 !
2 3 2 3/2 2 2 3/2
y 12 y 2 y 3 + 12 y 2
 
V = + 36y + 36y
2 3 3 0 3 3 3

V = 72

173
Now consider the volume of the solid obtained by rotating R about the x axis? This as the same as the volume of
the solid obtained by rotating R about the y axis. Geometrically we know this because R is symmetric about the line
y = x.
Now we justify it algebraically. Consider the phrase: Rotate the region x2 + xy + y 2 9 about the x axis. We
formally swap x and y to obtain: Rotate the region y 2 + yx + x2 9 about the y axis. Which is the original problem.

Solution 5.6
We find of the volume of the intersecting cylinders by summing the volumes of the two cylinders and then subracting the
volume of their intersection. The volume of each of the cylinders is 2. The intersection
is shown in Figure 5.13. If we
slice this solid along the plane z = const we have a square with side length 2 1 z 2 . The volume of the intersection
of the cylinders is
Z 1
4 1 z 2 dz.

1

We compute the volume of the intersecting cylinders.

1
0.5
0
-0.5
-1
1

0.5

-0.5

-1
-1
-0.5
0
0.5
1

Figure 5.13: The intersection of the two cylinders.

174
Z 1
4 1 z 2 dz

V = 2(2) 2
0
16
V = 4
3
Solution 5.7
The length of f (x) is Z p
L= 1 + 1/x2 dx.
1
p
Since 1 + 1/x2 > 1/x, the integral diverges. The length is infinite.
We find the area of S by integrating the length of circles.
Z
2
A= dx
1 x
This integral also diverges. The area is infinite.
Finally we find the volume of S by integrating the area of disks.
Z h i

V = dx = =
1 x2 x 1
Solution 5.8
First we write the formula for the work required to move the oil to the surface. We integrate over the mass of the oil.
Z
Work = (acceleration) (distance) d(mass)

Here (distance) is the distance of the differential of mass from the surface. The acceleration is that of gravity, g. The
differential of mass can be represented an a differential of volume time the density of the oil, 800 kg/m3 .
Z
Work = 800g(distance) d(volume)

175
We place the coordinate axis so that z = 0 coincides with the bottom of the cone. The oil lies between z = 0 and
z = 12. The cross sectional area of the oil deposit at a fixed depth is z 2 . Thus the differential of volume is z 2 dz.
This oil must me raised a distance of 24 z.
Z 12
W = 800 g (24 z) z 2 dz
0
W = 6912000g
kg m2
W 2.13 108
s2
Solution 5.9
The Jacobian in spherical coordinates is r2 sin .
Z 2 Z
area = R2 sin d d
0
Z0
= 2R2 sin d
0
= 2R2 [ cos ]0

area = 4R2
Z R Z 2 Z
volume = r2 sin d d dr
0 0 0
Z RZ
= 2 r2 sin d dr
0 0
 3 R
r
= 2 [ cos ]0
3 0
4
volume = R3
3

176
5.6 Quiz
Problem 5.1
What is the distance from the origin to the plane x + 2y + 3z = 4?
Solution
Problem 5.2
A bead of mass m slides frictionlessly on a wire determined parametrically by w(s). The bead moves under the force
of gravity. What is the acceleration of the bead as a function of the parameter s?
Solution

177
5.7 Quiz Solutions
Solution 5.1
Recall that the equation of a plane is x n = a n where a is a point in the plane and n is normal to the plane. We
are considering the plane x + 2y + 3z = 4. A normal to the plane is h1, 2, 3i. The unit normal is
1
n = h1, 2, 3i.
15
By substituting in x = y = 0, we see that a point in the plane is a = h0, 0, 4/3i. The distance of the plane from the
origin is a n = 415 .

Solution 5.2
The force of gravity is gk. The unit tangent to the wire is w0 (s)/|w0 (s)|. The component of the gravitational force
in the tangential direction is gk w0 (s)/|w0 (s)|. Thus the acceleration of the bead is

gk w0 (s)
.
m|w0 (s)|

178
Part III

Functions of a Complex Variable

179
Chapter 6

Complex Numbers

Im sorry. You have reached an imaginary number. Please rotate your phone 90 degrees and dial again.

-Message on answering machine of Cathy Vargas.

6.1 Complex Numbers


Shortcomings of real numbers. When you started algebra, you learned that the quadratic equation: x2 +2ax+b =
0 has either two, one or no solutions. For example:

x2 3x + 2 = 0 has the two solutions x = 1 and x = 2.

For x2 2x + 1 = 0, x = 1 is a solution of multiplicity two.

x2 + 1 = 0 has no solutions.

180
This is a little unsatisfactory. We can formally solve the general quadratic equation.

x2 + 2ax + b = 0
(x + a)2 = a2 b

x = a a2 b

However,the solutions are defined only when the discriminant a2 b is non-negative. This is because the square root
function x is a bijection from R0+ to R0+ . (See Figure 6.1.)


Figure 6.1: y = x

A new mathematical constant. We cannot solve x2 = 1 because the square root of 1is not defined. To
overcome this apparent shortcoming
of the real number system, we create a new symbolic constant 1.In performing

arithmetic, we will treat 1 as wewould2 a real constant like or a formal variable like x, i.e. 1 + 1 = 2 1.
This constant has the property: 1 = 1. Now we can express the solutions of x2 = 1 as x = 1 and
2 2 2
x = 1. These satisfy the equation since 1 = 1 and 1 = (1)2 1 = 1. Note that we

can express the square root of any negative real number in terms of 1: r = 1 r for r 0.

181

Eulers notation. Eulerintroduced the notation of using the letter i to denote 1. We will use the symbol , an
i without a dot, to denote 1. This helps us distinguish it from i used as a variable or index.1 We call any number
of the form b, b R, a pure imaginary number.2 Let a and b be real numbers. The product of a real number and an
imaginary number is an imaginary number: (a)(b) = (ab). The product of two imaginary numbers is a real number:
(a)(b) = ab. However the sum of a real number and an imaginary number a + b is neither real nor imaginary. We
call numbers of the form a + b complex numbers.3

The quadratic.
Now we return to the quadratic with real coefficients, x2 + 2ax + b = 0. It has the solutions
x = a a b. The solutions are real-valued only if a2 b
2 0. If not, then we can define solutions as complex
numbers. If the discriminant is negative, we write x = a b a2 . Thus every quadratic polynomial with real
coefficients has exactly two solutions, counting multiplicities. The fundamental theorem of algebra states that an nth
degree polynomial with complex coefficients has n, not necessarily distinct, complex roots. We will prove this result
later using the theory of functions of a complex variable.

Component operations. Consider the complex number z = x + y, (x, y R). The real part of z is <(z) = x;
the imaginary part of z is =(z) = y. Two complex numbers, z = x + y and = + , are equal if and only if x =
and y = . The complex conjugate 4 of z = x + y is z x y. The notation z x y is also used.

A little arithmetic. Consider two complex numbers: z = x + y, = + . It is easy to express the sum or
difference as a complex number.
z + = (x + ) + (y + ), z = (x ) + (y )
It is also easy to form the product.
z = (x + y)( + ) = x + x + y + 2 y = (x y) + (x + y)
1

Electrical engineering types prefer to use or j to denote 1.
2
Imaginary is an unfortunate term. Real numbers are artificial; constructs of the mind. Real numbers are no more real than
imaginary numbers.
3
Here complex means composed of two or more parts, not hard to separate, analyze, or solve. Those who disagree have a
complex number complex.
4
Conjugate: having features in common but opposite or inverse in some particular.

182
The quotient is a bit more difficult. (Assume that is nonzero.) How do we express z/ = (x + y)/( + ) as
the sum of a real number and an imaginary number? The trick is to multiply the numerator and denominator by the
complex conjugate of .
z x + y x + y x x y 2 y (x + y) (x + y) (x + y) x + y
= = = 2 2 2
= 2 2
= 2 2
2
+ + + + + + 2
Now we recognize it as a complex number.

Field properties. The set of complex numbers C form a field. That essentially means that we can do arithmetic
with complex numbers. When performing arithmetic, we simply treat as a symbolic constant with the property that
2 = 1. The field of complex numbers satisfy the following list of properties. Each one is easy to verify; some are
proved below. (Let z, , C.)
1. Closure under addition and multiplication.

z + = (x + y) + ( + )
= (x + ) + (y + ) C
z = (x + y) ( + )
= x + x + y + 2 y
= (x y) + (x + y) C

2. Commutativity of addition and multiplication. z + = + z. z = z.


3. Associativity of addition and multiplication. (z + ) + = z + ( + ). (z) = z ().
4. Distributive law. z ( + ) = z + z.
5. Identity with respect to addition and multiplication. Zero is the additive identity element, z + 0 = z; unity is the
muliplicative identity element, z(1) = z.
6. Inverse with respect to addition. z + (z) = (x + y) + (x y) = (x x) + (y y) = 0.

183
7. Inverse with respect to multiplication for nonzero numbers. zz 1 = 1, where

1 1 1 x y x y x y
z 1 = = = = 2 2
= 2 2
2
z x + y x + y x y x +y x +y x + y2

Properties of the complex conjugate. Using the field properties of complex numbers, we can derive the following
properties of the complex conjugate, z = x y.

1. (z) = z,

2. z + = z + ,

3. z = z,
 
z (z)
4. = .

6.2 The Complex Plane


Complex plane. We can denote a complex number z = x + y as an ordered pair of real numbers (x, y). Thus we
can represent a complex number as a point in R2 where the first component is the real part and the second component
is the imaginary part of z. This is called the complex plane or the Argand diagram. (See Figure 6.2.) A complex
number written as z = x + y is said to be in Cartesian form, or a + b form.

Recall that there are two ways of describing a point in the complex plane: an ordered pair of coordinates (x, y) that
give the horizontal and vertical offset from the origin or the distance r from the origin and the angle from the positive
horizontal axis. The angle is not unique. It is only determined up to an additive integer multiple of 2.

184
Im(z)

(x,y)
r
Re(z)

Figure 6.2: The complex plane.

Modulus. The magnitude p or modulus of a complex number is the distance of the point from the origin. It is
defined as |z| = |x + y| = x2 + y 2 . Note that zz = (x + y)(x y) = x2 + y 2 = |z|2 . The modulus has the
following properties.
1. |z| = |z| ||

z |z|
2. = for 6= 0.
||
3. |z + | |z| + ||
4. |z + | ||z| |||
We could prove the first two properties by expanding in x + y form, but it would be fairly messy. The proofs will
become simple after polar form has been introduced. The second two properties follow from the triangle inequalities in
geometry. This will become apparent after the relationship between complex numbers and vectors is introduced. One
can show that
|z1 z2 zn | = |z1 | |z2 | |zn |
and
|z1 + z2 + + zn | |z1 | + |z2 | + + |zn |
with proof by induction.

185
Argument. The argument of a complex number is the angle that the vector with tail at the origin and head at
z = x + y makes with the positive x-axis. The argument is denoted arg(z). Note that the argument is defined for all
nonzero numbers and is only determined up to an additive integer multiple of 2. That is, the argument of a complex
number is the set of values: { + 2n | n Z}. The principal argument of a complex number is that angle in the set
arg(z) which lies in the range (, ]. The principal argument is denoted Arg(z). We prove the following identities in
Exercise 6.10.

arg(z) = arg(z) + arg()


Arg(z) 6= Arg(z) + Arg()
arg z 2 = arg(z) + arg(z) 6= 2 arg(z)


Example 6.2.1 Consider the equation |z 1 | = 2. The set of points satisfying this equation is a circle of radius
2 and center at 1 + in the complex plane. You can see this by noting that |z 1 | is the distance from the point
(1, 1). (See Figure 6.3.)

-1 1 2 3

-1

Figure 6.3: Solution of |z 1 | = 2.

186
Another way to derive this is to substitute z = x + y into the equation.

|x + y 1 | = 2
p
(x 1)2 + (y 1)2 = 2
(x 1)2 + (y 1)2 = 4

This is the analytic geometry equation for a circle of radius 2 centered about (1, 1).

Example 6.2.2 Consider the curve described by

|z| + |z 2| = 4.

Note that |z| is the distance from the origin in the complex plane and |z 2| is the distance from z = 2. The equation
is
(distance from (0, 0)) + (distance from (2, 0)) = 4.

From geometry, we know that this is an ellipse with foci at (0, 0) and (2, 0), major axis 2, and minor axis 3. (See
Figure 6.4.)
We can use the substitution z = x + y to get the equation in algebraic form.

|z| + |z 2| = 4
|x + y| + |x + y 2| = 4
p p
x2 + y 2 + (x 2)2 + y 2 = 4
p
x2 + y 2 = 16 8 (x 2)2 + y 2 + x2 4x + 4 + y 2
p
x 5 = 2 (x 2)2 + y 2
x2 10x + 25 = 4x2 16x + 16 + 4y 2
1 1
(x 1)2 + y 2 = 1
4 3
Thus we have the standard form for an equation describing an ellipse.

187
2

-1 1 2 3

-1

-2

Figure 6.4: Solution of |z| + |z 2| = 4.

6.3 Polar Form


Polar form. A complex number written in Cartesian form, z = x + y, can be converted polar form, z = r(cos +
sin ), using trigonometry. Here r = |z| is the modulus and = arctan(x, y) is the argument of z. The argument is
the angle between the x axis and the vector with its head at (x, y). (See Figure 6.5.) Note that is not unique. If
z = r(cos + sin ) then z = r(cos( + 2n) + sin( + 2n)) for any n Z.

The arctangent. Note that arctan(x, y) is not the same thing as the old arctangent that you learned about in
trigonometry arctan(x, y) is sensitive to the quadrant of the point (x, y), while arctan xy is not. For example,

3
arctan(1, 1) = + 2n and arctan(1, 1) = + 2n,
4 4

188
Im( z ) (x,y)

r r sin
Re(z )
r cos

Figure 6.5: Polar form.

whereas    
1 1
arctan = arctan = arctan(1).
1 1

Eulers formula. Eulers formula, e = cos + sin ,5 allows us to write the polar form more compactly. Expressing
the polar form in terms of the exponential function of imaginary argument makes arithmetic with complex numbers
much more convenient.
z = r(cos + sin ) = r e
The exponential of an imaginary argument has all the nice properties that we know from studying functions of a real
variable, like ea eb = e(a+b) . Later on we will introduce the exponential of a complex number.
Using Eulers Formula, we can express the cosine and sine in terms of the exponential.
e + e (cos() + sin()) + (cos() + sin())
= = cos()
2 2
e e (cos() + sin()) (cos() + sin())
= = sin()
2 2

Arithmetic with complex numbers. Note that it is convenient to add complex numbers in Cartesian form.

z + = (x + y) + ( + ) = (x + ) + (y + )
5
See Exercise 6.17 for justification of Eulers formula.

189
However, it is difficult to multiply or divide them in Cartesian form.

z = (x + y) ( + ) = (x y) + (x + y)
z x + y (x + y) ( ) x + y y x
= = = 2 2
+ 2
+ ( + ) ( ) + + 2

On the other hand, it is difficult to add complex numbers in polar form.

z + = r e + e
= r (cos + sin ) + (cos + sin )
= r cos + cos + (r sin + sin )
q
= (r cos + cos )2 + (r sin + sin )2
e arctan(r cos + cos ,r sin + sin )
p
= r2 + 2 + 2 cos ( ) e arctan(r cos + cos ,r sin + sin )

However, it is convenient to multiply and divide them in polar form.

z = r e e = r e(+)
z r e r
= = e()
e

Keeping this in mind will make working with complex numbers a shade or two less grungy.

190
Result 6.3.1 Eulers formula is

e = cos + sin .

We can write the cosine and sine in terms of the exponential.


e + e e e
cos() = , sin() =
2 2
To change between Cartesian and polar form, use the identities

r e = r cos + r sin ,
p
x + y = x2 + y 2 e arctan(x,y) .

Cartesian form is convenient for addition. Polar form is convenient for multiplication and
division.

Example 6.3.1 We write 5 + 7 in polar form.



5 + 7 = 74 e arctan(5,7)

We write 2 e/6 in Cartesian form.


   
2 e/6 = 2 cos + 2 sin
6 6
= 3+

Example 6.3.2 We will prove the trigonometric identity


1 1 3
cos4 = cos(4) + cos(2) + .
8 2 8

191
We start by writing the cosine in terms of the exponential.

4
e + e

4
cos =
2
1 4
e +4 e2 +6 + 4 e2 + e4

=
16
1 e4 + e4 1 e2 + e2
  
3
= + +
8 2 2 2 8
1 1 3
= cos(4) + cos(2) +
8 2 8

n
By the definition of exponentiation, we have en = e We apply Eulers formula to obtain a result which is useful
in deriving trigonometric identities.

cos(n) + sin(n) = (cos + sin )n

Result 6.3.2 DeMoivres Theorem.a

cos(n) + sin(n) = (cos + sin )n


a
Its amazing what passes for a theorem these days. I would think that this would be a corollary at most.

Example 6.3.3 We will express cos(5) in terms of cos and sin(5) in terms of sin . We start with DeMoivres
theorem.
5
e5 = e

192
cos(5) + sin(5) = (cos + sin )5
       
5 5 5 4 5 3 2 5
= cos + cos sin cos sin cos2 sin3
0 1 2 3
   
5 4 5
+ cos sin + sin5
4 5
= cos 10 cos sin + 5 cos sin4 + 5 cos4 sin 10 cos2 sin3 + sin5
5 3 2
 

Then we equate the real and imaginary parts.


cos(5) = cos5 10 cos3 sin2 + 5 cos sin4
sin(5) = 5 cos4 sin 10 cos2 sin3 + sin5
Finally we use the Pythagorean identity, cos2 + sin2 = 1.
2
cos(5) = cos5 10 cos3 1 cos2 + 5 cos 1 cos2


cos(5) = 16 cos5 20 cos3 + 5 cos


2
sin(5) = 5 1 sin2 sin 10 1 sin2 sin3 + sin5


sin(5) = 16 sin5 20 sin3 + 5 sin

6.4 Arithmetic and Vectors


Addition. We can represent the complex number z = x + y = r e as a vector in Cartesian space with tail at the
origin and head at (x, y), or equivalently, the vector of length r and angle . With the vector representation, we can
add complex numbers by connecting the tail of one vector to the head of the other. The vector z + is the diagonal
of the parallelogram defined by z and . (See Figure 6.6.)

Negation. The negative of z = x + y is z = x y. In polar form we have z = r e and z = r e(+) , (more


generally, z = r e(+(2n+1)) , n Z. In terms of vectors, z has the same magnitude but opposite direction as z. (See
Figure 6.6.)

193
Multiplication. The product of z = r e and = e is z = r e(+) . The length of the vector z is the
product of the lengths of z and . The angle of z is the sum of the angles of z and . (See Figure 6.6.)
Note that arg(z) = arg(z) + arg(). Each of these arguments has an infinite number of values. If we write out
the multi-valuedness explicitly, we have

{ + + 2n : n Z} = { + 2n : n Z} + { + 2n : n Z}

The same is not true of the principal argument. In general, Arg(z) 6= Arg(z) + Arg(). Consider the case z = =
e3/4 . Then Arg(z) = Arg() = 3/4, however, Arg(z) = /2.

z =(xy )+i(x+y )
=r e i(+)
z+ =(x+ )+i(y+ ) =+i=ei
z=x+iy z=x+iy =rei
=+i z=x+iy =re i

z=xiy
=re i(+ )

Figure 6.6: Addition, negation and multiplication.

Multiplicative inverse. Assume that z is nonzero. The multiplicative inverse of z = r e is z1 = 1r e . The


length of z1 is the multiplicative inverse of the length of z. The angle of z1 is the negative of the angle of z. (See
Figure 6.7.)

194
Division. Assume that is nonzero. The quotient of z = r e and = e is z = r e() . The length of the
vector z is the quotient of the lengths of z and . The angle of z is the difference of the angles of z and . (See
Figure 6.7.)

Complex conjugate. The complex conjugate of z = x + y = r e is z = x y = r e . z is the mirror image


of z, reflected across the x axis. In other words, z has the same magnitude as z and the angle of z is the negative of
the angle of z. (See Figure 6.7.)

= e i
z=re i z=x+iy=re i
z=re i

_z = _r e i ()
_1 = e
1 i
z r
_
z=xiy=rei

Figure 6.7: Multiplicative inverse, division and complex conjugate.

6.5 Integer Exponents


Consider the product (a + b)n , n Z. If we know arctan(a, b) then it will be most convenient to expand the product
working in polar form. If not, we can write n in base 2 to efficiently do the multiplications.

195

20
Example 6.5.1 Suppose that we want to write 3+ in Cartesian form.6 We can do the multiplication directly.
 2n
Note that 20 is 10100 in base 2. That is, 20 = 24 + 22 . We first calculate the powers of the form 3+ by
successive squaring.
 2
3 + = 2 + 2 3
 4
3 + = 8 + 8 3
 8
3 + = 128 128 3
 16
3+ = 32768 + 32768 3
4 16
Next we multiply 3 + and 3+ to obtain the answer.
 20   
3+ = 32768 + 32768 3 8 + 8 3 = 524288 524288 3

Since we know that arctan 3, 1 = /6, it is easiest to do this problem by first changing to modulus-argument
form.
!20
 20 r 
2
3+ = 3 + 12 e arctan( 3,1)
20
= 2 e/6
= 220 e4/3
!
1 3
= 1048576
2 2

= 524288 524288 3
6
No, I have no idea why we would want to do that. Just humor me. If you pretend that youre interested, Ill do the same. Believe
me, expressing your real feelings here isnt going to do anyone any good.

196
Example 6.5.2 Consider (5 + 7)11 . We will do the exponentiation in polar form and write the result in Cartesian
form.
 11
11 arctan(5,7)
(5 + 7) = 74 e

= 745 74(cos(11 arctan(5, 7)) + sin(11 arctan(5, 7)))

= 2219006624 74 cos(11 arctan(5, 7)) + 2219006624 74 sin(11 arctan(5, 7))
The result is correct, but not very satisfying. This expression could be simplified. You could evaluate the trigonometric
functions with some fairly messy trigonometric identities. This would take much more work than directly multiplying
(5 + 7)11 .

6.6 Rational Exponents


In this section we consider complex numbers with rational exponents, z p/q , where p/q is a rational number. First we
consider unity raised to the 1/n power. We define 11/n as the set of numbers {z} such that z n = 1.
11/n = {z | z n = 1}
We can find these values by writing z in modulus-argument form.
zn = 1
rn en = 1
rn = 1 n = 0 mod 2
r=1 = 2k for k Z
1/n
 2k/n
1 = e |kZ
There are only n distinct values as a result of the 2 periodicity of e . e2 = e0 .
11/n = e2k/n | k = 0, . . . , n 1


These values are equally spaced points on the unit circle in the complex plane.

197
Example 6.6.1 11/6 has the 6 values,
e0 , e/3 , e2/3 , e , e4/3 , e5/3 .


In Cartesian form this is ( )


1 + 3 1 + 3 1 3 1 3
1, , , 1, , .
2 2 2 2
The sixth roots of unity are plotted in Figure 6.8.

-1 1

-1

Figure 6.8: The sixth roots of unity.

The nth roots of the complex number c = e are the set of numbers z = r e such that
z n = c = e
rn en = e

n
r= n = mod 2

r= n
= ( + 2k)/n for k = 0, . . . , n 1.
Thus  np o
c1/n = n
e(+2k)/n | k = 0, . . . , n 1 = n |c| e(Arg(c)+2k)/n | k = 0, . . . , n 1

198
Principal roots. The principal nth root is denoted

n
z n z e Arg(z)/n .

Thus the principal root has the property 


/n < Arg n
z /n.

This is consistent with the notation from functions of a real variable: n x denotes the positive nth root of a positive

real number. We adopt the convention that z 1/n denotes the nth roots of z, which is a set of n numbers and n z is
the principal nth root of z, which is a single number. The nth roots of z are the principal nth root of z times the nth
roots of unity.

z 1/n = n r e(Arg(z)+2k)/n | k = 0, . . . , n 1


z 1/n = n z e2k/n | k = 0, . . . , n 1


z 1/n = n z11/n

Rational exponents. We interpret z p/q to mean z (p/q) . That is, we first simplify the exponent, i.e. reduce the
fraction, before carrying out the exponentiation. Therefore z 2/4 = z 1/2 and z 10/5 = z 2 . If p/q is a reduced fraction, (p
and q are relatively prime, in other words, they have no common factors), then

z p/q (z p )1/q .

Thus z p/q is a set of q values. Note that for an un-reduced fraction r/s,
r
(z r )1/s 6= z 1/s .
1/2
The former expression is a set of s values while the latter is a set of no more that s values. For instance, (12 ) =
2
11/2 = 1 and 11/2 = (1)2 = 1.

Example 6.6.2 Consider 21/5 , (1 + )1/3 and (2 + )5/6 .



21/5 = 2 e2k/5 ,
5
for k = 0, 1, 2, 3, 4

199
 1/3
(1 + )1/3 = 2 e/4

2 e/12 e2k/3 ,
6
= for k = 0, 1, 2

 5/6
(2 + )5/6 = 5 e Arctan(2,1)
 1/6
= 55 e5 Arctan(2,1)

12 5
= 55 e 6 Arctan(2,1) ek/3 , for k = 0, 1, 2, 3, 4, 5

Example 6.6.3 We find the roots of z 5 + 4.

(4)1/5 = (4 e )1/5

= 4 e(1+2k)/5 ,
5
for k = 0, 1, 2, 3, 4

200
6.7 Exercises
Complex Numbers
Exercise 6.1
If z = x + y, write the following in the form a + b:
1. (1 + 2)7
1
2.
(zz)
z + z
3.
(3 + )9
Hint, Solution
Exercise 6.2
Verify that:
1 + 2 2 2
1. + =
3 4 5 5
2. (1 )4 = 4
Hint, Solution
Exercise 6.3
Write the following complex numbers in the form a + b.
 10
1. 1 + 3

2. (11 + 4)2
Hint, Solution

201
Exercise 6.4
Write the following complex numbers in the form a + b
 2
2+
1.
6 (1 2)
2. (1 )7
Hint, Solution
Exercise 6.5
If z = x + y, write the following in the form u(x, y) + v(x, y).
 
z
1.
z
z + 2
2.
2 z
Hint, Solution
Exercise 6.6
Quaternions are sometimes used as a generalization of complex numbers. A quaternion u may be defined as
u = u0 + u1 + u2 + ku3
where u0 , u1 , u2 and u3 are real numbers and , and k are objects which satisfy
2 = 2 = k 2 = 1, = k, = k
and the usual associative and distributive laws. Show that for any quaternions u, w there exists a quaternion v such
that
uv = w
except for the case u0 = u1 = u2 = u3 .
Hint, Solution

202
Exercise 6.7
Let 6= 0, 6= 0 be two complex numbers. Show that = t for some real number t (i.e. the vectors defined by

and are parallel) if and only if = = 0.
Hint, Solution

The Complex Plane


Exercise 6.8
Find and depict all values of

1. (1 + )1/3

2. 1/4

Identify the principal root.


Hint, Solution
Exercise 6.9
Sketch the regions of the complex plane:

1. |<(z)| + 2|=(z)| 1

2. 1 |z | 2

3. |z | |z + |

Hint, Solution
Exercise 6.10
Prove the following identities.

1. arg(z) = arg(z) + arg()

2. Arg(z) 6= Arg(z) + Arg()

203
3. arg (z 2 ) = arg(z) + arg(z) 6= 2 arg(z)

Hint, Solution
Exercise 6.11
Show, both by geometric and algebraic arguments, that for complex numbers z and the inequalities

||z| ||| |z + | |z| + ||

hold.
Hint, Solution
Exercise 6.12
Find all the values of

1. (1)3/4

2. 81/6

and show them graphically.


Hint, Solution
Exercise 6.13
Find all values of

1. (1)1/4

2. 161/8

and show them graphically.


Hint, Solution
Exercise 6.14
Sketch the regions or curves described by

204
1. 1 < |z 2| < 2

2. |<(z)| + 5|=(z)| = 1

3. |z | = |z + |

Hint, Solution
Exercise 6.15
Sketch the regions or curves described by

1. |z 1 + | 1

2. <(z) =(z) = 5

3. |z | + |z + | = 1

Hint, Solution
Exercise 6.16
Solve the equation
| e 1| = 2
for (0 ) and verify the solution geometrically.
Hint, Solution

Polar Form
Exercise 6.17
Show that Eulers formula, e = cos + sin , is formally consistent with the standard Taylor series expansions for the
real functions ex , cos x and sin x. Consider the Taylor series of ex about x = 0 to be the definition of the exponential
function for complex argument.
Hint, Solution

205
Exercise 6.18
Use de Moivres formula to derive the trigonometric identity

cos(3) = cos3 () 3 cos() sin2 ().

Hint, Solution
Exercise 6.19
Establish the formula
1 z n+1
1 + z + z2 + + zn = , (z 6= 1),
1z
for the sum of a finite geometric series; then derive the formulas
1 sin((n + 1/2))
1. 1 + cos() + cos(2) + + cos(n) = +
2 2 sin(/2)
1 cos((n + 1/2))
2. sin() + sin(2) + + sin(n) = cot
2 2 2 sin(/2)
where 0 < < 2.
Hint, Solution

Arithmetic and Vectors


Exercise 6.20
|z|
Prove |z| = |z||| and z = using polar form.

||
Hint, Solution
Exercise 6.21
Prove that
|z + |2 + |z |2 = 2 |z|2 + ||2 .


Interpret this geometrically.


Hint, Solution

206
Integer Exponents
Exercise 6.22
Write (1 + )10 in Cartesian form with the following two methods:

1. Just do the multiplication. If it takes you more than four multiplications, you suck.

2. Do the multiplication in polar form.

Hint, Solution

Rational Exponents
Exercise 6.23
1/2
Show that each of the numbers z = a + (a2 b) satisfies the equation z 2 + 2az + b = 0.
Hint, Solution

207
6.8 Hints
Complex Numbers
Hint 6.1

Hint 6.2

Hint 6.3

Hint 6.4

Hint 6.5

Hint 6.6

Hint 6.7

The Complex Plane


Hint 6.8

Hint 6.9

208
Hint 6.10
Write the multivaluedness explicitly.

Hint 6.11
Consider a triangle with vertices at 0, z and z + .

Hint 6.12

Hint 6.13

Hint 6.14

Hint 6.15

Hint 6.16

Polar Form
Hint 6.17
Find the Taylor series of e , cos and sin . Note that 2n = (1)n .

Hint 6.18

Hint 6.19

Arithmetic and Vectors

209
Hint 6.20
| e | = 1.

Hint 6.21
Consider the parallelogram defined by z and .

Integer Exponents
Hint 6.22
For the first part,
  2 2
(1 + )10 = (1 + )2 (1 + )2 .

Rational Exponents
Hint 6.23
Substitite the numbers into the equation.

210
6.9 Solutions
Complex Numbers
Solution 6.1
1. We can do the exponentiation by directly multiplying.

(1 + 2)7 = (1 + 2)(1 + 2)2 (1 + 2)4


= (1 + 2)(3 + 4)(3 + 4)2
= (11 2)(7 24)
= 29 + 278

We can also do the problem using De Moivres Theorem.


 7
(1 + 2)7 = 5 e arctan(1,2)

= 125 5 e7 arctan(1,2)

= 125 5 cos(7 arctan(1, 2)) + 125 5 sin(7 arctan(1, 2))

2.
1 1
=
(zz) (x y)2
1 (x + y)2
=
(x y)2 (x + y)2
(x + y)2
= 2
(x + y 2 )2
x2 y 2 2xy
= 2 2 2
+ 2
(x + y ) (x + y 2 )2

211
3. We can evaluate the expression using De Moivres Theorem.

z + z
9
= (y + x + x y)(3 + )9
(3 + )
 9
= (1 + )(x y) 10 e arctan(3,1)
1
= (1 + )(x y) e9 arctan(3,1)
10000 10
(1 + )(x y)
= (cos(9 arctan(3, 1)) sin(9 arctan(3, 1)))
10000 10
(x y)
= (cos(9 arctan(3, 1)) + sin(9 arctan(3, 1)))
10000 10
(x y)
+ (cos(9 arctan(3, 1)) sin(9 arctan(3, 1)))
10000 10

212
We can also do this problem by directly multiplying but its a little grungy.

z + z (y + x + x y)(3 )9
=
(3 + )9 109
 2
2
(1 + )(x y)(3 ) ((3 )2 )
=
109
2
(1 + )(x y)(3 ) (8 6)2
=
109
(1 + )(x y)(3 )(28 96)2
=
109
(1 + )(x y)(3 )(8432 5376)
=
109
(x y)(22976 38368)
=
109
359(y x) 1199(y x)
= +
15625000 31250000
Solution 6.2
1.

1 + 2 2 1 + 2 3 + 4 2
+ = +
3 4 5 3 4 3 + 4 5
5 + 10 1 2
= +
25 5
2
=
5

2.
(1 )4 = (2)2 = 4

213
Solution 6.3
1. First we do the multiplication in Cartesian form.

10 2  8 1
  
1+ 3 = 1+ 3 1+ 3

 4 1
 
= 2 + 2 3 2 + 2 3

 2 1
 
= 2 + 2 3 8 8 3
  1
= 2 + 2 3 128 + 128 3
 1
= 512 512 3
1 1
=
512 1 + 3

1 1 1 3
=
512 1 + 3 1 3

1 3
= +
2048 2048

214
Now we do the multiplication in modulus-argument, (polar), form.
 10 10
1+ 3 = 2 e/3
= 210 e10/3
    
1 10 10
= cos + sin
1024 3 3
    
1 4 4
= cos sin
1024 3 3
!
1 1 3
= +
1024 2 2

1 3
= +
2048 2048

2.
(11 + 4)2 = 105 + 88

Solution 6.4
1.
 2  2
2+ 2+
=
6 (1 2) 1 + 8
3 + 4
=
63 16
3 + 4 63 + 16
=
63 16 63 + 16
253 204
=
4225 4225

215
2.

2
(1 )7 = (1 )2 (1 )2 (1 )
= (2)2 (2)(1 )
= (4)(2 2)
= 8 + 8

Solution 6.5
1.

   
z x + y
=
z x + y
 
x y
=
x + y
x + y
=
x y
x + y x + y
=
x y x + y
x2 y 2 2xy
= 2 +
x + y2 x2 + y 2

216
2.
z + 2 x + y + 2
=
2 z 2 (x y)
x + (y + 2)
=
2 y x
x + (y + 2) 2 y + x
=
2 y x 2 y + x
x(2 y) (y + 2)x x2 + (y + 2)(2 y)
= +
(2 y)2 + x2 (2 y)2 + x2
2xy 4 + x2 y 2
= +
(2 y)2 + x2 (2 y)2 + x2
Solution 6.6
Method 1. We expand the equation uv = w in its components.
uv = w
(u0 + u1 + u2 + ku3 ) (v0 + v1 + v2 + kv3 ) = w0 + w1 + w2 + kw3

(u0 v0 u1 v1 u2 v2 u3 v3 ) + (u1 v0 + u0 v1 u3 v2 + u2 v3 ) + (u2 v0 + u3 v1 + u0 v2 u1 v3 )


+ k (u3 v0 u2 v1 + u1 v2 + u0 v3 ) = w0 + w1 + w2 + kw3
We can write this as a matrix equation.

u0 u1 u2 u3 v0 w0
u1 u0 u3 u2 v1 w1
=
u 2 u 3 u0 u1 v2 w2
u3 u2 u1 u0 v3 w3
This linear system of equations has a unique solution for v if and only if the determinant of the matrix is nonzero. The
2
determinant of the matrix is (u20 + u21 + u22 + u23 ) . This is zero if and only if u0 = u1 = u2 = u3 = 0. Thus there

217
exists a unique v such that uv = w if u is nonzero. This v is
v = (u0 w0 + u1 w1 + u2 w2 + u3 w3 ) + (u1 w0 + u0 w1 + u3 w2 u2 w3 ) + (u2 w0 u3 w1 + u0 w2 + u1 w3 )
+ k (u3 w0 + u2 w1 u1 w2 + u0 w3 ) / u20 + u21 + u22 + u23
 

Method 2. Note that uu is a real number.


uu = (u0 u1 u2 ku3 ) (u0 + u1 + u2 + ku3 )
= u20 + u21 + u22 + u23 + (u0 u1 u1 u0 u2 u3 + u3 u2 )


+ (u0 u2 + u1 u3 u2 u0 u3 u1 ) + k (u0 u3 u1 u2 + u2 u1 u3 u0 )
= u20 + u21 + u22 + u23


uu = 0 only if u = 0. We solve for v by multiplying by the conjugate of u and dividing by uu.


uv = w
uuv = uw
uw
v=
uu
(u0 u1 u2 ku3 ) (w0 + w1 + w2 + kw3 )
v=
u20 + u21 + u22 + u23

v = (u0 w0 + u1 w1 + u2 w2 + u3 w3 ) + (u1 w0 + u0 w1 + u3 w2 u2 w3 ) + (u2 w0 u3 w1 + u0 w2 + u1 w3 )


+ k (u3 w0 + u2 w1 u1 w2 + u0 w3 ) / u20 + u21 + u22 + u23
 

Solution 6.7 
If = t, then = t||2 , which is a real number. Hence = = 0.

Now assume that = = 0. This implies that = r for some r R. We multiply by and simplify.
||2 = r
r
=
||2
r
By taking t = ||2
We see that = t for some real number t.

218
The Complex Plane
Solution 6.8
1.
 1/3
1/3
(1 + ) = 2 e/4

2 e/12 11/3
6
=

2 e/12 e2k/3 , k = 0, 1, 2
6
=
n 3/4 o
6 /12 6 6 17/12
= 2e , 2e , 2e
The principal root is
2 e/12 .
3 6
1+=
The roots are depicted in Figure 6.9.
2.
1/4
1/4 = e/2
= e/8 11/4
= e/8 e2k/4 , k = 0, 1, 2, 3
= e/8 , e5/8 , e9/8 , e13/8


The principal root is


4
= e/8 .
The roots are depicted in Figure 6.10.
Solution 6.9
1.
|<(z)| + 2|=(z)| 1
|x| + 2|y| 1

219
1

-1 1

-1

Figure 6.9: (1 + )1/3

In the first quadrant, this is the triangle below the line y = (1x)/2. We reflect this triangle across the coordinate
axes to obtain triangles in the other quadrants. Explicitly, we have the set of points: {z = x + y | 1 x
1 |y| (1 |x|)/2}. See Figure 6.11.

2. |z | is the distance from the point in the complex plane. Thus 1 < |z | < 2 is an annulus centered at
z = between the radii 1 and 2. See Figure 6.12.

3. The points which are closer to z = than z = are those points in the upper half plane. See Figure 6.13.

Solution 6.10
Let z = r e and = e .

220
1

-1 1

-1

Figure 6.10: 1/4

1.

arg(z) = arg(z) + arg()


arg r e(+) = { + 2m} + { + 2n}


{ + + 2k} = { + + 2m}

2.

Arg(z) 6= Arg(z) + Arg()

Consider z = = 1. Arg(z) = Arg() = , however Arg(z) = Arg(1) = 0. The identity becomes 0 6= 2.

221
1

1 1

Figure 6.11: |<(z)| + 2|=(z)| 1


4
3
2
1
-3 -2 -1 1 2 3
-1
-2

Figure 6.12: 1 < |z | < 2

222
1

1 1

Figure 6.13: The upper half plane.

3.

arg z 2 = arg(z) + arg(z) 6= 2 arg(z)




arg r2 e2 = { + 2k} + { + 2m} =



6 2{ + 2n}
{2 + 2k} = {2 + 2m} = 6 {2 + 4n}

Solution 6.11
Consider a triangle in the complex plane with vertices at 0, z and z + . (See Figure 6.14.)
The lengths of the sides of the triangle are |z|, || and |z + | The second inequality shows that one side of the
triangle must be less than or equal to the sum of the other two sides.

|z + | |z| + ||

The first inequality shows that the length of one side of the triangle must be greater than or equal to the difference in

223
z+
z ||

|z| |z+ |

Figure 6.14: Triangle inequality.

the length of the other two sides.


|z + | ||z| |||

Now we prove the inequalities algebraically. We will reduce the inequality to an identity. Let z = r e , = e .

||z| ||| |z + | |z| + ||


|r | |r e + e | r +
(r )2 r e + e r e + e (r + )2
 

r2 + 2 2r r2 + 2 + r e() +r e(+) r2 + 2 + 2r
2r 2r cos ( ) 2r
1 cos( ) 1

224
Solution 6.12
1.

1/4
(1)3/4 = (1)3
= (1)1/4
= (e )1/4
= e/4 11/4
= e/4 ek/2 , k = 0, 1, 2, 3
= e/4 , e3/4 , e5/4 , e7/4

 
1 + 1 + 1 1
= , , ,
2 2 2 2

See Figure 6.15.

2.


81/6 = 811/6
6


= 2 ek/3 , k = 0, 1, 2, 3, 4, 5
n o
= 2, 2 e/3 , 2 e2/3 , 2 e , 2 e4/3 , 2 e5/3
( )
1 + 3 1 + 3 1 3 1 3
= 2, , , 2, ,
2 2 2 2

See Figure 6.16.

225
1

-1 1

-1

Figure 6.15: (1)3/4

Solution 6.13
1.

(1)1/4 = ((1)1 )1/4


= (1)1/4
= (e )1/4
= e/4 11/4
= e/4 ek/2 , k = 0, 1, 2, 3
= e/4 , e3/4 , e5/4 , e7/4

 
1 + 1 + 1 1
= , , ,
2 2 2 2
See Figure 6.17.

226
2

-2 -1 1 2

-1

-2

Figure 6.16: 81/6

2.

161/8 = 1611/8
8


= 2 ek/4 , k = 0, 1, 2, 3, 4, 5, 6, 7
n o
= 2, 2 e/4 , 2 e/2 , 2 e3/4 , 2 e , 2 e5/4 , 2 e3/2 , 2 e7/4
n o
= 2, 1 + , 2, 1 + , 2, 1 , 2, 1

See Figure 6.18.

Solution 6.14
1. |z 2| is the distance from the point 2 in the complex plane. Thus 1 < |z 2| < 2 is an annulus. See
Figure 6.19.

227
1

-1 1

-1

Figure 6.17: (1)1/4

2.

|<(z)| + 5|=(z)| = 1
|x| + 5|y| = 1

In the first quadrant this is the line y = (1 x)/5. We reflect this line segment across the coordinate axes to
obtain line segments in the other quadrants. Explicitly, we have the set of points: {z = x + y | 1 < x <
1 y = (1 |x|)/5}. See Figure 6.20.

3. The set of points equidistant from and is the real axis. See Figure 6.21.

Solution 6.15
1. |z 1 + | is the distance from the point (1 ). Thus |z 1 + | 1 is the disk of unit radius centered at
(1 ). See Figure 6.22.

228
1

-1 1

-1

Figure 6.18: 161/8


5
4
3
2
1
-3 -2 -1 1 2 3
-1

Figure 6.19: 1 < |z 2| < 2

229
0.4

0.2

-1 1
-0.2

-0.4

Figure 6.20: |<(z)| + 5|=(z)| = 1

-1 1

-1

Figure 6.21: |z | = |z + |

230
1

-1 1 2 3

-1

-2

-3

Figure 6.22: |z 1 + | < 1

2.

<(z) =(z) = 5
xy =5
y =x5

See Figure 6.23.

3. Since |z | + |z + | 2, there are no solutions of |z | + |z + | = 1.

231
5

-10 -5 5 10

-5

-10

-15

Figure 6.23: <(z) =(z) = 5

Solution 6.16

| e 1| = 2
e 1 e 1 = 4
 

1 e e +1 = 4
2 cos() = 2
=


e | 0 is a unit semi-circle in the upper half of the complex plane from 1 to 1. The only point on this
semi-circle that is a distance 2 from the point 1 is the point 1, which corresponds to = .

Polar Form

232
Solution 6.17
We recall the Taylor series expansion of ex about x = 0.

X xn
ex = .
n=0
n!
We take this as the definition of the exponential function for complex argument.

X ()n
e =
n=0
n!
n
X n
=
n=0
n!

X (1)n 2n
X (1)n 2n+1
= +
n=0
(2n)! n=0
(2n + 1)!
We compare this expression to the Taylor series for the sine and cosine.

X (1)n 2n
X (1)n 2n+1
cos = , sin = ,
n=0
(2n)! n=0
(2n + 1)!

Thus e and cos + sin have the same Taylor series expansions about = 0.
e = cos + sin
Solution 6.18

cos(3) + sin(3) = (cos + sin )3


cos(3) + sin(3) = cos3 + 3 cos2 sin 3 cos sin2 sin3
We equate the real parts of the equation.
cos(3) = cos3 3 cos sin2

233
Solution 6.19
Define the partial sum,
n
X
Sn (z) = zk .
k=0

Now consider (1 z)Sn (z).

n
X
(1 z)Sn (z) = (1 z) zk
k=0
n
X n+1
X
(1 z)Sn (z) = zk zk
k=0 k=1
n+1
(1 z)Sn (z) = 1 z

We divide by 1 z. Note that 1 z is nonzero.

1 z n+1
Sn (z) =
1z
1 z n+1
1 + z + z2 + + zn = , (z 6= 1)
1z

Now consider z = e where 0 < < 2 so that z is not unity.

n n+1
X
k
 1 e
e =
k=0
1 e
n
X 1 e(n+1)
ek =
k=0
1 e

234
In order to get sin(/2) in the denominator, we multiply top and bottom by e/2 .
n
X e/2 e(n+1/2)
(cos(k) + sin(k)) =
e/2 e/2
k=0
n n
X X cos(/2) sin(/2) cos((n + 1/2)) sin((n + 1/2))
cos(k) + sin(k) =
k=0 k=0
2 sin(/2)
n n  
X X 1 sin((n + 1/2)) 1 cos((n + 1/2))
cos(k) + sin(k) = + + cot(/2)
k=0 k=1
2 sin(/2) 2 sin(/2)

1. We take the real and imaginary part of this to obtain the identities.
n
X 1 sin((n + 1/2))
cos(k) = +
k=0
2 2 sin(/2)

2.
n
X 1 cos((n + 1/2))
sin(k) = cot(/2)
k=1
2 2 sin(/2)

Arithmetic and Vectors


Solution 6.20

|z| = |r e e |
= |r e(+) |
= |r|
= |r|||
= |z|||

235

z r e
=
e

r ()
= e


r
=

|r|
=
||
|z|
=
||

Solution 6.21

|z + |2 + |z |2 = (z + ) z + + (z ) z
 

= zz + z + z + + zz z z +
= 2 |z|2 + ||2


Consider the parallelogram defined by the vectors z and . The lengths of the sides are z and and the lengths of
the diagonals are z + and z . We know from geometry that the sum of the squared lengths of the diagonals of a
parallelogram is equal to the sum of the squared lengths of the four sides. (See Figure 6.24.)

Integer Exponents

236
z-
z
z+

Figure 6.24: The parallelogram defined by z and .

Solution 6.22
1.
  2 2
(1 + )10 = (1 + )2 (1 + )2
2
= (2)2 (2)
= (4)2 (2)
= 16(2)
= 32

2.
 10
(1 + )10 = 2 e/4
 10
= 2 e10/4
= 32 e/2
= 32

237
Rational Exponents
Solution 6.23
We substitite the numbers into the equation to obtain an identity.

z 2 + 2az + b = 0
 1/2 2  1/2 
a + a2 b + 2a a + a2 b +b=0
1/2 1/2
a2 2a a2 b + a2 b 2a2 + 2a a2 b +b=0
0=0

238
Chapter 7

Functions of a Complex Variable

If brute force isnt working, youre not using enough of it.


-Tim Mauch

In this chapter we introduce the algebra of functions of a complex variable. We will cover the trigonometric and
inverse trigonometric functions. The properties of trigonometric functions carry over directly from real-variable theory.
However, because of multi-valuedness, the inverse trigonometric functions are significantly trickier than their real-variable
counterparts.

7.1 Curves and Regions


In this section we introduce curves and regions in the complex plane. This material is necessary for the study of
branch points in this chapter and later for contour integration.

Curves. Consider two continuous functions x(t) and y(t) defined on the interval t [t0 ..t1 ]. The set of points in
the complex plane,
{z(t) = x(t) + y(t) | t [t0 . . . t1 ]},

239
defines a continuous curve or simply a curve. If the endpoints coincide ( z (t0 ) = z (t1 ) ) it is a closed curve. (We
assume that t0 6= t1 .) If the curve does not intersect itself, then it is said to be a simple curve.
If x(t) and y(t) have continuous derivatives and the derivatives do not both vanish at any point, then it is a smooth
curve.1 This essentially means that the curve does not have any corners or other nastiness.
A continuous curve which is composed of a finite number of smooth curves is called a piecewise smooth curve. We
will use the word contour as a synonym for a piecewise smooth curve.
See Figure 7.1 for a smooth curve, a piecewise smooth curve, a simple closed curve and a non-simple closed curve.

(a) (b) (c) (d)

Figure 7.1: (a) Smooth curve. (b) Piecewise smooth curve. (c) Simple closed curve. (d) Non-simple closed curve.

Regions. A region R is connected if any two points in R can be connected by a curve which lies entirely in R. A
region is simply-connected if every closed curve in R can be continuously shrunk to a point without leaving R. A region
which is not simply-connected is said to be multiply-connected region. Another way of defining simply-connected is
that a path connecting two points in R can be continuously deformed into any other path that connects those points.
Figure 7.2 shows a simply-connected region with two paths which can be continuously deformed into one another and
two multiply-connected regions with paths which cannot be deformed into one another.

Jordan curve theorem. A continuous, simple, closed curve is known as a Jordan curve. The Jordan Curve
Theorem, which seems intuitively obvious but is difficult to prove, states that a Jordan curve divides the plane into
1
Why is it necessary that the derivatives do not both vanish?

240
Figure 7.2: A simply-connected and two multiply-connected regions.

a simply-connected, bounded region and an unbounded region. These two regions are called the interior and exterior
regions, respectively. The two regions share the curve as a boundary. Points in the interior are said to be inside the
curve; points in the exterior are said to be outside the curve.

Traversal of a contour. Consider a Jordan curve. If you traverse the curve in the positive direction, then the
inside is to your left. If you traverse the curve in the opposite direction, then the outside will be to your left and you
will go around the curve in the negative direction. For circles, the positive direction is the counter-clockwise direction.
The positive direction is consistent with the way angles are measured in a right-handed coordinate system, i.e. for a
circle centered on the origin, the positive direction is the direction of increasing angle. For an oriented contour C, we
denote the contour with opposite orientation as C.

Boundary of a region. Consider a simply-connected region. The boundary of the region is traversed in the positive
direction if the region is to the left as you walk along the contour. For multiply-connected regions, the boundary may
be a set of contours. In this case the boundary is traversed in the positive direction if each of the contours is traversed
in the positive direction. When we refer to the boundary of a region we will assume it is given the positive orientation.
In Figure 7.3 the boundaries of three regions are traversed in the positive direction.

241
Figure 7.3: Traversing the boundary in the positive direction.

Two interpretations of a curve. Consider a simple closed curve as depicted in Figure 7.4a. By giving it an
orientation, we can make a contour that either encloses the bounded domain Figure 7.4b or the unbounded domain
Figure 7.4c. Thus a curve has two interpretations. It can be thought of as enclosing either the points which are inside
or the points which are outside.2

7.2 The Point at Infinity and the Stereographic Projection


Complex infinity. In real variables, there are only two ways to get to infinity. We can either go up or down the
number line. Thus signed infinity makes sense. By going up or down we respectively approach + and . In the
complex plane there are an infinite number of ways to approach infinity. We stand at the origin, point ourselves in any
direction and go straight. We could walk along the positive real axis and approach infinity via positive real numbers.
We could walk along the positive imaginary axis and approach infinity via pure imaginary numbers. We could generalize
the real variable notion of signed infinity to a complex variable notion of directional infinity, but this will not be useful
2
A farmer wanted to know the most efficient way to build a pen to enclose his sheep, so he consulted an engineer, a physicist
and a mathematician. The engineer suggested that he build a circular pen to get the maximum area for any given perimeter. The
physicist suggested that he build a fence at infinity and then shrink it to fit the sheep. The mathematician constructed a little fence
around himself and then defined himself to be outside.

242
(a) (b) (c)

Figure 7.4: Two interpretations of a curve.

for our purposes. Instead, we introduce complex infinity or the point at infinity as the limit of going infinitely far along
any direction in the complex plane. The complex plane together with the point at infinity form the extended complex
plane.

Stereographic projection. We can visualize the point at infinity with the stereographic projection. We place a
unit sphere on top of the complex plane so that the south pole of the sphere is at the origin. Consider a line passing
through the north pole and a point z = x + y in the complex plane. In the stereographic projection, the point point z
is mapped to the point where the line intersects the sphere. (See Figure 7.5.) Each point z = x + y in the complex
plane is mapped to a unique point (X, Y, Z) on the sphere.

4x 4y 2|z|2
X= 2 , Y = 2 , Z= 2
|z| + 4 |z| + 4 |z| + 4

The origin is mapped to the south pole. The point at infinity, |z| = , is mapped to the north pole.
In the stereographic projection, circles in the complex plane are mapped to circles on the unit sphere. Figure 7.6
shows circles along the real and imaginary axes under the mapping. Lines in the complex plane are also mapped to
circles on the unit sphere. The right diagram in Figure 7.6 shows lines emanating from the origin under the mapping.

243
y

Figure 7.5: The stereographic projection.

244
Figure 7.6: The stereographic projection of circles and lines.

245
The stereographic projection helps us reason about the point at infinity. When we consider the complex plane by
itself, the point at infinity is an abstract notion. We cant draw a picture of the point at infinity. It may be hard to
accept the notion of a jordan curve enclosing the point at infinity. However, in the stereographic projection, the point
at infinity is just an ordinary point (namely the north pole of the sphere).

7.3 A Gentle Introduction to Branch Points


In this section we will introduce the concepts of branches, branch points and branch cuts. These concepts (which are
notoriously difficult to understand for beginners) are typically defined in terms functions of a complex variable. Here
we will develop these ideas as they relate to the arctangent function arctan(x, y). Hopefully this simple example will
make the treatment in Section 7.9 more palateable.
First we review some properties of the arctangent. It is a mapping from R2 to R. It measures the angle around the
origin from the positive x axis. Thus it is a multi-valued function. For a fixed point in the domain, the function values
differ by integer multiples of 2. The arctangent is not defined at the origin nor at the point at infinity; it is singular
at these two points. If we plot some of the values of the arctangent, it looks like a corkscrew with axis through the
origin. A portion of this function is plotted in Figure 7.7.
Most of the tools we have for analyzing functions (continuity, differentiability, etc.) depend on the fact that the
function is single-valued. In order to work with the arctangent we need to select a portion to obtain a single-valued
function. Consider the domain (1..2) (1..4). On this domain we select the value of the arctangent that is between
0 and . The domain and a plot of the selected values of the arctangent are shown in Figure 7.8.
CONTINUE.

7.4 Cartesian and Modulus-Argument Form


We can write a function of a complex variable z as a function of x and y or as a function of r and with the substitutions
z = x + y and z = r e , respectively. Then we can separate the real and imaginary components or write the function
in modulus-argument form,

f (z) = u(x, y) + v(x, y), or f (z) = u(r, ) + v(r, ),

246
5
2
0
-5 1

-2 0 y
-1
0 -1
x 1
2 -2

Figure 7.7: Plots of <(log z) and a portion of =(log z).

5
4
3
2 2
1.5 6
1 1
0.5 4
0
-3 -2 -1 1 2 3 4 5 -2 2
-1 0
2 0
-2 4
-3

Figure 7.8: A domain and a selected value of the arctangent for the points in the domain.

247
f (z) = (x, y) e(x,y) , or f (z) = (r, ) e(r,) .
1
Example 7.4.1 Consider the functions f (z) = z, f (z) = z 3 and f (z) = 1z
. We write the functions in terms of x
and y and separate them into their real and imaginary components.

f (z) = z
= x + y

f (z) = z 3
= (x + y)3
= x3 + x2 y xy 2 y 3
= x3 xy 2 + x2 y y 3
 

1
f (z) =
1z
1
=
1 x y
1 1 x + y
=
1 x y 1 x + y
1x y
= 2 2
+
(1 x) + y (1 x)2 + y 2
1
Example 7.4.2 Consider the functions f (z) = z, f (z) = z 3 and f (z) = 1z
. We write the functions in terms of r
and and write them in modulus-argument form.

f (z) = z
= r e

248
f (z) = z 3
3
= r e
= r3 e3

1
f (z) =
1z
1
=
1 r e
1 1
=
1 r e 1 r e

1 r e
=
1 r e r e +r2
1 r cos + r sin
=
1 2r cos + r2
Note that the denominator is real and non-negative.
1
= 2
|1 r cos + r sin | e arctan(1r cos ,r sin )
1 2r cos + r q
1
= (1 r cos )2 + r2 sin2 e arctan(1r cos ,r sin )
1 2r cos + r2
1 p
= 1 2r cos + r2 cos2 + r2 sin2 e arctan(1r cos ,r sin )
1 2r cos + r2
1
= e arctan(1r cos ,r sin )
1 2r cos + r 2

7.5 Graphing Functions of a Complex Variable


We cannot directly graph functions of a complex variable as they are mappings from R2 to R2 . To do so would require
four dimensions. However, we can can use a surface plot to graph the real part, the imaginary part, the modulus or the

249
argument of a function of a complex variable. Each of these are scalar fields, mappings from R2 to R.

Example 7.5.1 Consider the identity function, f (z) = z. In Cartesian coordinates and Cartesian form, the function
is f (z) = x + y. The real and imaginary components are u(x, y) = x and v(x, y) = y. (See Figure 7.9.) In modulus

2 2
1 2 1 2
0 0
-1 1 -1 1
-2 -2
-2 0 y -2 0 y
-1 -1
-1 -1
x0 1 x0 1
2-2 2-2

Figure 7.9: The real and imaginary parts of f (z) = z = x + y.

argument form the function is p


f (z) = z = r e = x2 + y 2 e arctan(x,y) .
The modulus of f (z) is a single-valued function which is the distance from the origin. The argument of f (z) is a multi-
valued function. Recall that arctan(x, y) has an infinite number of values each of which differ by an integer multiple
of 2. A few branches of arg(f (z)) are plotted in Figure 7.10. The modulus and principal argument of f (z) = z are
plotted in Figure 7.11.

Example 7.5.2 Consider the function f (z) = z 2 . In Cartesian coordinates and separated into its real and imaginary
components the function is
f (z) = z 2 = (x + y)2 = x2 y 2 + 2xy.


Figure 7.12 shows surface plots of the real and imaginary parts of z 2 . The magnitude of z 2 is
p
|z 2 | = z 2 z 2 = zz = (x + y)(x y) = x2 + y 2 .

250
y 12
0
-1
-2
5
0
-5
-2
-1
0
x 1
2

Figure 7.10: A few branches of arg(z).

2 2 2 2
1 0
0 1 -2 1
-2 0y -2 0y
-1 -1 -1 -1
0 0
x 1 x 1
2-2 2 -2

Figure 7.11: Plots of |z| and Arg(z).

251
4
2 5
2 2
0 0
-2 1 -5 1
-4
-2 0 y -2 0 y
-1 -1
0 -1 0 -1
x 1 x 1
2 -2 2 -2

Figure 7.12: Plots of < (z 2 ) and = (z 2 ).

Note that 2
z 2 = r e = r2 e2 .
In Figure 7.13 are plots of |z 2 | and a branch of arg (z 2 ).

7.6 Trigonometric Functions


The exponential function. Consider the exponential function ez . We can use Eulers formula to write ez = ex+y
in terms of its real and imaginary parts.
ez = ex+y = ex ey = ex cos y + ex sin y
From this we see that the exponential function is 2 periodic: ez+2 = ez , and odd periodic: ez+ = ez .
Figure 7.14 has surface plots of the real and imaginary parts of ez which show this periodicity.
The modulus of ez is a function of x alone.

|ez | = ex+y = ex

252
8 5
6 2 2
4 0
2 1 1
0 -5
-2 0 y -2 0 y
-1 -1
0 -1 0 -1
x 1 x 1
2 -2 2 -2

Figure 7.13: Plots of |z 2 | and a branch of arg (z 2 ).

20 20
10 10
0 5 0 5
-10 -10
-20 -20
0 y 0 y
-2 -2
0 0
x -5 x -5
2 2

Figure 7.14: Plots of < (ez ) and = (ez ).

253
The argument of ez is a function of y alone.

arg (ez ) = arg ex+y = {y + 2n | n Z}




In Figure 7.15 are plots of | ez | and a branch of arg (ez ).

20 5
15 5 0 5
10
5 -5
0
0 y 0 y
-2 -2
x0 -5 x0 -5
2 2

Figure 7.15: Plots of | ez | and a branch of arg (ez ).

Example 7.6.1 Show that the transformation w = ez maps the infinite strip, < x < , 0 < y < , onto the
upper half-plane.
Method 1. Consider the line z = x + c, < x < . Under the transformation, this is mapped to

w = ex+c = ec ex , < x < .

This is a ray from the origin to infinity in the direction of ec . Thus we see that z = x is mapped to the positive, real
w axis, z = x + is mapped to the negative, real axis, and z = x + c, 0 < c < is mapped to a ray with angle c in
the upper half-plane. Thus the strip is mapped to the upper half-plane. See Figure 7.16.
Method 2. Consider the line z = c + y, 0 < y < . Under the transformation, this is mapped to

w = ec+y + ec ey , 0 < y < .

254
3 3
2 2
1 1

-3 -2 -1 1 2 3 -3 -2 -1 1 2 3

Figure 7.16: ez maps horizontal lines to rays.

This is a semi-circle in the upper half-plane of radius ec . As c , the radius goes to zero. As c , the radius
goes to infinity. Thus the strip is mapped to the upper half-plane. See Figure 7.17.

3 3
2 2
1 1

-1 1 -3 -2 -1 1 2 3

Figure 7.17: ez maps vertical lines to circular arcs.

255
The sine and cosine. We can write the sine and cosine in terms of the exponential function.

ez + ez cos(z) + sin(z) + cos(z) + sin(z)


=
2 2
cos(z) + sin(z) + cos(z) sin(z)
=
2
= cos z

ez ez cos(z) + sin(z) cos(z) sin(z)


=
2 2
cos(z) + sin(z) cos(z) + sin(z)
=
2
= sin z

We separate the sine and cosine into their real and imaginary parts.

cos z = cos x cosh y sin x sinh y sin z = sin x cosh y + cos x sinh y

For fixed y, the sine and cosine are oscillatory in x. The amplitude of the oscillations grows with increasing |y|. See
Figure 7.18 and Figure 7.19 for plots of the real and imaginary parts of the cosine and sine, respectively. Figure 7.20
shows the modulus of the cosine and the sine.

The hyperbolic sine and cosine. The hyperbolic sine and cosine have the familiar definitions in terms of the
exponential function. Thus not surprisingly, we can write the sine in terms of the hyperbolic sine and write the cosine
in terms of the hyperbolic cosine. Below is a collection of trigonometric identities.

256
5 5
2.5 2.5
0 2 0 2
-2.5 1 -2.5 1
-5 -5
0 y 0 y
-2 -2
0 -1 0 -1
x x
2 -2 2 -2

Figure 7.18: Plots of <(cos(z)) and =(cos(z)).

5 5
2.5 2.5
0 2 0 2
-2.5 1 -2.5 1
-5 -5
0 y 0 y
-2 -2
0 -1 0 -1
x x
2 -2 2 -2

Figure 7.19: Plots of <(sin(z)) and =(sin(z)).

257
4 4
2 2
2 2
1 1
0
0 y 0 y
-2 -2
0 -1 0 -1
x x
2 -2 2 -2

Figure 7.20: Plots of | cos(z)| and | sin(z)|.

Result 7.6.1

ez = ex (cos y + sin y)
ez + ez ez ez
cos z = sin z =
2 2
cos z = cos x cosh y sin x sinh y sin z = sin x cosh y + cos x sinh y
ez + ez ez ez
cosh z = sinh z =
2 2
cosh z = cosh x cos y + sinh x sin y sinh z = sinh x cos y + cosh x sin y
sin(z) = sinh z sinh(z) = sin z
cos(z) = cosh z cosh(z) = cos z
log z = ln |z| + arg(z) = ln |z| + Arg(z) + 2n, n Z

258
7.7 Inverse Trigonometric Functions
The logarithm. The logarithm, log(z), is defined as the inverse of the exponential function ez . The exponential
function is many-to-one and thus has a multi-valued inverse. From what we know of many-to-one functions, we conclude
that
elog z = z, but log (ez ) 6= z.
This is because elog z is single-valued but log (ez ) is not. Because ez is 2 periodic, the logarithm of a number is a set
of numbers which differ by integer multiples of 2. For instance, e2n = 1 so that log(1) = {2n : n Z}. The
logarithmic function has an infinite number of branches. The value of the function on the branches differs by integer
multiples of 2. It has singularities at zero and infinity. | log(z)| as either z 0 or z .
We will derive the formula for the complex variable logarithm. For now, let ln(x) denote the real variable logarithm
that is defined for positive real numbers. Consider w = log z. This means that ew = z. We write w = u + v in
Cartesian form and z = r e in polar form.
eu+v = r e
We equate the modulus and argument of this expression.

eu = r v = + 2n
u = ln r v = + 2n

With log z = u + v, we have a formula for the logarithm.

log z = ln |z| + arg(z)

If we write out the multi-valuedness of the argument function we note that this has the form that we expected.

log z = ln |z| + (Arg(z) + 2n), nZ

We check that our formula is correct by showing that elog z = z

elog z = eln |z|+ arg(z) = eln r++2n = r e = z

259
Note again that log (ez ) 6= z.

log (ez ) = ln | ez | + arg (ez ) = ln (ex ) + arg ex+y = x + (y + 2n) = z + 2n 6= z




The real part of the logarithm is the single-valued ln r; the imaginary part is the multi-valued arg(z). We define the
principal branch of the logarithm Log z to be the branch that satisfies < =(Log z) . For positive, real numbers
the principal branch, Log x is real-valued. We can write Log z in terms of the principal argument, Arg z.

Log z = ln |z| + Arg(z)

See Figure 7.21 for plots of the real and imaginary part of Log z.

1
2
0 2 2
0
-1 1 1
-2
-2
-2 0 y -2 0 y
-1 -1
0 -1 0 -1
x 1 x 1
2 -2 2 -2

Figure 7.21: Plots of <(Log z) and =(Log z).

The form: ab . Consider ab where a and b are complex and a is nonzero. We define this expression in terms of the
exponential and the logarithm as
ab = eb log a .

260
Note that the multi-valuedness of the logarithm may make ab multi-valued. First consider the case that the exponent
is an integer.
am = em log a = em(Log a+2n) = em Log a e2mn = em Log a
Thus we see that am has a single value where m is an integer.
Now consider the case that the exponent is a rational number. Let p/q be a rational number in reduced form.
p p p
ap/q = e q log a = e q (Log a+2n) = e q Log a e2np/q .

This expression has q distinct values as

e2np/q = e2mp/q if and only if n = m mod q.

Finally consider the case that the exponent b is an irrational number.

ab = eb log a = eb(Log a+2n) = eb Log a e2bn

Note that e2bn and e2bm are equal if and only if 2bn and 2bm differ by an integer multiple of 2, which means
that bn and bm differ by an integer. This occurs only when n = m. Thus e2bn has a distinct value for each different
integer n. We conclude that ab has an infinite number of values.
You may have noticed something a little fishy. If b is not an integer and a is any non-zero complex number, then
a is multi-valued. Then why have we been treating eb as single-valued, when it is merely the case a = e? The answer
b

is that in the realm of functions of a complex variable, ez is an abuse of notation. We write ez when we mean exp(z),
the single-valued exponential function. Thus when we write ez we do not mean the number e raised to the z power,
we mean the exponential function of z. We denote the former scenario as (e)z , which is multi-valued.

Logarithmic identities. Back in high school trigonometry when you thought that the logarithm was only defined
for positive real numbers you learned the identity log xa = a log x. This identity doesnt hold when the logarithm is
defined for nonzero complex numbers. Consider the logarithm of z a .

log z a = Log z a + 2n

261
a log z = a(Log z + 2n) = a Log z + 2an
Note that
log z a 6= a log z
Furthermore, since
Log z a = ln |z a | + Arg (z a ) , a Log z = a ln |z| + a Arg(z)
a
and Arg (z ) is not necessarily the same as a Arg(z) we see that
Log z a 6= a Log z.

Consider the logarithm of a product.


log(ab) = ln |ab| + arg(ab)
= ln |a| + ln |b| + arg(a) + arg(b)
= log a + log b
There is not an analogous identity for the principal branch of the logarithm since Arg(ab) is not in general the same as
Arg(a) + Arg(b). Pn
Using log(ab) = log(a) + log(b) we can deduce that log (an ) = k=1 log a = n log a, where n is a positive
integer. This result is simple, straightforward and wrong. I have led you down the merry path to damnation.3 In fact,
log (a2 ) 6= 2 log a. Just write the multi-valuedness explicitly,
log a2 = Log a2 + 2n,
 
2 log a = 2(Log a + 2n) = 2 Log a + 4n.

You can verify that  


1
log = log a.
a
We can use this and the product identity to expand the logarithm of a quotient.
a
log = log a log b
b
3
Dont feel bad if you fell for it. The logarithm is a tricky bastard.

262
For general values of a, log z a 6= a log z. However, for some values of a, equality holds. We already know that a = 1
and a = 1 work. To determine if equality holds for other values of a, we explicitly write the multi-valuedness.

log z a = log ea log z = a log z + 2k, k Z




a log z = a ln |z| + a Arg z + a2m, m Z

We see that log z a = a log z if and only if

{am | m Z} = {am + k | k, m Z}.

The sets are equal if and only if a = 1/n, n Z . Thus we have the identity:

 1
log z 1/n = log z, n Z
n

263
Result 7.7.1 Logarithmic Identities.

ab = eb log a
elog z = eLog z = z
log(ab) = log a + log b
log(1/a) = log a
log(a/b) = log a log b
  1
log z 1/n
= log z, n Z
n
Logarithmic Inequalities.

Log(uv) 6= Log(u) + Log(v)


log z a 6= a log z
Log z a 6= a Log z
log ez 6= z

Example 7.7.1 Consider 1 . We apply the definition ab = eb log a .


1 = e log(1)
= e(ln(1)+2n)
2
= e2n
Thus we see that 1 has an infinite number of values, all of which lie on the unit circle |z| = 1 in the complex plane.
However, the set 1 is not equal to the set |z| = 1. There are points in the latter which are not in the former. This is
analogous to the fact that the rational numbers are dense in the real numbers, but are a subset of the real numbers.

264
Example 7.7.2 We find the zeros of sin z.

ez ez
sin z = =0
2
ez = ez
e2z = 1
2z mod 2 = 0
z = n, nZ

Equivalently, we could use the identity

sin z = sin x cosh y + cos x sinh y = 0.

This becomes the two equations (for the real and imaginary parts)

sin x cosh y = 0 and cos x sinh y = 0.

Since cosh is real-valued and positive for real argument, the first equation dictates that x = n, n Z. Since
cos(n) = (1)n for n Z, the second equation implies that sinh y = 0. For real argument, sinh y is only zero at
y = 0. Thus the zeros are

z = n, nZ

Example 7.7.3 Since we can express sin z in terms of the exponential function, one would expect that we could express

265
the sin1 z in terms of the logarithm.
w = sin1 z
z = sin w
ew ew
z=
2
2w w
e 2z e 1 = 0

ew = z 1 z 2
 
w = log z 1 z 2

Thus we see how the multi-valued sin1 is related to the logarithm.


 
sin1 z = log z 1 z 2

Example 7.7.4 Consider the equation sin3 z = 1.


sin3 z = 1
sin z = 11/3
ez ez
= 11/3
2
ez 2(1)1/3 ez = 0
e2z 2(1)1/3 ez 1 = 0
p
z 2(1)1/3 4(1)2/3 + 4
e =
q2
ez = (1)1/3 1 (1)2/3
 p 
z = log (1)1/3 1 12/3

266
Note that there are three sources of multi-valuedness in the expression for z. The two values of the square root are
shown explicitly. There are three cube roots of unity. Finally, the logarithm has an infinite number of branches. To
show this multi-valuedness explicitly, we could write
 p 
2m/3 4m/3
z = Log e 1e + 2n, m = 0, 1, 2, n = . . . , 1, 0, 1, . . .

Example 7.7.5 Consider the harmless looking equation, z = 1.


Before we start with the algebra, note that the right side of the equation is a single number. z is single-valued only
when z is an integer. Thus we know that if there are solutions for z, they are integers. We now proceed to solve the
equation.
z = 1
z
e/2 = 1

Use the fact that z is an integer.

ez/2 = 1
z/2 = 2n, for some n Z
z = 4n, nZ
Here is a different approach. We write down the multi-valued form of z . We solve the equation by requiring that
all the values of z are 1.
z = 1
ez log = 1
z log = 2n, for some n Z
 
z + 2m = 2n, m Z, for some n Z
2

z + 2mz = 2n, m Z, for some n Z
2

267
The only solutions that satisfy the above equation are

z = 4k, k Z.

Now lets consider a slightly different problem: 1 z . For what values of z does z have 1 as one of its values.

1 z
1 ez log
1 {ez(/2+2n) | n Z}
z(/2 + 2n) = 2m, m, n Z
4m
z= , m, n Z
1 + 4n

There are an infinite set of rational numbers for which z has 1 as one of its values. For example,

4/5 = 11/5 = 1, e2/5 , e4/5 , e6/5 , e8/5




7.8 Riemann Surfaces


Consider the mapping w = log(z). Each nonzero point in the z-plane is mapped to an infinite number of points in
the w plane.
w = {ln |z| + arg(z)} = {ln |z| + (Arg(z) + 2n) | n Z}
This multi-valuedness makes it hard to work with the logarithm. We would like to select one of the branches of the
logarithm. One way of doing this is to decompose the z-plane into an infinite number of sheets. The sheets lie above
one another and are labeled with the integers, n Z. (See Figure 7.22.) We label the point z on the nth sheet as
(z, n). Now each point (z, n) maps to a single point in the w-plane. For instance, we can make the zeroth sheet map
to the principal branch of the logarithm. This would give us the following mapping.

log(z, n) = Log z + 2n

268
2
1
0
-1
-2

Figure 7.22: The z-plane decomposed into flat sheets.

This is a nice idea, but it has some problems. The mappings are not continuous. Consider the mapping on the
zeroth sheet. As we approach the negative real axis from above z is mapped to ln |z| + as we approach from below
it is mapped to ln |z| . (Recall Figure 7.21.) The mapping is not continuous across the negative real axis.
Lets go back to the regular z-plane for a moment. We start at the point z = 1 and selecting the branch of the
logarithm that maps to zero. (log(1) = 2n). We make the logarithm vary continuously as we walk around the origin
once in the positive direction and return to the point z = 1. Since the argument of z has increased by 2, the value
of the logarithm has changed to 2. If we walk around the origin again we will have log(1) = 4. Our flat sheet
decomposition of the z-plane does not reflect this property. We need a decomposition with a geometry that makes the
mapping continuous and connects the various branches of the logarithm.
Drawing inspiration from the plot of arg(z), Figure 7.10, we decompose the z-plane into an infinite corkscrew with
axis at the origin. (See Figure 7.23.) We define the mapping so that the logarithm varies continuously on this surface.
Consider a point z on one of the sheets. The value of the logarithm at that same point on the sheet directly above it
is 2 more than the original value. We call this surface, the Riemann surface for the logarithm. The mapping from
the Riemann surface to the w-plane is continuous and one-to-one.

269
Figure 7.23: The Riemann surface for the logarithm.

7.9 Branch Points


Example 7.9.1 Consider the function z 1/2 . For each value of z, there are two values of z 1/2 . We write z 1/2 in
modulus-argument and Cartesian form.
p
z 1/2 = |z| e arg(z)/2
p p
z 1/2 = |z| cos(arg(z)/2) + |z| sin(arg(z)/2)

Figure 7.24 shows the real and imaginary parts of z 1/2 from three different
 viewpoints. The second and third views are
1/2
looking down the x axis and y axis, respectively. Consider < z . This is a double layered sheet which intersects
1/2
itself on the negative real axis. (=(z ) has a similar structure, but intersects itself on the positive real axis.) Lets
start at a point on the positive real axis on the lower sheet. If we walk around the origin once and return to the positive
real axis, we will be on the upper sheet. If we do this again, we will return to the lower sheet.
Suppose we are at a point in the complex plane. We pick one of the two values of z 1/2 . If the function varies
continuously as we walk around the origin and back to our starting point, the value of z 1/2 will have changed. We will

270
be on the other branch. Because walking around the point z = 0 takes us to a different branch of the function, we
refer to z = 0 as a branch point.
Now consider the modulus-argument form of z 1/2 :
p
z 1/2 = |z| e arg(z)/2 .

Figure 7.25 shows the modulus and the principal argument of z 1/2 . We see that each time we walk around the origin,
the argument of z 1/2 changes by . This means that the value of the function changes by the factor e = 1, i.e.
the function changes sign. If we walk around the origin twice, the argument changes by 2, so that the value of the
function does not change, e2 = 1.
1/2
z 1/2 is a continuous function except at z = 0. Suppose we start at z = 1 = e0 and the function value (e0 ) = 1.
If we follow the first path in Figure 7.26, the argument of z varies from up to about 4 , down to about 4 and back
1/2
to 0. The value of the function is still (e0 ) .
Now suppose we follow a circular path around the origin in the positive, counter-clockwise, direction. (See the
second path in Figure 7.26.) The argument of z increases by 2. The value of the function at half turns on the path is
1/2
e0 = 1,
1/2
(e ) = e/2 = ,
1/2
e2 = e = 1

As we return to the point z = 1, the argument of the function has changed by and the value of the function has
changed from 1 to 1. If we were to walk along the circular path again, the argument of z would increase by another
2. The argument of the function would increase by another and the value of the function would return to 1.
1/2
e4 = e2 = 1

In general, any time we walk around the origin, the value of z 1/2 changes by the factor 1. We call z = 0 a branch
point. If we want a single-valued square root, we need something to prevent us from walking around the origin. We
achieve this by introducing a branch cut. Suppose we have the complex plane drawn on an infinite sheet of paper.
With a scissors we cut the paper from the origin to along the real axis. Then if we start at z = e0 , and draw a

271
1 1
2 2
0 0
-1 1 -1 1
-2 0 y -2 0 y
-1 -1
0 -1 0 -1
x 1 x 1
2 -2 2 -2

-20121
-1 0121
-1
-2
x x
0 0

-1 -1
-2 -1 0 1 2 -2 -1 0 1 2
y y

1210-1
-2 1210-1
-2
y y
0 0

-1 -1
2 1 0 -1 -2 2 1 0 -1 -2
x x

 
Figure 7.24: Plots of < z 1/2 (left) and = z 1/2 (right) from three viewpoints.

272
1 2 2 2
0.5 0
0 1 -2 1
-2-1 0y -2 0y
0 -1 -1 -1
x 1 0
2 -2 x 1
2 -2


Figure 7.25: Plots of |z 1/2 | and Arg z 1/2 .

Im(z) Im(z)

Re(z) Re(z)

Figure 7.26: A path that does not encircle the origin and a path around the origin.

273
continuous line without
 leaving the paper, the argument of z will always be in the range < arg z < . This means
1/2
that 2 < arg z < 2 . No matter what path we follow in this cut plane, z = 1 has argument zero and (1)1/2 = 1.
By never crossing the negative real axis, we have constructed a single valued branch of the square root function. We
call the cut along the negative real axis a branch cut.

Example 7.9.2 Consider the logarithmic function log z. For each value of z, there are an infinite number of values of
log z. We write log z in Cartesian form.
log z = ln |z| + arg z
Figure 7.27 shows the real and imaginary parts of the logarithm. The real part is single-valued. The imaginary part is
multi-valued and has an infinite number of branches. The values of the logarithm form an infinite-layered sheet. If we
start on one of the sheets and walk around the origin once in the positive direction, then the value of the logarithm
increases by 2 and we move to the next branch. z = 0 is a branch point of the logarithm.

1
0 2 5 2
-1 0
1 -5 1
-2
-2 0 y -2 0 y
-1 -1
0 -1 0 -1
x 1 x 1
2-2 2 -2

Figure 7.27: Plots of <(log z) and a portion of =(log z).

The logarithm is a continuous function except at z = 0. Suppose we start at z = 1 = e0 and the function value
log (e0 ) = ln(1) + 0 = 0. If we follow the first path in Figure 7.26, the argument of z and thus the imaginary part of
the logarithm varies from up to about 4 , down to about 4 and back to 0. The value of the logarithm is still 0.

274
Now suppose we follow a circular path around the origin in the positive direction. (See the second path in Fig-
ure 7.26.) The argument of z increases by 2. The value of the logarithm at half turns on the path is
log e0 = 0,


log (e ) = ,
log e2 = 2


As we return to the point z = 1, the value of the logarithm has changed by 2. If we were to walk along the circular
path again, the argument of z would increase by another 2 and the value of the logarithm would increase by another
2.

Result 7.9.1 A point z0 is a branch point of a function f (z) if the function changes value
when you walk around the point on any path that encloses no singularities other than the one
at z = z0 .

Branch points at infinity : mapping infinity to the origin. Up to this point we have considered only branch
points in the finite plane. Now we consider the possibility of a branch point at infinity. As a first method of approaching
this problem we map the point at infinity to the origin with the transformation = 1/z and examine the point = 0.

Example 7.9.3 Again consider the function z 1/2 . Mapping the point at infinity to the origin, we have f () =
(1/)1/2 = 1/2 . For each value of , there are two values of 1/2 . We write 1/2 in modulus-argument form.
1
1/2 = p e arg()/2
||
Like z 1/2 , 1/2 has a double-layered sheet of values. Figure 7.28 shows the modulus and the principal argument of
1/2 . We see that each time we walk around the origin, the argument of 1/2 changes by . This means that the
value of the function changes by the factor e = 1, i.e. the function changes sign. If we walk around the origin
twice, the argument changes by 2, so that the value of the function does not change, e2 = 1.
Since 1/2 has a branch point at zero, we conclude that z 1/2 has a branch point at infinity.

275
3
2.5 2
2 2 0 2
1.5 1 1
1 -2
-2 0 y -2 0 y
-1 -1
0 -1 0 -1
x 1 x 1
2 -2 2 -2

Figure 7.28: Plots of | 1/2 | and Arg 1/2 .




Example 7.9.4 Again consider the logarithmic function log z. Mapping the point at infinity to the origin, we have
f () = log(1/) = log(). From Example 7.9.2 we known that log() has a branch point at = 0. Thus log z
has a branch point at infinity.

Branch points at infinity : paths around infinity. We can also check for a branch point at infinity by following
a path that encloses the point at infinity and no other singularities. Just draw a simple closed curve that separates the
complex plane into a bounded component that contains all the singularities of the function in the finite plane. Then,
depending on orientation, the curve is a contour enclosing all the finite singularities, or the point at infinity and no
other singularities.

Example 7.9.5 Once again consider the function z 1/2 . We know that the function changes value on a curve that goes
once around the origin. Such a curve can be considered to be either a path around the origin or a path around infinity.
In either case the path encloses one singularity. There are branch points at the origin and at infinity. Now consider a
curve that does not go around the origin. Such a curve can be considered to be either a path around neither of the
branch points or both of them. Thus we see that z 1/2 does not change value when we follow a path that encloses
neither or both of its branch points.

276
1/2
Example 7.9.6 Consider f (z) = (z 2 1) . We factor the function.

f (z) = (z 1)1/2 (z + 1)1/2

There are branch points at z = 1. Now consider the point at infinity.


1/2 1/2
f 1 = 2 1 = 1 1 2


Since f ( 1 ) does not have a branch point at = 0, f (z) does not have a branch point at infinity. We could reach
the same conclusion by considering a path around infinity. Consider a path that circles the branch points at z = 1
once in the positive direction. Such a path circles the point at infinity once in the negative direction. In traversing this
1/2 1/2
path, the value of f (z) is multiplied by the factor (e2 ) (e2 ) = e2 = 1. Thus the value of the function does
not change. There is no branch point at infinity.

Diagnosing branch points. We have the definition of a branch point, but we do not have a convenient criterion
for determining if a particular function has a branch point. We have seen that log z and z for non-integer have
branch points at zero and infinity. The inverse trigonometric functions like the arcsine also have branch points, but they
can be written in terms of the logarithm and the square root. In fact all the elementary functions with branch points
can be written in terms of the functions log z and z . Furthermore, note that the multi-valuedness of z comes from
the logarithm, z = e log z . This gives us a way of quickly determining if and where a function may have branch points.

Result 7.9.2 Let f (z) be a single-valued function. Then log(f (z)) and (f (z)) may have
branch points only where f (z) is zero or singular.

Example 7.9.7 Consider the functions,


1/2
1. (z 2 )
2
2. z 1/2
3
3. z 1/2

277
Are they multi-valued? Do they have branch points?

1. 1/2
z2 = z 2 = z
Because of the ()1/2 , the function is multi-valued. The only possible branch points are at zero and infinity. If
 1/2  1/2
2 2 1/2
(e0 ) = 1, then (e2 ) = (e4 ) = e2 = 1. Thus we see that the function does not change
value when we walk around the origin. We can also consider this to be a path around infinity. This function is
multi-valued, but has no branch points.

2. 2 2
z 1/2 = z =z
This function is single-valued.

3. 3 3 3
z 1/2 = z = z
 3
1/2
This function is multi-valued. We consider the possible branch point at z = 0. If (e0 ) = 1, then
 3
1/2
(e2 ) = (e )3 = e3 = 1. Since the function changes value when we walk around the origin, it has a
branch point at z = 0. Since this is also a path around infinity, there is a branch point there.
1 1

Example 7.9.8 Consider the function f (z) = log z1 . Since z1 is only zero at infinity and its only singularity is at
z = 1, the only possibilities for branch points are at z = 1 and z = . Since
 
1
log = log(z 1)
z1

and log w has branch points at zero and infinity, we see that f (z) has branch points at z = 1 and z = .

Example 7.9.9 Consider the functions,

278
1. elog z
2. log ez .
Are they multi-valued? Do they have branch points?
1.
elog z = exp(Log z + 2n) = eLog z e2n = z
This function is single-valued.
2.
log ez = Log ez +2n = z + 2m
This function is multi-valued. It may have branch points only where ez is zero or infinite. This only occurs at
z = . Thus there are no branch points in the finite plane. The function does not change when traversing a
simple closed path. Since this path can be considered to enclose infinity, there is no branch point at infinity.
Consider (f (z)) where f (z) is single-valued and f (z) has either a zero or a singularity at z = z0 . (f (z)) may
have a branch point at z = z0 . If f (z) is not a power of z, then it may be difficult to tell if (f (z)) changes value when
we walk around z0 . Factor f (z) into f (z) = g(z)h(z) where h(z) is nonzero and finite at z0 . Then g(z) captures the
important behavior of f (z) at the z0 . g(z) tells us how fast f (z) vanishes or blows up. Since (f (z)) = (g(z)) (h(z))
and (h(z)) does not have a branch point at z0 , (f (z)) has a branch point at z0 if and only if (g(z)) has a branch
point there.
Similarly, we can decompose
log(f (z)) = log(g(z)h(z)) = log(g(z)) + log(h(z))
to see that log(f (z)) has a branch point at z0 if and only if log(g(z)) has a branch point there.

Result 7.9.3 Consider a single-valued function f (z) that has either a zero or a singularity at
z = z0 . Let f (z) = g(z)h(z) where h(z) is nonzero and finite. (f (z)) has a branch point
at z = z0 if and only if (g(z)) has a branch point there. log(f (z)) has a branch point at
z = z0 if and only if log(g(z)) has a branch point there.

279
Example 7.9.10 Consider the functions,

1. sin z 1/2

2. (sin z)1/2

3. z 1/2 sin z 1/2


1/2
4. (sin z 2 )

Find the branch points and the number of branches.

1. 
sin z 1/2 = sin z = sin z
sin z 1/2 is multi-valued. It has two branches. There may be branch points at zero and infinity.
 Consider
 the unit
0 1/2 2 1/2
circle which is a path around the origin or infinity. If sin (e ) = sin(1), then sin (e ) = sin (e ) =
sin(1) = sin(1). There are branch points at the origin and infinity.

2.
(sin z)1/2 = sin z
The function is multi-valued with two branches. The sine vanishes at z = n and is singular at infinity. There
could be branch points at these locations. Consider the point z = n. We can write
sin z
sin z = (z n)
z n
sin z
Note that zn
is nonzero and has a removable singularity at z = n.

sin z cos z
lim = lim = (1)n
zn z n zn 1
Since (z n)1/2 has a branch point at z = n, (sin z)1/2 has branch points at z = n.

280
Since the branch points at z = n go all the way out to infinity. It is not possible to make a path that encloses
infinity and no other singularities. The point at infinity is a non-isolated singularity. A point can be a branch
point only if it is an isolated singularity.
3.

z 1/2 sin z 1/2 = z sin z

= z sin z

= z sin z
The function is single-valued. Thus there could be no branch points.
4. 1/2
sin z 2 = sin z 2
This function is multi-valued. Since sin z 2 = 0 at z = (n)1/2 , there may be branch points there. First consider
the point z = 0. We can write
sin z 2
sin z 2 = z 2 2
z
2 2
where sin (z ) /z is nonzero and has a removable singularity at z = 0.
sin z 2 2z cos z 2
lim = lim = 1.
z0 z 2 z0 2z
1/2 1/2
Since (z 2 )does not have a branch point at z = 0, (sin z 2 ) does not have a branch point there either.

Now consider the point z = n.
 sin z 2
sin z 2 = z n
z n

sin (z 2 ) / (z n) in nonzero and has a removable singularity at z = n.
sin z 2 2z cos z 2
lim
= lim
= 2 n(1)n
z n z n z n 1

281
1/2 1/2
Since (z n) has a branch point at z = n, (sin z 2 ) also has a branch point there.
1/2
Thus
we
see that (sin z2) has branch points at z = (n)1/2 for n Z \ {0}. This is the set of numbers:
{ , 2, . . . , , 2, . . .}. The point at infinity is a non-isolated singularity.

Example 7.9.11 Find the branch points of


1/3
f (z) = z 3 z .

3
Introduce branch cuts. If f (2) = 6 then what is f (2)?
We expand f (z).
f (z) = z 1/3 (z 1)1/3 (z + 1)1/3 .
There are branch points at z = 1, 0, 1. We consider the point at infinity.
   1/3  1/3  1/3
1 1 1 1
f = 1 +1

1
= (1 )1/3 (1 + )1/3

Since f (1/) does not have a branch point at = 0, f (z) does not have a branch point at infinity. Consider the three
possible branch cuts in Figure 7.29.
The first and the third branch cuts will make the function single valued, the second will not. It is clear that the first
set makes the function single valued since it is not possible to walk around any of the branch points.
The second set of branch cuts would allow you to walk around the branch points at z = 1. If you walked around
these two once in the positive direction, the value of the function would change by the factor e4/3 .
The third set of branch cuts would allow you to walk around all three branch points together. You can verify that
if you walk around the three branch points, the value of the function will not change (e6/3 = e2 = 1).
Suppose we introduce the third set of branch cuts and are on the branch with f (2) = 3 6.
1/3 1/3 1/3
f (2) = 2 e0 1 e0 3 e0
3
= 6

282
1/3
Figure 7.29: Three Possible Branch Cuts for f (z) = (z 3 z) .

The value of f (2) is

f (2) = (2 e )1/3 (3 e )1/3 (1 e )1/3



= 2 e/3 3 e/3 1 e/3
3 3 3


= 6 e
3


3
= 6.
Example 7.9.12 Find the branch points and number of branches for
2
f (z) = z z .

2
z z = exp z 2 log z


There may be branch points at the origin and infinity due to the logarithm. Consider walking around a circle of radius
r centered at the origin in the positive direction. Since the logarithm changes by 2, the value of f (z) changes by the
2
factor e2r . There are branch points at the origin and infinity. The function has an infinite number of branches.

Example 7.9.13 Construct a branch of


1/3
f (z) = z 2 + 1

283
such that
1 
f (0) = 1 + 3 .
2
First we factor f (z).
f (z) = (z )1/3 (z + )1/3
There are branch points at z = . Figure 7.30 shows one way to introduce branch cuts.

1/3
Figure 7.30: Branch Cuts for f (z) = (z 2 + 1) .

Since it is not possible to walk around any branch point, these cuts make the function single valued. We introduce
the coordinates:
z = e , z + = r e .

1/3 1/3
f (z) = e r e

= 3 r e(+)/3

The condition
1 
f (0) = 1 + 3 = e(2/3+2n)
2

284
can be stated

1 e(+)/3 = e(2/3+2n)
3

+ = 2 + 6n

The angles must be defined to satisfy this relation. One choice is

5 3
<< , << .
2 2 2 2

Principal branches. We construct the principal branch of the logarithm by putting a branch cut on the negative
real axis choose z = r e , (, ). Thus the principal branch of the logarithm is

Log z = ln r + , < < .

Note that the if x is a negative real number, (and thus lies on the branch cut), then Log x is undefined.
The principal branch of z is
z = e Log z .
Note that there is a branch cut on the negative real axis.

< arg e Log z <





The principal branch of the z 1/2 is denoted z. The principal branch of z 1/n is denoted n
z.
1/2
Example 7.9.14 Construct 1 z 2 , the principal branch of (1 z 2 ) .
1/2
First note that since (1 z 2 ) = (1 z)1/2 (1 + z)1/2 there are branch points at z = 1 and z = 1. The
principal branch of the square root has a branch cut on the negative real axis. 1 z 2 is a negative real number for
z ( . . . 1) (1 . . . ). Thus we put branch cuts on ( . . . 1] and [1 . . . ).

285
7.10 Exercises
Cartesian and Modulus-Argument Form
Exercise 7.1
Find the image of the strip 2 < x < 3 under the mapping w = f (z) = z 2 . Does the image constitute a domain?
Hint, Solution
Exercise 7.2
For a given real number , 0 < 2, find the image of the sector 0 arg(z) < under the transformation w = z 4 .
How large should be so that the w plane is covered exactly once?
Hint, Solution

Trigonometric Functions
Exercise 7.3
In Cartesian coordinates, z = x + y, write sin(z) in Cartesian and modulus-argument form.
Hint, Solution
Exercise 7.4
Show that ez is nonzero for all finite z.
Hint, Solution
Exercise 7.5
Show that
z2 2
e e|z| .
When does equality hold?
Hint, Solution
Exercise 7.6
Solve coth(z) = 1.
Hint, Solution

286
Exercise 7.7
Solve 2 2z . That is, for what values of z is 2 one of the values of 2z ? Derive this result then verify your answer by
evaluating 2z for the solutions that your find.
Hint, Solution
Exercise 7.8
Solve 1 1z . That is, for what values of z is 1 one of the values of 1z ? Derive this result then verify your answer by
evaluating 1z for the solutions that your find.
Hint, Solution

Logarithmic Identities
Exercise 7.9
Show that if < (z1 ) > 0 and < (z2 ) > 0 then
Log(z1 z2 ) = Log(z1 ) + Log(z2 )
and illustrate that this relationship does not hold in general.
Hint, Solution
Exercise 7.10
Find the fallacy in the following arguments:
1

1. log(1) = log 1 = log(1) log(1) = log(1), therefore, log(1) = 0.

2. 1 = 11/2 = ((1)(1))1/2 = (1)1/2 (1)1/2 = = 1, therefore, 1 = 1.


Hint, Solution
Exercise 7.11
Write the following expressions in modulus-argument or Cartesian form. Denote any multi-valuedness explicitly.
 1/4
22/5 , 31+ , 3 , 1/4 .
Hint, Solution

287
Exercise 7.12
Solve cos z = 69.
Hint, Solution
Exercise 7.13
Solve cot z = 47.
Hint, Solution
Exercise 7.14
Determine all values of

1. log()

2. ()

3. 3

4. log(log())

and plot them in the complex plane.


Hint, Solution
Exercise 7.15
Evaluate and plot the following in the complex plane:

1. (cosh())2
 
1
2. log
1+

3. arctan(3)

Hint, Solution

288
Exercise 7.16
Determine all values of and log ((1 + ) ) and plot them in the complex plane.
Hint, Solution
Exercise 7.17
Find all z for which
1. ez =

2. cos z = sin z

3. tan2 z = 1
Hint, Solution
Exercise 7.18
Prove the following identities and identify the branch points of the functions in the extended complex plane.
 
+z
1. arctan(z) = log
2 z
 
1 1+z
2. arctanh(z) = log
2 1z
 1/2 
3. arccosh(z) = log z + z 2 1

Hint, Solution

Branch Points and Branch Cuts


Exercise 7.19
Identify the branch points of the function  
z(z + 1)
f (z) = log
z1

289
and introduce appropriate branch cuts to ensure that the function is single-valued.
Hint, Solution
Exercise 7.20
Identify all the branch points of the function
1/2
w = f (z) = z 3 + z 2 6z
in the extended complex plane. Give a polar description of f (z) and specify branch
cuts so that your choice of angles
gives a single-valued function that is continuous at z = 1 with f (1) = 6. Sketch the branch cuts in the
stereographic projection.
Hint, Solution
Exercise 7.21
Consider the mapping w = f (z) = z 1/3 and the inverse mapping z = g(w) = w3 .
1. Describe the multiple-valuedness of f (z).
2. Describe a region of the w-plane that g(w) maps one-to-one to the whole z-plane.
3. Describe and attempt to draw a Riemann surface on which f (z) is single-valued and to which g(w) maps one-
to-one. Comment on the misleading nature of your picture.
4. Identify the branch points of f (z) and introduce a branch cut to make f (z) single-valued.
Hint, Solution
Exercise 7.22
Determine the branch points of the function
1/2
f (z) = z 3 1 .
Construct cuts and define a branch so that z = 0 and z = 1 do not lie on a cut, and such that f (0) = . What is
f (1) for this branch?
Hint, Solution

290
Exercise 7.23
Determine the branch points of the function

w(z) = ((z 1)(z 6)(z + 2))1/2

Construct cuts and define a branch so that z = 4 does not lie on a cut, and such that w = 6 when z = 4.
Hint, Solution
Exercise 7.24
Give the number of branches and locations of the branch points for the functions

1. cos z 1/2


2. (z + )z

Hint, Solution
Exercise 7.25
Find the branch points of the following functions in the extended complex plane, (the complex plane including the point
at infinity).
1/2
1. z 2 + 1
1/2
2. z 3 z

3. log z 2 1


 
z+1
4. log
z1

Introduce branch cuts to make the functions single valued.


Hint, Solution

291
Exercise 7.26
Find all branch points and introduce cuts to make the following functions single-valued: For the first function, choose
cuts so that there is no cut within the disk |z| < 2.
1/2
1. f (z) = z 3 + 8
 1/2 !
z+1
2. f (z) = log 5 +
z1

3. f (z) = (z + 3)1/2

Hint, Solution
Exercise 7.27
Let f (z) have branch points at z = 0 and z = , but nowhere else in the extended complex plane. How does the
value and argument of f (z) change while traversing the contour in Figure 7.31? Does the branch cut in Figure 7.31
make the function single-valued?

Figure 7.31: Contour around the branch points and the branch cut.

292
Hint, Solution
Exercise 7.28
Let f (z) be analytic except for no more than a countably infinite number of singularities. Suppose that f (z) has only
one branch point in the finite complex plane. Does f (z) have a branch point at infinity? Now suppose that f (z) has
two or more branch points in the finite complex plane. Does f (z) have a branch point at infinity?
Hint, Solution
Exercise 7.29
1/4
Find all branch points of (z 4 + 1) in the extended complex plane. Which of the branch cuts in Figure 7.32 make the
function single-valued.

1/4
Figure 7.32: Four candidate sets of branch cuts for (z 4 + 1) .

Hint, Solution
Exercise 7.30
Find the branch points of
 1/3
z
f (z) = 2
z +1
in the extended complex plane. Introduce branch cuts that make the function single-valued and such that the function

293

is defined on the positive real axis. Define a branch such that f (1) = 1/ 3 2. Write down an explicit formula for the
value of the branch. What is f (1 + )? What is the value of f (z) on either side of the branch cuts?
Hint, Solution

Exercise 7.31
Find all branch points of
f (z) = ((z 1)(z 2)(z 3))1/2

in the extended complex plane. Which of the branch cuts in Figure 7.33 will make the function single-valued. Using
the first set of branch cuts in this figure define a branch on which f (0) = 6. Write out an explicit formula for the
value of the function on this branch.

Figure 7.33: Four candidate sets of branch cuts for ((z 1)(z 2)(z 3))1/2 .

Hint, Solution

294
Exercise 7.32
Determine the branch points of the function
1/3
z 2 2 (z + 2)

w= .

Construct and define a branch so that the resulting cut is one line of finite extent and w(2) = 2. What is w(3) for
this branch? What are the limiting values of w on either side of the branch cut?
Hint, Solution
Exercise 7.33
Construct the principal branch of arccos(z). (Arccos(z) has the property that if x [1, 1] then Arccos(x) [0, ].
In particular, Arccos(0) = 2 ).
Hint, Solution
Exercise 7.34 1/2
Find the branch points of z 1/2 1 in the finite complex plane. Introduce branch cuts to make the function
single-valued.
Hint, Solution
Exercise 7.35
For the linkage illustrated in Figure 7.34, use complex variables to outline a scheme for expressing the angular position,
velocity and acceleration of arm c in terms of those of arm a. (You neednt work out the equations.)
Hint, Solution
Exercise 7.36
Find the image of the strip |<(z)| < 1 and of the strip 1 < =(z) < 2 under the transformations:

1. w = 2z 2
z+1
2. w = z1

Hint, Solution

295
b
a
c

l

Figure 7.34: A linkage.

Exercise 7.37
Locate and classify all the singularities of the following functions:
(z + 1)1/2
1.
z+2
 
1
2. cos
1+z
1
3.
(1 ez )2
In each case discuss the possibility of a singularity at the point .
Hint, Solution
Exercise 7.38
Describe how the mapping w = sinh(z) transforms the infinite strip < x < , 0 < y < into the w-plane. Find
cuts in the w-plane which make the mapping continuous both ways. What are the images of the lines (a) y = /4; (b)
x = 1?
Hint, Solution

296
7.11 Hints
Cartesian and Modulus-Argument Form
Hint 7.1

Hint 7.2

Trigonometric Functions
Hint 7.3
Recall that sin(z) = 1
2
(ez ez ). Use Result 6.3.1 to convert between Cartesian and modulus-argument form.

Hint 7.4
Write ez in polar form.

Hint 7.5
The exponential is an increasing function for real variables.

Hint 7.6
Write the hyperbolic cotangent in terms of exponentials.

Hint 7.7
Write out the multi-valuedness of 2z . There is a doubly-infinite set of solutions to this problem.

Hint 7.8
Write out the multi-valuedness of 1z .

Logarithmic Identities

297
Hint 7.9

Hint 7.10
Write out the multi-valuedness of the expressions.

Hint 7.11
Do the exponentiations in polar form.

Hint 7.12
Write the cosine in terms of exponentials. Multiply by ez to get a quadratic equation for ez .

Hint 7.13
Write the cotangent in terms of exponentials. Get a quadratic equation for ez .

Hint 7.14

Hint 7.15

Hint 7.16
has an infinite number of real, positive values. = e log . log ((1 + ) ) has a doubly infinite set of values.
log ((1 + ) ) = log(exp( log(1 + ))).

Hint 7.17

Hint 7.18

Branch Points and Branch Cuts

298
Hint 7.19

Hint 7.20

Hint 7.21

Hint 7.22

Hint 7.23

Hint 7.24

Hint 7.25
1/2
1. (z 2 + 1) = (z )1/2 (z + )1/2
1/2
2. (z 3 z) = z 1/2 (z 1)1/2 (z + 1)1/2

3. log (z 2 1) = log(z 1) + log(z + 1)

4. log z+1

z1
= log(z + 1) log(z 1)

Hint 7.26

Hint 7.27
Reverse the orientation of the contour so that it encircles infinity and does not contain any branch points.

299
Hint 7.28
Consider a contour that encircles all the branch points in the finite complex plane. Reverse the orientation of the
contour so that it contains the point at infinity and does not contain any branch points in the finite complex plane.

Hint 7.29
Factor the polynomial. The argument of z 1/4 changes by /2 on a contour that goes around the origin once in the
positive direction.

Hint 7.30

Hint 7.31
To define the branch, define angles from each of the branch points in the finite complex plane.

Hint 7.32

Hint 7.33

Hint 7.34

Hint 7.35

Hint 7.36

Hint 7.37

300
Hint 7.38

301
7.12 Solutions
Cartesian and Modulus-Argument Form
Solution 7.1
Let w = u + v. We consider the strip 2 < x < 3 as composed of vertical lines. Consider the vertical line: z = c + y,
y R for constant c. We find the image of this line under the mapping.
w = (c + y)2
w = c2 y 2 + 2cy
u = c2 y 2 , v = 2cy
This is a parabola that opens to the left. We can parameterize the curve in terms of v.
1 2
u = c2 v , vR
4c2
The boundaries of the region, x = 2 and x = 3, are respectively mapped to the parabolas:
1 2 1 2
u=4 v , v R and u = 9 v , vR
16 36
We write the image of the mapping in set notation.
 
1 2 1 2
w = u + v : v R and 4 v < u < 9 v .
16 36

See Figure 7.35 for depictions of the strip and its image under the mapping. The mapping is one-to-one. Since the
image of the strip is open and connected, it is a domain.
Solution 7.2
We write the mapping w = z 4 in polar coordinates.
4
w = z 4 = r e = r4 e4

302
3 10
2
5
1

-1 1 2 3 4 5 -5 5 10 15
-1
-5
-2
-3 -10

Figure 7.35: The domain 2 < x < 3 and its image under the mapping w = z 2 .

Thus we see that

w : {r e | r 0, 0 < } {r4 e4 | r 0, 0 < } = {r e | r 0, 0 < 4}.

We can state this in terms of the argument.

w : {z | 0 arg(z) < } {z | 0 arg(z) < 4}

If = /2, the sector will be mapped exactly to the whole complex plane.

Trigonometric Functions

303
Solution 7.3

1 z
e ez

sin z =
2
1 y+x
eyx

= e
2
1 y
e (cos x + sin x) ey (cos x sin x)

=
2
1 y
e (sin x cos x) + ey (sin x + cos x)

=
2
= sin x cosh y + cos x sinh y

q
sin z = sin2 x cosh2 y + cos2 x sinh2 y exp( arctan(sin x cosh y, cos x sinh y))
q
= cosh2 y cos2 x exp( arctan(sin x cosh y, cos x sinh y))
r
1
= (cosh(2y) cos(2x)) exp( arctan(sin x cosh y, cos x sinh y))
2
Solution 7.4
In order that ez be zero, the modulus, ex must be zero. Since ex has no finite solutions, ez = 0 has no finite solutions.

Solution 7.5
We write the expressions in terms of Cartesian coordinates.

z2 (x+y)2
e = e

2 2
= ex y +2xy

2 y 2
= ex

304
2 2 2 +y 2
e|z| = e|x+y| = ex
2 y 2 2 +y 2
The exponential function is an increasing function for real variables. Since x2 y 2 x2 + y 2 , ex ex .

z2 2
e e|z|
Equality holds only when y = 0.
Solution 7.6

coth(z) = 1
(e + ez ) /2
z
=1
(ez ez ) /2
ez + ez = ez ez
ez = 0
There are no solutions.
Solution 7.7
We write out the multi-valuedness of 2z .
2 2z
eln 2 ez log(2)
eln 2 {ez(ln(2)+2n) | n Z}
ln 2 z{ln 2 + 2n + 2m | m, n Z}
 
ln(2) + 2m
z= | m, n Z
ln(2) + 2n
We verify this solution. Consider m and n to be fixed integers. We express the multi-valuedness in terms of k.
2(ln(2)+2m)/(ln(2)+2n) = e(ln(2)+2m)/(ln(2)+2n) log(2)
= e(ln(2)+2m)/(ln(2)+2n)(ln(2)+2k)

305
For k = n, this has the value, eln(2)+2m = eln(2) = 2.
Solution 7.8
We write out the multi-valuedness of 1z .
1 1z
1 ez log(1)
1 {ez2n | n Z}
The element corresponding to n = 0 is e0 = 1. Thus 1 1z has the solutions,
z C.
That is, z may be any complex number. We verify this solution.
1z = ez log(1) = ez2n
For n = 0, this has the value 1.

Logarithmic Identities
Solution 7.9
We write the relationship in terms of the natural logarithm and the principal argument.
Log(z1 z2 ) = Log(z1 ) + Log(z2 )
ln |z1 z2 | + Arg(z1 z2 ) = ln |z1 | + Arg(z1 ) + ln |z2 | + Arg(z2 )
Arg(z1 z2 ) = Arg(z1 ) + Arg(z2 )
< (zk ) > 0 implies that Arg(zk ) (/2 . . . /2). Thus Arg(z1 ) + Arg(z2 ) ( . . . ). In this case the relationship
holds.
The relationship does not hold in general because Arg(z1 ) + Arg(z2 ) is not necessarily in the interval ( . . . ].
Consider z1 = z2 = 1.
Arg((1)(1)) = Arg(1) = 0, Arg(1) + Arg(1) = 2
Log((1)(1)) = Log(1) = 0, Log(1) + Log(1) = 2

306
Solution 7.10
1. The algebraic manipulations are fine. We write out the multi-valuedness of the logarithms.
 
1
log(1) = log = log(1) log(1) = log(1)
1

{ + 2n : n Z} = { + 2n : n Z}
= {2n : n Z} { + 2n : n Z} = { 2n : n Z}
Thus log(1) = log(1). However this does not imply that log(1) = 0. This is because the logarithm is a
set-valued function log(1) = log(1) is really saying:
{ + 2n : n Z} = { 2n : n Z}

2. We consider
1 = 11/2 = ((1)(1))1/2 = (1)1/2 (1)1/2 = = 1.
There are three multi-valued expressions above.
11/2 = 1
((1)(1))1/2 = 1
(1)1/2 (1)1/2 = ()() = 1
Thus we see that the first and fourth equalities are incorrect.
1 6= 11/2 , (1)1/2 (1)1/2 6=
Solution 7.11

22/5 = 41/5

= 411/5
5


= 4 e2n/5 ,
5
n = 0, 1, 2, 3, 4

307
31+ = e(1+) log 3
= e(1+)(ln 3+2n)
= eln 32n e(ln 3+2n) , nZ

 1/4 1/4
3 = 2 e/6

= 2 e/24 11/4
4


= 2 e(n/2/24) ,
4
n = 0, 1, 2, 3

1/4 = e(/4) log 1


= e(/4)(2n)
= en/2 , nZ

308
Solution 7.12

cos z = 69
ez + ez
= 69
2
e2z 138 ez +1 = 0
1 
ez = 138 1382 4
2  
z = log 69 2 1190
   
z = ln 69 2 1190 + 2n
 
z = 2n ln 69 2 1190 , n Z

309
Solution 7.13

cot z = 47
(e + ez ) /2
z
= 47
(ez ez ) /(2)
ez + ez = 47 ez ez


46 e2z 48 = 0
24
2z = log
23
24
z = log
 2 23

24
z= ln + 2n , n Z
2 23
24
z = n ln , nZ
2 23
Solution 7.14
1.
log() = ln | | + arg()
 
= ln(1) + + 2n , n Z
2

log() = + 2n, n Z
2
These are equally spaced points in the imaginary axis. See Figure 7.36.
2.
() = e log()
= e(/2+2n) , nZ

310
10

-1 1
-10

Figure 7.36: The values of log().

() = e/2+2n , nZ
These are points on the positive real axis with an accumulation point at the origin. See Figure 7.37.
1

-1

Figure 7.37: The values of () .

3.
3 = e log(3)
= e(ln(3)+ arg(3))

311
3 = e(ln(3)+2n) , nZ

These points all lie on the circle of radius |e | centered about the origin in the complex plane. See Figure 7.38.

10

-10 -5 5 10

-5

-10

Figure 7.38: The values of 3 .

4.
  
log(log()) = log + 2m , m Z
2   
= ln + 2m + Arg + 2m + 2n, m, n Z

2 2

= ln + 2m + sign(1 + 4m) + 2n, m, n Z

2 2

These points all lie in the right half-plane. See Figure 7.39.

312
20
10

1 2 3 4 5
-10
-20

Figure 7.39: The values of log(log()).

Solution 7.15
1.

2
e + e

2
(cosh()) =
2
2
= (1)
= e2 log(1)
= e2(ln(1)++2n) , nZ
2(1+2n)
=e , nZ

These are points on the positive real axis with an accumulation point at the origin. See Figure 7.40.

313
1

1000

-1

Figure 7.40: The values of (cosh())2 .

2.

 
1
log = log(1 + )
1+
 
/4
= log 2 e
1
= ln(2) log e/4

2
1
= ln(2) /4 + 2n, nZ
2

These are points on a vertical line in the complex plane. See Figure 7.41.

314
10

-1 1
-10

1

Figure 7.41: The values of log 1+
.

3.

 
1 3
arctan(3) = log
2 + 3
 
1 1
= log
2 2
   
1 1
= ln + + 2n , nZ
2 2

= + n + ln(2)
2 2

These are points on a horizontal line in the complex plane. See Figure 7.42.

315
1

-5 5

-1

Figure 7.42: The values of arctan(3).

Solution 7.16

= e log()
= e(ln ||+ Arg()+2n) , nZ
(/2+2n)
=e , nZ
= e(1/2+2n) , nZ
These are points on the positive real axis. There is an accumulation point at z = 0. See Figure 7.43.

log ((1 + ) ) = log e log(1+)




= log(1 + ) + 2n, n Z
= (ln |1 + | + Arg(1 + ) + 2m) + 2n, m, n Z
 
1
= ln 2 + + 2m + 2n, m, n Z
2 4
   
2 1 1
= + 2m + ln 2 + 2n , m, n Z
4 2

316
1

25 50 75 100

-1

Figure 7.43: The values of .

See Figure 7.44 for a plot.

10
5

-40 -20 20
-5
-10

Figure 7.44: The values of log ((1 + ) ).

317
Solution 7.17
1.

ez =
z = log
z = ln || + arg()
 
z = ln(1) + + 2n , n Z
2

z = + 2n, n Z
2

2. We can solve the equation by writing the cosine and sine in terms of the exponential.

cos z = sin z
ez + ez ez ez
=
2 2
(1 + ) ez = (1 + ) ez
1 +
e2z =
1+
2z
e =
2z = log()

2z = + 2n, n Z
2

z = + n, n Z
4

318
3.

tan2 z = 1
sin2 z = cos2 z
cos z = sin z
ez + ez ez ez
=
2 2
ez = ez or ez = ez
ez = 0 or ez = 0
eyx = 0 or ey+x = 0
ey = 0 or ey = 0
z=

There are no solutions for finite z.

319
Solution 7.18
1.
w = arctan(z)
z = tan(w)
sin(w)
z=
cos(w)
(e ew ) /(2)
w
z=
(ew + ew ) /2
z ew +z ew = ew + ew
( + z) e2w = ( z)
 1/2
w z
e =
+z
 1/2
z
w = log
+z
 
+z
arctan(z) = log
2 z
We identify the branch points of the arctangent.

arctan(z) = (log( + z) log( z))
2
There are branch points at z = due to the logarithm terms. We examine the point at infinity with the change
of variables = 1/z.
 
+ 1/
arctan(1/) = log
2 1/
 
+ 1
arctan(1/) = log
2 1

320
As 0, the argument of the logarithm term tends to 1 The logarithm does not have a branch point at that
point. Since arctan(1/) does not have a branch point at = 0, arctan(z) does not have a branch point at
infinity.

2.

w = arctanh(z)
z = tanh(w)
sinh(w)
z=
cosh(w)
(e ew ) /2
w
z= w
(e + ew ) /2
z ew +z ew = ew ew
(z 1) e2w = z 1
 1/2
w z 1
e =
z1
 1/2
z+1
w = log
1z
 
1 1+z
arctanh(z) = log
2 1z

We identify the branch points of the hyperbolic arctangent.

1
arctanh(z) = (log(1 + z) log(1 z))
2

There are branch points at z = 1 due to the logarithm terms. We examine the point at infinity with the change

321
of variables = 1/z.
 
1 1 + 1/
arctanh(1/) = log
2 1 1/
 
1 +1
arctanh(1/) = log
2 1

As 0, the argument of the logarithm term tends to 1 The logarithm does not have a branch point at that
point. Since arctanh(1/) does not have a branch point at = 0, arctanh(z) does not have a branch point at
infinity.

3.

w = arccosh(z)
z = cosh(w)
ew + ew
z=
2
e2w 2z ew +1 = 0
1/2
ew = z + z 2 1
 1/2 
w = log z + z 2 1
 1/2 
arccosh(z) = log z + z 2 1

We identify the branch points of the hyperbolic arc-cosine.

arccosh(z) = log z + (z 1)1/2 (z + 1)1/2




First we consider branch points due to the square root. There are branch points at z = 1 due to the square
1/2
root terms. If we walk around the singularity at z = 1 and no other singularities, the (z 2 1) term changes

322
sign. This will change the value of arccosh(z). The same is true for the point z = 1. The point at infinity is
1/2
not a branch point for (z 2 1) . We factor the expression to verify this.
1/2 1/2 1/2
z2 1 = z2 1 z 2
1/2 1/2
(z 2 ) does not have a branch point at infinity. It is multi-valued, but it has no branch points. (1 z 2 ) does
not have a branch point at infinity, The argument of the square root function tends to unity there. In summary,
there are branch points at z = 1 due to the square root. If we walk around either one of the these branch
points. the square root term will change value. If we walk around both of these points, the square root term will
not change value.
Now we consider branch points due to logarithm. There may be branch points where the argument of the
logarithm vanishes or tends to infinity. We see if the argument of the logarithm vanishes.
1/2
z + z2 1 =0
2 2
z =z 1
1/2
z + (z 2 1) is non-zero and finite everywhere in the complex plane. The only possibility for a branch point
1/2
in the logarithm term is the point at infinity. We see if the argument of z + (z 2 1) changes when we walk
around infinity but no other singularity. We consider a circular path with center at the origin and radius greater
than unity. We can either say that this path encloses the two branch points at z = 1 and no other singularities
or we can say that this path encloses the point at infinity and no other singularities. We examine the value of
the argument of the logarithm on this path.
1/2 1/2 1/2
z + z2 1 = z + z2 1 z 2
1/2 1/2
Neither (z 2 ) nor (1 z 2 ) changes value as we walk the path. Thus we can use the principal branch of the
square root in the expression.
1/2
2
 
z+ z 1 2
=zz 1z =z 1 1z 2

323
First consider the + branch.  
z 1 + 1 z 2

As we walk the path around infinity, the argument of z changes by 2 while the argument of 1 + 1 z 2
1/2
does not change. Thus the argument of z + (z 2 1) changes by 2 when we go around infinity. This makes
the value of the logarithm change by 2. There is a branch point at infinity.
First consider the branch.

  

2
 1 2 4

z 1 1z =z 1 1 z +O z
2
 
1 2 4

=z z +O z
2
1
= z 1 1 + O z 2

2
As we walk the path around infinity, the argument of z 1 changes by 2 while the argument of (1 + O (z 2 ))
1/2
does not change. Thus the argument of z + (z 2 1) changes by 2 when we go around infinity. This makes
the value of the logarithm change by 2. Again we conclude that there is a branch point at infinity.
For the sole purpose of overkill, lets repeat the above analysis from a geometric viewpoint. Again we consider
the possibility of a branch point at infinity due to the logarithm. We walk along the circle shown in the first plot
of Figure 7.45. Traversing this path, we go around infinity, but no other singularities. We consider the mapping
1/2
w = z + (z 2 1) . Depending on the branch of the square root, the circle is mapped to one one of the contours
shown in the second plot. For each branch, the argument
 of w changes
 by 2 as we traverse the circle in the
1/2
z-plane. Therefore the value of arccosh(z) = log z + (z 2 1) changes by 2 as we traverse the circle.
We again conclude that there is a branch point at infinity due to the logarithm.
To summarize: There are branch points at z = 1 due to the square root and a branch point at infinity due to
the logarithm.

Branch Points and Branch Cuts

324
1

-1 1 -1 1
-1

-1

1/2
Figure 7.45: The mapping of a circle under w = z + (z 2 1) .

Solution 7.19
We expand the function to diagnose the branch points in the finite complex plane.
 
z(z + 1)
f (z) = log = log(z) + log(z + 1) log(z 1)
z1
The are branch points at z = 1, 0, 1. Now we examine the point at infinity. We make the change of variables z = 1/.
   
1 (1/)(1/ + 1)
f = log
(1/ 1)
 
1 (1 +
= log
1
= log(1 + ) log(1 ) log()
log() has a branch point at = 0. The other terms do not have branch points there. Since f (1/) has a branch point
at = 0 f (z) has a branch point at infinity.
Note that in walking around either z = 1 or z = 0 once in the positive direction, the argument of z(z + 1)/(z 1)
changes by 2. In walking around z = 1, the argument of z(z + 1)/(z 1) changes by 2. This argument does not

325
change if we walk around both z = 0 and z = 1. Thus we put a branch cut between z = 0 and z = 1. Next be put
a branch cut between z = 1 and the point at infinity. This prevents us from walking around either of these branch
points. These two branch cuts separate the branches of the function. See Figure 7.46

-3 -2 -1 1 2

 
z(z+1)
Figure 7.46: Branch cuts for log z1
.

Solution 7.20
First we factor the function.

f (z) = (z(z + 3)(z 2))1/2 = z 1/2 (z + 3)1/2 (z 2)1/2

There are branch points at z = 3, 0, 2. Now we examine the point at infinity.


     1/2
1 1 1 1
f = +3 2 = 3/2 ((1 + 3)(1 2))1/2

Since 3/2 has a branch point at = 0 and the rest of the terms are analytic there, f (z) has a branch point at infinity.
Consider the set of branch cuts in Figure 7.47. These cuts do not permit us to walk around any single branch point.
We can only walk around none or all of the branch points, (which is the same thing). The cuts can be used to define
a single-valued branch of the function.

326
3

-4 -2 2 4
-1

-2

-3

1/2
Figure 7.47: Branch cuts for (z 3 + z 2 6z) .

327
Now to define the branch. We make a choice of angles.

z + 3 = r1 e1 , < 1 <
3
z = r2 e2 , < 2 <
2 2
z 2 = r3 e3 , 0 < 3 < 2

The function is 1/2


f (z) = r1 e1 r2 e2 r3 e3 = r1 r2 r3 e(1 +2 +3 )/2 .
We evaluate the function at z = 1.
p
f (1) = (2)(1)(3) e(0++)/2 = 6

We see that our choice of angles gives us the desired branch.


The stereographic projection is the projection from the complex plane onto a unit sphere with south pole at the
origin. The point z = x + y is mapped to the point (X, Y, Z) on the sphere with

4x 4y 2|z|2
X= , Y = , Z= .
|z|2 + 4 |z|2 + 4 |z|2 + 4

Figure 7.48 first shows the branch cuts and their stereographic projections and then shows the stereographic projections
alone.
Solution 7.21
1. For each value of z, f (z) = z 1/3 has three values.

f (z) = z 1/3 = 3
z ek2/3 , k = 0, 1, 2

2.
g(w) = w3 = |w|3 e3 arg(w)

328
1
0
-1
2
2 4
0 1
-4 0 0
0 -1
0
4 -4 1

1/2
Figure 7.48: Branch cuts for (z 3 + z 2 6z) and their stereographic projections.

Any sector of the w plane of angle 2/3 maps one-to-one to the whole z-plane.

g : r e | r 0, 0 < 0 + 2/3 7 r3 e3 | r 0, 0 < 0 + 2/3


 

g : r e | r 0, 0 < 0 + 2/3 7 r e | r 0, 30 < 30 + 2


 

g : r e | r 0, 0 < 0 + 2/3 7 C


See Figure 7.49 to see how g(w) maps the sector 0 < 2/3.

3. See Figure 7.50 for a depiction of the Riemann surface for f (z) = z 1/3 . We show two views of the surface and a
curve that traces the edge of the shown portion of the surface. The depiction is misleading because the surface
is not self-intersecting. We would need four dimensions to properly visualize the this Riemann surface.

4. f (z) = z 1/3 has branch points at z = 0 and z = . Any branch cut which connects these two points would
prevent us from walking around the points singly and would thus separate the branches of the function. For
example, we could put a branch cut on the negative real axis. Defining the angle < < for the mapping

f r e = 3 r e/3

defines a single-valued branch of the function.

329
Figure 7.49: The function g(w) = w3 maps the sector 0 < 2/3 one-to-one to the whole z-plane.

330
Figure 7.50: Riemann surface for f (z) = z 1/3 .

Solution 7.22
The cube roots of 1 are ( )
1 + 3 1 3
1, e2/3 , e 4/3

= 1, , .
2 2
We factor the polynomial.
!1/2 !1/2
1/2 1 3 1+ 3
z3 1 = (z 1)1/2 z+ z+
2 2

There are branch points at each of the cube roots of unity.


( )
1 + 3 1 3
z = 1, ,
2 2

Now we examine the point at infinity. We make the change of variables z = 1/.
1/2 1/2
f (1/) = 1/ 3 1 = 3/2 1 3

331
1/2
3/2 has a branch point at = 0, while (1 3 ) is not singular there. Since f (1/) has a branch point at = 0,
f (z) has a branch point at infinity.

There are several ways of introducing branch cuts to separate the branches of the function. The easiest approach is
to put a branch cut from each of the three branch points in the finite complex plane out to the branch point at infinity.
See Figure 7.51a. Clearly this makes the function single valued as it is impossible to walk around any of the branch
points. Another approach is to have a branch cut from one of the branch points in the finite plane to the branch point
at infinity and a branch cut connecting the remaining two branch points. See Figure 7.51bcd. Note that in walking
around any one of the finite branch points, (in the positive direction), the argument of the function changes by . This
means that the value of the function changes by e , which is to say the value of the function changes sign. In walking
around any two of the finite branch points, (again in the positive direction), the argument of the function changes by
2. This means that the value of the function changes by e2 , which is to say that the value of the function does not
change. This demonstrates that the latter branch cut approach makes the function single-valued.

a b c d

1/2
Figure 7.51: Suitable branch cuts for (z 3 1) .

Now we construct a branch. We will use the branch cuts in Figure 7.51a. We introduce variables to measure radii

332
and angles from the three finite branch points.

z 1 = r1 e1 , 0 < 1 < 2

1 3 2
z+ = r2 e2 , < 2 <
2 3 3
1+ 3 2
z+ = r3 e3 , < 3 <
2 3 3
We compute f (0) to see if it has the desired value.

f (z) = r1 r2 r3 e(1 +2 +3 )/2
f (0) = e(/3+/3)/2 =

Since it does not have the desired value, we change the range of 1 .

z 1 = r1 e1 , 2 < 1 < 4

f (0) now has the desired value.


f (0) = e(3/3+/3)/2 =
We compute f (1).
f (1) = 2 e(32/3+2/3)/2 = 2

Solution 7.23
First we factor the function.

w(z) = ((z + 2)(z 1)(z 6))1/2 = (z + 2)1/2 (z 1)1/2 (z 6)1/2

There are branch points at z = 2, 1, 6. Now we examine the point at infinity.


     1/2    1/2
1 1 1 1 3/2 2 1 6
w = +2 1 6 = 1+ 1 1

333
Since 3/2 has a branch point at = 0 and the rest of the terms are analytic there, w(z) has a branch point at infinity.
Consider the set of branch cuts in Figure 7.52. These cuts let us walk around the branch points at z = 2 and
z = 1 together or if we change our perspective, we would be walking around the branch points at z = 6 and z =
together. Consider a contour in this cut plane that encircles the branch points at z = 2 and z = 1. Since the
argument of (z z0 )1/2 changes by when we walk around z0 , the argument of w(z) changes by 2 when we traverse
the contour. Thus the value of the function does not change and it is a valid set of branch cuts.

 
    

Figure 7.52: Branch cuts for ((z + 2)(z 1)(z 6))1/2 .

Now to define the branch. We make a choice of angles.

z + 2 = r1 e1 , 1 = 2 for z (1 . . . 6),
z 1 = r2 e2 , 2 = 1 for z (1 . . . 6),
z 6 = r3 e3 , 0 < 3 < 2

The function is 1/2


w(z) = r1 e1 r2 e2 r3 e3 = r1 r2 r3 e(1 +2 +3 )/2 .
We evaluate the function at z = 4.
p
w(4) = (6)(3)(2) e(2n+2n+)/2 = 6

We see that our choice of angles gives us the desired branch.

334
Solution 7.24
1.
 
cos z 1/2 = cos z = cos z


This is a single-valued function. There are no branch points.


2.
(z + )z = ez log(z+)
= ez(ln |z+|+ Arg(z+)+2n) , nZ
There is a branch point at z = . There are an infinite number of branches.
Solution 7.25
1.
1/2
f (z) = z 2 + 1 = (z + )1/2 (z )1/2
We see that there are branch points at z = . To examine the point at infinity, we substitute z = 1/ and
examine the point = 0.
 2 !1/2
1 1 2 1/2

+1 = 1 +
( 2 )1/2
Since there is no branch point at = 0, f (z) has no branch point at infinity.
A branch cut connecting z = would make the function single-valued. We could also accomplish this with two
branch cuts starting z = and going to infinity.
2. 1/2
f (z) = z 3 z = z 1/2 (z 1)1/2 (z + 1)1/2
There are branch points at z = 1, 0, 1. Now we consider the point at infinity.
   3 !1/2
1 1 1 1/2
f = = 3/2 1 2

335
There is a branch point at infinity.
One can make the function single-valued with three branch cuts that start at z = 1, 0, 1 and each go to infinity.
We can also make the function single-valued with a branch cut that connects two of the points z = 1, 0, 1 and
another branch cut that starts at the remaining point and goes to infinity.

3.
f (z) = log z 2 1 = log(z 1) + log(z + 1)


There are branch points at z = 1.


   
1 1
1 = log 2 + log 1 2
 
f = log 2

log ( 2 ) has a branch point at = 0.

log 2 = ln 2 + arg 2 = ln 2 2 arg()


 

Every time we walk around the point = 0 in the positive direction, the value of the function changes by 4.
f (z) has a branch point at infinity.
We can make the function single-valued by introducing two branch cuts that start at z = 1 and each go to
infinity.

4.  
z+1
f (z) = log = log(z + 1) log(z 1)
z1
There are branch points at z = 1.
     
1 1/ + 1 1+
f = log = log
1/ 1 1

There is no branch point at = 0. f (z) has no branch point at infinity.

336
We can make the function single-valued by introducing two branch cuts that start at z = 1 and each go to
infinity. We can also make the function single-valued with a branch cut that connects the points z = 1. This is
because log(z + 1) and log(z 1) change by 2 and 2, respectively, when you walk around their branch
points once in the positive direction.
Solution 7.26
1. The cube roots of 8 are
n o
2, 2 e2/3 , 2 e4/3 = 2, 1 + 3, 1 3 .


Thus we can write


1/2  1/2  1/2
z3 + 8 = (z + 2)1/2 z 1 3 z1+ 3 .

There are three branch points on the circle of radius 2.


n o
z = 2, 1 + 3, 1 3 .

We examine the point at infinity.


1/2 1/2
f (1/) = 1/ 3 + 8 = 3/2 1 + 8 3

Since f (1/) has a branch point at = 0, f (z) has a branch point at infinity.
There are several ways of introducing branch cuts outside of the disk |z| < 2 to separate the branches of the
function. The easiest approach is to put a branch cut from each of the three branch points in the finite complex
plane out to the branch point at infinity. See Figure 7.53a. Clearly this makes the function single valued as it
is impossible to walk around any of the branch points. Another approach is to have a branch cut from one of
the branch points in the finite plane to the branch point at infinity and a branch cut connecting the remaining
two branch points. See Figure 7.53bcd. Note that in walking around any one of the finite branch points, (in
the positive direction), the argument of the function changes by . This means that the value of the function
changes by e , which is to say the value of the function changes sign. In walking around any two of the finite

337
a b c d

1/2
Figure 7.53: Suitable branch cuts for (z 3 + 8) .

branch points, (again in the positive direction), the argument of the function changes by 2. This means that
the value of the function changes by e2 , which is to say that the value of the function does not change. This
demonstrates that the latter branch cut approach makes the function single-valued.
2.  1/2 !
z+1
f (z) = log 5 +
z1
First we deal with the function  1/2
z+1
g(z) =
z1
Note that it has branch points at z = 1. Consider the point at infinity.
 1/2  1/2
1/ + 1 1+
g(1/) = =
1/ 1 1
Since g(1/) has no branch point at = 0, g(z) has no branch point at infinity. This means that if we walk
around both of the branch points at z = 1, the function does not change value. We can verify this with another
method: When we walk around the point z = 1 once in the positive direction, the argument of z + 1 changes
by 2, the argument of (z + 1)1/2 changes by and thus the value of (z + 1)1/2 changes by e = 1. When we

338
walk around the point z = 1 once in the positive direction, the argument of z 1 changes by 2, the argument
of (z 1)1/2 changes by and thus the value of (z 1)1/2 changes by e = 1. f (z) has branch points
z+1 1/2

at z = 1. When we walk around both points z = 1 once in the positive direction, the value of z1 does
not change. Thus we can make the function single-valued with a branch cut which enables us to walk around
either none or both of these branch points. We put a branch cut from 1 to 1 on the real axis.
f (z) has branch points where
 1/2
z+1
5+
z1
is either zero or infinite. The only place in the extended complex plane where the expression becomes infinite is
at z = 1. Now we look for the zeros.
 1/2
z+1
5+ =0
z1
 1/2
z+1
= 5
z1
z+1
= 25
z1
z + 1 = 25z 25
13
z=
12

Note that  1/2


13/12 + 1
= 251/2 = 5.
13/12 1
On one branch, (which we call the positive branch), of the function g(z) the quantity
 1/2
z+1
5+
z1

339
is always nonzero. On the other (negative) branch of the function, this quantity has a zero at z = 13/12.
The logarithm introduces branch points at z = 1 on both the positive and negative branch of g(z). It introduces
a branch point at z = 13/12 on the negative branch of g(z). To determine if additional branch cuts are needed
to separate the branches, we consider
 1/2
z+1
w =5+
z1
and see where the branch cut between 1 gets mapped to in the w plane. We rewrite the mapping.
 1/2
2
w =5+ 1+
z1

The mapping is the following sequence of simple transformations:

(a) z 7 z 1
1
(b) z 7
z
(c) z 7 2z
(d) z 7 z + 1
(e) z 7 z 1/2
(f) z 7 z + 5

We show these transformations graphically below.

-1 1 -2 0 -1/2 -1

1
z 7 z 1 z 7 z 7 2z z 7 z + 1
z

340
0

z 7 z 1/2 z 7 z + 5
For the positive branch of g(z), the branch cut is mapped to the line x = 5 and the z plane is mapped to the
half-plane x > 5. log(w) has branch points at w = 0 and w = . It is possible to walk around only one of these
points in the half-plane x > 5. Thus no additional branch cuts are needed in the positive sheet of g(z).
For the negative branch of g(z), the branch cut is mapped to the line x = 5 and the z plane is mapped to the
half-plane x < 5. It is possible to walk around either w = 0 or w = alone in this half-plane. Thus we need an
additional branch cut. On the negative sheet of g(z), we put a branch cut beteen z = 1 and z = 13/12. This
puts a branch cut between w = and w = 0 and thus separates the branches of the logarithm.
Figure 7.54 shows the branch cuts in the positive and negative sheets of g(z).

Im(z) Im(z)
g(13/12)=5 g(13/12)=-5
Re(z) Re(z)

 
z+1 1/2

Figure 7.54: The branch cuts for f (z) = log 5 + z1
.

3. The function f (z) = (z + 3)1/2 has a branch point at z = 3. The function is made single-valued by connecting
this point and the point at infinity with a branch cut.
Solution 7.27
Note that the curve with opposite orientation goes around infinity in the positive direction and does not enclose any
branch points. Thus the value of the function does not change when traversing the curve, (with either orientation, of

341
course). This means that the argument of the function must change my an integer multiple of 2. Since the branch
cut only allows us to encircle all three or none of the branch points, it makes the function single valued.
Solution 7.28
We suppose that f (z) has only one branch point in the finite complex plane. Consider any contour that encircles this
branch point in the positive direction. f (z) changes value if we traverse the contour. If we reverse the orientation of
the contour, then it encircles infinity in the positive direction, but contains no branch points in the finite complex plane.
Since the function changes value when we traverse the contour, we conclude that the point at infinity must be a branch
point. If f (z) has only a single branch point in the finite complex plane then it must have a branch point at infinity.
If f (z) has two or more branch points in the finite complex plane then it may or may not have a branch point at
infinity. This is because the value of the function may or may not change on a contour that encircles all the branch
points in the finite complex plane.
Solution 7.29
First we factor the function,
 1/4  1/4  1/4  1/4
4
1/4 1+ 1 + 1 1
f (z) = z + 1 = z z z z .
2 2 2 2
1
There are branch points at z = .
2
We make the substitution z = 1/ to examine the point at infinity.
   1/4
1 1
f = +1
4
1 4 1/4

= 1 +
( 4 )1/4
4 1/4
1/4 has a removable singularity at the point = 0, but no branch point there. Thus (z 4 + 1) has no branch
point at infinity.
1/4
Note that the argument of (z 4 z0 ) changes by /2 on a contour that goes around the point z0 once in the
1/4
positive direction. The argument of (z 4 + 1) changes by n/2 on a contour that goes around n of its branch points.

342
Thus any set of branch cuts that permit you to walk around only one, two or three of the branch points will not make
the function single valued. A set of branch cuts that permit us to walk around only zero or all four of the branch points
will make the function single-valued. Thus we see that the first two sets of branch cuts in Figure 7.32 will make the
function single-valued, while the remaining two will not.
Consider the contour in Figure 7.32. There are two ways to see that the function does not change value while
traversing the contour. The first is to note that each of the branch points makes the argument of the function increase
1/4
by /2. Thus the argument of (z 4 + 1) changes by 4(/2) = 2 on the contour. This means that the value of the
function changes by the factor e2 = 1. If we change the orientation of the contour, then it is a contour that encircles
infinity once in the positive direction. There are no branch points inside the this contour with opposite orientation.
(Recall that the inside of a contour lies to your left as you walk around it.) Since there are no branch points inside this
contour, the function cannot change value as we traverse it.
Solution 7.30
 1/3
z
f (z) = 2
= z 1/3 (z )1/3 (z + )1/3
z +1
There are branch points at z = 0, .
1/3
1/3
  
1 1/
f = =
(1/)2 + 1 (1 + 2 )1/3
There is a branch point at = 0. f (z) has a branch point at infinity.
We introduce branch cuts from z = 0 to infinity on the negative real axis, from z = to infinity on the positive
imaginary axis and from z = to infinity on the negative imaginary axis. As we cannot walk around any of the branch
points, this makes the function single-valued.
We define a branch by defining angles from the branch points. Let
z = r e < < ,

(z ) = s e 3/2 < < /2,
(z + ) = t e /2 < < 3/2.

343
With

f (z) = z 1/3 (z )1/3 (z + )1/3


1 /3 1 /3
= 3 r e/3 e 3
e
3
s t
r
r ()/3
= 3 e
st
we have an explicit formula for computing the value of the function for this branch. Now we compute f (1) to see if we
chose the correct ranges for the angles. (If not, well just change one of them.)
s
1 1
f (1) = 3 e(0/4(/4))/3 = 3
2 2 2

We made the right choice for the angles. Now to compute f (1 + ).


s r
3 2 2
f (1 + ) = e(/40Arctan(2))/3 = 6 e(/4Arctan(2))/3
1 5 5

Consider the value of the function above and below the branch cut on the negative real axis. Above the branch cut the
function is r
x
f (x + 0) = 3 e()/3
2
x +1 x +12

Note that = so that r r


x x 1+ 3
f (x + 0) = 3
2
e/3 = 3
.
x +1 x2 + 1 2
Below the branch cut = and
r r
x x 1 3
f (x 0) = 3
2
e()/3 = 3
.
x +1 x2 + 1 2

344
For the branch cut along the positive imaginary axis,

r
y
f (y + 0) = 3 e(/2/2/2)/3
(y 1)(y + 1)
r
y
= 3 e/6
(y 1)(y + 1)

3
r
y
= 3 ,
(y 1)(y + 1) 2

r
y
f (y 0) = 3 e(/2(3/2)/2)/3
(y 1)(y + 1)
r
y
= 3 e/2
(y 1)(y + 1)
r
y
=3 .
(y 1)(y + 1)

For the branch cut along the negative imaginary axis,

r
y
f (y + 0) = 3 e(/2(/2)(/2))/3
(y + 1)(y 1)
r
y
= 3 e/6
(y + 1)(y 1)
r
y 3+
= 3 ,
(y + 1)(y 1) 2

345
r
y
f (y 0) = 3 e(/2(/2)(3/2))/3
(y + 1)(y 1)
r
y
= 3 e/2
(y + 1)(y 1)
r
y
= 3 .
(y + 1)(y 1)
Solution 7.31
First we factor the function.
f (z) = ((z 1)(z 2)(z 3))1/2 = (z 1)1/2 (z 2)1/2 (z 3)1/2
There are branch points at z = 1, 2, 3. Now we examine the point at infinity.
     1/2    1/2
1 1 1 1 3/2 1 2 3
f = 1 2 3 = 1 1 1

Since 3/2 has a branch point at = 0 and the rest of the terms are analytic there, f (z) has a branch point at infinity.
The first two sets of branch cuts in Figure 7.33 do not permit us to walk around any of the branch points, including
the point at infinity, and thus make the function single-valued. The third set of branch cuts lets us walk around the
branch points at z = 1 and z = 2 together or if we change our perspective, we would be walking around the branch
points at z = 3 and z = together. Consider a contour in this cut plane that encircles the branch points at z = 1
and z = 2. Since the argument of (z z0 )1/2 changes by when we walk around z0 , the argument of f (z) changes by
2 when we traverse the contour. Thus the value of the function does not change and it is a valid set of branch cuts.
Clearly the fourth set of branch cuts does not make the function single-valued as there are contours that encircle the
branch point at infinity and no other branch points. The other way to see this is to note that the argument of f (z)
changes by 3 as we traverse a contour that goes around the branch points at z = 1, 2, 3 once in the positive direction.
Now to define the branch. We make the preliminary choice of angles,
z 1 = r1 e1 , 0 < 1 < 2,
2
z 2 = r2 e , 0 < 2 < 2,
z 3 = r3 e3 , 0 < 3 < 2.

346
The function is
1/2
f (z) = r1 e1 r2 e2 r3 e3 = r1 r2 r3 e(1 +2 +3 )/2 .

The value of the function at the origin is



f (0) = 6 e(3)/2 = 6,

which is not what we wanted. We will change range of one of the angles to get the desired result.

z 1 = r1 e1 , 0 < 1 < 2,
z 2 = r2 e2 , 0 < 2 < 2,
3
z 3 = r3 e , 2 < 3 < 4.


f (0) = 6 e(5)/2 = 6,

Solution 7.32

2
1/3  1/3  1/3
(z + 2)1/3

w= z 2 (z + 2) z+ 2 z 2

There are branch points at z = 2 and z = 2. If we walk around any one of the branch points once in the positive
direction, the argument of w changes by 2/3 and thus the value of the function changes by e2/3 . If we walk around
3 2/3 = 2. The value of the function is unchanged as
all three branch points then the argument of w changes by
e2 = 1. Thus the branch cut on the real axis from 2 to 2 makes the function single-valued.
Now we define a branch. Let

z 2 = a e , z+ 2 = b e , z + 2 = c e .

We constrain the angles as follows: On the positive real axis, = = . See Figure 7.55.

347
Im(z)

c b a

Re(z)

1/3
Figure 7.55: A branch of ((z 2 2) (z + 2)) .

Now we determine w(2).

 1/3  1/3
w(2) = 2 2 2+ 2 (2 + 2)1/3
0 3
q q
3
2 + 2 e0 4 e0
3
= 2 2e
3

3
= 2 4
= 2.

Note that we didnt have to choose the angle from each of the branch points as zero. Choosing any integer multiple
of 2 would give us the same result.

348
1/3 
 1/3
w(3) = 3 2 3 + 2 (3 + 2)1/3
/3 3
q q
3
3 2 e/3 1 e/3
3
= 3 + 2e

= 7 e
3


3
= 7

The value of the function is


3
w= abc e(++)/3 .

Consider the interval 2 . . . 2 . As we approach the branch cut from above, the function has the value,


r 
3 /3
w= abc e = 3
2x x+ 2 (x + 2) e/3 .

As we approach the branch cut from below, the function has the value,


r 
/3
2 x x + 2 (x + 2) e/3 .
3
w = abc e = 3


Consider the interval 2 . . . 2 . As we approach the branch cut from above, the function has the value,


r 
3 2/3
w= abc e = 3
2x x 2 (x + 2) e2/3 .

As we approach the branch cut from below, the function has the value,


r 
2/3
2 x x 2 (x + 2) e2/3 .
3
w = abc e = 3

349
3
2.5
2
1.5
1
0.5
-1 -0.5 0.5 1

Figure 7.56: The principal branch of the arc cosine, Arccos(x).

Solution 7.33
Arccos(x) is shown in Figure 7.56 for real variables in the range [1 . . . 1].
First we write arccos(z) in terms of log(z). If cos(w) = z, then w = arccos(z).
cos(w) = z
ew + ew
=z
2
(ew )2 2z ew +1 = 0
1/2
ew = z + z 2 1
 1/2 
w = log z + z 2 1
Thus we have  1/2 
2
arccos(z) = log z + z 1 .

Since Arccos(0) = 2 , we must find the branch such that


 1/2 
log 0 + 02 1 =0
log (1)1/2 = 0.


350
Since  
log() = + 2n = + 2n
2 2
and  
log() = + 2n = + 2n
2 2
1/2
we must choose the branch of the square root such that (1) = and the branch of the logarithm such that

log() = 2 .
First we construct the branch of the square root.
1/2
z2 1 = (z + 1)1/2 (z 1)1/2
We see that there are branch points at z = 1 and z = 1. In particular we want the Arccos to be defined for z = x,
x [1 . . . 1]. Hence we introduce branch cuts on the lines < x 1 and 1 x < . Define the local
coordinates
z + 1 = r e , z 1 = e .
With the given branch cuts, the angles have the possible ranges
{} = {. . . , ( . . . ), ( . . . 3), . . .}, {} = {. . . , (0 . . . 2), (2 . . . 4), . . .}.
Now we choose ranges for and and see if we get the desired branch. If not, we choose a different range for one of
the angles. First we choose the ranges
( . . . ), (0 . . . 2).
If we substitute in z = 0 we get
1/2 1/2
02 1 = 1 e0 (1 e )1/2 = e0 e/2 =
Thus we see that this choice of angles gives us the desired branch.
Now we go back to the expression
 1/2 
2
arccos(z) = log z + z 1 .

351
= =0
= =2

1/2
Figure 7.57: Branch cuts and angles for (z 2 1) .

1/2
We have already seen that there are branch points at z = 1 and z = 1 because of (z 2 1) . Now we must
determine if the logarithm introduces additional branch points. The only possibilities for branch points are where the
argument of the logarithm is zero.
1/2
z + z2 1 =0
2 2
z =z 1
0 = 1

We see that the argument of the logarithm is nonzero and thus there are no additional branch points. Introduce the
1/2
variable, w = z + (z 2 1) . What is the image of the branch cuts in the w plane? We parameterize the branch cut
connecting z = 1 and z = + with z = r + 1, r [0 . . . ).
1/2
w = r + 1 + (r + 1)2 1
p
= r + 1 r(r + 2)
 p 
= r 1 r 1 + 2/r + 1
 p   p 
r 1 + 1 + 2/r + 1 is the interval [1 . . . ); r 1 1 + 2/r + 1 is the interval (0 . . . 1]. Thus we see that this
branch cut is mapped to the interval (0 . . . ) in the w plane. Similarly, we could show that the branch cut ( . . .1]

352
in the z plane is mapped to ( . . . 0) in the w plane. In the w plane there is a branch cut along the real w axis
from to . Thus cut makes the logarithm single-valued. For the branch of the square root that we chose, all the
points in the z plane get mapped to the upper half of the w plane.
With the branch cuts we have introduced so far and the chosen branch of the square root we have
 1/2 
2
arccos(0) = log 0 + 0 1
= log
 
= + 2n
2

= + 2n
2
Choosing the n = 0 branch of the logarithm will give us Arccos(z). We see that we can write
 1/2 
Arccos(z) = Log z + z 2 1 .

Solution 7.34 1/2


We consider the function f (z) = z 1/2 1 . First note that z 1/2 has a branch point at z = 0. We place a branch
cut on the negative real axis to make it single valued. f (z) will have a branch point where z 1/2 1 = 0. This occurs
at z = 1 on the branch of z 1/2 on which 11/2 = 1. (11/2 has the value 1 on one branch of z 1/2 and 1 on the other
branch.) For this branch we introduce a branch cut connecting z = 1 with the point at infinity. (See Figure 7.58.)

1/2 1/2
1 =1 1 =-1

1/2
Figure 7.58: Branch cuts for z 1/2 1 .

353
Solution 7.35
The distance between the end of rod a and the end of rod c is b. In the complex plane, these points are a e and
l + c e , respectively. We write this out mathematically.

l + c e a e = b
l + c e a e l + c e a e = b2
 

l2 + cl e al e +cl e +c2 ac e() al e ac e() +a2 = b2


1 2
b a2 c 2 l 2

cl cos ac cos( ) al cos =
2
This equation relates the two angular positions. One could differentiate the equation to relate the velocities and
accelerations.
Solution 7.36
1. Let w = u + v. First we do the strip: |<(z)| < 1. Consider the vertical line: z = c + y, y R. This line is
mapped to
w = 2(c + y)2
w = 2c2 2y 2 + 4cy
u = 2c2 2y 2 , v = 4cy
This is a parabola that opens to the left. For the case c = 0 it is the negative u axis. We can parametrize the
curve in terms of v.
1
u = 2c2 2 v 2 , v R
8c
The boundaries of the region are both mapped to the parabolas:
1
u = 2 v2, v R.
8
The image of the mapping is
 
1 2
w = u + v : v R and u < 2 v .
8

354
Note that the mapping is two-to-one.
Now we do the strip 1 < =(z) < 2. Consider the horizontal line: z = x + c, x R. This line is mapped to

w = 2(x + c)2
w = 2x2 2c2 + 4cx
u = 2x2 2c2 , v = 4cx

This is a parabola that opens upward. We can parametrize the curve in terms of v.
1 2
u= 2
v 2c2 , vR
8c
The boundary =(z) = 1 is mapped to
1
u = v 2 2, v R.
8
The boundary =(z) = 2 is mapped to
1 2
u= v 8, vR
32
The image of the mapping is
 
1 2 1 2
w = u + v : v R and v 8<u< v 2 .
32 8

2. We write the transformation as


z+1 2
=1+ .
z1 z1
Thus we see that the transformation is the sequence:

(a) translation by 1
(b) inversion

355
(c) magnification by 2
(d) translation by 1
Consider the strip |<(z)| < 1. The translation by 1 maps this to 2 < <(z) < 0. Now we do the inversion.
The left edge, <(z) = 0, is mapped to itself. The right edge, <(z) = 2, is mapped to the circle |z +1/4| = 1/4.
Thus the current image is the left half plane minus a circle:

1 1
<(z) < 0 and z + > .
4 4
The magnification by 2 yields
1 1
<(z) < 0 and z + > .
2 2
The final step is a translation by 1.

1 1
<(z) < 1 and z > .
2 2

Now consider the strip 1 < =(z) < 2. The translation by 1 does not change the domain. Now we do the
inversion. The bottom edge, =(z) = 1, is mapped to the circle |z + /2| = 1/2. The top edge, =(z) = 2, is
mapped to the circle |z + /4| = 1/4. Thus the current image is the region between two circles:
1 1
z + < and z + > .

2 2 4 4
The magnification by 2 yields 1
|z + | < 1 and z + > .

2 2
The final step is a translation by 1.
1
|z 1 + | < 1 and z 1 + > .

2 2

356
Solution 7.37
1. There is a simple pole at z = 2. The function has a branch point at z = 1. Since this is the only branch
point in the finite complex plane there is also a branch point at infinity. We can verify this with the substitution
z = 1/.

(1/ + 1)1/2
 
1
f =
1/ + 2
(1 + )1/2
1/2
=
1 + 2

Since f (1/) has a branch point at = 0, f (z) has a branch point at infinity.

2. cos z is an entire function with an essential singularity at infinity. Thus f (z) has singularities only where 1/(1 + z)
has singularities. 1/(1 + z) has a first order pole at z = 1. It is analytic everywhere else, including the point at
infinity. Thus we conclude that f (z) has an essential singularity at z = 1 and is analytic elsewhere. To explicitly
show that z = 1 is an essential singularity, we can find the Laurent series expansion of f (z) about z = 1.

(1)n
 
1 X
cos = (z + 1)2n
1+z n=0
(2n)!

3. 1 ez has simple zeros at z = 2n, n Z. Thus f (z) has second order poles at those points.
The point at infinity is a non-isolated singularity. To justify this: Note that
1
f (z) =
(1 ez )2
1
has second order poles at z = 2n, n Z. This means that f (1/) has second order poles at = 2n , n Z.
These second order poles get arbitrarily close to = 0. There is no deleted neighborhood around = 0 in which
f (1/) is analytic. Thus the point = 0, (z = ), is a non-isolated singularity. There is no Laurent series
expansion about the point = 0, (z = ).

357
The point at infinity is neither a branch point nor a removable singularity. It is not a pole either. If it were, there
would be an n such that limz z n f (z) = const 6= 0. Since z n f (z) has second order poles in every deleted
neighborhood of infinity, the above limit does not exist. Thus we conclude that the point at infinity is an essential
singularity.

Solution 7.38
We write sinh z in Cartesian form.

w = sinh z = sinh x cos y + cosh x sin y = u + v

Consider the line segment x = c, y (0 . . . ). Its image is

{sinh c cos y + cosh c sin y | y (0 . . . )}.

This is the parametric equation for the upper half of an ellipse. Also note that u and v satisfy the equation for an
ellipse.
u2 v2
+ =1
sinh2 c cosh2 c
The ellipse starts at the point (sinh(c), 0), passes through the point (0, cosh(c)) and ends at (sinh(c), 0). As c varies
from zero to or from zero to , the semi-ellipses cover the upper half w plane. Thus the mapping is 2-to-1.
Consider the infinite line y = c, x ( . . . ).Its image is

{sinh x cos c + cosh x sin c | x ( . . . )}.

This is the parametric equation for the upper half of a hyperbola. Also note that u and v satisfy the equation for a
hyperbola.
u2 v2
2 + =1
cos c sin2 c
As c varies from 0 to /2 or from /2 to , the semi-hyperbola cover the upper half w plane. Thus the mapping is
2-to-1.

358
We look for branch points of sinh1 w.

w = sinh z
ez ez
w=
2
e2z 2w ez 1 = 0
1/2
ez = w + w 2 + 1
z = log w + (w )1/2 (w + )1/2


1/2
There are branch points at w = . Since w + (w2 + 1) is nonzero and finite in the finite complex plane, the
logarithm does not introduce any branch points in the finite plane. Thus the only branch point in the upper half w
plane is at w = . Any branch cut that connects w = with the boundary of =(w) > 0 will separate the branches
under the inverse mapping.
Consider the line y = /4. The image under the mapping is the upper half of the hyperbola

2u2 + 2v 2 = 1.

Consider the segment x = 1.The image under the mapping is the upper half of the ellipse

u2 v2
+ = 1.
sinh2 1 cosh2 1

359
Chapter 8

Analytic Functions

Students need encouragement. So if a student gets an answer right, tell them it was a lucky guess. That way, they
develop a good, lucky feeling.1
-Jack Handey

8.1 Complex Derivatives


Functions of a Real Variable. The derivative of a function of a real variable is
d f (x + x) f (x)
f (x) = lim .
dx x0 x
If the limit exists then the function is differentiable at the point x. Note that x can approach zero from above or
below. The limit cannot depend on the direction in which x vanishes.
Consider f (x) = |x|. The function is not differentiable at x = 0 since
|0 + x| |0|
lim + =1
x0 x
1
Quote slightly modified.

360
and
|0 + x| |0|
lim = 1.
x0 x

Analyticity. The complex derivative, (or simply derivative if the context is clear), is defined,

d f (z + z) f (z)
f (z) = lim .
dz z0 z
The complex derivative exists if this limit exists. This means that the value of the limit is independent of the manner
in which z 0. If the complex derivative exists at a point, then we say that the function is complex differentiable
there.
A function of a complex variable is analytic at a point z0 if the complex derivative exists in a neighborhood about
that point. The function is analytic in an open set if it has a complex derivative at each point in that set. Note that
complex differentiable has a different meaning than analytic. Analyticity refers to the behavior of a function on an open
set. A function can be complex differentiable at isolated points, but the function would not be analytic at those points.
Analytic functions are also called regular or holomorphic. If a function is analytic everywhere in the finite complex
plane, it is called entire.

Example 8.1.1 Consider z n , n Z+ , Is the function differentiable? Is it analytic? What is the value of the derivative?
We determine differentiability by trying to differentiate the function. We use the limit definition of differentiation.
We will use Newtons binomial formula to expand (z + z)n .

d n (z + z)n z n
z = lim
dz z0
 z 
n(n1) n2
z n + nz n1 z + 2
z z 2 + + z n z n
= lim
z0
 z 
n1 n(n 1) n2
= lim nz + z z + + z n1
z0 2
= nz n1

361
The derivative exists everywhere. The function is analytic in the whole complex plane so it is entire. The value of the
d
derivative is dz = nz n1 .

Example 8.1.2 We will show that f (z) = z is not differentiable. Consider its derivative.

d f (z + z) f (z)
f (z) = lim .
dz z0 z

d z + z z
z = lim
dz z0 z
z
= lim
z0 z

First we take z = x and evaluate the limit.


x
lim =1
x0 x

Then we take z = y.
y
lim = 1
y0 y

Since the limit depends on the way that z 0, the function is nowhere differentiable. Thus the function is not
analytic.

Complex Derivatives in Terms of Plane Coordinates. Let z = (, ) be a system of coordinates in


the complex plane. (For example, we could have Cartesian coordinates z = (x, y) = x + y or polar coordinates
z = (r, ) = r e ). Let f (z) = (, ) be a complex-valued function. (For example we might have a function in the
form (x, y) = u(x, y) + v(x, y) or (r, ) = R(r, ) e(r,) .) If f (z) = (, ) is analytic, its complex derivative is

362
equal to the derivative in any direction. In particular, it is equal to the derivatives in the coordinate directions.
 1
df f (z + z) f (z) ( + , ) (, )
= lim = lim
=
dz 0,=0 z 0


 1
df f (z + z) f (z) (, + ) (, )
= lim = lim
=
dz =0,0 z 0


Example 8.1.3 Consider the Cartesian coordinates z = x + y. We write the complex derivative as derivatives in the
coordinate directions for f (z) = (x, y).
 1
df (x + y)
= =
dz x x x
 1
df (x + y)
= =
dz y y y
We write this in operator notation.
d
= = .
dz x y
d n
Example 8.1.4 In Example 8.1.1 we showed that z n , n Z+ , is an entire function and that dz
z = nz n1 . Now we
corroborate this by calculating the complex derivative in the Cartesian coordinate directions.
d n
z = (x + y)n
dz x
= n(x + y)n1
= nz n1

d n
z = (x + y)n
dz y
= n(x + y)n1
= nz n1

363
Complex Derivatives are Not the Same as Partial Derivatives Recall from calculus that

f g s g t
f (x, y) = g(s, t) = +
x s x t x

Do not make the mistake of using a similar formula for functions of a complex variable. If f (z) = (x, y) then

df x y
6= + .
dz x z y z

d
This is because the dz operator means The derivative in any direction in the complex plane. Since f (z) is analytic,
0
f (z) is the same no matter in which direction we take the derivative.

Rules of Differentiation. For an analytic function defined in terms of z we can calculate the complex derivative
using all the usual rules of differentiation that we know from calculus like the product rule,

d
f (z)g(z) = f 0 (z)g(z) + f (z)g 0 (z),
dz

or the chain rule,


d
f (g(z)) = f 0 (g(z))g 0 (z).
dz

This is because the complex derivative derives its properties from properties of limits, just like its real variable counterpart.

364
Result 8.1.1 The complex derivative is,
d f (z + z) f (z)
f (z) = lim .
dz z0 z
The complex derivative is defined if the limit exists and is independent of the manner in which
z 0. A function is analytic at a point if the complex derivative exists in a neighborhood
of that point.
Let z = (, ) define coordinates in the complex plane. The complex derivative in the
coordinate directions is  1  1
d
= = .
dz
In Cartesian coordinates, this is
d
= = .
dz x y
In polar coordinates, this is
d
= e = e
dz r r
Since the complex derivative is defined with the same limit formula as real derivatives, all the
rules from the calculus of functions of a real variable may be used to differentiate functions
of a complex variable.

d n
Example 8.1.5 We have shown that z n , n Z+ , is an entire function. Now we corroborate that dz
z = nz n1 by

365
calculating the complex derivative in the polar coordinate directions.
d n
z = e rn en
dz r

= e nrn1 en
= nrn1 e(n1)
= nz n1

d n
z = e rn en
dz r
n
= e r n en
r
= nrn1 e(n1)
= nz n1

Analytic Functions can be Written in Terms of z. Consider an analytic function expressed in terms of x and
y, (x, y). We can write as a function of z = x + y and z = x y.
 
z+z zz
f (z, z) = ,
2 2
We treat z and z as independent variables. We find the partial derivatives with respect to these variables.
 
x y 1
= + =
z z x z y 2 x y
 
x y 1
= + = +
z z x z y 2 x y
Since is analytic, the complex derivatives in the x and y directions are equal.

=
x y

366
The partial derivative of f (z, z) with respect to z is zero.
 
f 1
= + =0
z 2 x y
Thus f (z, z) has no functional dependence on z, it can be written as a function of z alone.
If we were considering an analytic function expressed in polar coordinates (r, ), then we could write it in Cartesian
coordinates with the substitutions: p
r = x2 + y 2 , = arctan(x, y).
Thus we could write (r, ) as a function of z alone.

Result 8.1.2 Any analytic function (x, y) or (r, ) can be written as a function of z alone.

8.2 Cauchy-Riemann Equations


If we know that a function is analytic, then we have a convenient way of determining its complex derivative. We just
express the complex derivative in terms of the derivative in a coordinate direction. However, we dont have a nice way
of determining if a function is analytic. The definition of complex derivative in terms of a limit is cumbersome to work
with. In this section we remedy this problem.

A necessary condition for analyticity. Consider a function f (z) = (x, y). If f (z) is analytic, the complex
derivative is equal to the derivatives in the coordinate directions. We equate the derivatives in the x and y directions
to obtain the Cauchy-Riemann equations in Cartesian coordinates.
x = y (8.1)
This equation is a necessary condition for the analyticity of f (z).
Let (x, y) = u(x, y) + v(x, y) where u and v are real-valued functions. We equate the real and imaginary parts
of Equation 8.1 to obtain another form for the Cauchy-Riemann equations in Cartesian coordinates.
ux = v y , uy = vx .

367
Note that this is a necessary and not a sufficient condition for analyticity of f (z). That is, u and v may satisfy the
Cauchy-Riemann equations but f (z) may not be analytic. At this point, Cauchy-Riemann equations give us an easy
test for determining if a function is not analytic.

Example 8.2.1 In Example 8.1.2 we showed that z is not analytic using the definition of complex differentiation. Now
we obtain the same result using the Cauchy-Riemann equations.

z = x y
ux = 1, vy = 1

We see that the first Cauchy-Riemann equation is not satisfied; the function is not analytic at any point.

A sufficient condition for analyticity. A sufficient condition for f (z) = (x, y) to be analytic at a point
z0 = (x0 , y0 ) is that the partial derivatives of (x, y) exist and are continuous in some neighborhood of z0 and satisfy
the Cauchy-Riemann equations there. If the partial derivatives of exist and are continuous then

(x + x, y + y) = (x, y) + xx (x, y) + yy (x, y) + o(x) + o(y).

Here the notation o(x) means terms smaller than x. We calculate the derivative of f (z).

f (z + z) f (z)
f 0 (z) = lim
z0 z
(x + x, y + y) (x, y)
= lim
x,y0 x + y
(x, y) + xx (x, y) + yy (x, y) + o(x) + o(y) (x, y)
= lim
x,y0 x + y
xx (x, y) + yy (x, y) + o(x) + o(y)
= lim
x,y0 x + y

368
Here we use the Cauchy-Riemann equations.

(x + y)x (x, y) o(x) + o(y)


= lim + lim
x,y0 x + y x,y0 x + y
= x (x, y)

Thus we see that the derivative is well defined.

Cauchy-Riemann Equations in General Coordinates Let z = (, ) be a system of coordinates in the


complex plane. Let (, ) be a function which we write in terms of these coordinates, A necessary condition for
analyticity of (, ) is that the complex derivatives in the coordinate directions exist and are equal. Equating the
derivatives in the and directions gives us the Cauchy-Riemann equations.

 1  1

=

We could separate this into two equations by equating the real and imaginary parts or the modulus and argument.

369
Result 8.2.1 A necessary condition for analyticity of (, ), where z = (, ), at z = z0
is that the Cauchy-Riemann equations are satisfied in a neighborhood of z = z0 .
 1  1

= .

(We could equate the real and imaginary parts or the modulus and argument of this to obtain
two equations.) A sufficient condition for analyticity of f (z) is that the Cauchy-Riemann
equations hold and the first partial derivatives of exist and are continuous in a neighborhood
of z = z0 .
Below are the Cauchy-Riemann equations for various forms of f (z).

f (z) = (x, y), x = y


f (z) = u(x, y) + v(x, y), ux = vy , uy = vx

f (z) = (r, ), r =
r
1
f (z) = u(r, ) + v(r, ), ur = v , u = rvr
r
R 1
f (z) = R(r, ) e(r,) , Rr = , R = Rr
r r
f (z) = R(x, y) e(x,y) , Rx = Ry , Ry = Rx

Example 8.2.2 Consider the Cauchy-Riemann equations for f (z) = u(r, ) + v(r, ). From Exercise 8.3 we know
that the complex derivative in the polar coordinate directions is
d
= e = e .
dz r r

370
From Result 8.2.1 we have the equation,


e [u + v] = e [u + v].
r r
We multiply by e and equate the real and imaginary components to obtain the Cauchy-Riemann equations.

1
ur = v , u = rvr
r
Example 8.2.3 Consider the exponential function.

ez = (x, y) = ex (cos y + sin(y))

We use the Cauchy-Riemann equations to show that the function is entire.

x = y
ex (cos y + sin(y)) = ex ( sin y + cos(y))
ex (cos y + sin(y)) = ex (cos y + sin(y))

Since the function satisfies the Cauchy-Riemann equations and the first partial derivatives are continuous everywhere
in the finite complex plane, the exponential function is entire.
Now we find the value of the complex derivative.

d z
e = = ex (cos y + sin(y)) = ez
dz x
The differentiability of the exponential function implies the differentiability of the trigonometric functions, as they can
be written in terms of the exponential.

In Exercise 8.13 you can show that the logarithm log z is differentiable for z 6= 0. This implies the differentiability
of z and the inverse trigonometric functions as they can be written in terms of the logarithm.

371
Example 8.2.4 We compute the derivative of z z .

d z d z log z
(z ) = e
dz dz
= (1 + log z) ez log z
= (1 + log z)z z
= z z + z z log z

8.3 Harmonic Functions


A function u is harmonic if its second partial derivatives exist, are continuous and satisfy Laplaces equation u = 0.2
(In Cartesian coordinates the Laplacian is u uxx + uyy .) If f (z) = u + v is an analytic function then u and v are
harmonic functions. To see why this is so, we start with the Cauchy-Riemann equations.

ux = v y , uy = vx

We differentiate the first equation with respect to x and the second with respect to y. (We assume that u and v are
twice continuously differentiable. We will see later that they are infinitely differentiable.)

uxx = vxy , uyy = vyx

Thus we see that u is harmonic.


u uxx + uyy = vxy vyx = 0
One can use the same method to show that v = 0.

If u is harmonic on some simply-connected domain, then there exists a harmonic function v such that f (z) = u + v
is analytic in the domain. v is called the harmonic conjugate of u. The harmonic conjugate is unique up to an additive
2
The capital Greek letter is used to denote the Laplacian, like u(x, y), and differentials, like x.

372
constant. To demonstrate this, let w be another harmonic conjugate of u. Both the pair u and v and the pair u and
w satisfy the Cauchy-Riemann equations.

ux = v y , uy = vx , ux = wy , uy = wx

We take the difference of these equations.

vx wx = 0, vy wy = 0

On a simply connected domain, the difference between v and w is thus a constant.


To prove the existence of the harmonic conjugate, we first write v as an integral.
Z (x,y)
v(x, y) = v (x0 , y0 ) + vx dx + vy dy
(x0 ,y0 )

On a simply connected domain, the integral is path independent and defines a unique v in terms of vx and vy . We use
the Cauchy-Riemann equations to write v in terms of ux and uy .
Z (x,y)
v(x, y) = v (x0 , y0 ) + uy dx + ux dy
(x0 ,y0 )

Changing the starting point (x0 , y0 ) changes v by an additive constant. The harmonic conjugate of u to within an
additive constant is Z
v(x, y) = uy dx + ux dy.

This proves the existence3 of the harmonic conjugate. This is not the formula one would use to construct the harmonic
conjugate of a u. One accomplishes this by solving the Cauchy-Riemann equations.
3
A mathematician returns to his office to find that a cigarette tossed in the trash has started a small fire. Being calm and a
quick thinker he notes that there is a fire extinguisher by the window. He then closes the door and walks away because the solution
exists.

373
Result 8.3.1 If f (z) = u + v is an analytic function then u and v are harmonic functions.
That is, the Laplacians of u and v vanish u = v = 0. The Laplacian in Cartesian and
polar coordinates is
2 2 1 2
 
1
= 2 + 2, = r + 2 2.
x y r r r r
Given a harmonic function u in a simply connected domain, there exists a harmonic function
v, (unique up to an additive constant), such that f (z) = u + v is analytic in the domain.
One can construct v by solving the Cauchy-Riemann equations.

Example 8.3.1 Is x2 the real part of an analytic function?


The Laplacian of x2 is
[x2 ] = 2 + 0
x2 is not harmonic and thus is not the real part of an analytic function.

Example 8.3.2 Show that u = ex (x sin y y cos y) is harmonic.

u
= ex sin y ex (x sin y y cos y)
x
= ex sin y x ex sin y + y ex cos y

2u
= ex sin y ex sin y + x ex sin y y ex cos y
x2
= 2 ex sin y + x ex sin y y ex cos y

u
= ex (x cos y cos y + y sin y)
y

374
2u
= ex (x sin y + sin y + y cos y + sin y)
y 2
= x ex sin y + 2 ex sin y + y ex cos y
2u 2u
Thus we see that x2
+ y 2
= 0 and u is harmonic.

Example 8.3.3 Consider u = cos x cosh y. This function is harmonic.

uxx + uyy = cos x cosh y + cos x cosh y = 0

Thus it is the real part of an analytic function, f (z). We find the harmonic conjugate, v, with the Cauchy-Riemann
equations. We integrate the first Cauchy-Riemann equation.

vy = ux = sin x cosh y
v = sin x sinh y + a(x)

Here a(x) is a constant of integration. We substitute this into the second Cauchy-Riemann equation to determine a(x).

vx = uy
cos x sinh y + a0 (x) = cos x sinh y
a0 (x) = 0
a(x) = c

Here c is a real constant. Thus the harmonic conjugate is

v = sin x sinh y + c.

The analytic function is


f (z) = cos x cosh y sin x sinh y + c
We recognize this as
f (z) = cos z + c.

375
Example 8.3.4 Here we consider an example that demonstrates the need for a simply connected domain. Consider
u = Log r in the multiply connected domain, r > 0. u is harmonic.
1 2
 
1
Log r = r Log r + 2 2 Log r = 0
r r r r
We solve the Cauchy-Riemann equations to try to find the harmonic conjugate.
1
ur = v , u = rvr
r
vr = 0, v = 1
v =+c
We are able to solve for v, but it is multi-valued. Any single-valued branch of that we choose will not be continuous
on the domain. Thus there is no harmonic conjugate of u = Log r for the domain r > 0.
If we had instead considered the simply-connected domain r > 0, | arg(z)| < then the harmonic conjugate would
be v = Arg(z) + c. The corresponding analytic function is f (z) = Log z + c.

Example 8.3.5 Consider u = x3 3xy 2 + x. This function is harmonic.


uxx + uyy = 6x 6x = 0
Thus it is the real part of an analytic function, f (z). We find the harmonic conjugate, v, with the Cauchy-Riemann
equations. We integrate the first Cauchy-Riemann equation.
vy = ux = 3x2 3y 2 + 1
v = 3x2 y y 3 + y + a(x)
Here a(x) is a constant of integration. We substitute this into the second Cauchy-Riemann equation to determine a(x).
vx = uy
6xy + a0 (x) = 6xy
a0 (x) = 0
a(x) = c

376
Here c is a real constant. The harmonic conjugate is

v = 3x2 y y 3 + y + c.

The analytic function is

f (z) = x3 3xy 2 + x + 3x2 y y 3 + y + c




f (z) = x3 + 3x2 y 3xy 2 y 2 + x + y + c


f (z) = z 3 + z + c

8.4 Singularities
Any point at which a function is not analytic is called a singularity. In this section we will classify the different flavors
of singularities.

Result 8.4.1 Singularities. If a function is not analytic at a point, then that point is a
singular point or a singularity of the function.

8.4.1 Categorization of Singularities


Branch Points. If f (z) has a branch point at z0 , then we cannot define a branch of f (z) that is continuous in a
neighborhood of z0 . Continuity is necessary for analyticity. Thus all branch points are singularities. Since function are
discontinuous across branch cuts, all points on a branch cut are singularities.

Example 8.4.1 Consider f(z) = z 3/2 . The origin and infinity are branch points and are thus singularities of f (z). We
choose the branch g(z) = z 3 . All the points on the negative real axis, including the origin, are singularities of g(z).

Removable Singularities.

377
Example 8.4.2 Consider
sin z
f (z) = .
z
This function is undefined at z = 0 because f (0) is the indeterminate form 0/0. f (z) is analytic everywhere in the
finite complex plane except z = 0. Note that the limit as z 0 of f (z) exists.

sin z cos z
lim = lim =1
z0 z z0 1
If we were to fill in the hole in the definition of f (z), we could make it differentiable at z = 0. Consider the function
(
sin z
z
z 6= 0,
g(z) =
1 z = 0.

We calculate the derivative at z = 0 to verify that g(z) is analytic there.

f (0) f (z)
f 0 (0) = lim
z0 z
1 sin(z)/z
= lim
z0 z
z sin(z)
= lim
z0 z2
1 cos(z)
= lim
z0 2z
sin(z)
= lim
z0 2
=0

We call the point at z = 0 a removable singularity of sin(z)/z because we can remove the singularity by defining the
value of the function to be its limiting value there.

378
Consider a function f (z) that is analytic in a deleted neighborhood of z = z0 . If f (z) is not analytic at z0 , but
limzz0 f (z) exists, then the function has a removable singularity at z0 . The function
(
f (z) z 6= z0
g(z) =
limzz0 f (z) z = z0
is analytic in a neighborhood of z = z0 . We show this by calculating g 0 (z0 ).
g (z0 ) g(z)
g 0 (z0 ) = lim
zz0 z0 z
0
g (z)
= lim
zz0 1
0
= lim f (z)
zz0

This limit exists because f (z) is analytic in a deleted neighborhood of z = z0 .

Poles. If a function f (z) behaves like c/ (z z0 )n near z = z0 then the function has an nth order pole at that point.
More mathematically we say
lim (z z0 )n f (z) = c 6= 0.
zz0
We require the constant c to be nonzero so we know that it is not a pole of lower order. We can denote a removable
singularity as a pole of order zero.
Another way to say that a function has an nth order pole is that f (z) is not analytic at z = z0 , but (z z0 )n f (z)
is either analytic or has a removable singularity at that point.

Example 8.4.3 1/ sin (z 2 ) has a second order pole at z = 0 and first order poles at z = (n)1/2 , n Z .
z2 2z
lim 2
= lim
z0 sin (z ) z0 2z cos (z 2 )
2
= lim
z0 2 cos (z 2 ) 4z 2 sin (z 2 )
=1

379
z (n)1/2 1
lim 2
= lim
z(n)1/2 sin (z ) z(n) 1/2 2z cos (z 2 )
1
=
2(n) (1)n
1/2

Example 8.4.4 e1/z is singular at z = 0. The function is not analytic as limz0 e1/z does not exist. We check if the
function has a pole of order n at z = 0.

e
lim z n e1/z = lim
z0 n

e
= lim
n!

Since the limit does not exist for any value of n, the singularity is not a pole. We could say that e1/z is more singular
than any power of 1/z.

Essential Singularities. If a function f (z) is singular at z = z0 , but the singularity is not a branch point, or a
pole, the the point is an essential singularity of the function.

The point at infinity. We can consider the point at infinity z by making the change of variables z = 1/
and considering 0. If f (1/) is analytic at = 0 then f (z) is analytic at infinity. We have encountered branch
points at infinity before (Section 7.9). Assume that f (z) is not analytic at infinity. If limz f (z) exists then f (z) has
a removable singularity at infinity. If limz f (z)/z n = c 6= 0 then f (z) has an nth order pole at infinity.

380
Result 8.4.2 Categorization of Singularities. Consider a function f (z) that has a singu-
larity at the point z = z0 . Singularities come in four flavors:
Branch Points. Branch points of multi-valued functions are singularities.
Removable Singularities. If limzz0 f (z) exists, then z0 is a removable singularity. It
is thus named because the singularity could be removed and thus the function made
analytic at z0 by redefining the value of f (z0 ).
Poles. If limzz0 (z z0 )n f (z) = const 6= 0 then f (z) has an nth order pole at z0 .
Essential Singularities. Instead of defining what an essential singularity is, we say what it
is not. If z0 neither a branch point, a removable singularity nor a pole, it is an essential
singularity.

A pole may be called a non-essential singularity. This is because multiplying the function by an integral power of
z z0 will make the function analytic. Then an essential singularity is a point z0 such that there does not exist an n
such that (z z0 )n f (z) is analytic there.

8.4.2 Isolated and Non-Isolated Singularities

Result 8.4.3 Isolated and Non-Isolated Singularities. Suppose f (z) has a singularity at
z0 . If there exists a deleted neighborhood of z0 containing no singularities then the point is
an isolated singularity. Otherwise it is a non-isolated singularity.

381
If you dont like the abstract notion of a deleted neighborhood, you can work with a deleted circular neighborhood.
However, this will require the introduction of more math symbols and a Greek letter. z = z0 is an isolated singularity
if there exists a > 0 such that there are no singularities in 0 < |z z0 | < .

Example 8.4.5 We classify the singularities of f (z) = z/ sin z.


z has a simple zero at z = 0. sin z has simple zeros at z = n. Thus f (z) has a removable singularity at z = 0
and has first order poles at z = n for n Z . We can corroborate this by taking limits.

z 1
lim f (z) = lim = lim =1
z0 z0 sin z z0 cos z

(z n)z
lim (z n)f (z) = lim
zn zn sin z
2z n
= lim
zn cos z
n
=
(1)n
6= 0

Now to examine the behavior at infinity. There is no neighborhood of infinity that does not contain first order
poles of f (z). (Another way of saying this is that there does not exist an R such that there are no singularities in
R < |z| < .) Thus z = is a non-isolated singularity.
We could also determine this by setting = 1/z and examining the point = 0. f (1/) has first order poles at
= 1/(n) for n Z \ {0}. These first order poles come arbitrarily close to the point = 0 There is no deleted
neighborhood of = 0 which does not contain singularities. Thus = 0, and hence z = is a non-isolated singularity.
The point at infinity is an essential singularity. It is certainly not a branch point or a removable singularity. It is not a
pole, because there is no n such that limz z n f (z) = const 6= 0. z n f (z) has first order poles in any neighborhood
of infinity, so this limit does not exist.

382
8.5 Application: Potential Flow
Example 8.5.1 We consider 2 dimensional uniform flow in a given direction. The flow corresponds to the complex
potential
(z) = v0 e0 z,
where v0 is the fluid speed and 0 is the direction. We find the velocity potential and stream function .

(z) = +
= v0 (cos(0 )x + sin(0 )y), = v0 ( sin(0 )x + cos(0 )y)

These are plotted in Figure 8.1 for 0 = /6.

1 1
0 1 0 1
-1 0.5 -1 0.5
-1 0 -1 0
-0.5 -0.5
0 -0.5 0 -0.5
0.5 0.5
1-1 1-1

Figure 8.1: The velocity potential and stream function for (z) = v0 e0 z.

Next we find the stream lines, = c.

v0 ( sin(0 )x + cos(0 )y) = c


c
y= + tan(0 )x
v0 cos(0 )

383
1

0.5

-0.5

-1
-1 -0.5 0 0.5 1

Figure 8.2: Streamlines for = v0 ( sin(0 )x + cos(0 )y).

Figure 8.2 shows how the streamlines go straight along the 0 direction. Next we find the velocity field.

v =
v = x x + y y
v = v0 cos(0 )x + v0 sin(0 )y

The velocity field is shown in Figure 8.3.

Example 8.5.2 Steady, incompressible, inviscid, irrotational flow is governed by the Laplace equation. We consider
flow around an infinite cylinder of radius a. Because the flow does not vary along the axis of the cylinder, this is a
two-dimensional problem. The flow corresponds to the complex potential

a2
 
(z) = v0 z+ .
z

384
Figure 8.3: Velocity field and velocity direction field for = v0 (cos(0 )x + sin(0 )y).

We find the velocity potential and stream function .

(z) = +
2
a2
   
a
= v0 r+ cos , = v0 r sin
r r

These are plotted in Figure 8.4.

385
 
a2
Figure 8.4: The velocity potential and stream function for (z) = v0 z + z
.

Next we find the stream lines, = c.

a2
 
v0 r sin = c
r
p
c c2 + 4v0 sin2
r=
2v0 sin

Figure 8.5 shows how the streamlines go around the cylinder. Next we find the velocity field.

386
 
a2
Figure 8.5: Streamlines for = v0 r r
sin .

v =

v = r r +
r 
a2 a2
  
v = v0 1 2 cos r v0 1+ 2 sin
r r

The velocity field is shown in Figure 8.6.

387
 
a2
Figure 8.6: Velocity field and velocity direction field for = v0 r + r
cos .

8.6 Exercises
Complex Derivatives
Exercise 8.1
Consider two functions f (z) and g(z) analytic at z0 with f (z0 ) = g(z0 ) = 0 and g 0 (z0 ) 6= 0.
1. Use the definition of the complex derivative to justify LHospitals rule:
f (z) f 0 (z0 )
lim = 0
zz0 g(z) g (z0 )
2. Evaluate the limits
1 + z2 sinh(z)
lim , lim
z 2 + 2z 6 z ez +1

388
Hint, Solution

Exercise 8.2
Show that if f (z) is analytic and (x, y) = f (z) is twice continuously differentiable then f 0 (z) is analytic.
Hint, Solution

Exercise 8.3
Find the complex derivative in the coordinate directions for f (z) = (r, ).
Hint, Solution

Exercise 8.4
Show that the following functions are nowhere analytic by checking where the derivative with respect to z exists.

1. sin x cosh y cos x sinh y

2. x2 y 2 + x + (2xy y)

Hint, Solution

Exercise 8.5
f (z) is analytic for all z, (|z| < ). f (z1 + z2 ) = f (z1 ) f (z2 ) for all z1 and z2 . (This is known as a functional
equation). Prove that f (z) = exp (f 0 (0)z).
Hint, Solution

Cauchy-Riemann Equations
Exercise 8.6
If f (z) is analytic in a domain and has a constant real part, a constant imaginary part, or a constant modulus, show
that f (z) is constant.
Hint, Solution

389
Exercise 8.7
Show that the function ( 4
ez for z 6= 0,
f (z) =
0 for z = 0.
satisfies the Cauchy-Riemann equations everywhere, including at z = 0, but f (z) is not analytic at the origin.
Hint, Solution
Exercise 8.8
Find the Cauchy-Riemann equations for the following forms.
1. f (z) = R(r, ) e(r,)
2. f (z) = R(x, y) e(x,y)
Hint, Solution
Exercise 8.9
1. Show that ez is not analytic.

2. f (z) is an analytic function of z. Show that f (z) = f (z) is also an analytic function of z.
Hint, Solution
Exercise 8.10
1. Determine all points z = x + y where the following functions are differentiable with respect to z:
(a) x3 + y 3
x1 y
(b) 2 2

(x 1) + y (x 1)2 + y 2
2. Determine all points z where these functions are analytic.
3. Determine which of the following functions v(x, y) are the imaginary part of an analytic function u(x, y)+v(x, y).
For those that are, compute the real part u(x, y) and re-express the answer as an explicit function of z = x + y:

390
(a) x2 y 2
(b) 3x2 y
Hint, Solution
Exercise 8.11
Let
(
x4/3 y 5/3 +x5/3 y 4/3
x2 +y 2
for z 6= 0,
f (z) =
0 for z = 0.
Show that the Cauchy-Riemann equations hold at z = 0, but that f is not differentiable at this point.
Hint, Solution
Exercise 8.12
Consider the complex function
x3 (1+)y 3 (1)
(
x2 +y 2
for z 6= 0,
f (z) = u + v =
0 for z = 0.
Show that the partial derivatives of u and v with respect to x and y exist at z = 0 and that ux = vy and uy = vx
there: the Cauchy-Riemann equations are satisfied at z = 0. On the other hand, show that

f (z)
lim
z0 z
does not exist, that is, f is not complex-differentiable at z = 0.
Hint, Solution
Exercise 8.13
Show that the logarithm log z is differentiable for z 6= 0. Find the derivative of the logarithm.
Hint, Solution

391
Exercise 8.14
Show that the Cauchy-Riemann equations for the analytic function f (z) = u(r, ) + v(r, ) are

ur = v /r, u = rvr .

Hint, Solution
Exercise 8.15
w = u + v is an analytic function of z. (x, y) is an arbitrary smooth function of x and y. When expressed in terms
of u and v, (x, y) = (u, v). Show that (w0 6= 0)
 1  
dw
= .
u v dz x y

Deduce 2  2
2 2 dw 2

+ = + .
u2 v 2 dz x2 y 2
Hint, Solution
Exercise 8.16 p
Show that the functions defined by f (z) = log |z|+ arg(z) and f (z) = |z| e arg(z)/2 are analytic in the sector |z| > 0,
| arg(z)| < . What are the corresponding derivatives df /dz?
Hint, Solution
Exercise 8.17
Show that the following functions are harmonic. For each one of them find its harmonic conjugate and form the
corresponding holomorphic function.

1. u(x, y) = x Log(r) y arctan(x, y) (r 6= 0)

2. u(x, y) = arg(z) (| arg(z)| < , r 6= 0)

3. u(x, y) = rn cos(n)

392
4. u(x, y) = y/r2 (r 6= 0)

Hint, Solution
Exercise 8.18
1. Use the Cauchy-Riemann equations to determine where the function

f (z) = (x y)2 + 2(x + y)

is differentiable and where it is analytic.

2. Evaluate the derivative of


2 y 2
f (z) = ex (cos(2xy) + sin(2xy))
and describe the domain of analyticity.
Hint, Solution
Exercise 8.19
Consider the function f (z) = u + v with real and imaginary parts expressed in terms of either x and y or r and .

1. Show that the Cauchy-Riemann equations

ux = vy , uy = vx

are satisfied and these partial derivatives are continuous at a point z if and only if the polar form of the Cauchy-
Riemann equations
1 1
ur = v , u = vr
r r
is satisfied and these partial derivatives are continuous there.

2. Show that it is easy to verify that Log z is analytic for r > 0 and < < using the polar form of the
Cauchy-Riemann equations and that the value of the derivative is easily obtained from a polar differentiation
formula.

393
3. Show that in polar coordinates, Laplaces equation becomes
1 1
rr + r + 2 = 0.
r r
Hint, Solution
Exercise 8.20
Determine which of the following functions are the real parts of an analytic function.
1. u(x, y) = x3 y 3
2. u(x, y) = sinh x cos y + x
3. u(r, ) = rn cos(n)
and find f (z) for those that are.
Hint, Solution
Exercise 8.21
Consider steady, incompressible, inviscid, irrotational flow governed by the Laplace equation. Determine the form of
the velocity potential and stream function contours for the complex potentials
1. (z) = (x, y) + (x, y) = log z + log z
2. (z) = log(z 1) + log(z + 1)
Plot and describe the features of the flows you are considering.
Hint, Solution
Exercise 8.22
1. Classify all the singularities (removable, poles, isolated essential, branch points, non-isolated essential) of the
following functions in the extended complex plane
z
(a) 2
z +1

394
1
(b)
sin z
(c) log 1 + z 2


(d) z sin(1/z)
tan1 (z)
(e)
z sinh2 (z)
2. Construct functions that have the following zeros or singularities:

(a) a simple zero at z = and an isolated essential singularity at z = 1.


(b) a removable singularity at z = 3, a pole of order 6 at z = and an essential singularity at z .
Hint, Solution

395
8.7 Hints
Complex Derivatives
Hint 8.1

Hint 8.2
Start with the Cauchy-Riemann equation and then differentiate with respect to x.

Hint 8.3
Read Example 8.1.3 and use Result 8.1.1.

Hint 8.4
Use Result 8.1.1.

Hint 8.5
Take the logarithm of the equation to get a linear equation.

Cauchy-Riemann Equations
Hint 8.6

Hint 8.7

Hint 8.8
For the first part use the result of Exercise 8.3.

Hint 8.9
Use the Cauchy-Riemann equations.

396
Hint 8.10

Hint 8.11
To evaluate ux (0, 0), etc. use the definition of differentiation. Try to find f 0 (z) with the definition of complex
differentiation. Consider z = r e .
Hint 8.12
To evaluate ux (0, 0), etc. use the definition of differentiation. Try to find f 0 (z) with the definition of complex
differentiation. Consider z = r e .
Hint 8.13

Hint 8.14

Hint 8.15

Hint 8.16

Hint 8.17

Hint 8.18

Hint 8.19

397
Hint 8.20

Hint 8.21

Hint 8.22
CONTINUE

398
8.8 Solutions
Complex Derivatives
Solution 8.1
1. We consider LHospitals rule.
f (z) f 0 (z0 )
lim = 0
zz0 g(z) g (z0 )
We start with the right side and show that it is equal to the left side. First we apply the definition of complex
differentiation.
f 0 (z0 ) lim0 f (z0 +)f

(z0 )
lim0 f (z0+)
= =
g 0 (z0 ) lim0 g(z0 +)g(z0 ) lim0 g(z0 +)

Since both of the limits exist, we may take the limits with  = .

f 0 (z0 ) f (z0 + )
= lim
g 0 (z0 ) 0 g(z0 + )
f 0 (z0 ) f (z)
0
= lim
g (z0 ) zz 0 g(z)

This proves LHospitals rule.

2.
1 + z2
 
2z 1
lim 6
= 5
=
z 2 + 2z 12z z= 6

 
sinh(z) cosh(z)
lim = =1
z ez +1 ez z=

399
Solution 8.2
We start with the Cauchy-Riemann equation and then differentiate with respect to x.
x = y
xx = yx
We interchange the order of differentiation.
(x )x = (x )y
(f 0 )x = (f 0 )y
Since f 0 (z) satisfies the Cauchy-Riemann equation and its partial derivatives exist and are continuous, it is analytic.

Solution 8.3
We calculate the complex derivative in the coordinate directions.
 !1
df r e
= = e ,
dz r r r
 !1
df r e
= = e .
dz r

We can write this in operator notation.


d
= e = e
dz r r
Solution 8.4
1. Consider f (x, y) = sin x cosh y cos x sinh y. The derivatives in the x and y directions are
f
= cos x cosh y + sin x sinh y
x
f
= cos x cosh y sin x sinh y
y

400
These derivatives exist and are everywhere continuous. We equate the expressions to get a set of two equations.

cos x cosh y = cos x cosh y, sin x sinh y = sin x sinh y


cos x cosh y = 0, sin x sinh y = 0
 
x = + n and (x = m or y = 0)
2
The function may be differentiable only at the points

x= + n, y = 0.
2
Thus the function is nowhere analytic.

2. Consider f (x, y) = x2 y 2 + x + (2xy y). The derivatives in the x and y directions are
f
= 2x + 1 + 2y
x
f
= 2y + 2x 1
y
These derivatives exist and are everywhere continuous. We equate the expressions to get a set of two equations.

2x + 1 = 2x 1, 2y = 2y.

Since this set of equations has no solutions, there are no points at which the function is differentiable. The
function is nowhere analytic.
Solution 8.5

f (z1 + z2 ) = f (z1 ) f (z2 )


log (f (z1 + z2 )) = log (f (z1 )) + log (f (z2 ))

401
We define g(z) = log(f (z)).

g (z1 + z2 ) = g (z1 ) + g (z2 )

This is a linear equation which has exactly the solutions:

g(z) = cz.
Thus f (z) has the solutions:
f (z) = ecz ,
where c is any complex constant. We can write this constant in terms of f 0 (0). We differentiate the original equation
with respect to z1 and then substitute z1 = 0.
f 0 (z1 + z2 ) = f 0 (z1 ) f (z2 )
f 0 (z2 ) = f 0 (0)f (z2 )
f 0 (z) = f 0 (0)f (z)

We substitute in the form of the solution.

c ecz = f 0 (0) ecz


c = f 0 (0)
Thus we see that
0
f (z) = ef (0)z .

Cauchy-Riemann Equations
Solution 8.6
Constant Real Part. First assume that f (z) has constant real part. We solve the Cauchy-Riemann equations to
determine the imaginary part.
ux = vy , uy = vx
vx = 0, vy = 0

402
We integrate the first equation to obtain v = a + g(y) where a is a constant and g(y) is an arbitrary function. Then
we substitute this into the second equation to determine g(y).

g 0 (y) = 0
g(y) = b

We see that the imaginary part of f (z) is a constant and conclude that f (z) is constant.
Constant Imaginary Part. Next assume that f (z) has constant imaginary part. We solve the Cauchy-Riemann
equations to determine the real part.

ux = vy , uy = vx
ux = 0, uy = 0

We integrate the first equation to obtain u = a + g(y) where a is a constant and g(y) is an arbitrary function. Then
we substitute this into the second equation to determine g(y).

g 0 (y) = 0
g(y) = b

We see that the real part of f (z) is a constant and conclude that f (z) is constant.
Constant Modulus. Finally assume that f (z) has constant modulus.

|f (z)| = constant

u2 + v 2 = constant
u2 + v 2 = constant

We differentiate this equation with respect to x and y.

2uux + 2vvx = 0, 2uuy + 2vvy = 0


  
ux v x u
=0
uy v y v

403
This system has non-trivial solutions for u and v only if the matrix is non-singular. (The trivial solution u = v = 0 is
the constant function f (z) = 0.) We set the determinant of the matrix to zero.

ux v y u y v x = 0

We use the Cauchy-Riemann equations to write this in terms of ux and uy .

u2x + u2y = 0
ux = uy = 0

Since its partial derivatives vanish, u is a constant. From the Cauchy-Riemann equations we see that the partial
derivatives of v vanish as well, so it is constant. We conclude that f (z) is a constant.
Constant Modulus. Here is another method for the constant modulus case. We solve the Cauchy-Riemann
equations in polar form to determine the argument of f (z) = R(x, y) e(x,y) . Since the function has constant modulus
R, its partial derivatives vanish.

Rx = Ry , Ry = Rx
Ry = 0, Rx = 0

The equations are satisfied for R = 0. For this case, f (z) = 0. We consider nonzero R.

y = 0, x = 0

We see that the argument of f (z) is a constant and conclude that f (z) is constant.

Solution 8.7
First we verify that the Cauchy-Riemann equations are satisfied for z 6= 0. Note that the form

fx = fy

will be far more convenient than the form


ux = v y , uy = vx

404
for this problem.
4
fx = 4(x + y)5 e(x+y)
4 4
fy = 4(x + y)5 e(x+y) = 4(x + y)5 e(x+y)
The Cauchy-Riemann equations are satisfied for z 6= 0.
Now we consider the point z = 0.
f (x, 0) f (0, 0)
fx (0, 0) = lim
x0 x
4
ex
= lim
x0 x
=0

f (0, y) f (0, 0)
fy (0, 0) = lim
y0 y
y 4
e
= lim
y0 y
=0
The Cauchy-Riemann equations are satisfied for z = 0.
f (z) is not analytic at the point z = 0. We show this by calculating the derivative.
f (z) f (0) f (z)
f 0 (0) = lim = lim
z0 z z0 z
Let z = r e , that is, we approach the origin at an angle of .


f r e
f 0 (0) = lim
r0 r e
4 4
er e
= lim
r0 r e

405
For most values of the limit does not exist. Consider = /4.
4
0 er
f (0) = lim =
r0 r e/4

Because the limit does not exist, the function is not differentiable at z = 0. Recall that satisfying the Cauchy-Riemann
equations is a necessary, but not a sufficient condition for differentiability.
Solution 8.8
1. We find the Cauchy-Riemann equations for

f (z) = R(r, ) e(r,) .

From Exercise 8.3 we know that the complex derivative in the polar coordinate directions is

d
= e = e .
dz r r
We equate the derivatives in the two directions.

   
e Re = e Re
r r

(Rr + Rr ) e = (R + R ) e
r
We divide by e and equate the real and imaginary components to obtain the Cauchy-Riemann equations.

R 1
Rr = , R = Rr
r r

2. We find the Cauchy-Riemann equations for

f (z) = R(x, y) e(x,y) .

406
We equate the derivatives in the x and y directions.

   
Re = Re
x y
(Rx + Ry ) e = (Rx + Ry ) e

We divide by e and equate the real and imaginary components to obtain the Cauchy-Riemann equations.

Rx = Ry , Ry = Rx

Solution 8.9
1. A necessary condition for analyticity in an open set is that the Cauchy-Riemann equations are satisfied in that
set. We write ez in Cartesian form.

ez = exy = ex cos y ex sin y.

Now we determine where u = ex cos y and v = ex sin y satisfy the Cauchy-Riemann equations.

ux = vy , uy = vx
x x
e cos y = e cos y, ex sin y = ex sin y
cos y = 0, sin y = 0

y = + m, y = n
2

Thus we see that the Cauchy-Riemann equations are not satisfied anywhere. ez is nowhere analytic.

2. Since f (z) = u + v is analytic, u and v satisfy the Cauchy-Riemann equations and their first partial derivatives
are continuous.
f (z) = f (z) = u(x, y) + v(x, y) = u(x, y) v(x, y)

407
We define f (z) (x, y) + (x, y) = u(x, y) v(x, y). Now we see if and satisfy the Cauchy-Riemann
equations.

x = y , y = x
(u(x, y))x = (v(x, y))y , (u(x, y))y = (v(x, y))x
ux (x, y) = vy (x, y), uy (x, y) = vx (x, y)
ux = vy , uy = vx

Thus we see that the Cauchy-Riemann equations for and are satisfied if and only if the Cauchy-Riemann
equations for u and v are satisfied. The continuity of the first partial derivatives of u and v implies the same of
and . Thus f (z) is analytic.

Solution 8.10
1. The necessary condition for a function f (z) = u + v to be differentiable at a point is that the Cauchy-Riemann
equations hold and the first partial derivatives of u and v are continuous at that point.

(a)
f (z) = x3 + y 3 + 0
The Cauchy-Riemann equations are

ux = vy and uy = vx
3x2 = 0 and 3y 2 = 0
x = 0 and y = 0

The first partial derivatives are continuous. Thus we see that the function is differentiable only at the point
z = 0.
(b)
x1 y
f (z) = 2 2

(x 1) + y (x 1)2 + y 2

408
The Cauchy-Riemann equations are

ux = vy and uy = vx
2 2
(x 1) + y (x 1)2 + y 2 2(x 1)y 2(x 1)y
= and =
((x 1)2 + y 2 )2 ((x 1)2 + y 2 )2 ((x 1)2 + y 2 )2 ((x 1)2 + y 2 )2
The Cauchy-Riemann equations are each identities. The first partial derivatives are continuous everywhere
except the point x = 1, y = 0. Thus the function is differentiable everywhere except z = 1.
2. (a) The function is not differentiable in any open set. Thus the function is nowhere analytic.
(b) The function is differentiable everywhere except z = 1. Thus the function is analytic everywhere except
z = 1.
3. (a) First we determine if the function is harmonic.

v = x2 y 2
vxx + vyy = 0
22=0

The function is harmonic in the complex plane and this is the imaginary part of some analytic function. By
inspection, we see that this function is

z 2 + c = 2xy + c + x2 y 2 ,


where c is a real constant. We can also find the function by solving the Cauchy-Riemann equations.

ux = vy and uy = vx
ux = 2y and uy = 2x

We integrate the first equation.

u = 2xy + g(y)

409
Here g(y) is a function of integration. We substitute this into the second Cauchy-Riemann equation to
determine g(y).

uy = 2x
2x + g 0 (y) = 2x
g 0 (y) = 0
g(y) = c
u = 2xy + c
f (z) = 2xy + c + x2 y 2


f (z) = z 2 + c

(b) First we determine if the function is harmonic.

v = 3x2 y
vxx + vyy = 6y

The function is not harmonic. It is not the imaginary part of some analytic function.

Solution 8.11
We write the real and imaginary parts of f (z) = u + v.

( (
x4/3 y 5/3 x5/3 y 4/3
x2 +y 2
for z 6= 0, x2 +y 2
for z 6= 0,
u= , v=
0 for z = 0. 0 for z = 0.

The Cauchy-Riemann equations are


ux = v y , uy = vx .

410
We calculate the partial derivatives of u and v at the point x = y = 0 using the definition of differentiation.
u(x, 0) u(0, 0) 00
ux (0, 0) = lim = lim =0
x0 x x0 x
v(x, 0) v(0, 0) 00
vx (0, 0) = lim = lim =0
x0 x x0 x
u(0, y) u(0, 0) 00
uy (0, 0) = lim = lim =0
y0 y y0 y

v(0, y) v(0, 0) 00
vy (0, 0) = lim = lim =0
y0 y y0 y
Since ux (0, 0) = uy (0, 0) = vx (0, 0) = vy (0, 0) = 0 the Cauchy-Riemann equations are satisfied.
f (z) is not analytic at the point z = 0. We show this by calculating the derivative there.
f (z) f (0) f (z)
f 0 (0) = lim = lim
z0 z z0 z

We let z = r e , that is, we approach the origin at an angle of . Then x = r cos and y = r sin .

0 f r e
f (0) = lim
r0 r e
r 4/3 cos4/3 r5/3 sin5/3 +r 5/3 cos5/3 r 4/3 sin4/3
r2
= lim
r0 r e
4/3 5/3 5/3 4/3
cos sin
+ cos sin
= lim
r0 e
The value of the limit depends on and is not a constant. Thus this limit does not exist. The function is not
differentiable at z = 0.
Solution 8.12
( (
x3 y 3 x3 +y 3
x2 +y 2
for z 6= 0, x2 +y 2
for z 6= 0,
u= , v=
0 for z = 0. 0 for z = 0.

411
The Cauchy-Riemann equations are
ux = v y , uy = vx .
The partial derivatives of u and v at the point x = y = 0 are,
u(x, 0) u(0, 0)
ux (0, 0) = lim
x0 x
x 0
= lim
x0 x
= 1,

v(x, 0) v(0, 0)
vx (0, 0) = lim
x0 x
x 0
= lim
x0 x
= 1,

u(0, y) u(0, 0)
uy (0, 0) = lim
y0 y
y 0
= lim
y0 y
= 1,

v(0, y) v(0, 0)
vy (0, 0) = lim
y0 y
y 0
= lim
y0 y
= 1.

412
We see that the Cauchy-Riemann equations are satisfied at x = y = 0
f (z) is not analytic at the point z = 0. We show this by calculating the derivative.

f (z) f (0) f (z)


f 0 (0) = lim = lim
z0 z z0 z

Let z = r e , that is, we approach the origin at an angle of . Then x = r cos and y = r sin .

0 f r e
f (0) = lim
r0 r e
(1+)r3 cos3 (1)r3 sin3
r2
= lim
r0 r e
3 3
(1 + ) cos (1 ) sin
= lim
r0 e

The value of the limit depends on and is not a constant. Thus this limit does not exist. The function is not
differentiable at z = 0. Recall that satisfying the Cauchy-Riemann equations is a necessary, but not a sufficient
condition for differentiability.

Solution 8.13
We show that the logarithm log z = (r, ) = Log r + satisfies the Cauchy-Riemann equations.


r =
r
1
=
r r
1 1
=
r r
Since the logarithm satisfies the Cauchy-Riemann equations and the first partial derivatives are continuous for z 6= 0,
the logarithm is analytic for z 6= 0.

413
Now we compute the derivative.
d
log z = e (Log r + )
dz r
1
= e
r
1
=
z
Solution 8.14
The complex derivative in the coordinate directions is
d
= e = e .
dz r r
We substitute f = u + v into this identity to obtain the Cauchy-Riemann equation in polar coordinates.
f f
e = e
r r
f f
=
r r

ur + vr = (u + v )
r
We equate the real and imaginary parts.
1 1
ur = v , vr = u
r r
1
ur = v , u = rvr
r
Solution 8.15
Since w is analytic, u and v satisfy the Cauchy-Riemann equations,
ux = vy and uy = vx .

414
Using the chain rule we can write the derivatives with respect to x and y in terms of u and v.

= ux + vx
x u v

= uy + vy
y u v
Now we examine x y .

x y = ux u + vx v (uy u + vy v )
x y = (ux uy ) u + (vx vy ) v
x y = (ux uy ) u (vy + vx ) v

We use the Cauchy-Riemann equations to write uy and vy in terms of ux and vx .

x y = (ux + vx ) u (ux + vx ) v

Recall that w0 = ux + vx = vy uy .

dw
x y = (u v )
dz

Thus we see that,


 1  
dw
= .
u v dz x y

We write this in operator notation.


 1  
dw
=
u v dz x y

415
The complex conjugate of this relation is
 1  
dw
+ = +
u v dz x y
Now we apply both these operators to = .
    1   1  
dw dw
+ = +
u v u v dz x y dz x y

2 2 2 2
 
+ +
u2 uv vu v 2
 1 "    1 !    1   #
dw dw dw
= + + +
dz x y dz x y dz x y x y

(w0 )1 is an analytic function. Recall that for analytic functions f , f 0 = fx = fy . So that fx + fy = 0.


 1 " 1  2 #
2 2 dw dw 2
+ = +
u2 v 2 dz dz x2 y 2
2  2
2 2 dw 2

+ = +
u2 v 2 dz x2 y 2
Solution 8.16
1. We consider
f (z) = log |z| + arg(z) = log r + .
The Cauchy-Riemann equations in polar coordinates are
1
ur = v , u = rvr .
r

416
We calculate the derivatives.
1 1 1
ur = , v =
r r r
u = 0, rvr = 0
Since the Cauchy-Riemann equations are satisfied and the partial derivatives are continuous, f (z) is analytic in
|z| > 0, | arg(z)| < . The complex derivative in terms of polar coordinates is
d
= e = e .
dz r r
We use this to differentiate f (z).
df 1 1
= e [log r + ] = e =
dz r r z
2. Next we consider p
f (z) = |z| e arg(z)/2 = r e/2 .
The Cauchy-Riemann equations for polar coordinates and the polar form f (z) = R(r, ) e(r,) are
R 1
Rr = , R = Rr .
r r

We calculate the derivatives for R = r, = /2.
1 R 1
Rr = , =
2 r r 2 r
1
R = 0, Rr = 0
r
Since the Cauchy-Riemann equations are satisfied and the partial derivatives are continuous, f (z) is analytic in
|z| > 0, | arg(z)| < . The complex derivative in terms of polar coordinates is
d
= e = e .
dz r r

417
We use this to differentiate f (z).
df 1 1
= e [ r e/2 ] = /2 =
dz r 2e r 2 z
Solution 8.17
1. We consider the function

u = x Log r y arctan(x, y) = r cos Log r r sin

We compute the Laplacian.


1 2u
 
1 u
u = r + 2 2
r r r r
1 1
= (cos (r + r Log r) sin ) + 2 (r( sin 2 cos ) r cos Log r)
r r r
1 1
= (2 cos + cos Log r sin ) + ( sin 2 cos cos Log r)
r r
=0

The function u is harmonic. We find the harmonic conjugate v by solving the Cauchy-Riemann equations.
1
vr = u , v = rur
r
vr = sin (1 + Log r) + cos , v = r (cos (1 + Log r) sin )

We integrate the first equation with respect to r to determine v to within the constant of integration g().

v = r(sin Log r + cos ) + g()

We differentiate this expression with respect to .

v = r (cos (1 + Log r) sin ) + g 0 ()

418
We compare this to the second Cauchy-Riemann equation to see that g 0 () = 0. Thus g() = c. We have
determined the harmonic conjugate.

v = r(sin Log r + cos ) + c

The corresponding analytic function is

f (z) = r cos Log r r sin + (r sin Log r + r cos + c).

On the positive real axis, ( = 0), the function has the value

f (z = r) = r Log r + c.

We use analytic continuation to determine the function in the complex plane.

f (z) = z log z + c

2. We consider the function


u = Arg(z) = .
We compute the Laplacian.
1 2u
 
1 u
u = r + =0
r r r r2 2
The function u is harmonic. We find the harmonic conjugate v by solving the Cauchy-Riemann equations.
1
v r = u , v = rur
r
1
vr = , v = 0
r
We integrate the first equation with respect to r to determine v to within the constant of integration g().

v = Log r + g()

419
We differentiate this expression with respect to .

v = g 0 ()

We compare this to the second Cauchy-Riemann equation to see that g 0 () = 0. Thus g() = c. We have
determined the harmonic conjugate.
v = Log r + c
The corresponding analytic function is
f (z) = Log r + c
On the positive real axis, ( = 0), the function has the value

f (z = r) = Log r + c

We use analytic continuation to determine the function in the complex plane.

f (z) = log z + c

3. We consider the function


u = rn cos(n)
We compute the Laplacian.

1 2u
 
1 u
u = r + 2 2
r r r r
1
= (nrn cos(n)) n2 rn2 cos(n)
r r
= n2 rn2 cos(n) n2 rn2 cos(n)
=0

420
The function u is harmonic. We find the harmonic conjugate v by solving the Cauchy-Riemann equations.
1
v r = u , v = rur
r
vr = nrn1 sin(n), v = nrn cos(n)

We integrate the first equation with respect to r to determine v to within the constant of integration g().

v = rn sin(n) + g()

We differentiate this expression with respect to .

v = nrn cos(n) + g 0 ()

We compare this to the second Cauchy-Riemann equation to see that g 0 () = 0. Thus g() = c. We have
determined the harmonic conjugate.
v = rn sin(n) + c
The corresponding analytic function is

f (z) = rn cos(n) + rn sin(n) + c

On the positive real axis, ( = 0), the function has the value

f (z = r) = rn + c

We use analytic continuation to determine the function in the complex plane.

f (z) = z n

4. We consider the function


y sin
u= =
r2 r

421
We compute the Laplacian.
1 2u
 
1 u
u = r + 2 2
r r r r
 
1 sin sin
= 3
r r r r
sin sin
= 3 3
r r
=0
The function u is harmonic. We find the harmonic conjugate v by solving the Cauchy-Riemann equations.
1
vr = u , v = rur
r
cos sin
v r = 2 , v =
r r
We integrate the first equation with respect to r to determine v to within the constant of integration g().
cos
v= + g()
r
We differentiate this expression with respect to .
sin
v = + g 0 ()
r
We compare this to the second Cauchy-Riemann equation to see that g 0 () = 0. Thus g() = c. We have
determined the harmonic conjugate.
cos
v= +c
r
The corresponding analytic function is
sin cos
f (z) = + + c
r r

422
On the positive real axis, ( = 0), the function has the value


f (z = r) = + c.
r

We use analytic continuation to determine the function in the complex plane.


f (z) = + c
z

Solution 8.18
1. We calculate the first partial derivatives of u = (x y)2 and v = 2(x + y).

ux = 2(x y)
uy = 2(y x)
vx =2
vy =2

We substitute these expressions into the Cauchy-Riemann equations.

ux = vy , uy = vx
2(x y) = 2, 2(y x) = 2
x y = 1, y x = 1
y =x1

Since the Cauchy-Riemann equation are satisfied along the line y = x1 and the partial derivatives are continuous,
the function f (z) is differentiable there. Since the function is not differentiable in a neighborhood of any point,
it is nowhere analytic.

423
2. We calculate the first partial derivatives of u and v.
2 y 2
u x = 2 ex (x cos(2xy) y sin(2xy))
x2 y 2
uy = 2 e (y cos(2xy) + x sin(2xy))
x2 y 2
vx = 2 e (y cos(2xy) + x sin(2xy))
x2 y 2
vy = 2 e (x cos(2xy) y sin(2xy))

Since the Cauchy-Riemann equations, ux = vy and uy = vx , are satisfied everywhere and the partial derivatives
are continuous, f (z) is everywhere differentiable. Since f (z) is differentiable in a neighborhood of every point, it
is analytic in the complex plane. (f (z) is entire.)
Now to evaluate the derivative. The complex derivative is the derivative in any direction. We choose the x
direction.

f 0 (z) = ux + vx
2 y 2 2 y 2
f 0 (z) = 2 ex (x cos(2xy) y sin(2xy)) + 2 ex (y cos(2xy) + x sin(2xy))
0 x2 y 2
f (z) = 2 e ((x + y) cos(2xy) + (y + x) sin(2xy))

Finding the derivative is easier if we first write f (z) in terms of the complex variable z and use complex differen-
tiation.
2 y 2
f (z) = ex (cos(2x, y) + sin(2xy))
2 y 2
f (z) = ex e2xy
2
f (z) = e(x+y)
2
f (z) = ez
2
f 0 (z) = 2z ez

424
Solution 8.19
1. Assume that the Cauchy-Riemann equations in Cartesian coordinates
ux = vy , uy = vx
are satisfied and these partial derivatives are continuous at a point z. We write the derivatives in polar coordinates
in terms of derivatives in Cartesian coordinates to verify the Cauchy-Riemann equations in polar coordinates. First
we calculate the derivatives.
x = r cos , y = r sin
x y
wr = wx + wy = cos wx + sin wy
r r
x y
w = wx + wy = r sin wx + r cos wy

Then we verify the Cauchy-Riemann equations in polar coordinates.
ur = cos ux + sin uy
= cos vy sin vx
1
= v
r

1
u = sin ux + cos uy
r
= sin vy cos vx
= vr
This proves that the Cauchy-Riemann equations in Cartesian coordinates hold only if the Cauchy-Riemann equa-
tions in polar coordinates hold. (Given that the partial derivatives are continuous.) Next we prove the converse.
Assume that the Cauchy-Riemann equations in polar coordinates
1 1
ur = v , u = vr
r r

425
are satisfied and these partial derivatives are continuous at a point z. We write the derivatives in Cartesian
coordinates in terms of derivatives in polar coordinates to verify the Cauchy-Riemann equations in Cartesian
coordinates. First we calculate the derivatives.
p
r = x2 + y 2 , = arctan(x, y)
r x y
wx = wr + w = wr 2 w
x x r r
r y x
wy = wr + w = wr + 2 w
y y r r
Then we verify the Cauchy-Riemann equations in Cartesian coordinates.
x y
ux = ur 2 u
r r
x y
= 2 v + vr
r r
= uy
y x
uy = ur + 2 u
r r
y x
= 2 v vr
r r
= ux
This proves that the Cauchy-Riemann equations in polar coordinates hold only if the Cauchy-Riemann equations
in Cartesian coordinates hold. We have demonstrated the equivalence of the two forms.
2. We verify that log z is analytic for r > 0 and < < using the polar form of the Cauchy-Riemann equations.
Log z = ln r +
1 1
ur = v , u = vr
r r
1 1 1
= 1, 0 = 0
r r r

426
Since the Cauchy-Riemann equations are satisfied and the partial derivatives are continuous for r > 0, log z is
analytic there. We calculate the value of the derivative using the polar differentiation formulas.
d 1 1
Log z = e (ln r + ) = e =
dz r r z
d 1
Log z = (ln r + ) = =
dz z z z
3. Let {xi } denote rectangular coordinates in two dimensions and let {i } be an orthogonal coordinate system .
The distance metric coefficients hi are defined
s 2  2
x1 x2
hi = + .
i i
The Laplacian is     
2 1 h2 u h1 u
u= + .
h1 h2 1 h1 1 2 h2 2
First we calculate the distance metric coefficients in polar coordinates.
s 
2  2 p
x y
hr = + = cos2 + sin2 = 1
r r
s 
2  2 p
x y
h = + = r2 sin2 + r2 cos2 = r

Then we find the Laplacian.   
2 1 1
= (rr ) +
r r r
In polar coordinates, Laplaces equation is
1 1
rr + r + 2 = 0.
r r

427
Solution 8.20
1. We compute the Laplacian of u(x, y) = x3 y 3 .

2 u = 6x 6y

Since u is not harmonic, it is not the real part of on analytic function.


2. We compute the Laplacian of u(x, y) = sinh x cos y + x.

2 u = sinh x cos y sinh x cos y = 0

Since u is harmonic, it is the real part of on analytic function. We determine v by solving the Cauchy-Riemann
equations.

vx = uy , vy = ux
vx = sinh x sin y, vy = cosh x cos y + 1

We integrate the first equation to determine v up to an arbitrary additive function of y.

v = cosh x sin y + g(y)

We substitute this into the second Cauchy-Riemann equation. This will determine v up to an additive constant.

vy = cosh x cos y + 1
cosh x cos y + g 0 (y) = cosh x cos y + 1
g 0 (y) = 1
g(y) = y + a
v = cosh x sin y + y + a
f (z) = sinh x cos y + x + (cosh x sin y + y + a)

Here a is a real constant. We write the function in terms of z.

f (z) = sinh z + z + a

428
3. We compute the Laplacian of u(r, ) = rn cos(n).
2 u = n(n 1)rn2 cos(n) + nrn2 cos(n) n2 rn2 cos(n) = 0
Since u is harmonic, it is the real part of on analytic function. We determine v by solving the Cauchy-Riemann
equations.
1
v r = u , v = rur
r
n1
vr = nr sin(n), v = nrn cos(n)
We integrate the first equation to determine v up to an arbitrary additive function of .
v = rn sin(n) + g()
We substitute this into the second Cauchy-Riemann equation. This will determine v up to an additive constant.
v = nrn cos(n)
nrn cos(n) + g 0 () = nrn cos(n)
g 0 () = 0
g() = a
v = rn sin(n) + a
f (z) = rn cos(n) + (rn sin(n) + a)
Here a is a real constant. We write the function in terms of z.
f (z) = z n + a
Solution 8.21
1. We find the velocity potential and stream function .
(z) = log z + log z
(z) = ln r + + (ln r + )
= ln r , = ln r +

429
Figure 8.7: The velocity potential and stream function for (z) = log z + log z.

A branch of these are plotted in Figure 8.7.


Next we find the stream lines, = c.
ln r + = c
r = ec
These are spirals which go counter-clockwise as we follow them to the origin. See Figure 8.8. Next we find the
velocity field.
v =

v = r r +
r
r
v=
r r

430
Figure 8.8: Streamlines for = ln r + .

The velocity field is shown in the first plot of Figure 8.9. We see that the fluid flows out from the origin along
the spiral paths of the streamlines. The second plot shows the direction of the velocity field.
2. We find the velocity potential and stream function .
(z) = log(z 1) + log(z + 1)
(z) = ln |z 1| + arg(z 1) + ln |z + 1| + arg(z + 1)
= ln |z 2 1|, = arg(z 1) + arg(z + 1)
The velocity potential and a branch of the stream function are plotted in Figure 8.10.
The stream lines, arg(z 1) + arg(z + 1) = c, are plotted in Figure 8.11.
Next we find the velocity field.
v =
2 2
2x(x + y 1) 2y(x2 + y 2 + 1)
v= x + y
x4 + 2x2 (y 2 1) + (y 2 + 1)2 x4 + 2x2 (y 2 1) + (y 2 + 1)2

431
Figure 8.9: Velocity field and velocity direction field for = ln r .

The velocity field is shown in the first plot of Figure 8.12. The fluid is flowing out of sources at z = 1. The
second plot shows the direction of the velocity field.
Solution 8.22
1. (a) We factor the denominator to see that there are first order poles at z = .
z z
=
z2 +1 (z )(z + )

432
2 6
1 2 4 2
0 2
-1 1 1
0
-2 0 -2 0
-1 -1
0 -1 0 -1
1 1
2-2 2-2

Figure 8.10: The velocity potential and stream function for (z) = log(z 1) + log(z + 1).

Since the function behaves like 1/z at infinity, it is analytic there.


(b) The denominator of 1/ sin z has first order zeros at z = n, n Z. Thus the function has first order poles
at these locations. Now we examine the point at infinity with the change of variables z = 1/.

1 1 2
= = /
sin z sin(1/) e e/

We see that the point at infinity is a singularity of the function. Since the denominator grows exponentially,
there is no multiplicative factor of n that will make the function analytic at = 0. We conclude that the
point at infinity is an essential singularity. Since there is no deleted neighborhood of the point at infinity
that does contain first order poles at the locations z = n, the point at infinity is a non-isolated singularity.
(c)
log 1 + z 2 = log(z + ) + log(z )


There are branch points at z = . Since the argument of the logarithm is unbounded as z there is
a branch point at infinity as well. Branch points are non-isolated singularities.

433
2

-1

-2
-2 -1 0 1 2

Figure 8.11: Streamlines for = arg(z 1) + arg(z + 1).

(d)
1
z sin(1/z) = z e/z + e/z

2
The point z = 0 is a singularity. Since the function grows exponentially at z = 0. There is no multiplicative
factor of z n that will make the function analytic. Thus z = 0 is an essential singularity.
There are no other singularities in the finite complex plane. We examine the point at infinity.
 
1 1
z sin = sin
z
The point at infinity is a singularity. We take the limit 0 to demonstrate that it is a removable

434
Figure 8.12: Velocity field and velocity direction field for = ln |z 2 1|.

singularity.
sin cos
lim = lim =1
0 0 1

(e)
log +z

tan1 (z) z
=
z sinh2 (z) 2z sinh2 (z)

435
There are branch points at z = due to the logarithm. These are non-isolated singularities. Note that
sinh(z) has first order zeros at z = n, n Z. The arctangent has a first order zero at z = 0. Thus there
is a second order pole at z = 0. There are second order poles at z = n, n Z \ {0} due to the hyperbolic
sine. Since the hyperbolic sine has an essential singularity at infinity, the function has an essential singularity
at infinity as well. The point at infinity is a non-isolated singularity because there is no neighborhood of
infinity that does not contain second order poles.

2. (a) (z ) e1/(z1) has a simple zero at z = and an isolated essential singularity at z = 1.


(b)
sin(z 3)
(z 3)(z + )6
has a removable singularity at z = 3, a pole of order 6 at z = and an essential singularity at z .

436
Chapter 9

Analytic Continuation

For every complex problem, there is a solution that is simple, neat, and wrong.

- H. L. Mencken

9.1 Analytic Continuation


Suppose there is a function, f1 (z) that is analytic in the domain D1 and another analytic function, f2 (z) that is
analytic in the domain D2 . (See Figure 9.1.)
If the two domains overlap and f1 (z) = f2 (z) in the overlap region D1 D2 , then f2 (z) is called an analytic
continuation of f1 (z). This is an appropriate name since f2 (z) continues the definition of f1 (z) outside of its original
domain of definition D1 . We can define a function f (z) that is analytic in the union of the domains D1 D2 . On the
domain D1 we have f (z) = f1 (z) and f (z) = f2 (z) on D2 . f1 (z) and f2 (z) are called function elements. There is an
analytic continuation even if the two domains only share an arc and not a two dimensional region.

With more overlapping domains D3 , D4 , . . . we could perhaps extend f1 (z) to more of the complex plane. Sometimes
it is impossible to extend a function beyond the boundary of a domain. This is known as a natural boundary. If a

437
Im(z)

D1
D2
Re(z)

Figure 9.1: Overlapping Domains

function f1 (z) is analytically continued to a domain Dn along two different paths, (See Figure 9.2.), then the two
analytic continuations are identical as long as the paths do not enclose a branch point of the function. This is the
uniqueness theorem of analytic continuation.

Dn
D1

Figure 9.2: Two Paths of Analytic Continuation

Consider an analytic function f (z) defined in the domain D. Suppose that f (z) = 0 on the arc AB, (see Figure 9.3.)
Then f (z) = 0 in all of D.
Consider a point on AB. The Taylor series expansion of f (z) about the point z = converges in a circle C at

438
D
C
B
A

Figure 9.3: Domain Containing Arc Along Which f (z) Vanishes

least up to the boundary of D. The derivative of f (z) at the point z = is

f ( + z) f ()
f 0 () = lim
z0 z

If z is in the direction of the arc, then f 0 () vanishes as well as all higher derivatives, f 0 () = f 00 () = f 000 () = = 0.
Thus we see that f (z) = 0 inside C. By taking Taylor series expansions about points on AB or inside of C we see that
f (z) = 0 in D.

Result 9.1.1 Let f1 (z) and f2 (z) be analytic functions defined in D. If f1 (z) = f2 (z) for
the points in a region or on an arc in D, then f1 (z) = f2 (z) for all points in D.

To prove Result 9.1.1, we define the analytic function g(z) = f1 (z) f2 (z). Since g(z) vanishes in the region or
on the arc, then g(z) = 0 and hence f1 (z) = f2 (z) for all points in D.

439
Result 9.1.2 Consider analytic functions f1 (z) and f2 (z) defined on the domains D1 and
D2 , respectively. Suppose that D1 D2 is a region or an arc and that f1 (z) = f2 (z) for all
z D1 D2 . (See Figure 9.4.) Then the function
(
f1 (z) for z D1 ,
f (z) =
f2 (z) for z D2 ,

is analytic in D1 D2 .

D1 D2 D1 D2

Figure 9.4: Domains that Intersect in a Region or an Arc

Result 9.1.2 follows directly from Result 9.1.1.

9.2 Analytic Continuation of Sums


Example 9.2.1 Consider the function

X
f1 (z) = zn.
n=0

The sum converges uniformly for D1 = |z| r < 1. Since the derivative also converges in this domain, the function is
analytic there.

440
Re(z) Re(z)

D2
D1
Im(z) Im(z)

P
Figure 9.5: Domain of Convergence for n=0 zn.

Now consider the function


1
f2 (z) = .
1z
This function is analytic everywhere except the point z = 1. On the domain D1 ,

1 X
f2 (z) = = z n = f1 (z)
1z n=0

Analytic continuation tells us that there is a function that is analytic on the union of the two domains. Here, the
domain is the entire z plane except the point z = 1 and the function is

1
f (z) = .
1z
1
P
1z
is said to be an analytic continuation of n=0 zn.

441
9.3 Analytic Functions Defined in Terms of Real Variables

Result 9.3.1 An analytic function, u(x, y) + v(x, y) can be written in terms of a function
of a complex variable, f (z) = u(x, y) + v(x, y).
Result 9.3.1 is proved in Exercise 9.1.

Example 9.3.1

f (z) = cosh y sin x (x ex cos y y ex sin y) cos x sinh y (y ex cos y + x ex sin y)


+ cosh y sin x (y ex cos y + x ex sin y) + cos x sinh y (x ex cos y y ex sin y)
 

is an analytic function. Express f (z) in terms of z.


On the real line, y = 0, f (z) is
f (z = x) = x ex sin x
(Recall that cos(0) = cosh(0) = 1 and sin(0) = sinh(0) = 0.)
The analytic continuation of f (z) into the complex plane is

f (z) = z ez sin z.

Alternatively, for x = 0 we have


f (z = y) = y sinh y(cos y sin y).
The analytic continuation from the imaginary axis to the complex plane is

f (z) = z sinh(z)(cos(z) sin(z))


= z sinh(z)(cos(z) + sin(z))
= z sin z ez .

442
Example 9.3.2 Consider u = ex (x sin y y cos y). Find v such that f (z) = u + v is analytic.
From the Cauchy-Riemann equations,
v u
= = ex sin y x ex sin y + y ex cos y
y x
v u
= = ex cos y x ex cos y y ex sin y
x y
Integrate the first equation with respect to y.

v = ex cos y + x ex cos y + ex (y sin y + cos y) + F (x)


= y ex sin y + x ex cos y + F (x)

F (x) is an arbitrary function of x. Substitute this expression for v into the equation for v/x.

y ex sin y x ex cos y + ex cos y + F 0 (x) = y ex sin y x ex cos y + ex cos y

Thus F 0 (x) = 0 and F (x) = c.


v = ex (y sin y + x cos y) + c

Example 9.3.3 Find f (z) in the previous example. (Up to the additive constant.)

Method 1

f (z) = u + v
= ex (x sin y y cos y) + ex (y sin y + x cos y)
e ey e + ey e ey e + ey
  y   y    y   y 
x x
=e x y + e y +x
2 2 2 2
(x+y)
= (x + y) e
= z ez

443
Method 2 f (z) = f (x + y) = u(x, y) + v(x, y) is an analytic function.
On the real axis, y = 0, f (z) is
f (z = x) = u(x, 0) + v(x, 0)
= ex (x sin 0 0 cos 0) + ex (0 sin 0 + x cos 0)
= x ex
Suppose there is an analytic continuation of f (z) into the complex plane. If such a continuation, f (z), exists, then it
must be equal to f (z = x) on the real axis An obvious choice for the analytic continuation is
f (z) = u(z, 0) + v(z, 0)
since this is clearly equal to u(x, 0) + v(x, 0) when z is real. Thus we obtain
f (z) = z ez
Example 9.3.4 Consider f (z) = u(x, y) + v(x, y). Show that f 0 (z) = ux (z, 0) uy (z, 0).
f 0 (z) = ux + vx
= ux uy
f 0 (z) is an analytic function. On the real axis, z = x, f 0 (z) is
f 0 (z = x) = ux (x, 0) uy (x, 0)
Now f 0 (z = x) is defined on the real line. An analytic continuation of f 0 (z = x) into the complex plane is
f 0 (z) = ux (z, 0) uy (z, 0).
Example 9.3.5 Again consider the problem of finding f (z) given that u(x, y) = ex (x sin y y cos y). Now we can
use the result of the previous example to do this problem.
u
ux (x, y) = = ex sin y x ex sin y + y ex cos y
x
u
uy (x, y) = = x ex cos y + y ex sin y ex cos y
y

444
f 0 (z) = ux (z, 0) uy (z, 0)
= 0 z ez ez


= z ez + ez


Integration yields the result


f (z) = z ez +c

Example 9.3.6 Find f (z) given that

u(x, y) = cos x cosh2 y sin x + cos x sin x sinh2 y


v(x, y) = cos2 x cosh y sinh y cosh y sin2 x sinh y

f (z) = u(x, y) + v(x, y) is an analytic function. On the real line, f (z) is

f (z = x) = u(x, 0) + v(x, 0)
= cos x cosh2 0 sin x + cos x sin x sinh2 0 + cos2 x cosh 0 sinh 0 cosh 0 sin2 x sinh 0


= cos x sin x

Now we know the definition of f (z) on the real line. We would like to find an analytic continuation of f (z) into the
complex plane. An obvious choice for f (z) is
f (z) = cos z sin z
Using trig identities we can write this as
sin(2z)
f (z) = .
2

Example 9.3.7 Find f (z) given only that

u(x, y) = cos x cosh2 y sin x + cos x sin x sinh2 y.

445
Recall that

f 0 (z) = ux + vx
= ux uy

Differentiating u(x, y),

ux = cos2 x cosh2 y cosh2 y sin2 x + cos2 x sinh2 y sin2 x sinh2 y


uy = 4 cos x cosh y sin x sinh y

f 0 (z) is an analytic function. On the real axis, f 0 (z) is

f 0 (z = x) = cos2 x sin2 x

Using trig identities we can write this as


f 0 (z = x) = cos(2x)
Now we find an analytic continuation of f 0 (z = x) into the complex plane.

f 0 (z) = cos(2z)

Integration yields the result


sin(2z)
f (z) = +c
2

9.3.1 Polar Coordinates


Example 9.3.8 Is
u(r, ) = r(log r cos sin )
the real part of an analytic function?

446
The Laplacian in polar coordinates is

1 2
 
1
= r + .
r r r r2 2

We calculate the partial derivatives of u.

u
= cos + log r cos sin
r
u
r = r cos + r log r cos r sin
 r 
u
r = 2 cos + log r cos sin
r r
 
1 u 1
r = (2 cos + log r cos sin )
r r r r
u
= r ( cos + sin + log r sin )

2u
= r (2 cos log r cos + sin )
2
1 2u 1
2 2
= (2 cos log r cos + sin )
r r
From the above we see that
1 2u
 
1 u
u = r + = 0.
r r r r2 2
Therefore u is harmonic and is the real part of some analytic function.

Example 9.3.9 Find an analytic function f (z) whose real part is

u(r, ) = r (log r cos sin ) .

447
Let f (z) = u(r, ) + v(r, ). The Cauchy-Riemann equations are
v
ur = , u = rvr .
r
Using the partial derivatives in the above example, we obtain two partial differential equations for v(r, ).
u
vr = = cos + sin + log r sin
r
v = rur = r (cos + log r cos sin )
Integrating the equation for v yields
v = r ( cos + log r sin ) + F (r)
where F (r) is a constant of integration.
Substituting our expression for v into the equation for vr yields
cos + log r sin + sin + F 0 (r) = cos + sin + log r sin
F 0 (r) = 0
F (r) = const
Thus we see that
f (z) = u + v
= r (log r cos sin ) + r ( cos + log r sin ) + const
f (z) is an analytic function. On the line = 0, f (z) is
f (z = r) = r(log r) + r(0) + const
= r log r + const
The analytic continuation into the complex plane is

f (z) = z log z + const

448
Example 9.3.10 Find the formula in polar coordinates that is analogous to

f 0 (z) = ux (z, 0) uy (z, 0).

We know that
df f
= e .
dz r
If f (z) = u(r, ) + v(r, ) then
df
= e (ur + vr )
dz
From the Cauchy-Riemann equations, we have vr = u /r.
df  u 
= e ur
dz r
f 0 (z) is an analytic function. On the line = 0, f (z) is
u (r, 0)
f 0 (z = r) = ur (r, 0)
r
The analytic continuation of f 0 (z) into the complex plane is

f 0 (z) = ur (z, 0) u (z, 0).
r

Example 9.3.11 Find an analytic function f (z) whose real part is

u(r, ) = r (log r cos sin ) .

ur (r, ) = (log r cos sin ) + cos


u (r, ) = r ( log r sin sin cos )

449

f 0 (z) = ur (z, 0) u (z, 0)
r
= log z + 1

Integrating f 0 (z) yields


f (z) = z log z + c.

9.3.2 Analytic Functions Defined in Terms of Their Real or Imaginary Parts


Consider an analytic function: f (z) = u(x, y) + v(x, y). We differentiate this expression.

f 0 (z) = ux (x, y) + vx (x, y)

We apply the Cauchy-Riemann equation vx = uy .

f 0 (z) = ux (x, y) uy (x, y). (9.1)

Now consider the function of a complex variable, g():

g() = ux (x, ) uy (x, ) = ux (x, + ) uy (x, + ).

This function is analytic where f () is analytic. To show this we first verify that the derivatives in the and directions
are equal.

g() = uxy (x, + ) uyy (x, + )


g() = (uxy (x, + ) + uyy (x, + )) = uxy (x, + ) uyy (x, + )

Since these partial derivatives are equal and continuous, g() is analytic. We evaluate the function g() at = x.
(Substitute y = x into Equation 9.1.)

f 0 (2x) = ux (x, x) uy (x, x)

450
We make a change of variables to solve for f 0 (x).

x x x x
f 0 (x) = ux , uy , .
2 2 2 2

If the expression is non-singular, then this defines the analytic function, f 0 (z), on the real axis. The analytic continuation
to the complex plane is

z z z z
f 0 (z) = ux , uy , .
2 2 2 2

d
Note that dz
2u(z/2, z/2) = ux (z/2, z/2) uy (z/2, z/2). We integrate the equation to obtain:

z
z
f (z) = 2u , + c.
2 2

We know that the real part of an analytic function determines that function to within an additive constant. Assuming
that the above expression is non-singular, we have found a formula for writing an analytic function in terms of its real
part. With the same method, we can find how to write an analytic function in terms of its imaginary part, v.
We can also derive formulas if u and v are expressed in polar coordinates:

f (z) = u(r, ) + v(r, ).

451
Result 9.3.2 If f (z) = u(x, y) + v(x, y) is analytic and the expressions are non-singular,
then
z z
f (z) = 2u , + const (9.2)
2
z 2
z
f (z) = 2v , + const. (9.3)
2 2
If f (z) = u(r, ) + v(r, ) is analytic and the expressions are non-singular, then

1/2 
f (z) = 2u z , log z + const (9.4)
2

1/2 
f (z) = 2v z , log z + const. (9.5)
2

Example 9.3.12 Consider the problem of finding f (z) given that u(x, y) = ex (x sin y y cos y).

z z
f (z) = 2u ,
2  2 
z/2 z z z  z 
= 2e sin + cos +c
2  z 2 2
z  2
= z ez/2 sin + cos +c
2 2
= z ez/2 ez/2 + c


= z ez +c

Example 9.3.13 Consider


1
Log x2 + y 2 + Arctan(x, y).

Log z =
2

452
We try to construct the analytic function from its real part using Equation 9.2.
z z
f (z) = 2u , +c
2 2
1  z 2  z 2 
= 2 Log + +c
2 2 2
= Log(0) + c

We obtain a singular expression, so the method fails.

Example 9.3.14 Again consider the logarithm, this time written in terms of polar coordinates.

Log z = Log r +

We try to construct the analytic function from its real part using Equation 9.4.

1/2 
f (z) = 2u z , log z + c
2
= 2 Log z 1/2 + c
= Log z + c

With this method we recover the analytic function.

453
9.4 Exercises
Exercise 9.1
Consider two functions, f (x, y) and g(x, y). They are said to be functionally dependent if there is a an h(g) such that

f (x, y) = h(g(x, y)).

f and g will be functionally dependent if and only if their Jacobian vanishes.


If f and g are functionally dependent, then the derivatives of f are

fx = h0 (g)gx
fy = h0 (g)gy .

Thus we have
(f, g) fx fy
= = fx gy fy gx = h0 (g)gx gy h0 (g)gy gx = 0.
(x, y) gx gy

If the Jacobian of f and g vanishes, then
fx gy fy gx = 0.
This is a first order partial differential equation for f that has the general solution

f (x, y) = h(g(x, y)).

Prove that an analytic function u(x, y) + v(x, y) can be written in terms of a function of a complex variable,
f (z) = u(x, y) + v(x, y).

Exercise 9.2
Which of the following functions are the real part of an analytic function? For those that are, find the harmonic
conjugate, v(x, y), and find the analytic function f (z) = u(x, y) + v(x, y) as a function of z.

1. x3 3xy 2 2xy + y

2. ex sinh y

454
3. ex (sin x cos y cosh y cos x sin y sinh y)

Exercise 9.3
For an analytic function, f (z) = u(r, ) + v(r, ) prove that under suitable restrictions:
 
f (z) = 2u z 1/2 , log z + const.
2

455
9.5 Hints
Hint 9.1
Show that u(x, y) + v(x, y) is functionally dependent on x + y so that you can write f (z) = f (x + y) = u(x, y) +
v(x, y).

Hint 9.2

Hint 9.3
Check out the derivation of Equation 9.2.

456
9.6 Solutions
Solution 9.1
u(x, y) + v(x, y) is functionally dependent on z = x + y if and only if

(u + v, x + y)
= 0.
(x, y)


(u + v, x + y) ux + vx uy + vy
=
(x, y) 1
= vx uy + (ux vy )

Since u and v satisfy the Cauchy-Riemann equations, this vanishes.

=0

Thus we see that u(x, y) + v(x, y) is functionally dependent on x + y so we can write

f (z) = f (x + y) = u(x, y) + v(x, y).

Solution 9.2
1. Consider u(x, y) = x3 3xy 2 2xy + y. The Laplacian of this function is

u uxx + uyy
= 6x 6x
=0

Since the function is harmonic, it is the real part of an analytic function. Clearly the analytic function is of the
form,
az 3 + bz 2 + cz + d,

457
with a, b and c complex-valued constants and d a real constant. Substituting z = x + y and expanding products
yields,
a x3 + 3x2 y 3xy 2 y 3 + b x2 + 2xy y 2 + c(x + y) + d.
 

By inspection, we see that the analytic function is

f (z) = z 3 + z 2 z + d.

The harmonic conjugate of u is the imaginary part of f (z),

v(x, y) = 3x2 y y 3 + x2 y 2 x + d.

We can also do this problem with analytic continuation. The derivatives of u are

ux = 3x2 3y 2 2y,
uy = 6xy 2x + 1.

The derivative of f (z) is

f 0 (z) = ux uy = 3x2 2y 2 2y + (6xy 2x + 1).

On the real axis we have


f 0 (z = x) = 3x2 2x + .
Using analytic continuation, we see that

f 0 (z) = 3z 2 2z + .

Integration yields
f (z) = z 3 z 2 + z + const

458
2. Consider u(x, y) = ex sinh y. The Laplacian of this function is

u = ex sinh y + ex sinh y
= 2 ex sinh y.

Since the function is not harmonic, it is not the real part of an analytic function.

3. Consider u(x, y) = ex (sin x cos y cosh y cos x sin y sinh y). The Laplacian of the function is

x
u = (e (sin x cos y cosh y cos x sin y sinh y + cos x cos y cosh y + sin x sin y sinh y))
x
x
+ (e ( sin x sin y cosh y cos x cos y sinh y + sin x cos y sinh y cos x sin y cosh y))
y
= 2 ex (cos x cos y cosh y + sin x sin y sinh y) 2 ex (cos x cos y cosh y + sin x sin y sinh y)
= 0.

Thus u is the real part of an analytic function. The derivative of the analytic function is

f 0 (z) = ux + vx = ux uy

From the derivatives of u we computed before, we have

f (z) = (ex (sin x cos y cosh y cos x sin y sinh y + cos x cos y cosh y + sin x sin y sinh y))
(ex ( sin x sin y cosh y cos x cos y sinh y + sin x cos y sinh y cos x sin y cosh y))

Along the real axis, f 0 (z) has the value,

f 0 (z = x) = ex (sin x + cos x).

By analytic continuation, f 0 (z) is


f 0 (z) = ez (sin z + cos z)

459
We obtain f (z) by integrating.
f (z) = ez sin z + const.
u is the real part of the analytic function

f (z) = ez sin z + c,

where c is a real constant. We find the harmonic conjugate of u by taking the imaginary part of f .

f (z) = ex (cosy + sin y)(sin x cosh y + cos x sinh y) + c

v(x, y) = ex sin x sin y cosh y + cos x cos y sinh y + c

Solution 9.3
We consider the analytic function: f (z) = u(r, ) + v(r, ). Recall that the complex derivative in terms of polar
coordinates is
d
= e = e .
dz r r
The Cauchy-Riemann equations are
1 1
ur = v , v r = u .
r r
We differentiate f (z) and use the partial derivative in r for the right side.

f 0 (z) = e (ur + vr )

We use the Cauchy-Riemann equations to right f 0 (z) in terms of the derivatives of u.


 
0 1
f (z) = e ur u (9.6)
r
Now consider the function of a complex variable, g():
   
1 1
g() = e ur (r, ) u (r, ) = e ur (r, + ) u (r, + )
r r

460
This function is analytic where f () is analytic. It is a simple calculus exercise to show that the complex derivative in

the direction, , and the complex derivative in the direction, , are equal. Since these partial derivatives are
equal and continuous, g() is analytic. We evaluate the function g() at = log r. (Substitute = log r into
Equation 9.6.)
 
0 ( log r)
 ( log r) 1
f re =e ur (r, log r) u (r, log r)
r
1
rf 0 r2 = ur (r, log r) u (r, log r)

r
If the expression is non-singular, then it defines the analytic function, f 0 (z), on a curve. The analytic continuation to
the complex plane is
1
zf 0 z 2 = ur (z, log z) u (z, log z).

z
2
We integrate to obtain an expression for f (z ).
1
f z 2 = u(z, log z) + const

2
We make a change of variables and solve for f (z).
 
f (z) = 2u z 1/2 , log z + const.
2

Assuming that the above expression is non-singular, we have found a formula for writing the analytic function in terms
of its real part, u(r, ). With the same method, we can find how to write an analytic function in terms of its imaginary
part, v(r, ).

461
Chapter 10

Contour Integration and the Cauchy-Goursat


Theorem

Between two evils, I always pick the one I never tried before.

- Mae West

10.1 Line Integrals


In this section we will recall the definition of a line integral in the Cartesian plane. In the next section we will use
this to define the contour integral in the complex plane.

Limit Sum Definition. First we develop a limit sum definition of a line integral. Consider a curve C in the Cartesian
plane joining the points (a0 , b0 ) and (a1 , b1 ). We partition the curve into n segments with the points (x0 , y0 ), . . . , (xn , yn )
where the first and last points are at the endpoints of the curve. We define the differences, xk = xk+1 xk and
yk = yk+1 yk , and let (k , k ) be points on the curve between (xk , yk ) and (xk+1 , yk+1 ). This is shown pictorially
in Figure 10.1.

462
y

(1 ,1 ) (x2 ,y2 ) (xn ,yn )

(x1 ,y1 )
(2 ,2 ) ( n1 ,n1 )
(0 ,0 )
(x0 ,y0 ) (xn1 ,yn1 )
x

Figure 10.1: A curve in the Cartesian plane.

Consider the sum


n1
X
(P (k , k )xk + Q(k , k )yk ) ,
k=0

where P and Q are continuous functions on the curve. (P and Q may be complex-valued.) In the limit as each of the
xk and yk approach zero the value of the sum, (if the limit exists), is denoted by
Z
P (x, y) dx + Q(x, y) dy.
C

This is a line integral along the curve C. The value of the line integral depends on the functions P (x, y) and Q(x, y),
the endpoints of the curve and the curve C. We can also write a line integral in vector notation.
Z
f (x) dx
C

Here x = (x, y) and f (x) = (P (x, y), Q(x, y)).

463
Evaluating Line Integrals with Parameterization. Let the curve C be parametrized by x = x(t), y = y(t)
for t0 t t1 . Then the differentials on the curve are dx = x0 (t) dt and dy = y 0 (t) dt. Using the parameterization we
can evaluate a line integral in terms of a definite integral.
Z Z t1
P (x(t), y(t))x0 (t) + Q(x(t), y(t))y 0 (t) dt

P (x, y) dx + Q(x, y) dy =
C t0

Example 10.1.1 Consider the line integral


Z
x2 dx + (x + y) dy,
C

where C is the semi-circle from (1, 0) to (1, 0) in the upper half plane. We parameterize the curve with x = cos t,
y = sin t for 0 t .
Z Z
2
cos2 t( sin t) + (cos t + sin t) cos t dt

x dx + (x + y) dy =
C 0
2
=
2 3

10.2 Contour Integrals


Limit Sum Definition. We develop a limit sum definition for contour integrals. It will be analogous to the definition
for line integrals except that the notation is cleaner in complex variables. Consider a contour C in the complex plane
joining the points c0 and c1 . We partition the contour into n segments with the points z0 , . . . , zn where the first and
last points are at the endpoints of the contour. We define the differences zk = zk+1 zk and let k be points on the
contour between zk and zk+1 . Consider the sum
n1
X
f (k )zk ,
k=0

464
where f is a continuous function on the contour. In the limit as each of the zk approach zero the value of the sum,
(if the limit exists), is denoted by Z
f (z) dz.
C
This is a contour integral along C.
We can write a contour integral in terms of a line integral. Let f (z) = (x, y). ( : R2 7 C.)
Z Z
f (z) dz = (x, y)(dx + dy)
Z C Z C
f (z) dz = ((x, y) dx + (x, y) dy) (10.1)
C C

Further, we can write a contour integral in terms of two real-valued line integrals. Let f (z) = u(x, y) + v(x, y).
Z Z
f (z) dz = (u(x, y) + v(x, y))(dx + dy)
C C
Z Z Z
f (z) dz = (u(x, y) dx v(x, y) dy) + (v(x, y) dx + u(x, y) dy) (10.2)
C C C

Evaluation. Let the contour C be parametrized by z = z(t) for t0 t t1 . Then the differential on the contour
is dz = z 0 (t) dt. Using the parameterization we can evaluate a contour integral in terms of a definite integral.
Z Z t1
f (z) dz = f (z(t))z 0 (t) dt
C t0

Example 10.2.1 Let C be the positively oriented unit circle about the origin in the complex plane. Evaluate:
R
1. C z dz

2. C z1 dz
R

3. C z1 |dz|
R

465
In each case we parameterize the contour and then do the integral.

1.
z = e , dz = e d
Z Z 2
z dz = e e d
C 0
 2
1 2
= e
2 0
 
1 4 1 0
= e e
2 2
=0

2. Z Z 2 Z 2
1 1
dz = e d = d = 2
C z 0 e 0

3.
|dz| = e d = e |d| = |d|
Since d is positive in this case, |d| = d.
Z Z 2
1 1 2
d = e 0 = 0

|dz| =
C z 0 e

10.2.1 Maximum Modulus Integral Bound


The absolute value of a real integral obeys the inequality
Z b Z b

f (x) dx |f (x)| |dx| (b a) max |f (x)|.

a

a axb

466
Now we prove the analogous result for the modulus of a contour integral.
Z n1


X
f (z) dz = lim f (k )zk

z0
C k=0

n1
X
lim |f (k )| |zk |
z0
k=0
Z
= |f (z)| |dz|
C
Z  
max |f (z)| |dz|
C zC
 Z
= max |f (z)| |dz|
zC C
 
= max |f (z)| (length of C)
zC

Result 10.2.1 Maximum Modulus Integral Bound.


Z Z  

f (z) dz |f (z)| |dz| max |f (z)| (length of C)

C

C zC

10.3 The Cauchy-Goursat Theorem


Let f (z) be analytic in a compact, closed, connected domain D. We consider the integral of f (z) on the boundary of
the domain. Z Z Z
f (z) dz = (x, y)(dx + dy) = dx + dy
D D D

467
Recall Greens Theorem. Z Z
P dx + Q dy = (Qx Py ) dx dy
D D

If we assume that f 0 (z) is continuous, we can apply Greens Theorem to the integral of f (z) on D.
Z Z Z
f (z) dz = dx + dy = (x y ) dx dy
D D D

Since f (z) is analytic, it satisfies the Cauchy-Riemann equation x = y . The integrand in the area integral,
x y , is zero. Thus the contour integral vanishes.
Z
f (z) dz = 0
D

This is known as Cauchys Theorem. The assumption that f 0 (z) is continuous is not necessary, but it makes the
proof much simpler because we can use Greens Theorem. If we remove this restriction the result is known as the
Cauchy-Goursat Theorem. The proof of this result is omitted.

Result 10.3.1 The Cauchy-Goursat Theorem. If f (z) is analytic in a compact, closed,


connected domain D then the integral of f (z) on the boundary of the domain vanishes.
I XI
f (z) dz = f (z) dz = 0
D k Ck

Here the set of contours {Ck } make up the positively oriented boundary D of the domain
D.
As a special case of the Cauchy-Goursat theorem we can consider a simply-connected region. For this the boundary
is a Jordan curve. We can state the theorem in terms of this curve instead of referring to the boundary.

468
Result 10.3.2 The Cauchy-Goursat Theorem for Jordan Curves. If f (z) is analytic
inside and on a simple, closed contour C, then
I
f (z) dz = 0
C

Example 10.3.1 Let C be the unit circle about the origin with positive orientation. In Example 10.2.1 we calculated
that Z
z dz = 0
C
Now we can evaluate the integral without parameterizing the curve. We simply note that the integrand is analytic
inside and on the circle, which is simple and closed. By the Cauchy-Goursat Theorem, the integral vanishes.
We cannot apply the Cauchy-Goursat theorem to evaluate
Z
1
dz = 2
C z

as the integrand is not analytic at z = 0.

Example 10.3.2 Consider the domain D = {z | |z| > 1}. The boundary R of the domain is the unit circle with negative
orientation. f (z) = 1/z is analytic on D and its boundary. However D f (z) dz does not vanish and we cannot apply
the Cauchy-Goursat Theorem. This is because the domain is not compact.

10.4 Contour Deformation


Path Independence. Consider a function f (z) that R is analytic on a simply connected domain a contour C in that
domain with end points a and b. The contour integral C f (z) dz is independent of the path connecting the end points
Rb
and can be denoted a f (z) dz. This result is a direct consequence of the Cauchy-Goursat Theorem. Let C1 and C2
be two different paths connecting the points. Let C2 denote the second contour with the opposite orientation. Let

469
C be the contour which is the union of C1 and C2 . By the Cauchy-Goursat theorem, the integral along this contour
vanishes.
Z Z Z
f (z) dz = f (z) dz + f (z) dz = 0
C C1 C2

This implies that the integrals along C1 and C2 are equal.

Z Z
f (z) dz = f (z) dz
C1 C2

Thus contour integrals on simply connected domains are independent of path. This result does not hold for multiply
connected domains.

Result 10.4.1 Path Independence. Let f (z) be analytic on a simply connected domain.
For points a and b in the domain, the contour integral,
Z b
f (z) dz
a

is independent of the path connecting the points.

Deforming Contours. Consider two simple, closed, positively oriented contours, C1 and C2 . Let C2 lie completely
within C1 . If f (z) is analytic on and between C1 and C2 then the integrals of f (z) along C1 and C2 are equal.

Z Z
f (z) dz = f (z) dz
C1 C2

470
Again, this is a direct consequence of the Cauchy-Goursat Theorem. Let D be the domain on and between C1 and C2 .
By the Cauchy-Goursat Theorem the integral along the boundary of D vanishes.

Z Z
f (z) dz + f (z) dz = 0
C1 C2
Z Z
f (z) dz = f (z) dz
C1 C2

R
By following this line of reasoning, we see that we can deform a contour C without changing the value of C
f (z) dz
as long as we stay on the domain where f (z) is analytic.

Result 10.4.2 Contour Deformation. Let f (z) be analytic on a domain D. If a set of


closed contours {Cm } can be continuously deformed on the domain D to a set of contours
{n } then the integrals along {Cm } and {n } are equal.
Z Z
f (z) dz = f (z) dz
{Cm } {n }

10.5 Moreras Theorem.

The converse of the Cauchy-Goursat theorem is Moreras Theorem. If the integrals of a continuous function f (z)
vanish along all possible simple, closed contours in a domain, then f (z) is analytic on that domain. To prove Moreras
Theorem we will assume that first partial derivatives of f (z) = u(x, y) + v(x, y) are continuous, although the result
can be derived without this restriction. Let the simple, closed contour C be the boundary of D which is contained in

471
the domain .
I I
f (z) dz = (u + v)(dx + dy)
C IC I
= u dx v dy + v dx + u dy
ZC C
Z
= (vx uy ) dx dy + (ux vy ) dx dy
D D
=0

Since the two integrands are continuous and vanish for all C in , we conclude that the integrands are identically zero.
This implies that the Cauchy-Riemann equations,

ux = v y , uy = vx ,

are satisfied. f (z) is analytic in .


The converse of the Cauchy-Goursat theorem is Moreras Theorem. If the integrals of a continuous function f (z)
vanish along all possible simple, closed contours in a domain, then f (z) is analytic on that domain. To prove Moreras
Theorem we will assume that first partial derivatives of f (z) = (x, y) are continuous, although the result can be
derived without this restriction. Let the simple, closed contour C be the boundary of D which is contained in the
domain .
I I
f (z) dz = ( dx + dy)
C
ZC
= (x y ) dx dy
D
=0

Since the integrand, x y is continuous and vanishes for all C in , we conclude that the integrand is identically
zero. This implies that the Cauchy-Riemann equation,

x = y ,

472
is satisfied. We conclude that f (z) is analytic in .

Result 10.5.1 Moreras Theorem. If f (z) is continuous in a simply connected domain


and I
f (z) dz = 0
C
for all possible simple, closed contours C in the domain, then f (z) is analytic in .

10.6 Indefinite Integrals

Consider a function f (z) which is analytic in a domain D. An anti-derivative or indefinite integral (or simply integral)
is a function F (z) which satisfies F 0 (z) = f (z). This integral exists and is unique up to an additive constant. Note
that if the domain is not connected, then the additive constants in each connected component are independent. The
indefinite integrals are denoted:
Z
f (z) dz = F (z) + c.

We will prove existence later by writing an indefinite integral as a contour integral. We briefly consider uniqueness
of the indefinite integral here. Let F (z) and G(z) be integrals of f (z). Then F 0 (z) G0 (z) = f (z) f (z) = 0.
Although we do not prove it, it certainly makes sense that F (z) G(z) is a constant on each connected component
of the domain. Indefinite integrals are unique up to an additive constant.
Integrals of analytic functions have all the nice properties of integrals of functions of a real variable. All the formulas
from integral tables, including things like integration by parts, carry over directly.

473
10.7 Fundamental Theorem of Calculus via Primitives
10.7.1 Line Integrals and Primitives
Here we review some concepts from vector calculus. Analagous to an integral in functions of a single variable is
a primitive in functions of several variables. Consider a function f (x). F (x) is an integral of f (x) if and only if
dF = f dx. Now we move to functions of x and y. Let P (x, y) and Q(x, y) be defined on a simply connected domain.
A primitive satisfies
d = P dx + Q dy.
A necessary and sufficient condition for the existence of a primitive is that Py = Qx . The definite integral can be
evaluated in terms of the primitive.
Z (c,d)
P dx + Q dy = (c, d) (a, b)
(a,b)

10.7.2 Contour Integrals


Now consider integral along the contour C of the function f (z) = (x, y).
Z Z
f (z) dz = ( dx + dy)
C C

A primitive of dx + dy exists if and only if y = x . We recognize this as the Cauch-Riemann equation,


x = y . Thus a primitive exists if and only if f (z) is analytic. If so, then

d = dx + dy.

How do we find the primitive that satisfies x = and y = ? Note that choosing (x, y) = F (z) where F (z)
is an anti-derivative of f (z), F 0 (z) = f (z), does the trick. We express the complex derivative as partial derivatives in
the coordinate directions to show this.

F 0 (z) = f (z) = (x, y), F 0 (z) = x = y

474
From this we see that x = and y = so (x, y) = F (z) is a primitive. Since we can evaluate the line integral
of ( dx + dy),
Z (c,d)
( dx + dy) = (c, d) (a, b),
(a,b)

We can evaluate a definite integral of f in terms of its indefinite integral, F .


Z b
f (z) dz = F (b) F (a)
a

This is the Fundamental Theorem of Calculus for functions of a complex variable.

10.8 Fundamental Theorem of Calculus via Complex Calculus

Result 10.8.1 Constructing an Indefinite Integral. If f (z) is analytic in a simply con-


nected domain D and a is a point in the domain, then
Z z
F (z) = f () d
a

is analytic in D and is an indefinite integral of f (z), (F 0 (z) = f (z)).


Now we consider anti-derivatives and definite integrals without using vector calculus. From real variables we know
that we can construct an integral of f (x) with a definite integral.
Z x
F (x) = f () d
a

Now we will prove the analogous property for functions of a complex variable.
Z z
F (z) = f () d
a

475
Rz
Let f (z) be analytic in a simply connected domain D and let a be a point in the domain. To show that F (z) = a
f () d
is an integral of f (z), we apply the limit definition of differentiation.
F (z + z) F (z)
F 0 (z) = lim
z0 z
Z z+z Z z 
1
= lim f () d f () d
z0 z a a
Z z+z
1
= lim f () d
z0 z z

The integral is independent of path. We choose a straight line connecting z and z + z. We add and subtract
R z+z
zf (z) = z f (z) d from the expression for F 0 (z).
 Z z+z 
0 1
F (z) = lim zf (z) + (f () f (z)) d
z0 z z
Z z+z
1
= f (z) + lim (f () f (z)) d
z0 z z

Since f (z) is analytic, it is certainly continuous. This means that

lim f () = 0.
z

The limit term vanishes as a result of this continuity.


Z z+z
1 1
lim
(f () f (z)) d lim |z| max |f () f (z)|
z0 z z z0 |z| [z...z+z]

= lim max |f () f (z)|


z0 [z...z+z]

=0

Thus F 0 (z) = f (z).

476
This results demonstrates the existence of the indefinite integral. We will use this to prove the Fundamental Theorem
of Calculus for functions of a complex variable.

Result 10.8.2 Fundamental Theorem of Calculus. If f (z) is analytic in a simply con-


nected domain D then Z b
f (z) dz = F (b) F (a)
a
where F (z) is any indefinite integral of f (z).
From Result 10.8.1 we know that Z b
f (z) dz = F (b) + c.
a
(Here we are considering b to be a variable.) The case b = a determines the constant.
Z a
f (z) dz = F (a) + c = 0
a
c = F (a)
This proves the Fundamental Theorem of Calculus for functions of a complex variable.

Example 10.8.1 Consider the integral Z


1
dz
C za
where C is any closed contour that goes around the point z = a once in the positive direction. We use the Fundamental
Theorem of Calculus to evaluate the integral. We start at a point on the contour z a = r e . When we traverse the
contour once in the positive direction we end at the point z a = r e(+2) .
Z
1 za=r e(+2)
dz = [log(z a)]za=r e
C za
= Log r + ( + 2) (Log r + )
= 2

477
10.9 Exercises
Exercise 10.1
C is the arc corresponding to the unit semi-circle, |z| = 1, =(z) 0, directed from z = 1 to z = 1. Evaluate
Z
1. z 2 dz
C
Z
2
2. z dz
C
Z
3. z 2 |dz|
C
Z
2
4. z |dz|
C

Hint, Solution
Exercise 10.2
Evaluate Z
2 +bx)
e(ax dx,

where a, b C and <(a) > 0. Use the fact that


Z
2
ex dx = .

Hint, Solution
Exercise 10.3
Evaluate Z Z
ax2 2
2 e cos(x) dx, and 2 x eax sin(x)dx,
0 0

478
where <(a) > 0 and R.
Hint, Solution
Exercise 10.4
Use an admissible parameterization to evaluate
Z
(z z0 )n dz, nZ
C

for the following cases:


1. C is the circle |z z0 | = 1 traversed in the counterclockwise direction.
2. C is the circle |z z0 2| = 1 traversed in the counterclockwise direction.
3. z0 = 0, n = 1 and C is the closed contour defined by the polar equation
 
2
r = 2 sin
4
Is this result compatible with the results of part (a)?
Hint, Solution
Exercise 10.5
1. Use bounding arguments to show that
Z
z + Log z
lim dz = 0
R CR z3 + 1
where CR is the positive closed contour |z| = R.
2. Place a bound on Z

Log z dz

C
where C is the arc of the circle |z| = 2 from 2 to 2.

479
3. Deduce that Z 2 2

z 1
dz r R + 1
z2 + 1 R2 1
C

where C is a semicircle of radius R > 1 centered at the origin.


Hint, Solution
Exercise 10.6
Let C denote the entire positively oriented boundary of the half disk 0 r 1, 0 in the upper half plane.
Consider the branch
3
f (z) = r e/2 , < <
2 2
of the multi-valued function z 1/2 . Show by separate parametric evaluation of the semi-circle and the two radii constituting
the boundary that Z
f (z) dz = 0.
C

Does the Cauchy-Goursat theorem apply here?


Hint, Solution
Exercise 10.7
Evaluate the following contour integrals using anti-derivatives and justify your approach for each.

1. Z
z 3 + z 3 dz,

C

where C is the line segment from z1 = 1 + to z2 = .

2. Z
sin2 z cos z dz
C

where C is a right-handed spiral from z1 = to z2 = .

480
3.
1 + e
Z

z dz = (1 )
C 2
with
z = e Log z , < Arg z < .
C joins z1 = 1 and z2 = 1, lying above the real axis except at the end points. (Hint: redefine z so that it
remains unchanged above the real axis and is defined continuously on the real axis.)

Hint, Solution

481
10.10 Hints
Hint 10.1

Hint 10.2
2
Let C be the parallelogram in the complex plane with corners at R and R + b/(2a). Consider the integral of eaz
on this contour. Take the limit as R .
Hint 10.3
Extend the range of integration to ( . . . ). Use ex = cos(x) + sin(x) and the result of Exercise 10.2.

Hint 10.4

Hint 10.5

Hint 10.6

Hint 10.7

482
10.11 Solutions

Solution 10.1
We parameterize the path with z = e , with ranging from to 0.

dz = e d
|dz| = | e d| = |d| = d

1.

Z Z 0
2
z dz = e2 e d
C
Z 0
= e3 d

 0
1 3
= e
3
1 0
e e3

=
3
1
= (1 (1))
3
2
=
3

483
2.
Z Z 0
2
|z | dz = | e2 | e d
C
Z 0
= e d

 0
= e
= 1 (1)
=2

3.
Z Z 0
2
z |dz| = e2 | e d|
C
Z 0
= e2 d

h i0
2
= e
2

= (1 1)
2
=0

4.
Z Z 0
2
|z | |dz| = | e2 || e d|
C
Z 0
= d

= []0
=

484
Solution 10.2
Z
2 +bx)
I= e(ax dx

First we complete the square in the argument of the exponential.
Z
b2 /(4a) 2
I=e ea(x+b/(2a)) dx

2
Consider the parallelogram in the complex plane with corners at R and R + b/(2a). The integral of eaz on this
contour vanishes as it is an entire function. We relate the integral along one side of the parallelogram to the integrals
along the other three sides.
Z R+b/(2a) Z R Z R Z R+b/(2a) !
2 2
eaz dz = + + eaz dz.
R+b/(2a) R+b/(2a) R R

The first and third integrals on the right side vanish as R because the integrand vanishes and the lengths of the
paths of integration are finite. Taking the limit as R we have,
Z +b/(2a) Z Z
az 2 a(x+b/(2a))2 2
e dz e dx = eax dx.
+b/(2a)

Now we have Z
b2 /(4a) 2
I=e eax dx.


We make the change of variables = ax.
Z
b2 /(4a) 1 2
I=e e d
a


Z r
(ax2 +bx) b2 /(4a)
e dx = e
a

485
Solution 10.3
Consider Z
2
I=2 eax cos(x) dx.
0
Since the integrand is an even function, Z
2
I= eax cos(x) dx.

2
Since eax sin(x) is an odd function, Z
2
I= eax ex dx.

We evaluate this integral with the result of Exercise 10.2.
Z r
2 2 /(4a)
2 eax cos(x) dx = e
0 a
Consider Z
2
I=2 x eax sin(x) dx.
0
Since the integrand is an even function, Z
2
I= x eax sin(x) dx.

2
Since x eax cos(x) is an odd function,
Z
2
I = x eax ex dx.

We add a dash of integration by parts to get rid of the x factor.
  Z  
1 ax2 x 1 ax2 x
I = e e + e e dx
2a 2a
ax2 x
Z
I= e e dx
2a

486

Z r
ax2 2 /(4a)
2 xe sin(x) dx = e
0 2a a

Solution 10.4
1. We parameterize the contour and do the integration.

z z0 = e , [0 . . . 2)

Z Z 2
n
(z z0 ) dz = en e d
C 0h
e(n+1) 2
i (
n+1
for n 6= 1 0 for n 6= 1
= 0 =
[]2 for n = 1 2 for n = 1
0

2. We parameterize the contour and do the integration.

z z0 = 2 + e , [0 . . . 2)

Z Z 2 n
n
(z z0 ) dz = 2 + e e d
C
0
n+1 2

(2+e )

for n 6= 1
= n+1 =0
0
log 2 + 2
e 0 for n = 1

3. We parameterize the contour and do the integration.


 
2
z = re , r = 2 sin , [0 . . . 4)
4

487
1

-1 1

-1

Figure 10.2: The contour: r = 2 sin2



4
.

The contour encircles the origin twice. See Figure 10.2.

Z Z 4
1 1
z dz =
(r0 () + r()) e d
C 0 r() e
Z 4  0 
r ()
= + d
0 r()
= [log(r()) + ]4
0

488
Since r() does not vanish, the argument of r() does not change in traversing the contour and thus the
logarithmic term has the same value at the beginning and end of the path.
Z
z 1 dz = 4
C

This answer is twice what we found in part (a) because the contour goes around the origin twice.

Solution 10.5
1. We parameterize the contour with z = R e and bound the modulus of the integral.
Z Z
z + Log z z + Log z
dz z 3 + 1 |dz|


CR z3 + 1 CR
Z 2
R + ln R +
R d
0 R3 1
R + ln R +
= 2r
R3 1
The upper bound on the modulus on the integral vanishes as R .

R + ln R +
lim 2r =0
R R3 1
We conclude that the integral vanishes as R .
Z
z + Log z
lim dz = 0
R C
R
z3 + 1

2. We parameterize the contour and bound the modulus of the integral.

z = 2 e , [/2 . . . /2]

489
Z Z

Log z dz |Log z| |dz|

C C
Z /2
= | ln 2 + |2 d
/2
Z /2
2 (ln 2 + ||) d
/2
Z /2
=4 (ln 2 + ) d
0

= ( + 4 ln 2)
2

3. We parameterize the contour and bound the modulus of the integral.

z = R e , [0 . . . 0 + ]

Z 2 Z 2
z 1 z 1
z 2 + 1 dz z 2 + 1 |dz|

C C
Z 0 + 2 2
R e 1
R2 e2 +1 |R d|

0
Z 0 + 2
R +1
R d
0 R2 1
R2 + 1
= r 2
R 1

490
Solution 10.6

Z Z 1
Z Z 0
/2
f (z) dz = r dr + e e d + r (dr)
C 0 0 1
 
2 2 2 2
= + +
3 3 3 3
=0

The Cauchy-Goursat theorem does not apply because the function is not analytic at z = 0, a point on the boundary.
Solution 10.7
1.

z 4
Z 
3 3
 1
z + z dz = 2
C 4 2z 1+
1
= +
2
In this example, the anti-derivative is single-valued.

2.

sin3 z
Z 
2
sin z cos z dz =
C 3
1
sin3 () sin3 ()

=
3
sinh3 ()
=
3
Again the anti-derivative is single-valued.

491
3. We choose the branch of z with /2 < arg(z) < 3/2. This matches the principal value of z above the real
axis and is defined continuously on the path of integration.
e0
z 1+
Z 

z dz =
C 1 + e
 e0
1 (1+) log z
= e
2 e
1 0
e e(1+)

=
2
1 + e
= (1 )
2

492
Chapter 11

Cauchys Integral Formula

If I were founding a university I would begin with a smoking room; next a dormitory; and then a decent reading room
and a library. After that, if I still had more money that I couldnt use, I would hire a professor and get some text books.

- Stephen Leacock

493
11.1 Cauchys Integral Formula

Result 11.1.1 Cauchys Integral Formula. If f () is analytic in a compact, closed, con-


nected domain D and z is a point in the interior of D then
I I
1 f () 1 X f ()
f (z) = d = d. (11.1)
2 D z 2 Ck z k

Here the set of contours {Ck } make up the positively oriented boundary D of the domain
D. More generally, we have
I XI
n! f () n! f ()
f (n) (z) = d = d. (11.2)
2 D ( z)n+1 2 Ck ( z)n+1
k

Cauchys Formula shows that the value of f (z) and all its derivatives in a domain are determined by the value of
f (z) on the boundary of the domain. Consider the first formula of the result, Equation 11.1. We deform the contour
to a circle of radius about the point = z.
I I
f () f ()
d = d
C z C z
f () f (z)
I I
f (z)
= d + d
C z C z

We use the result of Example 10.8.1 to evaluate the first integral.

f () f (z)
I I
f ()
d = 2f (z) + d
C z C z

494
The remaining integral along C vanishes as 0 because f () is continuous. We demonstrate this with the maximum
modulus integral bound. The length of the path of integration is 2.
I  
f () f (z) 1
lim d lim (2) max |f () f (z)|
0 C

z 0 |z|=
 
lim 2 max |f () f (z)|
0 |z|=

=0
This gives us the desired result. I
1 f ()
f (z) = d
2 C z

We derive the second formula, Equation 11.2, from the first by differentiating with respect to z. Note that the
integral converges uniformly for z in any closed subset of the interior of C. Thus we can differentiate with respect to
z and interchange the order of differentiation and integration.
1 dn
I
(n) f ()
f (z) = n
d
2 dz C z
dn f ()
I
1
= d
2 C dz n z
I
n! f ()
= d
2 C ( z)n+1
Example 11.1.1 Consider the following integrals where C is the positive contour on the unit circle. For the third
integral, the point z = 1 is removed from the contour.
I
sin cos z 5 dz

1.
C
I
1
2. dz
C (z 3)(3z 1)

495

Z
3. z dz
C

1. Since sin (cos (z 5 )) is an analytic function inside the unit circle,


I
sin cos z 5 dz = 0

C

1
2. has singularities at z = 3 and z = 1/3. Since z = 3 is outside the contour, only the singularity at
(z3)(3z1)
z = 1/3 will contribute to the value of the integral. We will evaluate this integral using the Cauchy integral
formula. I  
1 1
dz = 2 =
C (z 3)(3z 1) (1/3 3)3 4

3. Since the curve is not closed, we cannot apply the Cauchy integral formula. Note that z is single-valued and
analytic in the complex plane with a branch cut on the negative real axis. Thus we use the Fundamental Theorem
of Calculus.
 e

Z
2 3
z dz = z
C 3 e
2 3/2
e3/2

= e
3
2
= ( )
3
4
=
3

Cauchys Inequality. Suppose the f () is analytic in the closed disk | z| r. By Cauchys integral formula,
I
(n) n! f ()
f (z) = d,
2 C ( z)n+1

496
where C is the circle of radius r centered about the point z. We use this to obtain an upper bound on the modulus of
f (n) (z).
I
(n)
f (z) = n! f ()
d
2 C ( z)n+1


n! f ()
2r max
2 |z|=r ( z)n+1
n!
= n max |f ()|
r |z|=r

Result 11.1.2 Cauchys Inequality. If f () is analytic in | z| r then


(n) n!M

f (z) n
r
where |f ()| M for all | z| = r.

Liouvilles Theorem. Consider a function f (z) that is analytic and bounded, (|f (z)| M ), in the complex plane.
From Cauchys inequality,
M
|f 0 (z)|
r
for any positive r. By taking r , we see that f 0 (z) is identically zero for all z. Thus f (z) is a constant.

Result 11.1.3 Liouvilles Theorem. If f (z) is analytic and |f (z)| is bounded in the complex
plane then f (z) is a constant.

The Fundamental Theorem of Algebra. We will prove that every polynomial of degree n 1 has exactly n
roots, counting multiplicities. First we demonstrate that each such polynomial has at least one root. Suppose that an

497
nth degree polynomial p(z) has no roots. Let the lower bound on the modulus of p(z) be 0 < m |p(z)|. The function
f (z) = 1/p(z) is analytic, (f 0 (z) = p0 (z)/p2 (z)), and bounded, (|f (z)| 1/m), in the extended complex plane. Using
Liouvilles theorem we conclude that f (z) and hence p(z) are constants, which yields a contradiction. Therefore every
such polynomial p(z) must have at least one root.
Now we show that we can factor the root out of the polynomial. Let
n
X
p(z) = pk z k .
k=0

We note that
n1
X
n n
(z c ) = (z c) cn1k z k .
k=0
th
Suppose that the n degree polynomial p(z) has a root at z = c.
p(z) = p(z) p(c)
Xn n
X
= pk z k pk ck
k=0 k=0
n
X
pk z k ck

=
k=0
Xn k1
X
= pk (z c) ck1j z j
k=0 j=0

= (z c)q(z)
Here q(z) is a polynomial of degree n 1. By induction, we see that p(z) has exactly n roots.

Result 11.1.4 Fundamental Theorem of Algebra. Every polynomial of degree n 1 has


exactly n roots, counting multiplicities.

498
Gauss Mean Value Theorem. Let f () be analytic in | z| r. By Cauchys integral formula,
I
1 f ()
f (z) = d,
2 C z

where C is the circle | z| = r. We parameterize the contour with = z + r e .


Z 2
1 f (z + r e )
f (z) = r e d
2 0 r e
Writing this in the form, Z 2
1
f (z) = f (z + r e )r d,
2r 0

we see that f (z) is the average value of f () on the circle of radius r about the point z.

Result 11.1.5 Gauss Average Value Theorem. If f () is analytic in | z| r then


Z 2
1
f (z) = f (z + r e ) d.
2 0
That is, f (z) is equal to its average value on a circle of radius r about the point z.

Extremum Modulus Theorem. Let f (z) be analytic in closed, connected domain, D. The extreme values of the
modulus of the function must occur on the boundary. If |f (z)| has an interior extrema, then the function is a constant.
We will show this with proof by contradiction. Assume that |f (z)| has an interior maxima at the point z = c. This
means that there exists an neighborhood of the point z = c for which |f (z)| |f (c)|. Choose an  so that the set
|z c|  lies inside this neighborhood. First we use Gauss mean value theorem.
Z 2
1
f c +  e d

f (c) =
2 0

499
We get an upper bound on |f (c)| with the maximum modulus integral bound.

Z 2
1
f c +  e d

|f (c)|
2 0

Since z = c is a maxima of |f (z)| we can get a lower bound on |f (c)|.

Z 2
1
f c +  e d

|f (c)|
2 0

If |f (z)| < |f (c)| for any point on |z c| = , then the continuity of f (z) implies that |f (z)| < |f (c)| in a neighborhood
of that point which would make the value of the integral of |f (z)| strictly less than |f (c)|. Thus we conclude that
|f (z)| = |f (c)| for all |z c| = . Since we can repeat the above procedure for any circle of radius smaller than ,
|f (z)| = |f (c)| for all |z c| , i.e. all the points in the disk of radius  about z = c are also maxima. By recursively
repeating this procedure points in this disk, we see that |f (z)| = |f (c)| for all z D. This implies that f (z) is a
constant in the domain. By reversing the inequalities in the above method we see that the minimum modulus of f (z)
must also occur on the boundary.

Result 11.1.6 Extremum Modulus Theorem. Let f (z) be analytic in a closed, connected
domain, D. The extreme values of the modulus of the function must occur on the boundary.
If |f (z)| has an interior extrema, then the function is a constant.

500
11.2 The Argument Theorem

Result 11.2.1 The Argument Theorem. Let f (z) be analytic inside and on C except for
isolated poles inside the contour. Let f (z) be nonzero on C.
Z 0
1 f (z)
dz = N P
2 C f (z)
Here N is the number of zeros and P the number of poles, counting multiplicities, of f (z)
inside C.

First we will simplify the problem and consider a function f (z) that has one zero or one pole. Let f (z) be analytic
and nonzero inside and on A except for a zero of order n at z = a. Then we can write f (z) = (z a)n g(z) where g(z)
0 (z)
is analytic and nonzero inside and on A. The integral of ff (z) along A is

f 0 (z)
Z Z
1 1 d
dz = (log(f (z))) dz
2 A f (z) 2 A dz
Z
1 d
= (log((z a)n ) + log(g(z))) dz
2 A dz
Z
1 d
= (log((z a)n )) dz
2 A dz
Z
1 n
= dz
2 A z a
=n

501
Now let f (z) be analytic and nonzero inside and on B except for a pole of order p at z = b. Then we can write
g(z) f 0 (z)
f (z) = (zb) p where g(z) is analytic and nonzero inside and on B. The integral of f (z) along B is
Z 0 Z
1 f (z) 1 d
dz = (log(f (z))) dz
2 B f (z) 2 B dz
Z
1 d
log((z b)p ) + log(g(z)) dz

=
2 B dz
Z
1 d
log((z b)p )+ dz

=
2 B dz
p
Z
1
= dz
2 B z b
= p

Now consider a function f (z) that is analytic inside an on the contour C except for isolated poles at the points
b1 , . . . , bp . Let f (z) be nonzero except at the isolated points a1 , . . . , an . Let the contours Ak , k = 1, . . . , n, be simple,
positive contours which contain the zero at ak but no other poles or zeros of f (z). Likewise, let the contours Bk ,
k = 1, . . . , p be simple, positive contours which contain the pole at bk but no other poles of zeros of f (z). (See
Figure 11.1.) By deforming the contour we obtain
Z 0 n Z p Z
f (z) X f 0 (z) X f 0 (z)
dz = dz + dz.
C f (z) j=1 A j
f (z) k=1 B j
f (z)
From this we obtain Result 11.2.1.

11.3 Rouches Theorem

Result 11.3.1 Rouches Theorem. Let f (z) and g(z) be analytic inside and on a simple,
closed contour C. If |f (z)| > |g(z)| on C then f (z) and f (z) + g(z) have the same number
of zeros inside C and no zeros on C.

502
A1 C
B3
B1
A2
B2

Figure 11.1: Deforming the contour C.

First note that since |f (z)| > |g(z)| on C, f (z) is nonzero on C. The inequality implies that |f (z) + g(z)| > 0
on C so f (z) + g(z) has no zeros on C. We well count the number of zeros of f (z) and g(z) using the Argument
Theorem, (Result 11.2.1). The number of zeros N of f (z) inside the contour is
I 0
1 f (z)
N= dz.
2 C f (z)
Now consider the number of zeros M of f (z) + g(z). We introduce the function h(z) = g(z)/f (z).
I 0
1 f (z) + g 0 (z)
M= dz
2 C f (z) + g(z)
I 0
1 f (z) + f 0 (z)h(z) + f (z)h0 (z)
= dz
2 C f (z) + f (z)h(z)
I 0
h0 (z)
I
1 f (z) 1
= dz + dz
2 C f (z) 2 C 1 + h(z)
1
=N+ [log(1 + h(z))]C
2
=N

503
(Note that since |h(z)| < 1 on C, <(1 + h(z)) > 0 on C and the value of log(1 + h(z)) does not not change in
traversing the contour.) This demonstrates that f (z) and f (z) + g(z) have the same number of zeros inside C and
proves the result.

504
11.4 Exercises
Exercise 11.1
What is
(arg(sin z)) C
where C is the unit circle?
Exercise 11.2
Let C be the circle of radius 2 centered about the origin and oriented in the positive direction. Evaluate the following
integrals:
1. C zsin z
H
2 +5 dz

2. C z2z+1 dz
H

H 2
3. C z z+1 dz

Exercise 11.3
Let f (z) be analytic and bounded (i.e. |f (z)| < M ) for |z| > R, but not necessarily analytic for |z| R. Let the
points and lie inside the circle |z| = R. Evaluate
I
f (z)
dz
C (z )(z )

where C is any closed contour outside |z| = R, containing the circle |z| = R. [Hint: consider the circle at infinity] Now
suppose that in addition f (z) is analytic everywhere. Deduce that f () = f ().

Exercise 11.4
Using Rouches theorem show that all the roots of the equation p(z) = z 6 5z 2 + 10 = 0 lie in the annulus 1 < |z| < 2.

Exercise 11.5
Evaluate as a function of t
ezt
I
1
= dz,
2 C z 2 (z 2 + a2 )

505
where C is any positively oriented contour surrounding the circle |z| = a.

Exercise 11.6
Consider C1 , (the positively oriented circle |z| = 4), and C2 , (the positively oriented boundary of the square whose
sides lie along the lines x = 1, y = 1). Explain why
Z Z
f (z) dz = f (z) dz
C1 C2

for the functions


1
1. f (z) =
3z 2
+1
z
2. f (z) =
1 ez
Exercise 11.7
Show that if f (z) is of the form
k k1 1
f (z) = + + + + g(z), k1
zk z k1 z
where g is analytic inside and on C, (the positive circle |z| = 1), then
Z
f (z) dz = 21 .
C

Exercise 11.8
Show that if f (z) is analytic within and on a simple closed contour C and z0 is not on C then

f 0 (z)
Z Z
f (z)
dz = 2
dz.
C z z0 C (z z0 )

Note that z0 may be either inside or outside of C.

506
Exercise 11.9
If C is the positive circle z = e show that for any real constant a,
Z az
e
dz = 2
C z

and hence Z
ea cos cos(a sin ) d = .
0

Exercise 11.10
Use Cauchy-Goursat, the generalized Cauchy integral formula, and suitable extensions to multiply-connected domains
to evaluate the following integrals. Be sure to justify your approach in each case.
1. Z
z
3
dz
C z 9
where C is the positively oriented rectangle whose sides lie along x = 5, y = 3.
2. Z
sin z
dz,
C z 2 (z 4)
where C is the positively oriented circle |z| = 2.
3.
(z 3 + z + ) sin z
Z
dz,
C z 4 + z 3
where C is the positively oriented circle |z| = .
4.
ezt
Z
dz
C z 2 (z + 1)
where C is any positive simple closed contour surrounding |z| = 1.

507
Exercise 11.11
Use Liouvilles theorem to prove the following:

1. If f (z) is entire with <(f (z)) M for all z then f (z) is constant.

2. If f (z) is entire with |f (5) (z)| M for all z then f (z) is a polynomial of degree at most five.

Exercise 11.12
Find all functions f (z) analytic in the domain D : |z| < R that satisfy f (0) = e and |f (z)| 1 for all z in D.

Exercise 11.13
4 z k
Let f (z) =
P 
k=0 k 4
and evaluate the following contour integrals, providing justification in each case:
Z
1. cos(z)f (z) dz C is the positive circle |z 1| = 1.
C
Z
f (z)
2. dz C is the positive circle |z| = .
C z3

508
11.5 Hints
Hint 11.1
Use the argument theorem.

Hint 11.2

Hint 11.3
To evaluate the integral, consider the circle at infinity.

Hint 11.4

Hint 11.5

Hint 11.6

Hint 11.7

Hint 11.8

Hint 11.9

Hint 11.10

509
Hint 11.11

Hint 11.12

Hint 11.13

510
11.6 Solutions
Solution 11.1
Let f (z) be analytic inside and on the contour C. Let f (z) be nonzero on the contour. The argument theorem states
that Z 0
1 f (z)
dz = N P,
2 C f (z)
where N is the number of zeros and P is the number of poles, (counting multiplicities), of f (z) inside C. The theorem
is aptly named, as
Z 0
1 f (z) 1
dz = [log(f (z))]C
2 C f (z) 2
1
= [log |f (z)| + arg(f (z))]C
2
1
= [arg(f (z))]C .
2
Thus we could write the argument theorem as
Z 0
1 f (z) 1
dz = [arg(f (z))]C = N P.
2 C f (z) 2
Since sin z has a single zero and no poles inside the unit circle, we have
1
arg(sin(z)) C = 1 0
2

arg(sin(z)) = 2
C

Solution 11.2


1. Since the integrand zsin z
2 +5 is analytic inside and on the contour, (the only singularities are at z = 5 and at
infinity), the integral is zero by Cauchys Theorem.

511
2. First we expand the integrand in partial fractions.
z a b
= +
z2 +1 z z+

z 1 z 1
a= = , b= =
z + z= 2
z z= 2

Now we can do the integral with Cauchys formula.


Z Z Z
z 1/2 1/2
2
dz = dz + dz
C z +1 C z C z +
1 1
= 2 + 2
2 2
= 2

3.
z2 + 1
Z Z  
1
dz = z+ dz
C z C z
Z Z
1
= z dz + dz
C C z
= 0 + 2
= 2

Solution 11.3
Let C be the circle of radius r, (r > R), centered at the origin. We get an upper bound on the integral with the
Maximum Modulus Integral Bound, (Result 10.2.1).
I
f (z)
2r max
f (z)
2r M

(z )(z ) dz
C |z|=r (z )(z )
(r ||)(r ||)

512
By taking the limit as r we see that the modulus of the integral is bounded above by zero. Thus the integral
vanishes.
Now we assume that f (z) is analytic and evaluate the integral with Cauchys Integral Formula. (We assume that
6= .)
I
f (z)
dz = 0
C (z )(z )
I I
f (z) f (z)
dz + dz = 0
C (z )( ) C ( )(z )
f () f ()
2 + 2 =0

f () = f ()

Solution 11.4
Consider the circle |z| = 2. On this circle:

|z 6 | = 64
| 5z 2 + 10| | 5z 2 | + |10| = 30

Since |z 6 | < | 5z 2 + 10| on |z| = 2, p(z) has the same number of roots as z 6 in |z| < 2. p(z) has 6 roots in |z| < 2.
Consider the circle |z| = 1. On this circle:

|10| = 10
|z 6 5z 2 | |z 6 | + | 5z 2 | = 6

Since |z 6 5z 2 | < |10| on |z| = 1, p(z) has the same number of roots as 10 in |z| < 1. p(z) has no roots in |z| < 1.
On the unit circle,
|p(z)| |10| |z 6 | |5z 2 | = 4.
Thus p(z) has no roots on the unit circle.
We conclude that p(z) has exactly 6 roots in 1 < |z| < 2.

513
Solution 11.5
We evaluate the integral with Cauchys Integral Formula.

ezt
I
1
= dz
2 C z 2 (z 2 + a2 )
I  zt
ezt ezt

1 e
= + dz
2 C a2 z 2 2a3 (z a) 2a3 (z + a)
d ezt eat eat
 
= +
dz a2 z=0 2a3 2a3
t sin(at)
= 2
a a3
at sin(at)
=
a3
Solution 11.6
1. We factor the denominator of the integrand.
1 1
=
3z 2+1 3(z 3/3)(z + 3/3)
There are two first order poles which could contribute to the value of an integral on a closed path. Both poles
lie inside both contours. See Figure 11.2. We see that C1 can be continuously deformed to C2 on the domain
where the integrand is analytic. Thus the integrals have the same value.
2. We consider the integrand
z
.
1 ez
Since ez = 1 has the solutions z = 2n for n Z, the integrand has singularities at these points. There is a
removable singularity at z = 0 and first order poles at z = 2n for n Z \ {0}. Each contour contains only the
singularity at z = 0. See Figure 11.3. We see that C1 can be continuously deformed to C2 on the domain where
the integrand is analytic. Thus the integrals have the same value.

514
4

-4 -2 2 4
-2

-4

1
Figure 11.2: The contours and the singularities of 3z 2 +1
.

6
4
2

-6 -4 -2 2 4 6
-2
-4
-6

z
Figure 11.3: The contours and the singularities of 1ez
.

515
Solution 11.7
First we write the integral of f (z) as a sum of integrals.
Z Z 
k k1 1 
f (z) dz = + + + + g(z) dz
C C zk z k1 z
Z Z Z Z
k k1 1
= k
dz + k1
dz + + dz + g(z) dz
C z C z C z C

The integral of g(z) vanishes by the Cauchy-Goursat theorem. We evaluate the integral of 1 /z with Cauchys integral
formula. Z
1
dz = 21
C z

We evaluate the remaining n /z n terms with anti-derivatives. Each of these integrals vanish.
Z Z Z Z Z
k k1 1
f (z) dz = k
dz + k1
dz + + dz + g(z) dz
C C z C z C z C
 
k h i
2
= k1
+ + + 21
(k 1)z C z C
= 21

Solution 11.8
We evaluate the integrals with the Cauchy integral formula. (z0 is required to not be on C so the integrals exist.)
(
f 0 (z) 2f 0 (z0 )
Z
if z0 is inside C
dz =
C z z0 0 if z0 is outside C
(
Z 2 0
f (z) 1!
f (z0 ) if z0 is inside C
2
dz =
C (z z0 ) 0 if z0 is outside C

Thus we see that the integrals are equal.

516
Solution 11.9
First we evaluate the integral using the Cauchy Integral Formula.
Z az
e
dz = [eaz ]z=0 = 2
C z
Next we parameterize the path of integration. We use the periodicity of the cosine and sine to simplify the integral.
Z az
e
dz = 2
C z
Z 2 a e
e

e d = 2
0 e
Z 2
ea(cos + sin ) d = 2
0
Z 2
ea cos (cos(sin ) + sin(sin )) d = 2
0
Z 2
ea cos cos(sin ) d = 2
Z0
ea cos cos(sin ) d =
0

Solution 11.10
1. We factor the integrand to see that there are singularities at the cube roots of 9.
z z
=
z3 9
  
3 3
z 9 z 9 e2/3 z 3 9 e2/3

Let C1 , C2 and C3 be contours around z = 3 9, z = 3 9 e2/3 and z = 3 9 e2/3 . See Figure 11.4. Let D be
the domain between C, C1 and C2 , i.e. the boundary of D is the union of C, C1 and C2 . Since the integrand
is analytic in D, the integral along the boundary of D vanishes.
Z Z Z Z Z
z z z z z
3
dz = 3
dz + 3
dz + 3
dz + 3
dz = 0
D z 9 C z 9 C1 z 9 C2 z 9 C3 z 9

517
From this we see that the integral along C is equal to the sum of the integrals along C1 , C2 and C3 . (We could
also see this by deforming C onto C1 , C2 and C3 .)
Z Z Z Z
z z z z
3
dz = 3
dz + 3
dz + dz
C z 9 C1 z 9 C2 z 9 C3 z3 9

We use the Cauchy Integral Formula to evaluate the integrals along C1 , C2 and C2 .
Z Z
z z
dz =  dz
z3 9 3
 
C C1 z 9 z 9 e2/3 z 3 9 e2/3
3

Z
z
+ 3
 3
  dz
C2 z 9 z 9e 2/3 z 3 9 e2/3
Z
z
+ 3
   dz
C3 z 9 z 9 e2/3 z 3 9 e2/3
3

" #
z
= 2  
z 3 9 e2/3 z 3 9 e2/3 z= 3
9
" #
z
+ 2  
3
z 9 z 3 9 e2/3 z= 3
9 e2/3
" #
z
+ 2  
z 9 z 3 9 e2/3 z=
3
3
9 e2/3
5/3 /3 2/3

= 23 1e +e
=0

2. The integrand has singularities at z = 0 and z = 4. Only the singularity at z = 0 lies inside the contour. We use

518
4
C
C2 2
C1
-6 -4 -2 2 4 6
C3 -2

-4

z
Figure 11.4: The contours for z 3 9
.

the Cauchy Integral Formula to evaluate the integral.


Z  
sin z d sin z
2
dz = 2
C z (z 4) dz z 4 z=0
 
cos z sin z
= 2
z 4 (z 4)2 z=0

=
2

3. We factor the integrand to see that there are singularities at z = 0 and z = .


(z 3 + z + ) sin z (z 3 + z + ) sin z
Z Z
dz = dz
C z 4 + z 3 C z 3 (z + )
Let C1 and C2 be contours around z = 0 and z = . See Figure 11.5. Let D be the domain between C, C1 and
C2 , i.e. the boundary of D is the union of C, C1 and C2 . Since the integrand is analytic in D, the integral
along the boundary of D vanishes. Z Z Z Z
= + + =0
D C C1 C2

519
From this we see that the integral along C is equal to the sum of the integrals along C1 and C2 . (We could also
see this by deforming C onto C1 and C2 .)
Z Z Z
= +
C C1 C2

We use the Cauchy Integral Formula to evaluate the integrals along C1 and C2 .

(z 3 + z + ) sin z (z 3 + z + ) sin z (z 3 + z + ) sin z


Z Z Z
dz = dz + dz
C z 4 + z 3 C1 z 3 (z + ) C2 z 3 (z + )
 3
2 d2 (z 3 + z + ) sin z
  
(z + z + ) sin z
= 2 +
z3 z= 2! dz 2 z+ z=0
  2 3

3z + 1 z + z +
= 2( sinh(1)) + 2 cos z
z+ (z + )2
2(3z 2 + 1) 2(z 3 + z + ) z 3 + z +
  
6z
+ + sin z
z+ (z + )2 (z + )3 z+ z=0
= 2 sinh(1)

4. We consider the integral


ezt
Z
dz.
C z 2 (z + 1)
There are singularities at z = 0 and z = 1.

Let C1 and C2 be contours around z = 0 and z = 1. See Figure 11.6. We deform C onto C1 and C2 .
Z Z Z
= +
C C1 C2

520
4

C1 C
-4 -2 C2 2 4

-2

-4

(z 3 +z+) sin z
Figure 11.5: The contours for z 4 +z 3
.

We use the Cauchy Integral Formula to evaluate the integrals along C1 and C2 .
ezt ezt ezt
Z Z Z
2
dz = 2
dz + 2
dz
C z (z + 1) C1 z (z + 1) C1 z (z + 1)
 zt 
d ezt
 
e
= 2 2 + 2
z z=1 dz (z + 1) z=0
zt ezt
 
t te
= 2 e +2
(z + 1) (z + 1)2 z=0
= 2(et +t 1)

Solution 11.11
Liouvilles Theorem states that if f (z) is analytic and bounded in the complex plane then f (z) is a constant.

521
2

1
C2 C1 C
-2 -1 1 2
-1

-2

ezt
Figure 11.6: The contours for z 2 (z+1)
.

1. Since f (z) is analytic, ef (z) is analytic. The modulus of ef (z) is bounded.


e = e<(f (z)) eM
f (z)

By Liouvilles Theorem we conclude that ef (z) is constant and hence f (z) is constant.
2. We know that f (z) is entire and |f (5) (z)| is bounded in the complex plane. Since f (z) is analytic, so is f (5) (z).
We apply Liouvilles Theorem to f (5) (z) to conclude that it is a constant. Then we integrate to determine the
form of f (z).
f (z) = c5 z 5 + c4 z 4 + c3 z 3 + c2 z 2 + c1 z + c0
Here c5 is the value of f (5) (z) and c4 through c0 are constants of integration. We see that f (z) is a polynomial
of degree at most five.
Solution 11.12
For this problem we will use the Extremum Modulus Theorem: Let f (z) be analytic in a closed, connected domain, D.
The extreme values of the modulus of the function must occur on the boundary. If |f (z)| has an interior extrema, then
the function is a constant.

522
Since |f (z)| has an interior extrema, |f (0)| = | e | = 1, we conclude that f (z) is a constant on D. Since we know
the value at z = 0, we know that f (z) = e .

Solution 11.13
First we determine the radius of convergence of the series with the ratio test.
4 k

k /4
R = lim 4 k+1

k (k + 1) /4
k4
= 4 lim
k (k + 1)4
24
= 4 lim
k 24
=4

The series converges absolutely for |z| < 4.

1. Since the integrand is analytic inside and on the contour of integration, the integral vanishes by Cauchys Theorem.

2.
Z
Z X
f (z)  z k 1
dz = k4 dz
C z3 C k=0 4 z3

k 4 k3
Z X
= k
z dz
C k=1 4

(k + 3)4 k
Z X
= z dz
C k=2 4k+3

(k + 3)4 k
Z Z Z X
1 1
= 2
dz + dz + z dz
C 4z C z C k=0 4k+3

523
We can parameterize the first integral to show that it vanishes. The second integral has the value 2 by the
Cauchy-Goursat Theorem. The third integral vanishes by Cauchys Theorem as the integrand is analytic inside
and on the contour. Z
f (z)
3
dz = 2
C z

524
Chapter 12

Series and Convergence

You are not thinking. You are merely being logical.

- Neils Bohr

12.1 Series of Constants


12.1.1 Definitions
Convergence of Sequences. The infinite sequence {an }
n=0 a0 , a1 , a2 , . . . is said to converge if

lim an = a
n

for some constant a. If the limit does not exist, then the sequence diverges. Recall the definition of the limit in the
above formula: For any  > 0 there exists an N Z such that |a an | <  for all n > N .

Example 12.1.1 The sequence {sin(n)} is divergent. The sequence is bounded above and below, but boundedness
does not imply convergence.

525
Cauchy Convergence Criterion. Note that there is something a little fishy about the above definition. We
should be able to say if a sequence converges without first finding the constant to which it converges. We fix this
problem with the Cauchy convergence criterion. A sequence {an } converges if and only if for any  > 0 there exists an
N such that |an am | <  for all n, m > N . The Cauchy convergence criterion is equivalent to the definition we had
before. For some problems it is handier to use. Now we dont need to know the limit of a sequence to show that it
converges.
P PN 1
Convergence of Series. The series n=1 an converges if the sequence of partial sums, SN = n=0 an , converges.
That is,
N
X 1
lim SN = lim an = constant.
N N
n=0

If the limit does not exist, then the series diverges. A necessary condition for the convergence of a series is that

lim an = 0.
n

(See Exercise 12.1.) Otherwise the sequence of partial sums would not converge.

Example 12.1.2 The series n


P
n=0 (1) = 1 1 + 1 1 + is divergent because the sequence of partial sums,
{SN } = 1, 0, 1, 0, 1, 0, . . . is divergent.

Tail of a Series. AnPinfinite series,


P P
n=0 an , converges or diverges with its tail. That is, for fixed N , n=0 an

converges if and only if n=N an converges. This is because the sum of the first N terms of a series is just a number.
Adding or subtracting a number to a series does not change its convergence.

Absolute Convergence. The series


P P
n=0 an converges absolutely if n=0 |an | converges. Absolute convergence
implies convergence. If a series is convergent, but not absolutely convergent, then it is said to be conditionally
convergent.
The terms of an absolutely convergent series can be rearranged in any order and the series will still converge to
the same sum. This is not true of conditionally convergent series. Rearranging the terms of a conditionally convergent

526
series may change the sum. In fact, the terms of a conditionally convergent series may be rearranged to obtain any
desired sum.

Example 12.1.3 The alternating harmonic series,


1 1 1
1 + + ,
2 3 4
converges, (Exercise 12.4). Since
1 1 1
1+ + + +
2 3 4
diverges, (Exercise 12.5), the alternating harmonic series is not absolutely convergent. Thus the terms can be rearranged
to obtain any sum, (Exercise 12.6).

Finite Series and Residuals. Consider the series f (z) =


P
n=0 an (z). We will denote the sum of the first N
terms in the series as
N
X 1
SN (z) = an (z).
n=0
We will denote the residual after N terms as

X
RN (z) f (z) SN (z) = an (z).
n=N

12.1.2 Special Series


1
Geometric Series. One of the most important series in mathematics is the geometric series,

X
zn = 1 + z + z2 + z3 + .
n=0
1
The series is so named because the terms grow or decay geometrically. Each term in the series is a constant times the previous
term.

527
The series clearly diverges for |z| 1 since the terms do not vanish as n . Consider the partial sum, SN (z)
P N 1 n
n=0 z , for |z| < 1.

N
X 1
(1 z)SN (z) = (1 z) zn
n=0
N
X 1 XN
= zn zn
n=0 n=1
= 1 + z + + z N 1 z + z 2 + + z N
 

= 1 zN

N 1
X 1 zN 1
zn = as N .
n=0
1z 1z
1
The limit of the partial sums is 1z
.

X 1
zn = for |z| < 1
n=0
1z

Harmonic Series. Another important series is the harmonic series,



X 1 1 1

= 1 + + + .
n=1
n 2 3

The series is absolutely convergent for <() > 1 and absolutely divergent for <() 1, (see the Exercise 12.8). The
Riemann zeta function () is defined as the sum of the harmonic series.

X 1
() =
n=1
n

528
The alternating harmonic series is

X (1)n+1 1 1 1
=1 + + .
n=1
n 2 3 4

Again, the series is absolutely convergent for <() > 1 and absolutely divergent for <() 1.

12.1.3 Convergence Tests


The Comparison Test.
P
Result 12.1.1 The series of positive terms an converges if there exists a convergent series
P P
P bn such that an bn for all n. Similarly,
an diverges if there exists a divergent series
bn such that an bn for all n.

Example 12.1.4 Consider the series



X 1
.
n=1
2n2
We can rewrite this as
X 1
.
n=1
2n
n a perfect square

Then by comparing this series to the geometric series,



X 1
n
= 1,
n=1
2

we see that it is convergent.

529
Integral Test.

Result 12.1.2 If the coefficients an of a series


P
n=0 an are monotonically decreasing and
can be extended to a monotonically decreasing function of the continuous variable x,

a(x) = an for x Z0+ ,

then the series converges or diverges with the integral


Z
a(x) dx.
0

P 1
Example 12.1.5 Consider the series n=1 n2 . Define the functions sl (x) and sr (x), (left and right),

1 1
sl (x) = , sr (x) = .
(dxe)2 (bxc)2

Recall that bxc is the greatest integer function, the greatest integer which is less than or equal to x. dxe is the least
integer function, the least integer greater than or equal to x. We can express the series as integrals of these functions.

Z Z
X 1
= sl (x) dx = sr (x) dx
n=1
n2 0 1

In Figure 12.1 these functions are plotted against y = 1/x2 . From the graph, it is clear that we can obtain a lower and

530
upper bound for the series.
Z Z
1 X 1 1
dx 1+ dx
1 x2 n=1
n 2
1 x2

X 1
1 2
n=1
n2

1 1

1 2 3 4 1 2 3 4
P
Figure 12.1: Upper and Lower bounds to n=1 1/n2 .

In general, we have
Z
X Z
a(x) dx an am + a(x) dx.
m n=m m

Thus we see that the sum converges or diverges with the integral.

The Ratio Test.


P
Result 12.1.3 The series an converges absolutely if

an+1
lim < 1.
n an

If the limit is greater than unity, then the series diverges. If the limit is unity, the test fails.

531
If the limit is greater than unity, then the terms are eventually increasing with n. Since the terms do not vanish,
the sum is divergent. If the limit is less than unity, then there exists some N such that

an+1
an r < 1 for all n N.

From this we can show that


P
n=0 an is absolutely convergent by comparing it to the geometric series.


X
X
|an | |aN | rn
n=N n=0
1
= |aN |
1r

Example 12.1.6 Consider the series,



X en
.
n=1
n!

We apply the ratio test to test for absolute convergence.


n+1

= lim e
an+1 n!
lim

n
n an n e (n + 1)!
e
= lim
n n + 1

=0

The series is absolutely convergent.

Example 12.1.7 Consider the series,



X 1
,
n=1
n2

532
which we know to be absolutely convergent. We apply the ratio test.

2

an+1
lim = lim 1/(n + 1)
n an n 1/n2
n2
= lim 2
n n + 2n + 1
1
= lim
n 1 + 2/n + 1/n2

=1

The test fails to predict the absolute convergence of the series.

The Root Test.


P
Result 12.1.4 The series an converges absolutely if

lim |an |1/n < 1.


n

If the limit is greater than unity, then the series diverges. If the limit is unity, the test fails.
More generally, we can test that

lim sup |an |1/n < 1.


If the limit is greater than unity, then the terms in the series do not vanish as n . This implies that the sum
does not converge. If the limit is less than unity, then there exists some N such that

|an |1/n r < 1 for all n N.

533
We bound the tail of the series of |an |.

X X n
|an | = |an |1/n
n=N n=N
X
rn
n=N
N
r
=
1r
P
n=0 an is absolutely convergent.

Example 12.1.8 Consider the series



X
n a bn ,
n=0
where a and b are real constants. We use the root test to check for absolute convergence.
lim |na bn |1/n < 1
n

|b| lim na/n < 1


n
 
1 ln n
|b| exp lim <1
n n
|b| e0 < 1
|b| < 1
Thus we see that the series converges absolutely for |b| < 1. Note that the value of a does not affect the absolute
convergence.

Example 12.1.9 Consider the absolutely convergent series,



X 1
.
n=1
n2

534
We aply the root test.

1/n
1/n
1
lim |an | = lim 2
n n n

= lim n2/n
n
2
= lim e n ln n
n
= e0
=1

It fails to predict the convergence of the series.

Raabes Test

P
Result 12.1.5 The series an converges absolutely if
 
an+1
lim n 1 > 1.
n an
If the limit is less than unity, then the series diverges or converges conditionally. If the limit
is unity, the test fails.

535
Gauss Test
P
Result 12.1.6 Consider the series an . If
an+1 L bn
=1 + 2
an n n
where bn is bounded then the series converges absolutely if L > 1. Otherwise the series
diverges or converges conditionally.

12.2 Uniform Convergence


Continuous Functions. A function f (z) is continuous in a closed domain if, given any  > 0, there exists a > 0
such that |f (z) f ()| <  for all |z | < in the domain.
An equivalent definition is that f (z) is continuous in a closed domain if
lim f () = f (z)
z

for all z in the domain.

Convergence. Consider a series in which the terms are functions of z,


P
n=0 an (z). The series is convergent in a
domain if the series converges for each point z in the domain. We can then define the function f (z) =
P
n=0 n (z).
a
We can state the convergence criterion as: For any given  > 0 there exists a function N (z) such that

N (z)1
X
|f (z) SN (z) (z)| = f (z)
an (z) < 
n=0

for all z in the domain. Note that the rate of convergence, i.e. the number of terms, N (z) required for for the absolute
error to be less than , is a function of z.

536
Uniform Convergence. Consider a series
P
n=0 an (z) that is convergent in some domain. If the rate of convergence
is independent of z then the series is said to be uniformly convergent. Stating this a little more mathematically, the
series is uniformly convergent in the domain if for any given  > 0 there exists an N , independent of z, such that
N

X
|f (z) SN (z)| = f (z) an (z) < 


n=1

for all z in the domain.

12.2.1 Tests for Uniform Convergence


Weierstrass M-test. The Weierstrass M-test is useful in determining if a series is uniformly convergent. The series
P
Pn=0 an (z) is uniformly and absolutely convergent in a domain if there exists a convergent series of positive terms

n=0 Mn such that |an (z)| Mn for all z in the domain. This condition first implies that the series is absolutely
convergent for all z in the domain. The condition |an (z)| Mn also ensures that the rate of convergence is independent
of z, which is the criterion for uniform convergence.
Note that absolute convergence and uniform convergence are independent. A series of functions may be absolutely
convergent without being uniformly convergent or vice versa. The Weierstrass M-test is a sufficient but not a necessary
condition for uniform convergence. The Weierstrass M-test can succeed only if the series is uniformly and absolutely
convergent.

Example 12.2.1 The series



X sin x
f (x) =
n=1
n(n + 1)

sin x 1
P 1
is uniformly and absolutely convergent for all real x because | n(n+1) |< n2
and n=1 n2 converges.

537
Dirichlet Test. Consider a sequence of monotone decreasing, positive constants cn with limit zero. If all the partial
sums of an (z) are bounded in some closed domain, that is

X N
an (z) < constant


n=1

for all N , then


P
n=1 cn an (z) is uniformly convergent in that closed domain. Note that the Dirichlet test does not
imply that the series is absolutely convergent.

Example 12.2.2 Consider the series,



X sin(nx)
.
n=1
n

We cannot use the Weierstrass M-test to determine if the series is uniformly convergent on an interval. While it is easy
to bound the terms with | sin(nx)/n| 1/n, the sum

X 1
n=1
n

PN 1
does not converge. Thus we will try the Dirichlet test. Consider the sum n=1 sin(nx). This sum can be evaluated
in closed form. (See Exercise 12.9.)

1
N
(
X 0 for x = 2k
sin(nx) = cos(x/2)cos((N 1/2)x)
n=1 2 sin(x/2)
6 2k
for x =

The partial sums have infinite discontinuities at x = 2k, k Z. The partial sumsPare bounded on any closed interval
that does not contain an integer multiple of 2. By the Dirichlet test, the sum n=1
sin(nx)
n
is uniformly convergent
on any such closed interval. The series may not be uniformly convergent in neighborhoods of x = 2k.

538
12.2.2 Uniform Convergence and Continuous Functions.
Consider a series f (z) =
P
n=1 an (z) that is uniformly convergent in some domain and whose terms an (z) are continuous
functions. Since the series is uniformly convergent, for any given  > 0 there exists an N such that |RN | <  for all z
in the domain.
Since the finite sum SN is continuous, for that  there exists a > 0 such that |SN (z) SN ()| <  for all in the
domain satisfying |z | < .
We combine these two results to show that f (z) is continuous.

|f (z) f ()| = |SN (z) + RN (z) SN () RN ()|


|SN (z) SN ()| + |RN (z)| + |RN ()|
< 3 for |z | <

Result 12.2.1 A uniformly convergent series of continuous terms represents a continuous


function.

Example 12.2.3 Again consider sin(nx)


P
n=1 n
. In Example 12.2.2 we showed that the convergence is uniform in any
closed interval that does not contain an integer multiple of 2. In Figure 12.2 is a plot of the first 10 and then 50 terms
in the series and finally the function to which the series converges. We see that the function has jump discontinuities
at x = 2k and is continuous on any closed interval not containing one of those points.

12.3 Uniformly Convergent Power Series


Power Series. Power series are series of the form

X
an (z z0 )n .
n=0

539
P sin(nx)
Figure 12.2: Ten, Fifty and all the Terms of n=1 n
.

P
Domain of Convergence of a Power Series Consider the series n=0 an z n . Let the series converge at some
point z0 . Then |an z0n | is bounded by some constant A for all n, so
n n
z
< A z
n

|an z | = |an z0n |
z0 z0

This comparison test shows that the series converges absolutely for all z satisfying |z| < |z0 |.
Suppose that the series diverges at some point z1 . Then the series could not converge for any |z| > |z1 | since
this would imply convergence at z1 . Thus there exists some circle in the z plane such that the power series converges
absolutely inside the circle and diverges outside the circle.

Result 12.3.1 The domain of convergence of a power series is a circle in the complex plane.

Radius of Convergence of Power Series. Consider a power series



X
f (z) = an z n
n=0

540
Applying the ratio test, we see that the series converges if
|an+1 z n+1 |
lim <l
n |an z n |
|an+1 |
lim |z| < 1
n |an |

|an |
|z| < lim
n |an+1 |

Result 12.3.2 Ratio formula. The radius of convergence of the power series

X
an z n
n=0

is
|an |
R = lim
n |an+1 |

when the limit exists.

Result 12.3.3 Cauchy-Hadamard formula. The radius of convergence of the power series:

X
an z n
n=0

is
1
R= p .
lim sup n
|an |

541
Absolute Convergence of Power Series. Consider a power series

X
f (z) = an z n
n=0

M be the value of the greatest term, an z0n . Consider any point z such that |z| < |z0 |.
that converges for z = z0 . LetP
We can bound the residual of n
n=0 |an z |,


X
RN (z) = |an z n |
n=N

an z n
X
n
= an z n |an z0 |

n=N 0

X z n

M
z0
n=N

Since |z/z0 | < 1, this is a convergent geometric series.


N
z 1
= M
z0 1 |z/z0 |
0 as N

Thus the power series is absolutely convergent for |z| < |z0 |.
P n
Result 12.3.4 If the power series n=0 an z converges for z = z0 , then the series converges
absolutely for |z| < |z0 |.

Example 12.3.1 Find the radii of convergence of the following series.

542

X
1. nz n
n=1


X
2. n!z n
n=1


X
3. n!z n!
n=1

1. We apply the ratio test to determine the radius of convergence.



an
R = lim
= lim n = 1
n an+1 n n + 1

The series converges absolutely for |z| < 1.

2. We apply the ratio test to the series.



n!
R = lim

n (n + 1)!
1
= lim
n n + 1

=0

The series has a vanishing radius of convergence. It converges only for z = 0.

543
3. Again we apply the ration test to determine the radius of convergence.
(n + 1)!z (n+1)!

lim <1
n n!z n!
lim (n + 1)|z|(n+1)!n! < 1
n

lim (n + 1)|z|(n)n! < 1


n
lim (ln(n + 1) + (n)n! ln |z|) < 0
n
ln(n + 1)
ln |z| < lim
n (n)n!
ln |z| < 0
|z| < 1
The series converges absolutely for |z| < 1.
Alternatively we could determine the radius of convergence of the series with the comparison test.

X
n! X
n!z |nz n |
n=1 n=1
P n
n=1 nz has a radius of convergence of 1. Thus the series must have a radius of convergence of at least 1.
Note that if |z| > 1 then the terms in the series do not vanish as n . Thus the series must diverge for all
|z| 1. Again we see that the radius of convergence is 1.

Uniform Convergence of Power Series. Consider a power series n


P
n=0 an z that converges
Pin the disk |z| < r0 .
n n n
The sum converges absolutely for z in the closed disk, |z| r < r0 . Since |an z | |an r | and n=0 |an r | converges,
the power series is uniformly convergent in |z| r < r0 .
P n
Result 12.3.5 If the power series n=0 an z converges for |z| < r0 then the series converges
uniformly for |z| r < r0 .

544
Example 12.3.2 Convergence and Uniform Convergence. Consider the series


X zn
log(1 z) = .
n=1
n

This series converges for |z| 1, z 6= 1. Is the series uniformly convergent in this domain? The residual after N terms
RN is

X zn
RN (z) = .
n=N +1
n

We can get a lower bound on the absolute value of the residual for real, positive z.


X xn
|RN (x)| =
n=N +1
n
Z
x
d
N +1
= Ei((N + 1) ln x)

The exponential integral function, Ei(z), is defined


et
Z
Ei(z) = dt.
z t

The exponential integral function is plotted in Figure 12.3. Since Ei(z) diverges as z 0, by choosing x sufficiently
close to 1 the residual can be made arbitrarily large. Thus this series is not uniformly convergent in the domain
|z| 1, z 6= 1. The series is uniformly convergent for |z| r < 1.

545
-4 -3 -2 -1

-1

-2

-3

Figure 12.3: The Exponential Integral Function.

H
Analyticity. Recall that a sufficient condition for the analyticity of a function f (z) in a domain is that C f (z) dz = 0
for all simple, closed contours in the domain.
Consider a power series Hf (z) = n
P
n=0 an z that is uniformly convergent in |z| r. If C is any simple, closed
contour in the domain then C f (z) dz exists. Expanding f (z) into a finite series and a residual,

I I
f (z) dz = (SN (z) + RN (z)) dz.
C C

Since the series is uniformly convergent, for any given  > 0 there exists an N such that |RN | <  for all z in |z| r.
Let L be the length of the contour C.

I

RN (z) dz L 0 as N

C

546
1
I I N
!
X
f (z) dz = lim an z n + RN (z) dz
C N C n=0

I X
= an z n
C n=0

X I
= an z n dz
n=0 C

=0
Thus f (z) is analytic for |z| < r.

Result 12.3.6 A power series is analytic in its domain of uniform convergence.

12.4 Integration and Differentiation of Power Series


Consider a power series f (z) = n
P
n=0 an z that is convergent in the disk |z| < r0 . Let C be any contour of finite
length L lying entirely within the closed domain |z| r < r0 . The integral of f (z) along C is
Z Z
f (z) dz = (SN (z) + RN (z)) dz.
C C

Since the series is uniformly convergent in the closed disk, for any given  > 0, there exists an N such that
|RN (z)| <  for all |z| r.
We bound the absolute value of the integral of RN (z).
Z Z

RN (z) dz |RN (z)| dz

C C
< L
0 as N

547
Thus
Z N
Z X
f (z) dz = lim an z n dz
C N C n=0
N
X Z
= lim an z n dz
N C
n=0

X Z
= an z n dz
n=0 C

ResultP12.4.1 If C is a contour lying in the domain of uniform convergence of the power


series n
n=0 an z then
Z X
X Z
an z n dz = an z n dz.
C n=0 n=0 C

In the domain of uniform convergence of a series we can interchange the order of summation and a limit process.
That is,
X
X
lim an (z) = lim an (z).
zz0 zz0
n=0 n=0

We can do this because the rate of convergence does not depend on z. Since differentiation is a limit process,

d f (z + h) f (z)
f (z) = lim ,
dz h0 h
we would expect that we could differentiate a uniformly convergent series.
Since we showed that a uniformly convergent power series is equal to an analytic function, we can differentiate a
power series in its domain of uniform convergence.

548
Result 12.4.2 Power series can be differentiated in their domain of uniform convergence.

d X X
an z n = (n + 1)an+1 z n .
dz n=0 n=0

Example 12.4.1 Differentiating a Series. Consider the series from Example 12.3.2.


X zn
log(1 z) =
n=1
n

We differentiate this to obtain the geometric series.


1 X
= z n1
1z n=1

1 X
= zn
1z n=0

The geometric series is convergent for |z| < 1 and uniformly convergent for |z| r < 1. Note that the domain of
convergence is different than the series for log(1 z). The geometric series does not converge for |z| = 1, z 6= 1.
However, the domain of uniform convergence has remained the same.

549
12.5 Taylor Series

Result 12.5.1 Taylors Theorem. Let f (z) be a function that is single-valued and analytic
in |z z0 | < R. For all z in this open disk, f (z) has the convergent Taylor series

X f (n) (z0 )
f (z) = (z z0 )n . (12.1)
n=0
n!

We can also write this as



f (n) (z0 )
I
X
n 1 f (z)
f (z) = an (z z0 ) , an = = dz, (12.2)
n=0
n! 2 C (z z0 )n+1

where C is a simple, positive, closed contour in 0 < |z z0 | < R that goes once around the
point z0 .

Proof of Taylors Theorem. Lets see why Result 12.5.1 is true. Consider a function f (z) that is analytic in
|z| < R. (Considering z0 6= 0 is only trivially more general as we can introduce the change of variables = z z0 .)
According to Cauchys Integral Formula, (Result ??),

I
1 f ()
f (z) = d, (12.3)
2 C z

where C is a positive, simple, closed contour in 0 < | z| < R that goes once around z. We take this contour to be
the circle about the origin of radius r where |z| < r < R. (See Figure 12.4.)

550
Im(z)

r R
Re(z)
C z

Figure 12.4: Graph of Domain of Convergence and Contour of Integration.

1
We expand z
in a geometric series,

1 1/
=
z 1 z/
 n
1X z
= , for |z| < ||
n=0

X zn
= n+1
, for |z| < ||
n=0

We substitute this series into Equation 12.3.


!
f ()z n
I
1 X
f (z) = d
2 C n=0
n+1

551
The series converges uniformly so we can interchange integration and summation.


zn
I
X f ()
= d
n=0
2 C n+1

Now we have derived Equation 12.2. To obtain Equation 12.1, we apply Cauchys Integral Formula.


X f (n) (0)
= zn
n=0
n!

There is a table of some commonly encountered Taylor series in Appendix H.

Example 12.5.1 Consider the Taylor series P expansion of 1/(1 z) about z = 0. Previously, we showed that this
function is the sum of the geometric series n
n=0 z and we used the ratio test to show that the series converged
absolutely for |z| < 1. Now we find the series using Taylors theorem. Since the nearest singularity of the function is
at z = 1, the radius of convergence of the series is 1. The coefficients in the series are

1 dn 1
 
an =
n! dz n 1 z z=0
 
1 n!
=
n! (1 z)n z=0
=1

Thus we have

1 X
= zn, for |z| < 1.
1z n=0

552
12.5.1 Newtons Binomial Formula.

Result 12.5.2 For all |z| < 1, a complex:


     
a a a 2 a 3
(1 + z) = 1 + z+ z + z +
1 2 3
where  
a a(a 1)(a 2) (a r + 1)
= .
r r!
If a is complex, then the expansion is of the principle branch of (1 + z)a . We define
     
r 0 0
= 1, = 0, for r 6= 0, = 1.
0 r 0

Example 12.5.2 Evaluate limn (1 + 1/n)n .


First we expand (1 + 1/n)n using Newtons binomial formula.
 n        
1 n 1 n 1 n 1
lim 1 + = lim 1 + + + +
n n n 1 n 2 n2 3 n3
 
n(n 1) n(n 1)(n 2)
= lim 1 + 1 + + +
n 2!n2 3!n3
 
1 1
= 1 + 1 + + +
2! 3!

We recognize this as the Taylor series expansion of e1 .

=e

553
We can also evaluate the limit using LHospitals rule.
  x   x 
1 1
ln lim 1 + = lim ln 1+
x x x x
 
1
= lim x ln 1 +
x x
ln(1 + 1/x)
= lim
x 1/x
1/x2
1+1/x
= lim
x 1/x2

=1
 x
1
lim 1+ =e
x x

Example 12.5.3 Find the Taylor series expansion of 1/(1 + z) about z = 0.


For |z| < 1,
     
1 1 1 2 1 3
=1+ z+ z + z +
1+z 1 2 3
= 1 + (1)1 z + (1)2 z 2 + (1)3 z 3 +
= 1 z + z2 z3 +

Example 12.5.4 Find the first few terms in the Taylor series expansion of

1

z2 + 5z + 6

about the origin.

554
We factor the denominator and then apply Newtons binomial formula.

1 1 1
=
z 2 + 5z + 6 z+3 z+2
1 1
= p p
3 1 + z/3 2 1 + z/2
          
1 1/2 z 1/2  z 2 1/2 z 1/2  z 2
= 1+ + + 1+ + +
6 1 3 2 3 1 2 2 2
z z2 z 3z 2
  
1
= 1 + + 1 + +
6 6 24 4 32
 
1 5 17 2
= 1 z + z +
6 12 96

12.6 Laurent Series

Result 12.6.1 Let f (z) be single-valued and analytic in the annulus R1 < |z z0 | < R2 .
For points in the annulus, the function has the convergent Laurent series

X
f (z) = an z n ,
n=

where I
1 f (z)
an = dz
2 C (z z0 )n+1
and C is a positively oriented, closed contour around z0 lying in the annulus.
To derive this result, consider a function f () that is analytic in the annulus R1 < || < R2 . Consider any point z

555
in the annulus. Let C1 be a circle of radius r1 with R1 < r1 < |z|. Let C2 be a circle of radius r2 with |z| < r2 < R2 .
Let Cz be a circle around z, lying entirely between C1 and C2 . (See Figure 12.5 for an illustration.)
f () f ()
Consider the integral of z
around the C2 contour. Since the the only singularities of z
occur at = z and at
points outside the annulus,
I I I
f () f () f ()
d = d + d.
C2 z Cz z C1 z

By Cauchys Integral Formula, the integral around Cz is

I
f ()
d = 2f (z).
Cz z

This gives us an expression for f (z).

I I
1 f () 1 f ()
f (z) = d d (12.4)
2 C2 z 2 C1 z

On the C2 contour, |z| < ||. Thus

1 1/
=
z 1 z/
 n
1X z
= , for |z| < ||
n=0

X zn
= n+1
, for |z| < ||
n=0

556
On the C1 contour, || < |z|. Thus
1 1/z
=
z 1 /z
 n
1X
= , for || < |z|
z n=0 z

X n
= , for || < |z|
n=0
z n+1
1
X zn
= , for || < |z|
n=
n+1

We substitute these geometric series into Equation 12.4.



! 1
!
f ()z n f ()z n
I I
1 X 1 X
f (z) = d + d
2 C2 n=0 n+1 2 C1 n=
n+1

Since the sums converge uniformly, we can interchange the order of integration and summation.
I 1 I
1 X f ()z n 1 X f ()z n
f (z) = d + d
2 n=0 C2 n+1 2 n= C1 n+1

Since the only singularities of the integrands lie outside of the annulus, the C1 and C2 contours can be deformed to
any positive, closed contour C that lies in the annulus and encloses the origin. (See Figure 12.5.) Finally, we combine
the two integrals to obtain the desired result.
I 
X 1 f ()
f (z) = n+1
d z n
n=
2 C

For the case of arbitrary z0 , simply make the transformation z z z0 .

557
Im(z) Im(z)

r2 R2 R2

r1 R1 R1
Re(z) Re(z)
C1
z C
C2 Cz

Figure 12.5: Contours for a Laurent Expansion in an Annulus.

Example 12.6.1 Find the Laurent series expansions of 1/(1 + z).

For |z| < 1,

     
1 1 1 2 1 3
=1+ z+ z + z +
1+z 1 2 3
= 1 + (1)1 z + (1)2 z 2 + (1)3 z 3 +
= 1 z + z2 z3 +

558
For |z| > 1,

1 1/z
=
1+z 1 + 1/z
     
1 1 1 1 2
= 1+ z + z +
z 1 2
= z 1 z 2 + z 3

559
12.7 Exercises
12.7.1 Series of Constants
Exercise 12.1 P
Show that if an converges then limn an = 0. That is, limn an = 0 is a necessary condition for the convergence
of the series.
Hint, Solution
Exercise 12.2
Answer the following questions true or false. Justify your answers.

1. There exists a sequence which converges to both 1 and 1.

2. There exists a sequence {an } such that an > 1 for all n and limn an = 1.

3. There exists a divergent geometric series whose terms converge.

4. There exists a sequence whose even terms are greater than 1, whose odd terms are less than 1 and that converges
to 1.

5. There exists a divergent series of non-negative terms, n


P
n=0 an , such that an < (1/2) .

6. There exists a convergent sequence, {an }, such that limn (an+1 an ) 6= 0.

7. There exists a divergent sequence, {an }, such that limn |an | = 2.


P P P
8. There exists divergent series, an and bn , such that (an + bn ) is convergent.

9. There exists 2 different series of nonzero terms that have the same sum.

10. There exists a series of nonzero terms that converges to zero.

11. There exists a series with an infinite number of non-real terms which converges to a real number.

560
P
12. There exists a convergent series an with limn |an+1 /an | = 1.
P
13. There exists a divergent series an with limn |an+1 /an | = 1.
P p
14. There exists a convergent series an with limn n |an | = 1.
P p
15. There exists a divergent series an with limn n |an | = 1.
P P 2
16. There exists a convergent series of non-negative terms, an , for which an diverges.
P P
17. There exists a convergent series of non-negative terms, an , for which an diverges.
P P
18. There exists a convergent series, an , for which |an | diverges.

an (z z0 )n which converges for z = 0 and z = 3 but diverges for z = 2.


P
19. There exists a power series

an (z z0 )n which converges for z = 0 and z = 2 but diverges for z = 2.


P
20. There exists a power series

Hint, Solution
Exercise 12.3
Determine if the following series converge.

X 1
1.
n=2
n ln(n)


X 1
2.
n=2
ln (nn )


X
n
3. ln ln n
n=2

561

X 1
4.
n=10
n(ln n)(ln(ln n))

X ln (2n )
5.
n=1
ln (3n ) + 1

X 1
6.
n=0
ln(n + 20)

X 4n + 1
7.
n=0
3n 2

X
8. (Log 2)n
n=0


X n2 1
9.
n=2
n4 1

X n2
10.
n=2
(ln n)n
 
X
n1
11. (1) ln
n=2
n

X (n!)2
12.
n=2
(2n)!

X 3n + 4n + 5
13.
n=2
5n 4n 3

562

X n!
14.
n=2
(ln n)n

X en
15.
n=2
ln(n!)

X (n!)2
16.
n=1
(n2 )!

X n8 + 4n4 + 8
17.
n=1
3n9 n5 + 9n
 
X 1 1
18.
n=1
n n+1

X cos(n)
19.
n=1
n

X ln n
20.
n=2
n11/10
Hint, Solution
Exercise 12.4 (mathematica/fcv/series/constants.nb)
Show that the alternating harmonic series,

X (1)n+1 1 1 1
=1 + + ,
n=1
n 2 3 4
is convergent.
Hint, Solution

563
Exercise 12.5 (mathematica/fcv/series/constants.nb)
Show that the series

X 1
n=1
n
is divergent with the Cauchy convergence criterion.
Hint, Solution
Exercise 12.6
The alternating harmonic series has the sum:

X (1)n
= ln(2).
n=1
n

Show that the terms in this series can be rearranged to sum to .


Hint, Solution
Exercise 12.7 (mathematica/fcv/series/constants.nb)
Is the series,

X n!
,
n=1
nn
convergent?
Hint, Solution
Exercise 12.8
Show that the harmonic series,

X 1 1 1

= 1 +
+
+ ,
n=1
n 2 3
converges for > 1 and diverges for 1.
Hint, Solution

564
Exercise P12.9
1
Evaluate N n=1 sin(nx).
Hint, Solution
Exercise 12.10
Evaluate
n
X n
X
k
kz and k2z k
k=1 k=1

for z 6= 1.
Hint, Solution
Exercise 12.11
Which of the following series converge? Find the sum of those that do.

1 1 1 1
1. + + + +
2 6 12 20
2. 1 + (1) + 1 + (1) +

X 1 1 1
3. n1
n=1
2 3n 5n+1

Hint, Solution
Exercise 12.12
Evaluate the following sum.
X

X X 1

k1 =0 k2 =k1 kn =kn1
2kn

Hint, Solution

565
12.7.2 Uniform Convergence
12.7.3 Uniformly Convergent Power Series
Exercise 12.13
Determine the domain of convergence of the following series.

X zn
1.
n=0
(z + 3)n

X Log z
2.
n=2
ln n

X z
3.
n=1
n

X (z + 2)2
4.
n=1
n2

X (z e)n
5.
n=1
nn

X z 2n
6.
n=1
2nz

X z n!
7.
n=0
(n!)2

X z ln(n!)
8.
n=0
n!

566

X (z )2n+1 n
9.
n=0
n!

X ln n
10.
n=0
zn

Hint, Solution
Exercise 12.14
Find the circle of convergence of the following series.

z2 z3 z4
1. z + ( ) + ( )( 2) + ( )( 2)( 3) +
2! 3! 4!

X n
2. n
(z )n
n=1
2

X
3. nn z n
n=1


X n! n
4. z
n=1
nn

X
5. (3 + (1)n )n z n
n=1


X
6. (n + n ) z n (|| > 1)
n=1

Hint, Solution

567
Exercise 12.15
Find the circle of convergence of the following series:

X
1. kz k
k=0

X
2. kk z k
k=1

X k! k
3. k
z
k=1
k

X
4. (z + 5)2k (k + 1)2
k=0

X
5. (k + 2k )z k
k=0

Hint, Solution

12.7.4 Integration and Differentiation of Power Series


Exercise 12.16
Using the geometric series, show that

1 X
= (n + 1)z n , for |z| < 1,
(1 z)2 n=0

and
X zn
log(1 z) = , for |z| < 1.
n=1
n

568
Hint, Solution

12.7.5 Taylor Series


Exercise 12.17
1
Find the Taylor series of 1+z 2 about the z = 0. Determine the radius of convergence of the Taylor series from the

singularities of the function. Determine the radius of convergence with the ratio test.
Hint, Solution
Exercise 12.18
Use two methods to find the Taylor series expansion of log(1 + z) about z = 0 and determine the circle of convergence.
First directly apply Taylors theorem, then differentiate a geometric series.
Hint, Solution
Exercise 12.19
Let f (z) = (1 + z) be the branch for which f (0) = 1. Find its Taylor series expansion about z = 0. What is the
radius of convergence of the series? ( is an arbitrary complex number.)
Hint, Solution
Exercise 12.20
Find the Taylor series expansions about the point z = 1 for the following functions. What are the radii of convergence?

1
1.
z
2. Log z
1
3.
z2
4. z Log z z

Hint, Solution

569
Exercise 12.21
Find the Taylor series expansion about the point z = 0 for ez . What is the radius of convergence? Use this to find the
Taylor series expansions of cos z and sin z about z = 0.
Hint, Solution
Exercise 12.22
Find the Taylor series expansion about the point z = for the cosine and sine.
Hint, Solution
Exercise 12.23
Sum the following series.

X (ln 2)n
1.
n=0
n!

X (n + 1)(n + 2)
2.
n=0
2n

X (1)n
3.
n=0
n!

X (1)n 2n+1
4.
n=0
(2n + 1)!

X (1)n 2n
5.
n=0
(2n)!

X ()n
6.
n=0
(2n)!

Hint, Solution

570
Exercise 12.24
1. Find the first three terms in the following Taylor series and state the convergence properties for the following.

(a) ez around z0 = 0
1+z
(b) around z0 =
1z
ez
(c) around z0 = 0
z1

It may be convenient to use the Cauchy product of two Taylor series.

2. Consider a function f (z) analytic for |z z0 | < R. Show that the series obtained by differentiating the Taylor
series for f (z) termwise is actually the Taylor series for f 0 (z) and hence argue that this series converges uniformly
to f 0 (z) for |z z0 | < R.

3. Find the Taylor series for


1
(1 z)3
by appropriate differentiation of the geometric series and state the radius of convergence.

4. Consider the branch of f (z) = (z + 1) corresponding to f (0) = 1. Find the Taylor series expansion about z0 = 0
and state the radius of convergence.
Hint, Solution

12.7.6 Laurent Series


Exercise 12.25
Find the Laurent series about z = 0 of 1/(z ) for |z| < 1 and |z| > 1.
Hint, Solution

571
Exercise 12.26
Obtain the Laurent expansion of
1
f (z) =
(z + 1)(z + 2)
centered on z = 0 for the three regions:

1. |z| < 1

2. 1 < |z| < 2

3. 2 < |z|

Hint, Solution
Exercise 12.27
By comparing the Laurent expansion of (z + 1/z)m , m Z+ , with the binomial expansion of this quantity, show that
(
m
2

m n m and m n even
Z
m 2m1 (mn)/2
(cos ) cos(n) d =
0 0 otherwise

Hint, Solution
Exercise 12.28
The function f (z) is analytic in the entire z-plane, including , except at the point z = /2, where it has a simple
pole, and at z = 2, where it has a pole of order 2. In addition
I I I
f (z) dz = 2, f (z) dz = 0, (z 1)f (z) dz = 0.
|z|=1 |z|=3 |z|=3

Find f (z) and its complete Laurent expansion about z = 0.


Hint, Solution

572
Exercise 12.29
3 z k
Let f (z) =
P 
k=1 k 3
. Compute each of the following, giving justification in each case. The contours are circles of
radius one about the origin.
Z
1. ez f (z) dz
|z|=1
Z
f (z)
2. dz
|z|=1 z4

f (z) ez
Z
3. dz
|z|=1 z2
Hint, Solution
Exercise 12.30
1
1. Expand f (z) = z(1z)
in Laurent series that converge in the following domains:

(a) 0 < |z| < 1


(b) |z| > 1
(c) |z + 1| > 2

2. Without determining the series, specify the region of convergence for a Laurent series representing f (z) =
1/(z 4 + 4) in powers of z 1 that converges at z = .
Hint, Solution

573
12.8 Hints
Hint 12.1
Use the Cauchy convergence criterion for series. In particular, consider |SN +1 SN |.

Hint 12.2
CONTINUE

Hint 12.3
1.

X 1
n=2
n ln(n)

Use the integral test.

2.

X 1
n=2
ln (nn )

Simplify the summand.

3.

X
n
ln ln n
n=2

Simplify the summand. Use the comparison test.

4.

X 1
n=10
n(ln n)(ln(ln n))

Use the integral test.

574
5.
X ln (2n )
n=1
ln (3n ) + 1
Show that the terms in the sum do not vanish as n
6.
X 1
n=0
ln(n + 20)
Shift the indices.
7.
X 4n + 1
n=0
3n 2
Show that the terms in the sum do not vanish as n
8.
X
(Log 2)n
n=0
This is a geometric series.
9.
X n2 1
n=2
n4 1
Simplify the integrand. Use the comparison test.
10.
X n2
n=2
(ln n)n
Compare to a geometric series.

575
11.  
X 1
n
(1) ln
n=2
n
Group pairs of consecutive terms to obtain a series of positive terms.
12.
X (n!)2
n=2
(2n)!
Use the comparison test.
13.
X 3n + 4n + 5
n=2
5n 4n 3
Use the root test.
14.
X n!
n=2
(ln n)n
Show that the terms do not vanish as n .
15.
X en
n=2
ln(n!)
Show that the terms do not vanish as n .
16.
X (n!)2
n=1
(n2 )!
Apply the ratio test.

576
17.
X n8 + 4n4 + 8
n=1
3n9 n5 + 9n
Use the comparison test.
18.  
X 1 1

n=1
n n+1
Use the comparison test.
19.
X cos(n)
n=1
n
Simplify the integrand.
20.
X ln n
n=2
n11/10
Use the integral test.
Hint 12.4
Group the terms.
1 1
1 =
2 2
1 1 1
=
3 4 12
1 1 1
=
5 6 30

577
Hint 12.5
Show that
1
|S2n Sn | > .
2
Hint 12.6
The alternating harmonic series is conditionally convergent. Let {an } and
P {bn } be thePpositive and negative terms in
the sum, respectively, ordered in decreasing magnitude. Note that both a
n=1 n and
b
n=1 n are divergent. Devise a
method for alternately taking terms from {an } and {bn }.

Hint 12.7
Use the ratio test.

Hint 12.8
Use the integral test.

Hint 12.9
Note that sin(nx) = =(enx ). This substitute will yield a finite geometric series.

Hint 12.10
Let Sn be the sum. Consider Sn zSn . Use the finite geometric sum.

Hint 12.11
1. The summand is a rational function. Find the first few partial sums.

2.

3. This a geometric series.

Hint 12.12
CONTINUE

578
Hint 12.13
CONTINUE

X zn
1.
n=0
(z + 3)n

X Log z
2.
n=2
ln n

X z
3.
n=1
n

X (z + 2)2
4.
n=1
n2

X (z e)n
5.
n=1
nn

X z 2n
6.
n=1
2nz

X z n!
7.
n=0
(n!)2

X z ln(n!)
8.
n=0
n!

X (z )2n+1 n
9.
n=0
n!

579

X ln n
10.
n=0
zn

Hint 12.14

Hint 12.15
CONTINUE

Hint 12.16
Differentiate the geometric series. Integrate the geometric series.

Hint 12.17
The Taylor series is a geometric series.

Hint 12.18

Hint 12.19

Hint 12.20
1.
1 1
=
z 1 + (z 1)
The right side is the sum of a geometric series.

2. Integrate the series for 1/z.

3. Differentiate the series for 1/z.

4. Integrate the series for Log z.

580
Hint 12.21
Evaluate the derivatives of ez at z = 0. Use Taylors Theorem.
Write the cosine and sine in terms of the exponential function.
Hint 12.22

cos z = cos(z )
sin z = sin(z )
Hint 12.23
CONTINUE

Hint 12.24
CONTINUE

Hint 12.25

Hint 12.26

Hint 12.27

Hint 12.28

Hint 12.29

Hint 12.30
CONTINUE

581
12.9 Solutions
Solution
P 12.1
n=0 an converges only if the partial sums, Sn , are a Cauchy sequence.

 > 0 N s.t. m, n > N |Sm Sn | < ,


In particular, we can consider m = n + 1.
 > 0 N s.t. n > N |Sn+1 Sn | < 
Now we note that Sn+1 sn = an .
 > 0 N s.t. n > N |an | < 

P for the sequence {an }. Thus we see that limn an = 0 is a necessary


This is exactly the Cauchy convergence criterion
condition for the convergence of the series n=0 an .

Solution 12.2
CONTINUE

Solution 12.3
1.

X 1
n=2
n ln(n)
Since this is a series of positive, monotone decreasing terms, the sum converges or diverges with the integral,
Z Z
1 1
dx = d
2 x ln x ln 2

Since the integral diverges, the series also diverges.


2.
X 1 X 1
=
n=2
ln (nn ) n=2 n ln(n)

582
The sum converges.
3.
X
n
X 1 X 1
ln ln n = ln(ln n)
n=2 n=2
n n=2
n
The sum is divergent by the comparison test.
4.
X 1
n=10
n(ln n)(ln(ln n))
Since this is a series of positive, monotone decreasing terms, the sum converges or diverges with the integral,
Z Z Z
1 1 1
dx = dy = dz
10 x ln x ln(ln x) ln(10) y ln y ln(ln(10)) z

Since the integral diverges, the series also diverges.


5.
X ln (2n ) X n ln 2 X ln 2
= =
n=1
ln (3n ) + 1 n=1 n ln 3 + 1 n=1 ln 3 + 1/n
Since the terms in the sum do not vanish as n , the series is divergent.
6.
X 1 X 1
=
n=0
ln(n + 20) n=20 ln n
The series diverges.
7.
X 4n + 1
n=0
3n 2
Since the terms in the sum do not vanish as n , the series is divergent.

583
8.

X
(Log 2)n
n=0

This is a geometric series. Since | Log 2| < 1, the series converges.

9.

X n2 1 X 1 X 1
= <
n=2
n4 1 n=2
n2 + 1 n=2 n2

The series converges by comparison to the harmonic series.

10.
n
n2 n2/n
X X 
=
n=2
(ln n)n n=2
ln n

Since n2/n 1 as n , n2/n / ln n 0 as n . The series converges by comparison to a geometric


series.

11. We group pairs of consecutive terms to obtain a series of positive terms.


  X      X  
X
n 1 1 1 2n + 1
(1) ln = ln ln = ln
n=2
n n=1
2n 2n + 1 n=1
2n

The series on the right side diverges because the terms do not vanish as n .

12.

X (n!)2 X (1)(2) n X 1
= <
n=2
(2n)! n=2 (n + 1)(n + 2) (2n) n=2 2n

The series converges by comparison with a geometric series.

584
13.

X 3n + 4n + 5
n=2
5n 4n 3

We use the root test to check for convergence.

3 + 4n + 5 1/n
n
1/n
lim |an | = lim n
n n 5 4n 3
1/n
4 (3/4)n + 1 + 5/4n

= lim
n 5 1 (4/5)n 3/5n
4
=
5
<1

We see that the series is absolutely convergent.

14. We will use the comparison test.


p !n
X n! X (n/2)n/2 X n/2
> =
n=2
(ln n)n n=2
(ln n)n n=2
ln n

Since the terms in the series on the right side do not vanish as n , the series is divergent.

15. We will use the comparison test.



X en X en X en
> =
n=2
ln(n!) n=2 ln(nn ) n=2 n ln(n)

Since the terms in the series on the right side do not vanish as n , the series is divergent.

585
16.

X (n!)2
n=1
(n2 )!
We apply the ratio test.
((n + 1)!)2 (n2 )!

an+1
lim = lim
n an n ((n + 1)2 )!(n!)2

(n + 1)2


= lim

n ((n + 1)2 n2 )!
(n + 1)2

= lim

n (2n + 1)!

=0

The series is convergent.

17.

X n8 + 4n4 + 8 X 1 1 + 4n4 + 8n8
=
n=1
3n9 n5 + 9n n=1 n 3 n4 + 9n8

1X1
>
4 n=1 n

We see that the series is divergent by comparison to the harmonic series.

18.
 
X 1 1 X 1 X 1
= <
n=1
n n+1 n=1
n2 + n n=1 n2
The series converges by the comparison test.

586
19.
X cos(n) X (1)n
=
n=1
n n=1
n
We recognize this as the alternating harmonic series, which is conditionally convergent.
20.
X ln n
n=2
n11/10
Since this is a series of positive, monotone decreasing terms, the sum converges or diverges with the integral,
Z Z
ln x
11/10
dx = y ey/10 dy
2 x ln 2

Since the integral is convergent, so is the series.


Solution 12.4


(1)n+1

X X 1 1
=
n=1
n n=1
2n 1 2n

X 1
=
n=1
(2n 1)(2n)

X 1
<
n=1
(2n 1)2

1 X 1
<
2 n=1
n2
2

=
12
Thus the series is convergent.

587
Solution 12.5
Since

2n1
X 1
|S2n Sn | =

j


j=n
2n1
X 1

j=n
2n 1
n
=
2n 1
1
>
2
the series does not satisfy the Cauchy convergence criterion.

Solution 12.6
The alternating harmonic series is conditionally convergent. That is, the sum is convergent but not absolutely conver-
gent. Let {an } and
P {bn } be thePpositive and negative terms in the sum, respectively, ordered in decreasing magnitude.
Note that both a
n=1 n and
b
n=1 n are divergent. Otherwise the alternating harmonic series would be absolutely
convergent.
To sum the terms in the series to we repeat the following two steps indefinitely:

1. Take terms from {an } until the sum is greater than .

2. Take terms from {bn } until the sum is less than .

Each of these steps can always be accomplished because the sums,


P P
n=1 an and n=1 bn are both divergent. Hence the
tails of the series are divergent. No matter how many terms we take, the remaining terms in each series are divergent.
In each step a finite, nonzero number of terms from the respective series is taken. Thus all the terms will be used.
Since the terms in each series vanish as n , the running sum converges to .

588
Solution 12.7
Applying the ratio test,

(n + 1)!nn

an+1
lim = lim
n an n n!(n + 1)(n+1)
nn
= lim
n (n + 1)n
 n
n
= lim
n (n + 1)
1
=
e
< 1,

we see that the series is absolutely convergent.

Solution 12.8
The harmonic series,

X 1 1 1

= 1 + + + ,
n=1
n 2 3

converges or diverges absolutely with the integral,

x]
(
Z
1
Z
1 [ln 1 i for <() = 1,
dx = dx = h
x1<()

1 |x | 1 x<() 1<()
for <() 6= 1.
1

The integral converges only for <() > 1. Thus the harmonic series converges absolutely for <() > 1 and diverges
absolutely for <() 1.

589
Solution 12.9

N
X 1 N
X 1
sin(nx) = sin(nx)
n=1 n=0
N
X 1
= =(enx )
n=0
1
N
!
X
== (ex )n
( n=0
=(N ) for x = 2k
= 1enx

= 1ex for x 6= 2k
(
0  for x = 2k
= ex/2 e(N 1/2)x
= ex/2 ex/2
for x 6= 2k
(
0  for x = 2k
= x/2 (N 1/2)x
= e 2sin(x/2)
e
for x 6= 2k
(
0  for x = 2k
= ex/2 e(N 1/2)x
< 2 sin(x/2)
for x 6= 2k

1
N
(
X 0 for x = 2k
sin(nx) = cos(x/2)cos((N 1/2)x)
n=1 2 sin(x/2)
6 2k
for x =

590
Solution 12.10
Let
n
X
Sn = kz k .
k=1

n
X n
X
k
Sn zSn = kz kz k+1
k=1 k=1
n
X n+1
X
= kz k (k 1)z k
k=1 k=2
n
X
= z k nz n+1
k=1
z z n+1
= nz n+1
1z
n
X z(1 (n + 1)z n + nz n+1 )
kz k =
k=1
(1 z)2
Let n
X
Sn = k2z k .
k=1

n
X
Sn zSn = (k 2 (k 1)2 )z k n2 z n+1
k=1
Xn n
X
k
=2 kz z k n2 z n+1
k=1 k=1
z(1 (n + 1)z n + nz n+1 ) z z n+1
=2 n2 z n+1
(1 z)2 1z

591
n
X z(1 + z z n (1 + z + n(n(z 1) 2)(z 1)))
k2z k =
k=1
(1 z)3

Solution 12.11
1.

X 1 1 1 1
an = + + + +
n=1
2 6 12 20
We conjecture that the terms in the sum are rational functions of summation index. That is, an = 1/p(n) where
p(n) is a polynomial. We use divided differences to determine the order of the polynomial.
2 6 12 20
4 6 8
2 2
We see that the polynomial is second order. p(n) = an2 + bn + c. We solve for the coefficients.
a+b+c=2
4a + 2b + c = 6
9a + 3b + c = 12
p(n) = n2 + n
We examine the first few partial sums.
1
S1 =
2
2
S2 =
3
3
S3 =
4
4
S4 =
5

592
We conjecture that Sn = n/(n+1). We prove this with induction. The base case is n = 1. S1 = 1/(1+1) = 1/2.
Now we assume the induction hypothesis and calculate Sn+1 .

Sn+1 = Sn + an+1
n 1
= + 2
n + 1 (n + 1) + (n + 1)
n+1
=
n+2

This proves the induction hypothesis. We calculate the limit of the partial sums to evaluate the series.

X 1 n
= lim
n=1
n2 +n n n+1

X 1
=1
n=1
n2 +n

2.

X
(1)n = 1 + (1) + 1 + (1) +
n=0

Since the terms in the series do not vanish as n , the series is divergent.

3. We can directly sum this geometric series.


X 1 1 1 1 1 2
= =
n=1
2n1 3n 5n+1 75 1 1/30 145

CONTINUE

593
Solution 12.12
The innermost sum is a geometric series.

X 1 1 1
= = 21kn1
kn =kn1
2kn 2kn1 1 1/2

This gives us a relationship between n nested sums and n 1 nested sums.



X X
X X 1 X X 1
=2
k1 =0 k2 =k1 kn =kn1
2kn k =0 k =k k =k
2kn1
1 2 1 n1 n2

We evaluate the n nested sums by induction.



X
X X 1
= 2n
k1 =0 k2 =k1 kn =kn1
2kn

Solution 12.13
CONTINUE.

X zn
1.
n=0
(z + 3)n

X Log z
2.
n=2
ln n

X z
3.
n=1
n

X (z + 2)2
4.
n=1
n2

594

X (z e)n
5.
n=1
nn

X z 2n
6.
n=1
2nz

X z n!
7.
n=0
(n!)2

X z ln(n!)
8.
n=0
n!

X (z )2n+1 n
9.
n=0
n!

X ln n
10.
n=0
zn

Solution 12.14
1. We assume that 6= 0. We determine the radius of convergence with the ratio test.

an
R = lim
n an+1

( ) ( (n 1))/n!
= lim
n ( ) ( n)/(n + 1)!

n+1
= lim
n n
1
=
||

595
The series converges absolutely for |z| < 1/||.

2. By the ratio test formula, the radius of absolute convergence is


n

n/2
R = lim
n (n + 1)/2n+1

n
= 2 lim
n n + 1

=2

By the root test formula, the radius of absolute convergence is


1
R= p
limn |n/2n |
n

2
=
limn n n
=2

The series converges absolutely for |z | < 2.

3. We determine the radius of convergence with the Cauchy-Hadamard formula.


1
R= p
lim sup n |an |
1
= p
lim sup n |nn |
1
=
lim sup n
=0

The series converges only for z = 0.

596
4. By the ratio test formula, the radius of absolute convergence is
n

n!/n
R = lim
n (n + 1)!/(n + 1)n+1
(n + 1)n

= lim
n nn
 n
n+1
= lim
n n
  n 
n+1
= exp lim ln
n n
  
n+1
= exp lim n ln
n n
 
ln(n + 1) ln(n)
= exp lim
n 1/n
 
1/(n + 1) 1/n
= exp lim
n 1/n2
 
n
= exp lim
n n + 1
1
=e
The series converges absolutely in the circle, |z| < e.
5. By the Cauchy-Hadamard formula, the radius of absolute convergence is
1
R=
lim sup | (3 + (1)n )n |
pn

1
=
lim sup (3 + (1)n )
1
=
4

597
Thus the series converges absolutely for |z| < 1/4.

6. By the Cauchy-Hadamard formula, the radius of absolute convergence is


1
R= p
lim sup n
|n + n |
1
= p
lim sup || n |1 + n/n |
1
=
||

Thus the sum converges absolutely for |z| < 1/||.

Solution 12.15
1.

X
kz k
k=0

We determine the radius of convergence with the ratio formula.



k
R = lim
k k + 1
1
= lim
k 1
=1

The series converges absolutely for |z| < 1.

2.
X
kk z k
k=1

598
We determine the radius of convergence with the Cauchy-Hadamard formula.
1
R= p
lim sup k |k k |
1
=
lim sup k
=0
The series converges only for z = 0.
3.
X k! k
k
z
k=1
k
We determine the radius of convergence with the ratio formula.
k!/k k


R = lim
k (k + 1)!/(k + 1)(k+1)

(k + 1)k
= lim k
k
 k  
k+1
= exp lim k ln
k k
 
ln(k + 1) ln(k)
= exp lim
k 1/k
 
1/(k + 1) 1/k
= exp lim
k 1/k 2
 
k
= exp lim
k k + 1

= exp(1)
=e

599
The series converges absolutely for |z| < e.
4.
X
(z + 5)2k (k + 1)2
k=0
We use the ratio formula to determine the domain of convergence.
(z + 5)2(k+1) (k + 2)2

lim <1
k (z + 5)2k (k + 1)2
(k + 2)2

2
|z + 5| lim <1
k (k + 1)2

2(k + 2)
|z + 5|2 lim <1
k 2(k + 1)
2
|z + 5|2 lim < 1
k 2
|z + 5|2 < 1

5.
X
(k + 2k )z k
k=0
We determine the radius of convergence with the Cauchy-Hadamard formula.
1
R= p
lim sup |k + 2k |
k

1
= p
lim sup 2 |1 + k/2k |
k

1
=
2
The series converges for |z| < 1/2.

600
Solution 12.16
The geometric series is

1 X
= zn.
1z n=0

This series is uniformly convergent in the domain, |z| r < 1. Differentiating this equation yields,


1 X
= nz n1
(1 z)2 n=1

X
= (n + 1)z n for |z| < 1.
n=0

Integrating the geometric series yields


X z n+1
log(1 z) =
n=0
n+1

X zn
log(1 z) = , for |z| < 1.
n=1
n

Solution 12.17


1 X
2 n
X
(1)n z 2n

= z =
1 + z2 n=0 n=0

1 1
The function 1+z 2
= (1z)(1+z)
has singularities at z = . Thus the radius of convergence is 1. Now we use the ratio

601
test to corroborate that the radius of convergence is 1.

an+1 (z)
lim <1
n an (z)

(1)n+1 z 2(n+1)

lim <1
n (1)n z 2n

lim z 2 < 1
n
|z| < 1
Solution 12.18

Method 1.
 2
z2
  
d z d
log(1 + z) = [log(1 + z)]z=0 + log(1 + z) + log(1 + z) +
dz z=0 1! dz 2 z=0 2!
z2 z3
     
1 z 1 2
=0+ + + +
1 + z z=0 1! (1 + z)2 z=0 2! (1 + z)3 z=0 3!
z2 z3 z4
=z + +
2 3 4
n
n+1 z
X
= (1)
n=1
n
Since the nearest singularity of log(1 + z) is at z = 1, the radius of convergence is 1.
Method 2. We know the geometric series converges for |z| < 1.

1 X
= (1)n z n
1+z n=0

We integrate this equation to get the series for log(1 + z) in the domain |z| < 1.
n+1
X
n z
X zn
log(1 + z) = (1) = (1)n+1
n=0
n + 1 n=1 n

602
We calculate the radius of convergence with the ratio test.

an (n + 1)
R = lim = lim =1
n an+1 n n

Thus the series converges absolutely for |z| < 1.

Solution 12.19
The Taylor series expansion of f (z) about z = 0 is

X f (n) (0)
f (z) = zn.
n=0
n!

The derivatives of f (z) are !


n1
Y
f (n) (z) = ( k) (1 + z)n .
k=0

Thus f (n) (0) is


n1
Y
(n)
f (0) = ( k).
k=0

If = m is a non-negative integer, then only the first m + 1 terms are nonzero. The Taylor series is a polynomial and
the series has an infinite radius of convergence.
m Qn1
k=0 ( k) n
X
m
(1 + z) = z
n=0
n!

If is not a non-negative integer, then all of the terms in the series are non-zero.
Qn1
k=0 ( k) n
X
(1 + z) = z
n=0
n!

603
The radius of convergence of the series is the distance to the nearest singularity of (1 + z) . This occurs at z = 1.
Thus the series converges for |z| < 1. We can corroborate this with the ratio test. The radius of convergence is
Qn1 



Q k=0 ( k) /n! n+1
R = lim n = lim = 1.

n (
k=0 ( k)) /(n + 1)! n n

If we use the binomial coefficient, we can write the series in a compact form.
  Qn1
( k)
k=0
n n!

X  

(1 + z) = zn
n=0
n

Solution 12.20
1. We find the series for 1/z by writing it in terms of z 1 and using the geometric series.
1 1
=
z 1 + (z 1)

1 X
= (1)n (z 1)n for |z 1| < 1
z n=0

Since the nearest singularity is at z = 0, the radius of convergence is 1. The series converges absolutely for
|z 1| < 1. We could also determine the radius of convergence with the Cauchy-Hadamard formula.
1
R= p
lim sup n
|an |
1
= p
lim sup n
|(1)n |
=1

604
2. We integrate 1/ from 1 to z for in the circle |z 1| < 1.
Z z
1
d = [Log ]z1 = Log z
1

The series we derived for 1/z is uniformly convergent for |z 1| r < 1. We can integrate the series in this
domain.
Z zX
Log z = (1)n ( 1)n d
1 n=0

X Z z
n
= (1) ( 1)n d
n=0 1

X (z 1)n+1
= (1)n
n=0
n+1


X (1)n1 (z 1)n
Log z = for |z 1| < 1
n=1
n

3. The series we derived for 1/z is uniformly convergent for |z 1| r < 1. We can differentiate the series in this
domain.
1 d 1
2
=
z dz z

d X
= (1)n (z 1)n
dz n=0

X
= (1)n+1 n(z 1)n1
n=1

605

1 X
= (1)n (n + 1)(z 1)n for |z 1| < 1
z2 n=0

4. We integrate Log from 1 to z for in the circle |z 1| < 1.


Z z
Log d = [ Log ]z1 = z Log z z + 1
1
The series we derived for Log z is uniformly convergent for |z 1| r < 1. We can integrate the series in this
domain.
Z z
z Log z z = = 1 + Log d
1

zX
(1)n1 ( 1)n
Z
= 1 + d
1 n=1
n

X (1)n1 (z 1)n+1
= 1 +
n=1
n(n + 1)

X (1)n (z 1)n
z Log z z = 1 + for |z 1| < 1
n=2
n(n 1)

Solution 12.21
We evaluate the derivatives of ez at z = 0. Then we use Taylors Theorem.
dn z
e = ez
dz n
dn z


z
e = e =1
dz n
z=0

X zn
ez =
n=0
n!

606
Since the exponential function has no singularities in the finite complex plane, the radius of convergence is infinite.
We find the Taylor series for the cosine and sine by writing them in terms of the exponential function.

ez + ez
cos z =
2

!
1 X (z)n X (z)n
= +
2 n=0 n! n=0
n!

X (z)n
=
n=0
n!
even n


X (1)n z 2n
cos z =
n=0
(2n)!

ez ez
sin z =
2

!
1 X (z)n X (z)n
=
2 n=0 n! n=0
n!

X (z)n
=
n=0
n!
odd n


X (1)n z 2n+1
sin z =
n=0
(2n + 1)!

607
Solution 12.22

cos z = cos(z )

X (1)n (z )2n
=
n=0
(2n)!

X (1)n+1 (z )2n
=
n=0
(2n)!

sin z = sin(z )

X (1)n (z )2n+1
=
n=0
(2n + 1)!

X (1)n+1 (z )2n+1
=
n=0
(2n + 1)!
Solution 12.23
CONTINUE

Solution 12.24
1. (a)
f (z) = ez
f (0) = 1
f 0 (0) = 1
f 00 (0) = 1
z2
ez = 1 z + + O z3

2
Since ez is entire, the Taylor series converges in the complex plane.

608
(b)

1+z
f (z) = , f () =
1z
2
f 0 (z) = 2
, f 0 () =
(1 z)
4
f 00 (z) = , f 00 () = 1 +
(1 z)3

1+z 1 +
(z )2 + O (z )3

= + (z ) +
1z 2

distance of 2 from z0 = , the radius of convergence is 2.
Since the nearest singularity, (at z = 1), is a
The series converges absolutely for |z | < 2.
(c)

ez z2
 
3
1 + z + z2 + O z3
 
= 1+z+ +O z
z1 2
5 2
= 1 2z z + O z 3

2

Since the nearest singularity, (at z = 1), is a distance of 1 from z0 = 0, the radius of convergence is 1. The
series converges absolutely for |z| < 1.

2. Since f (z) is analytic in |z z0 | < R, its Taylor series converges absolutely on this domain.

X f (n) (z0 )z n
f (z) =
n=0
n!

The Taylor series converges uniformly on any closed sub-domain of |z z0 | < R. We consider the sub-domain

609
|z z0 | < R. On the domain of uniform convergence we can interchange differentiation and summation.

0 d X f (n) (z0 )z n
f (z) =
dz n=0 n!

0
X nf (n) (z0 )z n1
f (z) =
n=1
n!

X f (n+1) (z0 )z n
f 0 (z) =
n=0
n!

Note that this is the Taylor series that we could obtain directly for f 0 (z). Since f (z) is analytic on |z z0 | < R
so is f 0 (z).

0
X f (n+1) (z0 )z n
f (z) =
n=0
n!

3.

1 d2 1 1
=
(1 z)3 dz 2 2 1 z

1 d2 X n
= z
2 dz 2 n=0

1X
= n(n 1)z n2
2 n=2

1X
= (n + 2)(n + 1)z n
2 n=0

The radius of convergence is 1, which is the distance to the nearest singularity at z = 1.

610
4. The Taylor series expansion of f (z) about z = 0 is

X f (n) (0)
f (z) = zn.
n=0
n!

We compute the derivatives of f (z).


n1
!
Y
f (n) (z) = ( k) (1 + z)n .
k=0

Now we determine the coefficients in the series.


n1
Y
(n)
f (0) = ( k)
k=0
Qn1
X ( k)
(1 + z) = k=0
zn
n=0
n!

The radius of convergence of the series is the distance to the nearest singularity of (1 + z) . This occurs at
z = 1. Thus the series converges for |z| < 1. We can corroborate this with the ratio test. We compute the
radius of convergence.
Qn1 



Q k=0 ( k) /n! n + 1
R = lim n = lim =1

k=0 ( k)) /(n + 1)!
n ( n n

If we use the binomial coefficient,   Qn1


( k)
k=0 ,
n n!
then we can write the series in a compact form.
 

X
(1 + z) = zn
n=0
n

611
Solution 12.25
For |z| < 1:
1
=
z 1 + z
X
= (z)n
n=0

(Note that |z| < 1 | z| < 1.)


For |z| > 1:
1 1 1
=
z z (1 /z)

(Note that |z| > 1 | /z| < 1.)



1 X  n
=
z n=0 z
0
1 X n n
= z
z n=
0
X
= ()n z n1
n=
1
X
= ()n+1 z n
n=

Solution 12.26
We expand the function in partial fractions.
1 1 1
f (z) = =
(z + 1)(z + 2) z+1 z+2

612
The Taylor series about z = 0 for 1/(z + 1) is
1 1
=
1+z 1 (z)
X
= (z)n , for |z| < 1
n=0

X
= (1)n z n , for |z| < 1
n=0

The series about z = for 1/(z + 1) is


1 1/z
=
1+z 1 + 1/z

1X
= (1/z)n , for |1/z| < 1
z n=0

X
= (1)n z n1 , for |z| > 1
n=0
1
X
= (1)n+1 z n , for |z| > 1
n=

The Taylor series about z = 0 for 1/(z + 2) is


1 1/2
=
2+z 1 + z/2

1X
= (z/2)n , for |z/2| < 1
2 n=0

X (1)n
= zn, for |z| < 2
n=0
2n+1

613
The series about z = for 1/(z + 2) is

1 1/z
=
2+z 1 + 2/z

1X
= (2/z)n , for |2/z| < 1
z n=0

X
= (1)n 2n z n1 , for |z| > 2
n=0
1
X (1)n+1 n
= z , for |z| > 2
n=
2n+1

To find the expansions in the three regions, we just choose the appropriate series.

1.

1 1
f (z) =
1+z 2+z

X X (1)n n
= (1)n z n n+1
z , for |z| < 1
n=0 n=0
2
 
X 1
= (1) 1 n+1 z n , for |z| < 1
n

n=0
2


X 2n+1 1 n
f (z) = (1)n z , for |z| < 1
n=0
2n+1

614
2.
1 1
f (z) =
1+z 2+z
1
X
n+1 n
X (1)n
f (z) = (1) z zn, for 1 < |z| < 2
n= n=0
2n+1

3.
1 1
f (z) =
1+z 2+z
1 1
X X (1)n+1 n
= (1)n+1 z n n+1
z , for 2 < |z|
n= n=
2

1
X 2n+1 1 n
f (z) = (1)n+1 z , for 2 < |z|
n=
2n+1

Solution 12.27
Laurent Series. We assume that m is a non-negative integer and that n is an integer. The Laurent series about the
point z = 0 of  m
1
f (z) = z +
z
is
X
f (z) = an z n
n=

where I
1 f (z)
an = dz
2 C z n+1

615
and C is a contour going around the origin once in the positive direction. We manipulate the coefficient integral into
the desired form.

(z + 1/z)m
I
1
an = dz
2 C z n+1
Z 2
1 (e + e )m
= e d
2 0 e(n+1)
Z 2
1
= 2m cosm en d
2 0
2m1 2
Z
= cosm (cos(n) sin(n)) d
0

Note that cosm is even and sin(n) is odd about = .

2
2m1
Z
= cosm cos(n) d
0

Binomial Series. Now we find the binomial series expansion of f (z).


 m m    n
1 X m mn 1
z+ = z
z n=0
n z
m  
X m
= z m2n
n=0
n
m  
X m
= zn
n=m
(m n)/2
mn even

616
P
The coefficients in the series f (z) = n= an z n are
(
m

(mn)/2
m n m and m n even
an =
0 otherwise
By equating the coefficients found by the two methods, we evaluate the desired integral.
(
m
Z 2 
m 2m1 (mn)/2
m n m and m n even
(cos ) cos(n) d =
0 0 otherwise

Solution 12.28
First we write f (z) in the form
g(z)
f (z) = .
(z /2)(z 2)2
g(z) is an entire function which grows no faster that z 3 at infinity. By expanding g(z) in a Taylor series about the
origin, we see that it is a polynomial of degree no greater than 3.
z 3 + z 2 + z +
f (z) =
(z /2)(z 2)2
Since f (z) is a rational function we expand it in partial fractions to obtain a form that is convenient to integrate.
a b c
f (z) = + + +d
z /2 z 2 (z 2)2
We use the value of the integrals of f (z) to determine the constants, a, b, c and d.
I  
a b c
+ + + d dz = 2
|z|=1 z /2 z 2 (z 2)2
2a = 2
a=1

617
I  
1 b c
+ + + d dz = 0
|z|=3 z /2 z 2 (z 2)2
2(1 + b) = 0
b = 1

Note that by applying the second constraint, we can change the third constraint to
I
zf (z) dz = 0.
|z|=3

I  
1 1 c
z + + d dz = 0
|z|=3 z /2 z 2 (z 2)2
 
(z /2) + /2 (z 2) + 2 c(z 2) + 2c
I
+ dz = 0
|z|=3 z /2 z2 (z 2)2
 
2 2+c =0
2

c=2
2
Thus we see that the function is
1 1 2 /2
f (z) = + + d,
z /2 z 2 (z 2)2
where d is an arbitrary constant. We can also write the function in the form:

dz 3 + 15 8
f (z) = .
4(z /2)(z 2)2

Complete Laurent Series. We find the complete Laurent series about z = 0 for each of the terms in the partial

618
fraction expansion of f (z).

1 2
=
z /2 1 + 2z

X
= 2 (2z)n , for | 2z| < 1
n=0
X
= (2)n+1 z n , for |z| < 1/2
n=0

1 1/z
=
z /2 1 /(2z)

1 X  n
= , for |/(2z)| < 1
z n=0 2z
 
X n n1
= z , for |z| < 2
n=0
2
1  
X n1 n
= z , for |z| < 2
n=
2
1
X
= (2)n+1 z n , for |z| < 2
n=

619
1 1/2
=
z2 1 z/2

1 X  z n
= , for |z/2| < 1
2 n=0 2

X zn
= , for |z| < 2
n=0
2n+1

1 1/z
=
z2 1 2/z
 n
1X 2
= , for |2/z| < 1
z n=0 z

X
= 2n z n1 , for |z| > 2
n=0
1
X
= 2n1 z n , for |z| > 2
n=

620
2 /2 1
2
= (2 /2) (1 z/2)2
(z 2) 4
 
4 X 2  z n
= , for |z/2| < 1
8 n=0 n 2

4X
= (1)n (n + 1)(1)n 2n z n , for |z| < 2
8 n=0

4Xn+1 n
= z , for |z| < 2
8 n=0 2n

 2
2 /2 2 /2 2
2
= 1
(z 2) z2 z
   n
2 /2 X 2 2
= 2
, for |2/z| < 1
z n=0
n z

X
= (2 /2) (1)n (n + 1)(1)n 2n z n2 , for |z| > 2
n=0
2
X
= (2 /2) (n 1)2n2 z n , for |z| > 2
n=
2
X n+1 n
= (2 /2) z , for |z| > 2
n=
2n+2

We take the appropriate combination of these series to find the Laurent series expansions in the regions: |z| < 1/2,

621
1/2 < |z| < 2 and 2 < |z|. For |z| < 1/2, we have

X X zn
n+1 n 4Xn+1 n
f (z) = (2) z + + z +d
n=0 n=0
2n+1 8 n=0 2n
 
X
n+1 1 4n+1
f (z) = (2) + n+1 + n
zn + d
n=0
2 8 2
  
X
n+1 1 4
f (z) = (2) + n+1 1 + (n + 1) z n + d, for |z| < 1/2
n=0
2 4

For 1/2 < |z| < 2, we have


1
X X zn 4Xn+1 n
f (z) = (2)n+1 z n + n+1
+ n
z +d
n= n=0
2 8 n=0
2
1   
X
n+1 n
X 1 4
f (z) = (2) z + n+1
1+ (n + 1) z n + d, for 1/2 < |z| < 2
n= n=0
2 4

For 2 < |z|, we have


1 1 2
X
n+1 n
X
n1 n
X n+1 n
f (z) = (2) z 2 z (2 /2) z +d
n= n= n=
2n+2
2  
X
n+1 1
f (z) = (2) (1 + (1 /4)(n + 1)) z n + d, for 2 < |z|
n=
2n+1

Solution 12.29
The radius of convergence of the series for f (z) is
3 k k3

k /3
R = lim = 3 lim = 3.
n (k + 1)3 /3k+1 n (k + 1)3

622
Thus f (z) is a function which is analytic inside the circle of radius 3.
1. The integrand is analytic. Thus by Cauchys theorem the value of the integral is zero.
I
ez f (z) dz = 0
|z|=1

2. We use Cauchys integral formula to evaluate the integral.


2 3!33
I
f (z) 2 (3)
4
dz = f (0) = = 2
|z|=1 z 3! 3! 33
I
f (z)
4
dz = 2
|z|=1 z

3. We use Cauchys integral formula to evaluate the integral.


f (z) ez 1!13
I
2 d
z
dz = (f (z) e ) z=0
= 2
|z|=1 z2 1! dz 31
f (z) ez
I
2
2
dz =
|z|=1 z 3

Solution 12.30
1. (a)
1 1 1
= +
z(1 z) z 1z

1 X n
= + z , for 0 < |z| < 1
z n=0

1 X
= + zn, for 0 < |z| < 1
z n=1

623
(b)

1 1 1
= +
z(1 z) z 1z
1 1 1
=
z z 1 1/z
 n
1 1X 1
= , for |z| > 1
z z n=0 z

1 X n
= z , for |z| > 1
z n=1

X
= zn, for |z| > 1
n=2

624
(c)
1 1 1
= +
z(1 z) z 1z
1 1
= +
(z + 1) 1 2 (z + 1)
1 1 1 1
= , for |z + 1| > 1 and |z + 1| > 2
(z + 1) 1 1/(z + 1) (z + 1) 1 2/(z + 1)

1 X 1 1 X 2n
= , for |z + 1| > 1 and |z + 1| > 2
(z + 1) n=0 (z + 1)n (z + 1) n=0 (z + 1)n

1 X 1 2n
= , for |z + 1| > 2
(z + 1) n=0 (z + 1)n

X 1 2n
= , for |z + 1| > 2
n=1
(z + 1)n+1
X
1 2n1 (z + 1)n ,

= for |z + 1| > 2
n=2

2. First we factor the denominator of f (z) = 1/(z 4 + 4).

z 4 + 4 = (z 1 )(z 1 + )(z + 1 )(z + 1 + )

We look for an annulus about z = 1 containing the point z = where f (z) is analytic. The singularities at
z = 1 are a distance of 1 from z =
1; the singularities at z = 1 are at a distance of 5. Since f (z) is
analytic in the domain 1 < |z 1| < 5 there is a convergent Laurent series in that domain.

625
Chapter 13

The Residue Theorem

Man will occasionally stumble over the truth, but most of the time he will pick himself up and continue on.

- Winston Churchill

13.1 The Residue Theorem

We will find that many integrals on closed contours may be evaluated in terms of the residues of a function. We first
define residues and then prove the Residue Theorem.

626
Result 13.1.1 Residues. Let f (z) be single-valued an analytic in a deleted neighborhood
of z0 . Then f (z) has the Laurent series expansion

X
f (z) = an (z z0 )n ,
n=

1
The residue of f (z) at z = z0 is the coefficient of the zz0 term:

Res(f (z), z0 ) = a1 .

The residue at a branch point or non-isolated singularity is undefined as the Laurent series
does not exist. If f (z) has a pole of order n at z = z0 then we can use the Residue Formula:
dn1 
 
1 n

Res(f (z), z0 ) = lim (z z0 ) f (z) .
zz0 (n 1)! dz n1

See Exercise 13.4 for a proof of the Residue Formula.

Example 13.1.1 In Example 8.4.5 we showed that f (z) = z/ sin z has first order poles at z = n, n Z \ {0}. Now

627
C B

Figure 13.1: Deform the contour to lie in the deleted disk.

we find the residues at these isolated singularities.

 z   z 
Res , z = n = lim (z n)
sin z zn sin z
z n
= n lim
zn sin z
1
= n lim
zn cos z
1
= n
(1)n
= (1)n n

Residue Theorem. We can evaluate many integrals in terms of the residues of a function. Suppose f (z) has only
one singularity, (at z = z0 ), inside the simple, closed, positively oriented contour C. f (z) has a convergent
R Laurent
series in some deleted disk about z0 . We deform C to lie in the disk. See Figure 13.1. We now evaluate C f (z) dz by
deforming the contour and using the Laurent series expansion of the function.

628
Z Z
f (z) dz = f (z) dz
C B
Z
X
= an (z z0 )n dz
B n=
r e(+2)
(z z0 )n+1

e(+2)
X
= an + a1 [log(z z0 )]rr e
n=
n+1 r e
n6=1

= a1 2

Z
f (z) dz = 2 Res(f (z), z0 )
C

Now assume that f (z) has n singularities at {z1 , . . . , zn }. We deform C to n contours C1 , . . . , Cn which enclose the
R lie in deleted disks about the singularities in which f (z) has convergent Laurent series. See Figure 13.2.
singularities and
We evaluate C f (z) dz by deforming the contour.

Z n Z
X n
X
f (z) dz = f (z) dz = 2 Res(f (z), zk )
C k=1 Ck k=1

Now instead let f (z) be analytic outside and on C except for isolated singularities at {n } in the
R domain outside C
and perhaps an isolated singularity at infinity. Let a be any point in the interior of C. To evaluate C f (z) dz we make
the change of variables = 1/(z a). This maps the contour C to C 0 . (Note that C 0 is negatively oriented.) All
the points outside C are mapped to points inside C 0 and vice versa. We can then evaluate the integral in terms of the
singularities inside C 0 .

629
C2

C1 C3
C

Figure 13.2: Deform the contour n contours which enclose the n singularities.

 
1
I I
1
f (z) dz = f +a d
C C0 2
I  
1 1
= 2
f + a dz
C 0 z z
       
X 1 1 1 1 1
= 2 Res 2
f +a , + 2 Res 2
f + a ,0 .
n
z z n a z z

630
C

a C

Figure 13.3: The change of variables = 1/(z a).

Result 13.1.2 Residue Theorem. If f (z) is analytic in a compact, closed, connected


domain D except for isolated singularities at {zn } in the interior of D then
Z XI X
f (z) dz = f (z) dz = 2 Res(f (z), zn ).
D k Ck n

Here the set of contours {Ck } make up the positively oriented boundary D of the domain
D. If the boundary of the domain is a single contour C then the formula simplifies.
I X
f (z) dz = 2 Res(f (z), zn )
C n

If instead f (z) is analytic outside and on C except for isolated singularities at {n } in the
domain outside C and perhaps an isolated singularity at infinity then
I        
X 1 1 1 1 1
f (z) dz = 2 Res 2 f +a , + 2 Res 2 f + a ,0 .
C n
z z n a z z
631
Here a is a any point in the interior of C.
Example 13.1.2 Consider Z
1 sin z
dz
2 C z(z 1)
where C is the positively oriented circle of radius 2 centered at the origin. Since the integrand is single-valued with
only isolated singularities, the Residue Theorem applies. The value of the integral is the sum of the residues from
singularities inside the contour.
The only places that the integrand could have singularities are z = 0 and z = 1. Since

sin z cos z
lim = lim = 1,
z0 z z0 1
there is a removable singularity at the point z = 0. There is no residue at this point.
Now we consider the point z = 1. Since sin(z)/z is analytic and nonzero at z = 1, that point is a first order pole
of the integrand. The residue there is
 
sin z sin z
Res , z = 1 = lim (z 1) = sin(1).
z(z 1) z1 z(z 1)

There is only one singular point with a residue inside the path of integration. The residue at this point is sin(1).
Thus the value of the integral is Z
1 sin z
dz = sin(1)
2 C z(z 1)

Example 13.1.3 Evaluate the integral Z


cot z coth z
dz
C z3
where C is the unit circle about the origin in the positive direction.
The integrand is
cot z coth z cos z cosh z
=
z3 z 3 sin z sinh z

632
sin z has zeros at n. sinh z has zeros at n. Thus the only pole inside the contour of integration is at z = 0. Since
sin z and sinh z both have simple zeros at z = 0,

sin z = z + O(z 3 ), sinh z = z + O(z 3 )

the integrand has a pole of order 5 at the origin. The residue at z = 0 is

1 d4 1 d4
 
5 cot zcoth z
z 2 cot z coth z

lim z = lim
z0 4! dz 4 z3 z0 4! dz 4

1
= lim 24 cot(z) coth(z)csc(z)2 32z coth(z)csc(z)4
4! z0
16z cos(2z) coth(z)csc(z)4 + 22z 2 cot(z) coth(z)csc(z)4
+ 2z 2 cos(3z) coth(z)csc(z)5 + 24 cot(z) coth(z)csch(z)2
+ 24csc(z)2 csch(z)2 48z cot(z)csc(z)2 csch(z)2
48z coth(z)csc(z)2 csch(z)2 + 24z 2 cot(z) coth(z)csc(z)2 csch(z)2
+ 16z 2 csc(z)4 csch(z)2 + 8z 2 cos(2z)csc(z)4 csch(z)2
32z cot(z)csch(z)4 16z cosh(2z) cot(z)csch(z)4
+ 22z 2 cot(z) coth(z)csch(z)4 + 16z 2 csc(z)2 csch(z)4

2 2 4 2 5
+ 8z cosh(2z)csc(z) csch(z) + 2z cosh(3z) cot(z)csch(z)
 
1 56
=
4! 15
7
=
45

633
Since taking the fourth derivative of z 2 cot z coth z really sucks, we would like a more elegant way of finding the
residue. We expand the functions in the integrand in Taylor series about the origin.
  
z2 z4 z2 z4
cos z cosh z 1 2
+ 24
1 + 2
+ 24
+
= 3 5 3 z5
z 3 sin z sinh z z 3 z z6 + 120z
z + z6 + 120
 
+
z4
1 6
+
= 1 1
 
z3 z + z6
2
36
+ 60 +
4
1 1 z6 +
= z4
z 5 1 90 +
z4 z4
  
1
= 1 + 1+ +
z5 6 90
 
1 7 4
= 1 z +
z5 45
1 7 1
= 5
+
z 45 z
7
Thus we see that the residue is 45 . Now we can evaluate the integral.
Z
cot z coth z 14
3
dz =
C z 45

13.2 Cauchy Principal Value for Real Integrals


13.2.1 The Cauchy Principal Value
First we recap improper integrals. If f (x) has a singularity at x0 (a . . . b) then
Z b Z x0  Z b
f (x) dx lim+ f (x) dx + lim+ f (x) dx.
a 0 a 0 x0 +

634
For integrals on ( . . . ),
Z Z b
f (x) dx lim f (x) dx.
a, b a
R1 1
Example 13.2.1 1 x
dx is divergent. We show this with the definition of improper integrals.
Z 1 Z  Z 1
1 1 1
dx = lim+ dx + lim+ dx
1 x 0 1 x 0 x
= lim+ [ln |x|]
1 + lim+ [ln |x|]1
0 0
= lim+ ln  lim+ ln
0 0

The integral diverges because  and approach zero independently.


Since 1/x is an odd function, it appears that the area under the curve is zero. Consider what would happen if  and
were not independent. If they approached zero symmetrically, = , then the value of the integral would be zero.
Z  Z 1 
1
lim+ + dx = lim+ (ln  ln ) = 0
0 1  x 0

We could make the integral have any value we pleased by choosing = c. 1
Z  Z 1 
1
lim+ + dx = lim+ (ln  ln(c)) = ln c
0 1 c x 0

We have seen it is reasonable that Z 1


1
dx
1 x
has some meaning, and if we could evaluate the integral, the most reasonable value would be zero. The Cauchy principal
value provides us with a way of evaluating such integrals. If f (x) is continuous on (a, b) except at the point x0 (a, b)
1
This may remind you of conditionally convergent series. You can rearrange the terms to make the series sum to any number.

635
then the Cauchy principal value of the integral is defined
Z b Z x0  Z b 
f (x) dx = lim+ f (x) dx + f (x) dx .
a 0 a x0 +

The Cauchy principal value is obtained by approaching the singularity symmetrically. The principal value of the integral
may exist when the integral diverges.
R 1 If1 the integral exists, it is equal to the principal value of the integral.
The Cauchy principal value of 1 x dx is defined
Z 1 Z  Z 1 
1 1 1
dx lim+ dx + dx
1 x 0 1 x  x
= lim+ [log |x|] 1
1 [log |x|]
0
= lim+ (log | | log ||)
0
= 0.
R
(Another notation for the principal value of an integral is PV f (x) dx.) Since the limits of integration approach zero
symmetrically, the two halves of the integral cancel. If the limits of integration approached zero independently, (the
definition of the integral), then the two halves would both diverge.
R
Example 13.2.2 x2x+1 dx is divergent. We show this with the definition of improper integrals.
Z Z b
x x
2
dx = lim dx
x +1 a, b a +1 x2
 b
1 2
= lim ln(x + 1)
a, b 2
 2 a
1 b +1
= lim ln
2 a, b a2 + 1

636
The integral diverges because a and b approach infinity independently. Now consider what would happen if a and b
were not independent. If they approached zero symmetrically, a = b, then the value of the integral would be zero.

b2 + 1
 
1
lim ln =0
2 b b2 + 1

We could make the integral have any value we pleased by choosing a = cb.
R
We can assign a meaning to divergent integrals of the form
f (x) dx with the Cauchy principal value. The
Cauchy principal value of the integral is defined

Z Z a
f (x) dx = lim f (x) dx.
a a

The Cauchy principal value is obtained by approaching infinity symmetrically.


R
The Cauchy principal value of x2x+1 dx is defined

Z Z a
x x
2
dx = lim dx
x +1 a a +1 x2
 a
 
1 2
= lim ln x + 1
a 2
a
= 0.

637
Result 13.2.1 Cauchy Principal Value. If f (x) is continuous on (a, b) except at the point
x0 (a, b) then the integral of f (x) is defined
Z b Z x0  Z b
f (x) dx = lim+ f (x) dx + lim+ f (x) dx.
a 0 a 0 x0 +

The Cauchy principal value of the integral is defined


Z b Z x0  Z b 
f (x) dx = lim+ f (x) dx + f (x) dx .
a 0 a x0 +

If f (x) is continuous on (, ) then the integral of f (x) is defined


Z Z b
f (x) dx = lim f (x) dx.
a, b a

The Cauchy principal value of the integral is defined


Z Z a
f (x) dx = lim f (x) dx.
a a

The principal value of the integral may exist when the integral diverges. If the integral exists,
it is equal to the principal value of the integral.
R
Example 13.2.3 Clearly
x dx diverges, however the Cauchy principal value exists.
Z  2
x
x dx = lim a=0
a 2 a

638
In general, if f (x) is an odd function with no singularities on the finite real axis then

Z
f (x) dx = 0.

13.3 Cauchy Principal Value for Contour Integrals

Example 13.3.1 Consider the integral


Z
1
dz,
Cr z1

where Cr is the positively oriented circle of radius r and center at the origin. From the residue theorem, we know that
the integral is
Z (
1 0 for r < 1,
dz =
Cr z 1 2 for r > 1.

When r = 1, the integral diverges, as there is a first order pole on the path of integration. However, the principal value
of the integral exists.

Z Z 2
1 1
dz = lim+ e d
Cr z1 0  1 e

2
= lim+ log(e 1) 
0

639
We choose the branch of the logarithm with a branch cut on the positive real axis and arg log z (0, 2).

= lim+ log e(2) 1 log (e 1)


 
0
= lim+ log 1 i + O(2 ) 1 log 1 + i + O(2 ) 1
   
0
= lim+ log i + O(2 ) log i + O(2 )
 
0
= lim+ Log  + O(2 ) + arg  + O(2 ) Log  + O(2 ) arg  + O(2 )
   
0
3
=
2 2
=

Thus we obtain
Z
1 0
for r < 1,
dz = for r = 1,
Cr z1
2 for r > 1.

In the above example we evaluated the contour integral by parameterizing the contour. This approach is only
feasible when the integrand is simple. We would like to use the residue theorem to more easily evaluate the principal
value of the integral. But before we do that, we will need a preliminary result.

Result 13.3.1 Let f (z) have a first order pole at z = z0 and let (z z0 )f (z) be analytic in
some neighborhood of z0 . Let the contour C be a circular arc from z0 + e to z0 + e .
(We assume that > and < 2.)
Z
lim+ f (z) dz = ( ) Res(f (z), z0 )
0 C

The contour is shown in Figure 13.4. (See Exercise 13.9 for a proof of this result.)

640
C

z0

Figure 13.4: The C Contour

Cp

Figure 13.5: The indented contour.

Example 13.3.2 Consider


Z
1
dz
C z1
where C is the unit circle. Let Cp be the circular arc of radius 1 that starts and ends a distance of  from z = 1. Let
C be the positive, circular arc of radius  with center at z = 1 that joins the endpoints of Cp . Let Ci , be the union of
Cp and C . (Cp stands for Principal value Contour; Ci stands for Indented Contour.) Ci is an indented contour that
avoids the first order pole at z = 1. Figure 13.5 shows the three contours.

641
Note that the principal value of the integral is

Z Z
1 1
dz = lim+ dz.
C z1 0 Cp z1

We can calculate the integral along Ci with the residue theorem.

Z
1
dz = 2
Ci z1

We can calculate the integral along C using Result 13.3.1. Note that as  0+ , the contour becomes a semi-circle,
a circular arc of radians.
Z  
1 1
lim dz = Res , 1 =
0+ C z 1 z1

Now we can write the principal value of the integral along C in terms of the two known integrals.

Z Z Z
1 1 1
dz = dz dz
C z1 Ci z 1 C z 1
= 2
=

In the previous example, we formed an indented contour that included the first order pole. You can show that if we
had indented the contour to exclude the pole, we would obtain the same result. (See Exercise 13.11.)
We can extend the residue theorem to principal values of integrals. (See Exercise 13.10.)

642
Result 13.3.2 Residue Theorem for Principal Values. Let f (z) be analytic inside and
on a simple, closed, positive contour C, except for isolated singularities at z1 , . . . , zm inside
the contour and first order poles at 1 , . . . , n on the contour. Further, let the contour be C 1
at the locations of these first order poles. (i.e., the contour does not have a corner at any of
the first order poles.) Then the principal value of the integral of f (z) along C is
Z Xm Xn
f (z) dz = 2 Res(f (z), zj ) + Res(f (z), j ).
C j=1 j=1

13.4 Integrals on the Real Axis


Example 13.4.1 We wish to evaluate the integral
Z
1
dx.
x2 +1

We can evaluate this integral directly using calculus.


Z
1
dx = [arctan x]

x2 +1
=

Now we will evaluate the integral using contour integration. Let CR be the semicircular arc from R to R in the upper
half plane. Let C be the union of CR and the interval [R, R].
We can evaluate the integral along C with the residue theorem. The integrand has first order poles at z = . For

643
R > 1, we have
Z  
1 1
2
dz = 2 Res ,
C z +1 z2 + 1
1
= 2
2
= .

Now we examine the integral along CR . We use the maximum modulus integral bound to show that the value of the
integral vanishes as R .
Z
1 1

2+1
dz R max 2


CR z zC R z + 1
1
= R 2
R 1
0 as R .

Now we are prepared to evaluate the original real integral.


Z
1
2
dz =
C z +1
Z R Z
1 1
2
dx + 2
dz =
R x + 1 CR z + 1

We take the limit as R .


Z
1
dx =
x2 +1

We would get the same result by closing the path of integration in the lower half plane. Note that in this case the
closed contour would be in the negative direction.

644
If you are really observant, you may have noticed that we did something a little funny in evaluating
Z
1
2
dx.
x + 1

The definition of this improper integral is


Z Z 0 Z b
1 1 1
2
dx = lim 2
dx+ = lim 2
dx.
x + 1 a+ a x + 1 b+ 0 x + 1

In the above example we instead computed Z R


1
lim dx.
R+ R x2 + 1
x
Note that for some integrands, the former and latter are not the same. Consider the integral of x2 +1
.
Z Z 0 Z b
x x x
2
dx = lim 2
dx + lim 2
dx
x +1 a+ a x + 1 b+ 0 x + 1
   
1 2 1 2
= lim log |a + 1| + lim log |b + 1|
a+ 2 b+ 2
Note that the limits do not exist and hence the integral diverges. We get a different result if the limits of integration
approach infinity symmetrically.
Z R  
x 1 2 2
lim dx = lim (log |R + 1| log |R + 1|)
R+ R x2 + 1 R+ 2
=0

(Note that the integrand is an odd function, so the integral from R to R is zero.) We call this the principal value of
the integral and denote it by writing PV in front of the integral sign or putting a dash through the integral.
Z Z Z R
PV f (x) dx f (x) dx lim f (x) dx
R+ R

645
The principal value of an integral may exist when the integral diverges. If the integral does converge, then it is
equal to its principal value.
We can use the method of Example 13.4.1 to evaluate the principal value of integrals of functions that vanish fast
enough at infinity.

Result 13.4.1 Let f (z) be analytic except for isolated singularities, with only first order poles
on the real axis. Let CR be the semi-circle from R to R in the upper half plane. If
 
lim R max |f (z)| = 0
R zCR

then Z m n
X X
f (x) dx = 2 Res (f (z), zk ) + Res(f (z), xk )
k=1 k=1
where z1 , . . . zm are the singularities of f (z) in the upper half plane and x1 , . . . , xn are the
first order poles on the real axis.
Now let CR be the semi-circle from R to R in the lower half plane. If
 
lim R max |f (z)| = 0
R zCR

then Z m n
X X
f (x) dx = 2 Res (f (z), zk ) Res(f (z), xk )
k=1 k=1
where z1 , . . . zm are the singularities of f (z) in the lower half plane and x1 , . . . , xn are the
first order poles on the real axis.

646
This result is proved in Exercise 13.13. Of course we can use this result to evaluate the integrals of the form
Z
f (z) dz,
0
where f (x) is an even function.

13.5 Fourier Integrals


In order to do Fourier transforms, which are useful in solving differential equations, it is necessary to be able to calculate
Fourier integrals. Fourier integrals have the form
Z
ex f (x) dx.

We evaluate these integrals by closing the path of integration in the lower or upper half plane and using techniques of
contour integration.
Consider the integral
Z /2
eR sin d.
0
Since 2/ sin for 0 /2,
eR sin eR2/ for 0 /2
Z /2 Z /2
R sin
e d eR2/ d
0 0
h R2/ i/2
= e
2R 0
R
= (e 1)
2R


2R
0 as R

647
We can use this to prove the following Result 13.5.1. (See Exercise 13.17.)

Result 13.5.1 Jordans Lemma.


Z

eR sin d < .
0 R
Suppose that f (z) vanishes as |z| . If is a (positive/negative) real number and CR is
a semi-circle of radius R in the (upper/lower) half plane then the integral
Z
f (z) ez dz
CR

vanishes as R .
R
We can use Jordans Lemma and the Residue Theorem to evaluate many Fourier integrals. Consider f (x) ex dx,
where is a positive real number. Let f (z) be analytic except for isolated singularities, with only first order poles on
the real axis. Let C be the contour from R to R on the real axis and then back to R along a semi-circle in the
upper half plane. If R is large enough so that C encloses all the singularities of f (z) in the upper half plane then
Z m
X n
X
f (z) ez dz = 2 Res(f (z) ez , zk ) + Res(f (z) ez , xk )
C k=1 k=1

where z1 , . . . zm are the singularities of f (z) in the upper half plane and x1 , . . . , xn are the first order poles on the real
axis. If f (z) vanishes as |z| then the integral on CR vanishes as R by Jordans Lemma.
Z Xm n
X
x z
f (x) e dx = 2 Res(f (z) e , zk ) + Res(f (z) ez , xk )
k=1 k=1

For negative we close the path of integration in the lower half plane. Note that the contour is then in the negative
direction.

648
Result 13.5.2 Fourier Integrals. Let f (z) be analytic except for isolated singularities, with
only first order poles on the real axis. Suppose that f (z) vanishes as |z| . If is a
positive real number then
Z Xm n
X
x z
f (x) e dx = 2 Res(f (z) e , zk ) + Res(f (z) ez , xk )
k=1 k=1

where z1 , . . . zm are the singularities of f (z) in the upper half plane and x1 , . . . , xn are the
first order poles on the real axis. If is a negative real number then
Z Xm Xn
x z
f (x) e dx = 2 Res(f (z) e , zk ) Res(f (z) ez , xk )
k=1 k=1

where z1 , . . . zm are the singularities of f (z) in the lower half plane and x1 , . . . , xn are the
first order poles on the real axis.

13.6 Fourier Cosine and Sine Integrals


Fourier cosine and sine integrals have the form,

Z Z
f (x) cos(x) dx and f (x) sin(x) dx.
0 0

If f (x) is even/odd then we can evaluate the cosine/sine integral with the method we developed for Fourier integrals.

649
Let f (z) be analytic except for isolated singularities, with only first order poles on the real axis. Suppose that f (x)
is an even function and that f (z) vanishes as |z| . We consider real > 0.

Z
1
Z
f (x) cos(x) dx = f (x) cos(x) dx
0 2

Since f (x) sin(x) is an odd function,

1
Z
f (x) sin(x) dx = 0.
2

Thus
Z
1
Z
f (x) cos(x) dx = f (x) ex dx
0 2

Now we apply Result 13.5.2.

Z m n
X
z X
f (x) cos(x) dx = Res(f (z) e , zk ) + Res(f (z) ez , xk )
0 k=1
2 k=1

where z1 , . . . zm are the singularities of f (z) in the upper half plane and x1 , . . . , xn are the first order poles on the real
axis.
If f (x) is an odd function, we note that f (x) cos(x) is an odd function to obtain the analogous result for Fourier
sine integrals.

650
Result 13.6.1 Fourier Cosine and Sine Integrals. Let f (z) be analytic except for isolated
singularities, with only first order poles on the real axis. Suppose that f (x) is an even function
and that f (z) vanishes as |z| . We consider real > 0.
Z m n
X
z X
f (x) cos(x) dx = Res(f (z) e , zk ) + Res(f (z) ez , xk )
0 2
k=1 k=1

where z1 , . . . zm are the singularities of f (z) in the upper half plane and x1 , . . . , xn are the
first order poles on the real axis. If f (x) is an odd function then,
Z n
X
z X
f (x) sin(x) dx = Res(f (z) e , k ) + Res(f (z) ez , xk )
0 2
k=1 k=1

where 1 , . . . are the singularities of f (z) in the lower half plane and x1 , . . . , xn are the first
order poles on the real axis.

Now suppose that f (x) is neither even nor odd. We can evaluate integrals of the form:
Z Z
f (x) cos(x) dx and f (x) sin(x) dx

by writing them in terms of Fourier integrals


Z
1 1
Z Z
f (x) cos(x) dx = x
f (x) e dx + f (x) ex dx
2 2
Z
Z Z

f (x) sin(x) dx = f (x) ex dx + f (x) ex dx
2 2

651
CR

Figure 13.6:

13.7 Contour Integration and Branch Cuts


Example 13.7.1 Consider

xa
Z
dx, 0 < a < 1,
0 x+1
where xa denotes exp(a ln(x)). We choose the branch of the function

z a
f (z) = |z| > 0, 0 < arg z < 2
z+1

with a branch cut on the positive real axis.


Let C and CR denote the circular arcs of radius  and R where  < 1 < R. C is negatively oriented; CR is
positively oriented. Consider the closed contour C that is traced by a point moving from C to CR above the branch
cut, next around CR , then below the cut to C , and finally around C . (See Figure 13.6.)
We write f (z) in polar coordinates.

exp(a log z) exp(a(log r + i))


f (z) = =
z+1 r e +1

652
We evaluate the function above, (z = r e0 ), and below, (z = r e2 ), the branch cut.

exp[a(log r + i0)] ra
f (r e0 ) = =
r+1 r+1
a e2a
exp[a(log r + 2)] r
f (r e2 ) = = .
r+1 r+1
We use the residue theorem to evaluate the integral along C.
I
f (z) dz = 2 Res(f (z), 1)
C
Z R a Z R a 2a
r e
Z Z
r
dr + f (z) dz dr + f (z) dz = 2 Res(f (z), 1)
 r+1 CR  r+1 C

The residue is
Res(f (z), 1) = exp(a log(1)) = exp(a(log 1 + )) = ea .
We bound the integrals along C and CR with the maximum modulus integral bound.

a 1a
Z

f (z) dz 2
= 2

C 1 1
a
R1a
Z
2R R

f (z) dz = 2

CR
R1 R1

Since 0 < a < 1, the values of the integrals tend to zero as  0 and R . Thus we have
Z a
r ea
dr = 2
0 r+1 1 e2a
Z a
x
dx =
0 x+1 sin a

653
Result 13.7.1 Integrals from Zero to Infinity. Let f (z) be a single-valued analytic func-
tion with only isolated singularities and no singularities on the positive, real axis, [0, ). Let
a 6 Z. If the integrals exist then,
Z Xn
f (x) dx = Res (f (z) log z, zk ) ,
0 k=1
Z n
a 2 X
x f (x) dx = 2a
Res (z a f (z), zk ) ,
0 1 e
k=1
Z n n
1X 2
 X
f (x) log x dx = Res f (z) log z, zk + Res (f (z) log z, zk ) ,
0 2
k=1 k=1

Z n
a 2 X
x f (x) log x dx = 2a
Res (z a f (z) log z, zk )
0 1e
k=1
n
2a X
+ 2 Res (z a f (z), zk ) ,
sin (a) k=1
n
!
Z m
2 X
xa f (x) logm x dx = Res (z a f (z), zk ) ,
0 am 1 e2a
k=1
where z1 , . . . , zn are the singularities of f (z) and there is a branch cut on the positive real
axis with 0 < arg(z) < 2.

654
13.8 Exploiting Symmetry

We have already
R used symmetry of the integrand to evaluate certain integrals. For f (x) an even function we were
able to evaluate 0 f (x) dx by extending the range of integration from to . For

Z
x f (x) dx
0

we put a branch cut on the positive real axis and noted that the value of the integrand below the branch cut is a
constant multiple of the value of the function above the branch cut. This enabled us to evaluate the real integral with
contour integration. In this section we will use other kinds of symmetry to evaluate integrals. We will discover that
periodicity of the integrand will produce this symmetry.

13.8.1 Wedge Contours

We note that z n = rn en is periodic in with period 2/n. The real and imaginary parts of z n are odd periodic
in with period /n. This observation suggests that certain integrals on the positive real axis may be evaluated by
closing the path of integration with a wedge contour.

Example 13.8.1 Consider


Z
1
dx
0 1 + xn

655
where n N, n 2. We can evaluate this integral using Result 13.7.1.
Z n1  
1 X log z (1+2k)/n
dx = Res ,e
0 1 + xn k=0
1 + zn
n1
(z e(1+2k)/n ) log z
X  
= lim
k=0
ze(1+2k)/n 1 + zn
n1
log z + (z e(1+2k)/n )/z
X  
= lim
k=0
ze(1+2k)/n nz n1
n1  
X (1 + 2k)/n
=
n e(1+2k)(n1)/n
k=0
n1
X
= (1 + 2k) e2k/n
n2 e(n1)/n k=0
n1
2e/n X
= k e2k/n
n2 k=1
2 e/n n
= 2 2/n
n e 1

=
n sin(/n)
This is a bit grungy. To find a spiffier way to evaluate the integral we note that if we write the integrand as a function
of r and , it is periodic in with period 2/n.
1 1
n
=
1+z 1 + rn en
The integrand along the rays = 2/n, 4/n, 6/n, . . . has the same value as the integrand on the real axis. Consider
the contour C that is the boundary of the wedge 0 < r < R, 0 < < 2/n. There is one singularity inside the

656
contour. We evaluate the residue there.
z e/n
 
1
Res , e/n = lim
1 + zn ze/n 1 + z
n

1
= lim
ze /n nz n1
e/n
=
n
We evaluate the integral along C with the residue theorem.

2 e/n
Z
1
n
dz =
C 1+z n
Let CR be the circular arc. The integral along CR vanishes as R .
Z
1 2R 1

n
dz max

CR 1 + z n zCR 1 + z n
2R 1

n Rn 1
0 as R

We parametrize the contour to evaluate the desired integral.


Z Z 0
1 1 2/n 2 e/n
dx + e dx =
0 1 + xn 1+x
n n
Z /n
1 2 e
dx =
0 1 + xn n(1 e2/n )
Z
1
dx =
0 1 + xn n sin(/n)

657
13.8.2 Box Contours
Recall that ez = ex+y is periodic in y with period 2. This implies that the hyperbolic trigonometric functions
cosh z, sinh z and tanh z are periodic in y with period 2 and odd periodic in y with period . We can exploit this
property to evaluate certain integrals on the real axis by closing the path of integration with a box contour.

Example 13.8.2 Consider the integral


Z  x i
1 h 
dx = log tanh +
cosh x 4 2

= log(1) log(1)
= .

We will evaluate this integral using contour integration. Note that

ex+ + ex
cosh(x + ) = = cosh(x).
2

Consider the box contour C that is the boundary of the region R < x < R, 0 < y < . The only singularity of
the integrand inside the contour is a first order pole at z = /2. We evaluate the integral along C with the residue
theorem.
I  
1 1
dz = 2 Res ,
C cosh z cosh z 2
z /2
= 2 lim
z/2 cosh z
1
= 2 lim
z/2 sinh z

= 2

658
The integrals along the sides of the box vanish as R .
Z R+
1 1
dz max

R cosh z z[R...R+] cosh z

2
max R+y

y[0...] e + eRy
2
=
eR
eR


sinh R
0 as R
The value of the integrand on the top of the box is the negative of its value on the bottom. We take the limit as
R .
Z Z
1 1
dx + dx = 2
cosh x cosh x
Z
1
dx =
cosh x

13.9 Definite Integrals Involving Sine and Cosine


Example 13.9.1 For real-valued a, evaluate the integral:
Z 2
d
f (a) = .
0 1 + a sin
What is the value of the integral for complex-valued a.
Real-Valued a. For 1 < a < 1, the integrand is bounded, hence the integral exists. For |a| = 1, the integrand
has a second order pole on the path of integration. For |a| > 1 the integrand has two first order poles on the path of
integration. The integral is divergent for these two cases. Thus we see that the integral exists for 1 < a < 1.

659
For a = 0, the value of the integral is 2. Now consider a 6= 0. We make the change of variables z = e . The real
integral from = 0 to = 2 becomes a contour integral along the unit circle, |z| = 1. We write the sine, cosine and
the differential in terms of z.
z z 1 z + z 1 dz
sin = , cos = , dz = e d, d =
2 2 z
We write f (a) as an integral along C, the positively oriented unit circle |z| = 1.
I I
1/(z) 2/a
f (a) = 1
dz = 2
dz
C 1 + a(z z )/(2) C z + (2/a)z 1

We factor the denominator of the integrand.


I
2/a
f (a) = dz
C (z z1 )(z z2 )
   
1 + 1 a2 1 1 a2
z1 = , z2 =
a a
Because |a| < 1, the second root is outside the unit circle.

1 + 1 a2
|z2 | = > 1.
|a|
Since |z1 z2 | = 1, |z1 | < 1. Thus the pole at z1 is inside the contour and the pole at z2 is outside. We evaluate the
contour integral with the residue theorem.
I
2/a
f (a) = 2
dz
C z + (2/a)z 1
2/a
= 2
z1 z2
1
= 2
1 a2

660
2
f (a) =
1 a2
Complex-Valued a. We note that the integral converges except for real-valued a satisfying |a| 1. On any closed
subset of C \ {a R | |a| 1} the integral is uniformly convergent. Thus except for the values {a R | |a| 1},
we can differentiate the integral with respect to a. f (a) is analytic in the complex plane except for the set of points
on the real axis: a ( . . . 1] and a [1 . . . ). The value of the analytic function f (a) on the real axis for the
interval (1 . . . 1) is
2
f (a) = .
1 a2
By analytic continuation we see that the value of f (a) in the complex plane is the branch of the function

2
f (a) =
(1 a2 )1/2

where f (a) is positive, real-valued for a (1 . . . 1) and there are branch cuts on the real axis on the intervals:
( . . . 1] and [1 . . . ).

Result 13.9.1 For evaluating integrals of the form


Z a+2
F (sin , cos ) d
a

it may be useful to make the change of variables z = e . This gives us a contour integral
along the unit circle about the origin. We can write the sine, cosine and differential in terms
of z.
z z 1 z + z 1 dz
sin = , cos = , d =
2 2 z

661
13.10 Infinite Sums
The function g(z) = cot(z) has simple poles at z = n Z. The residues at these points are all unity.

(z n) cos(z)
Res( cot(z), n) = lim
zn sin(z)
cos(z) (z n) sin(z)
= lim
zn cos(z)
=1

Let Cn be the square contour with corners at z = (n + 1/2)(1 ). Recall that

cos z = cos x cosh y sin x sinh y and sin z = sin x cosh y + cos x sinh y.

First we bound the modulus of cot(z).



cos x cosh y sin x sinh y
| cot(z)| =
sin x cosh y + cos x sinh y
s
cos2 x cosh2 y + sin2 x sinh2 y
=
sin2 x cosh2 y + cos2 x sinh2 y
s
cosh2 y

sinh2 y
= | coth(y)|

The hyperbolic cotangent, coth(y), has a simple pole at y = 0 and tends to 1 as y .


Along the top and bottom of Cn , (z = x (n + 1/2)), we bound the modulus of g(z) = cot(z).

| cot(z)| coth((n + 1/2))

662
Along the left and right sides of Cn , (z = (n + 1/2) + y), the modulus of the function is bounded by a constant.

cos((n + 1/2)) cosh(y) sin((n + 1/2)) sinh(y)
|g((n + 1/2) + y)| =
sin((n + 1/2)) cosh(y) + cos((n + 1/2)) sinh(y)
= | tanh(y)|

Thus the modulus of cot(z) can be bounded by a constant M on Cn .

Let f (z) be analytic except for isolated singularities. Consider the integral,
I
cot(z)f (z) dz.
Cn

We use the maximum modulus integral bound.


I

cot(z)f (z) dz (8n + 4)M max |f (z)|

Cn zCn

Note that if
lim |zf (z)| = 0,
|z|

then I
lim cot(z)f (z) dz = 0.
n Cn

This implies that the sum of all residues of cot(z)f (z) is zero. Suppose further that f (z) is analytic at z = n Z.
The residues of cot(z)f (z) at z = n are f (n). This means

X
f (n) = ( sum of the residues of cot(z)f (z) at the poles of f (z) ).
n=

663
Result 13.10.1 If
lim |zf (z)| = 0,
|z|

then the sum of all the residues of cot(z)f (z) is zero. If in addition f (z) is analytic at
z = n Z then

X
f (n) = ( sum of the residues of cot(z)f (z) at the poles of f (z) ).
n=

Example 13.10.1 Consider the sum



X 1
, a 6 Z.
n=
(n + a)2

By Result 13.10.1 with f (z) = 1/(z + a)2 we have


 
X 1 1
2
= Res cot(z) 2
, a
n=
(n + a) (z + a)
d
= lim cot(z)
za dz
sin2 (z) cos2 (z)
= .
sin2 (z)


X 1 2
=
n=
(n + a)2 sin2 (a)

Example 13.10.2 Derive /4 = 1 1/3 + 1/5 1/7 + 1/9 .

664
Consider the integral Z
1 dw
In =
2 Cn w(w z) sin w
where Cn is the square with corners at w = (n + 1/2)(1 ), n Z+ . With the substitution w = x + y,

| sin w|2 = sin2 x + sinh2 y,

we see that |1/ sin w| 1 on Cn . Thus In 0 as n . We use the residue theorem and take the limit n .

(1)n (1)n

X 1 1
0= + + 2
n=1
n(n z) n(n + z) z sin z z


1 1 X (1)n
= 2z
sin z z n=1
n2 2 z 2

1 X (1)n (1)n

=
z n=1 n z n + z

We substitute z = /2 into the above expression to obtain

/4 = 1 1/3 + 1/5 1/7 + 1/9

665
13.11 Exercises
The Residue Theorem
Exercise 13.1
Evaluate the following closed contour integrals using Cauchys residue theorem.
Z
dz
1. 2
, where C is the contour parameterized by r = 2 cos(2), 0 2.
C z 1

ez
Z
2. 2
dz, where C is the positive circle |z| = 3.
C z (z 2)(z + 5)
Z
3. e1/z sin(1/z) dz, where C is the positive circle |z| = 1.
C

Hint, Solution
Exercise 13.2
Derive Cauchys integral formula from Cauchys residue theorem.
Hint, Solution
Exercise 13.3
Calculate the residues of the following functions at each of the poles in the finite part of the plane.
1
1.
z4 a4
sin z
2.
z2
1 + z2
3.
z(z 1)2
ez
4.
z 2 + a2

666
(1 cos z)2
5.
z7
Hint, Solution
Exercise 13.4
Let f (z) have a pole of order n at z = z0 . Prove the Residue Formula:

dn1
 
1 n
Res(f (z), z0 ) = lim [(z z0 ) f (z)] .
zz0 (n 1)! dz n1

Hint, Solution
Exercise 13.5
Consider the function
z4
f (z) = .
z2 + 1
Classify the singularities of f (z) in the extended complex plane. Calculate the residue at each pole and at infinity. Find
the Laurent series expansions and their domains of convergence about the points z = 0, z = and z = .
Hint, Solution
Exercise 13.6
Let P (z) be a polynomial none of whose roots lie on the closed contour . Show that

P 0 (z)
Z
1
dz = number of roots of P (z) which lie inside .
2 P (z)

where the roots are counted according to their multiplicity.


Hint: From the fundamental theorem of algebra, it is always possible to factor P (z) in the form P (z) = (z z1 )(z
z2 ) (z zn ). Using this form of P (z) the integrand P 0 (z)/P (z) reduces to a very simple expression.
Hint, Solution

667
Exercise 13.7
Find the value of
ez
I
dz
C (z ) tan z
where C is the positively-oriented circle

1. |z| = 2

2. |z| = 4

Hint, Solution

Cauchy Principal Value for Real Integrals


Solution 13.1
Show that the integral
Z 1
1
dx.
1 x
is divergent. Evaluate the integral
Z 1
1
dx, R, 6= 0.
1 x
Evaluate Z 1
1
lim+ dx
0 1 x
and Z 1
1
lim dx.
0 1 x
The integral exists for arbitrarily close to zero, but diverges when = 0. Plot the real and imaginary part of the
integrand. If one were to assign meaning to the integral for = 0, what would the value of the integral be?

668
Exercise 13.8
Do the principal values of the following integrals exist?
R1
1. 1 x12 dx,
R1
2. 1 x13 dx,
R1
3. 1 fx(x)
3 dx.

Assume that f (x) is real analytic on the interval (1, 1).


Hint, Solution

Cauchy Principal Value for Contour Integrals


Exercise 13.9
Let f (z) have a first order pole at z = z0 and let (z z0 )f (z) be analytic in some neighborhood of z0 . Let the contour
C be a circular arc from z0 + e to z0 + e . (Assume that > and < 2.) Show that
Z
lim+ f (z) dz = ( ) Res(f (z), z0 )
0 C

Hint, Solution
Exercise 13.10
Let f (z) be analytic inside and on a simple, closed, positive contour C, except for isolated singularities at z1 , . . . , zm
inside the contour and first order poles at 1 , . . . , n on the contour. Further, let the contour be C 1 at the locations of
these first order poles. (i.e., the contour does not have a corner at any of the first order poles.) Show that the principal
value of the integral of f (z) along C is
Z X m n
X
f (z) dz = 2 Res(f (z), zj ) + Res(f (z), j ).
C j=1 j=1

Hint, Solution

669
Exercise 13.11
Let C be the unit circle. Evaluate Z
1
dz
C z1
by indenting the contour to exclude the first order pole at z = 1.
Hint, Solution

Integrals on the Real Axis


Exercise 13.12
Evaluate the following improper integrals.
Z
x2
1. 2 2
dx =
0 (x + 1)(x + 4) 6
Z
dx
2. 2 2
, a>0
(x + b) + a

Hint, Solution
Exercise 13.13
Prove Result 13.4.1.
Hint, Solution
Exercise 13.14
Evaluate Z
2x
.
x2 +x+1
Hint, Solution
Exercise 13.15
Use contour integration to evaluate the integrals

670
Z
dx
1. ,
1 + x4

x2 dx
Z
2. ,
(1 + x2 )2
Z
cos(x)
3. dx.
1 + x2
Hint, Solution
Exercise 13.16
Evaluate by contour integration

x6
Z
dx.
0 (x4 + 1)2
Hint, Solution

Fourier Integrals
Exercise 13.17
Suppose that f (z) vanishes as |z| . If is a (positive / negative) real number and CR is a semi-circle of radius
R in the (upper / lower) half plane then show that the integral
Z
f (z) ez dz
CR

vanishes as R .
Hint, Solution
Exercise 13.18
Evaluate by contour integration Z
cos 2x
dx.
x

671
Hint, Solution

Fourier Cosine and Sine Integrals


Exercise 13.19
Evaluate Z
sin x
dx.
x
Hint, Solution
Exercise 13.20
Evaluate

1 cos x
Z
dx.
x2
Hint, Solution
Exercise 13.21
Evaluate
Z
sin(x)
dx.
0 x(1 x2 )
Hint, Solution

Contour Integration and Branch Cuts


Exercise 13.22
Evaluate the following integrals.
Z 2
ln x 3
1. dx =
0 1 + x2 8
Z
ln x
2. dx = 0
0 1 + x2

672
Hint, Solution
Exercise 13.23
By methods of contour integration find Z
dx
0 x2 + 5x + 6
R
[ Recall the trick of considering
f (z) log z dz with a suitably chosen contour and branch for log z. ]
Hint, Solution
Exercise 13.24
Show that

xa
Z
a
2
dx = for 1 < <(a) < 1.
0 (x + 1) sin(a)
From this derive that

log2 x 2
Z Z
log x
dx = 0, dx = .
0 (x + 1)2 0 (x + 1)2 3
Hint, Solution
Exercise 13.25
Consider the integral

xa
Z
I(a) = dx.
0 1 + x2
1. For what values of a does the integral exist?
2. Evaluate the integral. Show that

I(a) =
2 cos(a/2)
3. Deduce from your answer in part (b) the results
Z
log2 x 3
Z
log x
dx = 0, dx = .
0 1 + x2 0 1 + x2 8

673
You may assume that it is valid to differentiate under the integral sign.
Hint, Solution
Exercise 13.26
Let f (z) be a single-valued analytic function with only isolated singularities and no singularities on the positive real
axis, [0, ). Give sufficient conditions on f (x) for absolute convergence of the integral
Z
xa f (x) dx.
0

Assume that a is not an integer. Evaluate the integral by considering the integral of z a f (z) on a suitable contour.
(Consider the branch of z a on which 1a = 1.)
Hint, Solution
Exercise 13.27
Using the solution to Exercise 13.26, evaluate
Z
xa f (x) log x dx,
0

and Z
xa f (x) logm x dx,
0
where m is a positive integer.
Hint, Solution
Exercise 13.28
Using the solution to Exercise 13.26, evaluate Z
f (x) dx,
0
i.e. examine a = 0. The solution will suggest
R a way to evaluate the integral with contour integration. Do the contour
integration to corroborate the value of 0 f (x) dx.
Hint, Solution

674
Exercise 13.29
Let f (z) be an analytic function with only isolated singularities and no singularities on the positive real axis, [0, ).
Give sufficient conditions on f (x) for absolute convergence of the integral
Z
f (x) log x dx
0

Evaluate the integral with contour integration.


Hint, Solution
Exercise 13.30
For what values of a does the following integral exist?
Z
xa
dx.
0 1 + x4
Evaluate the integral. (Consider the branch of xa on which 1a = 1.)
Hint, Solution
Exercise 13.31
By considering the integral of f (z) = z 1/2 log z/(z + 1)2 on a suitable contour, show that
Z 1/2 Z
x log x x1/2
2
dx = , 2
dx = .
0 (x + 1) 0 (x + 1) 2
Hint, Solution

Exploiting Symmetry
Exercise 13.32
Evaluate by contour integration, the principal value integral
Z
eax
I(a) = x x
dx
e e

675
for a real and |a| < 1. [Hint: Consider the contour that is the boundary of the box, R < x < R, 0 < y < , but
indented around z = 0 and z = .
Hint, Solution
Exercise 13.33
Evaluate the following integrals.
Z
dx
1. ,
0 (1 + x2 )2
Z
dx
2. .
0 1 + x3
Hint, Solution
Exercise 13.34
Find the value of the integral I
Z
dx
I=
0 1 + x6
by considering the contour integral Z
dz
1 + z6
with an appropriately chosen contour .
Hint, Solution
Exercise 13.35
2
Let C be the boundary of the sector 0 < r < R, 0 < < /4. By integrating ez on C and letting R show
that Z Z Z
1 2
2
cos(x ) dx = 2
sin(x ) dx = ex dx.
0 0 2 0
Hint, Solution

676
Exercise 13.36
Evaluate Z
x
dx
sinh x
using contour integration.
Hint, Solution
Exercise 13.37
Show that

eax
Z

x
dx = for 0 < a < 1.
e +1 sin(a)
Use this to derive that Z
cosh(bx)
dx = for 1 < b < 1.
cosh x cos(b/2)
Hint, Solution
Exercise 13.38
Using techniques of contour integration find for real a and b:
Z
d
F (a, b) = 2
0 (a + b cos )

What are the restrictions on a and b if any? Can the result be applied for complex a, b? How?
Hint, Solution
Exercise 13.39
Show that Z
cos x
dx =
ex + ex e/2 + e/2
[ Hint: Begin by considering the integral of ez /(ez + ez ) around a rectangle with vertices: R, R + .]
Hint, Solution

677
Definite Integrals Involving Sine and Cosine
Exercise 13.40
Evaluate the following real integrals.
Z
d
1. = 2
1 + sin2
Z /2
2. sin4 d
0

Hint, Solution
Exercise 13.41
Use contour integration to evaluate the integrals
Z 2
d
1. ,
0 2 + sin()
Z
cos(n)
2. d for |a| < 1, n Z0+ .
1 2a cos() + a2

Hint, Solution
Exercise 13.42
By integration around the unit circle, suitably indented, show that
Z
cos(n) sin(n)
d = .
0 cos cos sin

Hint, Solution

678
Exercise 13.43
Evaluate
1
x2
Z
dx.
0 (1 + x2 ) 1 x2
Hint, Solution

Infinite Sums
Exercise 13.44
Evaluate

X 1
4
.
n=1
n
Hint, Solution
Exercise 13.45
Sum the following series using contour integration:

X 1
n=
n2 2

Hint, Solution

679
13.12 Hints
The Residue Theorem
Hint 13.1

Hint 13.2

Hint 13.3

Hint 13.4
Substitute the Laurent series into the formula and simplify.

Hint 13.5
Use that the sum of all residues of the function in the extended complex plane is zero in calculating the residue at
infinity. To obtain the Laurent series expansion about z = , write the function as a proper rational function, (numerator
has a lower degree than the denominator) and expand in partial fractions.

Hint 13.6

Hint 13.7

Cauchy Principal Value for Real Integrals


Hint 13.8

680
Hint 13.9
For the third part, does the integrand have a term that behaves like 1/x2 ?

Cauchy Principal Value for Contour Integrals


Hint 13.10
Expand f (z) in a Laurent series. Only the first term will make a contribution to the integral in the limit as  0+ .

Hint 13.11
Use the result of Exercise 13.9.

Hint 13.12
Look at Example 13.3.2.

Integrals on the Real Axis


Hint 13.13

Hint 13.14
Close the path of integration in the upper or lower half plane with a semi-circle. Use the maximum modulus integral
bound, (Result 10.2.1), to show that the integral along the semi-circle vanishes.

Hint 13.15
Make the change of variables x = 1/.

Hint 13.16
Use Result 13.4.1.

Hint 13.17

681
Fourier Integrals
Hint 13.18
Use
Z

eR sin d < .
0 R

Hint 13.19

Fourier Cosine and Sine Integrals


Hint 13.20
ex
Consider the integral of x
.

Hint 13.21
Show that
Z
1 cos x 1 ex
Z
dx = dx.
x2 x2

Hint 13.22
Show that

ex
Z Z
sin(x)
dx = dx.
0 x(1 x2 ) 2 x(1 x2 )

Contour Integration and Branch Cuts


Hint 13.23
Integrate a branch of log2 z/(1 + z 2 ) along the boundary of the domain  < r < R, 0 < < .

Hint 13.24

682
Hint 13.25
Note that
Z 1
xa dx
0

converges for <(a) > 1; and


Z
xa dx
1

converges for <(a) < 1.


Consider f (z) = z a /(z + 1)2 with a branch cut along the positive real axis and the contour in Figure ?? in the limit
as 0 and R .
To derive the last two integrals, differentiate with respect to a.

Hint 13.26

Hint 13.27
Consider the integral of z a f (z) on the contour in Figure ??.

Hint 13.28
Differentiate with respect to a.

Hint 13.29
Take the limit as a 0. Use LHospitals rule. To corroborate the result, consider the integral of f (z) log z on an
appropriate contour.

Hint 13.30
Consider the integral of f (z) log2 z on the contour in Figure ??.

683
Hint 13.31
Consider the integral of
za
f (z) =
1 + z4
on the boundary of the region  < r < R, 0 < < /2. Take the limits as  0 and R .

Hint 13.32
Consider the branch of f (z) = z 1/2 log z/(z + 1)2 with a branch cut on the positive real axis and 0 < arg z < 2.
Integrate this function on the contour in Figure ??.

Exploiting Symmetry
Hint 13.33

Hint 13.34
For the second part, consider the integral along the boundary of the region, 0 < r < R, 0 < < 2/3.

Hint 13.35

Hint 13.36
To show that the integral on the quarter-circle vanishes as R establish the inequality,
4
cos 2 1 , 0 .
4
Hint 13.37
Consider the box contour C this is the boundary of the rectangle, R x R, 0 y . The value of the integral
is 2 /2.

Hint 13.38
Consider the rectangular contour with corners at R and R + 2. Let R .

684
Hint 13.39

Hint 13.40

Definite Integrals Involving Sine and Cosine


Hint 13.41

Hint 13.42

Hint 13.43

Hint 13.44
Make the changes of variables x = sin and then z = e .

Infinite Sums
Hint 13.45
Use Result 13.10.1.

Hint 13.46

685
1

-1 1

-1

Figure 13.7: The contour r = 2 cos(2).

13.13 Solutions

The Residue Theorem


Solution 13.2
1. We consider

Z
dz
C z2 1

where C is the contour parameterized by r = 2 cos(2), 0 2. (See Figure 13.7.) There are first order

686
poles at z = 1. We evaluate the integral with Cauchys residue theorem.
Z     
dz 1 1
2
= 2 Res , z = 1 + Res , z = 1
C z 1 z2 1 z2 1
 
1 1
= 2 +
z + 1 z=1 z 1 z=1
=0

2. We consider the integral


ez
Z
2
dz,
C z (z 2)(z + 5)
where C is the positive circle |z| = 3. There is a second order pole at z = 0, and first order poles at z = 2 and
z = 5. The poles at z = 0 and z = 2 lie inside the contour. We evaluate the integral with Cauchys residue
theorem.
ez ez
Z   
2
dz = 2 Res ,z = 0
C z (z 2)(z + 5) z 2 (z 2)(z + 5)
ez
 
+ Res ,z = 2
z 2 (z 2)(z + 5)
ez ez
 
d
= 2 + 2
dz (z 2)(z + 5) z=0 z (z + 5) z=2

ez z
 
d e
= 2 + 2
dz (z 2)(z + 5) z=0 z (z + 5) z=2

(z 2 + (7 2)z 5 12) ez
   
1 5
= 2 2 2
+ e2
(z 2) (z + 5) 58 116
   z=0
3 1 5
= 2 + + e2
25 20 58 116
 
5 1 6 1 5
= + cos 2 sin 2 + + cos 2 + sin 2
10 58 29 25 29 58

687
3. We consider the integral Z
e1/z sin(1/z) dz
C
where C is the positive circle |z| = 1. There is an essential singularity at z = 0. We determine the residue there
by expanding the integrand in a Laurent series.
     
1/z 1 1 1 1
e sin(1/z) = 1 + + O + O
z z2 z z3
 
1 1
= +O
z z2
The residue at z = 0 is 1. We evaluate the integral with the residue theorem.
Z
e1/z sin(1/z) dz = 2
C

Solution 13.3
If f () is analytic in a compact, closed, connected domain D and z is a point in the interior of D then Cauchys integral
formula states I
(n) n! f ()
f (z) = d.
2 D ( z)n+1
To corroborate this, we evaluate the integral with Cauchys residue theorem. There is a pole of order n + 1 at the point
= z.
n! 2 dn
I
n! f ()
d. = f ()
2 D ( z)n+1 2 n! d n
=z

= f (n) (z)
Solution 13.4
1.
1 1
=
z4 a 4 (z a)(z + a)(z a)(z + a)

688
There are first order poles at z = a and z = a. We calculate the residues there.
 
1 1 1
Res 4 4
, z = a = = 3
z a (z + a)(z a)(z + a) z=a 4a

 
1 1 1
Res 4 4
, z = a = = 3
z a (z a)(z a)(z + a) z=a
4a
 
1 1
Res 4 4
, z = a = = 3
z a (z a)(z + a)(z + a) z=a 4a

 
1 1
Res 4 4
, z = a = = 3
z a (z a)(z + a)(z a) z=a
4a

2.
sin z
z2
Since denominator has a second order zero at z = 0 and the numerator has a first order zero there, the function
has a first order pole at z = 0. We calculate the residue there.
 
sin z sin z
Res 2
, z = 0 = lim
z z0 z
cos z
= lim
z0 1
=1

3.
1 + z2
z(z 1)2
There is a first order pole at z = 0 and a second order pole at z = 1.
1 + z2 1 + z 2
 
Res ,z = 0 = =1
z(z 1)2 (z 1)2 z=0

689
1 + z2 d 1 + z 2
 
Res ,z = 1 =
z(z 1)2 dz z z=1
 
1
= 1 2
z z=1
=0

4. ez / (z 2 + a2 ) has first order poles at z = a. We calculate the residues there.

ez ez ea
 
Res , z = a = =
z 2 + a2 z + a z=a 2a
z z
ea
 
e e
Res , z = a = =
z 2 + a2 z a z=a 2a

2
5. Since 1 cos z has a second order zero at z = 0, (1cos
z7
z)
has a third order pole at that point. We find the
residue by expanding the function in a Laurent series.

(1 cos z)2 z2 z4  2
  
7 6
=z 1 1 + +O z
z7 2 24
 2
 2
z4

7 z 6
=z +O z
2 24
 4
z6

7 z 8

=z +O z
4 24
1 1
= 3 + O(z)
4z 24z

The residue at z = 0 is 1/24.

690
Solution 13.5
Since f (z) has an isolated pole of order n at z = z0 , it has a Laurent series that is convergent in a deleted neighborhood
about that point. We substitute this Laurent series into the Residue Formula to verify it.

dn1
 
1 n
Res(f (z), z0 ) = lim [(z z0 ) f (z)]
zz0 (n 1)! dz n1
"
#!
1 dn1 X
= lim (z z0 )n ak (z z0 )k
zz0 (n 1)! dz n1 k=n
" #!
n1
1 d X
= lim akn (z z0 )k
zz0 (n 1)! dz n1 k=0

!
1 X k!
= lim akn (z z0 )kn+1
zz0 (n 1)! k=n1 (k n + 1)!

!
1 X (k + n 1)!
= lim ak1 (z z0 )k
zz0 (n 1)! k=0 k!
1 (n 1)!
= a1
(n 1)! 0!
= a1

This proves the Residue Formula.

Solution 13.6
Classify Singularities.
z4 z4
f (z) = = .
z2 + 1 (z )(z + )

There are first order poles at z = . Since the function behaves like z 2 at infinity, there is a second order pole there.

691
To see this more slowly, we can make the substitution z = 1/ and examine the point = 0.
4
 
1 1 1
f = 2 = 2 4
= 2
+1 + (1 + 2 )
f (1/) has a second order pole at = 0, which implies that f (z) has a second order pole at infinity.
Residues. The residues at z = are,
 4
z4

z
Res 2
, = lim = ,
z +1 z z + 2
 4
z4

z
Res 2
, = lim = .
z +1 z z 2
The residue at infinity is
   
1 1
Res(f (z), ) = Res f , = 0
2
1 4
 
= Res , = 0
2 2 + 1
4
 
= Res , = 0
1 + 2

Here we could use the residue formula, but its easier to find the Laurent expansion.

!
X
= Res 4 (1)n 2n , = 0
n=0
=0
We could also calculate the residue at infinity by recalling that the sum of all residues of this function in the extended
complex plane is zero.

+ + Res(f (z), ) = 0
2 2

692
Res(f (z), ) = 0
Laurent Series about z = 0. Since the nearest singularities are at z = , the Taylor series will converge in the
disk |z| < 1.

z4 1
2
= z4
z +1 1 (z)2
X
4
=z (z 2 )n
n=0

X
= z4 (1)n z 2n
n=0

X
= (1)n z 2n
n=2

This geometric series converges for | z 2 | < 1, or |z| < 1. The series expansion of the function is


z4 X
2
= (1)n z 2n for |z| < 1
z + 1 n=2

Laurent Series about z = . We expand f (z) in partial fractions. First we write the function as a proper rational
function, (i.e. the numerator has lower degree than the denominator). By polynomial division, we see that

1
f (z) = z 2 1 + .
z2 +1
Now we expand the last term in partial fractions.

/2 /2
f (z) = z 2 1 + +
z z+

693
Since the nearest singularity is at z = , the Laurent series will converge in the annulus 0 < |z | < 2.

z 2 1 = ((z ) + )2 1
= (z )2 + 2(z ) 2

/2 /2
=
z+ 2 + (z )
1/4
=
1 (z )/2
 n
1 X (z )
=
4 n=0 2

1 X n
= (z )n
4 n=0 2n

This geometric series converges for |(z )/2| < 1, or |z | < 2. The series expansion of f (z) is


/2 1 X n
f (z) = 2 + 2(z ) + (z )2 + (z )n .
z 4 n=0 2n


z4 /2 2 1 X n
2
= 2 + 2(z ) + (z ) + n
(z )n for |z | < 2
z +1 z 4 n=0 2

Laurent Series about z = . Since the nearest singularities are at z = , the Laurent series will converge in

694
the annulus 1 < |z| < .

z4 z2
=
z2 + 1 1 + 1/z 2
 n
2
X 1
=z 2
n=0
z
0
X
= (1)n z 2(n+1)
n=
1
X
= (1)n+1 z 2n
n=

This geometric series converges for | 1/z 2 | < 1, or |z| > 1. The series expansion of f (z) is

1
z4 X
= (1)n+1 z 2n for 1 < |z| <
z 2 + 1 n=

Solution 13.7
Method 1: Residue Theorem. We factor P (z). Let m be the number of roots, counting multiplicities, that lie
inside the contour . We find a simple expression for P 0 (z)/P (z).

n
Y
P (z) = c (z zk )
k=1
n Y
X n
0
P (z) = c (z zj )
k=1 j=1
j6=k

695
Pn Qn
0 c k=1 j=1 (z zj )
P (z) j6=k
=
c nk=1 (z zk )
Q
P (z)
Qn
n
X j6j=1 (z zj )
=k
= Qn
k=1 j=1 (z zj )
n
X 1
=
k=1
z zk

Now we do the integration using the residue theorem.

n
P 0 (z)
Z Z X
1 1 1
dz = dz
2 P (z) 2 k=1 z zk
n Z
X 1 1
= dz
k=1
2 z zk
Z
X 1 1
= dz
zk inside
2 z zk
X
= 1
zk inside

=m

Qn
Method 2: Fundamental Theorem of Calculus. We factor the polynomial, P (z) = c k=1 (z zk ). Let m be

696
the number of roots, counting multiplicities, that lie inside the contour .

P 0 (z)
Z
1 1
dz = [log P (z)]C
2 P (z) 2
" n
#
1 Y
= log (z zk )
2 k=1
" n #C
1 X
= log(z zk )
2 k=1
C

The value of the logarithm changes by 2 for the terms in which zk is inside the contour. Its value does not change
for the terms in which zk is outside the contour.

" #
1 X
= log(z zk )
2 zk inside C
1 X
= 2
2 zk inside

=m

Solution 13.8
1.
ez ez cos z
I I
dz = dz
C (z ) tan z C (z ) sin z

The integrand has first order poles at z = n, n Z, n 6= 1 and a double pole at z = . The only pole inside

697
the contour occurs at z = 0. We evaluate the integral with the residue theorem.
ez cos z ez cos z
I  
dz = 2 Res ,z = 0
C (z ) sin z (z ) sin z
ez cos z
= 2 lim z
z=0 (z ) sin z
z
= 2 lim
z=0 sin z
1
= 2 lim
z=0 cos z
= 2

ez
I
dz = 2
C (z ) tan z

2. The integrand has a first order poles at z = 0, and a second order pole at z = inside the contour. The
value of the integral is 2 times the sum of the residues at these points. From the previous part we know that
residue at z = 0.
ez cos z
 
1
Res ,z = 0 =
(z ) sin z
We find the residue at z = with the residue formula.
ez cos z ez cos z
 
Res , z = = lim (z + )
(z ) sin z z (z ) sin z

e (1) z+
= lim
2 z sin z
e 1
= lim
2 z cos z
e
=
2

698
We find the residue at z = by finding the first few terms in the Laurent series of the integrand.
ez cos z (e + e (z ) + O ((z )2 )) (1 + O ((z )2 ))
=
(z ) sin z (z ) ((z ) + O ((z )3 ))
e e (z ) + O ((z )2 )
=
(z )2 + O ((z )4 )
e e
(z)2
+ z + O(1)
=
1 + O ((z )2 )
e e
 
+ O(1) 1 + O (z )2

= 2
+
(z ) z
e e
= + + O(1)
(z )2 z
With this we see that
ez cos z
 
Res ,z = = e .
(z ) sin z
The integral is
ez cos z ez cos z ez cos z
I     
dz = 2 Res , z = + Res ,z = 0
C (z ) sin z (z ) sin z (z ) sin z
ez cos z
 
+ Res ,z =
(z ) sin z
1 e
 

= 2 +e
2

ez
I
dz = 2 e 2 e

C (z ) tan z

Cauchy Principal Value for Real Integrals

699
Solution 13.9
Consider the integral
Z 1
1
dx.
1 x
By the definition of improper integrals we have
Z 1 Z  Z 1
1 1 1
dx = lim+ dx + lim+ dx
1 x 0 1 x 0 x
= lim+ [log |x|] 1
1 + lim+ [log |x|]
0 0
= lim+ log  lim+ log
0 0

This limit diverges. Thus the integral diverges.


Now consider the integral Z 1
1
dx
1 x
where R, 6= 0. Since the integrand is bounded, the integral exists.
Z 1 Z 1
1 x +
dx = 2 2
dx
1 x 1 x +
Z 1

= 2 2
dx
1 x +
Z 1

= 2 2 2
dx
0 x +
Z 1/
1
= 2 2
d
0 +1
= 2 [arctan ]1/
 0
1
= 2 arctan

700
Figure 13.8: The real and imaginary part of the integrand for several values of .

Note that the integral exists for all nonzero real and that
Z 1
1
lim+ dx =
0 1 x

and Z 1
1
lim dx = .
0 1 x
The integral exists for arbitrarily close to zero, but diverges when = 0. The real part of the integrand is an odd
function with two humps that get thinner and taller with decreasing . The imaginary part of the integrand is an even
function with a hump that gets thinner and taller with decreasing . (See Figure 13.8.)
   
1 x 1
< = 2 , = =
x x + 2 x x2 + 2
Note that Z 1
1
< dx + as 0+
0 x
and Z 0
1
< dx as 0 .
1 x

701
However,
Z 1
1
lim < dx = 0
0 1 x

because the two integrals above cancel each other.


Now note that when = 0, the integrand is real. Of course the integral doesnt converge for this case, but if we
could assign some value to
Z 1
1
dx
1 x

it would be a real number. Since


Z 1 
1
lim < dx = 0,
0 1 x

This number should be zero.


Solution 13.10
1.

Z 1 Z  Z 1 
1 1 1
2 dx = lim+ 2
dx + 2
dx
1 x 0 1 x  x
   1 !
1 1
= lim+ +
0 x 1 x 
 
1 1
= lim+ 11+
0  

The principal value of the integral does not exist.

702
2.
Z 1 Z  Z 1 
1 1 1
3 dx = lim+ 3
dx + 3
dx
1 x 0 1 x  x
   1 !
1 1
= lim+ 2 + 2
0 2x 1 2x 
 
1 1 1 1
= lim+ + +
0 2()2 2(1)2 2(1)2 22
=0

3. Since f (x) is real analytic,



X
f (x) = fn xn for x (1, 1).
n=1

We can rewrite the integrand as

f (x) f0 f1 f2 f (x) f0 f1 x f2 x2
= + + + .
x3 x3 x2 x x3
Note that the final term is real analytic on (1, 1). Thus the principal value of the integral exists if and only if
f2 = 0.

Cauchy Principal Value for Contour Integrals


Solution 13.11
We can write f (z) as
f0 (z z0 )f (z) f0
f (z) = + .
z z0 z z0
Note that the second term is analytic in a neighborhood of z0 . Thus it is bounded on the contour. Let M be the

703
(zz0 )f (z)f0
maximum modulus of zz0
on C . By using the maximum modulus integral bound, we have
Z
(z z0 )f (z) f0
dz ( )M

C z z0
0 as  0+ .
Thus we see that Z Z
f0
lim+ f (z) dz lim+ dz.
0 C 0 C z z0
We parameterize the path of integration with
z = z0 + e , (, ).
Now we evaluate the integral.
Z Z
f0 f0
lim+ dz = lim+ e d
0 C z z0 0 e
Z
= lim+ f0 d
0
= ( )f0
( ) Res(f (z), z0 )
This proves the result.
Solution 13.12
Let Ci be the contour that is indented with circular arcs or radius  at each of the first order poles on C so as to enclose
these poles. Let A1 , . . . , An be these circular arcs of radius  centered at the points 1 , . . . , n . Let Cp be the contour,
(not necessarily connected), obtained by subtracting each of the Aj s from Ci .
Since the curve is C 1 , (or continuously differentiable), at each of the first order poles on C, the Aj s becomes
semi-circles as  0+ . Thus Z
f (z) dz = Res(f (z), j ) for j = 1, . . . , n.
Aj

704
CONTINUE

Figure 13.9: The Indented Contour.

The principal value of the integral along C is


Z Z
f (z) dz = lim+ f (z) dz
C 0 Cp
Z n Z
!
X
= lim+ f (z) dz f (z) dz
0 Ci Aj
j=1
m n
! n
X X X
= 2 Res(f (z), zj ) + Res(f (z), j ) Res(f (z), j )
j=1 j=1 j=1

Z m
X n
X
f (z) dz = 2 Res(f (z), zj ) + Res(f (z), j ).
C j=1 j=1

Solution 13.13
Consider Z
1
dz
C z1
where C is the unit circle. Let Cp be the circular arc of radius 1 that starts and ends a distance of  from z = 1. Let
C be the negative, circular arc of radius  with center at z = 1 that joins the endpoints of Cp . Let Ci , be the union of
Cp and C . (Cp stands for Principal value Contour; Ci stands for Indented Contour.) Ci is an indented contour that
avoids the first order pole at z = 1. Figure 13.9 shows the three contours.
Note that the principal value of the integral is
Z Z
1 1
dz = lim+ dz.
C z1 0 Cp z 1

705
We can calculate the integral along Ci with Cauchys theorem. The integrand is analytic inside the contour.
Z
1
dz = 0
Ci z1

We can calculate the integral along C using Result 13.3.1. Note that as  0+ , the contour becomes a semi-circle,
a circular arc of radians in the negative direction.

Z  
1 1
lim dz = Res , 1 =
0+ C z1 z1

Now we can write the principal value of the integral along C in terms of the two known integrals.
Z Z Z
1 1 1
dz = dz dz
C z1 Ci z 1 C z 1
= 0 ()
=

Integrals on the Real Axis


Solution 13.14
1. First we note that the integrand is an even function and extend the domain of integration.


x2 x2
Z Z
1
2 2
dx = dx
0 (x + 1)(x + 4) 2 (x2 + 1)(x2 + 4)

Next we close the path of integration in the upper half plane. Consider the integral along the boundary of the

706
domain 0 < r < R, 0 < < .
z2 z2
Z Z
1 1
dz = dz
2 C (z 2 + 1)(z 2 + 4) 2 C (z )(z + )(z 2)(z + 2)
z2
  
1
= 2 Res ,z =
2 (z 2 + 1)(z 2 + 4)
z2
 
+ Res , z = 2
(z 2 + 1)(z 2 + 4)
z2 2
 
z
= +
(z + )(z 2 + 4) z= (z 2 + 1)(z + 2) z=2
 
=
6 3

=
6
R RR R
Let CR be the circular arc portion of the contour. C = R + CR . We show that the integral along CR vanishes
as R with the maximum modulus bound.
2 2
Z
z
R max
z

2 + 1)(z 2 + 4)
dz (z 2 + 1)(z 2 + 4)


CR (z zC R

R2
= R 2
(R 1)(R2 4)
0 as R

We take the limit as R to evaluate the integral along the real axis.

1 R x2
Z

lim 2 2
dx =
R 2 R (x + 1)(x + 4) 6
Z 2
x
2 2
dx =
0 (x + 1)(x + 4) 6

707
2. We close the path of integration in the upper half plane. Consider the integral along the boundary of the domain
0 < r < R, 0 < < .
Z Z
dz dz
2 2
=
C (z + b) + a C (z + b a)(z + b + a)
 
1
= 2 Res , z = b + a
(z + b a)(z + b + a)

1
= 2
z + b + a z=b+a

=
a
R RR R
Let CR be the circular arc portion of the contour. C = R + CR . We show that the integral along CR vanishes
as R with the maximum modulus bound.
Z

dz
R max
1

(z + b)2 + a 2 zC (z + b)2 + a 2
CR R

1
= R
(R b)2 + a2
0 as R

We take the limit as R to evaluate the integral along the real axis.
Z R
dx
lim 2 2
=
R R (x + b) + a a
Z
dx
2 2
=
(x + b) + a a

Solution 13.15
Let CR be the semicircular arc from R to R in the upper half plane. Let C be the union of CR and the interval

708
[R, R]. We can evaluate the principal value of the integral along C with Result 13.3.2.
Z m
X n
X
f (x) dx = 2 Res (f (z), zk ) + Res(f (z), xk )
C k=1 k=1

We examine the integral along CR as R .


Z

f (z) dz R max |f (z)|

CR zCR

0 as R .

Now we are prepared to evaluate the real integral.


Z Z R
f (x) dx = lim f (x) dx
R R
Z
= lim f (z) dz
R C
Xm n
X
= 2 Res (f (z), zk ) + Res(f (z), xk )
k=1 k=1

If we close the path of integration in the lower half plane, the contour will be in the negative direction.
Z m
X n
X
f (x) dx = 2 Res (f (z), zk ) Res(f (z), xk )
k=1 k=1

Solution 13.16
We consider
Z
2x
dx.
x2 +x+1

709
With the change of variables x = 1/, this becomes
Z
2 1
 
1
2 1
d,
+ + 1 2
Z
2 1
2
d
+ + 1

There are first order poles at = 0 and = 1/2 3/2. We close the path of integration in the upper half plane
with a semi-circle. Since the integrand decays like 3 the integrand along the semi-circle vanishes as the radius tends
to infinity. The value of the integral is thus
!
2z 1 2z 1
 
1 3
Res , z = 0 + 2 Res , z = +
z2 + z + 1 z2 + z + 1 2 2

2z 1
   
2
lim + 2 lim
z0 z2 + z + 1 z(1+ 3)/2 z + (1 + 3)/2
Z
2x 2
2
dx =
x + x + 1 3

Solution 13.17
1. Consider Z
1
dx.
x4 +1
1
The integrand z 4 +1
is analytic on the real axis and has isolated singularities at the points

z = {e/4 , e3/4 , e5/4 , e7/4 }.


Let CR be the semi-circle of radius R in the upper half plane. Since
   
1 1
lim R max 4 = lim R = 0,
R zCR z + 1 R R4 1

710
we can apply Result 13.4.1.
Z     
1 1 /4 1 3/4
dx = 2 Res ,e + Res ,e
x4 + 1 z4 + 1 z4 + 1

The appropriate residues are,

z e/4
 
1
Res 4
, e/4 = lim 4
z +1 ze/4 z + 1
1
= lim 3
ze/4 4z
1
= e3/4
4
1
= ,
4 2

 
1 1
Res 4
, e3/4 =
z +1 4(e3/4 )3
1 /4
= e
4
1
= ,
4 2
We evaluate the integral with the residue theorem.
Z  
1 1 1
4
dx = 2 +
x + 1 4 2 4 2
Z
1
dx =
x4 +1 2

711
2. Now consider

x2
Z
dx.
(x2 + 1)2
The integrand is analytic on the real axis and has second order poles at z = . Since the integrand decays
sufficiently fast at infinity,

z2 R2
   

lim R max 2
= lim R 2 =0
R zCR (z + 1)2 R (R 1)2

we can apply Result 13.4.1.



x2 z2
Z  
dx = 2 Res ,z =
(x2 + 1)2 (z 2 + 1)2

z2 z2
   
d 2
Res , z = = lim (z ) 2
(z 2 + 1)2 z dz (z + 1)2
z2
 
d
= lim
z dz (z + )2
(z + )2 2z z 2 2(z + )
 
= lim
z (z + )4

=
4

x2
Z

2 2
dx =
(x + 1) 2

3. Since
sin(x)
1 + x2

712
is an odd function,

ex
Z Z
cos(x)
dx = dx
1 + x2 1 + x2
Since ez /(1 + z 2 ) is analytic except for simple poles at z = and the integrand decays sufficiently fast in the
upper half plane,  z   
e 1
lim R max = lim R 2 =0
R zCR 1 + z 2 R R 1
we can apply Result 13.4.1.

ex ez
Z  
dx = 2 Res ,z =
1 + x2 (z )(z + )
e1
= 2
2
Z
cos(x)
2
dx =
1+x e

Solution 13.18
Consider the function
z6
f (z) = .
(z 4 + 1)2
The value of the function on the imaginary axis:
y 6
(y 4 + 1)2
is a constant multiple of the value of the function on the real axis:

x6
.
(x4 + 1)2

713
Thus to evaluate the real integral we consider the path of integration, C, which starts at the origin, follows the real
axis to R, follows a circular path to R and thenfollows the imaginary axis
back down to the origin. f (z) has second
order poles at the fourth roots of 1: (1 )/ 2. Of these only (1 + )/ 2 lies inside the path of integration. We
evaluate the contour integral with the Residue Theorem. For R > 1:

z6 z6
Z  
/4
dz = 2 Res ,z = e
C (z 4 + 1)2 (z 4 + 1)2
z6
 
d /4 2
= 2 lim (z e ) 4
ze/4 dz (z + 1)2
z6
 
d
= 2 lim
ze/4 dz (z e3/4 )2 (z e5/4 )2 (z e7/4 )2
z6
= 2 lim
ze/4 (z e3/4 )2 (z e5/4 )2 (z e7/4 )2
 !
6 2 2 2

z z e3/4 z e5/4 z e7/4
!
6 2 2 2 2 2
= 2
(2)(4)(2) 1 + 2 2 + 2 2
3
= 2 (1 ) 2
32
3
= (1 + )
8 2

The integral along the circular part of the contour, CR , vanishes as R . We demonstrate this with the maximum

714
modulus integral bound.

z6 z6
Z  
R
dz
max

CR (z 4 + 1)2 4 zCR (z 4 + 1)2
R R6
=
4 (R4 1)2
0 as R

Taking the limit R , we have:

Z 0
x6 (y)6
Z
3
dx + dy = (1 + )
0 (x4 + 1)2 4
((y) + 1)
2
8 2
Z Z
x6 y6 3
4 2
dx + 4 2
dy = (1 + )
0 (x + 1) 0 (y + 1) 8 2
Z 6
x 3
(1 + ) 4 2
dx = (1 + )
0 (x + 1) 8 2
Z
x6 3
4 2
dx =
0 (x + 1) 8 2

Fourier Integrals
Solution 13.19
We know that
Z

eR sin d < .
0 R

715
First take the case that is positive and the semi-circle is in the upper half plane.
Z Z
z
z

f (z) e dz e dz max |f (z)|
zCR
CR CR
Z
R e
e R e d max |f (z)|

0 zCR
Z
R sin
=R e d max |f (z)|
0 zCR

<R max |f (z)|
R zCR

= max |f (z)|
zCR
0 as R
The procedure is almost the same for negative .
Solution 13.20
First we write the integral in terms of Fourier integrals.
Z Z Z
cos 2x e2x e2x
dx = dx + dx
x 2(x ) 2(x )
1
Note that 2(z) vanishes as |z| . We close the former Fourier integral in the upper half plane and the latter in
the lower half plane. There is a first order pole at z = in the upper half plane.
Z
e2x e2z
 
dx = 2 Res , z =
2(x ) 2(z )
e2
= 2
2
There are no singularities in the lower half plane.
Z
e2x
dx = 0
2(x )

716
Thus the value of the original real integral is
Z
cos 2x
dx = e2
x

Fourier Cosine and Sine Integrals


Solution 13.21
We are considering the integral Z
sin x
dx.
x
The integrand is an entire function. So it doesnt appear that the residue theorem would directly apply. Also the
integrand is unbounded as x + and x , so closing the integral in the upper or lower half plane is not
directly applicable. In order to proceed, we must write the integrand in a different form. Note that
Z
cos x
dx = 0
x
since the integrand is odd and has only a first order pole at x = 0. Thus
Z Z x
sin x e
dx = dx.
x x

Let CR be the semicircular arc in the upper half plane from R to R. Let C be the closed contour that is the union
of CR and the real interval [R, R]. If we close the path of integration with a semicircular arc in the upper half plane,
we have Z Z z
ez
Z 
sin x e
dx = lim dz dz ,
x R C z CR z
provided that all the integrals exist.
The integral along CR vanishes as R by Jordans lemma. By the residue theorem for principal values we have
Z z  z 
e e
dz = Res , 0 = .
z z

717
Combining these results,
Z
sin x
dx = .
x

Solution 13.22
Note that (1 cos x)/x2 has a removable singularity at x = 0. The integral decays like x12 at infinity, so the integral
exists. Since (sin x)/x2 is a odd function with a simple pole at x = 0, the principal value of its integral vanishes.
Z
sin x
2
dx = 0
x
Z Z Z
1 cos x 1 cos x sin x 1 ex
dx = dx = dx
x2 x2 x2

Let CR be the semi-circle of radius R in the upper half plane. Since


1 ez
 
lim R max = lim R 2 = 0
R zCR 2
z R R2

the integral along CR vanishes as R .

1 ez
Z
dz 0 as R
CR z2

We can apply Result 13.4.1.


Z
1 ex 1 ez 1 ez ez
 
dx = Res , z = 0 = lim = lim
x2 z2 z0 z z0 1


1 cos x
Z
dx =
x2

718
Solution 13.23
Consider
Z
sin(x)
dx.
0 x(1 x2 )

Note that the integrand has removable singularities at the points x = 0, 1 and is an even function.

Z Z
sin(x) 1 sin(x)
2
dx = dx.
0 x(1 x ) 2 x(1 x2 )

cos(x)
Note that is an odd function with first order poles at x = 0, 1.
x(1 x2 )

Z
cos(x)
2
dx = 0
x(1 x )

ex
Z Z
sin(x)
dx = dx.
0 x(1 x2 ) 2 x(1 x2 )

Let CR be the semi-circle of radius R in the upper half plane. Since

ez
 
1
lim R max 2
= lim R
2
=0
R zCR z(1 z ) R R(R 1)

the integral along CR vanishes as R .

ez
Z
dz 0 as R
CR z(1 z 2 )

719
We can apply Result 13.4.1.

ex ez ez
    

Z
dx = Res , z = 0 + Res ,z = 1
2 x(1 x2 ) 2 z(1 z 2 ) z(1 z 2 )
ez
 
+ Res , z = 1
z(1 z 2 )
ez ez ez
 

= lim lim + lim
2 z0 1 z 2 z0 z(1 + z) z0 z(1 z)
 
1 1
= 1 +
2 2 2
Z
sin(x)
dx =
0 x(1 x2 )

Contour Integration and Branch Cuts


Solution 13.24
Let C be the boundary of the region  < r < R, 0 < < . Choose the branch of the logarithm with a branch cut
on the negative imaginary axis and the angle range /2 < < 3/2. We consider the integral of log2 z/(1 + z 2 ) on
this contour.

log2 z
I  2 
log z
2
dz = 2 Res ,z =
C 1+z 1 + z2
log2 z
= 2 lim
z z +
(/2)2
= 2
2
3
=
4

720
Let CR be the semi-circle from R to R in the upper half plane. We show that the integral along CR vanishes as
R with the maximum modulus integral bound.
log2 z
Z 2
log z

2
dz R max
1 + z2

CR 1 + z zC R

ln2 R + 2 ln R + 2
R
R2 1
0 as R
Let C be the semi-circle from  to  in the upper half plane. We show that the integral along C vanishes as  0
with the maximum modulus integral bound.
log2 z
Z 2
log z

2
dz  max
1 + z2

C 1 + z zC 

ln2  2 ln  + 2

1 2
0 as  0
Now we take the limit as  0 and R for the integral along C.
log2 z 3
I
2
dz =
C 1+z 4
Z 2 Z 0
ln r (ln r + )2 3
dr + dr =
0 1 + r2 1 + r2 4
Z 2 Z Z
ln x ln x 2 1 3
2 dx + 2 dx = dx (13.1)
0 1 + x2 0 1 + x2 0 1 + x2 4
We evaluate the integral of 1/(1 + x2 ) by extending the path of integration to ( . . . ) and closing the path of
integration in the upper half plane. Since
   
1 1
lim R max lim R = 0,
R zCR 1 + z 2 R R2 1

721
the integral of 1/(1 + z 2 ) along CR vanishes as R . We evaluate the integral with the Residue Theorem.
Z
2 1
Z
2 1
dx = dx
0 1 + x2 2 1 + x2
2
 
1
= 2 Res ,z =
2 1 + z2
1
= 3 lim
z z +
3
=
2
Now we return to Equation 13.1.

ln2 x 3
Z Z
ln x
2 dx + 2 dx =
0 1 + x2 0 1 + x2 4
We equate the real and imaginary parts to solve for the desired integrals.
Z 2
ln x 3
dx =
0 1 + x2 8
Z
ln x
dx = 0
0 1 + x2

Solution 13.25
We consider the branch of the function
log z
f (z) =
z2 + 5z + 6
with a branch cut on the real axis and 0 < arg(z) < 2.
Let C and CR denote the circles of radius  and R where  < 1 < R. C is negatively oriented; CR is positively
oriented. Consider the closed contour, C, that is traced by a point moving from  to R above the branch cut, next
around CR back to R, then below the cut to , and finally around C back to . (See Figure 13.10.)

722
CR

Figure 13.10: The path of integration.

We can evaluate the integral of f (z) along C with the residue theorem. For R > 3, there are first order poles inside
the path of integration at z = 2 and z = 3.

Z     
log z log z log z
dz = 2 Res , z = 2 + Res , z = 3
C z 2 + 5z + 6 z 2 + 5z + 6 z 2 + 5z + 6
 
log z log z
= 2 lim + lim
z2 z + 3 z3 z + 2
 
log(2) log(3)
= 2 +
1 1
= 2 (log(2) + log(3) )
 
2
= 2 log
3

723
In the limit as  0, the integral along C vanishes. We demonstrate this with the maximum modulus theorem.
Z
log z log z

2
dz 2 max 2


C z + 5z + 6 zC  z + 5z + 6
2 log 
2
6 5 2
0 as  0

In the limit as R , the integral along CR vanishes. We again demonstrate this with the maximum modulus
theorem.
Z
log z
2R max
log z
dz

CRz 2 + 5z + 6 zCR z 2 + 5z + 6

log R + 2
2R
R2 5R 6
0 as R

Taking the limit as  0 and R , the integral along C is:


Z Z Z 0
log z log x log x + 2
2
dz = dx + dx
C z + 5z + 6 0 x2 + 5x + 6 2
x + 5x + 6
Z
log x
= 2 2
dx
0 x + 5x + 6
Now we can evaluate the real integral.
Z  
log x 2
2 2
dx = 2 log
0 x + 5x + 6 3
Z  
log x 3
dx = log
0 x2 + 5x + 6 2

724
Solution 13.26
We consider the integral

xa
Z
I(a) = dx.
0 (x + 1)2
To examine convergence, we split the domain of integration.
1
xa xa xa
Z Z Z
dx = dx + dx
0 (x + 1)2 0 (x + 1)2 1 (x + 1)2

First we work with the integral on (0 . . . 1).


1
Z 1
xa xa
Z

dx (x + 1)2 |dx|


0 (x + 1)2 0
Z 1
x<(a)
= 2
dx
0 (x + 1)
Z 1
x<(a) dx
0

This integral converges for <(a) > 1.


Next we work with the integral on (1 . . . ).
Z Z
xa xa



2
dx

(x + 1)2 |dx|


1 (x + 1) 1
Z
x<(a)
= dx
(x + 1)2
Z1
x<(a)2 dx
1

This integral converges for <(a) < 1.

725
Thus we see that the integral defining I(a) converges in the strip, 1 < <(a) < 1. The integral converges uniformly
in any closed subset of this domain. Uniform convergence means that we can differentiate the integral with respect to
a and interchange the order of integration and differentiation.


xa log x
Z
0
I (a) = dx
0 (x + 1)2

Thus we see that I(a) is analytic for 1 < <(a) < 1.


For 1 < <(a) < 1 and a 6= 0, z a is multi-valued. Consider the branch of the function f (z) = z a /(z + 1)2 with a
branch cut on the positive real axis and 0 < arg(z) < 2. We integrate along the contour in Figure ??.
The integral on C vanishes as  0. We show this with the maximum modulus integral bound. First we write z a
in modulus-argument form, z =  e , where a = + .

z a = ea log z
= e(+)(ln +)
= e ln +( ln +)
=  e e( log +)

Now we bound the integral.

za za
Z

dz 2 max


C (z + 1)2 zC (z + 1)2

 e2||
2
(1 )2
0 as  0

726
The integral on CR vanishes as R .

za za
Z


2
dz 2R max

2

CR (z + 1) zC R (z + 1)
R e2||
2R
(R 1)2
0 as R

Above the branch cut, (z = r e0 ), the integrand is


ra
f (r e0 ) = .
(r + 1)2

Below the branch cut, (z = r e2 ), we have,

e2a ra
f (r e2 ) = .
(r + 1)2

Now we use the residue theorem.


Z Z 0 2a a
ra za
 
e r
dr + dr = 2 Res , 1
0 (r + 1)2 (r + 1)
2 (z + 1)2
 ra
Z
2a d a
1e 2
dr = 2 lim (z )
0 (r + 1) z1 dz
Z
ra a e(a1)
dr = 2
(r + 1)2 1 e2a
Z0 a
r 2a
2
dr = a
0 (r + 1) e ea
Z
xa a
2
dx = for 1 < <(a) < 1, a 6= 0
0 (x + 1) sin(a)

727
The right side has a removable singularity at a = 0. We use analytic continuation to extend the answer to a = 0.
Z (
a
xa sin(a)
for 1 < <(a) < 1, a 6= 0
I(a) = 2
dx =
0 (x + 1) 1 for a = 0
We can derive the last two integrals by differentiating this formula with respect to a and taking the limit a 0.
Z a Z a 2
0 x log x 00 x log x
I (a) = 2
dx, I (a) = dx
0 (x + 1) 0 (x + 1)2
Z Z
0 log x 00 log2 x
I (0) = dx, I (0) = dx
0 (x + 1)2 0 (x + 1)2
We can find I 0 (0) and I 00 (0) either by differentiating the expression for I(a) or by finding the first few terms in the
Taylor series expansion of I(a) about a = 0. The latter approach is a little easier.

X I (n) (0) n
I(a) = a
n=0
n!
a
I(a) =
sin(a)
a
=
a (a)3 /6 + O(a5 )
1
=
1 (a) /6 + O(a4 )
2

2 a2
=1+ + O(a4 )
6
Z
log x
I 0 (0) = dx = 0
0 (x + 1)2
Z
00 log2 x 2
I (0) = dx =
0 (x + 1)2 3

728
Solution 13.27
1. We consider the integral

xa
Z
I(a) = dx.
0 1 + x2
To examine convergence, we split the domain of integration.
Z Z 1 Z
xa xa xa
dx = dx + dx
0 1 + x2 0 1+x
2
1 1 + x2

First we work with the integral on (0 . . . 1).


Z 1 Z 1 a
xa

x
dx 1 + x2 |dx|

1 + x 2
0 0
Z 1 <(a)
x
= 2
dx
0 1+x
Z 1
x<(a) dx
0

This integral converges for <(a) > 1.


Next we work with the integral on (1 . . . ).
Z a
Z a
x

x
dx 1 + x2 |dx|


1 1 + x2
1
Z <(a)
x
= dx
1 1 + x2
Z
x<(a)2 dx
1

This integral converges for <(a) < 1.

729
CR

Figure 13.11:

Thus we see that the integral defining I(a) converges in the strip, 1 < <(a) < 1. The integral converges
uniformly in any closed subset of this domain. Uniform convergence means that we can differentiate the integral
with respect to a and interchange the order of integration and differentiation.


xa log x
Z
0
I (a) = dx
0 1 + x2

Thus we see that I(a) is analytic for 1 < <(a) < 1.

2. For 1 < <(a) < 1 and a 6= 0, z a is multi-valued. Consider the branch of the function f (z) = z a /(1 + z 2 ) with
a branch cut on the positive real axis and 0 < arg(z) < 2. We integrate along the contour in Figure 13.11.

The integral on C vanishes are 0. We show this with the maximum modulus integral bound. First we write

730
z a in modulus-argument form, where z = e and a = + .

z a = ea log z
= e(+)(log +)
= e log +( log +)
= a e e( log +)

Now we bound the integral.


Z
a
a
z z
dz 2 max

C 1 + z 2 zC 1 + z 2

e2||
2
1 2
0 as 0

The integral on CR vanishes as R .


a
Z a
z 2R max z


2
dz

CR 1 + z
zCR 1 + z 2

R e2||
2R 2
R 1
0 as R

Above the branch cut, (z = r e0 ), the integrand is

ra
f (r e0 ) = .
1 + r2

731
Below the branch cut, (z = r e2 ), we have,

e2a ra
f (r e2 ) = .
1 + r2

Now we use the residue theorem.

0
ra e2a ra
Z Z   a   a 
z z
dr + dr = 2 Res , + Res ,
0 1 + r2 1+r
2 1 + z2 1 + z2
 xa za za
Z  
2a
1e dx = 2 lim + lim
0 1 + x2 z z + z z
Z a
 a/2
ea3/2

2a
 x e
1e dx = 2 +
0 1 + x2 2 2
Z
xa ea/2 ea3/2
dx =
0 1 + x2 1 e2a
Z
xa ea/2 (1 ea )
dx =
0 1 + x2 (1 + ea )(1 ea )
Z
xa
2
dx = a/2
0 1+x e + ea/2
Z
xa
2
dx = for 1 < <(a) < 1, a 6= 0
0 1+x 2 cos(a/2)

We use analytic continuation to extend the answer to a = 0.


xa
Z

I(a) = dx = for 1 < <(a) < 1
0 1 + x2 2 cos(a/2)

732
3. We can derive the last two integrals by differentiating this formula with respect to a and taking the limit a 0.

xa log x xa log2 x
Z Z
0 00
I (a) = dx, I (a) = dx
0 1 + x2 0 1 + x2
Z Z
0 log x 00 log2 x
I (0) = dx, I (0) = dx
0 1 + x2 0 1 + x2

We can find I 0 (0) and I 00 (0) either by differentiating the expression for I(a) or by finding the first few terms in
the Taylor series expansion of I(a) about a = 0. The latter approach is a little easier.

X I (n) (0)
I(a) = an
n=0
n!


I(a) =
2 cos(a/2)
1
=
2 1 (a/2) /2 + O(a4 )
2

1 + (a/2)2 /2 + O(a4 )

=
2
3 /8 2
= + a + O(a4 )
2 2

Z
0 log x
I (0) = dx = 0
0 1 + x2

log2 x 3
Z
I 00 (0) = dx =
0 1 + x2 8

733
Solution 13.28
Convergence. If xa f (x)  x as x 0 for some > 1 then the integral
Z 1
xa f (x) dx
0

will converge absolutely. If xa f (x)  x as x for some < 1 then the integral
Z
xa f (x)
1

will converge absolutely. These are sufficient conditions for the absolute convergence of
Z
xa f (x) dx.
0

Contour Integration. We put a branch cut on the positive real axis and choose 0 < arg(z) < 2. We consider
the integral of z a f (z) on the contour in Figure ??. Let the singularities of f (z) occur at z1 , . . . , zn . By the residue
theorem, Z n
X
z a f (z) dz = 2 Res (z a f (z), zk ) .
C k=1
1
On the circle of radius , the integrand is o( ). Since the length of C is 2, the integral on C vanishes as
 0. On the circle of radius R, the integrand is o(R1 ). Since the length of CR is 2R, the integral on CR vanishes
as R .
The value of the integrand below the branch cut, z = x e2 , is

f (x e2 ) = xa e2a f (x)

In the limit as  0 and R we have


Z Z 0 n
X
a a 2a
x f (x) dx + x e f (x) dx = 2 Res (z a f (z), zk ) .
0 k=1

734
Z n
a 2 X
x f (x) dx = Res (z a f (z), zk ) .
0 1 e2a k=1

Solution 13.29
In the interval of uniform convergence of th integral, we can differentiate the formula
Z n
2 X
xa f (x) dx = 2a
Res (z a f (z), zk ) ,
0 1 e
k=1

with respect to a to obtain,


Z n n
2 X 4 2 a e2a X
xa f (x) log x dx = a
Res (z f (z) log z, zk ) , Res (z a f (z), zk ) .
0 1 e2a k=1
(1 e2a )2 k=1

n n
2a X
Z
a 2 X a
x f (x) log x dx = Res (z f (z) log z, zk ) , + 2 Res (z a f (z), zk ) ,
0 1 e2a k=1
sin (a) k=1

Differentiating the solution of Exercise 13.26 m times with respect to a yields


n
!

m
Z
2 X
xa f (x) logm x dx = m Res (z a f (z), zk ) ,
0 a 1 e2a k=1

Solution 13.30
Taking the limit as a 0 Z in the solution of Exercise 13.26 yields
Z  Pn a

k=1 Res (z f (z), zk )
f (x) dx = 2 lim
0 a0 1 e2a
The numerator vanishes because the sum of all residues of z n f (z) is zero. Thus we can use LHospitals rule.
Z  Pn a

k=1 Res (z f (z) log z, zk )
f (x) dx = 2 lim
0 a0 2 e2a

735
Z n
X
f (x) dx = Res (f (z) log z, zk )
0 k=1

This suggests that we could have derived the result directly by considering the integral of f (z) log z on the contour
in Figure ??. We put a branch cut on the positive real axis and choose the branch arg z = 0. Recall that we have
assumed that f (z) has only isolated singularities and no singularities on the positive real axis, [0, ). By the residue
theorem,
Z Xn
f (z) log z dz = 2 Res (f (z) log z, z = zk ) .
C k=1

By assuming that f (z)  z as z 0 where > 1 the integral on C will vanish as  0. By assuming that
f (z)  z as z where < 1 the integral on CR will vanish as R . The value of the integrand below the
branch cut, z = x e2 is f (x)(log x + 2). Taking the limit as  0 and R , we have
Z Z 0 n
X
f (x) log x dx + f (x)(log x + 2) dx = 2 Res (f (z) log z, zk ) .
0 k=1

Thus we corroborate the result. n


Z X
f (x) dx = Res (f (z) log z, zk )
0 k=1

Solution 13.31
Consider the integral of f (z) log2 z on the contour in Figure ??. We put a branch cut on the positive real axis and
choose the branch 0 < arg z < 2. Let z1 , . . . zn be the singularities of f (z). By the residue theorem,
Z n
X
2
Res f (z) log2 z, zk .

f (z) log z dz = 2
C k=1

If f (z)  z as z 0 for some > 1 then the integral on C will vanish as  0. f (z)  z as z for some
< 1 then the integral on CR will vanish as R . Below the branch cut the integrand is f (x)(log x + 2)2 .

736
Thus we have
Z Z 0 n
X
2 2 2
Res f (z) log2 z, zk .

f (x) log x dx + f (x)(log x + 4 log x 4 ) dx = 2
0 k=1
Z Z n
X
2
Res f (z) log2 z, zk .

4 f (x) log x dx + 4 f (x) dx = 2
0 0 k=1
Z n n
1 X
2
 X
f (x) log x dx = Res f (z) log z, zk + Res (f (z) log z, zk )
0 2 k=1 k=1

Solution 13.32
Convergence. We consider

xa
Z
dx.
0 1 + x4
Since the integrand behaves like x near x = 0 we must have <(a) > 1. Since the integrand behaves like xa4 at
a

infinity we must have <(a 4) < 1. The integral converges for 1 < <(a) < 3.
Contour Integration. The function
za
f (z) =
1 + z4

has first order poles at z = (1 )/ 2 and a branch point at z = 0. We could evaluate the real integral by putting
a branch cut on the positive real axis with 0 < arg(z) < 2 and integrating f (z) on the contour in Figure 13.12.
Integrating on this contour would work because the value of the integrand below the branch cut is a constant times
the value of the integrand above the branch cut. After demonstrating that the integrals along C and CR vanish in the
limits as  0 and R we would see that the value of the integral is a constant times the sum of the residues at
the four poles. However, this is not the only, (and not the best), contour that can be used to evaluate the real integral.
Consider the value of the integral on the line arg(z) = .
ra ea
f (r e ) =
1 + r4 e4

737
CR

za
Figure 13.12: Possible path of integration for f (z) = 1+z 4

If is a integer multiple of /2 then the integrand is a constant multiple of

ra
f (x) = .
1 + r4

Thus any of the contours in Figure 13.13 can be used to evaluate the real integral. The only difference is how many
residues we have to calculate. Thus we choose the first contour in Figure 13.13. We put a branch cut on the negative
real axis and choose the branch < arg(z) < to satisfy f (1) = 1.
We evaluate the integral along C with the Residue Theorem.

za za
Z  
1+
dz = 2 Res , z =
C 1 + z4 1 + z4 2

Let a = + and z = r e . Note that

|z a | = |(r e )+ | = r e .

738
CR CR CR

C C C

za
Figure 13.13: Possible Paths of Integration for f (z) = 1+z 4

The integral on C vanishes as  0. We demonstrate this with the maximum modulus integral bound.
za
Z a
 z

4
dz
max
1+z
C 2 zC 1 + z 4
  e||/2

2 1 4
0 as  0
The integral on CR vanishes as R .
za
Z a
R z
dz max

CR 1 + z4 2 zCR 1 + z 4
R R e||/2

2 R4 1
0 as R

739
The value of the integrand on the positive imaginary axis, z = x e/2 , is
(x e/2 )a xa ea/2
= .
1 + (x e/2 )4 1 + x4
We take the limit as  0 and R .
Z Z 0 a a/2
xa
 a 
x e /2 z /4
dx + e dx = 2 Res ,e
0 1 + x4 1+x
4 1 + z4
 xa
 a
z (z e/2 )
Z 
(a+1)/2
1e dx = 2 lim
0 1 + x4 ze/4 1 + z4
Z
xa az a (z e/2 ) + z a
 
2
dx = lim
0 1 + x4 1 e(a+1)/2 ze/4 4z 3
Z
xa 2 ea/4
dx =
1 + x4 1 e(a+1)/2 4 e3/4
Z 0
xa
4
dx = (a+1)/4
0 1+x 2(e e(a+1)/4 )
Z
xa
 
(a + 1)
dx = csc
0 1 + x4 4 4
Solution 13.33
Consider the branch of f (z) = z 1/2 log z/(z + 1)2 with a branch cut on the positive real axis and 0 < arg z < 2. We
integrate this function on the contour in Figure ??.
We use the maximum modulus integral bound to show that the integral on C vanishes as 0.
Z
1/2
1/2
z log z z log z
dz 2 max

C (z + 1)2 (z + 1)2

C

1/2 (2 log )
= 2
(1 )2
0 as 0

740
The integral on CR vanishes as R .
1/2
Z 1/2
z log z z log z

2
dz 2R max

CR (z + 1) CR (z + 1)2
R1/2 (log R + 2)
= 2R
(R 1)2
0 as R

Above the branch cut, (z = x e0 ), the integrand is,

x1/2 log x
f (x e0 ) = .
(x + 1)2

Below the branch cut, (z = x e2 ), we have,

x1/2 (log x + )
f (x e2 ) = .
(x + 1)2

Taking the limit as 0 and R , the residue theorem gives us


0
x1/2 log x x1/2 (log x + 2) z 1/2 log z
Z Z  
dx + dx = 2 Res , 1 .
0 (x + 1)2 (x + 1)2 (z + 1)2
Z
x1/2 log x x1/2
Z
d 1/2
2 dx + 2 dx = 2 lim (z log z)
0 (x + 1)2 0 (x + 1)2 z1 dz
Z 1/2 Z
x1/2
 
x log x 1 1/2 1/2 1
2 dx + 2 dx = 2 lim z log z + z
0 (x + 1)2 0 (x + 1)2 z1 2 z
Z 1/2 Z
x1/2
 
x log x 1
2 2
dx + 2 2
dx = 2 ()()
0 (x + 1) 0 (x + 1) 2

741

x1/2 log x x1/2
Z Z
2 dx + 2 dx = 2 + 2
0 (x + 1)2 0 (x + 1) 2

Equating real and imaginary parts,



x1/2 log x x1/2
Z Z

dx = , dx = .
0 (x + 1)2 0 (x + 1)2 2

Exploiting Symmetry
Solution 13.34
Convergence. The integrand,
eaz eaz
= ,
ez ez 2 sinh(z)
has first order poles at z = n, n Z. To study convergence, we split the domain of integration.
Z Z 1 Z 1 Z
= + +
1 1

The principal value integral Z 1


eax
dx
1 ex ex
exists for any a because the integrand has only a first order pole on the path of integration.
Now consider the integral on (1 . . . ).
Z Z (a1)x
eax e

x
dx = dx

1
x
e e 1 1 e2x
Z
1
e(a1)x dx
1 e2 1
This integral converges for a 1 < 0; a < 1.

742
Finally consider the integral on ( . . . 1).

1
Z 1 (a+1)x
eax
Z
e
dx = dx

ex ex 1 e
2x
Z 1
1
e(a+1)x dx
1 e2

This integral converges for a + 1 > 0; a > 1.


Thus we see that the integral for I(a) converges for real a, |a| < 1.
Choice of Contour. Consider the contour C that is the boundary of the region: R < x < R, 0 < y < . The
integrand has no singularities inside the contour. There are first order poles on the contour at z = 0 and z = . The
value of the integral along the contour is times the sum of these two residues.
The integrals along the vertical sides of the contour vanish as R .

R+
eaz eaz
Z

dz max z

R ez ez z(R...R+) e ez

eaR
R
e eR
0 as R

R+
eaz eaz
Z

dz max

R ez ez z(R...R+) ez ez

eaR
R
e eR
0 as R

743
Evaluating the Integral. We take the limit as R and apply the residue theorem.
Z Z +
eax eaz
x
dx + dz
x
e e + ez ez
eaz eaz
   
= Res z , z = 0 + Res z , z =
e ez e ez


eax ea(x+ z eaz (z ) eaz
Z Z
x
dx + dz = lim + lim
x
e e ex+ ex z0 2 sinh(z) z 2 sinh(z)
Z ax az az
a e e +az e e +a(z ) eaz
az
(1 + e ) dx = lim + lim
ex ex z0 2 cosh(z) z 2 cosh(z)
Z ax a
e 1 e
(1 + ea ) x x
dx = +
e e 2 2
Z ax a
e (1 e )
x x
dx =
e e 2(1 + ea )
Z
eax (ea/2 ea/2 )
x x
dx =
e e 2 ea/2 + ea/2
Z
eax  a 
x x
dx = tan
e e 2 2
Solution 13.35
1.

1
Z Z
dx dx
2 =
0 (1 + x2 ) 2 (1 + x2 )2
We apply Result 13.4.1 to the integral on the real axis. First we verify that the integrand vanishes fast enough
in the upper half plane.
   
1 1
lim R max = lim R =0
R zCR (1 + z 2 )2 R (R2 1)2

744
Then we evaluate the integral with the residue theorem.

Z  
dx 1
= 2 Res ,z =
(1 + x2 )2 (1 + z 2 )2
 
1
= 2 Res ,z =
(z )2 (z + )2
d 1
= 2 lim
z dz (z + )2
2
= 2 lim
z (z + )3

=
2

Z
dx
2 =
0
2
(1 + x ) 4

2. We wish to evaluate
Z
dx
.
0 x3 + 1

Let the contour C be the boundary of the region 0 < r < R, 0 < < 2/3. We factor the denominator of the
integrand to see that the contour encloses the simple pole at e/3 for R > 1.

z 3 + 1 = (z e/3 )(z + 1)(z e/3 )

745
We calculate the residue at that point.
   
1 /3 /3 1
Res ,z = e = lim (z e ) 3
z3 + 1 ze/3 z +1
 
1
= lim
ze/3 (z + 1)(z e/3 )
1
= /3
(e +1)(e/3 e/3 )
e/3
=
3
We use the residue theorem to evaluate the integral.
2 e/3
I
dz
3
=
C z +1 3
Let CR be the circular arc portion of the contour.
Z R Z R 2/3
e
Z Z
dz dx dz dx
3
= 3
+ 3
3
C z +1 0 x +1 CR z + 1 0 x +1
Z R Z
dx dz
= (1 + e/3 ) 3
+ 3
0 x +1 CR z + 1

We show that the integral along CR vanishes as R with the maximum modulus integral bound.
Z
dz 2R 1


3 + 1
0 as R

CR z 3 R3 1
We take R and solve for the desired integral.
 dx 2 e/3
Z
/3
1+e =
x3 + 1 3
Z 0
dx 2
3
=
0 x +1 3 3

746
Figure 13.14: The semi-circle contour.

Solution 13.36
Method 1: Semi-Circle Contour. We wish to evaluate the integral
Z
dx
I= .
0 1 + x6

We note that the integrand is an even function and express I as an integral over the whole real axis.
Z
1 dx
I=
2 1 + x6

Now we will evaluate the integral using contour integration. We close the path of integration in the upper half plane.
Let R be the semicircular arc from R to R in the upper half plane. Let be the union of R and the interval
[R, R]. (See Figure 13.14.)
We can evaluate the integral along with the residue theorem. The integrand has first order poles at z = e(1+2k)/6 ,

747
k = 0, 1, 2, 3, 4, 5. Three of these poles are in the upper half plane. For R > 1, we have

Z 2  
1 X 1 (1+2k)/6
dz = 2 Res ,e
z6 + 1 k=0
z 6+1

2
X z e(1+2k)/6
= 2 lim
k=0
ze(1+2k)/6 z6 + 1

Since the numerator and denominator vanish, we apply LHospitals rule.

2
X 1
= 2 lim
k=0
ze(1+2k)/6 6z 5
2
X
= e5(1+2k)/6
3 k=0
5/6
+ e15/6 + e25/6

= e
3
5/6
+ e/2 + e/6

= e
3
!
3 3
= +
3 2 2
2
=
3

Now we examine the integral along R . We use the maximum modulus integral bound to show that the value of the

748
integral vanishes as R .
Z
1 1
dz R max 6

R z6 + 1 zR z + 1

1
= R 6
R 1
0 as R .

Now we are prepared to evaluate the original real integral.


Z
1 2
6
dz =
z +1 3
Z R Z
1 1 2
6
dx + 6
dz =
R x +1 R z + 1 3

We take the limit as R .


Z
1 2
dx =
x6 +1 3
Z

1
6
dx =
0 x +1 3

We would get the same result by closing the path of integration in the lower half plane. Note that in this case the
closed contour would be in the negative direction.
Method 2: Wedge Contour. Consider the contour , which starts at the origin, goes to the point R along the
real axis, then to the point R e/3 along a circle of radius R and then back to the origin along the ray = /3. (See
Figure 13.15.)
We can evaluate the integral along with the residue theorem. The integrand has one first order pole inside the

749
Figure 13.15: The wedge contour.

contour at z = e/6 . For R > 1, we have


Z  
1 1
6
dz = 2 Res 6
, e/6
z +1 z +1
z e/6
= 2 lim 6
ze/6 z + 1

Since the numerator and denominator vanish, we apply LHospitals rule.

1
= 2 lim
ze/6 6z 5
5/6
= e
3

= e/3
3
Now we examine the integral along the circular arc, R . We use the maximum modulus integral bound to show that

750
the value of the integral vanishes as R .
Z
1 R 1

6
dz max

R z + 1 3 zR z 6 + 1
R 1
=
3 R6 1
0 as R .
Now we are prepared to evaluate the original real integral.
Z
1 /3
6
dz = e
z +1 3
Z R Z Z 0
1 1 1
6
dx + 6
dz + 6
dz = e/3
0 x +1 R z + 1 R e/3 z + 1 3
Z R Z Z 0
1 1 1 /3 /3
6
dx + 6
dz + 6
e dx = e
0 x +1 R z + 1 R x +1 3
We take the limit as R .
Z
1
/3
dx = e/3

1e
0 +1 x6 3
Z /3
1 e
6
dx =
0 x +1 3 1 e/3


(1 3)/2
Z
1
dx =
0 x6 + 1 3 1 (1 + 3)/2
Z
1
6
dx =
0 x +1 3
Solution 13.37
First note that
4
cos(2) 1 , 0 .
4

751
Figure 13.16: cos(2) and 1 4

These two functions are plotted in Figure 13.16. To prove this inequality analytically, note that the two functions are
equal at the endpoints of the interval and that cos(2) is concave downward on the interval,

d2
cos(2) = 4 cos(2) 0 for 0 ,
d2 4
while 1 4/ is linear.
Let CR be the quarter circle of radius R from = 0 to = /4. The integral along this contour vanishes as
R .
Z Z /4
z 2 (R e )2


e dz
e R e d
CR 0
Z /4
2
R eR cos(2)
d
0
Z /4
2 (14/)
R eR d
0
h R2 (14/) i/4
= R 2e
4R 0
 R2

= 1e
4R
0 as R

752
Let C be the boundary of the domain 0 < r < R, 0 < < /4. Since the integrand is analytic inside C the integral
along C is zero. Taking the limit as R , the integral from r = 0 to along = 0 is equal to the integral from
r = 0 to along = /4.
Z Z 2
x2 1+
x 1+
e dx = e 2 dx
0 0 2
Z Z
x2 1+ 2
e dx = ex dx
0 2 0

Z Z
x2 1+
cos(x2 ) sin(x2 ) dx

e dx =
0 2 0

Z Z Z  Z Z 
x2 1 2 2 2 2
e dx = cos(x ) dx + sin(x ) dx + cos(x ) dx sin(x ) dx
0 2 0 0 2 0 0

We equate the imaginary part of this equation to see that the integrals of cos(x2 ) and sin(x2 ) are equal.
Z Z
2
cos(x ) dx = sin(x2 ) dx
0 0

The real part of the equation then gives us the desired identity.

Z Z Z
1 2
cos(x ) dx = 2 2
sin(x ) dx = ex dx
0 0 2 0

Solution 13.38
Consider the box contour C that is the boundary of the rectangle R x R, 0 y . There is a removable

753
singularity at z = 0 and a first order pole at z = . By the residue theorem,
Z
z  z 
dz = Res ,
C sinh z sinh z
z(z )
= lim
z sinh z
2z
= lim
z cosh z
2
=
The integrals along the side of the box vanish as R .
Z R+
z z
dz max

sinh z z[R,R+] sinh z

R
R+

sinh R
0 as R
The value of the integrand on the top of the box is
x + x +
= .
sinh(x + ) sinh x
Taking the limit as R , Z Z
x x +
dx + dx = 2 .
sinh x sinh x
Note that Z
1
dx = 0
sinh x
as there is a first order pole at x = 0 and the integrand is odd.
Z
x 2
dx =
sinh x 2

754
Solution 13.39
First we evaluate

eax
Z
dx.
ex +1
Consider the rectangular contour in the positive direction with corners at R and R + 2. With the maximum
modulus integral bound we see that the integrals on the vertical sides of the contour vanish as R .
Z R+2 az aR
e 2 e

dz 0 as R

R ez +1 eR 1
Z R az
aR
e 2 e

dz 0 as R
z
R+2 e +1
1 eR

In the limit as R tends to infinity, the integral on the rectangular contour is the sum of the integrals along the top and
bottom sides.
Z ax Z a(x+2)
eaz e e
Z
z
dz = x
dx + dx
C e +1 e +1 ex+2 +1
Z ax
eaz e
Z
2a
z
dz = (1 e ) x
dx
C e +1 e +1

The only singularity of the integrand inside the contour is a first order pole at z = . We use the residue theorem to
evaluate the integral.
eaz
 az 
e
Z
z
dz = 2 Res z ,
C e +1 e +1
(z ) eaz
= 2 lim
z ez +1
a(z ) eaz + eaz
= 2 lim
z ez
= 2 ea

755
We equate the two results for the value of the contour integral.
Z ax
2a e
(1 e ) x
dx = 2 ea
e +1
Z ax
e 2
dx = a
x
e +1 e ea
Z ax
e
x
dx =
e +1 sin(a)
Now we derive the value of, Z
cosh(bx)
dx.
cosh x
First make the change of variables x 2x in the previous result.
Z 2ax
e
2x
2 dx =
e +1 sin(a)
Z (2a1)x
e
x x
dx =
e + e sin(a)
Now we set b = 2a 1.

ebx
Z

dx = = for 1 < b < 1
cosh x sin((b + 1)/2) cos(b/2)
Since the cosine is an even function, we also have,
Z bx
e
dx = for 1 < b < 1
cosh x cos(b/2)
Adding these two equations and dividing by 2 yields the desired result.
Z
cosh(bx)
dx = for 1 < b < 1
cosh x cos(b/2)

756
Solution 13.40
Real-Valued Parameters. For b = 0, the integral has the value: /a2 . If b is nonzero, then we can write the integral
as
1
Z
d
F (a, b) = 2 .
b 0 (a/b + cos )2

We define the new parameter c = a/b and the function,


Z
2 d
G(c) = b F (a, b) = .
0 (c + cos )2

If 1 c 1 then the integrand has a double pole on the path of integration. The integral diverges. Otherwise
the integral exists. To evaluate the integral, we extend the range of integration to (0..2) and make the change of
variables, z = e to integrate along the unit circle in the complex plane.
Z 2
1 d
G(c) =
2 0 (c + cos )2

For this change of variables, we have,


z + z 1 dz
cos = , d = .
2 z

Z
1 dz/(z)
G(c) = 1 2
2 C (c + (z + z )/2)
Z
z
= 2 2 2
dz
C (2cz + z + 1)
Z
z
= 2 dz
C (z + c + c 1) (z + c c2 1)2
2 2

757

If c > 1, then c c2 1 is outside the unit circle and c + c2 1 is inside the unit circle. The integrand has
a second order pole inside the path of integration. We evaluate the integral with the residue theorem.


 
z
G(c) = 22 Res , z = c + c2 1
(z + c + c2 1)2 (z + c c2 1)2
d z
= 4 lim

zc+ c2 1 dz (z + c + c2 1)2
 
1 2z
= 4 lim

zc+ c2 1 (z + c + c2 1)2 (z + c + c2 1)3

c + c2 1 z
= 4 lim

zc+ c2 1 (z + c + c2 1)3
2c
= 4
(2 c2 1)3
c
=p
(c 1)3
2

758

If c < 1, then c c2 1 is inside the unit circle and c + c2 1 is outside the unit circle.

 
z
G(c) = 22 Res 2
, z = c c 1
(z + c + c2 1)2 (z + c c2 1)2
d z
= 4 lim

zc c2 1 dz (z + c c2 1)2
 
1 2z
= 4 lim

zc c2 1 (z + c c2 1)2 (z + c c2 1)3

c c2 1 z
= 4 lim

zc c2 1 (z + c c2 1)3
2c
= 4
(2 c2 1)3
c
= p
(c 1)3
2

Thus we see that


= c for c > 1,
(c2 1)3



G(c) = c for c < 1,
(c2 1)3

for 1 c 1.

is divergent
In terms of F (a, b), this is

= a for a/b > 1,
(a2 b2 )3



F (a, b) = a for a/b < 1,
(a2 b2 )3

for 1 a/b 1.

is divergent
Complex-Valued Parameters. Consider
Z
d
G(c) = ,
0 (c + cos )2

759
for complex c. Except for real-valued c between 1 and 1, the integral converges uniformly. We can interchange
differentiation and integration. The derivative of G(c) is
d
Z
0 d
G (c) =
dc (c + cos )2
Z 0
2
= 3
d
0 (c + cos )

Thus we see that G(c) is analytic in the complex plane with a cut on the real axis from 1 to 1. The value of the
function on the positive real axis for c > 1 is
c
G(c) = p .
(c2 1)3
We use analytic continuation to determine G(c) for complex c. By inspection we see that G(c) is the branch of
c
,
(c2 1)3/2
with a branch cut on the real axis from 1 to 1 and which is real-valued and positive for real c > 1. Using F (a, b) =
G(c)/b2 we can determine F for complex-valued a and b.

Solution 13.41
First note that

ex
Z Z
cos x
dx = dx
ex + ex ex + ex
x
since sin x/(e + ex ) is an odd function. For the function
ez
f (z) =
ez + ez
we have
ex ex
f (x + ) = = e = e f (x).
ex+ + ex ex + ex

760
Thus we consider the integral

ez
Z
dz
C ez + ez

where C is the box contour with corners at R and R + . We can evaluate this integral with the residue theorem.
We can write the integrand as

ez
.
2 cosh z

We see that the integrand has first order poles at z = (n + 1/2). The only pole inside the path of integration is at
z = /2.

ez ez
Z  

dz = 2 Res ,z =
C ez + ez ez + ez 2
z
(z /2) e
= 2 lim
z/2 ez + ez
e +(z /2) ez
z
= 2 lim
z/2 ez ez
e/2
= 2 /2
e e/2
= e/2

761
The integrals along the vertical sides of the box vanish as R .
R+
ez ez
Z

dz
max

R ez + ez z[R...R+] ez + ez

1
max R+y

Ry

y[0...] e +e

1
max R
y[0...] e + eR2y
1
=
2 sinh R
0 as R

Taking the limit as R , we have


Z +
ex ez
Z
dx + dz = e/2
ex + ex e z + ez
Z +x
e
(1 + e ) x x
dx = e/2
e + e
Z
ex
x
dx = /2
x
e + e e + e/2

Finally we have,
Z
cos x
x
dx = /2 .
ex+e e + e/2

Definite Integrals Involving Sine and Cosine


Solution 13.42
1. To evaluate the integral we make the change of variables z = e . The path of integration in the complex plane

762
is the positively oriented unit circle.
Z Z
d 1 dz
2 = 2
1 + sin 1 (z z ) /4 z
1
ZC
4z
= 4 2
dz
C z 6z + 1
Z
4z
=     dz
C z 1 2 z1+ 2 z+1 2 z+1+ 2

There are first order poles at z = 1 2. The poles at z = 1 + 2 and z = 1 2 are inside the path of
integration. We evaluate the integral with Cauchys Residue Formula.

Z   
4z 4z
4 2
dz = 2 Res , z = 1 + 2
C z 6z + 1 z 4 6z 2 + 1

 
4z
+ Res ,z = 1 2
z 4 6z 2 + 1

z
= 8   

z 1 2 z 1 + 2 z + 1 + 2 z=1+2
!
z
+   

z 1 2 z + 1 2 z + 1 + 2 z=12
 
1 1
= 8
8 2 8 2

= 2

2. First we use symmetry to expand the domain of integration.


Z /2
1 2 4
Z
4
sin d = sin d
0 4 0

763
Next we make the change of variables z = e . The path of integration in the complex plane is the positively
oriented unit circle. We evaluate the integral with the residue theorem.

Z 2 Z  4
1 4 1 1 1 dz
sin d = z
4 0 4 C 16 z z
(z 2 1)4
Z
1
= dz
64 C z5
 

Z
3 6 4 1
= z 4z + 3 + 5 dz
64 C z z z

= 2 6
64
3
=
16

Solution 13.43
1. Let C be the positively oriented unit circle about the origin. We parametrize this contour.

z = e , dz = e d, (0 . . . 2)

764
We write sin and the differential d in terms of z. Then we evaluate the integral with the Residue theorem.
Z 2 I
1 1 dz
d =
0 2 + sin 2 + (z 1/z)/(2) z
IC
2
= 2
dz
z + 4z 1
IC
2
=   dz
C z+ 2+ 3 z+ 2 3
       
= 2 Res z + 2 + 3 z + 2 3 , z = 2 + 3
2
= 2
2 3
2
=
3

2. First consider the case a = 0. (



for n Z+
Z
0
cos(n) d =
2 for n = 0

Now we consider |a| < 1, a 6= 0. Since


sin(n)
1 2a cos + a2
is an even function,

en
Z Z
cos(n)
d = d
1 2a cos + a2 1 2a cos + a2
Let C be the positively oriented unit circle about the origin. We parametrize this contour.

z = e , dz = e d, ( . . . )

765
We write the integrand and the differential d in terms of z. Then we evaluate the integral with the Residue
theorem.
Z
en zn
I
dz
2
d = 2
1 2a cos + a C 1 a(z + 1/z) + a z
zn
I
= 2 2
dz
C az + (1 + a )z a
zn
I

= dz
a C z 2 (a + 1/a)z + 1
zn
I

= dz
a C (z a)(z 1/a)
zn
 

= 2 Res ,z = a
a (z a)(z 1/a)
2 an
=
a a 1/a
2an
=
1 a2
We write the value of the integral for |a| < 1 and n Z0+ .
Z (
cos(n) 2 for a = 0, n = 0
2
d = 2an
1 2a cos + a 1a2
otherwise

Solution 13.44
Convergence. We consider the integral
Z
cos(n) sin(n)
I() = d = .
0 cos cos sin
We assume that is real-valued. If is an integer, then the integrand has a second order pole on the path of integration,
the principal value of the integral does not exist. If is real, but not an integer, then the integrand has a first order
pole on the path of integration. The integral diverges, but its principal value exists.

766
Contour Integration. We will evaluate the integral for real, non-integer .
Z
cos(n)
I() = d
0 cos cos
1 2 cos(n)
Z
= d
2 0 cos cos
Z 2
1 en
= < d
2 0 cos cos
We make the change of variables: z = e .
zn
Z
1 dz
I() = <
2 (z + 1/z)/2 cos z
Z C
z n
=< )
dz
C (z e )(z e

Now we use the residue theorem.


zn
   

= < () Res ,z = e
(z e )(z e )
zn
  

+ Res ,z = e
(z e )(z e )
zn zn
 
= < lim + lim
ze z e ze z e

en en
 
= < +
e e e e
e en
 n 
= <
e e
 
sin(n)
= <
sin()

767
Z
cos(n) sin(n)
I() = d = .
0 cos cos sin

Solution 13.45
Consider the integral
1
x2
Z
dx.
2 2
0 (1 + x ) 1 x

We make the change of variables x = sin to obtain,


Z /2
sin2
p cos d
0 (1 + sin2 ) 1 sin2
Z /2
sin2
d
0 1 + sin2
Z /2
1 cos(2)
d
0 3 cos(2)
1 2 1 cos
Z
d
4 0 3 cos
Now we make the change of variables z = e to obtain a contour integral on the unit circle.
 
1 (z + 1/z)/2
Z
1
dz
4 C 3 (z + 1/z)/2 z
(z 1)2
Z
dz
4 C z(z 3 + 2 2)(z 3 2 2)
There are two first order poles inside the contour. The value of the integral is
(z 1)2 (z 1)2
    

2 Res , 0 + Res ,z = 3 2 2
4 z(z 3 + 2 2)(z 3 2 2) z(z 3 + 2 2)(z 3 2 2)

768
(z 1)2 (z 1)2
    

lim + lim .
2 z0 (z 3 + 2 2)(z 3 2 2) z32 2 z(z 3 2 2)

1

x2 (2 2)
Z
dx =
0 (1 + x2 ) 1 x2 4

Infinite Sums
Solution 13.46
From Result 13.10.1 we see that the sum of the residues of cot(z)/z 4 is zero. This function has simples poles at
nonzero integers z = n with residue 1/n4 . There is a fifth order pole at z = 0. Finding the residue with the formula

1 d4
lim 4 (z cot(z))
4! z0 dz

would be a real pain. After doing the differentiation, we would have to apply LHospitals rule multiple times. A better
way of finding the residue is with the Laurent series expansion of the function. Note that

1 1
=
sin(z) z (z)3 /6 + (z)5 /120
1 1
=
z 1 (z) /6 + (z)4 /120
2
 2 2 !
4 4
  2
1 2 2 4 4
= 1+ z z + + z z + + .
z 6 120 6 120

769
Now we find the z 1 term in the Laurent series expansion of cot(z)/z 4 .
2 !
2 2 4 4
 2
4 4
  2
4 4
 
cos(z) 1 2 2
= 4 1 z + z 1+ z z + + z z + +
z 4 sin(z) z 2 24 z 6 120 6 120
4 4 4 4
   
1 4
= 5 + + + z +
z 120 36 12 24
4 1
= +
45 z
Thus the residue at z = 0 is 4 /45. Summing the residues,
1
X 1 4 X 1
+ = 0.
n=
n4 45 n=1 n4


X 1 4
=
n=1
n4 90

Solution 13.47
For this problem we will use the following result: If
lim |zf (z)| = 0,
|z|

then the sum of all the residues of cot(z)f (z) is zero. If in addition, f (z) is analytic at z = n Z then

X
f (n) = ( sum of the residues of cot(z)f (z) at the poles of f (z) ).
n=

We assume that is not an integer, otherwise the sum is not defined. Consider f (z) = 1/(z 2 2 ). Since

1
lim z 2 = 0,
|z| z 2

770
and f (z) is analytic at z = n, n Z, we have

X 1
= ( sum of the residues of cot(z)f (z) at the poles of f (z) ).
n=
n2 2

f (z) has first order poles at z = .


   
X 1 cot(z) cot(z)
2 2
= Res , z = Res , z =
n=
n z 2 2 z 2 2
cot(z) cot(z)
= lim lim
z z+ z z
cot() cot()
=
2 2

X 1 cot()
=
n=
n2 2

771
Part IV

Ordinary Differential Equations

772
Chapter 14

First Order Differential Equations

Dont show me your technique. Show me your heart.

-Tetsuyasu Uekuma

14.1 Notation
A differential equation is an equation involving a function, its derivatives, and independent variables. If there is only
one independent variable, then it is an ordinary differential equation. Identities such as

d dy dx
f 2 (x) = 2f (x)f 0 (x),

and =1
dx dx dy

are not differential equations.


The order of a differential equation is the order of the highest derivative. The following equations for y(x) are
first, second and third order, respectively.

y 0 = xy 2

773
y 00 + 3xy 0 + 2y = x2

y 000 = y 00 y

The degree of a differential equation is the highest power of the highest derivative in the equation. The following
equations are first, second and third degree, respectively.

y 0 3y 2 = sin x

(y 00 )2 + 2x cos y = ex

(y 0 )3 + y 5 = 0

An equation is said to be linear if it is linear in the dependent variable.

y 00 cos x + x2 y = 0 is a linear differential equation.

y 0 + xy 2 = 0 is a nonlinear differential equation.

A differential equation is homogeneous if it has no terms that are functions of the independent variable alone. Thus
an inhomogeneous equation is one in which there are terms that are functions of the independent variables alone.

y 00 + xy + y = 0 is a homogeneous equation.

y 0 + y + x2 = 0 is an inhomogeneous equation.

A first order differential equation may be written in terms of differentials. Recall that for the function y(x) the
differential dy is defined dy = y 0 (x) dx. Thus the differential equations

y 0 = x2 y and y 0 + xy 2 = sin(x)

can be denoted:
dy = x2 y dx and dy + xy 2 dx = sin(x) dx.

774
A solution of a differential equation is a function which when substituted into the equation yields an identity. For
example, y = x ln |x| is a solution of
y
y 0 = 1.
x
We verify this by substituting it into the differential equation.

ln |x| + 1 ln |x| = 1

We can also verify that y = c ex is a solution of y 00 y = 0 for any value of the parameter c.

c ex c ex = 0

14.2 Example Problems


In this section we will discuss physical and geometrical problems that lead to first order differential equations.

14.2.1 Growth and Decay


Example 14.2.1 Consider a culture of bacteria in which each bacterium divides once per hour. Let n(t) N denote
the population, let t denote the time in hours and let n0 be the population at time t = 0. The population doubles every
hour. Thus for integer t, the population is n0 2t . Figure 14.1 shows two possible populations when there is initially a
single bacterium. In the first plot, each of the bacteria divide at times t = m for m N . In the second plot, they
divide at times t = m 1/2. For both plots the population is 2t for integer t.
We model this problem by considering a continuous population y(t) R which approximates the discrete population.
In Figure 14.2 we first show the population when there is initially 8 bacteria. The divisions of bacteria is spread out
over each one second interval. For integer t, the populations is 8 2t . Next we show the population with a plot of the
continuous function y(t) = 8 2t . We see that y(t) is a reasonable approximation of the discrete population.
In the discrete problem, the growth of the population is proportional to its number; the population doubles every
hour. For the continuous problem, we assume that this is true for y(t). We write this as an equation:

y 0 (t) = y(t).

775
16 16
12 12
8 8
4 4
1 2 3 4 1 2 3 4

Figure 14.1: The population of bacteria.


128 128
96 96
64 64
32 32
1 2 3 4 1 2 3 4

Figure 14.2: The discrete population of bacteria and a continuous population approximation.

That is, the rate of change y 0 (t) in the population is proportional to the population y(t), (with constant of proportionality
). We specify the population at time t = 0 with the initial condition: y(0) = n0 . Note that y(t) = n0 et satisfies the
problem:
y 0 (t) = y(t), y(0) = n0 .

For our bacteria example, = ln 2.

Result 14.2.1 A quantity y(t) whose growth or decay is proportional to y(t) is modelled by
the problem:
y 0 (t) = y(t), y(t0 ) = y0 .
Here we assume that the quantity is known at time t = t0 . e is the factor by which the
quantity grows/decays in unit time. The solution of this problem is y(t) = y0 e(tt0 ) .

776
14.3 One Parameter Families of Functions
Consider the equation:
F (x, y(x), c) = 0, (14.1)
which implicitly defines a one-parameter family of functions y(x; c). Here y is a function of the variable x and the
parameter c. For simplicity, we will write y(x) and not explicitly show the parameter dependence.

Example 14.3.1 The equation y = cx defines family of lines with slope c, passing through the origin. The equation
x2 + y 2 = c2 defines circles of radius c, centered at the origin.
Consider a chicken dropped from a height h. The elevation y of the chicken at time t after its release is y(t) = hgt2 ,
where g is the acceleration due to gravity. This is family of functions for the parameter h.

It turns out that the general solution of any first order differential equation is a one-parameter family of functions.
This is not easy to prove. However, it is easy to verify the converse. We differentiate Equation 14.1 with respect to x.
Fx + Fy y 0 = 0
(We assume that F has a non-trivial dependence on y, that is Fy 6= 0.) This gives us two equations involving the
independent variable x, the dependent variable y(x) and its derivative and the parameter c. If we algebraically eliminate
c between the two equations, the eliminant will be a first order differential equation for y(x). Thus we see that every
one-parameter family of functions y(x) satisfies a first order differential equation. This y(x) is the primitive of the
differential equation. Later we will discuss why y(x) is the general solution of the differential equation.

Example 14.3.2 Consider the family of circles of radius c centered about the origin.
x2 + y 2 = c2
Differentiating this yields:
2x + 2yy 0 = 0.
It is trivial to eliminate the parameter and obtain a differential equation for the family of circles.
x + yy 0 = 0

777
y = x/y

Figure 14.3: A circle and its tangent.

We can see the geometric meaning in this equation by writing it in the form:
x
y0 = .
y

For a point on the circle, the slope of the tangent y 0 is the negative of the cotangent of the angle x/y. (See Figure 14.3.)

Example 14.3.3 Consider the one-parameter family of functions:

y(x) = f (x) + cg(x),

where f (x) and g(x) are known functions. The derivative is

y 0 = f 0 + cg 0 .

778
We eliminate the parameter.
gy 0 g 0 y = gf 0 g 0 f
g0 g0f
y0 y = f 0
g g
Thus we see that y(x) = f (x)+cg(x) satisfies a first order linear differential equation. Later we will prove the converse:
the general solution of a first order linear differential equation has the form: y(x) = f (x) + cg(x).

We have shown that every one-parameter family of functions satisfies a first order differential equation. We do not
prove it here, but the converse is true as well.
Result 14.3.1 Every first order differential equation has a one-parameter family of solutions
y(x) defined by an equation of the form:

F (x, y(x); c) = 0.

This y(x) is called the general solution. If the equation is linear then the general solution
expresses the totality of solutions of the differential equation. If the equation is nonlinear,
there may be other special singular solutions, which do not depend on a parameter.

This is strictly an existence result. It does not say that the general solution of a first order differential equation
can be determined by some method, it just says that it exists. There is no method for solving the general first order
differential equation. However, there are some special forms that are soluble. We will devote the rest of this chapter to
studying these forms.

14.4 Integrable Forms


In this section we will introduce a few forms of differential equations that we may solve through integration.

779
14.4.1 Separable Equations

Any differential equation that can written in the form

P (x) + Q(y)y 0 = 0

is a separable equation, (because the dependent and independent variables are separated). We can obtain an implicit
solution by integrating with respect to x.

Z Z
dy
P (x) dx + Q(y) dx = c
dx
Z Z
P (x) dx + Q(y) dy = c

Result 14.4.1 The separable equation P (x) + Q(y)y 0 = 0 may be solved by integrating with
respect to x. The general solution is
Z Z
P (x) dx + Q(y) dy = c.

Example 14.4.1 Consider the differential equation y 0 = xy 2 . We separate the dependent and independent variables

780
and integrate to find the solution.

dy
= xy 2
dx
y 2 dy = x dx
Z Z
2
y dy = x dx + c
x2
y 1 = +c
2
1
y= 2
x /2 + c

Example 14.4.2 The equation y 0 = y y 2 is separable.

y0
=1
y y2

We expand in partial fractions and integrate.

 
1 1
y0 = 1
y y1
ln |y| ln |y 1| = x + c

781
We have an implicit equation for y(x). Now we solve for y(x).

y
ln =x+c
y 1

y x+c
y 1 = e

y
= ex+c
y1
y
= c ex 1
y1
c ex
y= x
c e 1
1
y=
1 + c ex

14.4.2 Exact Equations


Any first order ordinary differential equation of the first degree can be written as the total differential equation,
P (x, y) dx + Q(x, y) dy = 0.
If this equation can be integrated directly, that is if there is a primitive, u(x, y), such that
du = P dx + Q dy,
then this equation is called exact. The (implicit) solution of the differential equation is
u(x, y) = c,
where c is an arbitrary constant. Since the differential of a function, u(x, y), is
u u
du dx + dy,
x y

782
P and Q are the partial derivatives of u:
u u
P (x, y) = , Q(x, y) = .
x y

In an alternate notation, the differential equation


dy
P (x, y) + Q(x, y) = 0, (14.2)
dx
is exact if there is a primitive u(x, y) such that
du u u dy dy
+ = P (x, y) + Q(x, y) .
dx x y dx dx
The solution of the differential equation is u(x, y) = c.

Example 14.4.3
dy
x+y =0
dx
is an exact differential equation since  
d 1 2 2 dy
(x + y ) = x + y
dx 2 dx
The solution of the differential equation is
1 2
(x + y 2 ) = c.
2
Example 14.4.4 , Let f (x) and g(x) be known functions.

g(x)y 0 + g 0 (x)y = f (x)

is an exact differential equation since


d
(g(x)y(x)) = gy 0 + g 0 y.
dx

783
The solution of the differential equation is
Z
g(x)y(x) = f (x) dx + c
Z
1 c
y(x) = f (x) dx + .
g(x) g(x)

A necessary condition for exactness. The solution of the exact equation P + Qy 0 = 0 is u = c where u is
the primitive of the equation, du dx
= P + Qy 0 . At present the only method we have for determining the primitive is
guessing. This is fine for simple equations, but for more difficult cases we would like a method more concrete than
divine inspiration. As a first step toward this goal we determine a criterion for determining if an equation is exact.
Consider the exact equation,
P + Qy 0 = 0,
with primitive u, where we assume that the functions P and Q are continuously differentiable. Since the mixed partial
derivatives of u are equal,
2u 2u
= ,
xy yx
a necessary condition for exactness is
P Q
= .
y x

A sufficient condition for exactness. This necessary condition for exactness is also a sufficient condition. We
demonstrate this by deriving the general solution of (14.2). Assume that P + Qy 0 = 0 is not necessarily exact, but
satisfies the condition Py = Qx . If the equation has a primitive,
du u u dy dy
+ = P (x, y) + Q(x, y) ,
dx x y dx dx
then it satisfies
u u
= P, = Q. (14.3)
x y

784
Integrating the first equation of (14.3), we see that the primitive has the form
Z x
u(x, y) = P (, y) d + f (y),
x0

for some f (y). Now we substitute this form into the second equation of (14.3).
u
= Q(x, y)
y
Z x
Py (, y) d + f 0 (y) = Q(x, y)
x0

Now we use the condition Py = Qx .


Z x
Qx (, y) d + f 0 (y) = Q(x, y)
x0
Q(x, y) Q(x0 , y) + f 0 (y) = Q(x, y)
f 0 (y) = Q(x0 , y)
Z y
f (y) = Q(x0 , ) d
y0

Thus we see that Z x Z y


u= P (, y) d + Q(x0 , ) d
x0 y0

is a primitive of the derivative; the equation is exact. The solution of the differential equation is
Z x Z y
P (, y) d + Q(x0 , ) d = c.
x0 y0

Even though there are three arbitrary constants: x0 , y0 and c, the solution is a one-parameter family. This is because
changing x0 or y0 only changes the left side by an additive constant.

785
Result 14.4.2 Any first order differential equation of the first degree can be written in the
form
dy
P (x, y) + Q(x, y) = 0.
dx
This equation is exact if and only if
Py = Qx .
In this case the solution of the differential equation is given by
Z x Z y
P (, y) d + Q(x0 , ) d = c.
x0 y0

Exercise 14.1
Solve the following differential equations by inspection. That is, group terms into exact derivatives and then integrate.
f (x) and g(x) are known functions.
y 0 (x)
1. y(x)
= f (x)

2. y (x)y 0 (x) = f (x)


y0
3. cos x
+ y tan x
cos x
= cos x

Hint, Solution

14.4.3 Homogeneous Coefficient Equations


Homogeneous coefficient, first order differential equations form another class of soluble equations. We will find that
a change of dependent variable will make such equations separable or we can determine an integrating factor that will
make such equations exact. First we define homogeneous functions.

786
Eulers Theorem on Homogeneous Functions. The function F (x, y) is homogeneous of degree n if
F (x, y) = n F (x, y).
From this definition we see that  y
F (x, y) = xn F 1, .
x
(Just formally substitute 1/x for .) For example,
x2 y + 2y 3
xy 2 , , x cos(y/x)
x+y
are homogeneous functions of orders 3, 2 and 1, respectively.
Eulers theorem for a homogeneous function of order n is:
xFx + yFy = nF.
To prove this, we define = x, = y. From the definition of homogeneous functions, we have
F (, ) = n F (x, y).
We differentiate this equation with respect to .
F (, ) F (, )
+ = nn1 F (x, y)

xF + yF = nn1 F (x, y)
Setting = 1, (and hence = x, = y), proves Eulers theorem.

Result 14.4.3 Eulers Theorem on Homogeneous Functions. If F (x, y) is a homoge-


neous function of degree n, then

xFx + yFy = nF.

787
Homogeneous Coefficient Differential Equations. If the coefficient functions P (x, y) and Q(x, y) are homo-
geneous of degree n then the differential equation,
dy
P (x, y) + Q(x, y) = 0, (14.4)
dx
is called a homogeneous coefficient equation. They are often referred to simply as homogeneous equations.

Transformation to a Separable Equation. We can write the homogeneous equation in the form,
 y  y  dy
n n
x P 1, + x Q 1, = 0,
x x dx
 y  y  dy
P 1, + Q 1, = 0.
x x dx
y(x)
This suggests the change of dependent variable u(x) = x
.
 
du
P (1, u) + Q(1, u) u + x =0
dx
This equation is separable.
du
P (1, u) + uQ(1, u) + xQ(1, u) =0
dx
1 Q(1, u) du
+ =0
x P (1, u) + uQ(1, u) dx
Z
1
ln |x| + du = c
u + P (1, u)/Q(1, u)

By substituting ln |c| for c, we can write this in a simpler form.


Z
1 c
du = ln .

u + P (1, u)/Q(1, u) x

788
Integrating Factor. One can show that

1
(x, y) =
xP (x, y) + yQ(x, y)

is an integrating factor for the Equation 14.4. The proof of this is left as an exercise for the reader. (See Exercise 14.2.)

Result 14.4.4 Homogeneous Coefficient Differential Equations. If P (x, y) and Q(x, y)


are homogeneous functions of degree n, then the equation
dy
P (x, y) + Q(x, y) =0
dx
is made separable by the change of independent variable u(x) = y(x)
x . The solution is deter-
mined by Z
1 c
du = ln .

u + P (1, u)/Q(1, u) x
Alternatively, the homogeneous equation can be made exact with the integrating factor
1
(x, y) = .
xP (x, y) + yQ(x, y)

Example 14.4.5 Consider the homogeneous coefficient equation

dy
x2 y 2 + xy = 0.
dx

789
The solution for u(x) = y(x)/x is determined by
Z
1 c
du = ln

1u2 x

u+ u
Z c
u du = ln

x
1 2 c
u = ln

2 p x
u = 2 ln |c/x|

Thus the solution of the differential equation is


p
y = x 2 ln |c/x|

Exercise 14.2
Show that
1
(x, y) =
xP (x, y) + yQ(x, y)
is an integrating factor for the homogeneous equation,

dy
P (x, y) + Q(x, y) = 0.
dx
Hint, Solution
Exercise 14.3 (mathematica/ode/first order/exact.nb)
Find the general solution of the equation
dy y  y 2
=2 + .
dt t t
Hint, Solution

790
14.5 The First Order, Linear Differential Equation
14.5.1 Homogeneous Equations
The first order, linear, homogeneous equation has the form
dy
+ p(x)y = 0.
dx
Note that if we can find one solution, then any constant times that solution also satisfies the equation. If fact, all the
solutions of this equation differ only by multiplicative constants. We can solve any equation of this type because it is
separable.
y0
= p(x)
y
Z
ln |y| = p(x) dx + c
R
y = e p(x) dx+c
R
y = c e p(x) dx

Result 14.5.1 First Order, Linear Homogeneous Differential Equations. The first
order, linear, homogeneous differential equation,
dy
+ p(x)y = 0,
dx
has the solution R
y = c e p(x) dx
. (14.5)
The solutions differ by multiplicative constants.

791
Example 14.5.1 Consider the equation
dy 1
+ y = 0.
dx x
We use Equation 14.5 to determine the solution.
R
y(x) = c e 1/x dx
, for x 6= 0
y(x) = c e ln |x|
c
y(x) =
|x|
c
y(x) =
x

14.5.2 Inhomogeneous Equations


The first order, linear, inhomogeneous differential equation has the form
dy
+ p(x)y = f (x). (14.6)
dx
This equation is not separable. Note that it is similar to the exact equation we solved in Example 14.4.4,

g(x)y 0 (x) + g 0 (x)y(x) = f (x).

To solve Equation 14.6, we multiply by an integrating factor. Multiplying a differential equation by its integrating factor
changes it to an exact equation. Multiplying Equation 14.6 by the function, I(x), yields,
dy
I(x) + p(x)I(x)y = f (x)I(x).
dx
In order that I(x) be an integrating factor, it must satisfy
d
I(x) = p(x)I(x).
dx

792
This is a first order, linear, homogeneous equation with the solution
R
p(x) dx
I(x) = c e .

This is an integrating factor for any constant c. For simplicity we will choose c = 1.

R
To solve Equation 14.6 we multiply by the integrating factor and integrate. Let P (x) = p(x) dx.

dy
eP (x) + p(x) eP (x) y = eP (x) f (x)
dx
d P (x) 
e y = eP (x) f (x)
dx Z
P (x)
y=e eP (x) f (x) dx + c eP (x)

y yp + c yh

Note that the general solution is the sum of a particular solution, yp , that satisfies y 0 + p(x)y = f (x), and an arbitrary
constant times a homogeneous solution, yh , that satisfies y 0 + p(x)y = 0.

Example 14.5.2 Consider the differential equation

1
y 0 + y = x2 , x > 0.
x

First we find the integrating factor.


Z 
1
I(x) = exp dx = eln x = x
x

793
10

-1 1
-5

-10

Figure 14.4: Solutions to y 0 + y/x = x2 .

We multiply by the integrating factor and integrate.

d
(xy) = x3
dx
1
xy = x4 + c
4
1 c
y = x3 + .
4 x

The particular and homogeneous solutions are

1 1
yp = x3 and yh = .
4 x
Note that the general solution to the differential equation is a one-parameter family of functions. The general solution
is plotted in Figure 14.4 for various values of c.

794
Exercise 14.4 (mathematica/ode/first order/linear.nb)
Solve the differential equation
1
y 0 y = x , x > 0.
x
Hint, Solution

14.5.3 Variation of Parameters.


We could also have found the particular solution with the method of variation of parameters. Although we can
solve first order equations without this method, it will become important in the study of higher order inhomogeneous
equations. We begin by assuming that the particular solution has the form yp = u(x)yh (x) where u(x) is an unknown
function. We substitute this into the differential equation.
d
yp + p(x)yp = f (x)
dx
d
(uyh ) + p(x)uyh = f (x)
dx
u0 yh + u(yh0 + p(x)yh ) = f (x)

Since yh is a homogeneous solution, yh0 + p(x)yh = 0.

f (x)
u0 =
yh
Z
f (x)
u= dx
yh (x)

Recall that the homogeneous solution is yh = eP (x) .


Z
u= eP (x) f (x) dx

795
Thus the particular solution is Z
P (x)
yp = e eP (x) f (x) dx.

14.6 Initial Conditions


In physical problems involving first order differential equations, the solution satisfies both the differential equation
and a constraint which we call the initial condition. Consider a first order linear differential equation subject to the
initial condition y(x0 ) = y0 . The general solution is
Z
P (x)
y = yp + cyh = e eP (x) f (x) dx + c eP (x) .

For the moment, we will assume that this problem is well-posed. A problem R is well-posed if there is a unique solution to
the differential equation thatRsatisfies the constraint(s). Recall that eP (x) f (x) dx denotes any integral of eP (x) f (x).
x
For convenience, we choose x0 eP () f () d. The initial condition requires that
Z x0
P (x0 )
y(x0 ) = y0 = e eP () f () d + c eP (x0 ) = c eP (x0 ) .
x0

Thus c = y0 eP (x0 ) . The solution subject to the initial condition is


Z x
P (x)
y=e eP () f () d + y0 eP (x0 )P (x) .
x0

Example 14.6.1 Consider the problem


y 0 + (cos x)y = x, y(0) = 2.
From Result 14.6.1, the solution subject to the initial condition is
Z x
sin x
y=e esin d + 2 e sin x .
0

796
14.6.1 Piecewise Continuous Coefficients and Inhomogeneities
If the coefficient function p(x) and the inhomogeneous term f (x) in the first order linear differential equation

dy
+ p(x)y = f (x)
dx
are continuous, then the solution is continuous and has a continuous first derivative. To see this, we note that the
solution Z
P (x)
y=e eP (x) f (x) dx + c eP (x)

is continuous since the integral of a piecewise continuous function is continuous. The first derivative of the solution
can be found directly from the differential equation.

y 0 = p(x)y + f (x)

Since p(x), y, and f (x) are continuous, y 0 is continuous.


If p(x) or f (x) is only piecewise continuous, then the solution will be continuous since the integral of a piecewise
continuous function is continuous. The first derivative of the solution will be piecewise continuous.

Example 14.6.2 Consider the problem

y 0 y = H(x 1), y(0) = 1,

where H(x) is the Heaviside function. (


1 for x > 0,
H(x) =
0 for x < 0.
To solve this problem, we divide it into two equations on separate domains.

y10 y1 = 0, y1 (0) = 1, for x < 1


y20 y2 = 1, y2 (1) = y1 (1), for x > 1

797
8
6
4
2

-1 1 2

Figure 14.5: Solution to y 0 y = H(x 1).

With the condition y2 (1) = y1 (1) on the second equation, we demand that the solution be continuous. The solution
to the first equation is y = ex . The solution for the second equation is
Z x
y=e x
e d + e1 ex1 = 1 + ex1 + ex .
1

Thus the solution over the whole domain is


(
ex for x < 1,
y=
(1 + e1 ) ex 1 for x > 1.

The solution is graphed in Figure 14.5.

Example 14.6.3 Consider the problem,

y 0 + sign(x)y = 0, y(1) = 1.

798
Recall that

1
for x < 0
sign x = 0 for x = 0

1 for x > 0.

Since sign x is piecewise defined, we solve the two problems,

0
y+ + y+ = 0, y+ (1) = 1, for x > 0
0
y y = 0, y (0) = y+ (0), for x < 0,

and define the solution, y, to be


(
y+ (x), for x 0,
y(x) =
y (x), for x 0.

The initial condition for y demands that the solution be continuous.


Solving the two problems for positive and negative x, we obtain

(
e1x , for x > 0,
y(x) =
e1+x , for x < 0.

This can be simplified to

y(x) = e1|x| .

This solution is graphed in Figure 14.6.

799
2

-3 -2 -1 1 2 3

Figure 14.6: Solution to y 0 + sign(x)y = 0.

Result 14.6.1 Existence, Uniqueness Theorem. Let p(x) and f (x) be piecewise contin-
uous on the interval [a, b] and let x0 [a, b]. Consider the problem,
dy
+ p(x)y = f (x), y(x0 ) = y0 .
dx
The general solution of the differential equation is
Z
P (x)
y=e eP (x) f (x) dx + c eP (x) .

The unique, continuous solution of the differential equation subject to the initial condition is
Z x
y = eP (x) eP () f () d + y0 eP (x0 )P (x) ,
x0
R
where P (x) = p(x) dx.
800
Exercise 14.5 (mathematica/ode/first order/exact.nb)
Find the solutions of the following differential equations which satisfy the given initial conditions:
dy
1. + xy = x2n+1 , y(1) = 1, nZ
dx
dy
2. 2xy = 1, y(0) = 1
dx
Hint, Solution
Exercise 14.6 (mathematica/ode/first order/exact.nb)
Show that if > 0 and > 0, then for any real , every solution of
dy
+ y(x) = ex
dx
satisfies limx+ y(x) = 0. (The case = requires special treatment.) Find the solution for = = 1 which
satisfies y(0) = 1. Sketch this solution for 0 x < for several values of . In particular, show what happens when
0 and .
Hint, Solution

14.7 Well-Posed Problems


Example 14.7.1 Consider the problem,
1
y 0 y = 0, y(0) = 1.
x
The general solution is y = cx. Applying the initial condition demands that 1 = c 0, which cannot be satisfied. The
general solution for various values of c is plotted in Figure 14.7.

Example 14.7.2 Consider the problem


1 1
y0 y = , y(0) = 1.
x x

801
1

-1 1

-1

Figure 14.7: Solutions to y 0 y/x = 0.

The general solution is


y = 1 + cx.
The initial condition is satisfied for any value of c so there are an infinite number of solutions.

Example 14.7.3 Consider the problem


1
y 0 + y = 0, y(0) = 1.
x
The general solution is y = xc . Depending on whether c is nonzero, the solution is either singular or zero at the origin
and cannot satisfy the initial condition.

The above problems in which there were either no solutions or an infinite number of solutions are said to be ill-posed.
If there is a unique solution that satisfies the initial condition, the problem is said to be well-posed. We should have
suspected that we would run into trouble in the above examples as the initial condition was given at a singularity of
the coefficient function, p(x) = 1/x.

802
Consider the problem,
y 0 + p(x)y = f (x), y(x0 ) = y0 .
We assume that f (x) bounded in a neighborhood of x = x0 . The differential equation has the general solution,
Z
P (x)
y=e eP (x) f (x) dx + c eP (x) .

If the homogeneous solution, eP (x) , is nonzero and finite at x = x0 , then there is a unique value of c for which the
initial condition is satisfied. If the homogeneous solution vanishes at x = x0 then either the initial condition cannot be
satisfied or the initial condition is satisfied for all values of c. The homogeneous solution can vanish or be infinite only
if P (x) as x x0 . This can occur only if the coefficient function, p(x), is unbounded at that point.

Result 14.7.1 If the initial condition is given where the homogeneous solution to a first
order, linear differential equation is zero or infinite then the problem may be ill-posed. This
may occur only if the coefficient function, p(x), is unbounded at that point.

14.8 Equations in the Complex Plane


14.8.1 Ordinary Points
Consider the first order homogeneous equation

dw
+ p(z)w = 0,
dz
where p(z), a function of a complex variable, is analytic in some domain D. The integrating factor,
Z 
I(z) = exp p(z) dz ,

803
is an analytic function in that domain. As with the case of real variables, multiplying by the integrating factor and
integrating yields the solution,  Z 
w(z) = c exp p(z) dz .

We see that the solution is analytic in D.

Example 14.8.1 It does not make sense to pose the equation

dw
+ |z|w = 0.
dz
For the solution to exist, w and hence w0 (z) must be analytic. Since p(z) = |z| is not analytic anywhere in the complex
plane, the equation has no solution.

Any point at which p(z) is analytic is called an ordinary point of the differential equation. Since the solution is
analytic we can expand it in a Taylor series about an ordinary point. The radius of convergence of the series will be at
least the distance to the nearest singularity of p(z) in the complex plane.

Example 14.8.2 Consider the equation


dw 1
w = 0.
dz 1z
c
The general solution is w = 1z . Expanding this solution about the origin,

c X
w= =c zn.
1z n=0

The radius of convergence of the series is,


an
R = lim = 1,
n an+1

1
which is the distance from the origin to the nearest singularity of p(z) = 1z
.

804
We do not need to solve the differential equation to find the Taylor series expansion of the homogeneous solution.
We could substitute a general Taylor series expansion into the differential equation and solve for the coefficients. Since
we can always solve first order equations, this method is of limited usefulness. However, when we consider higher order
equations in which we cannot solve the equations exactly, this will become an important method.

Example 14.8.3 Again consider the equation


dw 1
w = 0.
dz 1z
P
Since we know that the solution has a Taylor series expansion about z = 0, we substitute w = n=0 an z n into the
differential equation.

d X n
X
(1 z) an z an z n = 0
dz n=0 n=0

X
X
X
nan z n1 nan z n an z n = 0
n=1 n=1 n=0

X
X
X
n n
(n + 1)an+1 z nan z an z n = 0
n=0 n=0 n=0

X
((n + 1)an+1 (n + 1)an ) z n = 0.
n=0

Now we equate powers of z to zero. For z n , the equation is (n + 1)an+1 (n + 1)an = 0, or an+1 = an . Thus we have
that an = a0 for all n 1. The solution is then

X
w = a0 zn,
n=0

which is the result we obtained by expanding the solution in Example 14.8.2.

805
Result 14.8.1 Consider the equation
dw
+ p(z)w = 0.
dz
If p(z) is analytic at z = z0 then z0 is called an ordinary point of the differential
Pequation. Then
Taylor series expansion of the solution can be found by substituting w = n=0 an (z z0 )
into the equation and equating powers of (z z0 ). The radius of convergence of the series is
at least the distance to the nearest singularity of p(z) in the complex plane.

Exercise 14.7
Find the Taylor series expansion about the origin of the solution to
dw 1
+ w=0
dz 1z
P
with the substitution w = n=0 an z n . What is the radius of convergence of the series? What is the distance to the
1
nearest singularity of 1z ?
Hint, Solution

14.8.2 Regular Singular Points


If the coefficient function p(z) has a simple pole at z = z0 then z0 is a regular singular point of the first order
differential equation.

Example 14.8.4 Consider the equation


dw
+ w = 0, 6= 0.
dz z
This equation has a regular singular point at z = 0. The solution is w = cz . Depending on the value of , the
solution can have three different kinds of behavior.

806
is a negative integer. The solution is analytic in the finite complex plane.
is a positive integer The solution has a pole at the origin. w is analytic in the annulus, 0 < |z|.
is not an integer. w has a branch point at z = 0. The solution is analytic in the cut annulus 0 < |z| < ,
0 < arg z < 0 + 2.

Consider the differential equation


dw
+ p(z)w = 0,
dz
where p(z) has a simple pole at the origin and is analytic in the annulus, 0 < |z| < r, for some positive r. Recall that
the solution is
 Z 
w = c exp p(z) dz
 Z 
b0 b0
= c exp + p(z) dz
z z
 
zp(z) b0
Z
= c exp b0 log z dz
z
 Z 
b0 zp(z) b0
= cz exp dz
z
The exponential factor has a removable singularity at z = 0 and is analytic in |z| < r. We consider the following
cases for the z b0 factor:
b0 is a negative integer. Since z b0 is analytic at the origin, the solution to the differential equation is analytic in
the circle |z| < r.
b0 is a positive integer. The solution has a pole of order b0 at the origin and is analytic in the annulus 0 < |z| < r.
b0 is not an integer. The solution has a branch point at the origin and thus is not single-valued. The solution is
analytic in the cut annulus 0 < |z| < r, 0 < arg z < 0 + 2.

807
Since the exponential factor has a convergent Taylor series in |z| < r, the solution can be expanded in a series of
the form


X
b0
w=z an z n , where a0 6= 0 and b0 = lim z p(z).
z0
n=0

In the case of a regular singular point at z = z0 , the series is


X
b0
w = (z z0 ) an (z z0 )n , where a0 6= 0 and b0 = lim (z z0 ) p(z).
zz0
n=0

Series of this form are known as Frobenius series. Since we can write the solution as

 Z   
b0 b0
w = c(z z0 ) exp p(z) dz ,
z z0

we see that the Frobenius expansion of the solution will have a radius of convergence at least the distance to the nearest
singularity of p(z).

808
Result 14.8.2 Consider the equation,
dw
+ p(z)w = 0,
dz
where p(z) has a simple pole at z = z0 , p(z) is analytic in some annulus, 0 < |z z0 | < r,
and limzz0 (z z0 )p(z) = . The solution to the differential equation has a Frobenius series
expansion of the form

X

w = (z z0 ) an (z z0 )n , a0 6= 0.
n=0

The radius of convergence of the expansion will be at least the distance to the nearest
singularity of p(z).

Example 14.8.5 We will find the first two nonzero terms in the series solution about z = 0 of the differential equation,
dw 1
+ w = 0.
dz sin z
First we note that the coefficient function has a simple pole at z = 0 and
z 1
lim = lim = 1.
z0 sin z z0 cos z

Thus we look for a series solution of the form



X
w = z 1 an z n , a0 6= 0.
n=0

The nearest singularities of 1/ sin z in the complex plane are at z = . Thus the radius of convergence of the series
will be at least .

809
Substituting the first three terms of the expansion into the differential equation,
d 1
(a0 z 1 + a1 + a2 z) + (a0 z 1 + a1 + a2 z) = O(z).
dz sin z
Recall that the Taylor expansion of sin z is sin z = z 61 z 3 + O(z 5 ).

z3
 
z + O(z ) (a0 z 2 + a2 ) + (a0 z 1 + a1 + a2 z) = O(z 2 )
5
6
 a0 
a0 z 1 + a2 + z + a0 z 1 + a1 + a2 z = O(z 2 )
6
 a0 
a1 + 2a2 + z = O(z 2 )
6
a0 is arbitrary. Equating powers of z,
z0 : a1 = 0.
a0
z1 : 2a2 + = 0.
6
Thus the solution has the expansion,
 z
w = a0 z 1 + O(z 2 ).
12
In Figure 14.8 the exact solution is plotted in a solid line and the two term approximation is plotted in a dashed line.
The two term approximation is very good near the point x = 0.

Example 14.8.6 Find the first two nonzero terms in the series expansion about z = 0 of the solution to
cos z
w0 i w = 0.
z
Since cosz z has a simple pole at z = 0 and limz0 i cos z = i we see that the Frobenius series will have the form

X
i
w=z an z n , a0 6= 0.
n=0

810
4

2 4 6
-2

-4

Figure 14.8: Plot of the exact solution and the two term approximation.

P (1)n z 2n
Recall that cos z has the Taylor expansion n=0 (2n)!
. Substituting the Frobenius expansion into the differential
equation yields


!
!
!
X X X (1)n z 2n X
z iz i1 an z n + z i nan z n1 i zi an z n =0
n=0 n=0 n=0
(2n)! n=0

!
!
X X (1)n z 2n X
(n + i)an z n i an z n = 0.
n=0 n=0
(2n)! n=0

Equating powers of z,

z 0 : ia0 ia0 = 0 a0 is arbitrary


1
z : (1 + i)a1 ia1 = 0 a1 = 0
i i
z 2 : (2 + i)a2 ia2 + a0 = 0 a2 = a0 .
2 4

811
Thus the solution is
 
i i 2 3
w = a0 z 1 z + O(z ) .
4

14.8.3 Irregular Singular Points


If a point is not an ordinary point or a regular singular point then it is called an irregular singular point. The following
equations have irregular singular points at the origin.

w0 + zw = 0

w0 z 2 w = 0

w0 + exp(1/z)w = 0

Example 14.8.7 Consider the differential equation

dw
+ z w = 0, 6= 0, 6= 1, 0, 1, 2, . . .
dz
This equation has an irregular singular point at the origin. Solving this equation,
 Z  
d
exp z dz w = 0
dz
n
(1)n
  
+1 X
w = c exp z =c z (+1)n .
+1 n=0
n! + 1

If is not an integer, then the solution has a branch point at the origin. If is an integer, < 1, thenPthe solution
has an essential singularity at the origin. The solution cannot be expanded in a Frobenius series, w = z n
n=0 an z .

812
Although we will not show it, this result holds for any irregular singular point of the differential equation. We cannot
approximate the solution near an irregular singular point using a Frobenius expansion.

Now would be a good time to summarize what we have discovered about solutions of first order differential equations
in the complex plane.

Result 14.8.3 Consider the first order differential equation


dw
+ p(z)w = 0.
dz
Ordinary Points If p(z) is analytic at z = z0 then z0 is an ordinary pointPof the differential
equation. The solution can be expanded in the Taylor series w = n
n=0 an (z z0 ) .
The radius of convergence of the series is at least the distance to the nearest singularity
of p(z) in the complex plane.
Regular Singular Points If p(z) has a simple pole at z = z0 and is analytic in some annulus
0 < |z z0 | < r then z0 is a regular singular point of the differential equation. The
solution at z0 will either be analytic, have a pole, or have
Pa branch point.
The solution
n
can be expanded in the Frobenius series w = (z z0 ) n=0 an (z z0 ) where a0 6= 0
and = limzz0 (z z0 )p(z). The radius of convergence of the Frobenius series will be
at least the distance to the nearest singularity of p(z).
Irregular Singular Points If the point z = z0 is not an ordinary point or a regular singular
point, then it is an irregular singular point of the differential equation. The solution
cannot be expanded in a Frobenius series about that point.

813
14.8.4 The Point at Infinity
Now we consider the behavior of first order linear differential equations at the point at infinity. Recall from complex
variables that the complex plane together with the point at infinity is called the extended complex plane. To study the
behavior of a function f (z) at infinity, we make the transformation z = 1 and study the behavior of f (1/) at = 0.

Example 14.8.8 Lets examine the behavior of sin z at infinity. We make the substitution z = 1/ and find the
Laurent expansion about = 0.

X (1)n
sin(1/) =
n=0
(2n + 1)! (2n+1)

Since sin(1/) has an essential singularity at = 0, sin z has an essential singularity at infinity.

We use the same approach if we want to examine the behavior at infinity of a differential equation. Starting with
the first order differential equation,
dw
+ p(z)w = 0,
dz
we make the substitution
1 d d
z= , = 2 , w(z) = u()
dz d

to obtain

du
2 + p(1/)u = 0
d
du p(1/)
u = 0.
d 2

814
Result 14.8.4 The behavior at infinity of
dw
+ p(z)w = 0
dz
is the same as the behavior at = 0 of
du p(1/)
u = 0.
d 2

Example 14.8.9 We classify the singular points of the equation


dw 1
+ 2 w = 0.
dz z +9
We factor the denominator of the fraction to see that z = 3 and z = 3 are regular singular points.
dw 1
+ w=0
dz (z 3)(z + 3)

We make the transformation z = 1/ to examine the point at infinity.

du 1 1
2 u=0
d (1/)2 + 9
du 1
2 u=0
d 9 + 1
Since the equation for u has a ordinary point at = 0, z = is a ordinary point of the equation for w.

815
14.9 Additional Exercises
Exact Equations
Exercise 14.8 (mathematica/ode/first order/exact.nb)
Find the general solution y = y(x) of the equations

dy x2 + xy + y 2
1. = ,
dx x2
2. (4y 3x) dx + (y 2x) dy = 0.

Hint, Solution
Exercise 14.9 (mathematica/ode/first order/exact.nb)
Determine whether or not the following equations can be made exact. If so find the corresponding general solution.

1. (3x2 2xy + 2) dx + (6y 2 x2 + 3) dy = 0


dy ax + by
2. =
dx bx + cy
Hint, Solution
Exercise 14.10 (mathematica/ode/first order/exact.nb)
Find the solutions of the following differential equations which satisfy the given initial condition. In each case determine
the interval in which the solution is defined.
dy
1. = (1 2x)y 2 , y(0) = 1/6.
dx
2. x dx + y ex dy = 0, y(0) = 1.

Hint, Solution

816
Exercise 14.11
Are the following equations exact? If so, solve them.
1. (4y x)y 0 (9x2 + y 1) = 0
2. (2x 2y)y 0 + (2x + 4y) = 0.
Hint, Solution
Exercise 14.12 (mathematica/ode/first order/exact.nb)
Find all functions f (t) such that the differential equation
dy
y 2 sin t + yf (t) =0 (14.7)
dt
is exact. Solve the differential equation for these f (t).
Hint, Solution

The First Order, Linear Differential Equation


Exercise 14.13 (mathematica/ode/first order/linear.nb)
Solve the differential equation
y
y0 + = 0.
sin x
Hint, Solution

Initial Conditions
Well-Posed Problems
Exercise 14.14
Find the solutions of
dy
t+ Ay = 1 + t2 , t>0
dt
which are bounded at t = 0. Consider all (real) values of A.
Hint, Solution

817
Equations in the Complex Plane
Exercise 14.15
Classify the singular points of the following first order differential equations, (include the point at infinity).

1. w0 + sin z
z
w =0

2. w0 + 1
z3
w =0

3. w0 + z 1/2 w = 0

Hint, Solution
Exercise 14.16
Consider the equation
w0 + z 2 w = 0.
The point z = 0 is an irregular singular point of the differential equation. Thus we know that we cannot expand the
solution about z = 0 in a Frobenius series. Try substituting the series solution

X

w=z an z n , a0 6= 0
n=0

into the differential equation anyway. What happens?


Hint, Solution

818
14.10 Hints
Hint 14.1

d 1
1. dx
ln |u| = u

2. d c
dx
u = uc1 u0

Hint 14.2

Hint 14.3
The equation is homogeneous. Make the change of variables u = y/t.

Hint 14.4
Make sure you consider the case = 0.

Hint 14.5

Hint 14.6

Hint 14.7
1
The radius of convergence of the series and the distance to the nearest singularity of 1z
are not the same.

Exact Equations
Hint 14.8
1.

2.

819
Hint 14.9
1. The equation is exact. Determine the primitive u by solving the equations ux = P , uy = Q.

2. The equation can be made exact.

Hint 14.10
1. This equation is separable. Integrate to get the general solution. Apply the initial condition to determine the
constant of integration.

2. Ditto. You will have to numerically solve an equation to determine where the solution is defined.

Hint 14.11

Hint 14.12

The First Order, Linear Differential Equation


Hint 14.13
Look in the appendix for the integral of csc x.

Initial Conditions
Well-Posed Problems
Hint 14.14

Equations in the Complex Plane


Hint 14.15

820
Hint 14.16
Try to find the value of by substituting the series into the differential equation and equating powers of z.

821
14.11 Solutions
Solution 14.1

1.

y 0 (x)
= f (x)
y(x)
d
ln |y(x)| = f (x)
dx Z
ln |y(x)| = f (x) dx + c
R
f (x) dx+c
y(x) = e
R
f (x) dx
y(x) = c e

2.

y (x)y 0 (x) = f (x)


y +1 (x)
Z
= f (x) dx + c
+1
 Z 1/(+1)
y(x) = ( + 1) f (x) dx + a

822
3.
y0 tan x
+y = cos x
cos x cos x
d  y 
= cos x
dx cos x
y
= sin x + c
cos x
y(x) = sin x cos x + c cos x

Solution 14.2
We consider the homogeneous equation,
dy
= 0.
P (x, y) + Q(x, y)
dx
That is, both P and Q are homogeneous of degree n. We hypothesize that multiplying by

1
(x, y) =
xP (x, y) + yQ(x, y)

will make the equation exact. To prove this we use the result that

dy
M (x, y) + N (x, y) =0
dx
is exact if and only if My = Nx .
 
P
My =
y xP + yQ
Py (xP + yQ) P (xPy + Q + yQy )
=
(xP + yQ)2

823
 
Q
Nx =
x xP + yQ
Qx (xP + yQ) Q(P + xPx + yQx )
=
(xP + yQ)2

M y = Nx
Py (xP + yQ) P (xPy + Q + yQy ) = Qx (xP + yQ) Q(P + xPx + yQx )
yPy Q yP Qy = xP Qx xPx Q
xPx Q + yPy Q = xP Qx + yP Qy
(xPx + yPy )Q = P (xQx + yQy )

With Eulers theorem, this reduces to an identity.

nP Q = P nQ

Thus the equation is exact. (x, y) is an integrating factor for the homogeneous equation.

Solution 14.3
We note that this is a homogeneous differential equation. The coefficient of dy/dt and the inhomogeneity are homo-
geneous of degree zero.
dy  y   y 2
=2 + .
dt t t
We make the change of variables u = y/t to obtain a separable equation.

tu0 + u = 2u + u2
u0 1
2
=
u +u t

824
Now we integrate to solve for u.

u0 1
=
u(u + 1) t
0 0
u u 1
=
u u+1 t
ln |u| ln |u + 1| = ln |t| + c

u
ln = ln |ct|
u + 1
u
= ct
u+1
u
= ct
u+1
ct
u=
1 ct
t
u=
ct
t2
y=
ct

Solution 14.4
We consider
1
y 0 y = x , x > 0.
x
First we find the integrating factor.
Z 
1 1
I(x) = exp dx = exp ( ln x) = .
x x

825
We multiply by the integrating factor and integrate.
1 0 1
y 2 y = x1
x  x
d 1
y = x1
dx x
Z
1
y = x1 dx + c
x
Z
y = x x1 dx + cx
(
x+1

+ cx for 6= 0,
y=
x ln x + cx for = 0.

Solution 14.5
1.
y 0 + xy = x2n+1 , y(1) = 1, nZ
We find the integrating factor.
2 /2
R
x dx
I(x) = e = ex
We multiply by the integrating factor and integrate. Since the initial condition is given at x = 1, we will take the
lower bound of integration to be that point.
d  x2 /2  2
e y = x2n+1 ex /2
dx Z
x
x2 /2 2 2
y= e 2n+1 e /2 d + c ex /2
1

We choose the constant of integration to satisfy the initial condition.


Z x
x2 /2 2 2
y=e 2n+1 e /2 d + e(1x )/2
1

826
If n 0 then we can use integration by parts to write the integral as a sum of terms. If n < 0 we can write the
integral in terms of the exponential integral function. However, the integral form above is as nice as any other
and we leave the answer in that form.

2.
dy
2xy(x) = 1, y(0) = 1.
dx
We determine the integrating factor and then integrate the equation.
2
R
I(x) = e 2x dx = ex
d  x2  2
e y = ex
dx Z
x
2 2 2
y = ex e d + c ex
0

We choose the constant of integration to satisfy the initial condition.

 Z x 
x2 2
y=e 1+ e d
0

We can write the answer in terms of the Error function,


Z x
2 2
erf(x) e d.
0
 
x2
y=e 1+ erf(x)
2

827
Solution 14.6
We determine the integrating factor and then integrate the equation.
R
I(x) = e dx = ex
d x
(e y) = e()x
dx Z
y = ex e()x dx + c ex

First consider the case 6= .

e()x
y = ex + c ex


y= ex +c ex

Clearly the solution vanishes as x .


Next consider = .

y = ex x + c ex
y = (c + x) ex

We use LHospitals rule to show that the solution vanishes as x .

c + x
lim x
= lim =0
x e x ex
For = = 1, the solution is (
1
1
ex +c ex for 6= 1,
y=
(c + x) ex for = 1.

828
1

4 8 12 16

Figure 14.9: The Solution for a Range of

The solution which satisfies the initial condition is

(
(ex +( 2) ex ) for 6= 1,
1
1
y=
(1 + x) ex for = 1.

In Figure 14.9 the solution is plotted for = 1/16, 1/8, . . . , 16.


Consider the solution in the limit as 0.

1
ex +( 2) ex

lim y(x) = lim
0 0 1

= 2 ex

829
1

1 2 3 4 1 2 3 4

Figure 14.10: The Solution as 0 and

In the limit as we have,

1
ex +( 2) ex

lim y(x) = lim
1
2 x
= lim e
1
(
1 for x = 0,
=
0 for x > 0.

This behavior is shown in Figure 14.10. The first graph plots the solutions for = 1/128, 1/64, . . . , 1. The second
graph plots the solutions for = 1, 2, . . . , 128.

830
Solution 14.7
We substitute w = n dw 1
P
n=0 an z into the equation dz
+ 1z
w = 0.

d X n 1 X
an z + an z n = 0
dz n=0 1 z n=0

X
X
n1
(1 z) nan z + an z n = 0
n=1 n=0

X
X
X
n n
(n + 1)an+1 z nan z + an z n = 0
n=0 n=0 n=0

X
((n + 1)an+1 (n 1)an ) z n = 0
n=0

Equating powers of z to zero, we obtain the relation,


n1
an+1 = an .
n+1
a0 is arbitrary. We can compute the rest of the coefficients from the recurrence relation.
1
a1 = a0 = a0
1
0
a2 = a1 = 0
2
We see that the coefficients are zero for n 2. Thus the Taylor series expansion, (and the exact solution), is

w = a0 (1 z).

1
The radius of convergence of the series in infinite. The nearest singularity of 1z is at z = 1. Thus we see the radius
of convergence can be greater than the distance to the nearest singularity of the coefficient function, p(z).

831
Exact Equations

Solution 14.8
1.

dy x2 + xy + y 2
=
dx x2

Since the right side is a homogeneous function of order zero, this is a homogeneous differential equation. We
make the change of variables u = y/x and then solve the differential equation for u.

xu0 + u = 1 + u + u2
du dx
2
=
1+u x
arctan(u) = ln |x| + c
u = tan(ln(|cx|))
y = x tan(ln(|cx|))

2.

(4y 3x) dx + (y 2x) dy = 0

Since the coefficients are homogeneous functions of order one, this is a homogeneous differential equation. We

832
make the change of variables u = y/x and then solve the differential equation for u.
 y  y 
4 3 dx + 2 dy = 0
x x
(4u 3) dx + (u 2)(u dx + x du) = 0
(u2 + 2u 3) dx + x(u 2) du = 0
dx u2
+ du = 0
x (u + 3)(u 1)
 
dx 5/4 1/4
+ du = 0
x u+3 u1
5 1
ln(x) + ln(u + 3) ln(u 1) = c
4 4
x4 (u + 3)5
=c
u1
x4 (y/x + 3)5
=c
y/x 1
(y + 3x)5
=c
yx

Solution 14.9
1.
(3x2 2xy + 2) dx + (6y 2 x2 + 3) dy = 0
We check if this form of the equation, P dx + Q dy = 0, is exact.

Py = 2x, Qx = 2x

Since Py = Qx , the equation is exact. Now we find the primitive u(x, y) which satisfies

du = (3x2 2xy + 2) dx + (6y 2 x2 + 3) dy.

833
The primitive satisfies the partial differential equations

ux = P, uy = Q. (14.8)

We integrate the first equation of 14.8 to determine u up to a function of integration.

ux = 3x2 2xy + 2
u = x3 x2 y + 2x + f (y)

We substitute this into the second equation of 14.8 to determine the function of integration up to an additive
constant.

x2 + f 0 (y) = 6y 2 x2 + 3
f 0 (y) = 6y 2 + 3
f (y) = 2y 3 + 3y

The solution of the differential equation is determined by the implicit equation u = c.

x3 x2 y + 2x + 2y 3 + 3y = c

2.
dy ax + by
=
dx bx + cy
(ax + by) dx + (bx + cy) dy = 0

We check if this form of the equation, P dx + Q dy = 0, is exact.

Py = b, Qx = b

Since Py = Qx , the equation is exact. Now we find the primitive u(x, y) which satisfies

du = (ax + by) dx + (bx + cy) dy

834
The primitive satisfies the partial differential equations
ux = P, uy = Q. (14.9)
We integrate the first equation of 14.9 to determine u up to a function of integration.
ux = ax + by
1
u = ax2 + bxy + f (y)
2
We substitute this into the second equation of 14.9 to determine the function of integration up to an additive
constant.
bx + f 0 (y) = bx + cy
f 0 (y) = cy
1
f (y) = cy 2
2
The solution of the differential equation is determined by the implicit equation u = d.
ax2 + 2bxy + cy 2 = d
Solution 14.10
Note that since these equations are nonlinear, we cannot predict where the solutions will be defined from the equation
alone.
1. This equation is separable. We integrate to get the general solution.
dy
= (1 2x)y 2
dx
dy
= (1 2x) dx
y2
1
= x x2 + c
y
1
y= 2
x xc

835
Now we apply the initial condition.
1 1
y(0) = =
c 6
1
y= 2
x x6
1
y=
(x + 2)(x 3)

The solution is defined on the interval (2 . . . 3).

2. This equation is separable. We integrate to get the general solution.

x dx + y ex dy = 0
x ex dx + y dy = 0
1
(x 1) ex + y 2 = c
p 2
y = 2(c + (1 x) ex )

We apply the initial condition to determine the constant of integration.


p
y(0) = 2(c + 1) = 1
1
c=
2
p
y = 2(1 x) ex 1

The function 2(1 x) ex 1 is plotted in Figure 14.11. We see that the argument of the square root in the
solution is non-negative only on an interval about the origin. Because 2(1 x) ex 1 == 0 is a mixed algebraic
/ transcendental equation, we cannot solve it analytically. The solution of the differential equation is defined on
the interval (1.67835 . . . 0.768039).

836
1
-5 -4 -3 -2 -1 1
-1
-2
-3

Figure 14.11: The function 2(1 x) ex 1.

Solution 14.11
1. We consider the differential equation,
(4y x)y 0 (9x2 + y 1) = 0.


1 y 9x2 = 1

Py =
y

Qx = (4y x) = 1
x
This equation is exact. It is simplest to solve the equation by rearranging terms to form exact derivatives.
4yy 0 xy 0 y + 1 9x2 = 0
d  2
2y xy + 1 9x2 = 0

dx
2y 2 xy + x 3x3 + c = 0
1 p 
y= x x2 8(c + x 3x3 )
4

2. We consider the differential equation,


(2x 2y)y 0 + (2x + 4y) = 0.

837

Py = (2x + 4y) = 4
y

Qx = (2x 2y) = 2
x
6 Qx , this is not an exact equation.
Since Py =

Solution 14.12
Recall that the differential equation
P (x, y) + Q(x, y)y 0 = 0
is exact if and only if Py = Qx . For Equation 14.7, this criterion is
2y sin t = yf 0 (t)
f 0 (t) = 2 sin t
f (t) = 2(a cos t).
In this case, the differential equation is
y 2 sin t + 2yy 0 (a cos t) = 0.
We can integrate this exact equation by inspection.
d 2 
y (a cos t) = 0
dt
y 2 (a cos t) = c
c
y =
a cos t

The First Order, Linear Differential Equation


Solution 14.13
Consider the differential equation
y
y0 + = 0.
sin x

838
We use Equation 14.5 to determine the solution.

R
1/ sin x dx
y = ce
y = c e ln | tan(x/2)|
 x 
y = c cot

2

x
y = c cot
2

Initial Conditions
Well-Posed Problems

Solution 14.14
First we write the differential equation in the standard form.

dy A 1
+ y = + t, t>0
dt t t

We determine the integrating factor.

R
A/t dt
I(t) = e = eA ln t = tA

839
We multiply the differential equation by the integrating factor and integrate.
dy A 1
+ y = +t
dt t t
d A
t y = tA1 + tA+1

dtA A+2
tA + tA+2 + c,
A 6= 0, 2
A 1 2
t y = ln t + 2 t + c, A=0

1 2
t + ln t + c, A = 2
2 2
1 t A
A + A+2 + ct , A 6= 2

y = ln t + 21 t2 + c, A=0

1 2 2
2 + t ln t + ct , A = 2
For positive A, the solution is bounded at the origin only for c = 0. For A = 0, there are no bounded solutions. For
negative A, the solution is bounded there for any value of c and thus we have a one-parameter family of solutions.
In summary, the solutions which are bounded at the origin are:

1 t2
A + A+2 ,
A>0
1 t2 A
y = A + A+2 + ct , A < 0, A 6= 2

1
2 + t2 ln t + ct2 , A = 2

Equations in the Complex Plane


Solution 14.15

1. Consider the equation w0 + sinz z w = 0. The point z = 0 is the only point we need to examine in the finite plane.
Since sinz z has a removable singularity at z = 0, there are no singular points in the finite plane. The substitution
z = 1 yields the equation
sin(1/)
u0 u = 0.

840
Since sin(1/)

has an essential singularity at = 0, the point at infinity is an irregular singular point of the original
differential equation.
2. Consider the equation w0 + z3 1
w = 0. Since z31
has a simple pole at z = 3, the differential equation has a
regular singular point there. Making the substitution z = 1/, w(z) = u()
1
u0 u=0
2 (1/ 3)
1
u0 u = 0.
(1 3)
Since this equation has a simple pole at = 0, the original equation has a regular singular point at infinity.
3. Consider the equation w0 + z 1/2 w = 0. There is an irregular singular point at z = 0. With the substitution
z = 1/, w(z) = u(),
1/2
u0 u=0
2
u0 5/2 u = 0.
We see that the point at infinity is also an irregular singular point of the original differential equation.
Solution 14.16
We start with the equation
w0 + z 2 w = 0.
P
Substituting w = z n=0 an z n , a0 6= 0 yields

!
d
X
n 2
X
z an z +z z an z n = 0
dz n=0 n=0

X
X
X
z 1 an z n + z nan z n1 + z an z n2 = 0
n=0 n=1 n=0

841
The lowest power of z in the expansion is z 2 . The coefficient of this term is a0 . Equating powers of z demands that
a0 = 0 which contradicts our initial assumption that it was nonzero. Thus we cannot find a such that the solution
can be expanded in the form,
X

w=z an z n , a0 6= 0.
n=0

842
14.12 Quiz
Problem 14.1
What is the general solution of a first order differential equation?
Solution
Problem 14.2
Write a statement about the functions P and Q to make the following statement correct.
The first order differential equation
dy
P (x, y) + Q(x, y) =0
dx
is exact if and only if . It is separable if .
Solution
Problem 14.3
Derive the general solution of
dy
+ p(x)y = f (x).
dx
Solution
Problem 14.4
Solve y 0 = y y 2 .
Solution

843
14.13 Quiz Solutions
Solution 14.1
The general solution of a first order differential equation is a one-parameter family of functions which satisfies the
equation.

Solution 14.2
The first order differential equation
dy
P (x, y) + Q(x, y) =0
dx

is exact if and only if Py = Qx . It is separable if P = P (x) and Q = Q(y).

Solution 14.3

dy
+ p(x)y = f (x)
dx
R 
We multiply by the integrating factor (x) = exp(P (x)) = exp p(x) dx , and integrate.

dy P (x)
e +p(x)y eP (x) = eP (x) f (x)
dx
d
y eP (x) = eP (x) f (x)

dx Z
y eP (x) = eP (x) f (x) dx + c
Z
P (x)
y=e eP (x) f (x) dx + c eP (x)

844
Solution 14.4
y 0 = y y 2 is separable.

y0 = y y2
y0
=1
y y2
y0 y0
=1
y y1
ln y ln(y 1) = x + c

We do algebraic simplifications and rename the constant of integration to write the solution in a nice form.
y
= c ex
y1
y = (y 1)c ex
c ex
y=
1 c ex
ex
y= x
e c
1
y=
1 c ex

845
Chapter 15

First Order Linear Systems of Differential


Equations

We all agree that your theory is crazy, but is it crazy enough?

- Niels Bohr

15.1 Introduction
In this chapter we consider first order linear systems of differential equations. That is, we consider equations of the
form,

x0 (t) = Ax(t) + f (t),



a11 a12 . . . a1n
x1 (t) a21 a22 . . . a2n
x(t) = ... , A = .. .. .

.. ..
. . . .
xn (t)
an1 an2 . . . ann

846
Initially we will consider the homogeneous problem, x0 (t) = Ax(t). (Later we will find particular solutions with variation
of parameters.) The best way to solve these equations is through the use of the matrix exponential. Unfortunately,
using the matrix exponential requires knowledge of the Jordan canonical form and matrix functions. Fortunately, we
can solve a certain class of problems using only the concepts of eigenvalues and eigenvectors of a matrix. We present
this simple method in the next section. In the following section we will take a detour into matrix theory to cover Jordan
canonical form and its applications. Then we will be able to solve the general case.

15.2 Using Eigenvalues and Eigenvectors to find Homogeneous So-


lutions
If you have forgotten what eigenvalues and eigenvectors are and how to compute them, go find a book on linear
algebra and spend a few minutes re-aquainting yourself with the rudimentary material.

Recall that the single differential equation x0 (t) = Ax has the general solution x = c eAt . Maybe the system of
differential equations
x0 (t) = Ax(t) (15.1)
has similiar solutions. Perhaps it has a solution of the form x(t) = xi et for some constant vector xi and some value
. Lets substitute this into the differential equation and see what happens.

x0 (t) = Ax(t)
xi et = Axi et
Axi = xi

We see that if is an eigenvalue of A with eigenvector xi then x(t) = xi et satisfies the differential equation. Since
the differential equation is linear, cxi et is a solution.
Suppose that the n n matrix A has the eigenvalues {k } with a complete set of linearly independent eigenvectors
{xik }. Then each of xik ek t is a homogeneous solution of Equation 15.1. We note that each of these solutions is

847
linearly independent. Without any kind of justification I will tell you that the general solution of the differential equation
is a linear combination of these n linearly independent solutions.

Result 15.2.1 Suppose that the n n matrix A has the eigenvalues {k } with a complete
set of linearly independent eigenvectors {xik }. The system of differential equations,

x0 (t) = Ax(t),

has the general solution,


n
X
x(t) = ck xik ek t
k=1

Example 15.2.1 (mathematica/ode/systems/systems.nb) Find the solution of the following initial value problem.
Describe the behavior of the solution as t .
   
0 2 1 1
x = Ax x, x(0) = x0
5 4 3

The matrix has the distinct eigenvalues 1 = 1, 2 = 3. The corresponding eigenvectors are
   
1 1
x1 = , x2 = .
1 5

The general solution of the system of differential equations is


   
1 t 1 3t
x = c1 e +c2 e .
1 5

848
We apply the initial condition to determine the constants.
    
1 1 c1 1
=
1 5 c2 3
1 1
c 1 = , c2 =
2 2
The solution subject to the initial condition is
   
1 1 t 1 1 3t
x= e + e
2 1 2 5

For large t, the solution looks like  


1 1 3t
x e .
2 5
Both coordinates tend to infinity.
Figure 15.1 shows some homogeneous solutions in the phase plane.

Example 15.2.2 (mathematica/ode/systems/systems.nb) Find the solution of the following initial value problem.
Describe the behavior of the solution as t .

1 1 2 2
0
x = Ax 0 2 2 x, x(0) = x0 0

1 1 3 1

The matrix has the distinct eigenvalues 1 = 1, 2 = 2, 3 = 3. The corresponding eigenvectors are

0 1 2
x1 = 2 , x2 = 1 ,
x3 = 2 .

1 0 1

849
10

7.5

2.5

-10 -7.5 -5 -2.5 2.5 5 7.5 10

-2.5

-5

-7.5

-10

Figure 15.1: Homogeneous solutions in the phase plane.

The general solution of the system of differential equations is


0 1 2
t 2t
x = c1 2 e +c2 1 e +c3 2 e3t .

1 0 1

850
We apply the initial condition to determine the constants.

0 1 2 c1 2
2 1 2 c2 = 0
1 0 1 c3 1
c1 = 1, c2 = 2, c3 = 0

The solution subject to the initial condition is



0 1
x = 2 et +2 1 e2t .
1 0

As t , all coordinates tend to infinity.

Exercise 15.1 (mathematica/ode/systems/systems.nb)


Find the solution of the following initial value problem. Describe the behavior of the solution as t .
   
0 1 5 1
x = Ax x, x(0) = x0
1 3 1

Hint, Solution
Exercise 15.2 (mathematica/ode/systems/systems.nb)
Find the solution of the following initial value problem. Describe the behavior of the solution as t .

3 0 2 1
x0 = Ax 1 1 0 x, x(0) = x0 0
2 1 0 0

Hint, Solution

851
Exercise 15.3
Use the matrix form of the method of variation of parameters to find the general solution of
   3 
dx 4 2 t
= x+ , t > 0.
dt 8 4 t2

Hint, Solution

15.3 Matrices and Jordan Canonical Form


Functions of Square Matrices. Consider a function f (x) with a Taylor series.

X f (n) (0)
f (x) = xn
n=0
n!

We can define the function to take square matrices as arguments. The function of the square matrix A is defined in
terms of the Taylor series.

X f (n) (0) n
f (A) = A
n=0
n!

(Note that this definition is usually not the most convenient method for computing a function of a matrix. Use the
Jordan canonical form for that.)

Eigenvalues and Eigenvectors. Consider a square matrix A. A nonzero vector x is an eigenvector of the matrix
with eigenvalue if
Ax = x.
Note that we can write this equation as
(A I)x = 0.

852
This equation has solutions for nonzero x if and only if A I is singular, (det(A I) = 0). We define the
characteristic polynomial of the matrix () as this determinant.

() = det(A I)

The roots of the characteristic polynomial are the eigenvalues of the matrix. The eigenvectors of distinct eigenvalues
are linearly independent. Thus if a matrix has distinct eigenvalues, the eigenvectors form a basis.
If is a root of () of multiplicity m then there are up to m linearly independent eigenvectors corresponding to
that eigenvalue. That is, it has from 1 to m eigenvectors.

Diagonalizing Matrices. Consider an nn matrix A that has a complete set of n linearly independent eigenvectors.
A may or may not have distinct eigenvalues. Consider the matrix S with eigenvectors as columns.

S = x1 x2 xn

A is diagonalized by the similarity transformation:

= S1 AS.

is a diagonal matrix with the eigenvalues of A as the diagonal elements. Furthermore, the k th diagonal element is
k , the eigenvalue corresponding to the the eigenvector, xk .

Generalized Eigenvectors. A vector xk is a generalized eigenvector of rank k if

(A I)k xk = 0 but (A I)k1 xk 6= 0.

Eigenvectors are generalized eigenvectors of rank 1. An nn matrix has n linearly independent generalized eigenvectors.
A chain of generalized eigenvectors generated by the rank m generalized eigenvector xm is the set: {x1 , x2 , . . . , xm },
where
xk = (A I)xk+1 , for k = m 1, . . . , 1.

853
Computing Generalized Eigenvectors. Let be an eigenvalue of multiplicity m. Let n be the smallest integer
such that
rank (nullspace ((A I)n )) = m.
Let Nk denote the number of eigenvalues of rank k. These have the value:

Nk = rank nullspace (A I)k rank nullspace (A I)k1


 
.

One can compute the generalized eigenvectors of a matrix by looping through the following three steps until all the
the Nk are zero:

1. Select the largest k for which Nk is positive. Find a generalized eigenvector xk of rank k which is linearly
independent of all the generalized eigenvectors found thus far.

2. From xk generate the chain of eigenvectors {x1 , x2 , . . . , xk }. Add this chain to the known generalized eigenvec-
tors.

3. Decrement each positive Nk by one.

Example 15.3.1 Consider the matrix


1 1 1
A = 2 1 1 .
3 2 4
The characteristic polynomial of the matrix is

1 1 1

() = 2 1 1
3 2 4
= (1 )2 (4 ) + 3 + 4 + 3(1 ) 2(4 ) + 2(1 )
= ( 2)3 .

854
Thus we see that = 2 is an eigenvalue of multiplicity 3. A 2I is

1 1 1
A 2I = 2 1 1
3 2 2

The rank of the nullspace space of A 2I is less than 3.



0 0 0
(A 2I)2 = 1 1 1
1 1 1

The rank of nullspace((A 2I)2 ) is less than 3 as well, so we have to take one more step.

0 0 0
(A 2I)3 = 0 0 0
0 0 0

The rank of nullspace((A 2I)3 ) is 3. Thus there are generalized eigenvectors of ranks 1, 2 and 3. The generalized
eigenvector of rank 3 satisfies:

(A 2I)3 x3 = 0

0 0 0
0 0 0 x3 = 0
0 0 0

We choose the solution


1
x3 = 0 .

0

855
Now to compute the chain generated by x3 .


1
x2 = (A 2I)x3 = 2
3

0
x1 = (A 2I)x2 = 1

1

Thus a set of generalized eigenvectors corresponding to the eigenvalue = 2 are


0 1 1
x1 = 1 , x2 = 2 , x3 = 0 .
1 3 0

Jordan Block. A Jordan block is a square matrix which has the constant, , on the diagonal and ones on the first
super-diagonal:

1 0 0 0
0
1 0 0
..

0 0 . 0 0

. . .. .. . . ..
.. .. . . . .

..
0 0 0 . 1
0 0 0 0

856
Jordan Canonical Form. A matrix J is in Jordan canonical form if all the elements are zero except for Jordan
blocks Jk along the diagonal.
J1 0 0 0
.
0 J2 . . 0 0

. . ..
J= .. .. ... ... .

0 0 . . . Jn1 0

0 0 0 Jn
The Jordan canonical form of a matrix is obtained with the similarity transformation:

J = S1 AS,

where S is the matrix of the generalized eigenvectors of A and the generalized eigenvectors are grouped in chains.

Example 15.3.2 Again consider the matrix



1 1 1
A = 2 1 1 .
3 2 4

Since = 2 is an eigenvalue of multiplicity 3, the Jordan canonical form of the matrix is



2 1 0
J = 0 2 1 .
0 0 2

In Example 15.3.1 we found the generalized eigenvectors of A. We define the matrix with generalized eigenvectors as
columns:
0 1 1
S = 1
2 0 .
1 3 0

857
We can verify that J = S1 AS.
J = S1 AS

0 3 2 1 1 1 0 1 1
= 0 1 1 2 1 1 1 2 0
1 1 1 3 2 4 1 3 0

2 1 0
= 0 2 1
0 0 2

Functions of Matrices in Jordan Canonical Form. The function of an n n Jordan block is the upper-
triangular matrix:
0 00 (n2) (n1)
f () f 1!() f 2!() f (n2)!() f (n1)!()

0 (n3) (n2)
f () f 1!() f (n3)!() f (n2)!()

0

. . . f (n4) () f (n3) ()
f (Jk ) = 0 0 f ()

(n4)! (n3)!

. . .
.. .. .. .. .
..
. . . .



0 . .. 0
f ()

0 0 f () 1!
0 0 0 0 f ()
The function of a matrix in Jordan canonical form is

f (J1 ) 0 0 0
.
0 f (J2 ) . . 0 0

. .. .. .. ..
f (J) = .
. . . . .
..
0 0 . f (Jn1 ) 0
0 0 0 f (Jn )
The Jordan canonical form of a matrix satisfies:
f (J) = S1 f (A)S,

858
where S is the matrix of the generalized eigenvectors of A. This gives us a convenient method for computing functions
of matrices.

Example 15.3.3 Consider the matrix exponential function eA for our old friend:

1 1 1
A= 2 1 1 .
3 2 4

In Example 15.3.2 we showed that the Jordan canonical form of the matrix is

2 1 0
J = 0 2 1 .
0 0 2

Since all the derivatives of e are just e , it is especially easy to compute eJ .


2 2 2
e e e /2
eJ = 0 e 2 e 2
0 0 e2

We find eA with a similarity transformation of eJ . We use the matrix of generalized eigenvectors found in Example 15.3.2.

eA = S eJ S1
2 2 2
0 1 1 e e e /2 0 3 2
eA = 1 2 0 0 e2 e2 0 1 1
1 3 0 0 0 e2 1 1 1

0 2 2
e2
eA = 3 1 1
2
5 3 5

859
15.4 Using the Matrix Exponential
The homogeneous differential equation
x0 (t) = Ax(t)
has the solution
x(t) = eAt c
where c is a vector of constants. The solution subject to the initial condition, x(t0 ) = x0 is

x(t) = eA(tt0 ) x0 .

The homogeneous differential equation


1
x0 (t) = Ax(t)
t
has the solution
x(t) = tA c eA Log t c,
where c is a vector of constants. The solution subject to the initial condition, x(t0 ) = x0 is
 A
t
x(t) = x0 eA Log(t/t0 ) x0 .
t0
The inhomogeneous problem
x0 (t) = Ax(t) + f (t), x(t0 ) = x0
has the solution Z t
x(t) = e A(tt0 )
x0 + e At
eA f ( ) d.
t0

Example 15.4.1 Consider the system


1 1 1
dx
= 2 1 1 x.
dt
3 2 4

860
The general solution of the system of differential equations is

x(t) = eAt c.

In Example 15.3.3 we found eA . At is just a constant times A. The eigenvalues of At are {k t} where {k } are the
eigenvalues of A. The generalized eigenvectors of At are the same as those of A.
Consider eJt . The derivatives of f () = et are f 0 () = t et and f 00 () = t2 et . Thus we have
2t
e t e2t t2 e2t /2
eJt = 0 e2t t e2t
0 0 e2t

1 t t2 /2
eJt = 0 1 t e2t
0 0 1

We find eAt with a similarity transformation.

eAt = S eJt S1

0 1 1 1 t t2 /2 0 3 2
eAt = 1 2 0 0 1 t e2t 0 1 1
1 3 0 0 0 1 1 1 1

1t t t
eAt = 2t t2 /2 1 t + t2 /2 t + t2 /2 e2t
3t + t /2 2t t /2 1 + 2t t2 /2
2 2

The solution of the system of differential equations is



1t t t
x(t) = c1 2t t2 /2 + c2 1 t + t2 /2 + c3 t + t2 /2 e2t
3t + t2 /2 2t t2 /2 1 + 2t t2 /2

861
Example 15.4.2 Consider the Euler equation system
 
dx 1 1 1 0
= Ax x.
dt t t 1 1

The solution is x(t) = tA c. Note that A is almost in Jordan canonical form. It has a one on the sub-diagonal instead
of the super-diagonal. It is clear that a function of A is defined
 
f (1) 0
f (A) = .
f 0 (1) f (1)

The function f () = t has the derivative f 0 () = t log t. Thus the solution of the system is
      
t 0 c1 t 0
x(t) = = c1 + c2
t log t t c2 t log t t

Example 15.4.3 Consider an inhomogeneous system of differential equations.


   3 
dx 4 2 t
= Ax + f (t) x+ , t > 0.
dt 8 4 t2

The general solution is Z


x(t) = e At
c+e At
eAt f (t) dt.

First we find homogeneous solutions. The characteristic equation for the matrix is

4 2
() = = 2 = 0
8 4

= 0 is an eigenvalue of multiplicity 2. Thus the Jordan canonical form of the matrix is


 
0 1
J= .
0 0

862
Since rank(nullspace(A 0I)) = 1 there is only one eigenvector. A generalized eigenvector of rank 2 satisfies

(A 0I)2 x2 = 0
 
0 0
x =0
0 0 2

We choose  
1
x2 =
0
Now we generate the chain from x2 .  
4
x1 = (A 0I)x2 =
8
We define the matrix of generalized eigenvectors S.
 
4 1
S=
8 0

The derivative of f () = et is f 0 () = t et . Thus


 
Jt 1 t
e =
0 1

The homogeneous solution of the differential equation system is xh = eAt c where

eAt = S eJt S1
    
4 1 1 t 0 1/8
eAt = .
8 0 0 1 1 1/2
 
At 1 + 4t 2t
e =
8t 1 4t

863
The general solution of the inhomogeneous system of equations is
Z
At
x(t) = e c + e At
eAt f (t) dt
   Z    3 
1 + 4t 2t 1 + 4t 2t 1 4t 2t t
x(t) = c+ dt
8t 1 4t 8t 1 4t 8t 1 + 4t t2
2 2 Log t + 6t 2t12
     
1 + 4t 2t
x(t) = c1 + c2 +
8t 1 4t 4 4 Log t + 13t

We can tidy up the answer a little bit. First we take linear combinations of the homogeneous solutions to obtain a
simpler form.

2 2 Log t + 6t 2t12
     
1 2t
x(t) = c1 + c2 +
2 4t 1 4 4 Log t + 13t

Then we subtract 2 times the first homogeneous solution from the particular solution.

2 Log t + 6t 2t12
     
1 2t
x(t) = c1 + c2 +
2 4t 1 4 Log t + 13t

864
15.5 Exercises
Exercise 15.4 (mathematica/ode/systems/systems.nb)
Find the solution of the following initial value problem.
   
0 2 1 1
x = Ax x, x(0) = x0
5 4 3
Hint, Solution
Exercise 15.5 (mathematica/ode/systems/systems.nb)
Find the solution of the following initial value problem.

1 1 2 2
x0 = Ax 0 2 2 x, x(0) = x0 0
1 1 3 1
Hint, Solution
Exercise 15.6 (mathematica/ode/systems/systems.nb)
Find the solution of the following initial value problem. Describe the behavior of the solution as t .
   
0 1 5 1
x = Ax x, x(0) = x0
1 3 1
Hint, Solution
Exercise 15.7 (mathematica/ode/systems/systems.nb)
Find the solution of the following initial value problem. Describe the behavior of the solution as t .

3 0 2 1
0
x = Ax 1 1 0 x, x(0) = x0 0

2 1 0 0
Hint, Solution

865
Exercise 15.8 (mathematica/ode/systems/systems.nb)
Find the solution of the following initial value problem. Describe the behavior of the solution as t .
   
0 1 4 3
x = Ax x, x(0) = x0
4 7 2
Hint, Solution
Exercise 15.9 (mathematica/ode/systems/systems.nb)
Find the solution of the following initial value problem. Describe the behavior of the solution as t .

1 0 0 1
x0 = Ax 4 1 0 x, x(0) = x0 2
3 6 2 30
Hint, Solution
Exercise 15.10
1. Consider the system
1 1 1
x0 = Ax = 2 1 1 x. (15.2)
3 2 4
(a) Show that = 2 is an eigenvalue of multiplicity 3 of the coefficient matrix A, and that there is only one
corresponding eigenvector, namely
0
(1)
xi = 1 .
1
(b) Using the information in part (i), write down one solution x(1) (t) of the system (15.2). There is no other
solution of a purely exponential form x = xi et .
(c) To find a second solution use the form x = xit e2t + e2t , and find appropriate vectors xi and . This gives
a solution of the system (15.2) which is independent of the one obtained in part (ii).

866
(d) To find a third linearly independent solution use the form x = xi(t2 /2) e2t +t e2t + e2t . Show that xi,
and satisfy the equations

(A 2I)xi = 0, (A 2I) = xi, (A 2I) = .

The first two equations can be taken to coincide with those obtained in part (iii). Solve the third equation,
and write down a third independent solution of the system (15.2).

2. Consider the system



5 3 2
x0 = Ax = 8 5 4 x. (15.3)
4 3 3

(a) Show that = 1 is an eigenvalue of multiplicity 3 of the coefficient matrix A, and that there are only two
linearly independent eigenvectors, which we may take as

1 0
xi(1) = 0 , xi(2) = 2
2 3

Find two independent solutions of equation (15.3).


(b) To find a third solution use the form x = xit et +et ; then show that xi and must satisfy

(A I)xi = 0, (A I) = xi.

Show that the most general solution of the first of these equations is xi = c1 xi1 + c2 xi2 , where c1 and c2
are arbitrary constants. Show that, in order to solve the second of these equations it is necessary to take
c1 = c2 . Obtain such a vector , and use it to obtain a third independent solution of the system (15.3).
Hint, Solution

867
Exercise 15.11 (mathematica/ode/systems/systems.nb)
Consider the system of ODEs
dx
= Ax, x(0) = x0
dt
where A is the constant 3 3 matrix
1 1 1
A= 2 1 1
8 5 3

1. Find the eigenvalues and associated eigenvectors of A. [HINT: notice that = 1 is a root of the characteristic
polynomial of A.]

2. Use the results from part (a) to construct eAt and therefore the solution to the initial value problem above.

3. Use the results of part (a) to find the general solution to

dx 1
= Ax.
dt t

Hint, Solution
Exercise 15.12 (mathematica/ode/systems/systems.nb)
1. Find the general solution to
dx
= Ax
dt
where
2 0 1
A = 0 2 0
0 1 3

2. Solve
dx
= Ax + g(t), x(0) = 0
dt

868
using A from part (a).
Hint, Solution
Exercise 15.13
Let A be an n n matrix of constants. The system

dx 1
= Ax, (15.4)
dt t
is analogous to the Euler equation.

1. Verify that when A is a 2 2 constant matrix, elimination of (15.4) yields a second order Euler differential
equation.

2. Now assume that A is an n n matrix of constants. Show that this system, in analogy with the Euler equation
has solutions of the form x = at where a is a constant vector provided a and satisfy certain conditions.

3. Based on your experience with the treatment of multiple roots in the solution of constant coefficient systems,
what form will the general solution of (15.4) take if is a multiple eigenvalue in the eigenvalue problem derived
in part (b)?

4. Verify your prediction by deriving the general solution for the system
 
dx 1 1 0
= x.
dt t 1 1

Hint, Solution

869
15.6 Hints
Hint 15.1

Hint 15.2

Hint 15.3

Hint 15.4

Hint 15.5

Hint 15.6

Hint 15.7

Hint 15.8

Hint 15.9

Hint 15.10

870
Hint 15.11

Hint 15.12

Hint 15.13

871
15.7 Solutions
Solution 15.1
We consider an initial value problem.
   
0 1 5 1
x = Ax x, x(0) = x0
1 3 1
The matrix has the distinct eigenvalues 1 = 1 , 2 = 1 + . The corresponding eigenvectors are
   
2 2+
x1 = , x2 = .
1 1
The general solution of the system of differential equations is
   
2 (1)t 2 + (1+)t
x = c1 e +c2 e .
1 1
We can take the real and imaginary parts of either of these solution to obtain real-valued solutions.
     
2 + (1+)t 2 cos(t) sin(t) t cos(t) + 2 sin(t) t
e = e + e
1 cos(t) sin(t)
   
2 cos(t) sin(t) t cos(t) + 2 sin(t) t
x = c1 e +c2 e
cos(t) sin(t)
We apply the initial condition to determine the constants.
    
2 1 c1 1
=
1 0 c2 1
c1 = 1, c2 = 1
The solution subject to the initial condition is
 
cos(t) 3 sin(t) t
x= e .
cos(t) sin(t)
Plotted in the phase plane, the solution spirals in to the origin as t increases. Both coordinates tend to zero as t .

872
Solution 15.2
We consider an initial value problem.


3 0 2 1
x0 = Ax 1 1 0 x, x(0) = x0 0

2 1 0 0


The matrix has the distinct eigenvalues 1 = 2, 2 = 1 2, 3 = 1 + 2. The corresponding eigenvectors
are

2 2 + 2 2 2
x1 = 2 , x2 = 1 + 2 , x3 = 1 2 .
1 3 3

The general solution of the system of differential equations is


2 2 + 2 2 2
x = c1 2 e2t +c2 1 + 2 e(1 2)t +c3 1 2 e(1+ 2)t .
1 3 3

We can take the real and imaginary parts of the second or third solution to obtain two real-valued solutions.


2 + 2 2 cos( 2t) + 2 sin( 2t) 2 cos( 2t) 2 sin(
2t)
1 + 2 e(1 2)t = cos( 2t) + 2 sin( 2t) et + 2 cos( 2t) + sin( 2t) et

3 3 cos( 2t) 3 sin( 2t)

2 2 cos( 2t) + 2 sin( 2t) 2 cos( 2t) 2 sin( 2t) t
2t t
x = c1 2 e +c2 cos( 2t) + 2 sin( 2t) e +c3
2 cos( 2t) + sin( 2t) e

1 3 cos( 2t) 3 sin( 2t)

873
We apply the initial condition to determine the constants.

2 2 2 c1 1
2 1 2 c2 = 0
1 3 0 c3 0
1 1 5
c 1 = , c 2 = , c3 =
3 9 9 2
The solution subject to the initial condition is

2 2 cos( 2t) 4 2 sin( 2t)
1 1
x = 2 e2t + 4 cos( 2t) + 2 sin( 2t) et .
3 6
1 2 cos( 2t) 5 2 sin( 2t)

As t , all coordinates tend to infinity. Plotted in the phase plane, the solution would spiral in to the origin.
Solution 15.3
Homogeneous Solution, Method 1. We designate the inhomogeneous system of differential equations
x0 = Ax + g(t).
First we find homogeneous solutions. The characteristic equation for the matrix is

4 2
() = = 2 = 0
8 4
= 0 is an eigenvalue of multiplicity 2. The eigenvectors satisfy
    
4 2 1 0
= .
8 4 2 0
Thus we see that there is only one linearly independent eigenvector. We choose
 
1
xi = .
2

874
One homogeneous solution is then    
1 0t 1
x1 = e = .
2 2
We look for a second homogeneous solution of the form

x2 = xit + .

We substitute this into the homogeneous equation.

x02 = Ax2
xi = A(xit + )

We see that xi and satisfy


Axi = 0, A = xi.
We choose xi to be the eigenvector that we found previously. The equation for is then
    
4 2 1 1
= .
8 4 2 2

is determined up to an additive multiple of xi. We choose


 
0
= .
1/2

Thus a second homogeneous solution is    


1 0
x2 = t+ .
2 1/2
The general homogeneous solution of the system is
   
1 t
xh = c1 + c2
2 2t 1/2

875
We can write this in matrix notation using the fundamental matrix (t).
  
1 t c1
xh = (t)c =
2 2t 1/2 c2
Homogeneous Solution, Method 2. The similarity transform c1 Ac with
 
1 0
c=
2 1/2
will convert the matrix  
4 2
A=
8 4
to Jordan canonical form. We make the change of variables,
 
1 0
y= x.
2 1/2
The homogeneous system becomes
   
dy 1 0 4 2 1 0
= y
dt 4 2 8 4 2 1/2
 0   
y1 0 1 y1
0 =
y2 0 0 y2
The equation for y2 is
y20 = 0.
y2 = c2
The equation for y1 becomes
y10 = c2 .
y1 = c1 + c2 t

876
The solution for y is then    
1 t
y = c1 + c2 .
0 1
We multiply this by c to obtain the homogeneous solution for x.
   
1 t
xh = c1 + c2
2 2t 1/2

Inhomogeneous Solution. By the method of variation of parameters, a particular solution is


Z
xp = (t) 1 (t)g(t) dt.
 Z    3 
1 t 1 4t 2t t
xp = dt
2 2t 1/2 4 2 t2
2t1 4t2 + t3
 Z  
1 t
xp = dt
2 2t 1/2 2t2 + 4t3
2 log t + 4t1 21 t2
  
1 t
xp =
2 2t 1/2 2t1 2t2
2 2 log t + 2t1 21 t2
 
xp =
4 4 log t + 5t1

By adding 2 times our first homogeneous solution, we obtain

2 log t + 2t1 12 t2
 
xp =
4 log t + 5t1

The general solution of the system of differential equations is

2 log t + 2t1 21 t2
     
1 t
x = c1 + c2 +
2 2t 1/2 4 log t + 5t1

877
Solution 15.4
We consider an initial value problem.
   
0 2 1 1
x = Ax x, x(0) = x0
5 4 3
The Jordan canonical form of the matrix is
 
1 0
J= .
0 3
The solution of the initial value problem is x = eAt x0 .
x = eAt x0
= S eJt S1 x0
   t    
1 1 e 0 1 5 1 1
= 3t
1 5 0 e 4 1 1 3
 t 3t

1 e +e
=
2 et +5 e3t
   
1 1 t 1 1 3t
x= e + e
2 1 2 5

Solution 15.5
We consider an initial value problem.

1 1 2 2
x0 = Ax 0 2 2 x, x(0) = x0 0
1 1 3 1
The Jordan canonical form of the matrix is

1 0 0
J = 0 2 0 .
0 0 3

878
The solution of the initial value problem is x = eAt x0 .

x = eAt x0
= S eJt S1 x0
t
0 1 2 e 0 0 1 1 0 2
1
= 2 1 2 0 e2t 0 4 2 4 0
2
1 0 1 0 0 e3t 1 1 2 1
2t

2e
= 2 et +2 e2t

et



0 2
t
x = 2
e + 2 e2t .

1 0

Solution 15.6
We consider an initial value problem.

   
0 1 5 1
x = Ax x, x(0) = x0
1 3 1

The Jordan canonical form of the matrix is

 
1 0
J= .
0 1 +

879
The solution of the initial value problem is x = eAt x0 .

x = eAt x0
= S eJt S1 x0
   (1)t    
2 2+ e 0 1 1 2 1
= (1+)t
1 1 0 e 2 1 + 2 1
t
 
(cos(t) 3 sin(t)) e
=
(cos(t) sin(t)) et

   
1 t 3 t
x= e cos(t) e sin(t)
1 1

Solution 15.7
We consider an initial value problem.


3 0 2 1
x0 = Ax 1 1 0 x, x(0) = x0 0

2 1 0 0

The Jordan canonical form of the matrix is


2 0 0
J = 0 1 2 0 .
0 0 1 + 2

880
The solution of the initial value problem is x = eAt x0 .

x = eAt x0
= S eJt S1 x0
2t
e

6 2 + 2 2 2 0 0
1
= 6 1 + 2 1 2 0 e(1 2)t 0
3 (1+ 2)t
3 3 3 0 0 e

2 2 2 1
1
1 52/2 1 22 4 + 2 0
6
1 + 5 2/2 1 + 2 2 4 2 0



2 2 cos( 2t) 4 2 sin( 2t)
1 1
x = 2 e2t + 4 cos( 2t) + 2 sin( 2t) et .
3 6
1 2 cos( 2t) 5 2 sin( 2t)

Solution 15.8
We consider an initial value problem.

   
0 1 4 3
x = Ax x, x(0) = x0
4 7 2

Method 1. Find Homogeneous Solutions. The matrix has the double eigenvalue 1 = 2 = 3. There is only

881
one corresponding eigenvector. We compute a chain of generalized eigenvectors.

(A + 3I)2 x2 = 0
0x2 = 0
 
1
x2 =
0
(A + 3I)x2 = x1
 
4
x1 =
4

The general solution of the system of differential equations is


     
1 3t 4 1
x = c1 e +c2 t+ e3t .
1 4 0

We apply the initial condition to determine the constants.


    
1 1 c1 3
=
1 0 c2 2
c1 = 2, c2 = 1

The solution subject to the initial condition is


 
3 + 4t 3t
x= e .
2 + 4t

Both coordinates tend to zero as t .


Method 2. Use the Exponential Matrix. The Jordan canonical form of the matrix is
 
3 1
J= .
0 3

882
The solution of the initial value problem is x = eAt x0 .

x = eAt x0
= S eJt S1 x0
  3t
t e3t
   
1 1/4 e 0 1 3
=
1 0 0 e3t 4 4 2

 
3 + 4t 3t
x= e .
2 + 4t

Solution 15.9
We consider an initial value problem.

1 0 0 1
x0 = Ax 4 1 0 x, x(0) = x0 2
3 6 2 30

Method 1. Find Homogeneous Solutions. The matrix has the distinct eigenvalues 1 = 1, 2 = 1, 3 = 2.
The corresponding eigenvectors are

1 0 0
x1 = 2 , x2 = 1 , x3 = 0 .
5 6 1

The general solution of the system of differential equations is



1 0 0
t
x = c1 2 e +c2 1 e +c3 0 e2t .
t
5 6 1

883
We apply the initial condition to determine the constants.

1 0 0 c1 1
2 1 0 c2 = 2
5 6 1 c3 30
c1 = 1, c2 = 4, c3 = 11
The solution subject to the initial condition is

1 0 0
t t
x = 2 e 4 1
e 11 0 e2t .

5 6 1

As t , the first coordinate vanishes, the second coordinate tends to and the third coordinate tends to
Method 2. Use the Exponential Matrix. The Jordan canonical form of the matrix is

1 0 0
J = 0 1 0 .
0 0 2

The solution of the initial value problem is x = eAt x0 .


x = eAt x0
= S eJt S1 x0
t
1 0 0 e 0 0 1 0 0 1
1
= 2 1 0 0 et 0 2 1 0 2
2
5 6 1 0 0 e2t 7 6 1 30

1 0 0
x = 2 et 4 1 et 11 0 e2t .
5 6 1

884
Solution 15.10
1. (a) We compute the eigenvalues of the matrix.

1 1 1

() = 2 1 1 = 3 + 62 12 + 8 = ( 2)3
3 2 4
= 2 is an eigenvalue of multiplicity 3. The rank of the null space of A 2I is 1. (The first two rows are
linearly independent, but the third is a linear combination of the first two.)

1 1 1
A 2I = 2 1 1
3 2 2
Thus there is only one eigenvector.

1 1 1 1
2 1 1 2 = 0
3 2 2 3

0
xi(1) = 1
1
(b) One solution of the system of differential equations is

0
x(1) = 1 e2t .
1
(c) We substitute the form x = xit e2t + e2t into the differential equation.
x0 = Ax
xi e2t +2xit e2t +2 e2t = Axit e2t +A e2t
(A 2I)xi = 0, (A 2I) = xi

885
We already have a solution of the first equation, we need the generalized eigenvector . Note that is only
determined up to a constant times xi. Thus we look for the solution whose second component vanishes to
simplify the algebra.

(A 2I) = xi

1 1 1 1 0
2 1 1 0 = 1
3 2 2 3 1
1 + 3 = 0, 21 3 = 1, 31 + 23 = 1

1
= 0

1

A second linearly independent solution is


0 1
x(2) = 1 t e2t + 0 e2t .
1 1

(d) To find a third solution we substutite the form x = xi(t2 /2) e2t +t e2t + e2t into the differential equation.

x0 = Ax
2xi(t2 /2) e2t +(xi + 2)t e2t +( + 2) e2t = Axi(t2 /2) e2t +At e2t +A e2t
(A 2I)xi = 0, (A 2I) = xi, (A 2I) =

We have already solved the first two equations, we need the generalized eigenvector . Note that is only
determined up to a constant times xi. Thus we look for the solution whose second component vanishes to

886
simplify the algebra.

(A 2I) =

1 1 1 1 1
2 1 1 0 = 0
3 2 2 3 1
1 + 3 = 1, 21 3 = 0, 31 + 23 = 1

1
= 0

2

A third linearly independent solution is



0 1 1
x(3) = 1 (t /2) + 0 t + 0 e2t
2
e 2t e 2t
1 1 2

2. (a) We compute the eigenvalues of the matrix.



5 3 2

() = 8 5 4 = 3 + 32 3 + 1 = ( 1)3
4 3 3

= 1 is an eigenvalue of multiplicity 3. The rank of the null space of A I is 2. (The second and third
rows are multiples of the first.)

4 3 2
A I = 8 6 4
4 3 2

887
Thus there are two eigenvectors.

4 3 2 1
8 6 4 2 = 0
4 3 2 3

1 0
(1) (2)
xi = 0 , xi =
2
2 3
Two linearly independent solutions of the differential equation are

1 0
(1) t (2)
x = 0 e , x = 2 et .
2 3

(b) We substitute the form x = xit et + et into the differential equation.


x0 = Ax
xi et +xit et + et = Axit et +A et
(A I)xi = 0, (A I) = xi
The general solution of the first equation is a linear combination of the two solutions we found in the previous
part.
xi = c1 xi1 + c2 xi2
Now we find the generalized eigenvector, . Note that is only determined up to a linear combination of
xi1 and xi2 . Thus we can take the first two components of to be zero.

4 3 2 0 1 0
8 6 4 0 = c1 0 + c2 2
4 3 2 3 2 3
23 = c1 , 43 = 2c2 , 23 = 2c1 3c2
c1
c1 = c2 , 3 =
2

888
We see that we must take c1 = c2 in order to obtain a solution. We choose c1 = c2 = 2 A third linearly
independent solution of the differential equation is

2 0
(3)
x = 4 t + 0 et .
e t
2 1

Solution 15.11
1. The characteristic polynomial of the matrix is

1 1 1

() = 2 1 1
8 5 3
= (1 )2 (3 ) + 8 10 5(1 ) 2(3 ) 8(1 )
= 3 2 + 4 + 4
= ( + 2)( + 1)( 2)
Thus we see that the eigenvalues are = 2, 1, 2. The eigenvectors xi satisfy
(A I)xi = 0.

For = 2, we have
(A + 2I)xi = 0.

3 1 1 1 0
2 3 1 2 = 0

8 5 1 3 0
If we take 3 = 1 then the first two rows give us the system,
    
3 1 1 1
=
2 3 2 1

889
which has the solution 1 = 4/7, 2 = 5/7. For the first eigenvector we choose:

4
xi = 5
7

For = 1, we have

(A + I)xi = 0.

2 1 1 1 0
2 2 1 2 = 0

8 5 2 3 0

If we take 3 = 1 then the first two rows give us the system,


    
2 1 1 1
=
2 2 2 1

which has the solution 1 = 3/2, 2 = 2. For the second eigenvector we choose:

3
xi = 4
2

For = 2, we have

(A + I)xi = 0.

1 1 1 1 0
2 1 1 2 = 0
8 5 5 3 0

890
If we take 3 = 1 then the first two rows give us the system,
    
1 1 1 1
=
2 1 2 1
which has the solution 1 = 0, 2 = 1. For the third eigenvector we choose:

0
xi = 1

1

In summary, the eigenvalues and eigenvectors are



4 3 0
= {2, 1, 2}, xi = 5 , 4 , 1

7 2 1

2. The matrix is diagonalized with the similarity transformation

J = S1 AS,

where S is the matrix with eigenvectors as columns:



4 3 0
S= 5 4 1
7 2 1

The matrix exponential, eAt is given by


eA = S eJ S1 .
2t
4 3 0 e 0 0 6 3 3
1
eA = 5 4 1 0 et 0 12 4 4 .
2t 12
7 2 1 0 0 e 18 13 1

891
2 e2t +3 et e2t + et e2t + et

2t 8 et +3 et 15 e2t 16 et +13 et 15 e2t 16 et + et
eAt = 5 e 2 12 12
7 e2t 4 et 3 et 21 e2t 8 et 13 et 21 e2t 8 et et
2 12 12

The solution of the initial value problem is eAt x0 .

3. The general solution of the Euler equation is



4 3 0
2 1
c1 5 t + c2 4 t + c3 1 t2 .

7 2 1

We could also write the solution as


x = tA c eA log t c,

Solution 15.12
1. The characteristic polynomial of the matrix is

2 0 1

() = 0 2 0
0 1 3
= (2 )2 (3 )

Thus we see that the eigenvalues are = 2, 2, 3. Consider



0 0 1
A 2I = 0 0 0 .
0 1 3

Since rank(nullspace(A 2I)) = 1 there is one eigenvector and one generalized eigenvector of rank two for

892
= 2. The generalized eigenvector of rank two satisfies

(A 2I)2 xi2 = 0

0 1 1
0 0 0 xi2 = 0
0 1 1
We choose the solution
0
xi2 = 1 .
1
The eigenvector for = 2 is
1
xi1 = (A 2I)xi2 = 0 .
0
The eigenvector for = 3 satisfies

(A 3I)2 xi = 0

1 0 1
0 1 0 xi = 0
0 1 0
We choose the solution
1
xi = 0 .

1
The eigenvalues and generalized eigenvectors are

1 0 1
= {2, 2, 3}, xi = 0 , 1 , 0 .

0 1 1

893
The matrix of eigenvectors and its inverse is

1 0 1 1 1 1
S = 0 1 0 , S1 = 0 1 0 .
0 1 1 0 1 1

The Jordan canonical form of the matrix, which satisfies J = S1 AS is



2 1 0
J = 0 2 0
0 0 3

Recall that the function of a Jordan block is:


f 0 () f 00 () f 000 ()

1 0 0 f () 1! 2! 3!
0 f 0 () f 00 ()
1 0 = 0
f ()
f 1! 0 () ,
2!

0 0 1 f
0 0 f () 1!

0 0 0 0 0 0 f ()

and that the function of a matrix in Jordan canonical form is



J1 0 0 0 f (J1 ) 0 0 0
0 J2 0 0 = 0 f (J2 ) 0 0

f
0
.
0 J 3 0 0 0 f (J3 ) 0
0 0 0 J4 0 0 0 f (J4 )

We want to compute eJt so we consider the function f () = et , which has the derivative f 0 () = t et . Thus
we see that 2t
e t e2t 0
eJt = 0 e2t 0
0 0 e3t

894
The exponential matrix is

eAt = S eJt S1 ,

e2t (1 + t) e2t + e3t e2t + e3t
eAt =0 e2t 0 .
2t 3t 3t
0 e +e e

The general solution of the homogeneous differential equation is

x = eAt c.

2. The solution of the inhomogeneous differential equation subject to the initial condition is
Z t
x=e At
0+e At
eA g( ) d
0
Z t
x = eAt eA g( ) d
0

Solution 15.13
1.

dx 1
= Ax
 0 dt  t   
x a b x1
t 10 =
x2 c d x2

The first component of this equation is


tx01 = ax1 + bx2 .

895
We differentiate and multiply by t to obtain a second order coupled equation for x1 . We use (15.4) to eliminate
the dependence on x2 .

t2 x001 + tx01 = atx01 + btx02


t2 x001 + (1 a)tx01 = b(cx1 + dx2 )
t2 x001 + (1 a)tx01 bcx1 = d(tx01 ax1 )
t2 x001 + (1 a d)tx01 + (ad bc)x1 = 0

Thus we see that x1 satisfies a second order, Euler equation. By symmetry we see that x2 satisfies,

t2 x002 + (1 b c)tx02 + (bc ad)x2 = 0.

2. We substitute x = at into (15.4).


1
at1 = Aat
t
Aa = a

Thus we see that x = at is a solution if is an eigenvalue of A with eigenvector a.


3. Suppose that = is an eigenvalue of multiplicity 2. If = has two linearly independent eigenvectors, a and
b then at and bt are linearly independent solutions. If = has only one linearly independent eigenvector,
a, then at is a solution. We look for a second solution of the form

x = xit log t + t .

Substituting this into the differential equation yields

xit1 log t + xit1 + t1 = Axit1 log t + At1

We equate coefficients of t1 log t and t1 to determine xi and .

(A I)xi = 0, (A I) = xi

896
These equations have solutions because = has generalized eigenvectors of first and second order.
Note that the change of independent variable = log t, y( ) = x(t), will transform (15.4) into a constant
coefficient system.
dy
= Ay
d
Thus all the methods for solving constant coefficient systems carry over directly to solving (15.4). In the case of
eigenvalues with multiplicity greater than one, we will have solutions of the form,

xit , xit log t + t , xit (log t)2 + t log t + t , ...,

analogous to the form of the solutions for a constant coefficient system,

xi e , xi e + e , xi 2 e + e + e , ....

4. Method 1. Now we consider  


dx 1 1 0
= x.
dt t 1 1
The characteristic polynomial of the matrix is

1 0
() = = (1 )2 .
1 1

= 1 is an eigenvalue of multiplicity 2. The equation for the associated eigenvectors is


    
0 0 1 0
= .
1 0 2 0

There is only one linearly independent eigenvector, which we choose to be


 
0
a= .
1

897
One solution of the differential equation is  
0
x1 = t.
1
We look for a second solution of the form
x2 = at log t + t.
satisfies the equation    
0 0 0
(A I) = = .
1 0 1
The solution is determined only up to an additive multiple of a. We choose
 
1
= .
0
Thus a second linearly independent solution is
   
0 1
x2 = t log t + t.
1 0
The general solution of the differential equation is
      
0 0 1
x = c1 t + c2 t log t + t .
1 1 0

Method 2. Note that the matrix is lower triangular.


 0   
x1 1 1 0 x1
= (15.5)
x02 t 1 1 x2
We have an uncoupled equation for x1 .
1
x01 = x1
t
x1 = c1 t

898
By substituting the solution for x1 into (15.5), we obtain an uncoupled equation for x2 .
1
x02 = (c1 t + x2 )
t
1
x02 x2 = c1
 t0
1 c1
x2 =
t t
1
x2 = c1 log t + c2
t
x2 = c1 t log t + c2 t

Thus the solution of the system is


 
c1 t
x= ,
c1 t log t + c2 t
   
t 0
x = c1 + c2 ,
t log t t

which is equivalent to the solution we obtained previously.

899
Chapter 16

Theory of Linear Ordinary Differential


Equations

A little partyin is good for the soul.

-Matt Metz

16.1 Exact Equations


Exercise 16.1
Consider a second order, linear, homogeneous differential equation:

P (x)y 00 + Q(x)y 0 + R(x)y = 0. (16.1)

Show that P 00 Q0 + R = 0 is a necessary and sufficient condition for this equation to be exact.
Hint, Solution
Exercise 16.2
Determine an equation for the integrating factor (x) for Equation 16.1.

900
Hint, Solution
Exercise 16.3
Show that
y 00 + xy 0 + y = 0
is exact. Find the solution.
Hint, Solution

16.2 Nature of Solutions

Result 16.2.1 Consider the nth order ordinary differential equation of the form
dn y dn1 y dy
L[y] = n + pn1 (x) n1 + + p1 (x) + p0 (x)y = f (x). (16.2)
dx dx dx
If the coefficient functions pn1 (x), . . . , p0 (x) and the inhomogeneity f (x) are continuous on
some interval a < x < b then the differential equation subject to the conditions,

y(x0 ) = v0 , y 0 (x0 ) = v1 , ... y (n1) (x0 ) = vn1 , a < x0 < b,

has a unique solution on the interval.

Exercise 16.4
On what intervals do the following problems have unique solutions?

1. xy 00 + 3y = x

2. x(x 1)y 00 + 3xy 0 + 4y = 2

901
3. ex y 00 + x2 y 0 + y = tan x

Hint, Solution

Linearity of the Operator. The differential operator L is linear. To verify this,

dn dn1 d
L[cy] = n (cy) + pn1 (x) n1 (cy) + + p1 (x) (cy) + p0 (x)(cy)
dx dx dx
dn dn1 d
= c n y + cpn1 (x) n1 y + + cp1 (x) y + cp0 (x)y
dx dx dx
= cL[y]
dn dn1 d
L[y1 + y2 ] = n (y1 + y2 ) + pn1 (x) n1 (y1 + y2 ) + + p1 (x) (y1 + y2 ) + p0 (x)(y1 + y2 )
dx dx dx
dn dn1 d
= n (y1 ) + pn1 (x) n1 (y1 ) + + p1 (x) (y1 ) + p0 (x)(y1 )
dx dx dx
dn dn1 d
+ n (y2 ) + pn1 (x) n1 (y2 ) + + p1 (x) (y2 ) + p0 (x)(y2 )
dx dx dx
= L[y1 ] + L[y2 ].

Homogeneous Solutions. The general homogeneous equation has the form

dn y dn1 y dy
L[y] = n
+ p n1 (x) n1
+ + p1 (x) + p0 (x)y = 0.
dx dx dx
From the linearity of L, we see that if y1 and y2 are solutions to the homogeneous equation then c1 y1 + c2 y2 is also a
solution, (L[c1 y1 + c2 y2 ] = 0).
On any interval where the coefficient functions are continuous, the nth order linear homogeneous equation has n
linearly independent solutions, y1 , y2 , . . . , yn . (We will study linear independence in Section 16.4.) The general solution
to the homogeneous problem is then
yh = c1 y1 + c2 y2 + + cn yn .

902
Particular Solutions. Any function, yp , that satisfies the inhomogeneous equation, L[yp ] = f (x), is called a
particular solution or particular integral of the equation. Note that for linear differential equations the particular solution
is not unique. If yp is a particular solution then yp +yh is also a particular solution where yh is any homogeneous solution.
The general solution to the problem L[y] = f (x) is the sum of a particular solution and a linear combination of the
homogeneous solutions
y = yp + c1 y1 + + cn yn .

Example 16.2.1 Consider the differential equation

y 00 y 0 = 1.

You can verify that two homogeneous solutions are ex and 1. A particular solution is x. Thus the general solution is

y = x + c1 ex +c2 .
Exercise 16.5
Suppose you are able to find three linearly independent particular solutions u1 (x), u2 (x) and u3 (x) of the second order
linear differential equation L[y] = f (x). What is the general solution?
Hint, Solution

Real-Valued Solutions. If the coefficient function and the inhomogeneity in Equation 16.2 are real-valued, then
the general solution can be written in terms of real-valued functions. Let y be any, homogeneous solution, (perhaps
complex-valued). By taking the complex conjugate of the equation L[y] = 0 we show that y is a homogeneous solution
as well.

L[y] = 0
L[y] = 0
y (n) + pn1 y (n1) + + p0 y = 0
y (n) + pn1 y (n1) + + p0 y = 0
L [y] = 0

903
For the same reason, if yp is a particular solution, then yp is a particular solution as well.
Since the real and imaginary parts of a function y are linear combinations of y and y,

y + y y y
<(y) = , =(y) = ,
2 2

if y is a homogeneous solution then both <y and =(y) are homogeneous solutions. Likewise, if yp is a particular solution
then <(yp ) is a particular solution.

 
yp + yp f f
L [<(yp )] = L = + =f
2 2 2

Thus we see that the homogeneous solution, the particular solution and the general solution of a linear differential
equation with real-valued coefficients and inhomogeneity can be written in terms of real-valued functions.

Result 16.2.2 The differential equation


dn y dn1 y dy
L[y] = n + pn1 (x) n1 + + p1 (x) + p0 (x)y = f (x)
dx dx dx
with continuous coefficients and inhomogeneity has a general solution of the form

y = yp + c1 y1 + + cn yn

where yp is a particular solution, L[yp ] = f , and the yk are linearly independent homogeneous
solutions, L[yk ] = 0. If the coefficient functions and inhomogeneity are real-valued, then the
general solution can be written in terms of real-valued functions.

904
16.3 Transformation to a First Order System
Any linear differential equation can be put in the form of a system of first order differential equations. Consider
y (n) + pn1 y (n1) + + p0 y = f (x).
We introduce the functions,
y1 = y, y2 = y 0 , ,..., yn = y (n1) .
The differential equation is equivalent to the system
y10 = y2
y20 = y3
.. ..
.=.
0
yn = f (x) pn1 yn p0 y1 .
The first order system is more useful when numerically solving the differential equation.

Example 16.3.1 Consider the differential equation


y 00 + x2 y 0 + cos x y = sin x.
The corresponding system of first order equations is
y10 = y2
y20 = sin x x2 y2 cos x y1 .

16.4 The Wronskian


16.4.1 Derivative of a Determinant.
Before investigating the Wronskian, we will need a preliminary result from matrix theory. Consider an n n matrix A
whose elements aij (x) are functions of x. We will denote the determinant by [A(x)]. We then have the following
theorem.

905
Result 16.4.1 Let aij (x), the elements of the matrix A, be differentiable functions of x.
Then n
d X
[A(x)] = k [A(x)]
dx
k=1

where k [A(x)] is the determinant of the matrix A with the k th row replaced by the derivative
of the k th row.

Example 16.4.1 Consider the the matrix  


x x2
A(x) =
x2 x4
The determinant is x5 x4 thus the derivative of the determinant is 5x4 4x3 . To check the theorem,

d d x x2
[A(x)] =
dx dx x2 x4

1 2x x x2
= 2 4 +
x x 2x 4x3
= x4 2x3 + 4x4 2x3
= 5x4 4x3 .

16.4.2 The Wronskian of a Set of Functions.


A set of functions {y1 , y2 , . . . , yn } is linearly dependent on an interval if there are constants c1 , . . . , cn not all zero such
that
c1 y1 + c2 y2 + + cn yn = 0 (16.3)
identically on the interval. The set is linearly independent if all of the constants must be zero to satisfy c1 y1 + cn yn = 0
on the interval.

906
Consider a set of functions {y1 , y2 , . . . , yn } that are linearly dependent on a given interval and n 1 times differ-
entiable. There are a set of constants, not all zero, that satisfy equation 16.3
Differentiating equation 16.3 n 1 times gives the equations,
c1 y10 + c2 y20 + + cn yn0 = 0
c1 y100 + c2 y200 + + cn yn00 = 0

(n1) (n1)
c1 y1 + c2 y2 + + cn yn(n1) = 0.
We could write the problem to find the constants as

y1 y2 ... yn c1
y10 y20 ... yn0 c2
00
y
1 y200 ... yn00 c3
= 0
.. .. ... .
. . . . . ..
(n1) (n1) (n1)
y1 y2 . . . yn cn
From linear algebra, we know that this equation has a solution for a nonzero constant vector only if the determinant of
the matrix is zero. Here we define the Wronskian ,W (x), of a set of functions.

y1 y2 ... yn
0
y
1 y20 ... yn0
W (x) = .. .. ..
.

. . ...
(n1) (n1) (n1)
y1 y2 . . . yn
Thus if a set of functions is linearly dependent on an interval, then the Wronskian is identically zero on that interval.
Alternatively, if the Wronskian is identically zero, then the above matrix equation has a solution for a nonzero constant
vector. This implies that the the set of functions is linearly dependent.
Result 16.4.2 The Wronskian of a set of functions vanishes identically over an interval if
and only if the set of functions is linearly dependent on that interval. The Wronskian of a set
of linearly independent functions does not vanish except possibly at isolated points.

907
Example 16.4.2 Consider the set, {x, x2 }. The Wronskian is

x x2
W (x) =

1 2x
= 2x2 x2
= x2 .

Thus the functions are independent.

Example 16.4.3 Consider the set {sin x, cos x, ex }. The Wronskian is



x
sin x
cos x e
W (x) = cos x sin x ex .
sin x cos x ex

Since the last row is a constant multiple of the first row, the determinant is zero. The functions are dependent. We
could also see this with the identity ex = cos x + sin x.

16.4.3 The Wronskian of the Solutions to a Differential Equation


Consider the nth order linear homogeneous differential equation

y (n) + pn1 (x)y (n1) + + p0 (x)y = 0.

Let {y1 , y2 , . . . , yn } be any set of n linearly independent solutions. Let Y (x) be the matrix such that W (x) = [Y (x)].
Now lets differentiate W (x).
d
W 0 (x) = [Y (x)]
dx
Xn
= k [Y (x)]
k=1

908
We note that the all but the last term in this sum is zero. To see this, lets take a look at the first term.
0
y1
0 y20 yn0
y
1 y20 yn0
1 [Y (x)] = .. .. .. ..
. . . .
(n1) (n1) (n1)
y 1 y2 yn
The first two rows in the matrix are identical. Since the rows are dependent, the determinant is zero.
The last term in the sum is
y1 y 2 y n
. . .. .

. . . .
. . .

n [Y (x)] = (n2) (n2) (n2) .

(n) y2 (n) yn (n)
y 1
y
1 y2 yn
(n) (n1)
In the last row of this matrix we make the substitution yi = pn1 (x)yi p0 (x)yi . Recalling that we
can add a multiple of a row to another without changing the determinant, we add p0 (x) times the first row, and p1 (x)
times the second row, etc., to the last row. Thus we have the determinant,

y 1 y 2 y n
.. .. .. ..


. . . .

W 0 (x) =

(n2) (n2) (n2)
y 1 y 2 y n
(n1) (n1) (n1)

pn1 (x)y pn1 (x)y2 pn1 (x)yn
1

y1 y2 yn
.. .. .. ..

. . . .
= pn1 (x) (n2) (n2) (n2)
(n1) y2(n1) yn(n1)
y 1
y
1 y2 yn
= pn1 (x)W (x)
Thus the Wronskian satisfies the first order differential equation,
W 0 (x) = pn1 (x)W (x).

909
Solving this equation we get a result known as Abels formula.
 Z 
W (x) = c exp pn1 (x) dx

Thus regardless of the particular set of solutions that we choose, we can compute their Wronskian up to a constant
factor.
Result 16.4.3 The Wronskian of any linearly independent set of solutions to the equation

y (n) + pn1 (x)y (n1) + + p0 (x)y = 0

is, (up to a multiplicative constant), given by


 Z 
W (x) = exp pn1 (x) dx .

Example 16.4.4 Consider the differential equation

y 00 3y 0 + 2y = 0.

The Wronskian of the two independent solutions is


 Z 
W (x) = c exp 3 dx

= c e3x .

For the choice of solutions {ex , e2x }, the Wronskian is


x
e e2x
W (x) = x
= 2 e3x e3x = e3x .
e 2 e2x

910
16.5 Well-Posed Problems
Consider the initial value problem for an nth order linear differential equation.
dn y dn1 y dy
n
+ pn1 (x) n1 + + p1 (x) + p0 (x)y = f (x)
dx dx dx
y(x0 ) = v1 , y 0 (x0 ) = v2 , . . . , y (n1) (x0 ) = vn

Since the general solution to the differential equation is a linear combination of the n homogeneous solutions plus the
particular solution
y = yp + c1 y1 + c2 y2 + + cn yn ,
the problem to find the constants ci can be written

y1 (x0 ) y2 (x0 ) ... yn (x0 ) c1 yp (x0 ) v1
y 0 (x0 ) 0 0 0
1 y2 (x0 ) ... yn (x0 ) c2 yp (x0 ) v2

.. .. .. + .. = .. .

..
.

. . ... . . .
(n1) (n1) (n1) (n1)
y1 (x0 ) y2 (x0 ) . . . yn (x0 ) cn yp (x0 ) vn

From linear algebra we know that this system of equations has a unique solution only if the determinant of the matrix
is nonzero. Note that the determinant of the matrix is just the Wronskian evaluated at x0 . Thus if the Wronskian
vanishes at x0 , the initial value problem for the differential equation either has no solutions or infinitely many solutions.
Such problems are said to be ill-posed. From Abels formula for the Wronskian
 Z 
W (x) = exp pn1 (x) dx ,

we see that the only way the Wronskian can vanish is if the value of the integral goes to .

Example 16.5.1 Consider the initial value problem


2 2
y 00 y 0 + 2 y = 0, y(0) = y 0 (0) = 1.
x x

911
The Wronskian
 Z 
2
W (x) = exp dx = exp (2 log x) = x2
x

vanishes at x = 0. Thus this problem is not well-posed.


The general solution of the differential equation is

y = c1 x + c2 x2 .

We see that the general solution cannot satisfy the initial conditions. If instead we had the initial conditions y(0) = 0,
y 0 (0) = 1, then there would be an infinite number of solutions.

Example 16.5.2 Consider the initial value problem

2
y 00 y = 0, y(0) = y 0 (0) = 1.
x2

The Wronskian
 Z 
W (x) = exp 0 dx = 1

does not vanish anywhere. However, this problem is not well-posed.


The general solution,
y = c1 x1 + c2 x2 ,

cannot satisfy the initial conditions. Thus we see that a non-vanishing Wronskian does not imply that the problem is
well-posed.

912
Result 16.5.1 Consider the initial value problem
dn y dn1 y dy
+ p n1 (x) + + p 1 (x) + p0 (x)y = 0
dxn dxn1 dx
y(x0 ) = v1 , y 0 (x0 ) = v2 , . . . , y (n1) (x0 ) = vn .

If the Wronskian  Z 
W (x) = exp pn1 (x) dx

vanishes at x = x0 then the problem is ill-posed. The problem may be ill-posed even if the
Wronskian does not vanish.

16.6 The Fundamental Set of Solutions


Consider a set of linearly independent solutions {u1 , u2 , . . . , un } to an nth order linear homogeneous differential equation.
This is called the fundamental set of solutions at x0 if they satisfy the relations
u1 (x0 ) = 1 u2 (x0 ) = 0 ... un (x0 ) = 0
u01 (x0 ) = 0 u02 (x0 ) = 1 ... u0n (x0 ) = 0
.. .. .. ..
. . . .
(n1) (n1) (n1)
u1 (x0 ) = 0 u2 (x0 ) = 0 . . . un (x0 ) = 1
Knowing the fundamental set of solutions is handy because it makes the task of solving an initial value problem
trivial. Say we are given the initial conditions,
y(x0 ) = v1 , y 0 (x0 ) = v2 , ..., y (n1) (x0 ) = vn .
If the ui s are a fundamental set then the solution that satisfies these constraints is just
y = v1 u1 (x) + v2 u2 (x) + + vn un (x).

913
Of course in general, a set of solutions is not the fundamental set. If the Wronskian of the solutions is nonzero and
finite we can generate a fundamental set of solutions that are linear combinations of our original set. Consider the case
of a second order equation Let {y1 , y2 } be two linearly independent solutions. We will generate the fundamental set of
solutions, {u1 , u2 }.
    
u1 c11 c12 y1
=
u2 c21 c22 y2
For {u1 , u2 } to satisfy the relations that define a fundamental set, it must satisfy the matrix equation

u1 (x0 ) u01 (x0 ) y1 (x0 ) y10 (x0 )


      
c11 c12 1 0
= =
u2 (x0 ) u02 (x0 ) c21 c22 y2 (x0 ) y20 (x0 ) 0 1
1
y1 (x0 ) y10 (x0 )
  
c11 c12
=
c21 c22 y2 (x0 ) y20 (x0 )
If the Wronskian is non-zero and finite, we can solve for the constants, cij , and thus find the fundamental set of
solutions. To generalize this result to an equation of order n, simply replace all the 2 2 matrices and vectors of length
2 with n n matrices and vectors of length n. I presented the case of n = 2 simply to save having to write out all the
ellipses involved in the general case. (It also makes for easier reading.)

Example 16.6.1 Two linearly independent solutions to the differential equation y 00 + y = 0 are y1 = ex and y2 = ex .

y1 (0) y10 (0)


   
1
=
y2 (0) y20 (0) 1 i

To find the fundamental set of solutions, {u1 , u2 }, at x = 0 we solve the equation


   1
c11 c12 1
=
c21 c22 1
   
c11 c12 1
=
c21 c22 2 1 1

914
The fundamental set is
ex + ex ex ex
u1 = , u2 = .
2 2
Using trigonometric identities we can rewrite these as
u1 = cos x, u2 = sin x.

Result 16.6.1 The fundamental set of solutions at x = x0 , {u1 , u2 , . . . , un }, to an nth order


linear differential equation, satisfy the relations
u1 (x0 ) = 1 u2 (x0 ) = 0 ... un (x0 ) = 0
0
u1 (x0 ) = 0 u02 (x0 ) = 1 ... u0n (x0 ) = 0
.. .. ... ..
. . .
(n1) (n1) (n1)
u1 (x0 ) = 0 u2 (x0 ) = 0 . . . un (x0 ) = 1.
If the Wronskian of the solutions is nonzero and finite at the point x0 then you can generate
the fundamental set of solutions from any linearly independent set of solutions.

Exercise 16.6
Two solutions of y 00 y = 0 are ex and ex . Show that the solutions are independent. Find the fundamental set of
solutions at x = 0.
Hint, Solution

16.7 Adjoint Equations


For the nth order linear differential operator
dn y dn1 y
L[y] = pn + p n1 + + p0 y
dxn dxn1

915
(where the pj are complex-valued functions) we define the adjoint of L

dn n1 d
n1
L [y] = (1)n (p n y) + (1) (pn1 y) + + p0 y.
dxn dxn1

Here f denotes the complex conjugate of f .

Example 16.7.1
1
L[y] = xy 00 + y 0 + y
x
has the adjoint

d2
 
d 1
L [y] = 2 [xy] y +y
dx dx x
1 1
= xy 00 + 2y 0 y 0 + 2 y + y
 x x  
00 1 0 1
= xy + 2 y + 1 + 2 y.
x x

Taking the adjoint of L yields

d2
    
d 1 1
L [y] = 2 [xy] 2 y + 1+ 2 y
dx dx x x
     
00 0 1 0 1 1
= xy + 2y 2 y y+ 1+ 2 y
x x2 x
1
= xy 00 + y 0 + y.
x
Thus by taking the adjoint of L , we obtain the original operator.

In general, L = L.

916
Consider L[y] = pn y (n) + + p0 y. If each of the pk is k times continuously differentiable and u and v are n times
continuously differentiable on some interval, then on that interval

d
vL[u] uL [v] = B[u, v]
dx
where B[u, v], the bilinear concomitant, is the bilinear form
n
X X
B[u, v] = (1)j u(k) (pm v)(j) .
m=1 j+k=m1
j0,k0

This equation is known as Lagranges identity. If L is a second order operator then

d
up1 v + u0 p2 v u(p2 v)0

vL[u] uL [v] =
dx
= u00 p2 v + u0 p1 v + u p2 v 00 + (2p02 + p1 )v 0 + (p002 + p01 )v .
 

Example 16.7.2 Verify Lagranges identity for the second order operator, L[y] = p2 y 00 + p1 y 0 + p0 y.
 
00 0 d2 d
vL[u] uL [v] = v(p2 u + p1 u + p0 u) u 2
(p2 v) (p1 v) + p0 v
dx dx
= v(p2 u00 + p1 u0 + p0 u) u(p2 v 00 + (2p2 0 p1 )v 0 + (p2 00 p1 0 + p0 )v)
= u00 p2 v + u0 p1 v + u p2 v 00 + (2p02 + p1 )v 0 + (p002 + p01 )v .
 

We will not verify Lagranges identity for the general case.

Integrating Lagranges identity on its interval of validity gives us Greens formula.


Z b 
vL[u] uL [v] dx = B[u, v] x=b B[u, v] x=a
a

917
Result 16.7.1 The adjoint of the operator
dn y dn1 y
L[y] = pn n + pn1 n1 + + p0 y
dx dx
is defined
dn n1 d
n1
L [y] = (1)n (p n y) + (1) (pn1 y) + + p0 y.
dxn dxn1
If each of the pk is k times continuously differentiable and u and v are n times continuously
differentiable, then Lagranges identity states
n
d d X X
vL[y] uL [v] = B[u, v] = (1)j u(k) (pm v)(j) .
dx dx m=1
j+k=m1
j0,k0

Integrating Lagranges identity on its domain of validity yields Greens formula,


Z b 
vL[u] uL [v] dx = B[u, v] x=b B[u, v] x=a .
a

918
16.8 Additional Exercises
Exact Equations
Nature of Solutions
Transformation to a First Order System
The Wronskian
Well-Posed Problems
The Fundamental Set of Solutions
Adjoint Equations
Exercise 16.7
Find the adjoint of the Bessel equation of order ,

x2 y 00 + xy 0 + (x2 2 )y = 0,

and the Legendre equation of order ,

(1 x2 )y 00 2xy 0 + ( + 1)y = 0.

Hint, Solution
Exercise 16.8
Find the adjoint of
x2 y 00 xy 0 + 3y = 0.
Hint, Solution

919
16.9 Hints
Hint 16.1

Hint 16.2

Hint 16.3

Hint 16.4

Hint 16.5
The difference of any two of the ui s is a homogeneous solution.

Hint 16.6

Exact Equations
Nature of Solutions
Transformation to a First Order System
The Wronskian
Well-Posed Problems
The Fundamental Set of Solutions
Adjoint Equations
Hint 16.7

920
Hint 16.8

921
16.10 Solutions
Solution 16.1
The second order, linear, homogeneous differential equation is

P (x)y 00 + Q(x)y 0 + R(x)y = 0. (16.4)

An exact equation can be written in the form:


d
[a(x)y 0 + b(x)y] = 0.
dx
If Equation 16.4 is exact, then we can write it in the form:
d
[P (x)y 0 + f (x)y] = 0
dx
for some function f (x). We carry out the differentiation to write the equation in standard form:

P (x)y 00 + (P 0 (x) + f (x)) y 0 + f 0 (x)y = 0 (16.5)

We equate the coefficients of Equations 16.4 and 16.5 to obtain a set of equations.

P 0 (x) + f (x) = Q(x), f 0 (x) = R(x).

In order to eliminate f (x), we differentiate the first equation and substitute in the expression for f 0 (x) from the second
equation. This gives us a necessary condition for Equation 16.4 to be exact:

P 00 (x) Q0 (x) + R(x) = 0 (16.6)

Now we demonstrate that Equation 16.6 is a sufficient condition for exactness. Suppose that Equation 16.6 holds.
Then we can replace R by Q0 P 00 in the differential equation.

P y 00 + Qy 0 + (Q0 P 00 )y = 0

922
We recognize the right side as an exact differential.

(P y 0 + (Q P 0 )y)0 = 0

Thus Equation 16.6 is a sufficient condition for exactness. We can integrate to reduce the problem to a first order
differential equation.
P y 0 + (Q P 0 )y = c

Solution 16.2
Suppose that there is an integrating factor (x) that will make

P (x)y 00 + Q(x)y 0 + R(x)y = 0

exact. We multiply by this integrating factor.

(x)P (x)y 00 + (x)Q(x)y 0 + (x)R(x)y = 0. (16.7)

We apply the exactness condition from Exercise 16.1 to obtain a differential equation for the integrating factor.

(P )00 (Q)0 + R = 0
00 P + 20 P 0 + P 00 0 Q Q0 + R = 0
P 00 + (2P 0 Q)0 + (P 00 Q0 + R) = 0

Solution 16.3
We consider the differential equation,
y 00 + xy 0 + y = 0.

Since
(1)00 (x)0 + 1 = 0

923
we see that this is an exact equation. We rearrange terms to form exact derivatives and then integrate.
(y 0 )0 + (xy)0 = 0
y 0 + xy = c
d h x2 /2 i 2
e y = c ex /2
dx
Z
x2 /2 2 2
y = ce ex /2 dx + d ex /2

Solution 16.4
Consider the initial value problem,
y 00 + p(x)y 0 + q(x)y = f (x),
y(x0 ) = y0 , y 0 (x0 ) = y1 .
If p(x), q(x) and f (x) are continuous on an interval (a . . . b) with x0 (a . . . b), then the problem has a unique solution
on that interval.
1.
xy 00 + 3y = x
3
y 00 + y = 1
x
Unique solutions exist on the intervals ( . . . 0) and (0 . . . ).
2.
x(x 1)y 00 + 3xy 0 + 4y = 2
3 4 2
y 00 + y0 + y=
x1 x(x 1) x(x 1)
Unique solutions exist on the intervals ( . . . 0), (0 . . . 1) and (1 . . . ).

924
3.

ex y 00 + x2 y 0 + y = tan x
y 00 + x2 ex y 0 + ex y = ex tan x
 
(2n1) (2n+1)
Unique solutions exist on the intervals 2
... 2
for n Z.

Solution 16.5
We know that the general solution is
y = yp + c1 y1 + c2 y2 ,

where yp is a particular solution and y1 and y2 are linearly independent homogeneous solutions. Since yp can be
any particular solution, we choose yp = u1 . Now we need to find two homogeneous solutions. Since L[ui ] = f (x),
L[u1 u2 ] = L[u2 u3 ] = 0. Finally, we note that since the ui s are linearly independent, y1 = u1 u2 and y2 = u2 u3
are linearly independent. Thus the general solution is

y = u1 + c1 (u1 u2 ) + c2 (u2 u3 ).

Solution 16.6
The Wronskian of the solutions is
x
x
e e
W (x) = x = 2.
e ex

Since the Wronskian is nonzero, the solutions are independent.


The fundamental set of solutions, {u1 , u2 }, is a linear combination of ex and ex .
    x 
u1 c11 c12 e
=
u2 c21 c22 ex

925
The coefficients are
   1
c11 c12 e0 e0
= 0
c21 c22 e e0
 1
1 1
=
1 1
 
1 1 1
=
2 1 1
 
1 1 1
=
2 1 1

1 1
u1 = (ex + ex ), u2 = (ex ex ).
2 2
The fundamental set of solutions at x = 0 is
{cosh x, sinh x}.

Exact Equations
Nature of Solutions
Transformation to a First Order System
The Wronskian
Well-Posed Problems
The Fundamental Set of Solutions
Adjoint Equations
Solution 16.7
1. The Bessel equation of order is
x2 y 00 + xy 0 + (x2 2 )y = 0.

926
The adjoint equation is

x2 00 + (4x x)0 + (2 1 + x2 2 ) = 0
x2 00 + 3x0 + (1 + x2 2 ) = 0.

2. The Legendre equation of order is

(1 x2 )y 00 2xy 0 + ( + 1)y = 0

The adjoint equation is

(1 x2 )00 + (4x + 2x)0 + (2 + 2 + ( + 1)) = 0


(1 x2 )00 2x0 + ( + 1) = 0

Solution 16.8
The adjoint of
x2 y 00 xy 0 + 3y = 0
is
d2 2 d
(x y) + (xy) + 3y = 0
dx2 dx
(x2 y 00 + 4xy 0 + 2y) + (xy 0 + y) + 3y = 0
x2 y 00 + 5xy 0 + 6y = 0.

927
16.11 Quiz
Problem 16.1
What is the differential equation whose solution is the two parameter family of curves y = c1 sin(2x + c2 )?
Solution

928
16.12 Quiz Solutions
Solution 16.1
We take the first and second derivative of y = c1 sin(2x + c2 ).

y 0 = 2c1 cos(2x + c2 )
y 00 = 4c1 sin(2x + c2 )

This gives us three equations involving x, y, y 0 , y 00 and the parameters c1 and c2 . We eliminate the the parameters to
obtain the differential equation. Clearly we have,

y 00 + 4y = 0.

929
Chapter 17

Techniques for Linear Differential Equations

My new goal in life is to take the meaningless drivel out of human interaction.

-Dave Ozenne

The nth order linear homogeneous differential equation can be written in the form:

y (n) + an1 (x)y (n1) + + a1 (x)y 0 + a0 (x)y = 0.

In general it is not possible to solve second order and higher linear differential equations. In this chapter we will examine
equations that have special forms which allow us to either reduce the order of the equation or solve it.

17.1 Constant Coefficient Equations


The nth order constant coefficient differential equation has the form:

y (n) + an1 y (n1) + + a1 y 0 + a0 y = 0.

930
We will find that solving a constant coefficient differential equation is no more difficult than finding the roots of a
polynomial. For notational simplicity, we will first consider second order equations. Then we will apply the same
techniques to higher order equations.

17.1.1 Second Order Equations


Factoring the Differential Equation. Consider the second order constant coefficient differential equation:

y 00 + 2ay 0 + by = 0. (17.1)

Just as we can factor a second degree polynomial:



2 + 2a + b = ( )( ), = a + a2 b and = a a2 b,

we can factor Equation 17.1.


d2
    
d d d
+ 2a +b y = y
dx2 dx dx dx
Once we have factored
 the differential equation, we can solve it by solving a series of two first order differential equations.
d
We set u = dx y to obtain a first order equation:
 
d
u = 0,
dx

which has the solution:


u = c1 ex .
To find the solution of Equation 17.1, we solve
 
d
y = u = c1 ex .
dx

931
We multiply by the integrating factor and integrate.
d x 
e y = c1 e()x
dx Z
x
y = c1 e e()x dx + c2 ex

We first consider the case that and are distinct.


1
y = c1 ex e()x +c2 ex

We choose new constants to write the solution in a simpler form.

y = c1 ex +c2 ex

Now we consider the case = .


Z
x
y = c1 e 1 dx + c2 ex

y = c1 x ex +c2 ex

The solution of Equation 17.1 is (


c1 ex +c2 ex , 6= ,
y= x x
(17.2)
c1 e +c2 x e , = .

Example 17.1.1 Consider the differential equation: y 00 + y = 0. To obtain the general solution, we factor the equation
and apply the result in Equation 17.2.
  
d d
+ y =0
dx dx
y = c1 ex +c2 ex .

932
Example 17.1.2 Next we solve y 00 = 0.
  
d d
0 0 y =0
dx dx
y = c1 e0x +c2 x e0x
y = c1 + c2 x

Substituting the Form of the Solution into the Differential Equation. Note that if we substitute y = ex
into the differential equation (17.1), we will obtain the quadratic polynomial (17.1.1) for .

y 00 + 2ay 0 + by = 0
2 ex +2a ex +b ex = 0
2 + 2a + b = 0

This gives us a superficially different method for solving constant coefficient equations. We substitute y = ex into
the differential equation. Let and be the roots of the quadratic in . If the roots are distinct, then the linearly
independent solutions are y1 = ex and y2 = ex . If the quadratic has a double root at = , then the linearly
independent solutions are y1 = ex and y2 = x ex .

Example 17.1.3 Consider the equation:


y 00 3y 0 + 2y = 0.
The substitution y = ex yields
2 3 + 2 = ( 1)( 2) = 0.
Thus the solutions are ex and e2x .

Example 17.1.4 Next consider the equation:

y 00 2y 0 + 4y = 0.

933
The substitution y = ex yields
2 2 + 4 = ( 2)2 = 0.
Because the polynomial has a double root, the solutions are e2x and x e2x .

Result 17.1.1 Consider the second order constant coefficient differential equation:

y 00 + 2ay 0 + by = 0.

We can factor the differential equation into the form:


  
d d
y = 0,
dx dx
which has the solution: (
c1 ex +c2 ex , 6= ,
y=
c1 ex +c2 x ex , = .
We can also determine and by substituting y = ex into the differential equation and
factoring the polynomial in .

Shift Invariance. Note that if u(x) is a solution of a constant coefficient equation, then u(x + c) is also a solution.
This is useful in applying initial or boundary conditions.

Example 17.1.5 Consider the problem


y 00 3y 0 + 2y = 0, y(0) = a, y 0 (0) = b.
We know that the general solution is
y = c1 ex +c2 e2x .

934
Applying the initial conditions, we obtain the equations,

c1 + c2 = a, c1 + 2c2 = b.

The solution is
y = (2a b) ex +(b a) e2x .
Now suppose we wish to solve the same differential equation with the boundary conditions y(1) = a and y 0 (1) = b. All
we have to do is shift the solution to the right.

y = (2a b) ex1 +(b a) e2(x1) .

17.1.2 Real-Valued Solutions


If the coefficients of the differential equation are real, then the solution can be written in terms of real-valued functions
(Result 16.2.2). For a real root = of the polynomial in , the corresponding solution, y = ex , is real-valued.
Now recall that the complex roots of a polynomial with real coefficients occur in complex conjugate pairs. Assume
that are roots of
n + an1 n1 + + a1 + a0 = 0.
The corresponding solutions of the differential equation are e(+)x and e()x . Note that the linear combinations

e(+)x + e()x e(+)x e()x


= ex cos(x), = ex sin(x),
2 2
are real-valued solutions of the differential equation. We could also obtain real-valued solution by taking the real and
imaginary parts of either e(+)x or e()x .

< e(+)x = ex cos(x), = e(+)x = ex sin(x)


 

Example 17.1.6 Consider the equation


y 00 2y 0 + 2y = 0.

935
The substitution y = ex yields

2 2 + 2 = ( 1 )( 1 + ) = 0.

The linearly independent solutions are

e(1+)x , and e(1)x .

We can write the general solution in terms of real functions.

y = c1 ex cos x + c2 ex sin x

Exercise 17.1
Find the general solution of

y 00 + 2ay 0 + by = 0

for a, b R. There are three distinct forms of the solution depending on the sign of a2 b.
Hint, Solution

Exercise 17.2
Find the fundamental set of solutions of

y 00 + 2ay 0 + by = 0

at the point x = 0, for a, b R. Use the general solutions obtained in Exercise 17.1.
Hint, Solution

936
Result 17.1.2 . Consider the second order constant coefficient equation

y 00 + 2ay 0 + by = 0.

The general solution of this differential equation is


 
ax a2 b x a2 b x
e

c1 e +c2 e if a2 > b,

y = eax c1 cos( b a2 x) + c2 sin( b a2 x)

if a2 < b,
eax (c + c x)

if a2 = b.
1 2

The fundamental set of solutions at x = 0 is


n 
ax cosh( a2 b x) + a
 o
e sinh( a2 b x) , eax 1 sinh( a2 b x) if a2 > b,
a2 b a2 b
n
ax
 a

ax 1
o

e cos( b a2 x) +
ba2
sin( b a2 x) , e
ba2
sin( b a 2 x) if a2 < b,

ax
{(1 + ax) e , x e }ax if a2 = b.

To obtain the fundamental set of solutions at the point x = , substitute (x ) for x in


the above solutions.

17.1.3 Higher Order Equations

The constant coefficient equation of order n has the form

L[y] = y (n) + an1 y (n1) + + a1 y 0 + a0 y = 0. (17.3)

937
The substitution y = ex will transform this differential equation into an algebraic equation.

L[ex ] = n ex +an1 n1 ex + + a1 ex +a0 ex = 0


n + an1 n1 + + a1 + a0 ex = 0


n + an1 n1 + + a1 + a0 = 0

Assume that the roots of this equation, 1 , . . . , n , are distinct. Then the n linearly independent solutions of Equa-
tion 17.3 are
e1 x , . . . , en x .
If the roots of the algebraic equation are not distinct then we will not obtain all the solutions of the differential
equation. Suppose that 1 = is a double root. We substitute y = ex into the differential equation.

L[ex ] = [( )2 ( 3 ) ( n )] ex = 0

Setting = will make the left side of the equation zero. Thus y = ex is a solution. Now we differentiate both sides
of the equation with respect to and interchange the order of differentiation.
 
d x d x
= L x ex
 
L[e ] = L e
d d
 
Let p() = ( 3 ) ( n ). We calculate L x ex by applying L and then differentiating with respect to .

d
L x ex = L[ex ]
 
d
d
= [( )2 ( 3 ) ( n )] ex
d
d
= [( )2 p()] ex
d
= 2( )p() + ( )2 p0 () + ( )2 p()x ex


= ( ) [2p() + ( )p0 () + ( )p()x] ex

938
Since setting = will make this expression zero, L[x ex ] = 0, x ex is a solution of Equation 17.3. You can verify
that ex and x ex are linearly independent. Now we have generated all of the solutions for the differential equation.
If = is a root of multiplicity m then by repeatedly differentiating with respect to you can show that the
corresponding solutions are
ex , x ex , x2 ex , . . . , xm1 ex .

Example 17.1.7 Consider the equation


y 000 3y 0 + 2y = 0.
The substitution y = ex yields
3 3 + 2 = ( 1)2 ( + 2) = 0.
Thus the general solution is
y = c1 ex +c2 x ex +c3 e2x .

Result 17.1.3 Consider the nth order constant coefficient equation


dn y dn1 y dy
+ a n1 + + a 1 + a0 y = 0.
dxn dxn1 dx
Let the factorization of the algebraic equation obtained with the substitution y = ex be

( 1 )m1 ( 2 )m2 ( p )mp = 0.

A set of linearly independent solutions is given by

{e1 x , x e1 x , . . . , xm1 1 e1 x , . . . , ep x , x ep x , . . . , xmp 1 ep x }.

If the coefficients of the differential equation are real, then we can find a real-valued set of
solutions.

939
Example 17.1.8 Consider the equation
d4 y d2 y
+ 2 + y = 0.
dx4 dx2
The substitution y = ex yields
4 + 22 + 1 = ( i)2 ( + i)2 = 0.
Thus the linearly independent solutions are

ex , x ex , ex and x ex .

Noting that
ex = cos(x) + sin(x),
we can write the general solution in terms of sines and cosines.

y = c1 cos x + c2 sin x + c3 x cos x + c4 x sin x

17.2 Euler Equations


Consider the equation
d2 y dy
L[y] = x2 2
+ ax + by = 0, x > 0.
dx dx
Lets say, for example, that y has units of distance and x has units of time. Note that each term in the differential
equation has the same dimension.

(distance) (distance)
(time)2 2
= (time) = (distance)
(time) (time)

Thus this is a second order Euler, or equidimensional equation. We know that the first order Euler equation, xy 0 +ay = 0,
has the solution y = cxa . Thus for the second order equation we will try a solution of the form y = x . The substitution

940
y = x will transform the differential equation into an algebraic equation.

d2 d
L[x ] = x2 2
[x ] + ax [x ] + bx = 0
dx dx
( 1)x + ax + bx = 0

( 1) + a + b = 0

Factoring yields

( 1 )( 2 ) = 0.

If the two roots, 1 and 2 , are distinct then the general solution is

y = c1 x1 + c2 x2 .

If the roots are not distinct, 1 = 2 = , then we only have the one solution, y = x . To generate the other solution
we use the same approach as for the constant coefficient equation. We substitute y = x into the differential equation
and differentiate with respect to .
d d
L[x ] = L[ x ]
d d
= L[ln x x ]

Note that
d d ln x
x = e = ln x e ln x = ln x x .
d d
Now we apply L and then differentiate with respect to .
d d
L[x ] = ( )2 x
d d
= 2( )x + ( )2 ln x x

941
Equating these two results,
L[ln x x ] = 2( )x + ( )2 ln x x .
Setting = will make the right hand side zero. Thus y = ln x x is a solution.
If you are in the mood for a little algebra you can show by repeatedly differentiating with respect to that if =
is a root of multiplicity m in an nth order Euler equation then the associated solutions are

x , ln x x , (ln x)2 x , . . . , (ln x)m1 x .

Example 17.2.1 Consider the Euler equation


y
xy 00 y 0 + = 0.
x
The substitution y = x yields the algebraic equation

( 1) + 1 = ( 1)2 = 0.

Thus the general solution is


y = c1 x + c2 x ln x.

17.2.1 Real-Valued Solutions


If the coefficients of the Euler equation are real, then the solution can be written in terms of functions that are real-valued
when x is real and positive, (Result 16.2.2). If are the roots of

( 1) + a + b = 0

then the corresponding solutions of the Euler equation are

x+ and x .

We can rewrite these as


x e ln x and x e ln x .

942
Note that the linear combinations
x e ln x +x e ln x x e ln x x e ln x
= x cos( ln x), and = x sin( ln x),
2 2
are real-valued solutions when x is real and positive. Equivalently, we could take the real and imaginary parts of either
x+ or x .
< x e ln x = x cos( ln x), = x e ln x = x sin( ln x)
 

Result 17.2.1 Consider the second order Euler equation

x2 y 00 + (2a + 1)xy 0 + by = 0.

The general solution of this differential equation is


 
a a 2 b a2 b
x

c1 x + c2 x if a2 > b,

y = xa c1 cos b a2 ln x + c2 sin b a2 ln x
 
if a2 < b,
xa (c + c ln x)

if a2 = b.
1 2

The fundamental set of solutions at x = is


n  a     
x 2 b ln x + a 2 b ln x


cosh a sinh a ,
 a2 b


 a o

x
sinh a2 b ln x if a2 > b,



a2 b

n 
x
 a     
cos b a 2 ln x + a sin b a2 ln x ,
y= ba 2
 a  o

x
2 sin b a2 ln x if a2 < b,




ba
   

a   a
x x x x
1 + a ln , ln if a2 = b.


943
Example 17.2.2 Consider the Euler equation
x2 y 00 3xy 0 + 13y = 0.
The substitution y = x yields
( 1) 3 + 13 = ( 2 3)( 2 + 3) = 0.
The linearly independent solutions are
x2+3 , x23 .


We can put this in a more understandable form.


x2+3 = x2 e3 ln x = x2 cos(3 ln x) + x2 sin(3 ln x)
We can write the general solution in terms of real-valued functions.
y = c1 x2 cos(3 ln x) + c2 x2 sin(3 ln x)

Result 17.2.2 Consider the nth order Euler equation


n n1
nd y n1 d y dy
x + an1 x + + a1 x + a0 y = 0.
dxn dxn1 dx
Let the factorization of the algebraic equation obtained with the substitution y = x be

( 1 )m1 ( 2 )m2 ( p )mp = 0.

A set of linearly independent solutions is given by

{x1 , ln x x1 , . . . , (ln x)m1 1 x1 , . . . , xp , ln x xp , . . . , (ln x)mp 1 xp }.

If the coefficients of the differential equation are real, then we can find a set of solutions that
are real valued when x is real and positive.

944
17.3 Exact Equations
Exact equations have the form
d
F (x, y, y 0 , y 00 , . . .) = f (x).
dx

If you can write an equation in the form of an exact equation, you can integrate to reduce the order by one, (or solve
the equation for first order). We will consider a few examples to illustrate the method.

Example 17.3.1 Consider the equation


y 00 + x2 y 0 + 2xy = 0.

We can rewrite this as


d 0
y + x2 y = 0.

dx
Integrating yields a first order inhomogeneous equation.

y 0 + x2 y = c1
R
We multiply by the integrating factor I(x) = exp( x2 dx) to make this an exact equation.

d  x3 /3  3
e y = c1 ex /3
dx Z
3 /3 3 /3
ex y = c1 ex dx + c2
Z
3 /3 3 /3 3 /3
y = c1 ex ex dx + c2 ex

945
Result 17.3.1 If you can write a differential equation in the form
d
F (x, y, y 0 , y 00 , . . .) = f (x),
dx
then you can integrate to reduce the order of the equation.
Z
F (x, y, y 0 , y 00 , . . .) = f (x) dx + c

17.4 Equations Without Explicit Dependence on y


Example 17.4.1 Consider the equation
y 00 + xy 0 = 0.
This is a second order equation for y, but note that it is a first order equation for y 0 . We can solve directly for y 0 .
   
d 2 3/2 0
exp x y =0
dx 3
 
0 2 3/2
y = c1 exp x
3
Now we just integrate to get the solution for y.
Z  
2
y = c1 exp x3/2 dx + c2
3

Result 17.4.1 If an nth order equation does not explicitly depend on y then you can consider
it as an equation of order n 1 for y 0 .

946
17.5 Reduction of Order
Consider the second order linear equation

L[y] y 00 + p(x)y 0 + q(x)y = f (x).

Suppose that we know one homogeneous solution y1 . We make the substitution y = uy1 and use that L[y1 ] = 0.

L[uy1 ] = 0u00 y1 + 2u0 y10 + uy100 + p(u0 y1 + uy10 ) + quy1 = 0


u00 y1 + u0 (2y10 + py1 ) + u(y100 + py10 + qy1 ) = 0
u00 y1 + u0 (2y10 + py1 ) = 0

Thus we have reduced the problem to a first order equation for u0 . An analogous result holds for higher order equations.

Result 17.5.1 Consider the nth order linear differential equation

y (n) + pn1 (x)y (n1) + + p1 (x)y 0 + p0 (x)y = f (x).

Let y1 be a solution of the homogeneous equation. The substitution y = uy1 will transform
the problem into an (n 1)th order equation for u0 . For the second order problem

y 00 + p(x)y 0 + q(x)y = f (x)

this reduced equation is


u00 y1 + u0 (2y10 + py1 ) = f (x).

Example 17.5.1 Consider the equation


y 00 + xy 0 y = 0.

947
By inspection we see that y1 = x is a solution. We would like to find another linearly independent solution. The
substitution y = xu yields
xu00 + (2 + x2 )u0 = 0
 
00 2
u + + x u0 = 0
x
The integrating factor is I(x) = exp(2 ln x + x2 /2) = x2 exp(x2 /2).
d  2 x2 /2 0 
x e u =0
dx
2
u0 = c1 x2 ex /2
Z
2
u = c1 x2 ex /2 dx + c2
Z
2
y = c1 x x2 ex /2 dx + c2 x

Thus we see that a second solution is


Z
2 /2
y2 = x x2 ex dx.

17.6 *Reduction of Order and the Adjoint Equation


Let L be the linear differential operator
dn y dn1 y
L[y] = pn + p n1 + + p0 y,
dxn dxn1
where each pj is a j times continuously differentiable complex valued function. Recall that the adjoint of L is
dn n1 d
n1
L [y] = (1)n (p n y) + (1) (pn1 y) + + p0 y.
dxn dxn1

948
If u and v are n times continuously differentiable, then Lagranges identity states
d
vL[u] uL [v] = B[u, v],
dx
where n
X X
B[u, v] = (1)j u(k) (pm v)(j) .
m=1 j+k=m1
j0,k0

For second order equations,


B[u, v] = up1 v + u0 p2 v u(p2 v)0 .
(See Section 16.7.)
If we can find a solution to the homogeneous adjoint equation, L [y] = 0, then we can reduce the order of the
equation L[y] = f (x). Let satisfy L [] = 0. Substituting u = y, v = into Lagranges identity yields
d
L[y] yL [] = B[y, ]
dx
d
L[y] = B[y, ].
dx
The equation L[y] = f (x) is equivalent to the equation
d
B[y, ] = f
dx Z
B[y, ] = (x)f (x) dx,

which is a linear equation in y of order n 1.

Example 17.6.1 Consider the equation

L[y] = y 00 x2 y 0 2xy = 0.

949
Method 1. Note that this is an exact equation.
d 0
(y x2 y) = 0
dx
y 0 x2 y = c1
d  x3 /3  3
e y = c1 ex /3
dx
Z
x3 /3 3 3
y = c1 e ex /3 dx + c2 ex /3

Method 2. The adjoint equation is


L [y] = y 00 + x2 y 0 = 0.
By inspection we see that = (constant) is a solution of the adjoint equation. To simplify the algebra we will choose
= 1. Thus the equation L[y] = 0 is equivalent to

B[y, 1] = c1
d d
y(x2 ) + [y](1) y [1] = c1
dx dx
y 0 x2 y = c1 .

By using the adjoint equation to reduce the order we obtain the same solution as with Method 1.

950
17.7 Additional Exercises
Constant Coefficient Equations
Exercise 17.3 (mathematica/ode/techniques linear/constant.nb)
Find the solution of each one of the following initial value problems. Sketch the graph of the solution and describe its
behavior as t increases.
1. 6y 00 5y 0 + y = 0, y(0) = 4, y 0 (0) = 0
2. y 00 2y 0 + 5y = 0, y(/2) = 0, y 0 (/2) = 2
3. y 00 + 4y 0 + 4y = 0, y(1) = 2, y 0 (1) = 1
Hint, Solution
Exercise 17.4 (mathematica/ode/techniques linear/constant.nb)
Substitute y = ex to find two linearly independent solutions to

y 00 4y 0 + 13y = 0.

that are real-valued when x is real-valued.


Hint, Solution
Exercise 17.5 (mathematica/ode/techniques linear/constant.nb)
Find the general solution to
y 000 y 00 + y 0 y = 0.
Write the solution in terms of functions that are real-valued when x is real-valued.
Hint, Solution
Exercise 17.6
Substitute y = ex to find the fundamental set of solutions at x = 0 for each of the equations:
1. y 00 + y = 0,

951
2. y 00 y = 0,
3. y 00 = 0.
What are the fundamental set of solutions at x = 1 for each of these equations.
Hint, Solution
Exercise 17.7
Consider a ball of mass m hanging by an ideal spring of spring constant k. The ball is suspended in a fluid which
damps the motion. This resistance has a coefficient of friction, . Find the differential equation for the displacement
of the mass from its equilibrium position by balancing forces. Denote this displacement by y(t). If the damping force
is weak, the mass will have a decaying, oscillatory motion. If the damping force is strong, the mass will not oscillate.
The displacement will decay to zero. The value of the damping which separates these two behaviors is called critical
damping.
Find the solution which satisfies the initial conditions y(0) = 0, y 0 (0) = 1. Use the solutions obtained in Exercise 17.2
or refer to Result 17.1.2.
Consider the case m = k = 1. Find the coefficient of friction for which the displacement of the mass decays most
rapidly. Plot the displacement for strong, weak and critical damping.
Hint, Solution
Exercise 17.8
Show that y = c cos(x ) is the general solution of y 00 + y = 0 where c and are constants of integration. (It is not
sufficient to show that y = c cos(x ) satisfies the differential equation. y = 0 satisfies the differential equation, but
is is certainly not the general solution.) Find constants c and such that y = sin(x).
Is y = c cosh(x ) the general solution of y 00 y = 0? Are there constants c and such that y = sinh(x)?
Hint, Solution
Exercise 17.9 (mathematica/ode/techniques linear/constant.nb)
Let y(t) be the solution of the initial-value problem
y 00 + 5y 0 + 6y = 0; y(0) = 1, y 0 (0) = V.
For what values of V does y(t) remain nonnegative for all t > 0?

952
Hint, Solution
Exercise 17.10 (mathematica/ode/techniques linear/constant.nb)
Find two linearly independent solutions of

y 00 + sign(x)y = 0, < x < .

where sign(x) = 1 according as x is positive or negative. (The solution should be continuous and have a continuous
first derivative.)
Hint, Solution

Euler Equations
Exercise 17.11
Find the general solution of
x2 y 00 + xy 0 + y = 0, x > 0.
Hint, Solution
Exercise 17.12
Substitute y = x to find the general solution of

x2 y 00 2xy + 2y = 0.

Hint, Solution
Exercise 17.13 (mathematica/ode/techniques linear/constant.nb)
Substitute y = x to find the general solution of

1
xy 000 + y 00 + y 0 = 0.
x
Write the solution in terms of functions that are real-valued when x is real-valued and positive.
Hint, Solution

953
Exercise 17.14
Find the general solution of
x2 y 00 + (2a + 1)xy 0 + by = 0.
Hint, Solution
Exercise 17.15
Show that
ex ex
y1 = eax , y2 = lim
a
are linearly indepedent solutions of
y 00 a2 y = 0
for all values of a. It is common to abuse notation and write the second solution as
eax eax
y2 =
a
where the limit is taken if a = 0. Likewise show that
xa xa
y1 = xa , y2 =
a
are linearly indepedent solutions of
x2 y 00 + xy 0 a2 y = 0
for all values of a.
Hint, Solution
Exercise 17.16 (mathematica/ode/techniques linear/constant.nb)
Find two linearly independent solutions (i.e., the general solution) of

(a) x2 y 00 2xy 0 + 2y = 0, (b) x2 y 00 2y = 0, (c) x2 y 00 xy 0 + y = 0.

Hint, Solution

954
Exact Equations
Exercise 17.17
Solve the differential equation
y 00 + y 0 sin x + y cos x = 0.
Hint, Solution

Equations Without Explicit Dependence on y


Reduction of Order
Exercise 17.18
Consider
(1 x2 )y 00 2xy 0 + 2y = 0, 1 < x < 1.
Verify that y = x is a solution. Find the general solution.
Hint, Solution
Exercise 17.19
Consider the differential equation
x+1 0 1
y 00 y + y = 0.
x x
Since the coefficients sum to zero, (1 x+1
x
+ x1 = 0), y = ex is a solution. Find another linearly independent solution.
Hint, Solution
Exercise 17.20
One solution of
(1 2x)y 00 + 4xy 0 4y = 0
is y = x. Find the general solution.
Hint, Solution

955
Exercise 17.21
Find the general solution of
(x 1)y00 xy0 + y = 0,
given that one solution is y = ex . (you may assume x > 1)
Hint, Solution

*Reduction of Order and the Adjoint Equation

956
17.8 Hints
Hint 17.1
Substitute y = ex into the differential equation.

Hint 17.2
The fundamental set of solutions is a linear combination of the homogeneous solutions.

Constant Coefficient Equations


Hint 17.3

Hint 17.4

Hint 17.5
It is a constant coefficient equation.

Hint 17.6
Use the fact that if u(x) is a solution of a constant coefficient equation, then u(x + c) is also a solution.

Hint 17.7
The force on the mass due to the spring is ky(t). The frictional force is y 0 (t).
Note that the initial conditions describe the second fundamental solution at t = 0.
Note that for large t, t et is much small than et if < . (Prove this.)

Hint 17.8
By definition, the general solution of a second order differential equation is a two parameter family of functions that
satisfies the differential equation. The trigonometric identities in Appendix M may be useful.

957
Hint 17.9

Hint 17.10

Euler Equations
Hint 17.11

Hint 17.12

Hint 17.13

Hint 17.14
Substitute y = x into the differential equation. Consider the three cases: a2 > b, a2 < b and a2 = b.

Hint 17.15

Hint 17.16

Exact Equations
Hint 17.17
It is an exact equation.

Equations Without Explicit Dependence on y

958
Reduction of Order
Hint 17.18

Hint 17.19
Use reduction of order to find the other solution.

Hint 17.20
Use reduction of order to find the other solution.

Hint 17.21

*Reduction of Order and the Adjoint Equation

959
17.9 Solutions
Solution 17.1
We substitute y = ex into the differential equation.
y 00 + 2ay 0 + by = 0
2 + 2a + b = 0

= a a2 b
If a2 > b then the two roots are distinct and real. The general solution is

y = c1 e(a+ a2 b)x
+c2 e(a a2 b)x
.
If a2 < b then the two roots are distinct and complex-valued. We can write them as

= a b a2 .
The general solution is
y = c1 e(a+ ba2 )x
+c2 e(a ba2 )x
.
By taking the sum and difference of the two linearly independent solutions above, we can write the general solution as
   
y = c1 eax cos b a2 x + c2 eax sin b a2 x .

If a2 = b then the only root is = a. The general solution in this case is then
y = c1 eax +c2 x eax .
In summary, the general solution is
 
ax a2 b x a2 b x

e c 1 e +c 2 e if a2 > b,


y= e ax
 
c 1 cos b a 2 x + c sin
2 b a 2x if a2 < b,
eax (c + c x)

if a2 = b.
1 2

960
Solution 17.2
First we note that the general solution can be written,

ax
 
e c1 cosh a2 b x + c2 sinh a2 b x if a2 > b,


y = eax c1 cos b a2 x + c2 sin b a2 x
 
if a2 < b,

ax
e (c1 + c2 x) if a2 = b.

We first consider the case a2 > b. The derivative is


0 ax
       
y = e 2
ac1 + a b c2 cosh 2 2
a b x + ac2 + a b c1 sinh 2
a bx .

The conditions, y1 (0) = 1 and y10 (0) = 0, for the first solution become,

c1 = 1, ac1 + a2 b c2 = 0,
a
c1 = 1, c2 = .
a2 b

The conditions, y2 (0) = 0 and y20 (0) = 1, for the second solution become,

c1 = 0, ac1 + a2 b c2 = 1,
1
c1 = 0, c2 = .
a2 b
The fundamental set of solutions is
    a   1  
ax ax
e cosh a bx +
2 sinh 2
a bx ,e sinh 2
a bx .
a2 b a2 b

Now consider the case a2 < b. The derivative is


0 ax
       
y = e 2
ac1 + b a c2 cos 2 2
b a x + ac2 b a c1 sin 2
ba x .

961
Clearly, the fundamental set of solutions is
 
    
ax
 a ax 1
e cos b a2 x + sin b a2 x ,e sin b a2 x .
b a2 b a2

Finally we consider the case a2 = b. The derivative is

y 0 = eax (ac1 + c2 + ac2 x).

The conditions, y1 (0) = 1 and y10 (0) = 0, for the first solution become,

c1 = 1, ac1 + c2 = 0,
c1 = 1, c2 = a.

The conditions, y2 (0) = 0 and y20 (0) = 1, for the second solution become,

c1 = 0, ac1 + c2 = 1,
c1 = 0, c2 = 1.

The fundamental set of solutions is


(1 + ax) eax , x eax .


In summary, the fundamental set of solutions at x = 0 is


n
ax
  ax 1 o
2 bx + a


e cosh a a2 b
sinh a 2 bx , e
a2 b
sinh a 2 bx if a2 > b,

n  
ax
 a
 ax 1
o

e 2
cos b a x + ba2 sin b a x , e
2
ba2
sin b a x 2 if a2 < b,

{(1 + ax) eax , x eax } if a2 = b.

Constant Coefficient Equations

962
Solution 17.3
1. We consider the problem
6y 00 5y 0 + y = 0, y(0) = 4, y 0 (0) = 0.
We make the substitution y = ex in the differential equation.
62 5 + 1 = 0
(2 1)(3 1) = 0
 
1 1
= ,
3 2
The general solution of the differential equation is
y = c1 et/3 +c2 et/2 .
We apply the initial conditions to determine the constants.
c1 c2
c1 + c2 = 4, + =0
3 2
c1 = 12, c2 = 8
The solution subject to the initial conditions is

y = 12 et/3 8 et/2 .
The solution is plotted in Figure 17.1. The solution tends to as t .
2. We consider the problem
y 00 2y 0 + 5y = 0, y(/2) = 0, y 0 (/2) = 2.
We make the substitution y = ex in the differential equation.
2 2 + 5 = 0

=1 15
= {1 + 2, 1 2}

963
1 2 3 4 5
-5
-10
-15
-20
-25
-30

Figure 17.1: The solution of 6y 00 5y 0 + y = 0, y(0) = 4, y 0 (0) = 0.

The general solution of the differential equation is

y = c1 et cos(2t) + c2 et sin(2t).

We apply the initial conditions to determine the constants.

y(/2) = 0 c1 e/2 = 0 c1 = 0
0
y (/2) = 2 2c2 e /2
=2 c2 = e/2

The solution subject to the initial conditions is

y = et/2 sin(2t).

The solution is plotted in Figure 17.2. The solution oscillates with an amplitude that tends to as t .

3. We consider the problem


y 00 + 4y 0 + 4y = 0, y(1) = 2, y 0 (1) = 1.

964
50
40
30
20
10
3 4 5 6
-10

Figure 17.2: The solution of y 00 2y 0 + 5y = 0, y(/2) = 0, y 0 (/2) = 2.

We make the substitution y = ex in the differential equation.

2 + 4 + 4 = 0
( + 2)2 = 0
= 2

The general solution of the differential equation is

y = c1 e2t +c2 t e2t .

We apply the initial conditions to determine the constants.

c1 e2 c2 e2 = 2, 2c1 e2 +3c2 e2 = 1
c1 = 7 e2 , c2 = 5 e2

The solution subject to the initial conditions is

y = (7 + 5t) e2(t+1)

965
2

1.5

0.5

-1 1 2 3 4 5

Figure 17.3: The solution of y 00 + 4y 0 + 4y = 0, y(1) = 2, y 0 (1) = 1.

The solution is plotted in Figure 17.3. The solution vanishes as t .

7 + 5t 5
lim (7 + 5t) e2(t+1) = lim 2(t+1)
= lim 2(t+1)
=0
t t e t 2 e

Solution 17.4

y 00 4y 0 + 13y = 0.
With the substitution y = ex we obtain

2 ex 4 ex +13 ex = 0
2 4 + 13 = 0
= 2 3i.

Thus two linearly independent solutions are

e(2+3i)x , and e(23i)x .

966
Noting that
e(2+3i)x = e2x [cos(3x) + sin(3x)]
e(23i)x = e2x [cos(3x) sin(3x)],

we can write the two linearly independent solutions

y1 = e2x cos(3x), y2 = e2x sin(3x).

Solution 17.5
We note that
y 000 y 00 + y 0 y = 0
is a constant coefficient equation. The substitution, y = ex , yields
3 2 + 1 = 0
( 1)( i)( + i) = 0.
The corresponding solutions are ex , ex , and ex . We can write the general solution as
y = c1 ex +c2 cos x + c3 sin x.

Solution 17.6
We start with the equation y 00 + y = 0. We substitute y = ex into the differential equation to obtain
2 + 1 = 0, = i.
A linearly independent set of solutions is
{ex , ex }.
The fundamental set of solutions has the form
y1 = c1 ex +c2 ex ,
y2 = c3 ex +c4 ex .

967
By applying the constraints

y1 (0) = 1, y10 (0) = 0,


y2 (0) = 0, y20 (0) = 1,

we obtain
ex + ex
y1 = = cos x,
2
ex + ex
y2 = = sin x.
2
Now consider the equation y 00 y = 0. By substituting y = ex we find that a set of solutions is

{ex , ex }.

By taking linear combinations of these we see that another set of solutions is

{cosh x, sinh x}.

Note that this is the fundamental set of solutions.


Next consider y 00 = 0. We can find the solutions by substituting y = ex or by integrating the equation twice. The
fundamental set of solutions as x = 0 is
{1, x}.
Note that if u(x) is a solution of a constant coefficient differential equation, then u(x + c) is also a solution. Also
note that if u(x) satisfies y(0) = a, y 0 (0) = b, then u(x x0 ) satisfies y(x0 ) = a, y 0 (x0 ) = b. Thus the fundamental
sets of solutions at x = 1 are
1. {cos(x 1), sin(x 1)},

2. {cosh(x 1), sinh(x 1)},

3. {1, x 1}.

968
Solution 17.7
Let y(t) denote the displacement of the mass from equilibrium. The forces on the mass are ky(t) due to the spring
and y 0 (t) due to friction. We equate the external forces to my 00 (t) to find the differential equation of the motion.

my 00 = ky y 0
0 k
y 00 + y + y=0
m m

The solution which satisfies the initial conditions y(0) = 0, y 0 (0) = 1 is


p 
t/(2m) 2m

e sinh 2 4km t/(2m) if 2 > km,

2 4km p 
y(t) = et/(2m) 2m sin 4km 2 t/(2m) if 2 < km,

4km2
t et/(2m)

if 2 = km.

We respectively call these cases: strongly damped, weakly damped and critically damped. In the case that m = k = 1
the solution is p 
e t/2 2 2 4 t/2

sinh if > 2,

2 4 p 
y(t) = et/2 2 sin 4 2 t/2 if < 2,
4 2

t et

if = 2.
Note that when t is large, t et is much smaller than et/2 for < 2. To prove this we examine the ratio of these
functions as t .
t et t
lim = lim (1/2)t
t et/2 t e
1
= lim
t (1 /2) e(1)t

=0

969
0.5
Strong
0.4 Weak
0.3 Critical
0.2
0.1

2 4 6 8 10
-0.1

Figure 17.4: Strongly, weakly and critically damped solutions.

Using this result, we see that the critically damped solution decays faster than the weakly damped solution.
We can write the strongly damped solution as

2  
2 2
et/2 p e 4 t/2 e 4 t/2 .
2 4

2 4 t/2
For large t, the dominant factor is e . Note that for > 2,
p p
2 4 = ( + 2)( 2) > 2.

Therefore we have the bounds p


2 < 2 4 < 0.
This shows that the critically damped solution decays faster than the strongly damped solution. = 2 gives the fastest
decaying solution. Figure 17.4 shows the solution for = 4, = 1 and = 2.

970
Solution 17.8
Clearly y = c cos(x ) satisfies the differential equation y 00 + y = 0. Since it is a two-parameter family of functions,
it must be the general solution.
Using a trigonometric identity we can rewrite the solution as

y = c cos cos x + c sin sin x.

Setting this equal to sin x gives us the two equations

c cos = 0,
c sin = 1,

which has the solutions c = 1, = (2n + 1/2), and c = 1, = (2n 1/2), for n Z.
Clearly y = c cosh(x ) satisfies the differential equation y 00 y = 0. Since it is a two-parameter family of
functions, it must be the general solution.
Using a trigonometric identity we can rewrite the solution as

y = c cosh cosh x + c sinh sinh x.

Setting this equal to sinh x gives us the two equations

c cosh = 0,
c sinh = 1,

which has the solutions c = i, = (2n + 1/2), and c = i, = (2n 1/2), for n Z.

Solution 17.9
We substitute y = et into the differential equation.

2 et +5 et +6 et = 0
2 + 5 + 6 = 0
( + 2)( + 3) = 0

971
The general solution of the differential equation is

y = c1 e2t +c2 e3t .

The initial conditions give us the constraints:

c1 + c2 = 1,
2c1 3c2 = V.

The solution subject to the initial conditions is

y = (3 + V ) e2t (2 + V ) e3t .

This solution will be non-negative for t > 0 if V 3.


Solution 17.10
For negative x, the differential equation is
y 00 y = 0.
We substitute y = ex into the differential equation to find the solutions.

2 1 = 0
= 1
y = ex , ex


We can take linear combinations to write the solutions in terms of the hyperbolic sine and cosine.

y = {cosh(x), sinh(x)}

For positive x, the differential equation is


y 00 + y = 0.

972
We substitute y = ex into the differential equation to find the solutions.

2 + 1 = 0
=
y = ex , ex


We can take linear combinations to write the solutions in terms of the sine and cosine.

y = {cos(x), sin(x)}

We will find the fundamental set of solutions at x = 0. That is, we will find a set of solutions, {y1 , y2 } that satisfy
the conditions:

y1 (0) = 1 y10 (0) = 0


y2 (0) = 0 y20 (0) = 1

Clearly, these solutions are


( (
cosh(x) x < 0 sinh(x) x < 0
y1 = y2 =
cos(x) x0 sin(x) x0

Euler Equations
Solution 17.11
We consider an Euler equation,
x2 y 00 + xy 0 + y = 0, x > 0.
We make the change of independent variable = ln x, u() = y(x) to obtain

u00 + u = 0.

973
We make the substitution u() = e .

2 + 1 = 0
= i

A set of linearly independent solutions for u() is

{e , e }.

Since
e + e e e
cos = and sin = ,
2 2
another linearly independent set of solutions is
{cos , sin }.
The general solution for y(x) is
y(x) = c1 cos(ln x) + c2 sin(ln x).

Solution 17.12
Consider the differential equation
x2 y 00 2xy + 2y = 0.
With the substitution y = x this equation becomes

( 1) 2 + 2 = 0
2 3 + 2 = 0
= 1, 2.

The general solution is then


y = c1 x + c2 x2 .

974
Solution 17.13
We note that
1
xy 000 + y 00 + y 0 = 0
x
is an Euler equation. The substitution y = x yields

3 32 + 2 + 2 + = 0
3 22 + 2 = 0.

The three roots of this algebraic equation are

= 0, = 1 + i, =1

The corresponding solutions to the differential equation are

y = x0 y = x1+ y = x1
y=1 y = x e ln x y = x e ln x .

We can write the general solution as

y = c1 + c2 x cos(ln x) + c3 sin(ln x).

Solution 17.14
We substitute y = x into the differential equation.

x2 y 00 + (2a + 1)xy 0 + by = 0
( 1) + (2a + 1) + b = 0
2 + 2a + b = 0

= a a2 b

975
For a2 > b then the general solution is

a2 b a2 b
y = c1 xa+ + c2 xa .

For a2 < b, then the general solution is



ba2 ba2
y = c1 xa+ + c2 xa .

By taking the sum and difference of these solutions, we can write the general solution as
   
y = c1 xa cos b a2 ln x + c2 xa sin b a2 ln x .

For a2 = b, the quadratic in lambda has a double root at = a. The general solution of the differential equation is

y = c1 xa + c2 xa ln x.

In summary, the general solution is:


 
a a2 b a2 b

x c 1 x + c 2 x if a2 > b,


y = xa c1 cos b a2 ln x + c2 sin b a2 ln x
 
if a2 < b,
xa (c + c ln x)

if a2 = b.
1 2

Solution 17.15
For a 6= 0, two linearly independent solutions of

y 00 a2 y = 0

are
y1 = eax , y2 = eax .
For a = 0, we have
y1 = e0x = 1, y2 = x e0x = x.

976
In this case the solution are defined by
 
ax d ax
y1 = [e ]a=0 , y2 = e .
da a=0

By the definition of differentiation, f 0 (0) is


f (a) f (a)
f 0 (0) = lim .
a0 2a
Thus the second solution in the case a = 0 is
eax eax
y2 = lim
a0 a
Consider the solutions
ex ex
y1 = eax , y2 = lim .
a
Clearly y1 is a solution for all a. For a 6= 0, y2 is a linear combination of eax and eax and is thus a solution. Since the
coefficient of eax in this linear combination is non-zero, it is linearly independent to y1 . For a = 0, y2 is one half the
derivative of eax evaluated at a = 0. Thus it is a solution.
For a 6= 0, two linearly independent solutions of
x2 y 00 + xy 0 a2 y = 0
are
y1 = xa , y2 = xa .
For a = 0, we have  
a d a
y1 = [x ]a=0 = 1, y2 = x = ln x.
da a=0
Consider the solutions
xa xa
y1 = xa , y2 =
a
Clearly y1 is a solution for all a. For a 6= 0, y2 is a linear combination of xa and xa and is thus a solution. For a = 0,
y2 is one half the derivative of xa evaluated at a = 0. Thus it is a solution.

977
Solution 17.16
1.
x2 y 00 2xy 0 + 2y = 0
We substitute y = x into the differential equation.

( 1) 2 + 2 = 0
2 3 + 2 = 0
( 1)( 2) = 0
y = c1 x + c2 x2

2.
x2 y 00 2y = 0
We substitute y = x into the differential equation.

( 1) 2 = 0
2 2 = 0
( + 1)( 2) = 0
c1
y= + c2 x2
x

3.
x2 y 00 xy 0 + y = 0
We substitute y = x into the differential equation.

( 1) + 1 = 0
2 2 + 1 = 0
( 1)2 = 0

978
Since there is a double root, the solution is:

y = c1 x + c2 x ln x.

Exact Equations
Solution 17.17
We note that
y 00 + y 0 sin x + y cos x = 0

is an exact equation.

d 0
[y + y sin x] = 0
dx
y 0 + y sin x = c1
d  cos x 
ye = c1 e cos x
dx
Z
y = c1 e cos x
e cos x dx + c2 ecos x

Equations Without Explicit Dependence on y


Reduction of Order
Solution 17.18

(1 x2 )y 00 2xy 0 + 2y = 0, 1 < x < 1

We substitute y = x into the differential equation to check that it is a solution.

(1 x2 )(0) 2x(1) + 2x = 0

979
We look for a second solution of the form y = xu. We substitute this into the differential equation and use the fact
that x is a solution.

(1 x2 )(xu00 + 2u0 ) 2x(xu0 + u) + 2xu = 0


(1 x2 )(xu00 + 2u0 ) 2x(xu0 ) = 0
(1 x2 )xu00 + (2 4x2 )u0 = 0
u00 2 4x2
=
u0 x(x2 1)
00
u 2 1 1
= +
u0 x 1x 1+x
0
ln(u ) = 2 ln(x) ln(1 x) ln(1 + x) + const
 
0 c
ln(u ) = ln
x2 (1 x)(1 + x)
c
u0 = 2
x (1 x)(1 + x)
 
0 1 1 1
u =c + +
x2 2(1 x) 2(1 + x)
 
1 1 1
u = c ln(1 x) + ln(1 + x) + const
x 2 2
  
1 1 1+x
u = c + ln + const
x 2 1x

A second linearly independent solution is


 
x 1+x
y = 1 + ln .
2 1x

980
Solution 17.19
We are given that y = ex is a solution of
x+1 0 1
y 00 y + y = 0.
x x
To find another linearly independent solution, we will use reduction of order. Substituting

y = u ex
y 0 = (u0 + u) ex
y 00 = (u00 + 2u0 + u) ex

into the differential equation yields

x+1 0 1
u00 + 2u0 + u (u + u) + u = 0.
x x
x 1
u00 + u0 = 0
 Z  x  
d 0 1
u exp 1 dx =0
dx x
u0 exln x = c1
u0 = c1 x ex
Z
u = c1 x ex dx + c2

u = c1 (x ex + ex ) + c2
y = c1 (x + 1) + c2 ex

Thus a second linearly independent solution is

y = x + 1.

981
Solution 17.20
We are given that y = x is a solution of

(1 2x)y 00 + 4xy 0 4y = 0.

To find another linearly independent solution, we will use reduction of order. Substituting

y = xu
y 0 = xu0 + u
y 00 = xu00 + 2u0

into the differential equation yields

(1 2x)(xu00 + 2u0 ) + 4x(xu0 + u) 4xu = 0,


(1 2x)xu00 + (4x2 4x + 2)u0 = 0,
u00 4x2 4x + 2
= ,
u0 x(2x 1)
u00 2 2
0
=2 + ,
u x 2x 1
ln(u0 ) = 2x 2 ln x + ln(2x 1) + const,
 
0 2 1
u = c1 e2x ,
x x2
1
u = c1 e2x +c2 ,
x
y = c1 e2x +c2 x.

Solution 17.21
One solution of
(x 1)y00 xy0 + y = 0,

982
is y1 = ex . We find a second solution with reduction of order. We make the substitution y2 = u ex in the differential
equation. We determine u up to an additive constant.

(x 1)(u00 + 2u0 + u) ex x(u0 + u) ex +u ex = 0


(x 1)u00 + (x 2)u0 = 0
u00 x2 1
0
= = 1 +
u x1 x1
0
ln |u | = x + ln |x 1| + c
u0 = c(x 1) ex
u = cx ex

The second solution of the differential equation is y2 = x.

*Reduction of Order and the Adjoint Equation

983
Chapter 18

Techniques for Nonlinear Differential


Equations

In mathematics you dont understand things. You just get used to them.

- Johann von Neumann

18.1 Bernoulli Equations


Sometimes it is possible to solve a nonlinear equation by making a change of the dependent variable that converts it
into a linear equation. One of the most important such equations is the Bernoulli equation

dy
+ p(t)y = q(t)y , 6= 1.
dt

The change of dependent variable u = y 1 will yield a first order linear equation for u which when solved will give us
an implicit solution for y. (See Exercise 18.4.)

984
Result 18.1.1 The Bernoulli equation y 0 + p(t)y = q(t)y , 6= 1 can be transformed to
the first order linear equation
du
+ (1 )p(t)u = (1 )q(t)
dt
with the change of variables u = y 1 .

Example 18.1.1 Consider the Bernoulli equation

2
y0 = y + y2.
x

First we divide by y 2 .

2 1
y 2 y 0 = y +1
x

We make the change of variable u = y 1 .

2
u0 = u+1
x
2
u0 + u = 1
x

985
2
R
The integrating factor is I(x) = exp( x
dx) = x2 .

d 2
(x u) = x2
dx
1
x2 u = x3 + c
3
1 c
u= x+ 2
3 x
 1
1 c
y = x+ 2
3 x
Thus the solution for y is

3x2
y= .
c x2

18.2 Riccati Equations


Factoring Second Order Operators. Consider the second order linear equation
 2 
d d
L[y] = + p(x) + q(x) y = y 00 + p(x)y 0 + q(x)y = f (x).
dx2 dx
If we were able to factor the linear operator L into the form
  
d d
L= + a(x) + b(x) , (18.1)
dx dx
then we would be able to solve the differential equation. Factoring reduces the problem to a system of first order
equations. We start with the factored equation
  
d d
+ a(x) + b(x) y = f (x).
dx dx

986
d 
We set u = dx
+ b(x) y and solve the problem

 
d
+ a(x) u = f (x).
dx

Then to obtain the solution we solve


 
d
+ b(x) y = u.
dx

Example 18.2.1 Consider the equation


   
00 1 0 1
y + x y + 1 y = 0.
x x2

Lets say by some insight or just random luck we are able to see that this equation can be factored into
  
d d 1
+x y = 0.
dx dx x

We first solve the equation


 
d
+ x u = 0.
dx
u0 + xu = 0
d  x2 /2 
e u =0
dx
2
u = c1 ex /2

987
Then we solve for y with the equation
 
d 1 2
y = u = c1 ex /2 .
dx x
1 2
y 0 y = c1 ex /2
x
d 2
x y = c1 x1 ex /2
1

dx
Z
2
y = c1 x x1 ex /2 dx + c2 x

If we were able to solve for a and b in Equation 18.1 in terms of p and q then we would be able to solve any second
order differential equation. Equating the two operators,
d2
  
d d d
+p +q = +a +b
dx2 dx dx dx
d2 d
= 2 + (a + b) + (b0 + ab).
dx dx
Thus we have the two equations
a + b = p, and b0 + ab = q.
Eliminating a,

b0 + (p b)b = q
b0 = b2 pb + q

Now we have a nonlinear equation for b that is no easier to solve than the original second order linear equation.

Riccati Equations. Equations of the form

y 0 = a(x)y 2 + b(x)y + c(x)

988
are called Riccati equations. From the above derivation we see that for every second order differential equation there
is a corresponding Riccati equation. Now we will show that the converse is true.
We make the substitution
u0 u00 (u0 )2 a0 u0
y= , y0 = + + 2 ,
au au au2 au
in the Riccati equation.

y 0 = ay 2 + by + c
u00 (u0 )2 a0 u0 (u0 )2 u0
+ + = a b +c
au au2 a2 u a2 u 2 au
u00 a0 u0 u0
+ 2 +b c=0
au  a u  au
a0
u00 + b u0 + acu = 0
a

Now we have a second order linear equation for u.


0
u
Result 18.2.1 The substitution y = au transforms the Riccati equation

y 0 = a(x)y 2 + b(x)y + c(x)

into the second order linear equation


a0
 
u00 + b u0 + acu = 0.
a

Example 18.2.2 Consider the Riccati equation


1 1
y0 = y2 + y + 2 .
x x

989
0
With the substitution y = uu we obtain
1 1
u00 u0 + 2 u = 0.
x x

This is an Euler equation. The substitution u = x yields

( 1) + 1 = ( 1)2 = 0.

Thus the general solution for u is


u = c1 x + c2 x log x.
0
Since y = uu ,

c1 + c2 (1 + log x)
y=
c1 x + c2 x log x
1 + c(1 + log x)
y=
x + cx log x

18.3 Exchanging the Dependent and Independent Variables


Some differential equations can be put in a more elementary form by exchanging the dependent and independent
variables. If the new equation can be solved, you will have an implicit solution for the initial equation. We will consider
a few examples to illustrate the method.

Example 18.3.1 Consider the equation


1
y0 = .
y3 xy 2

990
Instead of considering y to be a function of x, consider x to be a function of y. That is, x = x(y), x0 = dx
dy
.

dy 1
= 3
dx y xy 2
dx
= y 3 xy 2
dy
x0 + y 2 x = y 3

Now we have a first order equation for x.


d  y3 /3  3
e x = y 3 ey /3
dy
Z
y 3 /3 3 3
x=e y 3 ey /3 dy + c ey /3

Example 18.3.2 Consider the equation


y
y0 = .
y2
+ 2x
Interchanging the dependent and independent variables yields
1 y
=
x0 y 2 + 2x
x
x0 = y + 2
y
x
x0 2 = y
y
d 2
(y x) = y 1
dy
y 2 x = log y + c
x = y 2 log y + cy 2

991
Result 18.3.1 Some differential equations can be put in a simpler form by exchanging the
dependent and independent variables. Thus a differential equation for y(x) can be written as
an equation for x(y). Solving the equation for x(y) will give an implicit solution for y(x).

18.4 Autonomous Equations


Autonomous equations have no explicit dependence on x. The following are examples.
y 00 + 3y 0 2y = 0
y 00 = y + (y 0 )2
y 000 + y 00 y = 0
The change of variables u(y) = y 0 reduces an nth order autonomous equation in y to a non-autonomous equation
of order n 1 in u(y). Writing the derivatives of y in terms of u,
y 0 = u(y)
d
y 00 = u(y)
dx
dy d
= u(y)
dx dy
= y 0 u0
= u0 u
y 000 = (u00 u + (u0 )2 )u.
Thus we see that the equation for u(y) will have an order of one less than the original equation.

Result 18.4.1 Consider an autonomous differential equation for y(x), (autonomous equa-
tions have no explicit dependence on x.) The change of variables u(y) = y 0 reduces an nth
order autonomous equation in y to a non-autonomous equation of order n 1 in u(y).

992
Example 18.4.1 Consider the equation
y 00 = y + (y 0 )2 .
With the substitution u(y) = y 0 , the equation becomes

u 0 u = y + u2
u0 = u + yu1 .

We recognize this as a Bernoulli equation. The substitution v = u2 yields


1 0
v =v+y
2
v 0 2v = 2y
d 2y 
e v = 2y e2y
dy
Z
2y
v(y) = c1 e + e 2y
2y e2y dy
 Z 
2y 2y 2y 2y
v(y) = c1 e + e y e + e dy
 
2y 2y 2y 1 2y
v(y) = c1 e + e y e e
2
1
v(y) = c1 e2y y .
2
Now we solve for u.
 1/2
2y 1
u(y) = c1 e y .
2
 1/2
dy 2y 1
= c1 e y
dx 2

993
This equation is separable.
dy
dx =
1 1/2

c1 e2y y 2
Z
1
x + c2 = dy
1 1/2

c1 e2y y 2

Thus we finally have arrived at an implicit solution for y(x).

Example 18.4.2 Consider the equation


y 00 + y 3 = 0.
With the change of variables, u(y) = y 0 , the equation becomes
u0 u + y 3 = 0.
This equation is separable.
u du = y 3 dy
1 2 1
u = y 4 + c1
2 4
 1/2
1 4
u = 2c1 y
2
 1/2
0 1 4
y = 2c1 y
2
dy
= dx
(2c1 21 y 4 )1/2
Integrating gives us the implicit solution
Z
1
dy = x + c2 .
(2c1 12 y 4 )1/2

994
18.5 *Equidimensional-in-x Equations
Differential equations that are invariant under the change of variables x = c are said to be equidimensional-in-x. For
a familiar example from linear equations, we note that the Euler equation is equidimensional-in-x. Writing the new
derivatives under the change of variables,
d 1 d d2 1 d2
x = c , = , = , ....
dx c d dx2 c2 d 2
Example 18.5.1 Consider the Euler equation
2 3
y 00 + y 0 + 2 y = 0.
x x
Under the change of variables, x = c , y(x) = u(), this equation becomes
1 00 2 1 0 3
2
u + u + 2 2u = 0
c c c c
2 3
u00 + u0 + 2 u = 0.

Thus this equation is invariant under the change of variables x = c .

Example 18.5.2 For a nonlinear example, consider the equation


y 00 y0
y 00 y 0 + + 2 = 0.
xy x
With the change of variables x = c , y(x) = u() the equation becomes
u00 u0 u00 u0
+ + =0
c2 c c3 u c3 2
u00 u0
u00 u0 + + 2 = 0.
u
We see that this equation is also equidimensional-in-x.

995
You may recall that the change of variables x = et reduces an Euler equation to a constant coefficient equation. To
generalize this result to nonlinear equations we will see that the same change of variables reduces an equidimensional-in-x
equation to an autonomous equation.
Writing the derivatives with respect to x in terms of t,

d dt d d
x = et , = = et
dx dx dt dt

d d
=x
dx dt
d2 d2
 
d d d d
x2 2 = x x x = 2 .
dx dx dx dx dt dt

Example 18.5.3 Consider the equation in Example 18.5.2

y 00 y0
y 00 y 0 + + 2 = 0.
xy x

Applying the change of variables x = et , y(x) = u(t) yields an autonomous equation for u(t).

x2 y 00
x2 y 00 x y 0 + + x y0 = 0
y
u00 u0
(u00 u0 )u0 + + u0 = 0
u

Result 18.5.1 A differential equation that is invariant under the change of variables x = c
is equidimensional-in-x. Such an equation can be reduced to autonomous equation of the
same order with the change of variables, x = et .

996
18.6 *Equidimensional-in-y Equations
A differential equation is said to be equidimensional-in-y if it is invariant under the change of variables y(x) = c v(x).
Note that all linear homogeneous equations are equidimensional-in-y.

Example 18.6.1 Consider the linear equation


y 00 + p(x)y 0 + q(x)y = 0.
With the change of variables y(x) = cv(x) the equation becomes
cv 00 + p(x)cv 0 + q(x)cv = 0
v 00 + p(x)v 0 + q(x)v = 0
Thus we see that the equation is invariant under the change of variables.

Example 18.6.2 For a nonlinear example, consider the equation


y 00 y + (y 0 )2 y 2 = 0.
Under the change of variables y(x) = cv(x) the equation becomes.
cv 00 cv + (cv 0 )2 (cv)2 = 0
v 00 v + (v 0 )2 v 2 = 0.
Thus we see that this equation is also equidimensional-in-y.

The change of variables y(x) = eu(x) reduces an nth order equidimensional-in-y equation to an equation of order
n 1 for u0 . Writing the derivatives of eu(x) ,
d u
e = u 0 eu
dx
d2 u
2
e = (u00 + (u0 )2 ) eu
dx
d3 u
e = (u000 + 3u00 u00 + (u0 )3 ) eu .
dx3

997
Example 18.6.3 Consider the linear equation in Example 18.6.1

y 00 + p(x)y 0 + q(x)y = 0.

Under the change of variables y(x) = eu(x) the equation becomes

(u00 + (u0 )2 ) eu +p(x)u0 eu +q(x) eu = 0


u00 + (u0 )2 + p(x)u0 + q(x) = 0.

Thus we have a Riccati equation for u0 . This transformation might seem rather useless since linear equations are
usually easier to work with than nonlinear equations, but it is often useful in determining the asymptotic behavior of
the equation.

Example 18.6.4 From Example 18.6.2 we have the equation

y 00 y + (y 0 )2 y 2 = 0.

The change of variables y(x) = eu(x) yields

(u00 + (u0 )2 ) eu eu +(u0 eu )2 (eu )2 = 0


u00 + 2(u0 )2 1 = 0
u00 = 2(u0 )2 + 1

998
v0
Now we have a Riccati equation for u0 . We make the substitution u0 = 2v
.

v 00 (v 0 )2 (v 0 )2
= 2 +1
2v 2v 2 4v 2
v 00 2v = 0

v = c1 e 2x +c2 e 2x

c 1 e 2x
c 2 e 2x
u0 = 2 2
c1 e 2x +c2 e 2x

c1 2 e 2x c2 2 e 2x
Z
u=2 dx + c3
c1 e 2x +c2 e 2x
 
u = 2 log c1 e 2x +c2 e 2x + c3
 2
y = c1 e 2x +c2 e 2x ec3

The constants are redundant, the general solution is

 2
y = c1 e 2x +c2 e 2x

Result 18.6.1 A differential equation is equidimensional-in-y if it is invariant under the


change of variables y(x) = cv(x). An nth order equidimensional-in-y equation can be re-
duced to an equation of order n 1 in u0 with the change of variables y(x) = eu(x) .

999
18.7 *Scale-Invariant Equations

Result 18.7.1 An equation is scale invariant if it is invariant under the change of variables,
x = c, y(x) = c v(), for some value of . A scale-invariant equation can be transformed
to an equidimensional-in-x equation with the change of variables, y(x) = x u(x).

Example 18.7.1 Consider the equation


y 00 + x2 y 2 = 0.
Under the change of variables x = c, y(x) = c v() this equation becomes
c 00
v () + c2 x2 c2 v 2 () = 0.
c2
Equating powers of c in the two terms yields = 4.
Introducing the change of variables y(x) = x4 u(x) yields

d2  4
+ x2 (x4 u(x))2 = 0

2
x u(x)
dx
x4 u00 8x5 u0 + 20x6 u + x6 u2 = 0
x2 u00 8xu0 + 20u + u2 = 0.

We see that the equation for u is equidimensional-in-x.

1000
18.8 Exercises
Exercise 18.1
1. Find the general solution and the singular solution of the Clairaut equation,

y = xp + p2 .

2. Show that the singular solution is the envelope of the general solution.
Hint, Solution

Bernoulli Equations
Exercise 18.2 (mathematica/ode/techniques nonlinear/bernoulli.nb)
Consider the Bernoulli equation
dy
+ p(t)y = q(t)y .
dt
1. Solve the Bernoulli equation for = 1.

2. Show that for 6= 1 the substitution u = y 1 reduces Bernoullis equation to a linear equation.

3. Find the general solution to the following equations.


dy
t2 + 2ty y 3 = 0, t > 0
dt
(a)
dy
+ 2xy + y 2 = 0
dx
(b)

Hint, Solution

1001
Exercise 18.3
Consider a population, y. Let the birth rate of the population be proportional to y with constant of proportionality 1.
Let the death rate of the population be proportional to y 2 with constant of proportionality 1/1000. Assume that the
population is large enough so that you can consider y to be continuous. What is the population as a function of time
if the initial population is y0 ?
Hint, Solution
Exercise 18.4
Show that the transformation u = y 1n reduces the equation to a linear first order equation. Solve the equations

dy
1. t2 + 2ty y 3 = 0 t > 0
dt
dy
2. = ( cos t + T ) y y 3 , and T are real constants. (From a fluid flow stability problem.)
dt
Hint, Solution

Riccati Equations
Exercise 18.5
1. Consider the Ricatti equation,
dy
= a(x)y 2 + b(x)y + c(x).
dx
Substitute
1
y = yp (x) +
u(x)
into the Ricatti equation, where yp is some particular solution to obtain a first order linear differential equation
for u.

2. Consider a Ricatti equation,


y 0 = 1 + x2 2xy + y 2 .

1002
Verify that yp (x) = x is a particular solution. Make the substitution y = yp + 1/u to find the general solution.
What would happen if you continued this method, taking the general solution for yp ? Would you be able to find
a more general solution?

3. The substitution
u0
y=
au
gives us the second order, linear, homogeneous differential equation,
 0 
00 a
u + b u0 + acu = 0.
a

The general solution for u has two constants of integration. However, the solution for y should only have one
constant of integration as it satisfies a first order equation. Write y in terms of the solution for u and verify tha
y has only one constant of integration.
Hint, Solution

Exchanging the Dependent and Independent Variables


Exercise 18.6
Solve the differential equation

0 y
y = .
xy + y
Hint, Solution

Autonomous Equations
*Equidimensional-in-x Equations
*Equidimensional-in-y Equations
*Scale-Invariant Equations

1003
18.9 Hints
Hint 18.1

Bernoulli Equations
Hint 18.2

Hint 18.3
The differential equation governing the population is

dy y2
=y , y(0) = y0 .
dt 1000
This is a Bernoulli equation.
Hint 18.4

Riccati Equations
Hint 18.5

Exchanging the Dependent and Independent Variables


Hint 18.6
Exchange the dependent and independent variables.

Autonomous Equations
*Equidimensional-in-x Equations

1004
*Equidimensional-in-y Equations
*Scale-Invariant Equations

1005
18.10 Solutions
Solution 18.1
We consider the Clairaut equation,
y = xp + p2 . (18.2)

1. We differentiate Equation 18.2 with respect to x to obtain a second order differential equation.

y 0 = y 0 + xy 00 + 2y 0 y 00
y 00 (2y 0 + x) = 0

Equating the first or second factor to zero will lead us to two distinct solutions.
x
y 00 = 0 or y 0 =
2
If y 00 = 0 then y 0 p is a constant, (say y 0 = c). From Equation 18.2 we see that the general solution is,

y(x) = cx + c2 . (18.3)

Recall that the general solution of a first order differential equation has one constant of integration.
If y 0 = x/2 then y = x2 /4 + const. We determine the constant by substituting the expression into Equa-
tion 18.2.
x2  x   x 2
+c=x +
4 2 2
Thus we see that a singular solution of the Clairaut equation is

1
y(x) = x2 . (18.4)
4

Recall that a singular solution of a first order nonlinear differential equation has no constant of integration.

1006
2

-4 -2 2 4

-2

-4

Figure 18.1: The Envelope of y = cx + c2 .

2. Equating the general and singular solutions, y(x), and their derivatives, y 0 (x), gives us the system of equations,
1 1
cx + c2 = x2 , c = x.
4 2
Since the first equation is satisfied for c = x/2, we see that the solution y = cx + c2 is tangent to the solution
y = x2 /4 at the point (2c, |c|). The solution y = cx + c2 is plotted for c = . . . , 1/4, 0, 1/4, . . . in
Figure 18.1.
The envelope of a one-parameter family F (x, y, c) = 0 is given by the system of equations,
F (x, y, c) = 0, Fc (x, y, c) = 0.
For the family of solutions y = cx + c2 these equations are
y = cx + c2 , 0 = x + 2c.
Substituting the solution of the second equation, c = x/2, into the first equation gives the envelope,
   2
1 1 1
y = x x + x = x2 .
2 2 4

1007
Thus we see that the singular solution is the envelope of the general solution.

Bernoulli Equations
Solution 18.2
1.

dy
+ p(t)y = q(t)y
dt
dy
= (q p) dt
y
Z
ln y = (q p) dt + c
R
(qp) dt
y = ce

2. We consider the Bernoulli equation,


dy
+ p(t)y = q(t)y , 6= 1.
dt
We divide by y .
y y 0 + p(t)y 1 = q(t)

This suggests the change of dependent variable u = y 1 , u0 = (1 )y y 0 .

1 d 1
y + p(t)y 1 = q(t)
1 dt
du
+ (1 )p(t)u = (1 )q(t)
dt
Thus we obtain a linear equation for u which when solved will give us an implicit solution for y.

1008
3. (a)
dy
t2 + 2ty y 3 = 0, t > 0
dt
y0 1
t2 3 + 2t 2 = 1
y y
We make the change of variables u = y 2 .
1
t2 u0 + 2tu = 1
2
4 2
u0 u = 2
t t
The integrating factor is R
=e (4/t) dt
= e4 ln t = t4 .
We multiply by the integrating factor and integrate to obtain the solution.
d 4 
t u = 2t6
dt
2
u = t1 + ct4
5
2
y 2 = t1 + ct4
5

1 5t
y = q y =
2 1
t + ct4 2 + ct5
5

(b)
dy
+ 2xy + y 2 = 0
dx
y0 2x
+ = 1
y2 y

1009
We make the change of variables u = y 1 .

u0 2xu = 1

The integrating factor is


2
R
=e (2x) dx
= ex .
We multiply by the integrating factor and integrate to obtain the solution.
d  x2  2
e u = ex
dx Z
2 2 2
u = ex ex dx + c ex
2
ex
y = R x2
e dx + c

Solution 18.3
The differential equation governing the population is

dy y2
=y , y(0) = y0 .
dt 1000
We recognize this as a Bernoulli equation. The substitution u(t) = 1/y(t) yields
du 1 1
=u , u(0) = .
dt 1000 y0
1
u0 + u =
1000
t
1 t et
Z
u= e + e d
y0 1000 0
 
1 1 1
u= + et
1000 y0 1000

1010
Solving for y(t),
   1
1 1 1 t
y(t) = + e .
1000 y0 1000

As a check, we see that as t , y(t) 1000, which is an equilibrium solution of the differential equation.
dy y2
=0=y y = 1000.
dt 1000
Solution 18.4
1.
dy
t2 + 2ty y 3 = 0
dt
dy
+ 2t1 y = t2 y 3
dt
We make the change of variables u(t) = y 2 (t).

u0 4t1 u = 2t2
This gives us a first order, linear equation. The integrating factor is
4t1 dt
R
I(t) = e = e4 log t = t4 .
We multiply by the integrating factor and integrate.
d 4 
t u = 2t6
dt
2
t4 u = t5 + c
5
2 1
u = t + ct4
5

1011
Finally we write the solution in terms of y(t).
1
y(t) = q
2 1
5
t + ct4

5t
y(t) =
2 + ct5

2.
dy
( cos t + T ) y = y 3
dt
We make the change of variables u(t) = y 2 (t).

u0 + 2 ( cos t + T ) u = 2
This gives us a first order, linear equation. The integrating factor is
R
2( cos t+T ) dt
I(t) = e = e2( sin t+T t)
We multiply by the integrating factor and integrate.
d 2( sin t+T t) 
e u = 2 e2( sin t+T t)
dt Z 
2( sin t+T t) 2( sin t+T t)
u = 2e e dt + c

Finally we write the solution in terms of y(t).

e sin t+T t
y = q R 
2 e2( sin t+T t) dt + c

1012
Riccati Equations
Solution 18.5
We consider the Ricatti equation,
dy
= a(x)y 2 + b(x)y + c(x). (18.5)
dx
1. We substitute
1
y = yp (x) +
u(x)
into the Ricatti equation, where yp is some particular solution.
u0
   
0 2 yp 1 1
yp 2 = +a(x) yp + 2 + 2 + b(x) yp + + c(x)
u u u u
u0
 
1 yp 1
2 = b(x) + a(x) 2 + 2
u u u u
u0 = (b + 2ayp ) u a

We obtain a first order linear differential equation for u whose solution will contain one constant of integration.
2. We consider a Ricatti equation,
y 0 = 1 + x2 2xy + y 2 . (18.6)
We verify that yp (x) = x is a solution.
1 = 1 + x2 2xx + x2
Substituting y = yp + 1/u into Equation 18.6 yields,

u0 = (2x + 2x) u 1
u = x + c
1
y =x+
cx

1013
1
What would happen if we continued this method? Since y = x + cx is a solution of the Ricatti equation we can
make the substitution,
1 1
y =x+ + , (18.7)
c x u(x)
which will lead to a solution for y which has two constants of integration. Then we could repeat the process,
substituting the sum of that solution and 1/u(x) into the Ricatti equation to find a solution with three constants
of integration. We know that the general solution of a first order, ordinary differential equation has only one
constant of integration. Does this method for Ricatti equations violate this theorem? Theres only one way to
find out. We substitute Equation 18.7 into the Ricatti equation.
  
0 1
u = 2x + 2 x + u1
cx
2
u0 = u1
cx
2
u0 + u = 1
cx
The integrating factor is
1
I(x) = e2/(cx) = e2 log(cx) = .
(c x)2
Upon multiplying by the integrating factor, the equation becomes exact.
 
d 1 1
2
u =
dx (c x) (c x)2
1
u = (c x)2 + b(c x)2
cx
u = x c + b(c x)2
Thus the Ricatti equation has the solution,
1 1
y =x+ + .
c x x c + b(c x)2

1014
It appears that we we have found a solution that has two constants of integration, but appearances can be
deceptive. We do a little algebraic simplification of the solution.
1 1
y =x+ +
c x (b(c x) 1)(c x)
(b(c x) 1) + 1
y =x+
(b(c x) 1)(c x)
b
y =x+
b(c x) 1
1
y =x+
(c 1/b) x

This is actually a solution, (namely the solution we had before), with one constant of integration, (namely c1/b).
Thus we see that repeated applications of the procedure will not produce more general solutions.

3. The substitution
u0
y=
au
gives us the second order, linear, homogeneous differential equation,
 0 
00 a
u + b u0 + acu = 0.
a
The solution to this linear equation is a linear combination of two homogeneous solutions, u1 and u2 .

u = c1 u1 (x) + c2 u2 (x)

The solution of the Ricatti equation is then


c1 u01 (x) + c2 u02 (x)
y= .
a(x)(c1 u1 (x) + c2 u2 (x))

1015
Since we can divide the numerator and denominator by either c1 or c2 , this answer has only one constant of
integration, (namely c1 /c2 or c2 /c1 ).

Exchanging the Dependent and Independent Variables

Solution 18.6
Exchanging the dependent and independent variables in the differential equation,


0 y
y = ,
xy + y

yields

x0 (y) = y 1/2 x + y 1/2 .

1016
This is a first order differential equation for x(y).

x0 y 1/2 x = y 1/2
2y 3/2 2y 3/2
    
d 1/2
x exp = y exp
dy 3 3
2y 3/2 2y 3/2
   
x exp = exp + c1
3 3
 3/2 
2y
x = 1 + c1 exp
3
 3/2 
x+1 2y
= exp
c1 3
 
x+1 2
log = y 3/2
c1 3
  2/3
3 x+1
y= log
2 c1
 2/3
3
y = c + log(x + 1)
2

Autonomous Equations
*Equidimensional-in-x Equations
*Equidimensional-in-y Equations
*Scale-Invariant Equations

1017
Chapter 19

Transformations and Canonical Forms

Prize intensity more than extent. Excellence resides in quality not in quantity. The best is always few and rare -
abundance lowers value. Even among men, the giants are usually really dwarfs. Some reckon books by the thickness,
as if they were written to exercise the brawn more than the brain. Extent alone never rises above mediocrity; it is the
misfortune of universal geniuses that in attempting to be at home everywhere are so nowhere. Intensity gives eminence
and rises to the heroic in matters sublime.

-Balthasar Gracian

19.1 The Constant Coefficient Equation


The solution of any second order linear homogeneous differential equation can be written in terms of the solutions to
either
y 00 = 0, or y 00 y = 0
Consider the general equation
y 00 + ay 0 + by = 0.

1018
We can solve this differential equation by making the substitution y = ex . This yields the algebraic equation
2 + a + b = 0.
1 
= a a2 4b
2
There are two cases to consider. If a2 =
6 4b then the solutions are

a2 4b)x/2 a2 4b)x/2
y1 = e(a+ , y2 = e(a
If a2 = 4b then we have
y1 = eax/2 , y2 = x eax/2
Note that regardless of the values of a and b the solutions are of the form
y = eax/2 u(x)

We would like to write the solutions to the general differential equation in terms of the solutions to simpler differential
equations. We make the substitution
y = ex u
The derivatives of y are
y 0 = ex (u0 + u)
y 00 = ex (u00 + 2u0 + 2 u)
Substituting these into the differential equation yields
u00 + (2 + a)u0 + (2 + a + b)u = 0
In order to get rid of the u0 term we choose
a
= .
2
The equation is then
a2
 
00
u + b u = 0.
4
There are now two cases to consider.

1019
Case 1. If b = a2 /4 then the differential equation is

u00 = 0

which has solutions 1 and x. The general solution for y is then

y = eax/2 (c1 + c2 x).

Case 2. If b 6= a2 /4 then the differential equation is


 2 
00 a
u b u = 0.
4

We make the change variables


u(x) = v(), x = .
The derivatives in terms of are
d d d 1 d
= =
dx dx d d
2
d 1 d 1 d 1 d2
= = .
dx2 d d 2 d 2
The differential equation for v is
a2
 
1 00
v b v =0
2 4
 2 
00 2 a
v b v =0
4
We choose 1/2
a2

= b
4

1020
to obtain
v 00 v = 0
which has solutions e . The solution for y is

y = ex c1 ex/ +c2 ex/




 2 
2
y = eax/2 c1 e a /4b x +c2 e a /4b x

19.2 Normal Form


19.2.1 Second Order Equations
Consider the second order equation
y 00 + p(x)y 0 + q(x)y = 0. (19.1)
Through a change of dependent variable, this equation can be transformed to

u00 + I(x)y = 0.

This is known as the normal form of (19.1). The function I(x) is known as the invariant of the equation.
Now to find the change of variables that will accomplish this transformation. We make the substitution y(x) =
a(x)u(x) in (19.1).
au00 + 2a0 u0 + a00 u + p(au0 + a0 u) + qau = 0
 0  00
pa0
 
00 a 0 a
u + 2 +p u + + +q u=0
a a a
To eliminate the u0 term, a(x) must satisfy
a0
2 +p=0
a
1
a0 + pa = 0
2

1021
 Z 
1
a = c exp p(x) dx .
2
For this choice of a, our differential equation for u becomes
p2 p0
 
00
u + q u = 0.
4 2
Two differential equations having the same normal form are called equivalent.
Result 19.2.1 The change of variables
 Z 
1
y(x) = exp p(x) dx u(x)
2
transforms the differential equation

y 00 + p(x)y 0 + q(x)y = 0

into its normal form


u00 + I(x)u = 0
where the invariant of the equation, I(x), is
p2 p0
I(x) = q .
4 2

19.2.2 Higher Order Differential Equations


Consider the third order differential equation
y 000 + p(x)y 00 + q(x)y 0 + r(x)y = 0.

1022
We can eliminate the y 00 term. Making the change of dependent variable
 Z 
1
y = u exp p(x) dx
3
   Z 
0 0 1 1
y = u pu exp p(x) dx
3 3
   Z 
00 00 2 0 1 2 0 1
y = u pu + (p 3p )u exp p(x) dx
3 9 3
   Z 
00 000 00 1 2 0 0 1 0 00 3 1
y = u pu + (p 3p )u + (9p 9p p )u exp p(x) dx
3 27 3

yields the differential equation

1 1
u000 + (3q 3p0 p2 )u0 + (27r 9pq 9p00 + 2p3 )u = 0.
3 27

Result 19.2.2 The change of variables


 Z 
1
y(x) = exp pn1 (x) dx u(x)
n
transforms the differential equation

y (n) + pn1 (x)y (n1) + pn2 (x)y (n2) + + p0 (x)y = 0

into the form

u(n) + an2 (x)u(n2) + an3 (x)u(n3) + + a0 (x)u = 0.

1023
19.3 Transformations of the Independent Variable
19.3.1 Transformation to the form u + a(x) u = 0
Consider the second order linear differential equation

y 00 + p(x)y 0 + q(x)y = 0.

We make the change of independent variable

= f (x), u() = y(x).

The derivatives in terms of are


d d d d
= = f0
dx dx d d
2 2
d 0 d 0 d 0 2 d d
2
= f f = (f ) 2
+ f 00
dx d d d d
The differential equation becomes
(f 0 )2 u00 + f 00 u0 + pf 0 u0 + qu = 0.
In order to eliminate the u0 term, f must satisfy

f 00 + pf 0 = 0
 Z 
0
f = exp p(x) dx
Z  Z 
f= exp p(x) dx dx.

The differential equation for u is then


q
u00 + u=0
(f 0 )2

1024
 Z 
00
u () + q(x) exp 2 p(x) dx u() = 0.

Result 19.3.1 The change of variables


Z  Z 
= exp p(x) dx dx, u() = y(x)

transforms the differential equation

y 00 + p(x)y 0 + q(x)y = 0

into  Z 
u00 () + q(x) exp 2 p(x) dx u() = 0.

19.3.2 Transformation to a Constant Coefficient Equation


Consider the second order linear differential equation
y 00 + p(x)y 0 + q(x)y = 0.
With the change of independent variable
= f (x), u() = y(x),
the differential equation becomes
(f 0 )2 u00 + (f 00 + pf 0 )u0 + qu = 0.
For this to be a constant coefficient equation we must have
(f 0 )2 = c1 q, and f 00 + pf 0 = c2 q,

1025
for some constants c1 and c2 . Solving the first condition,

f 0 = c q,
Z p
f =c q(x) dx.

The second constraint becomes


f 00 + pf 0
= const
q
1 1/2 0
2
cq q + pcq 1/2
= const
q
q 0 + 2pq
= const.
q 3/2

Result 19.3.2 Consider the differential equation

y 00 + p(x)y 0 + q(x)y = 0.

If the expression
q 0 + 2pq
q 3/2
is a constant then the change of variables
Z p
=c q(x) dx, u() = y(x),

will yield a constant coefficient differential equation. (Here c is an arbitrary constant.)

1026
19.4 Integral Equations
Volterras Equations. Volterras integral equation of the first kind has the form
Z x
N (x, )f () d = f (x).
a

The Volterra equation of the second kind is


Z x
y(x) = f (x) + N (x, )y() d.
a

N (x, ) is known as the kernel of the equation.

Fredholms Equations. Fredholms integral equations of the first and second kinds are
Z b
N (x, )f () d = f (x),
a

Z b
y(x) = f (x) + N (x, )y() d.
a

19.4.1 Initial Value Problems


Consider the initial value problem

y 00 + p(x)y 0 + q(x)y = f (x), y(a) = , y 0 (a) = .

Integrating this equation twice yields


Z xZ Z x Z
00 0
y () + p()y () + q()y() d d = f () d d
a a a a

1027
Z x Z x
00 0
(x )[y () + p()y () + q()y()] d = (x )f () d.
a a
Now we use integration by parts.
Z x Z x
0
x 0
x
[(x )p0 () p()]y() d
 
(x )y () a y () d + (x )p()y() a
a a
Z x Z x
+ (x )q()y() d = (x )f () d.
a a

Z x
0
(x a)y (a) + y(x) y(a) (x a)p(a)y(a) [(x )p0 () p()]y() d
a
Z x Z x
+ (x )q()y() d = (x )f () d.
a a

We obtain a Volterra integral equation of the second kind for y(x).


Z x Z x
(x )[p0 () q()] p() y() d.

y(x) = (x )f () d + (x a)(p(a) + ) + +
a a

Note that the initial conditions for the differential equation are built into the Volterra equation. Setting x = a in
the Volterra equation yields y(a) = . Differentiating the Volterra equation,
Z x Z x
0
y (x) = f () d + (p(a) + ) p(x)y(x) + [p0 () q()] p()y() d
a a

and setting x = a yields


y 0 (a) = p(a) + p(a) = .
(Recall from calculus that
Z x Z x
d
g(x, ) d = g(x, x) + [g(x, )] d.)
dx x

1028
Result 19.4.1 The initial value problem

y 00 + p(x)y 0 + q(x)y = f (x), y(a) = , y 0 (a) = .

is equivalent to the Volterra equation of the second kind


Z x
y(x) = F (x) + N (x, )y() d
a

where
Z x
F (x) = (x )f () d + (x a)(p(a) + ) +
a
N (x, ) = (x )[p0 () q()] p().

19.4.2 Boundary Value Problems


Consider the boundary value problem
y 00 = f (x), y(a) = , y(b) = . (19.2)
To obtain a problem with homogeneous boundary conditions, we make the change of variable

y(x) = u(x) + + (x a)
ba
to obtain the problem
u00 = f (x), u(a) = u(b) = 0.
Now we will use Greens functions to write the solution as an integral. First we solve the problem
G00 = (x ), G(a|) = G(b|) = 0.

1029
The homogeneous solutions of the differential equation that satisfy the left and right boundary conditions are
c1 (x a) and c2 (x b).
Thus the Greens function has the form
(
c1 (x a), for x
G(x|) =
c2 (x b), for x

Imposing continuity of G(x|) at x = and a unit jump of G(x|) at x = , we obtain


(
(xa)(b)
ba
, for x
G(x|) = (xb)(a)
ba
, for x

Thus the solution of the (19.2) is


b

Z
y(x) = + (x a) + G(x|)f () d.
ba a

Now consider the boundary value problem


y 00 + p(x)y 0 + q(x)y = 0, y(a) = , y(b) = .
From the above result we can see that the solution satisfies
Z b

y(x) = + (x a) + G(x|)[f () p()y 0 () q()y()] d.
ba a

Using integration by parts, we can write


Z b Z b 
0
 b G(x|) 0
G(x|)p()y () d = G(x|)p()y() a + p() + G(x|)p () y() d
a a
Z b 
G(x|) 0
= p() + G(x|)p () y() d.
a

1030
Substituting this into our expression for y(x),
b Z b 

Z
G(x|) 0
y(x) = + (x a) + G(x|)f () d + p() + G(x|)[p () q()] y() d,
ba a a

we obtain a Fredholm integral equation of the second kind.


Result 19.4.2 The boundary value problem

y 00 + p(x)y 0 + q(x)y = f (x), y(a) = , y(b) = .

is equivalent to the Fredholm equation of the second kind


Z b
y(x) = F (x) + N (x, )y() d
a

where
Z b

F (x) = + (x a) + G(x|)f () d,
ba a
Z b
N (x, ) = H(x|)y() d,
(a
(xa)(b)
ba , for x
G(x|) = (xb)(a)
, for x ,
( ba
(xa) (xa)(b) 0
ba p() + ba [p () q()] for x
H(x|) = (xb) (xb)(a) 0
ba p() + ba [p () q()] for x .

1031
19.5 Exercises
The Constant Coefficient Equation
Normal Form
Exercise 19.1
Solve the differential equation  
4 1
y + 2 + x y0 +
00
24 + 12x + 4x2 y = 0.

3 9
Hint, Solution

Transformations of the Independent Variable


Integral Equations
Exercise 19.2
Show that the solution of the differential equation
y 00 + 2(a + bx)y 0 + (c + dx + ex2 )y = 0
can be written in terms of one of the following canonical forms:
v 00 + ( 2 + A)v = 0
v 00 = v
v 00 + v = 0
v 00 = 0.
Hint, Solution
Exercise 19.3
Show that the solution of the differential equation
   
00 b 0 d e
y +2 a+ y + c+ + 2 y =0
x x x

1032
can be written in terms of one of the following canonical forms:
 
00 A B
v + 1+ + 2 v =0

 
1 A
v 00 + + v=0
2
A
v 00 + 2 v = 0

Hint, Solution
Exercise 19.4
Show that the second order Euler equation

d2 y dy
x2 2
+ a1 x + a0 y = 0
dx dx
can be transformed to a constant coefficient equation.
Hint, Solution
Exercise 19.5
Solve Bessels equation of order 1/2,  
00 1 0 1
y + y + 1 2 y = 0.
x 4x
Hint, Solution

1033
19.6 Hints
The Constant Coefficient Equation
Normal Form
Hint 19.1
Transform the equation to normal form.

Transformations of the Independent Variable


Integral Equations
Hint 19.2
Transform the equation to normal form and then apply the scale transformation x = + .

Hint 19.3
Transform the equation to normal form and then apply the scale transformation x = .

Hint 19.4
Make the change of variables x = et , y(x) = u(t). Write the derivatives with respect to x in terms of t.

x = et
dx = et dt
d d
= et
dx dt
d d
x =
dx dt
Hint 19.5
Transform the equation to normal form.

1034
19.7 Solutions
The Constant Coefficient Equation
Normal Form
Solution 19.1
 
4 1
y + 2 + x y0 +
00
24 + 12x + 4x2 y = 0

3 9
To transform the equation to normal form we make the substitution
 Z   
1 4
y = exp 2 + x dx u
2 3
2 /3
= exx u

The invariant of the equation is


 2  
1  1 4 1 d 4
I(x) = 24 + 12x + 4x2 2+ x 2+ x
9 4 3 2 dx 3
= 1.

The normal form of the differential equation is then

u00 + u = 0

which has the general solution


u = c1 cos x + c2 sin x
Thus the equation for y has the general solution
2 /3 2 /3
y = c1 exx cos x + c2 exx sin x.

1035
Transformations of the Independent Variable
Integral Equations
Solution 19.2
The substitution that will transform the equation to normal form is
 Z 
1
y = exp 2(a + bx) dx u
2
2 /2
= eaxbx u.
The invariant of the equation is
1 1 d
I(x) = c + dx + ex2 (2(a + bx))2 (2(a + bx))
4 2 dx
= c b a2 + (d 2ab)x + (e b2 )x2
+ x + x2
The normal form of the differential equation is
u00 + ( + x + x2 )u = 0
We consider the following cases:
= 0.
= 0.
= 0. We immediately have the equation
u00 = 0.
6= 0. With the change of variables
v() = u(x), x = 1/2 ,
we obtain
v 00 + v = 0.

1036
6= 0. We have the equation
y 00 + ( + x)y = 0.
The scale transformation x = + yields

v 00 + 2 ( + ( + ))y = 0

v 00 = [3 + 2 ( + )]v.
Choosing

= ()1/3 , =

yields the differential equation
v 00 = v.

6= 0. The scale transformation x = + yields

v 00 + 2 [ + ( + ) + ( + )2 ]v = 0

v 00 + 2 [ + + 2 + ( + 2) + 2 2 ]v = 0.
Choosing

= 1/4 , =
2
yields the differential equation
v 00 + ( 2 + A)v = 0
where
1
A = 1/2 3/2 .
4

1037
Solution 19.3
The substitution that will transform the equation to normal form is
 Z   
1 b
y = exp 2 a+ dx u
2 x
= xb eax u.

The invariant of the equation is


  2   
d e 1 b 1 d b
I(x) = c + + 2 2 a+ 2 a+
x x 4 x 2 dx x
2
d 2ab e + b b
= c ax + +
x x2

+ + 2.
x x
The invariant form of the differential equation is
 
00
u + + + 2 u = 0.
x x

We consider the following cases:

= 0.

= 0. We immediately have the equation



u00 + u = 0.
x2
6= 0. We have the equation  
00
u + + 2 u = 0.
x x

1038
The scale transformation u(x) = v(), x = yields
 
00
v + + 2 u = 0.

Choosing = 1 , we obtain  
00 1
v + + 2 u = 0.

6= 0. The scale transformation x = yields
 
00 2
v + + + 2 v = 0.

Choosing = 1/2 , we obtain


1/2
 
00
v + 1+ + 2 v = 0.

Solution 19.4
We write the derivatives with respect to x in terms of t.

x = et
dx = et dt
d d
= et
dx dt
d d
x =
dx dt
d 2
Now we express x2 dx 2 in terms of t.

d2 d2
 
2 d d d d
x 2
=x x x = 2
dx dx dx dx dt dt

1039
Thus under the change of variables, x = et , y(x) = u(t), the Euler equation becomes

u00 u0 + a1 u0 + a0 u = 0
u00 + (a1 1)u0 + a0 u = 0.

Solution 19.5
The transformation  
Z
1 1
y = exp dx = x1/2 u
2 x
will put the equation in normal form. The invariant is
   
1 1 1 1 1
I(x) = 1 2 2
= 1.
4x 4 x 2 x2

Thus we have the differential equation


u00 + u = 0,
with the solution
u = c1 cos x + c2 sin x.
The solution of Bessels equation of order 1/2 is

y = c1 x1/2 cos x + c2 x1/2 sin x.

1040
Chapter 20

The Dirac Delta Function

I do not know what I appear to the world; but to myself I seem to have been only like a boy playing on a seashore,
and diverting myself now and then by finding a smoother pebble or a prettier shell than ordinary, whilst the great ocean
of truth lay all undiscovered before me.

- Sir Issac Newton

20.1 Derivative of the Heaviside Function


The Heaviside function H(x) is defined (
0 for x < 0,
H(x) =
1 for x > 0.
The derivative of the Heaviside function is zero for x 6= 0. At x = 0 the derivative is undefined. We will represent
the derivative of the Heaviside function by the Dirac delta function, (x). The delta function is zero for x 6= 0 and
infinite at the point x = 0. Since the derivative of H(x) is undefined, (x) is not a function in the conventional sense
of the word. One can derive the properties of the delta function rigorously, but the treatment in this text will be almost
entirely heuristic.

1041
The Dirac delta function is defined by the properties

(

for x 6= 0,
Z
0
(x) = and (x) dx = 1.
for x = 0,

The second property comes from the fact that (x) represents the derivative of H(x). The Dirac delta function is
conceptually pictured in Figure 20.1.

Figure 20.1: The Dirac Delta Function.

Let f (x) be a continuous function that vanishes at infinity. Consider the integral

Z
f (x)(x) dx.

1042
We use integration by parts to evaluate the integral.
Z Z

f 0 (x)H(x) dx

f (x)(x) dx = f (x)H(x)

Z
= f 0 (x) dx
0
= [f (x)]
0
= f (0)
We assumed that f (x) vanishes at infinity in order to use integration by parts to evaluate the integral. However,
since the delta function is zero for x 6= 0, the integrand is nonzero only at x = 0. ThusR the behavior of the function

at infinity should not affect the value of the integral. Thus it is reasonable that f (0) = f (x)(x) dx holds for all
continuous functions. By changing variables and noting that (x) is symmetric we can derive a more general formula.
Z
f (0) = f ()() d

Z
f (x) = f ( + x)() d

Z
f (x) = f ()( x) d

Z
f (x) = f ()(x ) d

This formula is very important in solving inhomogeneous differential equations.

20.2 The Delta Function as a Limit


Consider a function b(x, ) defined by (
0 for |x| > /2
b(x, ) = 1

for |x| < /2.

1043
The graph of b(x, 1/10) is shown in Figure 20.2.

10

-1 1

Figure 20.2: Graph of b(x, 1/10).

The Dirac delta function (x) can be thought of as b(x, ) in the limit as  0. Note that the delta function so
defined satisfies the properties,

(

0 for x 6= 0
Z
(x) = and (x) dx = 1
for x = 0

Delayed Limiting Process. When the Dirac delta function appears inside an integral, we can think of the delta
function as a delayed limiting process.

Z Z
f (x)(x) dx lim f (x)b(x, ) dx.
0

1044
Let f (x) be a continuous function and let F 0 (x) = f (x). We compute the integral of f (x)(x).


1 /2
Z Z
f (x)(x) dx = lim f (x) dx
0  /2

1 /2
= lim [F (x)]/2
0 
F (/2) F (/2)
= lim
0 
= F 0 (0)
= f (0)

20.3 Higher Dimensions


We can define a Dirac delta function in n-dimensional Cartesian space, n (x), x Rn . It is defined by the following
two properties.

n (x) = 0 for x 6= 0
Z
n (x) dx = 1
Rn

It is easy to verify, that the n-dimensional Dirac delta function can be written as a product of 1-dimensional Dirac delta
functions.
n
Y
n (x) = (xk )
k=1

1045
20.4 Non-Rectangular Coordinate Systems
We can derive Dirac delta functions in non-rectangular coordinate systems by making a change of variables in the
relation, Z
n (x) dx = 1
Rn
Where the transformation is non-singular, one merely divides the Dirac delta function by the Jacobian of the transfor-
mation to the coordinate system.

Example 20.4.1 Consider the Dirac delta function in cylindrical coordinates, (r, , z). The Jacobian is J = r.
Z Z 2 Z
3 (x x0 ) r dr d dz = 1
0 0

For r0 6= 0, the Dirac Delta function is


1
3 (x x0 ) = (r r0 ) ( 0 ) (z z0 )
r
since it satisfies the two defining properties.
1
(r r0 ) ( 0 ) (z z0 ) = 0 for (r, , z) 6= (r0 , 0 , z0 )
r

Z Z 2 Z
1
(r r0 ) ( 0 ) (z z0 ) r dr d dz
0 0 r
Z Z 2 Z
= (r r0 ) dr ( 0 ) d (z z0 ) dz = 1
0 0

For r0 = 0, we have
1
3 (x x0 ) = (r) (z z0 )
2r

1046
since this again satisfies the two defining properties.
1
(r) (z z0 ) = 0 for (r, z) 6= (0, z0 )
2r
Z Z 2 Z Z Z 2 Z
1 1
(r) (z z0 ) r dr d dz = (r) dr d (z z0 ) dz = 1
0 0 2r 2 0 0

1047
20.5 Exercises
Exercise 20.1
Let f (x) be a function that is continuous except for a jump discontinuity at x = 0. Using a delayed limiting process,
show that Z
f (0 ) + f (0+ )
= f (x)(x) dx.
2
Hint, Solution
Exercise 20.2
Show that the Dirac delta function is symmetric.
(x) = (x)
Hint, Solution
Exercise 20.3
Show that
(x)
(cx) = .
|c|
Hint, Solution
Exercise 20.4
We will consider the Dirac delta function with a function as on argument, (y(x)). Assume that y(x) has simple zeros
at the points {xn }.
y(xn ) = 0, y 0 (xn ) 6= 0
Further assume that y(x) has no multiple zeros. (If y(x) has multiple zeros (y(x)) is not well-defined in the same
sense that 1/0 is not well-defined.) Prove that
X (x xn )
(y(x)) = .
n
|y 0 (xn )|
Hint, Solution

1048
Exercise 20.5
Justify the identity Z
f (x) (n) (x) dx = (1)n f (n) (0)

From this show that


(n) (x) = (1)n (n) (x) and x (n) (x) = n (n1) (x).
Hint, Solution
Exercise 20.6
Consider x = (x1 , . . . , xn ) Rn and the curvilinear coordinate system = (1 , . . . , n ). Show that

( )
(x a) =
|J|

where a and are corresponding points in the two coordinate systems and J is the Jacobian of the transformation
from x to .
x
J

Hint, Solution
Exercise 20.7
Determine the Dirac delta function in spherical coordinates, (r, , ).

x = r cos sin , y = r sin sin , z = r cos

Hint, Solution

1049
20.6 Hints
Hint 20.1

Hint 20.2
Verify that (x) satisfies the two properties of the Dirac delta function.

Hint 20.3
Evaluate the integral,
Z
f (x)(cx) dx,

by noting that the Dirac delta function is symmetric and making a change of variables.

Hint 20.4
Let the points {m } partition the interval ( . . . ) such that y 0 (x) is monotone on each interval (m . . . m+1 ).
Consider some such interval, (a . . . b) (m . . . m+1 ). Show that
(R
Z b (y)
|y 0 (xn )|
dy if y(xn ) = 0 for a < xn < b
(y(x)) dx =
a 0 otherwise

for = min(y(a), y(b)) and = max(y(a), y(b)). Now consider the integral on the interval ( . . . ) as the sum
of integrals on the intervals {(m . . . m+1 )}.

Hint 20.5
Justify the identity,
Z
f (x) (n) (x) dx = (1)n f (n) (0),

with integration by parts.

1050
Hint 20.6
The Dirac delta function is defined by the following two properties.

(x a) = 0 for x 6= a
Z
(x a) dx = 1
Rn

Verify that ( )/|J| satisfies these properties in the coordinate system.

Hint 20.7
Consider the special cases 0 = 0, and r0 = 0.

1051
20.7 Solutions
Solution 20.1
Let F 0 (x) = f (x).

Z Z
1
f (x)(x) dx = lim f (x)b(x, ) dx
0 
!
Z 0 Z /2
1
= lim f (x)b(x, ) dx + f (x)b(x, ) dx
0  /2 0
1
= lim ((F (0) F (/2)) + (F (/2) F (0)))
0 
 
1 F (0) F (/2) F (/2) F (0)
= lim +
0 2 /2 /2
0 0 +
F (0 ) + F (0 )
=
2

f (0 ) + f (0+ )
=
2

Solution 20.2
(x) satisfies the two properties of the Dirac delta function.

(x) = 0 for x 6= 0
Z Z Z
(x) dx = (x) (dx) = (x) dx = 1

Therefore (x) = (x).

1052
Solution 20.3
We note the the Dirac delta function is symmetric and we make a change of variables to derive the identity.

Z Z
(cx) dx = (|c|x) dx

Z

(x)
= dx
|c|

(x)
(cx) =
|c|

Solution 20.4
Let the points {m } partition the interval ( . . . ) such that y 0 (x) is monotone on each interval (m . . . m+1 ).
Consider some such interval, (a . . . b) (m . . . m+1 ). Note that y 0 (x) is either entirely positive or entirely negative in
the interval. First consider the case when it is positive. In this case y(a) < y(b).

Z b Z y(b)  1
dy
(y(x)) dx = (y) dy
a y(a) dx
Z y(b)
(y)
= dy
y(a) y 0 (x)
(R y(b)
(y)
y(a) y 0 (xn )
dy for y(xn ) = 0 if y(a) < 0 < y(b)
=
0 otherwise

1053
Now consider the case that y 0 (x) is negative on the interval so y(a) > y(b).

Z b Z y(b)  1
dy
(y(x)) dx = (y) dy
a y(a) dx
Z y(b)
(y)
= dy
y(a) y 0 (x)
Z y(a)
(y)
= dy
y(b) y 0 (x)
(R y(a)
(y)
y(b) y 0 (xn )
dy for y(xn ) = 0 if y(b) < 0 < y(a)
=
0 otherwise

We conclude that

(R
b (y)
dy if y(xn ) = 0 for a < xn < b
Z
|y 0 (xn )|
(y(x)) dx =
a 0 otherwise

for = min(y(a), y(b)) and = max(y(a), y(b)).

1054
Now we turn to the integral of (y(x)) on ( . . . ). Let m = min(y(m ), y(m )) and m = max(y(m ), y(m )).

Z XZ m+1
(y(x)) dx = (y(x)) dx
m m
X Z m+1
= (y(x)) dx
m m
xn (m ...m+1 )
Z m+1
X (y)
= dy
m m |y 0 (xn )|
xn (m ...m+1 )
X Z
(y)
= dy
|y 0 (xn )|
Zn X
(y)
= 0
dy
n |y (xn )|

X (x xn )
(y(x)) =
n
|y 0 (xn )|

Solution 20.5
To justify the identity,

Z
f (x) (n) (x) dx = (1)n f (n) (0),

1055
we will use integration by parts.

Z 
Z
(n) (n1)
f 0 (x) (n1) (x) dx

f (x) (x) dx = f (x) (x)

Z
= f 0 (x) (n1) (x) dx

Z
n
= (1) f (n) (x)(x) dx

n (n)
= (1) f (0)

CONTINUE HERE

(n) (x) = (1)n (n) (x) and x (n) (x) = n (n1) (x).

Solution 20.6
The Dirac delta function is defined by the following two properties.

(x a) = 0 for x 6= a
Z
(x a) dx = 1
Rn

We verify that ( )/|J| satisfies these properties in the coordinate system.

( ) (1 1 ) (n n )
=
|J| |J|
= 0 for 6=

1056
( )
Z Z
|J| d = ( ) d
|J|
Z
= (1 1 ) (n n ) d
Z Z
= (1 1 ) d1 (n n ) dn

=1
We conclude that ( )/|J| is the Dirac delta function in the coordinate system.
( )
(x a) =
|J|
Solution 20.7
We consider the Dirac delta function in spherical coordinates, (r, , ). The Jacobian is J = r2 sin().
Z Z 2 Z
3 (x x0 ) r2 sin() dr d d = 1
0 0 0

For r0 6= 0, and 0 6= 0, , the Dirac Delta function is


1
3 (x x0 ) = (r r0 ) ( 0 ) ( 0 )
r2 sin()
since it satisfies the two defining properties.
1
(r r0 ) ( 0 ) ( 0 ) = 0 for (r, , ) 6= (r0 , 0 , 0 )
r2 sin()

Z Z 2 Z
1
(r r0 ) ( 0 ) ( 0 ) r2 sin() dr d d
0 0 0 r2 sin()
Z Z 2 Z
= (r r0 ) dr ( 0 ) d ( 0 ) d = 1
0 0 0

1057
For 0 = 0 or 0 = , the Dirac delta function is
1
3 (x x0 ) = (r r0 ) ( 0 ) .
2r2 sin()

We check that the value of the integral is unity.


Z Z 2 Z
1
(r r0 ) ( 0 ) r2 sin() dr d d
0 0 0 2r2 sin()
Z Z 2 Z
1
= (r r0 ) dr d ( 0 ) d = 1
2 0 0 0

For r0 = 0 the Dirac delta function is


1
3 (x) = (r)
4r2
We verify that the value of the integral is unity.
Z Z 2 Z Z Z 2 Z
1 2 1
(r r0 ) r sin() dr d d = (r) dr d sin() d = 1
0 0 0 4r2 4 0 0 0

1058
Chapter 21

Inhomogeneous Differential Equations

Feelin stupid? I know I am!


-Homer Simpson

21.1 Particular Solutions


Consider the nth order linear homogeneous equation
L[y] y (n) + pn1 (x)y (n1) + + p1 (x)y 0 + p0 (x)y = 0.
Let {y1 , y2 , . . . , yn } be a set of linearly independent homogeneous solutions, L[yk ] = 0. We know that the general
solution of the homogeneous equation is a linear combination of the homogeneous solutions.
n
X
yh = ck yk (x)
k=1

Now consider the nth order linear inhomogeneous equation


L[y] y (n) + pn1 (x)y (n1) + + p1 (x)y 0 + p0 (x)y = f (x).

1059
Any function yp which satisfies this equation is called a particular solution of the differential equation. We want to
know the general solution of the inhomogeneous equation. Later in this chapter we will cover methods of constructing
this solution; now we consider the form of the solution.
Let yp be a particular solution. Note that yp + h is a particular solution if h satisfies the homogeneous equation.

L[yp + h] = L[yp ] + L[h] = f + 0 = f

Therefore yp + yh satisfies the homogeneous equation. We show that this is the general solution of the inhomogeneous
equation. Let yp and p both be solutions of the inhomogeneous equation L[y] = f . The difference of yp and p is a
homogeneous solution.
L[yp p ] = L[yp ] L[p ] = f f = 0
yp and p differ by a linear combination of the homogeneous solutions {yk }. Therefore the general solution of L[y] = f
is the sum of any particular solution yp and the general homogeneous solution yh .
n
X
yp + yh = yp (x) + ck yk (x)
k=1

Result 21.1.1 The general solution of the nth order linear inhomogeneous equation L[y] =
f (x) is
y = yp + c1 y1 + c2 y2 + + cn yn ,
where yp is a particular solution, {y1 , . . . , yn } is a set of linearly independent homogeneous
solutions, and the ck s are arbitrary constants.

Example 21.1.1 The differential equation


y 00 + y = sin(2x)
has the two homogeneous solutions
y1 = cos x, y2 = sin x,

1060
and a particular solution
1
yp = sin(2x).
3
We can add any combination of the homogeneous solutions to yp and it will still be a particular solution. For example,

1 1
p = sin(2x) sin x
3  3  
2 3x x
= sin cos
3 2 2

is a particular solution.

21.2 Method of Undetermined Coefficients


The first method we present for computing particular solutions is the method of undetermined coefficients. For some
simple differential equations, (primarily constant coefficient equations), and some simple inhomogeneities we are able
to guess the form of a particular solution. This form will contain some unknown parameters. We substitute this form
into the differential equation to determine the parameters and thus determine a particular solution.
Later in this chapter we will present general methods which work for any linear differential equation and any
inhogeneity. Thus one might wonder why I would present a method that works only for some simple problems. (And
why it is called a method if it amounts to no more than guessing.) The answer is that guessing an answer is less
grungy than computing it with the formulas we will develop later. Also, the process of this guessing is not random,
there is rhyme and reason to it.

Consider an nth order constant coefficient, inhomogeneous equation.

L[y] y (n) + an1 y (n1) + + a1 y 0 + a0 y = f (x)

If f (x) is one of a few simple forms, then we can guess the form of a particular solution. Below we enumerate some
cases.

1061
f = p(x). If f is an mth order polynomial, f (x) = pm xm + + p1 x + p0 , then guess
yp = cm xm + c1 x + c0 .

f = p(x) eax . If f is a polynomial times an exponential then guess


yp = (cm xm + c1 x + c0 ) eax .

f = p(x) eax cos (bx). If f is a cosine or sine times a polynomial and perhaps an exponential, f (x) = p(x) eax cos(bx)
or f (x) = p(x) eax sin(bx) then guess
yp = (cm xm + c1 x + c0 ) eax cos(bx) + (dm xm + d1 x + d0 ) eax sin(bx).
Likewise for hyperbolic sines and hyperbolic cosines.

Example 21.2.1 Consider


y 00 2y 0 + y = t2 .
The homogeneous solutions are y1 = et and y2 = t et . We guess a particular solution of the form
yp = at2 + bt + c.
We substitute the expression into the differential equation and equate coefficients of powers of t to determine the
parameters.
yp00 2yp0 + yp = t2
(2a) 2(2at + b) + (at2 + bt + c) = t2
(a 1)t2 + (b 4a)t + (2a 2b + c) = 0
a 1 = 0, b 4a = 0, 2a 2b + c = 0
a = 1, b = 4, c = 6
A particular solution is
yp = t2 + 4t + 6.

1062
If the inhomogeneity is a sum of terms, L[y] = f f1 + +fk , you can solve the problems L[y] = f1 , . . . , L[y] = fk
independently and then take the sum of the solutions as a particular solution of L[y] = f .

Example 21.2.2 Consider


L[y] y 00 2y 0 + y = t2 + e2t . (21.1)
The homogeneous solutions are y1 = et and y2 = t et . We already know a particular solution to L[y] = t2 . We seek a
particular solution to L[y] = e2t . We guess a particular solution of the form

yp = a e2t .

We substitute the expression into the differential equation to determine the parameter.

yp00 2yp0 + yp = e2t


4ae2t 4a e2t +a e2t = e2t
a=1

A particular solution of L[y] = e2t is yp = e2t . Thus a particular solution of Equation 21.1 is

yp = t2 + 4t + 6 + e2t .

The above guesses will not work if the inhomogeneity is a homogeneous solution. In this case, multiply the guess by
the lowest power of x such that the guess does not contain homogeneous solutions.

Example 21.2.3 Consider


L[y] y 00 2y 0 + y = et .
The homogeneous solutions are y1 = et and y2 = t et . Guessing a particular solution of the form yp = a et would not
work because L[et ] = 0. We guess a particular solution of the form

yp = at2 et

1063
We substitute the expression into the differential equation and equate coefficients of like terms to determine the
parameters.

yp00 2yp0 + yp = et
(at2 + 4at + 2a) et 2(at2 + 2at) et +at2 et = et
2a et = et
1
a=
2
A particular solution is
t2 t
yp = e .
2
Example 21.2.4 Consider
1 1
y 00 + y 0 + 2 y = x, x > 0.
x x
The homogeneous solutions are y1 = cos(ln x) and y2 = sin(ln x). We guess a particular solution of the form

yp = ax3

We substitute the expression into the differential equation and equate coefficients of like terms to determine the
parameter.
1 1
yp00 + yp0 + 2 yp = x
x x
6ax + 3ax + ax = x
1
a=
10
A particular solution is
x3
yp = .
10

1064
21.3 Variation of Parameters
In this section we present a method for computing a particular solution of an inhomogeneous equation given that we
know the homogeneous solutions. We will first consider second order equations and then generalize the result for nth
order equations.

21.3.1 Second Order Differential Equations


Consider the second order inhomogeneous equation,

L[y] y 00 + p(x)y 0 + q(x)y = f (x), on a < x < b.

We assume that the coefficient functions in the differential equation are continuous on [a . . . b]. Let y1 (x) and y2 (x)
be two linearly independent solutions to the homogeneous equation. Since the Wronskian,
 Z 
W (x) = exp p(x) dx ,

is non-vanishing, we know that these solutions exist. We seek a particular solution of the form,

yp = u1 (x)y1 + u2 (x)y2 .

We compute the derivatives of yp .

yp0 = u01 y1 + u1 y10 + u02 y2 + u2 y20


yp00 = u001 y1 + 2u01 y10 + u1 y100 + u002 y2 + 2u02 y20 + u2 y200

We substitute the expression for yp and its derivatives into the inhomogeneous equation and use the fact that y1 and
y2 are homogeneous solutions to simplify the equation.

u001 y1 + 2u01 y10 + u1 y100 + u002 y2 + 2u02 y20 + u2 y200 + p(u01 y1 + u1 y10 + u02 y2 + u2 y20 ) + q(u1 y1 + u2 y2 ) = f
u001 y1 + 2u01 y10 + u002 y2 + 2u02 y20 + p(u01 y1 + u02 y2 ) = f

1065
This is an ugly equation for u1 and u2 , however, we have an ace up our sleeve. Since u1 and u2 are undetermined
functions of x, we are free to impose a constraint. We choose this constraint to simplify the algebra.

u01 y1 + u02 y2 = 0

This constraint simplifies the derivatives of yp ,

yp0 = u01 y1 + u1 y10 + u02 y2 + u2 y20


= u1 y10 + u2 y20
yp00 = u01 y10 + u1 y100 + u02 y20 + u2 y200 .

We substitute the new expressions for yp and its derivatives into the inhomogeneous differential equation to obtain a
much simpler equation than before.

u01 y10 + u1 y100 + u02 y20 + u2 y200 + p(u1 y10 + u2 y20 ) + q(u1 y1 + u2 y2 ) = f (x)
u01 y10 + u02 y20 + u1 L[y1 ] + u2 L[y2 ] = f (x)
u01 y10 + u02 y20 = f (x).

With the constraint, we have a system of linear equations for u01 and u02 .

u01 y1 + u02 y2 = 0
u01 y10 + u02 y20 = f (x).
  0  
y1 y2 u1 0
=
y10 y20 u02 f
We solve this system using Kramers rule. (See Appendix O.)

f (x)y2 f (x)y1
u01 = u02 =
W (x) W (x)

1066
Here W (x) is the Wronskian.

y 1 y2
W (x) = 0

y1 y20

We integrate to get u1 and u2 . This gives us a particular solution.


Z Z
f (x)y2 (x) f (x)y1 (x)
yp = y1 dx + y2 dx.
W (x) W (x)

Result 21.3.1 Let y1 and y2 be linearly independent homogeneous solutions of

L[y] = y 00 + p(x)y 0 + q(x)y = f (x).

A particular solution is
Z Z
f (x)y2 (x) f (x)y1 (x)
yp = y1 (x) dx + y2 (x) dx,
W (x) W (x)
where W (x) is the Wronskian of y1 and y2 .

Example 21.3.1 Consider the equation,


y 00 + y = cos(2x).

The homogeneous solutions are y1 = cos x and y2 = sin x. We compute the Wronskian.

cos x sin x
W (x) = = cos2 x + sin2 x = 1
sin x cos x

1067
We use variation of parameters to find a particular solution.
Z Z
yp = cos(x) cos(2x) sin(x) dx + sin(x) cos(2x) cos(x) dx
Z Z
1  1 
= cos(x) sin(3x) sin(x) dx + sin(x) cos(3x) + cos(x) dx
2 2
   
1 1 1 1
= cos(x) cos(3x) + cos(x) + sin(x) sin(3x) + sin(x)
2 3 2 3
1  1
sin2 (x) cos2 (x) +

= cos(3x) cos(x) + sin(3x) sin(x)
2 6
1 1
= cos(2x) + cos(2x)
2 6
1
= cos(2x)
3
The general solution of the inhomogeneous equation is

1
y = cos(2x) + c1 cos(x) + c2 sin(x).
3

21.3.2 Higher Order Differential Equations


Consider the nth order inhomogeneous equation,

L[y] = y(n) + pn1 (x)y (n1) + + p1 (x)y 0 + p0 (x)y = f (x), on a < x < b.

We assume that the coefficient functions in the differential equation are continuous on [a . . . b]. Let {y1 , . . . , yn } be a
set of linearly independent solutions to the homogeneous equation. Since the Wronskian,
 Z 
W (x) = exp pn1 (x) dx ,

1068
is non-vanishing, we know that these solutions exist. We seek a particular solution of the form

y p = u1 y 1 + u 2 y 2 + + un y n .

Since {u1 , . . . , un } are undetermined functions of x, we are free to impose n1 constraints. We choose these constraints
to simplify the algebra.

u01 y1 +u02 y2 + +u0n yn =0


u01 y10 +u02 y20 + +u0n yn0 =0
.. . . .
. + .. + .. + .. =0
(n2) (n2)
u01 y1 +u02 y2 + +u0n yn(n2) =0

We differentiate the expression for yp , utilizing our constraints.

yp =u1 y1 +u2 y2 + +un yn


yp0 =u1 y10 +u2 y20 + +un yn0
yp00 =u1 y100 +u2 y200 + +un yn00
.. . . . .
. = .. + .. + .. + ..
(n) (n) (n1) (n1)
yp(n) =u1 y1 +u2 y2 + +un yn(n) + u01 y1 + u02 y2 + + u0n yn(n1)

We substitute yp and its derivatives into the inhomogeneous differential equation and use the fact that the yk are
homogeneous solutions.
(n) (n1) (n1)
u1 y1 + + un yn(n) + u01 y1 + + u0n yn(n1) + pn1 (u1 y1 + + un yn(n1) ) + + p0 (u1 y1 + un yn ) = f
(n1) (n1)
u1 L[y1 ] + u2 L[y2 ] + + un L[yn ] + u01 y1 + u02 y2 + + u0n yn(n1) = f
(n1) (n1)
u01 y1 + u02 y2 + + u0n yn(n1) = f.

1069
With the constraints, we have a system of linear equations for {u1 , . . . , un }.

y1 y2 yn u01 0
y0 y20 0 0
y n u2 .
1 ..
.. .. .. .. .. = .
. . . . . 0
(n1) (n1) (n1)
y1 y2 yn u0n f
We solve this system using Kramers rule. (See Appendix O.)
W [y1 , . . . , yk1 , yk+1 , . . . , yn ]
u0k = (1)n+k+1 f, for k = 1, . . . , n,
W [y1 , y2 , . . . , yn ]
Here W is the Wronskian.
We integrating to obtain the uk s.
Z
n+k+1 W [y1 , . . . , yk1 , yk+1 , . . . , yn ](x)
uk = (1) f (x) dx, for k = 1, . . . , n
W [y1 , y2 , . . . , yn ](x)

Result 21.3.2 Let {y1 , . . . , yn } be linearly independent homogeneous solutions of

L[y] = y(n) + pn1 (x)y (n1) + + p1 (x)y 0 + p0 (x)y = f (x), on a < x < b.

A particular solution is
y p = u 1 y 1 + u2 y 2 + + u n y n .
where
Z
W [y1 , . . . , yk1 , yk+1 , . . . , yn ](x)
uk = (1)n+k+1 f (x) dx, for k = 1, . . . , n,
W [y1 , y2 , . . . , yn ](x)
and W [y1 , y2 , . . . , yn ](x) is the Wronskian of {y1 (x), . . . , yn (x)}.

1070
21.4 Piecewise Continuous Coefficients and Inhomogeneities
Example 21.4.1 Consider the problem

y 00 y = e|x| , y() = 0, > 0, 6= 1.

The homogeneous solutions of the differential equation are ex and ex . We use variation of parameters to find a
particular solution for x > 0.
Z x Z x
x e e x e e
yp = e d + e d
2 2
1 x x (+1) 1 x x (1)
Z Z
= e e d e e d
2 2
1 1
= ex + ex
2( + 1) 2( 1)
e x
= 2 , for x > 0
1
A particular solution for x < 0 is
ex
yp = , for x < 0.
2 1
Thus a particular solution is
e|x|
yp = .
2 1
The general solution is
1
y= e|x| +c1 ex +c2 ex .
1 2
Applying the boundary conditions, we see that c1 = c2 = 0. Apparently the solution is

e|x|
y= .
2 1

1071
-4 -2 2 4
0.3
-0.05
0.25
-0.1
0.2
-0.15
0.15
-0.2
0.1
-0.25
0.05
-0.3

-4 -2 2 4

Figure 21.1: The Incorrect and Correct Solution to the Differential Equation.

This function is plotted in Figure 21.1. This function satisfies the differential equation for positive and negative x. It
also satisfies the boundary conditions. However, this is NOT a solution to the differential equation. Since the differential
equation has no singular points and the inhomogeneous term is continuous, the solution must be twice continuously
differentiable. Since the derivative of e|x| /(2 1) has a jump discontinuity at x = 0, the second derivative does not
exist. Thus this function could not possibly be a solution to the differential equation. In the next example we examine
the right way to solve this problem.

Example 21.4.2 Again consider


y 00 y = e|x| , y() = 0, > 0, 6= 1.

1072
Separating this into two problems for positive and negative x,
00
y y = ex , y () = 0, on < x 0,
00 x
y+ y+ = e , y+ () = 0, on 0 x < .
In order for the solution over the whole domain to be twice differentiable, the solution and its first derivative must be
continuous. Thus we impose the additional boundary conditions
0 0
y (0) = y+ (0), y (0) = y+ (0).
The solutions that satisfy the two differential equations and the boundary conditions at infinity are
ex ex
y = + c ex , y+ = + c+ ex .
2 1 2 1
The two additional boundary conditions give us the equations
y (0) = y+ (0) c = c+
0 0
y (0) = y+ (0) 2
+ c = 2 c+ .
1 1
We solve these two equations to determine c and c+ .

c = c+ =
2 1
Thus the solution over the whole domain is
(
ex ex
2 1
for x < 0,
y= ex ex
2 1
for x > 0

e|x| e|x|
y= .
2 1
This function is plotted in Figure 21.1. You can verify that this solution is twice continuously differentiable.

1073
21.5 Inhomogeneous Boundary Conditions
21.5.1 Eliminating Inhomogeneous Boundary Conditions
Consider the nth order equation
L[y] = f (x), for a < x < b,
subject to the linear inhomogeneous boundary conditions

Bj [y] = j , for j = 1, . . . , n,

where the boundary conditions are of the form

B[y] 0 y(a) + 1 y 0 (a) + + yn1 y (n1) (a) + 0 y(b) + 1 y 0 (b) + + n1 y (n1)

Let g(x) be an n-times continuously differentiable function that satisfies the boundary conditions. Substituting y = u+g
into the differential equation and boundary conditions yields

L[u] = f (x) L[g], Bj [u] = bj Bj [g] = 0 for j = 1, . . . , n.

Note that the problem for u has homogeneous boundary conditions. Thus a problem with inhomogeneous boundary
conditions can be reduced to one with homogeneous boundary conditions. This technique is of limited usefulness for
ordinary differential equations but is important for solving some partial differential equation problems.

Example 21.5.1 Consider the problem

y 00 + y = cos 2x, y(0) = 1, y() = 2.


x
g(x) =
+ 1 satisfies the boundary conditions. Substituting y = u + g yields
x
u00 + u = cos 2x 1, y(0) = y() = 0.

1074
Example 21.5.2 Consider
y 00 + y = cos 2x, y 0 (0) = y() = 1.
g(x) = sin x cos x satisfies the inhomogeneous boundary conditions. Substituting y = u + sin x cos x yields

u00 + u = cos 2x, u0 (0) = u() = 0.

Note that since g(x) satisfies the homogeneous equation, the inhomogeneous term in the equation for u is the same as
that in the equation for y.

Example 21.5.3 Consider


2 4
y 00 + y = cos 2x, y(0) = , y() = .
3 3
1 1
g(x) = cos x 3
satisfies the boundary conditions. Substituting y = u + cos x 3
yields

1
u00 + u = cos 2x + , u(0) = u() = 0.
3

Result 21.5.1 The nth order differential equation with boundary conditions

L[y] = f (x), Bj [y] = bj , for j = 1, . . . , n

has the solution y = u + g where u satisfies

L[u] = f (x) L[g], Bj [u] = 0, for j = 1, . . . , n

and g is any n-times continuously differentiable function that satisfies the inhomogeneous
boundary conditions.

1075
21.5.2 Separating Inhomogeneous Equations and Inhomogeneous Boundary Con-
ditions
Now consider a problem with inhomogeneous boundary conditions

L[y] = f (x), B1 [y] = 1 , B2 [y] = 2 .

In order to solve this problem, we solve the two problems

L[u] = f (x), B1 [u] = B2 [u] = 0, and

L[v] = 0, B1 [v] = 1 , B2 [v] = 2 .


The solution for the problem with an inhomogeneous equation and inhomogeneous boundary conditions will be the sum
of u and v. To verify this,

L[u + v] = L[u] + L[v] = f (x) + 0 = f (x),


Bi [u + v] = Bi [u] + Bi [v] = 0 + i = i .

This will be a useful technique when we develop Green functions.

Result 21.5.2 The solution to

L[y] = f (x), B1 [y] = 1 , B2 [y] = 2 ,

is y = u + v where

L[u] = f (x), B1 [u] = 0, B2 [u] = 0, and


L[v] = 0, B1 [v] = 1 , B2 [v] = 2 .

1076
21.5.3 Existence of Solutions of Problems with Inhomogeneous Boundary Con-
ditions
Consider the nth order homogeneous differential equation

L[y] = y (n) + pn1 y (n1) + + p1 y 0 + p0 y = f (x), for a < x < b,

subject to the n inhomogeneous boundary conditions

Bj [y] = j , for j = 1, . . . , n

where each boundary condition is of the form

B[y] 0 y(a) + 1 y 0 (a) + + n1 y (n1) (a) + 0 y(b) + 1 y 0 (b) + + n1 y (n1) (b).

We assume that the coefficients in the differential equation are continuous on [a, b]. Since the Wronskian of the solutions
of the differential equation,  Z 
W (x) = exp pn1 (x) dx ,

is non-vanishing on [a, b], there are n linearly independent solution on that range. Let {y1 , . . . , yn } be a set of linearly
independent solutions of the homogeneous equation. From Result 21.3.2 we know that a particular solution yp exists.
The general solution of the differential equation is

y = yp + c1 y1 + c2 y2 + + cn yn .

The n boundary conditions impose the matrix equation,



B1 [y1 ] B1 [y2 ] B1 [yn ] c1 1 B1 [yp ]
B2 [y1 ] B2 [y2 ] B2 [yn ] c2 2 B2 [yp ]
.. =

.. .. ... .. ..
. . . . .
Bn [y1 ] Bn [y2 ] Bn [yn ] cn n Bn [yp ]

1077
This equation has a unique solution if and only if the equation


B1 [y1 ] B1 [y2 ] B1 [yn ] c1 0
B2 [y1 ] B2 [y2 ] B2 [yn ] c2 0
.. = ..

.. .. .. ..
. . . . . .
Bn [y1 ] Bn [y2 ] Bn [yn ] cn 0

has only the trivial solution. (This is the case if and only if the determinant of the matrix is nonzero.) Thus the problem

L[y] = y (n) + pn1 y (n1) + + p1 y 0 + p0 y = f (x), for a < x < b,

subject to the n inhomogeneous boundary conditions

Bj [y] = j , for j = 1, . . . , n,

has a unique solution if and only if the problem

L[y] = y (n) + pn1 y (n1) + + p1 y 0 + p0 y = 0, for a < x < b,

subject to the n homogeneous boundary conditions

Bj [y] = 0, for j = 1, . . . , n,

has only the trivial solution.

1078
Result 21.5.3 The problem

L[y] = y (n) + pn1 y (n1) + + p1 y 0 + p0 y = f (x), for a < x < b,

subject to the n inhomogeneous boundary conditions

Bj [y] = j , for j = 1, . . . , n,

has a unique solution if and only if the problem

L[y] = y (n) + pn1 y (n1) + + p1 y 0 + p0 y = 0, for a < x < b,

subject to
Bj [y] = 0, for j = 1, . . . , n,
has only the trivial solution.

21.6 Green Functions for First Order Equations


Consider the first order inhomogeneous equation

L[y] y 0 + p(x)y = f (x), for x > a, (21.2)

subject to a homogeneous initial condition, B[y] y(a) = 0.


The Green function G(x|) is defined as the solution to

L[G(x|)] = (x ) subject to G(a|) = 0.

We can represent the solution to the inhomogeneous problem in Equation 21.2 as an integral involving the Green

1079
function. To show that Z
y(x) = G(x|)f () d
a

is the solution, we apply the linear operator L to the integral. (Assume that the integral is uniformly convergent.)
Z  Z
L G(x|)f () d = L[G(x|)]f () d
a a
Z
= (x )f () d
a
= f (x)

The integral also satisfies the initial condition.


Z  Z
B G(x|)f () d = B[G(x|)]f () d
a a
Z
= (0)f () d
a
=0

Now we consider the qualitiative behavior of the Green function. For x 6= , the Green function is simply a
homogeneous solution of the differential equation, however at x = we expect some singular behavior. G0 (x|) will
have a Dirac delta function type singularity. This means that G(x|) will have a jump discontinuity at x = . We
integrate the differential equation on the vanishing interval ( . . . + ) to determine this jump.

G0 + p(x)G = (x )
Z +
+
G( |) G( |) + p(x)G(x|) dx = 1

G( + |) G( |) = 1 (21.3)

1080
The homogeneous solution of the differential equation is
R
y h = e p(x) dx

Since the Green function satisfies the homogeneous equation for x 6= , it will be a constant times this homogeneous
solution for x < and x > .
( R
c1 e p(x) dx a<x<
G(x|) =
R
p(x) dx
c2 e <x

In order to satisfy the homogeneous initial condition G(a|) = 0, the Green function must vanish on the interval
(a . . . ).
(
0 a<x<
G(x|) = R
c e p(x) dx <x

The jump condition, (Equation 21.3), gives us the constraint G( + |) = 1. This determines the constant in the
homogeneous solution for x > .
(
0 R a<x<
G(x|) = x p(t) dt
e <x

We can use the Heaviside function to write the Green function without using a case statement.
Rx
G(x|) = e p(t) dt
H(x )

Clearly the Green function is of little value in solving the inhomogeneous differential equation in Equation 21.2, as
we can solve that problem directly. However, we will encounter first order Green function problems in solving some
partial differential equations.

1081
Result 21.6.1 The first order inhomogeneous differential equation with homogeneous initial
condition
L[y] y 0 + p(x)y = f (x), for a < x, y(a) = 0,
has the solution Z
y= G(x|)f () d,
a
where G(x|) satisfies the equation

L[G(x|)] = (x ), for a < x, G(a|) = 0.

The Green function is Rx


p(t) dt
G(x|) = e H(x )

21.7 Green Functions for Second Order Equations


Consider the second order inhomogeneous equation

L[y] = y 00 + p(x)y 0 + q(x)y = f (x), for a < x < b, (21.4)

subject to the homogeneous boundary conditions

B1 [y] = B2 [y] = 0.

The Green function G(x|) is defined as the solution to

L[G(x|)] = (x ) subject to B1 [G] = B2 [G] = 0.

1082
The Green function is useful because you can represent the solution to the inhomogeneous problem in Equation 21.4
as an integral involving the Green function. To show that
Z b
y(x) = G(x|)f () d
a

is the solution, we apply the linear operator L to the integral. (Assume that the integral is uniformly convergent.)
Z b  Z b
L G(x|)f () d = L[G(x|)]f () d
a a
Z b
= (x )f () d
a
= f (x)

The integral also satisfies the boundary conditions.


Z b  Z b
Bi G(x|)f () d = Bi [G(x|)]f () d
a a
Z b
= [0]f () d
a
=0

One of the advantages of using Green functions is that once you find the Green function for a linear operator and
certain homogeneous boundary conditions,

L[G] = (x ), B1 [G] = B2 [G] = 0,

you can write the solution for any inhomogeneity, f (x).

L[f ] = f (x), B1 [y] = B2 [y] = 0

1083
You do not need to do any extra work to obtain the solution for a different inhomogeneous term.
Qualitatively, what kind of behavior will the Green function for a second order differential equation have? Will it
have a delta function singularity; will it be continuous? To answer these questions we will first look at the behavior of
integrals and derivatives of (x).
The integral of (x) is the Heaviside function, H(x).
Z x (
0 for x < 0
H(x) = (t) dt =
1 for x > 0
The integral of the Heaviside function is the ramp function, r(x).
Z x (
0 for x < 0
r(x) = H(t) dt =
x for x > 0
The derivative of the delta function is zero for x 6= 0. At x = 0 it goes from 0 up to +, down to and then back
up to 0.
In Figure 21.2 we see conceptually the behavior of the ramp function, the Heaviside function, the delta function,
and the derivative of the delta function.
We write the differential equation for the Green function.
G00 (x|) + p(x)G0 (x|) + q(x)G(x|) = (x )
we see that only the G00 (x|) term can have a delta function type singularity. If one of the other terms had a delta
function type singularity then G00 (x|) would be more singular than a delta function and there would be nothing in the
right hand side of the equation to match this kind of singularity. Analogous to the progression from a delta function
to a Heaviside function to a ramp function, we see that G0 (x|) will have a jump discontinuity and G(x|) will be
continuous.
Let y1 and y2 be two linearly independent solutions to the homogeneous equation, L[y] = 0. Since the Green
function satisfies the homogeneous equation for x 6= , it will be a linear combination of the homogeneous solutions.
(
c1 y1 + c2 y2 for x <
G(x|) =
d1 y1 + d2 y2 for x >

1084
d
Figure 21.2: r(x), H(x), (x) and dx
(x)

We require that G(x|) be continuous.


G(x|) x = G(x|) x+

We can write this in terms of the homogeneous solutions.

c1 y1 () + c2 y2 () = d1 y1 () + d2 y2 ()

We integrate L[G(x|)] = (x ) from to +.

Z + Z +
00 0
[G (x|) + p(x)G (x|) + q(x)G(x|)] dx = (x ) dx.

1085
Since G(x|) is continuous and G0 (x|) has only a jump discontinuity two of the terms vanish.
Z + Z +
0
p(x)G (x|) dx = 0 and q(x)G(x|) dx = 0

Z + Z +
G00 (x|) dx = (x ) dx

 0 +  +
G (x|) = H(x )
G0 ( + |) G0 ( |) = 1

We write this jump condition in terms of the homogeneous solutions.

d1 y10 () + d2 y20 () c1 y10 () c2 y20 () = 1


Combined with the two boundary conditions, this gives us a total of four equations to determine our four constants,
c1 , c2 , d1 , and d2 .
Result 21.7.1 The second order inhomogeneous differential equation with homogeneous
boundary conditions

L[y] = y 00 + p(x)y 0 + q(x)y = f (x), for a < x < b, B1 [y] = B2 [y] = 0,

has the solution Z b


y= G(x|)f () d,
a
where G(x|) satisfies the equation

L[G(x|)] = (x ), for a < x < b, B1 [G(x|)] = B2 [G(x|)] = 0.

G(x|) is continuous and G0 (x|) has a jump discontinuity of height 1 at x = .

1086
Example 21.7.1 Solve the boundary value problem

y 00 = f (x), y(0) = y(1) = 0,

using a Green function.


A pair of solutions to the homogeneous equation are y1 = 1 and y2 = x. First note that only the trivial solution
to the homogeneous equation satisfies the homogeneous boundary conditions. Thus there is a unique solution to this
problem.
The Green function satisfies

G00 (x|) = (x ), G(0|) = G(1|) = 0.

The Green function has the form (


c1 + c2 x for x <
G(x|) =
d1 + d2 x for x > .
Applying the two boundary conditions, we see that c1 = 0 and d1 = d2 . The Green function now has the form
(
cx for x <
G(x|) =
d(x 1) for x > .

Since the Green function must be continuous,



c = d( 1) d=c .
1
From the jump condition,
d d
c (x 1) cx =1

dx 1 x= dx x=

c c=1
1
c = 1.

1087
0.1 0.1 0.1 0.1
0.5 1 0.5 1 0.5 1 0.5 1
-0.1 -0.1 -0.1 -0.1
-0.2 -0.2 -0.2 -0.2
-0.3 -0.3 -0.3 -0.3

Figure 21.3: Plot of G(x|0.05),G(x|0.25),G(x|0.5) and G(x|0.75).

Thus the Green function is


(
( 1)x for x <
G(x|) =
(x 1) for x > .

The Green function is plotted in Figure 21.3 for various values of . The solution to y 00 = f (x) is
Z 1
y(x) = G(x|)f () d
0

Z x Z 1
y(x) = (x 1) f () d + x ( 1)f () d.
0 x

Example 21.7.2 Solve the boundary value problem

y 00 = f (x), y(0) = 1, y(1) = 2.

In Example 21.7.1 we saw that the solution to

u00 = f (x), u(0) = u(1) = 0

1088
is Z x Z 1
u(x) = (x 1) f () d + x ( 1)f () d.
0 x

Now we have to find the solution to

v 00 = 0, v(0) = 1, u(1) = 2.

The general solution is


v = c1 + c2 x.
Applying the boundary conditions yields
v = 1 + x.
Thus the solution for y is
Z x Z 1
y = 1 + x + (x 1) f () d + x ( 1)f ( xi) d.
0 x

Example 21.7.3 Consider


y 00 = x, y(0) = y(1) = 0.

Method 1. Integrating the differential equation twice yields

1
y = x3 + c1 x + c2 .
6
Applying the boundary conditions, we find that the solution is

1
y = (x3 x).
6

1089
Method 2. Using the Green function to find the solution,
Z x Z 1
2
y = (x 1) d + x ( 1) d
0 x
 
1 3 1 1 1 3 1 2
= (x 1) x + x x + x
3 3 2 3 2

1
y = (x3 x).
6

Example 21.7.4 Find the solution to the differential equation

y 00 y = sin x,

that is bounded for all x.


The Green function for this problem satisfies

G00 (x|) G(x|) = (x ).

The homogeneous solutions are y1 = ex , and y2 = ex . The Green function has the form
(
c1 ex +c2 ex for x <
G(x|) = x x
d1 e +d2 e for x > .

Since the solution must be bounded for all x, the Green function must also be bounded. Thus c2 = d1 = 0. The Green
function now has the form (
c ex for x <
G(x|) = x
de for x > .
Requiring that G(x|) be continuous gives us the condition

c e = d e d = c e2 .

1090
G(x|) has a jump discontinuity of height 1 at x = .

d 2 x d x
ce e ce =1
dx x= dx x=

c e2 e c e = 1
1
c = e
2

The Green function is then


(
12 ex for x <
G(x|) =
12 ex+ for x >

1
G(x|) = e|x| .
2

A plot of G(x|0) is given in Figure 21.4. The solution to y 00 y = sin x is


Z
1
y(x) = e|x| sin d
2
Z x Z 
1 x x+
= sin e d + sin e d
2 x
1 sin x + cos x sin x + cos x
= ( + )
2 2 2

1
y= sin x.
2

1091
0.6

0.4

0.2

-4 -2 2 4
-0.2

-0.4

-0.6

Figure 21.4: Plot of G(x|0).

21.7.1 Green Functions for Sturm-Liouville Problems


Consider the problem
0
L[y] = (p(x)y 0 ) + q(x)y = f (x), subject to
B1 [y] = 1 y(a) + 2 y 0 (a) = 0, B2 [y] = 1 y(b) + 2 y 0 (b) = 0.

This is known as a Sturm-Liouville problem. Equations of this type often occur when solving partial differential equations.
The Green function associated with this problem satisfies

L[G(x|)] = (x ), B1 [G(x|)] = B2 [G(x|)] = 0.

1092
Let y1 and y2 be two non-trivial homogeneous solutions that satisfy the left and right boundary conditions, respectively.

L[y1 ] = 0, B1 [y1 ] = 0, L[y2 ] = 0, B2 [y2 ] = 0.

The Green function satisfies the homogeneous equation for x 6= and satisfies the homogeneous boundary conditions.
Thus it must have the following form.
(
c1 ()y1 (x) for a x ,
G(x|) =
c2 ()y2 (x) for x b,

Here c1 and c2 are unknown functions of .


The first constraint on c1 and c2 comes from the continuity condition.

G( |) = G( + |)
c1 ()y1 () = c2 ()y2 ()

We write the inhomogeneous equation in the standard form.


p0 0 q (x )
G00 (x|) + G (x|) + G(x|) =
p p p
The second constraint on c1 and c2 comes from the jump condition.
1
G0 ( + |) G0 ( |) =
p()
1
c2 ()y20 () c1 ()y10 () =
p()
Now we have a system of equations to determine c1 and c2 .

c1 ()y1 () c2 ()y2 () = 0
1
c1 ()y10 () c2 ()y20 () =
p()

1093
We solve this system with Kramers rule.

y2 () y1 ()
c1 () = , c2 () =
p()(W ()) p()(W ())

Here W (x) is the Wronskian of y1 (x) and y2 (x). The Green function is
(y
1 (x)y2 ()
p()W ()
for a x ,
G(x|) = y2 (x)y1 ()
p()W ()
for x b.

The solution of the Sturm-Liouville problem is


Z b
y= G(x|)f () d.
a

Result 21.7.2 The problem


0
L[y] = (p(x)y 0 ) + q(x)y = f (x), subject to
B1 [y] = 1 y(a) + 2 y 0 (a) = 0, B2 [y] = 1 y(b) + 2 y 0 (b) = 0.

has the Green function


(y
1 (x)y2 ()
p()W () for a x ,
G(x|) = y2 (x)y1 ()
p()W () for x b,

where y1 and y2 are non-trivial homogeneous solutions that satisfy B1 [y1 ] = B2 [y2 ] = 0, and
W (x) is the Wronskian of y1 and y2 .

1094
Example 21.7.5 Consider the equation

y 00 y = f (x), y(0) = y(1) = 0.

A set of solutions to the homogeneous equation is {ex , ex }. Equivalently, one could use the set {cosh x, sinh x}. Note
that sinh x satisfies the left boundary condition and sinh(x 1) satisfies the right boundary condition. The Wronskian
of these two homogeneous solutions is

sinh x sinh(x 1)
W (x) =
cosh x cosh(x 1)
= sinh x cosh(x 1) cosh x sinh(x 1)
1 1
= [sinh(2x 1) + sinh(1)] [sinh(2x 1) sinh(1)]
2 2
= sinh(1).

The Green function for the problem is then


( sinh x sinh(1)
sinh(1)
for 0 x
G(x|) = sinh(x1) sinh
sinh(1)
for x 1.

The solution to the problem is

x 1
sinh(x 1)
Z Z
sinh(x)
y= sinh()f () d + sinh( 1)f () d.
sinh(1) 0 sinh(1) x

21.7.2 Initial Value Problems


Consider
L[y] = y 00 + p(x)y 0 + q(x)y = f (x), for a < x < b,

1095
subject the the initial conditions
y(a) = 1 , y 0 (a) = 2 .
The solution is y = u + v where
u00 + p(x)u0 + q(x)u = f (x), u(a) = 0, u0 (a) = 0,
and
v 00 + p(x)v 0 + q(x)v = 0, v(a) = 1 , v 0 (a) = 2 .
Since the Wronskian  Z 
W (x) = c exp p(x) dx

is non-vanishing, the solutions of the differential equation for v are linearly independent. Thus there is a unique solution
for v that satisfies the initial conditions.

The Green function for u satisfies


G00 (x|) + p(x)G0 (x|) + q(x)G(x|) = (x ), G(a|) = 0, G0 (a|) = 0.
The continuity and jump conditions are
G( |) = G( + |), G0 ( |) + 1 = G0 ( + |).
Let u1 and u2 be two linearly independent solutions of the differential equation. For x < , G(x|) is a linear combination
of these solutions. Since the Wronskian is non-vanishing, only the trivial solution satisfies the homogeneous initial
conditions. The Green function must be
(
0 for x <
G(x|) =
u (x) for x > ,

where u (x) is the linear combination of u1 and u2 that satisfies


u () = 0, u0 () = 1.

1096
Note that the non-vanishing Wronskian ensures a unique solution for u . We can write the Green function in the form

G(x|) = H(x )u (x).

This is known as the causal solution. The solution for u is


Z b
u= G(x|)f () d
a
Z b
= H(x )u (x)f () d
Za x
= u (x)f () d
a

Now we have the solution for y, Z x


y=v+ u (x)f () d.
a

Result 21.7.3 The solution of the problem

y 00 + p(x)y 0 + q(x)y = f (x), y(a) = 1 , y 0 (a) = 2 ,

is Z x
y = yh + y (x)f () d
a
where yh is the combination of the homogeneous solutions of the equation that satisfy the
initial conditions and y (x) is the linear combination of homogeneous solutions that satisfy
y () = 0, y0 () = 1.

1097
21.7.3 Problems with Unmixed Boundary Conditions
Consider
L[y] = y 00 + p(x)y 0 + q(x)y = f (x), for a < x < b,
subject the the unmixed boundary conditions

1 y(a) + 2 y 0 (a) = 1 , 1 y(b) + 2 y 0 (b) = 2 .

The solution is y = u + v where

u00 + p(x)u0 + q(x)u = f (x), 1 u(a) + 2 u0 (a) = 0, 1 u(b) + 2 u0 (b) = 0,

and
v 00 + p(x)v 0 + q(x)v = 0, 1 v(a) + 2 v 0 (a) = 1 , 1 v(b) + 2 v 0 (b) = 2 .
The problem for v may have no solution, a unique solution or an infinite number of solutions. We consider only the
case that there is a unique solution for v. In this case the homogeneous equation subject to homogeneous boundary
conditions has only the trivial solution.

The Green function for u satisfies

G00 (x|) + p(x)G0 (x|) + q(x)G(x|) = (x ),

1 G(a|) + 2 G0 (a|) = 0, 1 G(b|) + 2 G0 (b|) = 0.


The continuity and jump conditions are

G( |) = G( + |), G0 ( |) + 1 = G0 ( + |).

Let u1 and u2 be two solutions of the homogeneous equation that satisfy the left and right boundary conditions,
respectively. The non-vanishing of the Wronskian ensures that these solutions exist. Let W (x) denote the Wronskian

1098
of u1 and u2 . Since the homogeneous equation with homogeneous boundary conditions has only the trivial solution,
W (x) is nonzero on [a, b]. The Green function has the form
(
c 1 u1 for x < ,
G(x|) =
c 2 u2 for x > .

The continuity and jump conditions for Green function gives us the equations

c1 u1 () c2 u2 () = 0
c1 u01 () c2 u02 () = 1.

Using Kramers rule, the solution is


u2 () u1 ()
c1 = , c2 = .
W () W ()

Thus the Green function is


(u
1 (x)u2 ()
W ()
for x < ,
G(x|) = u1 ()u2 (x)
W ()
for x > .

The solution for u is


Z b
u= G(x|)f () d.
a

Thus if there is a unique solution for v, the solution for y is

Z b
y=v+ G(x|)f () d.
a

1099
Result 21.7.4 Consider the problem

y 00 + p(x)y 0 + q(x)y = f (x),

1 y(a) + 2 y 0 (a) = 1 , 1 y(b) + 2 y 0 (b) = 2 .


If the homogeneous differential equation subject to the inhomogeneous boundary conditions
has the unique solution yh , then the problem has the unique solution
Z b
y = yh + G(x|)f () d
a

where (u
1 (x)u2 ()
W () for x < ,
G(x|) = u1 ()u2 (x)
W () for x > ,
u1 and u2 are solutions of the homogeneous differential equation that satisfy the left and right
boundary conditions, respectively, and W (x) is the Wronskian of u1 and u2 .

21.7.4 Problems with Mixed Boundary Conditions


Consider
L[y] = y 00 + p(x)y 0 + q(x)y = f (x), for a < x < b,
subject the the mixed boundary conditions

B1 [y] = 11 y(a) + 12 y 0 (a) + 11 y(b) + 12 y 0 (b) = 1 ,

B2 [y] = 21 y(a) + 22 y 0 (a) + 21 y(b) + 22 y 0 (b) = 2 .

1100
The solution is y = u + v where

u00 + p(x)u0 + q(x)u = f (x), B1 [u] = 0, B2 [u] = 0,

and
v 00 + p(x)v 0 + q(x)v = 0, B1 [v] = 1 , B2 [v] = 2 .

The problem for v may have no solution, a unique solution or an infinite number of solutions. Again we consider
only the case that there is a unique solution for v. In this case the homogeneous equation subject to homogeneous
boundary conditions has only the trivial solution.
Let y1 and y2 be two solutions of the homogeneous equation that satisfy the boundary conditions B1 [y1 ] = 0 and
B2 [y2 ] = 0. Since the completely homogeneous problem has no solutions, we know that B1 [y2 ] and B2 [y1 ] are nonzero.
The solution for v has the form
v = c1 y1 + c2 y2 .
Applying the two boundary conditions yields
2 1
v= y1 + y2 .
B2 [y1 ] B1 [y2 ]

The Green function for u satisfies

G00 (x|) + p(x)G0 (x|) + q(x)G(x|) = (x ), B1 [G] = 0, B2 [G] = 0.

The continuity and jump conditions are

G( |) = G( + |), G0 ( |) + 1 = G0 ( + |).

We write the Green function as the sum of the causal solution and the two homogeneous solutions

G(x|) = H(x )y (x) + c1 y1 (x) + c2 y2 (x)

1101
With this form, the continuity and jump conditions are automatically satisfied. Applying the boundary conditions yields

B1 [G] = B1 [H(x )y ] + c2 B1 [y2 ] = 0,


B2 [G] = B2 [H(x )y ] + c1 B2 [y1 ] = 0,

B1 [G] = 11 y (b) + 12 y0 (b) + c2 B1 [y2 ] = 0,


B2 [G] = 21 y (b) + 22 y0 (b) + c1 B2 [y1 ] = 0,

21 y (b) + 22 y0 (b) 11 y (b) + 12 y0 (b)


G(x|) = H(x )y (x) y1 (x) y2 (x).
B2 [y1 ] B1 [y2 ]

Note that the Green function is well defined since B2 [y1 ] and B1 [y2 ] are nonzero. The solution for u is

Z b
u= G(x|)f () d.
a

Thus if there is a unique solution for v, the solution for y is

Z b
2 1
y= G(x|)f () d + y1 + y2 .
a B2 [y1 ] B1 [y2 ]

1102
Result 21.7.5 Consider the problem

y 00 + p(x)y 0 + q(x)y = f (x),

B1 [y] = 11 y(a) + 12 y 0 (a) + 11 y(b) + 12 y 0 (b) = 1 ,


B2 [y] = 21 y(a) + 22 y 0 (a) + 21 y(b) + 22 y 0 (b) = 2 .
If the homogeneous differential equation subject to the homogeneous boundary conditions
has no solution, then the problem has the unique solution
Z b
2 1
y= G(x|)f () d + y1 + y2 ,
a B2 [y1 ] B1 [y2 ]
where
21 y (b) + 22 y0 (b)
G(x|) = H(x )y (x) y1 (x)
B2 [y1 ]
11 y (b) + 12 y0 (b)
y2 (x),
B1 [y2 ]
y1 and y2 are solutions of the homogeneous differential equation that satisfy the first and sec-
ond boundary conditions, respectively, and y (x) is the solution of the homogeneous equation
that satisfies y () = 0, y0 () = 1.

1103
21.8 Green Functions for Higher Order Problems
Consider the nth order differential equation

L[y] = y (n) + pn1 (x)y (n1) + + p1 (x)y 0 + p0 y = f (x) on a < x < b,

subject to the n independent boundary conditions

Bj [y] = j

where the boundary conditions are of the form

n1
X n1
X
B[y] k y (k) (a) + k y (k) (b).
k=0 k=0

We assume that the coefficient functions in the differential equation are continuous on [a, b]. The solution is y = u + v
where u and v satisfy
L[u] = f (x), with Bj [u] = 0,

and
L[v] = 0, with Bj [v] = j

From Result 21.5.3, we know that if the completely homogeneous problem

L[w] = 0, with Bj [w] = 0,

has only the trivial solution, then the solution for y exists and is unique. We will construct this solution using Green
functions.

1104
First we consider the problem for v. Let {y1 , . . . , yn } be a set of linearly independent solutions. The solution for v
has the form
v = c1 y1 + + cn yn
where the constants are determined by the matrix equation

B1 [y1 ] B1 [y2 ] B1 [yn ] c1 1
B2 [y1 ] B2 [y2 ] B2 [yn ] c2 2
.. = .. .

.. .. . . ..
. . . . . .
Bn [y1 ] Bn [y2 ] Bn [yn ] cn n

To solve the problem for u we consider the Green function satisfying

L[G(x|)] = (x ), with Bj [G] = 0.

Let y (x) be the linear combination of the homogeneous solutions that satisfy the conditions

y () = 0
y0 () = 0
.. .
. = ..
(n2)
y () = 0
(n1)
y () = 1.

The causal solution is then


yc (x) = H(x )y (x).
The Green function has the form

G(x|) = H(x )y (x) + d1 y1 (x) + + dn yn (x)

1105
The constants are determined by the matrix equation

B1 [y1 ] B1 [y2 ] B1 [yn ] d1 B1 [H(x )y (x)]
B2 [y1 ] B2 [y2 ] B2 [yn ] d2 B2 [H(x )y (x)]
.. = .

.. .. .. .. ..
. . . . . .
Bn [y1 ] Bn [y2 ] Bn [yn ] dn Bn [H(x )y (x)]
The solution for u then is Z b
u= G(x|)f () d.
a

Result 21.8.1 Consider the nth order differential equation

L[y] = y (n) + pn1 (x)y (n1) + + p1 (x)y 0 + p0 y = f (x) on a < x < b,

subject to the n independent boundary conditions

Bj [y] = j

If the homogeneous differential equation subject to the homogeneous boundary conditions


has only the trivial solution, then the problem has the unique solution
Z b
y= G(x|)f () d + c1 y1 + cn yn
a

where
G(x|) = H(x )y (x) + d1 y1 (x) + + dn yn (x),
{y1 , . . . , yn } is a set of solutions of the homogeneous differential equation, and the constants
cj and dj can be determined by solving sets of linear equations.

1106
Example 21.8.1 Consider the problem
y 000 y 00 + y 0 y = f (x),
y(0) = 1, y 0 (0) = 2, y(1) = 3.
The completely homogeneous associated problem is

w000 w00 + w0 w = 0, w(0) = w0 (0) = w(1) = 0.

The solution of the differential equation is

w = c1 cos x + c2 sin x + c2 ex .

The boundary conditions give us the equation



1 0 1 c1 0
0 1 1 c2 = 0 .

cos 1 sin 1 e c3 0
The determinant of the matrix is e cos 1 sin 1 6= 0. Thus the homogeneous problem has only the trivial solution
and the inhomogeneous problem has a unique solution.
We separate the inhomogeneous problem into the two problems

u000 u00 + u0 u = f (x), u(0) = u0 (0) = u(1) = 0,

v 000 v 00 + v 0 v = 0, v(0) = 1, v 0 (0) = 2, v(1) = 3,


First we solve the problem for v. The solution of the differential equation is

v = c1 cos x + c2 sin x + c2 ex .

The boundary conditions yields the equation



1 0 1 c1 1
0 1 1 c2 = 2 .

cos 1 sin 1 e c3 3

1107
The solution for v is
1
(e + sin 1 3) cos x + (2e cos 1 3) sin x + (3 cos 1 2 sin 1) ex .
 
v=
e cos 1 sin 1
Now we find the Green function for the problem in u. The causal solution is
1
(sin cos ) cos x (sin + cos ) sin + e ex ,

H(x )u (x) = H(x )
2
1
H(x )u (x) = H(x ) ex cos(x ) sin(x ) .
 
2
The Green function has the form

G(x|) = H(x )u (x) + c1 cos x + c2 sin x + c3 ex .

The constants are determined by the three conditions

c1 cos x + c2 sin x + c3 ex x=0 = 0,


 
 
x
(c1 cos x + c2 sin x + c3 e ) = 0,
x x=0
u (x) + c1 cos x + c2 sin x + c3 ex x=1 = 0.
 

The Green function is


1  cos(1 ) + sin(1 ) e1 
G(x|) = H(x ) ex cos(x ) sin(x ) + cos x + sin x ex
 
2 2(cos 1 + sin 1 e)

The solution for v is Z 1


v= G(x|)f () d.
0

Thus the solution for y is

1108
Z 1
1 
y= G(x|)f () d + (e + sin 1 3) cos x
0 e cos 1 sin 1
+ (2e cos 1 3) sin x + (3 cos 1 2 sin 1) ex .


21.9 Fredholm Alternative Theorem


Orthogonality. Two real vectors, u and v are orthogonal if u v = 0. Consider two functions, u(x) and v(x),
defined in [a, b]. The dot product in vector space is analogous to the integral
Z b
u(x)v(x) dx
a

in function space. Thus two real functions are orthogonal if


Z b
u(x)v(x) dx = 0.
a

Consider the nth order linear inhomogeneous differential equation

L[y] = f (x) on [a, b],

subject to the linear inhomogeneous boundary conditions

Bj [y] = 0, for j = 1, 2, . . . n.

The Fredholm alternative theorem tells us if the problem has a unique solution, an infinite number of solutions, or
no solution. Before presenting the theorem, we will consider a few motivating examples.

1109
No Nontrivial Homogeneous Solutions. In the section on Green functions we showed that if the completely
homogeneous problem has only the trivial solution then the inhomogeneous problem has a unique solution.

Nontrivial Homogeneous Solutions Exist. If there are nonzero solutions to the homogeneous problem L[y] = 0
that satisfy the homogeneous boundary conditions Bj [y] = 0 then the inhomogeneous problem L[y] = f (x) subject to
the same boundary conditions either has no solution or an infinite number of solutions.
Suppose there is a particular solution yp that satisfies the boundary conditions. If there is a solution yh to the
homogeneous equation that satisfies the boundary conditions then there will be an infinite number of solutions since
yp + cyh is also a particular solution.

The question now remains: Given that there are homogeneous solutions that satisfy the boundary conditions, how
do we know if a particular solution that satisfies the boundary conditions exists? Before we address this question we
will consider a few examples.

Example 21.9.1 Consider the problem

y 00 + y = cos x, y(0) = y() = 0.

The two homogeneous solutions of the differential equation are

y1 = cos x, and y2 = sin x.

y2 = sin x satisfies the boundary conditions. Thus we know that there are either no solutions or an infinite number of

1110
solutions. A particular solution is

cos2 x
Z Z
cos x sin x
yp = cos x dx + sin x dx
1 1
Z Z  
1 1 1
= cos x sin(2x) dx + sin x + cos(2x) dx
2 2 2
 
1 1 1
= cos x cos(2x) + sin x x + sin(2x)
4 2 4
1 1  
= x sin x + cos x cos(2x) + sin x sin(2x)
2 4
1 1
= x sin x + cos x
2 4

The general solution is


1
y = x sin x + c1 cos x + c2 sin x.
2
Applying the two boundary conditions yields

1
y = x sin x + c sin x.
2

Thus there are an infinite number of solutions.

Example 21.9.2 Consider the differential equation

y 00 + y = sin x, y(0) = y() = 0.

The general solution is


1
y = x cos x + c1 cos x + c2 sin x.
2

1111
Applying the boundary conditions,

y(0) = 0 c1 = 0
1
y() = 0 cos() + c2 sin() = 0
2

= 0.
2

Since this equation has no solution, there are no solutions to the inhomogeneous problem.

In both of the above examples there is a homogeneous solution y = sin x that satisfies the boundary conditions.
In Example 21.9.1, the inhomogeneous term is cos x and there are an infinite number of solutions. In Example 21.9.2,
the inhomogeneity is sin x and there are no solutions. In general, if the inhomogeneous term is orthogonal to all the
homogeneous solutions that satisfy the boundary conditions then there are an infinite number of solutions. If not, there
are no inhomogeneous solutions.

1112
Result 21.9.1 Fredholm Alternative Theorem. Consider the nth order inhomogeneous
problem

L[y] = f (x) on [a, b] subject to Bj [y] = 0 for j = 1, 2, . . . , n,

and the associated homogeneous problem

L[y] = 0 on [a, b] subject to Bj [y] = 0 for j = 1, 2, . . . , n.

If the homogeneous problem has only the trivial solution then the inhomogeneous problem has
a unique solution. If the homogeneous problem has m independent solutions, {y1 , y2 , . . . , ym },
then there are two possibilities:
If f (x) is orthogonal to each of the homogeneous solutions then there are an infinite
number of solutions of the form
m
X
y = yp + cj yj .
j=1

If f (x) is not orthogonal to each of the homogeneous solutions then there are no inho-
mogeneous solutions.

Example 21.9.3 Consider the problem


y 00 + y = cos 2x, y(0) = 1, y() = 2.
cos x and sin x are two linearly independent solutions to the homogeneous equation. sin x satisfies the homogeneous
boundary conditions. Thus there are either an infinite number of solutions, or no solution.

1113
x
To transform this problem to one with homogeneous boundary conditions, we note that g(x) =
+ 1 and make
the change of variables y = u + g to obtain
x
u00 + u = cos 2x 1, y(0) = 0, y() = 0.

Since cos 2x x 1 is not orthogonal to sin x, there is no solution to the inhomogeneous problem.
To check this, the general solution is
1
y = cos 2x + c1 cos x + c2 sin x.
3
Applying the boundary conditions,
4
y(0) = 1 c1 =
3
1 4
y() = 2 = 2.
3 3
Thus we see that the right boundary condition cannot be satisfied.

Example 21.9.4 Consider


y 00 + y = cos 2x, y 0 (0) = y() = 1.
There are no solutions to the homogeneous equation that satisfy the homogeneous boundary conditions. To check this,
note that all solutions of the homogeneous equation have the form uh = c1 cos x + c2 sin x.

u0h (0) = 0 c2 = 0
uh () = 0 c1 = 0.

From the Fredholm Alternative Theorem we see that the inhomogeneous problem has a unique solution.
To find the solution, start with
1
y = cos 2x + c1 cos x + c2 sin x.
3

1114
y 0 (0) = 1 c2 = 1
1
y() = 1 c1 = 1
3
Thus the solution is
1 4
y = cos 2x cos x + sin x.
3 3

Example 21.9.5 Consider


2 4
y 00 + y = cos 2x,
y(0) = , y() = .
3 3
cos x and sin x satisfy the homogeneous differential equation. sin x satisfies the homogeneous boundary conditions.
Since g(x) = cos x 1/3 satisfies the boundary conditions, the substitution y = u + g yields

1
u00 + u = cos 2x + , y(0) = 0, y() = 0.
3

Now we check if sin x is orthogonal to cos 2x + 31 .


Z   Z
1 1 1 1
sin x cos 2x + dx = sin 3x sin x + sin x dx
0 3 2 2 3
0 
1 1
= cos 3x + cos x
6 6 0
=0

Since sin x is orthogonal to the inhomogeneity, there are an infinite number of solutions to the problem for u, (and
hence the problem for y).
As a check, then general solution for y is

1
y = cos 2x + c1 cos x + c2 sin x.
3

1115
Applying the boundary conditions,
2
y(0) = c1 = 1
3
4 4 4
y() = = .
3 3 3
Thus we see that c2 is arbitrary. There are an infinite number of solutions of the form

1
y = cos 2x + cos x + c sin x.
3

1116
21.10 Exercises
Undetermined Coefficients
Exercise 21.1 (mathematica/ode/inhomogeneous/undetermined.nb)
Find the general solution of the following equations.

1. y 00 + 2y 0 + 5y = 3 sin(2t)

2. 2y 00 + 3y 0 + y = t2 + 3 sin(t)

Hint, Solution
Exercise 21.2 (mathematica/ode/inhomogeneous/undetermined.nb)
Find the solution of each one of the following initial value problems.

1. y 00 2y 0 + y = t et +4, y(0) = 1, y 0 (0) = 1

2. y 00 + 2y 0 + 5y = 4 et cos(2t), y(0) = 1, y 0 (0) = 0

Hint, Solution

Variation of Parameters
Exercise 21.3 (mathematica/ode/inhomogeneous/variation.nb)
Use the method of variation of parameters to find a particular solution of the given differential equation.

1. y 00 5y 0 + 6y = 2 et

2. y 00 + y = tan(t), 0 < t < /2

3. y 00 5y 0 + 6y = g(t), for a given function g.

Hint, Solution

1117
Exercise 21.4 (mathematica/ode/inhomogeneous/variation.nb)
Solve
y 00 (x) + y(x) = x, y(0) = 1, y 0 (0) = 0.
Hint, Solution
Exercise 21.5 (mathematica/ode/inhomogeneous/variation.nb)
Solve
x2 y 00 (x) xy 0 (x) + y(x) = x.
Hint, Solution
Exercise 21.6 (mathematica/ode/inhomogeneous/variation.nb)
1. Find the general solution of y 00 + y = ex .

2. Solve y 00 + 2 y = sin x, y(0) = y 0 (0) = 0. is an arbitrary real constant. Is there anything special about = 1?
Hint, Solution
Exercise 21.7 (mathematica/ode/inhomogeneous/variation.nb)
Consider the problem of solving the initial value problem

y 00 + y = g(t), y(0) = 0, y 0 (0) = 0.

1. Show that the general solution of y 00 + y = g(t) is


 Z t   Z t 
y(t) = c1 g( ) sin d cos t + c2 + g( ) cos d sin t,
a b

where c1 and c2 are arbitrary constants and a and b are any conveniently chosen points.

2. Using the result of part (a) show that the solution satisfying the initial conditions y(0) = 0 and y 0 (0) = 0 is given
by Z t
y(t) = g( ) sin(t ) d.
0

1118
Notice that this equation gives a formula for computing the solution of the original initial value problem for any
given inhomogeneous term g(t). The integral is referred to as the convolution of g(t) with sin t.
3. Use the result of part (b) to solve the initial value problem,
y 00 + y = sin(t), y(0) = 0, y 0 (0) = 0,
where is a real constant. How does the solution for = 1 differ from that for 6= 1? The = 1 case provides
an example of resonant forcing. Plot the solution for resonant and non-resonant forcing.
Hint, Solution
Exercise 21.8
Find the variation of parameters solution for the third order differential equation
y 000 + p2 (x)y 00 + p1 (x)y 0 + p0 (x)y = f (x).
Hint, Solution

Green Functions
Exercise 21.9
Use a Green function to solve
y 00 = f (x), y() = y 0 () = 0.
Verify the the solution satisfies the differential equation.
Hint, Solution
Exercise 21.10
Solve the initial value problem
1 1
y 00 + y 0 2 y = x2 , y(0) = 0, y 0 (0) = 1.
x x
First use variation of parameters, and then solve the problem with a Green function.
Hint, Solution

1119
Exercise 21.11
What are the continuity conditions at x = for the Green function for the problem

y 000 + p2 (x)y 00 + p1 (x)y 0 + p0 (x)y = f (x).

Hint, Solution
Exercise 21.12
Use variation of parameters and Green functions to solve

x2 y 00 2xy 0 + 2y = ex , y(1) = 0, y 0 (1) = 1.

Hint, Solution
Exercise 21.13
Find the Green function for
y 00 y = f (x), y 0 (0) = y(1) = 0.
Hint, Solution
Exercise 21.14
Find the Green function for
y 00 y = f (x), y(0) = y() = 0.
Hint, Solution
Exercise 21.15
Find the Green function for each of the following:

a) xu00 + u0 = f (x), u(0+ ) bounded, u(1) = 0.

b) u00 u = f (x), u(a) = u(a) = 0.

c) u00 u = f (x), u(x) bounded as |x| .

1120
d) Show that the Green function for (b) approaches that for (c) as a .
Hint, Solution
Exercise 21.16
1. For what values of does the problem

y 00 + y = f (x), y(0) = y() = 0, (21.5)

have a unique solution? Find the Green functions for these cases.

2. For what values of does the problem

y 00 + 9y = 1 + x, y(0) = y() = 0,

have a solution? Find the solution.

3. For = n2 , n Z+ state in general the conditions on f in Equation 21.5 so that a solution will exist. What is
the appropriate modified Green function (in terms of eigenfunctions)?
Hint, Solution
Exercise 21.17
Show that the inhomogeneous boundary value problem:

Lu (pu0 )0 + qu = f (x), a < x < b, u(a) = , u(b) =

has the solution: Z b


u(x) = g(x; )f () d p(a)g (x; a) + p(b)g (x; b).
a
Hint, Solution
Exercise 21.18
The Green function for
u00 k 2 u = f (x), < x <

1121
subject to |u()| < is
1 k|x|
G(x; ) =
e .
2k
(We assume that k > 0.) Use the image method to find the Green function for the same equation on the semi-infinite
interval 0 < x < satisfying the boundary conditions,

i) u(0) = 0 |u()| < ,


ii) u0 (0) = 0 |u()| < .

Express these results in simplified forms without absolute values.


Hint, Solution
Exercise 21.19
1. Determine the Green function for solving:

y 00 a2 y = f (x), y(0) = y 0 (L) = 0.

2. Take the limit as L to find the Green function on (0, ) for the boundary conditions: y(0) = 0, y 0 () = 0.
We assume here that a > 0. Use the limiting Green function to solve:

y 00 a2 y = ex , y(0) = 0, y 0 () = 0.

Check that your solution satisfies all the conditions of the problem.
Hint, Solution

1122
21.11 Hints
Undetermined Coefficients
Hint 21.1

Hint 21.2

Variation of Parameters
Hint 21.3

Hint 21.4

Hint 21.5

Hint 21.6

Hint 21.7

Hint 21.8
Look for a particular solution of the form
y p = u 1 y 1 + u 2 y 2 + u3 y 3 ,

1123
where the yj s are homogeneous solutions. Impose the constraints

u01 y1 + u02 y2 + u03 y3 = 0


u01 y10 + u02 y20 + u03 y30 = 0.

To avoid some messy algebra when solving for u0j , use Kramers rule.

Green Functions
Hint 21.9

Hint 21.10

Hint 21.11

Hint 21.12

Hint 21.13
cosh(x) and sinh(x 1) are homogeneous solutions that satisfy the left and right boundary conditions, respectively.

Hint 21.14
sinh(x) and ex are homogeneous solutions that satisfy the left and right boundary conditions, respectively.

Hint 21.15
The Green function for the differential equation

d
L[y] (p(x)y 0 ) + q(x)y = f (x),
dx

1124
subject to unmixed, homogeneous boundary conditions is

y1 (x< )y2 (x> )


G(x|) = ,
p()W ()
(y
1 (x)y2 ()
p()W ()
for a x ,
G(x|) = y1 ()y2 (x)
p()W ()
for x b,
where y1 and y2 are homogeneous solutions that satisfy the left and right boundary conditions, respectively.
Recall that if y(x) is a solution of a homogeneous, constant coefficient differential equation then y(x + c) is also a
solution.
Hint 21.16
The problem has a Green function if and only if the inhomogeneous problem has a unique solution. The inhomogeneous
problem has a unique solution if and only if the homogeneous problem has only the trivial solution.
Hint 21.17
Show that g (x; a) and g (x; b) are solutions of the homogeneous differential equation. Determine the value of these
solutions at the boundary.
Hint 21.18

Hint 21.19

1125
21.12 Solutions
Undetermined Coefficients
Solution 21.1
1. We consider
y 00 + 2y 0 + 5y = 3 sin(2t).
We first find the homogeneous solution with the substitition y = et .

2 + 2 + 5 = 0
= 1 2i

The homogeneous solution is


yh = c1 et cos(2t) + c2 et sin(2t).
We guess a particular solution of the form

yp = a cos(2t) + b sin(2t).

We substitute this into the differential equation to determine the coefficients.

yp00 + 2yp0 + 5yp = 3 sin(2t)


4a cos(2t) 4b sin(2t) 4a sin(2t) + 4b sin(2t) + 5a cos(2t) + 5b sin(2t) = 3 sin(2t)
(a + 4b) cos(2t) + (3 4a + b) sin(2t) = 0
a + 4b = 0, 4a + b = 3
12 3
a= , b=
17 17
A particular solution is
3
yp = (sin(2t) 4 cos(2t)).
17

1126
The general solution of the differential equation is

3
y = c1 et cos(2t) + c2 et sin(2t) + (sin(2t) 4 cos(2t)).
17

2. We consider
2y 00 + 3y 0 + y = t2 + 3 sin(t)
We first find the homogeneous solution with the substitition y = et .

22 + 3 + 1 = 0
= {1, 1/2}

The homogeneous solution is


yh = c1 et +c2 et/2 .
We guess a particular solution of the form

yp = at2 + bt + c + d cos(t) + e sin(t).

We substitute this into the differential equation to determine the coefficients.

2yp00 + 3yp0 + yp = t2 + 3 sin(t)

2(2a d cos(t) e sin(t)) + 3(2at + b d sin(t) + e cos(t))


+ at2 + bt + c + d cos(t) + e sin(t) = t2 + 3 sin(t)

(a 1)t2 + (6a + b)t + (4a + 3b + c) + (d + 3e) cos(t) (3 + 3d + e) sin(t) = 0


a 1 = 0, 6a + b = 0, 4a + 3b + c = 0, d + 3e = 0, 3 + 3d + e = 0
9 3
a = 1, b = 6, c = 14, d = , e =
10 10

1127
A particular solution is
3
yp = t2 6t + 14 (3 cos(t) + sin(t)).
10
The general solution of the differential equation is
3
y = c1 et +c2 et/2 +t2 6t + 14 (3 cos(t) + sin(t)).
10
Solution 21.2
1. We consider the problem
y 00 2y 0 + y = t et +4, y(0) = 1, y 0 (0) = 1.
First we solve the homogeneous equation with the substitution y = et .
2 2 + 1 = 0
( 1)2 = 0
=1
The homogeneous solution is
yh = c1 et +c2 t et .
We guess a particular solution of the form
yp = at3 et +bt2 et +4.
We substitute this into the inhomogeneous differential equation to determine the coefficients.
yp00 2yp0 + yp = t et +4
(a(t3 + 6t2 + 6t) + b(t2 + 4t + 2)) et 2(a(t2 + 3t) + b(t + 2)) et at3 et +bt2 et +4 = t et +4
(6a 1)t + 2b = 0
6a 1 = 0, 2b = 0
1
a= , b=0
6

1128
A particular solution is
t3 t
yp = e +4.
6
The general solution of the differential equation is

t3 t
y = c1 et +c2 t et + e +4.
6
We use the initial conditions to determine the constants of integration.

y(0) = 1, y 0 (0) = 1
c1 + 4 = 1, c1 + c2 = 1
c1 = 3, c2 = 4

The solution of the initial value problem is

t3
 
y= + 4t 3 et +4.
6

2. We consider the problem

y 00 + 2y 0 + 5y = 4 et cos(2t), y(0) = 1, y 0 (0) = 0.

First we solve the homogeneous equation with the substitution y = et .

2 + 2 + 5 = 0

= 1 1 5
= 1 2

The homogeneous solution is


yh = c1 et cos(2t) + c2 et sin(2t).

1129
We guess a particular solution of the form
yp = t et (a cos(2t) + b sin(2t))
We substitute this into the inhomogeneous differential equation to determine the coefficients.
yp00 + 2yp0 + 5yp = 4 et cos(2t)

et (((2 + 3t)a + 4(1 t)b) cos(2t) + (4(t 1)a (2 + 3t)b) sin(2t))


+ 2 et (((1 t)a + 2tb) cos(2t) + (2ta + (1 t)b) sin(2t))
+ 5(et (ta cos(2t) + tb sin(2t))) = 4 et cos(2t)

4(b 1) cos(2t) 4a sin(2t) = 0


a = 0, b = 1
A particular solution is
yp = t et sin(2t).
The general solution of the differential equation is
y = c1 et cos(2t) + c2 et sin(2t) + t et sin(2t).
We use the initial conditions to determine the constants of integration.
y(0) = 1, y 0 (0) = 0
c1 = 1, c1 + 2c2 = 0
1
c1 = 1, c2 =
2
The solution of the initial value problem is
1 t
y= e (2 cos(2t) + (2t + 1) sin(2t)) .
2

1130
Variation of Parameters
Solution 21.3
1. We consider the equation
y 00 5y 0 + 6y = 2 et .
We find homogeneous solutions with the substitution y = et .

2 5 + 6 = 0
= {2, 3}

The homogeneous solutions are


y1 = e2t , y2 = e3t .
We compute the Wronskian of these solutions.
2t
e e3t
W (t) = 2t
= e5t
2 e 3 e3t

We find a particular solution with variation of parameters.

2 et e3t 2 et e2t
Z Z
2t 3t
yp = e dt + e dt
e5t e5t
Z Z
= 2 e2t et dt + 2 e3t e2t dt

= 2 et et

y p = et

2. We consider the equation



y 00 + y = tan(t), 0<t< .
2

1131
We find homogeneous solutions with the substitution y = et .

2 + 1 = 0
= i

The homogeneous solutions are


y1 = cos(t), y2 = sin(t).
We compute the Wronskian of these solutions.

cos(t) sin(t)
W (t) = = cos2 (t) + sin2 (t) = 1
sin(t) cos(t)

We find a particular solution with variation of parameters.


Z Z
yp = cos(t) tan(t) sin(t) dt + sin(t) tan(t) cos(t) dt

sin2 (t)
Z Z
= cos(t) dt + sin(t) sin(t) dt
cos(t)
  
cos(t/2) sin(t/2)
= cos(t) ln + sin(t) sin(t) cos(t)
cos(t/2) + sin(t/2)
 
cos(t/2) sin(t/2)
yp = cos(t) ln
cos(t/2) + sin(t/2)

3. We consider the equation


y 00 5y 0 + 6y = g(t).
The homogeneous solutions are
y1 = e2t , y2 = e3t .

1132
The Wronskian of these solutions is W (t) = e5t . We find a particular solution with variation of parameters.
g(t) e3t g(t) e2t
Z Z
2t 3t
yp = e dt + e dt
e5t e5t
Z Z
2t
yp = e 2t
g(t) e dt + e 3t
g(t) e3t dt

Solution 21.4
Solve
y 00 (x) + y(x) = x, y(0) = 1, y 0 (0) = 0.
The solutions of the homogeneous equation are
y1 (x) = cos x, y2 (x) = sin x.
The Wronskian of these solutions is

cos x sin x
W [cos x, sin x] =

sin x cos x
= cos2 x + sin2 x
= 1.
The variation of parameters solution for the particular solution is
Z Z
yp = cos x x sin x dx + sin x x cos x dx
 Z   Z 
= cos x x cos x + cos x dx + sin x x sin x sin x dx

= cos x (x cos x + sin x) + sin x (x sin x + cos x)


= x cos2 x cos x sin x + x sin2 x + cos x sin x
=x

1133
The general solution of the differential equation is thus
y = c1 cos x + c2 sin x + x.
Applying the two initial conditions gives us the equations
c1 = 1, c2 + 1 = 0.
The solution subject to the initial conditions is
y = cos x sin x + x.
Solution 21.5
Solve
x2 y 00 (x) xy 0 (x) + y(x) = x.
The homogeneous equation is
x2 y 00 (x) xy 0 (x) + y(x) = 0.
Substituting y = x into the homogeneous differential equation yields
x2 ( 1)x2 xx + x = 0
2 2 + 1 = 0
( 1)2 = 0
= 1.
The homogeneous solutions are
y1 = x, y2 = x log x.
The Wronskian of the homogeneous solutions is

x x log x
W [x, x log x] =

1 1 + log x
= x + x log x x log x
= x.

1134
Writing the inhomogeneous equation in the standard form:

1 1 1
y 00 (x) y 0 (x) + 2 y(x) = .
x x x
Using variation of parameters to find the particular solution,
Z Z
log x 1
yp = x dx + x log x dx
x x
1
= x log2 x + x log x log x
2
1
= x log2 x.
2
Thus the general solution of the inhomogeneous differential equation is

1
y = c1 x + c2 x log x + x log2 x.
2

Solution 21.6
1. First we find the homogeneous solutions. We substitute y = ex into the homogeneous differential equation.

y 00 + y = 0
2 + 1 = 0
=
y = ex , ex


We can also write the solutions in terms of real-valued functions.

y = {cos x, sin x}

1135
The Wronskian of the homogeneous solutions is

cos x sin x
W [cos x, sin x] = = cos2 x + sin2 x = 1.
sin x cos x

We obtain a particular solution with the variation of parameters formula.


Z Z
yp = cos x e sin x dx + sin x ex cos x dx
x

1 1
yp = cos x ex (sin x cos x) + sin x ex (sin x + cos x)
2 2
1 x
yp = e
2
The general solution is the particular solution plus a linear combination of the homogeneous solutions.

1 x
y= e + cos x + sin x
2

2.
y 00 + 2 y = sin x, y(0) = y 0 (0) = 0
Assume that is positive. First we find the homogeneous solutions by substituting y = ex into the homogeneous
differential equation.

y 00 + 2 y = 0
2 + 2 = 0
=
y = ex , ex


y = {cos(x), sin(x)}

1136
The Wronskian of these homogeneous solution is

cos(x) sin(x)
W [cos(x), sin(x)] =
= cos2 (x) + sin2 (x) = .
sin(x) cos(x)

We obtain a particular solution with the variation of parameters formula.


Z Z
sin(x) sin x cos(x) sin x
yp = cos(x) dx + sin(x) dx

We evaluate the integrals for 6= 1.

cos(x) sin(x) sin x cos(x) cos(x) cos(x) + sin x sin(x)


yp = cos(x) 2
+ sin(x)
( 1) (2 1)
sin x
yp = 2
1
The general solution for 6= 1 is
sin x
y= + c1 cos(x) + c2 sin(x).
2 1
The initial conditions give us the constraints:

c1 = 0,
1
+ c2 = 0,
2 1
For 6= 1, (non-resonant forcing), the solution subject to the initial conditions is

sin(x) sin(x)
y= .
(2 1)

1137
Now consider the case = 1. We obtain a particular solution with the variation of parameters formula.
Z Z
2
yp = cos(x) sin (x) dx + sin(x) cos(x) sin x dx
 
1 1 2
yp = cos(x) (x cos(x) sin(x)) + sin(x) cos (x)
2 2
1
yp = x cos(x)
2
The general solution for = 1 is
1
y = x cos(x) + c1 cos(x) + c2 sin(x).
2
The initial conditions give us the constraints:
c1 = 0
1
+ c2 = 0
2
For = 1, (resonant forcing), the solution subject to the initial conditions is

1
y = (sin(x) x cos x).
2
Solution 21.7
1. A set of linearly independent, homogeneous solutions is {cos t, sin t}. The Wronskian of these solutions is

cos t sin t
W (t) =
= cos2 t + sin2 t = 1.
sin t cos t
We use variation of parameters to find a particular solution.
Z Z
yp = cos t g(t) sin t dt + sin t g(t) cos t dt

1138
The general solution can be written in the form,
 Z t   Z t 
y(t) = c1 g( ) sin d cos t + c2 + g( ) cos d sin t.
a b

2. Since the initial conditions are given at t = 0 we choose the lower bounds of integration in the general solution
to be that point.  Z t   Z t 
y = c1 g( ) sin d cos t + c2 + g( ) cos d sin t
0 0

The initial condition y(0) = 0 gives the constraint, c1 = 0. The derivative of y(t) is then,
Z t  Z t 
0
y (t) = g(t) sin t cos t + g( ) sin d sin t + g(t) cos t sin t + c2 + g( ) cos d cos t,
0 0
Z t  Z t 
0
y (t) = g( ) sin d sin t + c2 + g( ) cos d cos t.
0 0

The initial condition y 0 (0) = 0 gives the constraint c2 = 0. The solution subject to the initial conditions is
Z t
y= g( )(sin t cos cos t sin ) d
0
Z t
y= g( ) sin(t ) d
0

3. The solution of the initial value problem

y 00 + y = sin(t), y(0) = 0, y 0 (0) = 0,

is Z t
y= sin( ) sin(t ) d.
0

1139
Figure 21.5: Non-resonant Forcing

For 6= 1, this is

1 t
Z

y= cos(t ) cos(t + ) d
2 0
 t
1 sin(t ) sin(t + )
= +
2 1+ 1 0
 
1 sin(t) sin(t) sin(t) + sin(t)
= +
2 1+ 1

sin t sin(t)
y= 2
+ . (21.6)
1 1 2

The solution is the sum of two periodic functions of period 2 and 2/. This solution is plotted in Figure 21.5
on the interval t [0, 16] for the values = 1/4, 7/8, 5/2.

1140
Figure 21.6: Resonant Forcing

For = 1, we have

1 t
Z

y= cos(t 2 ) cos(tau) d
2 0
 t
1 1
= sin(t 2 ) cos t
2 2 0

1
y= (sin t t cos t) . (21.7)
2
The solution has both a periodic and a transient term. This solution is plotted in Figure 21.5 on the interval
t [0, 16].
Note that we can derive (21.7) from (21.6) by taking the limit as 0.

sin(t) sin t t cos(t) sin t


lim 2
= lim
1 1 1 2
1
= (sin t t cos t)
2

1141
Solution 21.8
Let y1 , y2 and y3 be linearly independent homogeneous solutions to the differential equation

L[y] = y 000 + p2 y 00 + p1 y 0 + p0 y = f (x).

We will look for a particular solution of the form

y p = u 1 y 1 + u 2 y 2 + u3 y 3 .

Since the uj s are undetermined functions, we are free to impose two constraints. We choose the constraints to simplify
the algebra.

u01 y1 + u02 y2 + u03 y3 = 0


u01 y10 + u02 y20 + u03 y30 = 0

Differentiating the expression for yp ,

yp0 = u01 y1 + u1 y10 + u02 y2 + u2 y20 + u03 y3 + u3 y30


= u1 y10 + u2 y20 + u3 y30
yp00 = u01 y10 + u1 y100 + u02 y20 + u2 y200 + u03 y30 + u3 y300
= u1 y100 + u2 y200 + u3 y300
yp000 = u01 y100 + u1 y1000 + u02 y200 + u2 y2000 + u03 y300 + u3 y3000

Substituting the expressions for yp and its derivatives into the differential equation,

u01 y100 + u1 y1000 + u02 y200 + u2 y2000 + u03 y300 + u3 y3000 + p2 (u1 y100 + u2 y200 + u3 y300 ) + p1 (u1 y10 + u2 y20 + u3 y30 )
+ p0 (u1 y1 + u2 y2 + u3 y3 ) = f (x)

u01 y100 + u02 y200 + u03 y300 + u1 L[y1 ] + u2 L[y2 ] + u3 L[y3 ] = f (x)
u01 y100 + u02 y200 + u03 y300 = f (x).

1142
With the two constraints, we have the system of equations,

u01 y1 + u02 y2 + u03 y3 = 0


u01 y10 + u02 y20 + u03 y30 = 0
u01 y100 + u02 y200 + u03 y300 = f (x)

We solve for the u0j using Kramers rule.

(y2 y30 y20 y3 )f (x) (y1 y30 y10 y3 )f (x) (y1 y20 y10 y2 )f (x)
u01 = , u02 = , u03 =
W (x) W (x) W (x)

Here W (x) is the Wronskian of {y1 , y2 , y3 }. Integrating the expressions for u0j , the particular solution is

(y2 y30 y20 y3 )f (x) (y3 y10 y30 y1 )f (x) (y1 y20 y10 y2 )f (x)
Z Z Z
yp = y1 dx + y2 dx + y3 dx.
W (x) W (x) W (x)

Green Functions
Solution 21.9
We consider the Green function problem

G00 = f (x), G(|) = G0 (|) = 0.

The homogeneous solution is y = c1 + c2 x. The homogeneous solution that satisfies the boundary conditions is y = 0.
Thus the Green function has the form (
0 x < ,
G(x|) =
c1 + c2 x x > .
The continuity and jump conditions are then

G( + |) = 0, G0 ( + |) = 1.

1143
Thus the Green function is (
0 x < ,
G(x|) = = (x )H(x ).
x x>

The solution of the problem


y 00 = f (x), y() = y 0 () = 0.
is
Z
y= f ()G(x|) d

Z
y= f ()(x )H(x ) d

Z x
y= f ()(x ) d

We differentiate this solution to verify that it satisfies the differential equation.


Z x Z x
0
y = [f ()(x )]=x + (f ()(x )) d = f () d
x
y 00 = [f ()]=x = f (x)

Solution 21.10
Since we are dealing with an Euler equation, we substitute y = x to find the homogeneous solutions.

( 1) + 1 = 0
( 1)( + 1) = 0
1
y1 = x, y2 =
x

1144
Variation of Parameters. The Wronskian of the homogeneous solutions is

x 1/x 1 1 2
W (x) = 2 = = .

1 1/x x x x

A particular solution is

x2 (1/x) x2 x
Z Z
1
yp = x dx + dx
2/x x 2/x
x2 x4
Z Z
1
= x dx + dx
2 x 2
4 4
x x
=
6 10
x4
= .
15

The general solution is


x4 1
y= + c1 x + c2 .
15 x
Applying the initial conditions,

y(0) = 0 c2 = 0
y 0 (0) = 0 c1 = 1.

Thus we have the solution


x4
y= + x.
15

1145
Green Function. Since this problem has both an inhomogeneous term in the differential equation and inhomoge-
neous boundary conditions, we separate it into the two problems

1 1
u00 + u0 2 u = x2 , u(0) = u0 (0) = 0,
x x
00 1 0 1
v + v 2 v = 0, v(0) = 0, v 0 (0) = 1.
x x

First we solve the inhomogeneous differential equation with the homogeneous boundary conditions. The Green
function for this problem satisfies

L[G(x|)] = (x ), G(0|) = G0 (0|) = 0.

Since the Green function must satisfy the homogeneous boundary conditions, it has the form
(
0 for x <
G(x|) =
cx + d/x for x > .

From the continuity condition,


0 = c + d/.

The jump condition yields


c d/ 2 = 1.

Solving these two equations, we obtain


(
0 for x <
G(x|) = 1 2
2
x 2x
for x >

1146
Thus the solution is
Z
u(x) = G(x|) 2 d
0
Z x
2

1
= x 2 d
0 2 2x
1 4 1
= x x4
6 10
x4
= .
15

Now to solve the homogeneous differential equation with inhomogeneous boundary conditions. The general solution
for v is
v = cx + d/x.
Applying the two boundary conditions gives
v = x.
Thus the solution for y is
x4
y =x+ .
15

Solution 21.11
The Green function satisfies
G000 (x|) + p2 (x)G00 (x|) + p1 (x)G0 (x|) + p0 (x)G(x|) = (x ).
First note that only the G000 (x|) term can have a delta function singularity. If a lower derivative had a delta function
type singularity, then G000 (x|) would be more singular than a delta function and there would be no other term in the
equation to balance that behavior. Thus we see that G000 (x|) will have a delta function singularity; G00 (x|) will have
a jump discontinuity; G0 (x|) will be continuous at x = . Integrating the differential equation from to + yields
Z + Z +
000
G (x|) dx = (x ) dx

1147
G00 ( + |) G00 ( |) = 1.
Thus we have the three continuity conditions:

G00 ( + |) = G00 ( |) + 1
G0 ( + |) = G0 ( |)
G( + |) = G( |)

Solution 21.12
Variation of Parameters. Consider the problem

x2 y 00 2xy 0 + 2y = ex , y(1) = 0, y 0 (1) = 1.

Previously we showed that two homogeneous solutions are

y1 = x, y 2 = x2 .

The Wronskian of these solutions is


x x2
W (x) =
= 2x2 x2 = x2 .
1 2x
In the variation of parameters formula, we will choose 1 as the lower bound of integration. (This will simplify the
algebra in applying the initial conditions.)
Z x 2 Z x
e 2 e
yp = x d + x d
1 4 1 4
Z x Z x
e 2 e
= x 2
d + x d
1 1 3
Z x 
ex ex 1 x e
  x 
e e
Z
1 2
= x e d + x 2+ d
x 1 2x 2x 2 1
 Z x
x + x2

1 1 x e
= x e + (1 + x) e + d
2 2 1

1148
If you wanted to, you could write the last integral in terms of exponential integral functions.
The general solution is
 Z x
x2

2 1 1 x e
y = c1 x + c2 x x e + (1 + x) e + x + d
2 2 1
Applying the boundary conditions,

y(1) = 0 c1 + c2 = 0
y 0 (1) = 1 c1 + 2c2 = 1,

we find that c1 = 1, c2 = 1.
Thus the solution subject to the initial conditions is
 Z x
x2

1 12 x e
y = (1 + e )x + x + (1 + x) e + x + d
2 2 1

Green Functions. The solution to the problem is y = u + v where


2 2 ex
u00 u0 + 2 u = 2 , u(1) = 0, u0 (1) = 0,
x x x
and
2 2
v 00 v 0 + 2 v = 0, v(1) = 0, v 0 (1) = 1.
x x
The problem for v has the solution
v = x + x2 .
The Green function for u is
G(x|) = H(x )u (x)
where
u () = 0, and u0 () = 1.

1149
Thus the Green function is
x2
 
G(x|) = H(x ) x + .

The solution for u is then

e
Z
u= G(x|) 2 d

Z1 x 
x2 e

= x + d
1 2
 Z x
x2

1 1 x e
= x e + (1 + x) e + x + d.
2 2 1
Thus we find the solution for y is
 Z x
x2

1 1 2 x e
y = (1 + e )x + x + (1 + x) e + x + d
2 2 1
Solution 21.13
The differential equation for the Green function is
G00 G = (x ), Gx (0|) = G(1|) = 0.
Note that cosh(x) and sinh(x 1) are homogeneous solutions that satisfy the left and right boundary conditions,
respectively. The Wronskian of these two solutions is

cosh(x) sinh(x 1)
W (x) =
sinh(x) cosh(x 1)
= cosh(x) cosh(x 1) sinh(x) sinh(x 1)
1
ex + ex ex1 + ex+1 ex ex ex1 ex+1
   
=
4
1 1
e + e1

=
2
= cosh(1).

1150
The Green function for the problem is then

cosh(x< ) sinh(x> 1)
G(x|) = ,
cosh(1)
( cosh(x) sinh(1)
cosh(1)
for 0 x ,
G(x|) = cosh() sinh(x1)
cosh(1)
for x 1.

Solution 21.14
The differential equation for the Green function is

G00 G = (x ), G(0|) = G(|) = 0.

Note that sinh(x) and ex are homogeneous solutions that satisfy the left and right boundary conditions, respectively.
The Wronskian of these two solutions is

sinh(x) ex
W (x) =
cosh(x) ex
= sinh(x) ex cosh(x) ex
1 1
= ex ex ex ex + ex ex
 
2 2
= 1

The Green function for the problem is then

G(x|) = sinh(x< ) ex>


(
sinh(x) e for 0 x ,
G(x|) =
sinh() ex for x .

1151
Solution 21.15

a) The Green function problem is

xG00 (x|) + G0 (x|) = (x ), G(0|) bounded, G(1|) = 0.

First we find the homogeneous solutions of the differential equation.

xy 00 + y 0 = 0

This is an exact equation.


d
[xy 0 ] = 0
dx
c1
y0 =
x
y = c1 log x + c2
The homogeneous solutions y1 = 1 and y2 = log x satisfy the left and right boundary conditions, respectively.
The Wronskian of these solutions is
1 log x 1
W (x) = = .
0 1/x x
The Green function is
1 log x>
G(x|) = ,
(1/)

G(x|) = log x> .

b) The Green function problem is

G00 (x|) G(x|) = (x ), G(a|) = G(a|) = 0.

1152
{ex , ex } and {cosh x, sinh x} are both linearly independent sets of homogeneous solutions. sinh(x + a) and
sinh(x a) are homogeneous solutions that satisfy the left and right boundary conditions, respectively. The
Wronskian of these two solutions is,

sinh(x + a) sinh(x a)
W (x) =

cosh(x + a) cosh(x a)
= sinh(x + a) cosh(x a) sinh(x a) cosh(x + a)
= sinh(2a)

The Green function is


sinh(x< + a) sinh(x> a)
G(x|) = .
sinh(2a)

c) The Green function problem is

G00 (x|) G(x|) = (x ), G(x|) bounded as |x| .

ex and ex are homogeneous solutions that satisfy the left and right boundary conditions, respectively. The
Wronskian of these solutions is x
e e x
W (x) = x = 2.
e ex
The Green function is
ex< ex>
G(x|) = ,
2
1
G(x|) = ex< x> .
2

d) The Green function from part (b) is,

sinh(x< + a) sinh(x> a)
G(x|) = .
sinh(2a)

1153
We take the limit as a .

sinh(x< + a) sinh(x> a) (ex< +a ex< a ) (ex> a ex> +a )


lim = lim
a sinh(2a) a 2 (e2a e2a )
ex< x> + ex< +x> 2a + ex< x> 2a ex< +x> 4a
= lim
a 2 2 e4a
ex< x>
=
2

Thus we see that the solution from part (b) approaches the solution from part (c) as a .

Solution 21.16
1. The problem,
y 00 + y = f (x), y(0) = y() = 0,
has a Green function if and only if it has a unique solution. This inhomogeneous problem has a unique solution
if and only if the homogeneous problem has only the trivial solution.
First consider the case = 0. We find the general solution of the homogeneous differential equation.

y = c1 + c2 x

Only the trivial solution satisfies the boundary conditions. The problem has a unique solution for = 0.
Now consider non-zero . We find the general solution of the homogeneous differential equation.
   
y = c1 cos x + c2 sin x .

The solution that satisfies the left boundary condition is


 
y = c sin x .

1154
We apply the right boundary condition and find nontrivial solutions.
 
sin = 0
= n2 , n Z+
Thus the problem has a unique solution for all complex except = n2 , n Z+ .
Consider the case = 0. We find solutions of the homogeneous equation that satisfy the left and right boundary
conditions, respectively.
y1 = x, y2 = x .
We compute the Wronskian of these functions.

x x
W (x) = = .
1 1
The Green function for this case is
x< (x> )
G(x|) = .

We consider the case 6= n2 , 6= 0. We find the solutions of the homogeneous equation that satisfy the left
and right boundary conditions, respectively.
   
y1 = sin x , y2 = sin (x ) .

We compute the Wronskian of these functions.


   
sin x sin (x )  

W (x) =    
= sin

cos x cos (x )

The Green function for this case is


   
sin x< sin (x> )
G(x|) =   .
sin

1155
2. Now we consider the problem
y 00 + 9y = 1 + x, y(0) = y() = 0.
The homogeneous solutions of the problem are constant multiples of sin(3x). Thus for each value of , the
problem either has no solution or an infinite number of solutions. There will be an infinite number of solutions if
the inhomogeneity 1 + x is orthogonal to the homogeneous solution sin(3x) and no solution otherwise.
Z
+ 2
(1 + x) sin(3x) dx =
0 3
The problem has a solution only for = 2/. For this case the general solution of the inhomogeneous differential
equation is  
1 2x
y= 1 + c1 cos(3x) + c2 sin(3x).
9
The one-parameter family of solutions that satisfies the boundary conditions is
 
1 2x
y= 1 cos(3x) + c sin(3x).
9

3. For = n2 , n Z+ , y = sin(nx) is a solution of the homogeneous equation that satisfies the boundary
conditions. Equation 21.5 has a (non-unique) solution only if f is orthogonal to sin(nx).
Z
f (x) sin(nx) dx = 0
0

The modified Green function satisfies


sin(nx) sin(n)
G00 + n2 G = (x ) .
/2
We expand G in a series of the eigenfunctions.

X
G(x|) = gk sin(kx)
k=1

1156
We substitute the expansion into the differential equation to determine the coefficients. This will not determine
gn . We choose gn = 0, which is one of the choices that will make the modified Green function symmetric in x
and .

X
2 2
 2X
gk n k sin(kx) = sin(kx) sin(k)
k=1
k=1
k6=n

2 X sin(kx) sin(k)
G(x|) =
k=1 n2 k 2
k6=n

The solution of the inhomogeneous problem is


Z
y(x) = f ()G(x|) d.
0

Solution 21.17
We separate the problem for u into the two problems:

Lv (pv 0 )0 + qv = f (x), a < x < b, v(a) = 0, v(b) = 0


Lw (pw0 )0 + qw = 0, a < x < b, w(a) = , w(b) =

and note that the solution for u is u = v + w.


The problem for v has the solution, Z b
v= g(x; )f () d,
a
with the Green function, (v
1 (x)v2 ()
v1 (x< )v2 (x> ) p()W ()
for a x ,
g(x; ) = v1 ()v2 (x)
p()W () p()W ()
for x b.
Here v1 and v2 are homogeneous solutions that respectively satisfy the left and right homogeneous boundary conditions.

1157
Since g(x; ) is a solution of the homogeneous equation for x 6= , g (x; ) is a solution of the homogeneous
equation for x 6= . This is because for x 6= ,
 

L g = L[g] = (x ) = 0.

If is outside of the domain, (a, b), then g(x; ) and g (x; ) are homogeneous solutions on that domain. In particular
g (x; a) and g (x; b) are homogeneous solutions,

L [g (x; a)] = L [g (x; b)] = 0.

Now we use the definition of the Green function and v1 (a) = v2 (b) = 0 to determine simple expressions for these
homogeneous solutions.

v10 (a)v2 (x) (p0 (a)W (a) + p(a)W 0 (a))v1 (a)v2 (x)
g (x; a) =
p(a)W (a) (p(a)W (a))2
0
v (a)v2 (x)
= 1
p(a)W (a)
v10 (a)v2 (x)
=
p(a)(v1 (a)v20 (a) v10 (a)v2 (a))
v10 (a)v2 (x)
=
p(a)v10 (a)v2 (a)
v2 (x)
=
p(a)v2 (a)

We note that this solution has the boundary values,

v2 (a) 1 v2 (b)
g (a; a) = = , g (b; a) = = 0.
p(a)v2 (a) p(a) p(a)v2 (a)

1158
We examine the second solution.
v1 (x)v20 (b) (p0 (b)W (b) + p(b)W 0 (b))v1 (x)v2 (b)
g (x; b) =
p(b)W (b) (p(b)W (b))2
v1 (x)v20 (b)
=
p(b)W (b)
v1 (x)v20 (b)
=
p(b)(v1 (b)v20 (b) v10 (b)v2 (b))
v1 (x)v20 (b)
=
p(b)v1 (b)v20 (b)
v1 (x)
=
p(b)v1 (b)
This solution has the boundary values,
v1 (a) v1 (b) 1
g (a; b) = = 0, g (b; b) = = .
p(b)v1 (b) p(b)v1 (b) p(b)
Thus we see that the solution of

Lw = (pw0 )0 + qw = 0, a < x < b, w(a) = , w(b) = ,

is
w = p(a)g (x; a) + p(b)g (x; b).
Therefore the solution of the problem for u is
Z b
u= g(x; )f () d p(a)g (x; a) + p(b)g (x; b).
a

Solution 21.18
Figure 21.7 shows a plot of G(x; 1) and G(x; 1) for k = 1.

1159
-4 -2 2 4

-0.1

-0.2

-0.3

-0.4

-0.5

Figure 21.7: G(x; 1) and G(x; 1)

First we consider the boundary condition u(0) = 0. Note that the solution of
G00 k 2 G = (x ) (x + ), |G(; )| < ,
satisfies the condition G(0; ) = 0. Thus the Green function which satisfies G(0; ) = 0 is
1 1
G(x; ) = ek|x| + ek|x+| .
2k 2k
Since x, > 0 we can write this as
1 1
G(x; ) = ek|x| + ek(x+)
( 2k 2k
1 k(x) 1 k(x+)
2k e + 2k e , for x <
= 1 k(x) 1 k(x+)
2k e + 2k e , for < x
(
k1 ek sinh(kx), for x <
=
k1 ekx sinh(k), for < x

1160
1
G(x; ) = ekx> sinh(kx< )
k
Now consider the boundary condition u0 (0) = 0. Note that the solution of
G00 k 2 G = (x ) + (x + ), |G(; )| < ,
satisfies the boundary condition G0 (x; ) = 0. Thus the Green function is
1 k|x| 1 k|x+|
G(x; ) = e e .
2k 2k
Since x, > 0 we can write this as
1 1
G(x; ) = ek|x| ek(x+)
( 2k 2k
1 k(x) 1 k(x+)
2k e 2k e , for x <
= 1 k(x) 1 k(x+)
2k e 2k e , for < x
(
k1 ek cosh(kx), for x <
=
k1 ekx cosh(k), for < x

1
G(x; ) = ekx> cosh(kx< )
k
The Green functions which satisfies G(0; ) = 0 and G0 (0; ) = 0 are shown in Figure 21.8.

Solution 21.19
1. The Green function satisfies
g 00 a2 g = (x ), g(0; ) = g 0 (L; ) = 0.
We can write the set of homogeneous solutions as
 ax ax
e ,e or {cosh(ax), sinh(ax)} .

1161
1 2 3 4 5 1 2 3 4 5
-0.1 -0.1
-0.2
-0.2 -0.3
-0.3 -0.4
-0.4 -0.5

Figure 21.8: G(x; 1) and G(x; 1)

The solutions that respectively satisfy the left and right boundary conditions are
u1 = sinh(ax), u2 = cosh(a(x L)).
The Wronskian of these solutions is
 
sinh(ax) cosh(a(x L))
W (x) = = a cosh(aL).
a cosh(ax) a sinh(a(x L))
Thus the Green function is
( sinh(ax) cosh(a(L))
a cosh(aL)
for x , sinh(ax< ) cosh(a(x> L))
g(x; ) = = .
sinh(a) cosh(a(xL))
a cosh(aL)
for x. a cosh(aL)

2. We take the limit as L .


sinh(ax< ) cosh(a(x> L))
g(x; ) = lim
L a cosh(aL)
sinh(ax< ) cosh(ax> ) cosh(aL) sinh(ax> ) sinh(aL)
= lim
L a cosh(aL)
sinh(ax< )
= (cosh(ax> ) sinh(ax> ))
a

1162
1
g(x; ) = sinh(ax< ) eax>
a
The solution of
y 00 a2 y = ex , y(0) = y 0 () = 0
is
Z
y= g(x; ) e d
0
1
Z
= sinh(ax< ) eax> e d
a 0
Z x Z 
1 ax a
= sinh(a) e e d + sinh(ax) e e d
a 0 x

We first consider the case that a 6= 1.


 ax 
1 e x
 1 (a+1)x
= a + e (a cosh(ax) + sinh(ax)) + e sinh(ax)
a a2 1 a+1
eax ex
=
a2 1
For a = 1, we have
 
1 2x
 1 2x
y= e x 1 + 2x + e + e sinh(x)
4 2
1
= x ex .
2
Thus the solution of the problem is
( ax x
e e
a2 1
for a 6= 1,
y= 1 x
2x e for a = 1.
We note that this solution satisfies the differential equation and the boundary conditions.

1163
21.13 Quiz
Problem 21.1
Find the general solution of
y 00 y = f (x),
where f (x) is a known function.
Solution

1164
21.14 Quiz Solutions
Solution 21.1

y 00 y = f (x)
We substitute y = ex into the homogeneous differential equation.

y 00 y = 0
2 ex ex = 0
= 1

The homogeneous solutions are ex and ex . The Wronskian of these solutions is


x x

e
x e x = 2.
e e

We find a particular solution with variation of parameters.


Z x Z x
x e f (x) x e f (x)
yp = e dx + e dx
2 2
The general solution is
ex f (x) ex f (x)
Z Z
x
x
y = c1 e +c2 e e x
dx + ex dx.
2 2

1165
Chapter 22

Difference Equations

Televisions should have a dial to turn up the intelligence. There is a brightness knob, but it doesnt work.

-?

22.1 Introduction
Example 22.1.1 Gamblers ruin problem. Consider a gambler that initially has n dollars. He plays a game in which
he has a probability p of winning a dollar and q of losing a dollar. (Note that p + q = 1.) The gambler has decided
that if he attains N dollars he will stop playing the game. In this case we will say that he has succeeded. Of course
if he runs out of money before that happens, we will say that he is ruined. What is the probability of the gamblers
ruin? Let us denote this probability by an . We know that if he has no money left, then his ruin is certain, so a0 = 1. If
he reaches N dollars he will quit the game, so that aN = 0. If he is somewhere in between ruin and success then the
probability of his ruin is equal to p times the probability of his ruin if he had n + 1 dollars plus q times the probability
of his ruin if he had n 1 dollars. Writing this in an equation,

an = pan+1 + qan1 subject to a0 = 1, aN = 0.

1166
This is an example of a difference equation. You will learn how to solve this particular problem in the section on constant
coefficient equations.

Consider the sequence a1 , a2 , a3 , . . . Analogous to a derivative of a continuous function, we can define a discrete
derivative on the sequence
Dan = an+1 an .
The second discrete derivative is then defined as

D2 an = D[an+1 an ] = an+2 2an+1 + an .

The discrete integral of an is


n
X
ai .
i=n0

Corresponding to Z
df
dx = f () f (),
dx
in the discrete realm we have
1 1
X X
D[an ] = (an+1 an ) = a a .
n= n=

Linear difference equations have the form

Dr an + pr1 (n)Dr1 an + + p1 (n)Dan + p0 (n)an = f (n).

From the definition of the discrete derivative an equivalent form is

an+r + qr1 (n)anr 1 + + q1 (n)an+1 + q0 (n)an = f (n).

Besides being important in their own right, we will need to solve difference equations in order to develop series
solutions of differential equations. Also, some methods of solving differential equations numerically are based on
approximating them with difference equations.

1167
There are many similarities between differential and difference equations. Like differential equations, an rth order
homogeneous difference equation has r linearly independent solutions. The general solution to the rth order inho-
mogeneous equation is the sum of the particular solution and an arbitrary linear combination of the homogeneous
solutions.
For an rth order difference equation, the initial condition is given by specifying the values of the first r an s.

Example 22.1.2 Consider the difference equation an2 an1 an = 0 subject to the initial condition a1 = a2 = 1.
Note that although we may not know a closed-form formula for the an we can calculate the an in order by substituting
into the difference equation. The first few an are 1, 1, 2, 3, 5, 8, 13, 21, . . . We recognize this as the Fibonacci sequence.

22.2 Exact Equations


Consider the sequence a1 , a2 , . . .. Exact difference equations on this sequence have the form

D[F (an , an+1 , . . . , n)] = g(n).

We can reduce the order of, (or solve for first order), this equation by summing from 1 to n 1.

n1
X n1
X
D[F (aj , aj+1 , . . . , j)] = g(j)
j=1 j=1
n1
X
F (an , an+1 , . . . , n) F (a1 , a2 , . . . , 1) = g(j)
j=1
n1
X
F (an , an+1 , . . . , n) = g(j) + F (a1 , a2 , . . . , 1)
j=1

1168
Result 22.2.1 We can reduce the order of the exact difference equation

D[F (an , an+1 , . . . , n)] = g(n), for n 1

by summing both sides of the equation to obtain


n1
X
F (an , an+1 , . . . , n) = g(j) + F (a1 , a2 , . . . , 1).
j=1

Example 22.2.1 Consider the difference equation, D[nan ] = 1. Summing both sides of this equation

n1
X n1
X
D[jaj ] = 1
j=1 j=1

nan a1 = n 1
n + a1 1
an = .
n

22.3 Homogeneous First Order

Consider the homogeneous first order difference equation

an+1 = p(n)an , for n 1.

1169
We can directly solve for an .
an1 an2 a1
an = an
an1 an2 a1
an an1 a2
= a1
an1 an2 a1
= a1 p(n 1)p(n 2) p(1)
n1
Y
= a1 p(j)
j=1

Alternatively, we could solve this equation by making it exact. Analogous to an integrating factor for differential
equations, we multiply the equation by the summing factor
" n #1
Y
S(n) = p(j) .
j=1

an+1 p(n)an = 0
an+1 an
Qn Qn1 =0
j=1 p(j) j=1 p(j)
" #
an
D Qn1 =0
j=1 p(j)

Now we sum from 1 to n 1.


an
Qn1 a1 = 0
j=1 p(j)
n1
Y
an = a1 p(j)
j=1

1170
Result 22.3.1 The solution of the homogeneous first order difference equation

an+1 = p(n)an , for n 1,

is
n1
Y
an = a1 p(j).
j=1

Example 22.3.1 Consider the equation an+1 = nan with the initial condition a1 = 1.
n1
Y
an = a1 j = (1)(n 1)! = (n)
j=1

Recall that (z) is the generalization of the factorial function. For positive integral values of the argument, (n) =
(n 1)!.

22.4 Inhomogeneous First Order


Consider the equation
an+1 = p(n)an + q(n) for n 1.
hQ i1
n
Multiplying by S(n) = j=1 p(j) yields
a a q(n)
Qn n+1 Qn1n = Qn .
j=1 p(j) j=1 p(j) j=1 p(j)
The left hand side is a discrete derivative.
" #
an q(n)
D Qn1 = Qn
j=1 p(j) j=1 p(j)

1171
Summing both sides from 1 to n 1,

n1
" #
an X q(k)
Qn1 a1 = Qk
j=1 p(j) k=1 j=1 p(j)

" n1 # " n1 " # #


Y X q(k)
an = p(m) Qk + a1 .
m=1 k=1 j=1 p(j)

Result 22.4.1 The solution of the inhomogeneous first order difference equation

an+1 = p(n)an + q(n) for n 1

is " n1 # " n1 " # #


Y X q(k)
an = p(m) Qk + a1 .
m=1 k=1 j=1 p(j)

Example 22.4.1 Consider the equation an+1 = nan + 1 for n 1. The summing factor is

" n
#1
Y 1
S(n) = j = .
j=1
n!

1172
Multiplying the difference equation by the summing factor,

an+1 an 1
=
n! (n 1)! n!
 
an 1
D =
(n 1)! n!
n1
an X 1
a1 =
(n 1)! k=1
k!
" n1 #
X 1
an = (n 1)! + a1 .
k=1
k!

Example 22.4.2 Consider the equation

an+1 = an + , for n 0.

From the above result, (with the products and sums starting at zero instead of one), the solution is
" n1 # " n1 " # #
Y X
a0 = Qk + a0
m=0 k=0 j=0
" n1 #
X h i
= n k+1
+ a0

 k=0n1
1

n
= + a0
1 1
 n 
n 1
= + a0
1
1 n
= + a0 n .
1

1173
22.5 Homogeneous Constant Coefficient Equations
Homogeneous constant coefficient equations have the form
an+N + pN 1 an+N 1 + + p1 an+1 + p0 an = 0.
The substitution an = rn yields
rN + pN 1 rN 1 + + p1 r + p0 = 0
(r r1 )m1 (r rk )mk = 0.
If r1 is a distinct root then the associated linearly independent solution is r1n . If r1 is a root of multiplicity m > 1
then the associated solutions are r1n , nr1n , n2 r1n , . . . , nm1 r1n .

Result 22.5.1 Consider the homogeneous constant coefficient difference equation

an+N + pN 1 an+N 1 + + p1 an+1 + p0 an = 0.

The substitution an = rn yields the equation

(r r1 )m1 (r rk )mk = 0.

A set of linearly independent solutions is

{r1n , nr1n , . . . , nm1 1 r1n , . . . , rkn , nrkn , . . . , nmk 1 rkn }.

Example 22.5.1 Consider the equation an+2 3an+1 + 2an = 0 with the initial conditions a1 = 1 and a2 = 3. The
substitution an = rn yields
r2 3r + 2 = (r 1)(r 2) = 0.
Thus the general solution is
an = c1 1n + c2 2n .

1174
The initial conditions give the two equations,

a1 = 1 = c1 + 2c2
a2 = 3 = c1 + 4c2

Since c1 = 1 and c2 = 1, the solution to the difference equation subject to the initial conditions is

an = 2n 1.

Example 22.5.2 Consider the gamblers ruin problem that was introduced in Example 22.1.1. The equation for the
probability of the gamblers ruin at n dollars is

an = pan+1 + qan1 subject to a0 = 1, aN = 0.

We assume that 0 < p < 1. With the substitution an = rn we obtain

r = pr2 + q.

The roots of this equation are



1 1 4pq
r=
2p
p
1 1 4p(1 p)
=
2p
p
1 (1 2p)2
=
2p
1 |1 2p|
= .
2p

We will consider the two cases p 6= 1/2 and p = 1/2.

1175
p 6= 1/2. If p < 1/2, the roots are
1 (1 2p)
r=
2p
1p q
r1 = = , r2 = 1.
p p
If p > 1/2 the roots are
1 (2p 1)
r=
2p
p + 1 q
r1 = 1, r2 = = .
p p
Thus the general solution for p 6= 1/2 is  n
q
an = c 1 + c 2 .
p
The boundary condition a0 = 1 requires that c1 + c2 = 1. From the boundary condition aN = 0 we have
 N
q
(1 c2 ) + c2 =0
p
1
c2 =
1 + (q/p)N
pN
c2 = N .
p qN
Solving for c1 ,
pN
c1 = 1
pN q N
q N
c1 = N .
p qN

1176
Thus we have  n
q N pN q
an = N + N .
p qN p qN p

p = 1/2. In this case, the two roots of the polynomial are both 1. The general solution is

an = c1 + c2 n.

The left boundary condition demands that c1 = 1. From the right boundary condition we obtain

1 + c2 N = 0
1
c2 = .
N
Thus the solution for this case is
n
an = 1 .
N
N/2
As a check that this formula makes sense, we see that for n = N/2 the probability of ruin is 1 N
= 12 .

22.6 Reduction of Order


Consider the difference equation

(n + 1)(n + 2)an+2 3(n + 1)an+1 + 2an = 0 for n 0 (22.1)

We see that one solution to this equation is an = 1/n!. Analogous to the reduction of order for differential equations,
the substitution an = bn /n! will reduce the order of the difference equation.
(n + 1)(n + 2)bn+2 3(n + 1)bn+1 2bn
+ =0
(n + 2)! (n + 1)! n!
bn+2 3bn+1 + 2bn = 0 (22.2)

1177
At first glance it appears that we have not reduced the order of the equation, but writing it in terms of discrete
derivatives
D2 bn Dbn = 0
shows that we now have a first order difference equation for Dbn . The substitution bn = rn in equation 22.2 yields the
algebraic equation
r2 3r + 2 = (r 1)(r 2) = 0.
Thus the solutions are bn = 1 and bn = 2n . Only the bn = 2n solution will give us another linearly independent solution
for an . Thus the second solution for an is an = bn /n! = 2n /n!. The general solution to equation 22.1 is then

1 2n
an = c 1 + c2 .
n! n!

Result 22.6.1 Let an = sn be a homogeneous solution of a linear difference equation. The


substitution an = sn bn will yield a difference equation for bn that is of order one less than the
equation for an .

1178
22.7 Exercises
Exercise 22.1
Find a formula for the nth term in the Fibonacci sequence 1, 1, 2, 3, 5, 8, 13, . . ..
Hint, Solution
Exercise 22.2
Solve the difference equation
2
an+2 = an , a1 = a2 = 1.
n
Hint, Solution

1179
22.8 Hints
Hint 22.1
The difference equation corresponding to the Fibonacci sequence is

an+2 an+1 an = 0, a1 = a2 = 1.
Hint 22.2
Consider this exercise as two first order difference equations; one for the even terms, one for the odd terms.

1180
22.9 Solutions
Solution 22.1
We can describe the Fibonacci sequence with the difference equation
an+2 an+1 an = 0, a1 = a2 = 1.

With the substitution an = rn we obtain the equation

r2 r 1 = 0.

This equation has the two distinct roots



1+ 5 1 5
r1 = , r2 = .
2 2
Thus the general solution is
!n !n
1+ 5 1 5
an = c 1 + c2 .
2 2
From the initial conditions we have
c1 r1 +c2 r2 = 1
c1 r12 +c2 r22 = 1.
Solving for c2 in the first equation,
1
c2 = (1 c1 r1 ).
r2
We substitute this into the second equation.
1
c1 r12 + (1 c1 r1 )r22 = 1
r2
c1 (r12 r1 r2 ) = 1 r2

1181
1 r2
c1 =
r12 r1 r2

1 5
1 2
=
1+ 5
2
5

1+ 5
2
=
1+ 5
2
5
1
=
5
Substitute this result into the equation for c2 .
 
1 1
c2 = 1 r1
r2 5
!
2 1 1+ 5
= 1
1 5 5 2
!
2 1 5
=
1 5 2 5
1
=
5

Thus the nth term in the Fibonacci sequence has the formula
!n !n
1 1+ 5 1 1 5
an = .
5 2 5 2

It is interesting to note that although the Fibonacci sequence is defined in terms of integers, one cannot express the
formula form the nth element in terms of rational numbers.

1182
Solution 22.2
We can consider
2
an+2 = an , a1 = a2 = 1
n
to be a first order difference equation. First consider the odd terms.

a1 = 1
2
a3 =
1
22
a5 =
31
2(n1)/2
an =
(n 2)(n 4) (1)

For the even terms,

a2 = 1
2
a4 =
2
22
a6 =
42
2(n2)/2
an = .
(n 2)(n 4) (2)

Thus (
2(n1)/2
(n2)(n4)(1)
for odd n
an = 2(n2)/2
(n2)(n4)(2)
for even n.

1183
Chapter 23

Series Solutions of Differential Equations

Skill beats honesty any day.


-?

23.1 Ordinary Points


Big O and Little o Notation. The notation O(z n ) means terms no bigger than z n . This gives us a convenient
shorthand for manipulating series. For example,
z3
sin z = z + O(z 5 )
6
1
= 1 + O(z)
1z
The notation o(z n ) means terms smaller that z n . For example,
cos z = 1 + o(1)
ez = 1 + z + o(z)

1184
Example 23.1.1 Consider the equation
w00 (z) 3w0 (z) + 2w(z) = 0.
The general solution to this constant coefficient equation is
w = c1 ez +c2 e2z .
The functions ez and e2z are analytic in the finite complex plane. Recall that a function is analytic at a point z0 if and
only if the function has a Taylor series about z0 with a nonzero radius of convergence. If we substitute the Taylor series
expansions about z = 0 of ez and e2z into the general solution, we obtain

X zn X 2n z n
w = c1 + c2 .
n=0
n! n=0
n!

Thus we have a series solution of the differential equation.


Alternatively, we
P could try substituting a Taylor series into the differential equation and solving for the coefficients.
Substituting w = n=0 an z n into the differential equation yields

d2 X n d X n
X
an z 3 an z + 2 an z n = 0
dz 2 n=0 dz n=0 n=0

X
X
X
n2 n1
n(n 1)an z 3 nan z +2 an z n = 0
n=2 n=1 n=0

X
X
X
(n + 2)(n + 1)an+2 z n 3 (n + 1)an+1 z n + 2 an z n = 0
n=0 n=0 n=0
h
X i
(n + 2)(n + 1)an+2 3(n + 1)an+1 + 2an z n = 0.
n=0

Equating powers of z, we obtain the difference equation


(n + 2)(n + 1)an+2 3(n + 1)an+1 + 2an = 0, n 0.

1185
We see that an = 1/n! is one solution since
(n + 2)(n + 1) n+1 1 13+2
3 +2 = = 0.
(n + 2)! (n + 1)! n! n!
We use reduction of order for difference equations to find the other solution. Substituting an = bn /n! into the difference
equation yields
bn+2 bn+1 bn
(n + 2)(n + 1) 3(n + 1) +2 =0
(n + 2)! (n + 1)! n!
bn+2 3bn+1 + 2bn = 0.

At first glance it appears that we have not reduced the order of the difference equation. However writing this equation
in terms of discrete derivatives,
D2 bn Dbn = 0
we see that this is a first order difference equation for Dbn . Since this is a constant coefficient difference equation we
substitute bn = rn into the equation to obtain an algebraic equation for r.

r2 3r + 2 = (r 1)(r 2) = 0

Thus the two solutions are bn = 1n b0 and bn = 2n b0 . Only bn = 2n b0 will give us a second independent solution for an .
Thus the two solutions for an are
a0 2n a0
an = and an = .
n! n!
Thus we can write the general solution to the differential equation as

X zn X 2n z n
w = c1 + c2 .
n=0
n! n=0
n!

We recognize these two sums as the Taylor expansions of ez and e2z . Thus we obtain the same result as we did solving
the differential equation directly.

1186
Of course it would be pretty silly to go through all the grunge involved in developing a series expansion of the solution
in a problem like Example 23.1.1 since we can solve the problem exactly. However if we could not solve a differential
equation, then having a Taylor series expansion of the solution about a point z0 would be useful in determining the
behavior of the solutions near that point.
For this method of substituting a Taylor series into the differential equation to be useful we have to know at what
points the solutions are analytic. Lets say we were considering a second order differential equation whose solutions
were
1
w1 = , and w2 = log z.
z
Trying to find a Taylor series expansion of the solutions about the point z = 0 would fail because the solutions are not
analytic at z = 0. This brings us to two important questions.
1. Can we tell if the solutions to a linear differential equation are analytic at a point without knowing the solutions?
2. If there are Taylor series expansions of the solutions to a differential equation, what are the radii of convergence
of the series?
In order to answer these questions, we will introduce the concept of an ordinary point. Consider the nth order linear
homogeneous equation
dn w dn1 w dw
n
+ p n1 (z) n1
+ + p1 (z) + p0 (z)w = 0.
dz dz dz
If each of the coefficient functions pi (z) are analytic at z = z0 then z0 is an ordinary point of the differential equation.
For reasons of typography we will restrict our attention to second order equations and the point z0 = 0 for a while.
The generalization to an nth order equation will be apparent. Considering the point z0 6= 0 is only trivially more general
as we could introduce the transformation z z0 z to move the point to the origin.
In the chapter on first order differential equations we showed that the solution is analytic at ordinary points. One
would guess that this remains true for higher order equations. Consider the second order equation
y 00 + p(z)y 0 + q(z)y = 0,
where p and q are analytic at the origin.

X
X
n
p(z) = pn z , and q(z) = qn z n
n=0 n=0

1187
Assume that one of the solutions is not analytic at the origin and behaves like z at z = 0 where 6= 0, 1, 2, . . .. That
is, we can approximate the solution with w(z) = z + o(z ). Lets substitute w = z + o(z ) into the differential
equation and look at the lowest power of z in each of the terms.

X X
( 1)z 2 + o(z 2 ) + z 1 + o(z 1 ) pn z n + z + o(z ) qn z n = 0.
   
n=0 n=0

We see that the solution could not possibly behave like z , 6= 0, 1, 2, because there is no term on the left to
cancel out the z 2 term. The terms on the left side could not add to zero.
You could also check that a solution could not possibly behave like log z at the origin. Though we will not prove
it, if z0 is an ordinary point of a homogeneous differential equation, then all the solutions are analytic at the point z0 .
Since the solution is analytic at z0 we can expand it in a Taylor series.

Now we are prepared to answer our second question. From complex variables, we know that the radius of convergence
of the Taylor series expansion of a function is the distance to the nearest singularity of that function. Since the solutions
to a differential equation are analytic at ordinary points of the equation, the series expansion about an ordinary point
will have a radius of convergence at least as large as the distance to the nearest singularity of the coefficient functions.

Example 23.1.2 Consider the equation


1
w00 + w0 + z 2 w = 0.
cos z
If we expand the solution to the differential equation in Taylor series about z = 0, the radius of convergence will be at
least /2. This is because the coefficient functions are analytic at the origin, and the nearest singularities of 1/ cos z
are at z = /2.

23.1.1 Taylor Series Expansion for a Second Order Differential Equation


Consider the differential equation
w00 + p(z)w0 + q(z)w = 0

1188
where p(z) and q(z) are analytic in some neighborhood of the origin.

X
X
n
p(z) = pn z and q(z) = qn z n
n=0 n=0

We substitute a Taylor series and its derivatives



X
w= an z n
n=0

X
X
0 n1
w = nzn z = (n + 1)an+1 z n
n=1 n=0

X
X
00 n2
w = n(n 1)an z = (n + 2)(n + 1)an+2 z n
n=2 n=0

into the differential equation to obtain



!
!
X X X
(n + 2)(n + 1)an+2 z n + pn z n (n + 1)an+1 z n
n=0 n=0 n=0

!
!
X X
+ qn z n an z n =0
n=0 n=0

n
! n
!
X X X X X
(n + 2)(n + 1)an+2 z n + (m + 1)am+1 pnm z n + am qnm z n = 0
n=0 n=0 m=0 n=0 m=0

" n
#
X X
z n = 0.

(n + 2)(n + 1)an+2 + (m + 1)am+1 pnm + am qnm
n=0 m=0

1189
Equating coefficients of powers of z,
n
X 
(n + 2)(n + 1)an+2 + (m + 1)am+1 pnm + am qnm = 0 for n 0.
m=0

We see that a0 and a1 are arbitrary and the rest of the coefficients are determined by the recurrence relation
n
1 X
an+2 = ((m + 1)am+1 pnm + am qnm ) for n 0.
(n + 1)(n + 2) m=0

Example 23.1.3 Consider the problem


1 0
y 00 + y + ex y = 0, y(0) = y 0 (0) = 1.
cos x
Lets expand the solution in a Taylor series about the origin.

X
y(x) = an x n
n=0

Since y(0) = a0 and y 0 (0) = a1 , we see that a0 = a1 = 1. The Taylor expansions of the coefficient functions are
1
= 1 + O(x), and ex = 1 + O(x).
cos x
Now we can calculate a2 from the recurrence relation.
0
1 X
a2 = ((m + 1)am+1 p0m + am q0m )
1 2 m=0
1
= (1 1 1 + 1 1)
2
= 1

1190
1.2

1.1

0.2 0.4 0.6 0.8 1 1.2 1.4


0.9

0.8

0.7

Figure 23.1: Plot of the Numerical Solution and the First Three Terms in the Taylor Series.

Thus the solution to the problem is


y(x) = 1 + x x2 + O(x3 ).

In Figure 23.1 the numerical solution is plotted in a solid line and the sum of the first three terms of the Taylor series
is plotted in a dashed line.

The general recurrence relation for the an s is useful if you only want to calculate the first few terms in the Taylor
expansion. However, for many problems substituting the Taylor series for the coefficient functions into the differential
equation will enable you to find a simpler form of the solution. We consider the following example to illustrate this
point.

1191
Example 23.1.4 Develop a series expansion of the solution to the initial value problem

1
w00 + w = 0, w(0) = 1, w0 (0) = 0.
(z 2 + 1)

Solution using the General Recurrence Relation. The coefficient function has the Taylor expansion

1 X
= (1)n z 2n .
1 + z2 n=0

From the initial condition we obtain a0 = 1 and a1 = 0. Thus we see that the solution is

X
w= an z n ,
n=0

where
n
1 X
an+2 = am qnm
(n + 1)(n + 2) m=0
and (
0 for odd n
qn =
(1)(n/2) for even n.
Although this formula is fine if you only want to calculate the first few an s, it is just a tad unwieldy to work with.
Lets see if we can get a better expression for the solution.

Substitute the Taylor Series into the Differential Equation. Substituting a Taylor series for w yields

d2 X n 1 X
an z + 2 an z n = 0.
dz 2 n=0 (z + 1) n=0

1192
Note that the algebra will be easier if we multiply by z 2 + 1. The polynomial z 2 + 1 has only two terms, but the Taylor
series for 1/(z 2 + 1) has an infinite number of terms.

d2 X X
(z 2 + 1) a n z n
+ an z n = 0
dz 2 n=0 n=0

X
X
X
n n2
n(n 1)an z + n(n 1)an z + an z n = 0
n=2 n=2 n=0

X
X
X
n n
n(n 1)an z + (n + 2)(n + 1)an+2 z + an z n = 0
n=0 n=0 n=0
h
X i
(n + 2)(n + 1)an+2 + n(n 1)an + an z n = 0
n=0

Equating powers of z gives us the difference equation


n2 n + 1
an+2 = an , for n 0.
(n + 2)(n + 1)
From the initial conditions we see that a0 = 1 and a1 = 0. All of the odd terms in the series will be zero. For the
even terms, it is easier to reformulate the problem with the change of variables bn = a2n . In terms of bn the difference
equation is
(2n)2 2n + 1
bn+1 = bn , b0 = 1.
(2n + 2)(2n + 1)
This is a first order difference equation with the solution
n 
4j 2 2j + 1
Y 
bn = .
j=0
(2j + 2)(2j + 1)

Thus we have that (Qn/2  


2
4j 2j+1
j=0 (2j+2)(2j+1) for even n,
an =
0 for odd n.

1193
Note that the nearest singularities of 1/(z 2 + 1) in the complex plane are at z = i. Thus the radius of convergence
must be at least 1. Applying the ratio test, the series converges for values of |z| such that
an+2 z n+2

lim <1
n an z n
2

n n + 1 2
lim |z| < 1
n (n + 2)(n + 1)
|z|2 < 1.

The radius of convergence is 1.


The first few terms in the Taylor expansion are

1 1 13 6
w = 1 z2 + z4 z + .
2 8 240
In Figure 23.2 the plot of the first two nonzero terms is shown in a short dashed line, the plot of the first four
nonzero terms is shown in a long dashed line, and the numerical solution is shown in a solid line.

In general, if the coefficient functions are rational functions, that is they are fractions of polynomials, multiplying
the equations by the quotient will reduce the algebra involved in finding the series solution.

Example 23.1.5 If we were going to find the Taylor series expansion about z = 0 of the solution to
z 1
w00 + w0 + w = 0,
1+z 1 z2
we would first want to multiply the equation by 1 z 2 to obtain

(1 z 2 )w00 + z(1 z)w00 + w = 0.

Example 23.1.6 Find the series expansions about z = 0 of the fundamental set of solutions for

w00 + z 2 w = 0.

1194
0.2 0.4 0.6 0.8 1 1.2
0.9

0.8

0.7

0.6

0.5

0.4

0.3

Figure 23.2: Plot of the solution and approximations.

Recall that the fundamental set of solutions {w1 , w2 } satisfy


w1 (0) = 1 w2 (0) = 0
w10 (0) = 0 w20 (0) = 1.
Thus if
X X
w1 = an z n and w2 = bn z n ,
n=0 n=0
then the coefficients must satisfy
a0 = 1, a1 = 0, and b0 = 0, b1 = 1.

1195
P n
Substituting the Taylor expansion w = n=0 cn z into the differential equation,

X
X
n2
n(n 1)cn z + cn z n+2 = 0
n=2 n=0

X
X
(n + 2)(n + 1)cn+2 z n + cn2 z n = 0
n=0 n=2
h
X i
2c2 + 6c3 z + (n + 2)(n + 1)cn+2 + cn2 z n = 0
n=2

Equating coefficients of powers of z,


z 0 : c2 = 0
z 1 : c3 = 0
z n : (n + 2)(n + 1)cn+2 + cn2 = 0, for n 2
cn
cn+4 =
(n + 4)(n + 3)
For our first solution we have the difference equation
an
a0 = 1, a1 = 0, a2 = 0, a3 = 0, an+4 = .
(n + 4)(n + 3)
For our second solution,
bn
b0 = 0, b1 = 1, b2 = 0, b3 = 0, bn+4 = .
(n + 4)(n + 3)
The first few terms in the fundamental set of solutions are
1 4 1 8 1 5 1 9
w1 = 1 z + z , w2 = z z + z .
12 672 20 1440
In Figure 23.3 the five term approximation is graphed in a coarse dashed line, the ten term approximation is graphed
in a fine dashed line, and the numerical solution of w1 is graphed in a solid line. The same is done for w2 .

1196
1.5 1.5

1 1

0.5 0.5

1 2 3 4 5 6 1 2 3 4 5 6

-0.5 -0.5

-1 -1

Figure 23.3: The graph of approximations and numerical solution of w1 and w2 .

Result 23.1.1 Consider the nth order linear homogeneous equation


dn w dn1 w dw
+ p n1 (z) + + p 1 (z) + p0 (z)w = 0.
dz n dz n1 dz
If each of the coefficient functions pi (z) are analytic at z = z0 then z0 is an ordinary point
of the differential equation. The solution is analytic in some region containing z0 and can
be expanded in a Taylor series. The radius of convergence of the series will be at least the
distance to the nearest singularity of the coefficient functions in the complex plane.

1197
23.2 Regular Singular Points of Second Order Equations
Consider the differential equation
p(z) 0 q(z)
w00 + w + w = 0.
z z0 (z z0 )2

If z = z0 is not an ordinary point but both p(z) and q(z) are analytic at z = z0 then z0 is a regular singular point of
the differential equation. The following equations have a regular singular point at z = 0.

w00 + z1 w0 + z 2 w = 0

w00 + 1
sin z
w0 w =0

w00 zw0 + 1
z sin z
w =0

Concerning regular singular points of second order linear equations there is good news and bad news.

The Good News. We will find that with the use of the Frobenius method we can always find series expansions of
two linearly independent solutions at a regular singular point. We will illustrate this theory with several examples.

The Bad News. Instead of a tidy little theory like we have for ordinary points, the solutions can be of several
different forms. Also, for some of the problems the algebra can get pretty ugly.

Example 23.2.1 Consider the equation


3(1 + z)
w00 + w = 0.
16z 2
P
We wish to find series solutions about the point z = 0. First we try a Taylor series w = n=0 an z n . Substituting this

1198
into the differential equation,

2
X
n2 3 X
z n(n 1)an z + (1 + z) an z n = 0
n=2
16 n=0

X 3 X 3 X
n(n 1)an z n + an z n + an+1 z n = 0.
n=0
16 n=0 16 n=1

Equating powers of z,

z0 : a0 = 0
 
3 3
zn : n(n 1) + an + an+1 = 0
16 16
 
16
an+1 = n(n 1) + 1 an .
3

This difference equation has the solution an = 0 for all n. Thus we have obtained only the trivial solution to the
differential equation. We must try an expansion of a more general form. We recall that for regular
P singular points of
n
first order equations we can always find a solution in the form of a Frobenius series w = z n=0 an z , a0 6= 0. We
substitute this series into the differential equation.

X 3 X
z2 ( 1) + 2n + n(n 1) an z n+2 + (1 + z)z an z n = 0
 
n=0
16 n=0

X  n 3 X n 3 X
an1 z n = 0

( 1) + 2n + n(n 1) an z + an z +
n=0
16 n=0
16 n=1

Equating the z 0 term to zero yields the equation


 
3
( 1) + a0 = 0.
16

1199
Since we have assumed that a0 6= 0, the polynomial in must be zero. The two roots of the polynomial are
p p
1 + 1 3/4 3 1 1 3/4 1
1 = = , 2 = = .
2 4 2 4
Thus our two series solutions will be of the form

X
X
w1 = z 3/4 an z n , w2 = z 1/4 bn z n .
n=0 n=0

Substituting the first series into the differential equation,


 
X 3 3 n 3 X
+ 2n + n(n 1) + an z + an1 z n = 0.
n=0
16 16 16 n=1

Equating powers of z, we see that a0 is arbitrary and


3
an = an1 for n 1.
16n(n + 1)
This difference equation has the solution
n  
Y 3
an = a0
j=1
16j(j + 1)
 n Yn
3 1
= a0
16 j=1 j(j + 1)
 n
3 1
= a0 for n 1.
16 n!(n + 1)!
Substituting the second series into the differential equation,
 
X 3 3 3 X
+ 2n + n(n 1) + bn z n + bn1 z n = 0.
n=0
16 16 16 n=1

1200
We see that the difference equation for bn is the same as the equation for an . Thus we can write the general solution
to the differential equation as
 n !  n !
X 3 1 X 3 1
w = c1 z 3/4 1 + z n + c2 z 1/4 1 + zn
n=1
16 n!(n + 1)! n=1
16 n!(n + 1)!
 n !
X 3 1
c1 z 3/4 + c2 z 1/4 1 + zn .


n=1
16 n!(n + 1)!

23.2.1 Indicial Equation


Now lets consider the general equation for a regular singular point at z = 0
p(z) 0 q(z)
w00 +w + 2 w = 0.
z z
Since p(z) and q(z) are analytic at z = 0 we can expand them in Taylor series.

X
X
n
p(z) = pn z , q(z) = qn z n
n=0 n=0

P n
Substituting a Frobenius series w = z n=0 an z , a0 6= 0 and the Taylor series for p(z) and q(z) into the differential
equation yields
h
! !
! !
X i X X X X
( + n)( + n 1) an z n + pn z n ( + n)an z n + qn z n an z n = 0
n=0 n=0 n=0 n=0 n=0

h
X i
( + n)2 ( + n) + p0 ( + n) + q0 an z n
n=0

!
!
!
!
X X X X
+ pn z n ( + n)an z n + qn z n an z n =0
n=1 n=0 n=1 n=0

1201
h n1
! n1
!
X i X X X X
( + n)2 + (p0 1)(n ) + q0 an z n + ( + j)aj pnj zn + aj qnj zn = 0
n=0 n=1 j=0 n=1 j=0

Equating powers of z,
h i
z0 : 2 + (p0 1) + q0 a0 = 0
h i n1 h
X i
n 2
z : ( + n) + (p0 1)( + n) + q0 an = ( + j)pnj + qnj aj .
j=0

Let
I() = 2 + (p0 1) + q0 = 0.
This is known as the indicial equation. The indicial equation gives us the form of the solutions. The equation for
a0 is I()a0 = 0. Since we assumed that a0 is nonzero, I() = 0. Let the two roots of I() be 1 and 2 where
<(1 ) <(2 ).
Rewriting the difference equation for an (),
n1 h
X i
I( + n)an () = ( + j)pnj + qnj aj () for n 1. (23.1)
j=0

If the roots are distinct and do not differ by an integer then we can use Equation 23.1 to solve for an (1 ) and
an (2 ), which will give us the two solutions

X
X
1 n 2
w1 = z an (1 )z , and w2 = z an (2 )z n .
n=0 n=0

If the roots are not distinct, 1 = 2 , we will only have one solution and will have to generate another. If the roots
differ by an integer, 1 2 = N , there is one solution corresponding to 1 , but when we try to solve Equation 23.1
for an (2 ), we will encounter the equation
N
X 1 h i
I(2 + N )aN (2 ) = I(1 )aN (2 ) = 0 aN (2 ) = ( + n)pnj + qnj aj (2 ).
j=0

1202
If the right side of the equation is nonzero, then aN (2 ) is undefined. On the other hand, if the right side is zero then
aN (2 ) is arbitrary. The rest of this section is devoted to considering the cases 1 = 2 and 1 2 = N .

23.2.2 The Case: Double Root


Consider a second order equation L[w] = 0 with a regular singular point at z = 0. Suppose the indicial equation has a
double root.
I() = ( 1 )2 = 0
One solution has the form
X
1
w1 = z an z n .
n=0
In order to find the second solution, we will differentiate with respect to the parameter, . Let an () satisfy Equa-
tion 23.1 Substituting the Frobenius expansion into the differential equation,
"
#
X
L z an ()z n = 0.
n=0

Setting = 1 will make the left hand side of the equation zero. Differentiating this equation with respect to ,
"
#
X
L z an ()z n = 0.
n=0

Interchanging the order of differentiation,


"
#
X X dan ()
L log z z an ()z n + z z n = 0.
n=0 n=0
d

Since setting = 1 will make the left hand side of this equation zero, the second linearly independent solution is


X X da n ()
w2 = log z z 1 an (1 )z n + z 1 zn

d

n=0 n=0 =1

1203

X
w2 = w1 log z + z 1
a0n (1 )z n .
n=0

Example 23.2.2 Consider the differential equation

1+z
w00 + w = 0.
4z 2

There is a regular singular point at z = 0. The indicial equation is


 2
1 1
( 1) + = = 0.
4 2

One solution will have the form



X
1/2
w1 = z an z n , a0 6= 0.
n=0

Substituting the Frobenius expansion



X

z an ()z n
n=0

into the differential equation yields

1
z 2 w00 + (1 + z)w = 0
4

X 1X 1X
( 1) + 2n + n(n 1) an ()z n+ + an ()z n+ + an ()z n++1 = 0.
 
n=0
4 n=0
4 n=0

1204
Divide by z and adjust the summation indices.

X 1X n n 1X
[( 1) + 2n + n(n 1)] an ()z + an ()z + an1 ()z n = 0
n=0
4 n=0 4 n=1
    
1 X 1 1
( 1)a0 + a0 + ( 1) + 2n + n(n 1) + an () + an1 () z n = 0
4 n=1
4 4

Equating the coefficient of z 0 to zero yields I()a0 = 0. Equating the coefficients of z n to zero yields the difference
equation
 
1 1
( 1) + 2n + n(n 1) + an () + an1 () = 0
4 4
 
n(n + 1) ( 1) 1
an () = + + an1 ().
4 4 16
The first few an s are
    
9 25 9
a0 , ( 1) + a0 , ( 1) + ( 1) + a0 , . . .
16 16 16
Setting = 1/2, the coefficients for the first solution are
5 105
a0 , a0 , a0 , ...
16 16

The second solution has the form


X
w2 = w1 log z + z 1/2
a0n (1/2)z n .
n=0

Differentiating the an (),


   
da0 da1 () da2 () 9 25
= 0, = (2 1)a0 , = (2 1) ( 1) + + ( 1) + a0 , ...
d d d 16 16

1205
Setting = 1/2 in this equation yields

a00 = 0, a01 (1/2) = 0, a02 (1/2) = 0, ...

Thus the second solution is


w2 = w1 log z.
The first few terms in the general solution are
 
5 105 2
(c1 + c2 log z) 1 z + z .
16 16

23.2.3 The Case: Roots Differ by an Integer


Consider the case in which the roots of the indicial equation 1 and 2 differ by an integer. (1 2 = N ) Recall the
equation that determines an ()

h i n1 h
X i
2
I( + n)an = ( + n) + (p0 1)( + n) + q0 an = ( + j)pnj + qnj aj .
j=0

When = 2 the equation for aN is


N
X 1 h i
I(2 + N )aN (2 ) = 0 aN (2 ) = ( + j)pN j + qN j aj .
j=0

If the right hand side of this equation is zero, then aN is arbitrary. There will be two solutions of the Frobenius form.

X
X
1 n 2
w1 = z an (1 )z and w2 = z an (2 )z n .
n=0 n=0

1206
If the right hand side of the equation is nonzero then aN (2 ) will be undefined. We will have to generate the second
solution. Let
X

w(z, ) = z an ()z n ,
n=0
where an () satisfies the recurrence formula. Substituting this series into the differential equation yields
L[w(z, )] = 0.
We will multiply by ( 2 ), differentiate this equation with respect to and then set = 2 . This will generate a
linearly independent solution.
 

L[( 2 )w(z, )] = L ( 2 )w(z, )

"
#
X
=L ( 2 )z an ()z n
n=0
"
#
X X d
= L log z z ( 2 )an ()z n + z [( 2 )an ()]z n
n=0 n=0
d
Setting = 2 with make this expression zero, thus
 

X
n 2
X d
log z z lim {( 2 )an ()} z + z lim [( 2 )an ()] z n
n=0
2
n=0
2 d
is a solution. Now lets look at the first term in this solution

X
log z z lim {( 2 )an ()} z n .
2
n=0

The first N terms in the sum will be zero. That is because a0 , . . . , aN 1 are finite, so multiplying by ( 2 ) and
taking the limit as 2 will make the coefficients vanish. The equation for aN () is
N
X 1 h i
I( + N )aN () = ( + j)pN j + qN j aj ().
j=0

1207
Thus the coefficient of the N th term is
" N 1
#
( 2 ) X h i
lim ( 2 )aN () = lim ( + j)pN j + qN j aj ()
2 2 I( + N ) j=0
" N 1 h
#
( 2 ) X i
= lim ( + j)pN j + qN j aj ()
2 ( + N 1 )( + N 2 ) j=0

2
Since 1 = 2 + N , lim2 +N 1
= 1.

N 1 h
1 X i
= (2 + j)pN j + qN j aj (2 ).
(1 2 ) j=0

Using this you can show that the first term in the solution can be written

d1 log z w1 ,

where d1 is a constant. Thus the second linearly independent solution is


X
w2 = d1 log z w1 + z 2 dn z n ,
n=0

where
N 1 h
1 1 X i
d1 = (2 + j)pN j + qN j aj (2 )
a0 (1 2 ) j=0

and  i
d h
dn = lim ( 2 )an () for n 0.
2 d

1208
Example 23.2.3 Consider the differential equation
 
00 2 2
w + 1 w0 + 2 w = 0.
z z
The point z = 0 is a regular singular point. In order to find series expansions of the solutions, we first calculate the
indicial equation. We can write the coefficient functions in the form
p(z) 1 q(z) 1
= (2 + z), and 2
= 2 (2).
z z z z
Thus the indicial equation is
2 + (2 1) + 2 = 0
( 1)( 2) = 0.

The First Solution. The first solution will have the Frobenius form

X
2
w1 = z an (1 )z n .
n=0

Substituting a Frobenius series into the differential equation,


z 2 w00 + (z 2 2z)w0 + 2w = 0

X X
X
(n + )(n + 1)z n+ + (z 2 2z) (n + )z n+1 + 2 an z n = 0
n=0 n=0 n=0
h
X i
[2 3 + 2]a0 + (n + )(n + 1)an + (n + 1)an1 2(n + )an + 2an z n = 0.
n=1

Equating powers of z,
h i
(n + )(n + 1) 2(n + ) + 2 an = (n + 1)an1
an1
an = .
n+2

1209
Setting = 1 = 2, the recurrence relation becomes

an1 (1 )
an (1 ) =
n
(1)n
= a0 .
n!

The first solution is



X (1)n
w1 = a0 z n = a0 ez .
n=0
n!

The Second Solution. The equation for a1 (2 ) is

0 a1 (2 ) = 2a0 .

Since the right hand side of this equation is not zero, the second solution will have the form
 
2
X d
w2 = d1 log z w1 + z lim [( 2 )an ()] z n
n=0
2 d

First we will calculate d1 as we defined it previously.

1 1
d1 = a0 = 1.
a0 2 1

The expression for an () is


(1)n a0
an () = .
( + n 2)( + n 1) ( 1)

1210
The first few an () are

a0
a1 () =
1
a0
a2 () =
( 1)
a0
a3 () = .
( + 1)( 1)

We would like to calculate

 i
d h
dn = lim ( 1)an () .
1 d

1211
The first few dn are

 
d h i
d0 = lim ( 1)a0
1 d
= a0
   
d a0
d1 = lim ( 1)
1 d 1
 h 
d i
= lim a0
1 d
=0
   
d a0
d2 = lim ( 1)
1 d ( 1)
 
d a0
h i
= lim
1 d
= a0
   
d a0
d3 = lim ( 1)
1 d ( + 1)( 1)
  
d a0
= lim
1 d ( + 1)
3
= a0 .
4

It will take a little work to find the general expression for dn . We will need the following relations.

n1
0
X 1
(n) = (n 1)!, (z) = (z)(z), (n) = + .
k=1
k

1212
See the chapter on the Gamma function for explanations of these equations.

(1)n a0
  
d
dn = lim ( 1)
1 d ( + n 2)( + n 1) ( 1)
(1)n a0
  
d
= lim
1 d ( + n 2)( + n 1) ()
d (1)n a0 ()
  
= lim
1 d ( + n 1)
 
n ()() ()( + n 1)
= (1) a0 lim
1 ( + n 1) ( + n 1)
 
n ()[() ( + n 1)]
= (1) a0 lim
1 ( + n 1)
(1) (n)
= (1)n a0
(n 1)!
n1
(1)n+1 a0 X 1
=
(n 1)! k=0 k

Thus the second solution is


n1
!
X (1)n+1 a0 X 1
w2 = log z w1 + z zn.
n=0
(n 1)! k=0 k

The general solution is


n1
!
X (1)n+1 X 1
w = c1 ez c2 log z ez +c2 z zn.
n=0
(n 1)! k=0 k

We see that even in problems that are chosen for their simplicity, the algebra involved in the Frobenius method can
be pretty involved.

1213
Example 23.2.4 Consider a series expansion about the origin of the equation
1z 0 1
w00 + w 2 w = 0.
z z
The indicial equation is

2 1 = 0
= 1.

Substituting a Frobenius series into the differential equation,



X
X
X
2 n2 2 n1
z (n + )(n + 1)an z + (z z ) (n + )an z an z n = 0
n=0 n=0 n=0

X
X
X
X
(n + )(n + 1)an z n + (n + )an z n (n + 1)an1 z n an z n = 0
n=0 n=0 n=1 n=0
h i h
X i
( 1) + 1 a0 + n + )(n + 1)an + (n + 1)an (n + 1)an1 z n = 0.
n=1

Equating powers of z to zero,


an1 ()
an () = .
n++1
We know that the first solution has the form

X
w1 = z an z n .
n=0

Setting = 1 in the reccurence formula,


an1 2a0
an = = .
n+2 (n + 2)!

1214
Thus the first solution is

X 2a0
w1 = z zn
n=0
(n + 2)!

1 X z n+2
= 2a0
z n=0 (n + 2)!

!
2a0 X zn
= 1z
z n=0
n!
2a0 z
= (e 1 z).
z

Now to find the second solution. Setting = 1 in the reccurence formula,


an1 a0
an = = .
n n!

We see that in this case there is no trouble in defining a2 (2 ). The second solution is

a0 X z n a0 z
w2 = = e .
z n=0 n! z

Thus we see that the general solution is


c1 z c2
w= (e 1 z) + ez
z z
 
d1 z 1
w= e +d2 1 + .
z z

1215
23.3 Irregular Singular Points
If a point z0 of a differential equation is not ordinary or regular singular, then it is an irregular singular point. At least
one of the solutions at an irregular singular point will not be of the Frobenius form. We will examine how to obtain
series expansions about an irregular singular point in the chapter on asymptotic expansions.

23.4 The Point at Infinity


If we want to determine the behavior of a function f (z) at infinity, we can make the transformation = 1/z and
examine the point = 0.

Example 23.4.1 Consider the behavior of f (z) = sin z at infinity. This is the same as considering the point = 0 of
sin(1/), which has the series expansion


(1)n
  X
1
sin = .
n=0
(2n + 1)! 2n+1

Thus we see that the point = 0 is an essential singularity of sin(1/). Hence sin z has an essential singularity at
z = .

Example 23.4.2 Consider the behavior at infinity of z e1/z . We make the transformation = 1/z.


1 1 X n
e =
n=0 n!

Thus z e1/z has a pole of order 1 at infinity.

1216
In order to classify the point at infinity of a differential equation in w(z), we apply the transformation = 1/z,
u() = w(z). We write the derivatives with respect to z in terms of .
1
z=

1
dz = 2 d

d d
= 2
dz d
d2
 
2 d 2 d
=
dz 2 d d
2
d d
= 4 2 + 2 3
d d
Now we apply the transformation to the differential equation.
w00 + p(z)w0 + q(z)w = 0
4 u00 + 2 3 u0 + p(1/)( 2 )u0 + q(1/)u = 0
 
00 2 p(1/) q(1/)
u + 2
u0 + u=0
4
Example 23.4.3 Classify the singular points of the differential equation
1
w00 + w0 + 2w = 0.
z
There is a regular singular point at z = 0. To examine the point at infinity we make the transformation = 1/z,
u() = w(z).
 
00 2 1 2
u + u0 + 4 u = 0

1 2
u00 + u0 + 4 u = 0

1217
Thus we see that the differential equation for w(z) has an irregular singular point at infinity.

1218
23.5 Exercises
Exercise 23.1 (mathematica/ode/series/series.nb)
f (x) satisfies the Hermite equation
d2 f df
2
2x + 2f = 0.
dx dx
Construct two linearly independent solutions of the equation as Taylor series about x = 0. For what values of x do the
series converge?
Show that for certain values of , called eigenvalues, one of the solutions is a polynomial, called an eigenfunction.
Calculate the first four eigenfunctions H0 (x), H1 (x), H2 (x), H3 (x), ordered by degree.
Hint, Solution
Exercise 23.2
Consider the Legendre equation
(1 x2 )y 00 2xy 0 + ( + 1)y = 0.
1. Find two linearly independent solutions in the form of power series about x = 0.
2. Compute the radius of convergence of the series. Explain why it is possible to predict the radius of convergence
without actually deriving the series.
3. Show that if = 2n, with n an integer and n 0, the series for one of the solutions reduces to an even
polynomial of degree 2n.
4. Show that if = 2n + 1, with n an integer and n 0, the series for one of the solutions reduces to an odd
polynomial of degree 2n + 1.
5. Show that the first 4 polynomial solutions Pn (x) (known as Legendre polynomials) ordered by their degree and
normalized so that Pn (1) = 1 are
P0 = 1 P1 = x
1 1
P2 = (3x2 1) P4 = (5x3 3x)
2 2

1219
6. Show that the Legendre equation can also be written as

((1 x2 )y 0 )0 = ( + 1)y.

Note that two Legendre polynomials Pn (x) and Pm (x) must satisfy this relation for = n and = m respectively.
By multiplying the first relation by Pm (x) and the second by Pn (x) and integrating by parts show that Legendre
polynomials satisfy the orthogonality relation
Z 1
Pn (x)Pm (x) dx = 0 if n 6= m.
1

If n = m, it can be shown that the value of the integral is 2/(2n + 1). Verify this for the first three polynomials
(but you neednt prove it in general).

Hint, Solution
Exercise 23.3
Find the forms of two linearly independent series expansions about the point z = 0 for the differential equation

1 1z
w00 + w0 + 2 w = 0,
sin z z
such that the series are real-valued on the positive real axis. Do not calculate the coefficients in the expansions.
Hint, Solution
Exercise 23.4
Classify the singular points of the equation
w0
w00 + + 2w = 0.
z1
Hint, Solution

1220
Exercise 23.5
Find the series expansions about z = 0 for

5 0 z1
w00 + w + w = 0.
4z 8z 2

Hint, Solution
Exercise 23.6
Find the series expansions about z = 0 of the fundamental solutions of

w00 + zw0 + w = 0.

Hint, Solution
Exercise 23.7
Find the series expansions about z = 0 of the two linearly independent solutions of

1 0 1
w00 + w + w = 0.
2z z

Hint, Solution
Exercise 23.8
Classify the singularity at infinity of the differential equation
 
00 2 3 1
w + + 2 w0 + w = 0.
z z z2

Find the forms of the series solutions of the differential equation about infinity that are real-valued when z is real-valued
and positive. Do not calculate the coefficients in the expansions.
Hint, Solution

1221
Exercise 23.9
Consider the second order differential equation

d2 y dy
x 2
+ (b x) ay = 0,
dx dx
where a, b are real constants.

1. Show that x = 0 is a regular singular point. Determine the location of any additional singular points and classify
them. Include the point at infinity.

2. Compute the indicial equation for the point x = 0.

3. By solving an appropriate recursion relation, show that one solution has the form

ax (a)2 x2 (a)n xn
y1 (x) = 1 + + + + +
b (b)2 2! (b)n n!

where the notation (a)n is defined by

(a)n = a(a + 1)(a + 2) (a + n 1), (a)0 = 1.

Assume throughout this problem that b 6= n where n is a non-negative integer.

4. Show that when a = m, where m is a non-negative integer, that there are polynomial solutions to this equation.
Compute the radius of convergence of the series above when a 6= m. Verify that the result you get is in accord
with the Frobenius theory.

5. Show that if b = n + 1 where n = 0, 1, 2, . . ., then the second solution of this equation has logarithmic terms.
Indicate the form of the second solution in this case. You need not compute any coefficients.

Hint, Solution

1222
Exercise 23.10
Consider the equation
xy 00 + 2xy 0 + 6 ex y = 0.
Find the first three non-zero terms in each of two linearly independent series solutions about x = 0.
Hint, Solution

1223
23.6 Hints
Hint 23.1

Hint 23.2

Hint 23.3

Hint 23.4

Hint 23.5

Hint 23.6

Hint 23.7

Hint 23.8

Hint 23.9

Hint 23.10

1224
23.7 Solutions
Solution 23.1
f (x) is a Taylor series about x = 0.

X
f (x) = an x n
n=0

X
f 0 (x) = nan xn1
n=1

X
= nan xn1
n=0
X
f 00 (x) = n(n 1)an xn2
n=2

X
= (n + 2)(n + 1)an+2 xn
n=0

We substitute the Taylor series into the differential equation.

f 00 (x) 2xf 0 (x) + 2f = 0



X X
X
n n
(n + 2)(n + 1)an+2 x 2 nan x + 2 an x n
n=0 n=0 n=0

Equating coefficients gives us a difference equation for an :

(n + 2)(n + 1)an+2 2nan + 2an = 0


n
an+2 = 2 an .
(n + 1)(n + 2)

1225
The first two coefficients, a0 and a1 are arbitrary. The remaining coefficients are determined by the recurrence relation.
We will find the fundamental set of solutions at x = 0. That is, for the first solution we choose a0 = 1 and a1 = 0; for
the second solution we choose a0 = 0, a1 = 1. The difference equation for y1 is
n
an+2 = 2 an , a0 = 1, a1 = 0,
(n + 1)(n + 2)
which has the solution Qn
2n k=0 (2(n k) )
a2n = , a2n+1 = 0.
(2n)!
The difference equation for y2 is
n
an+2 = 2 an , a0 = 0, a1 = 1,
(n + 1)(n + 2)
which has the solution Qn1
2n k) 1 )
k=0 (2(n
a2n = 0, a2n+1 = .
(2n + 1)!
A set of linearly independent solutions, (in fact the fundamental set of solutions at x = 0), is

2n nk=0 (2(n k) ) 2n 2n n1
Q Q
k=0 (2(n k) 1 ) 2n+1
X X
y1 (x) = x , y2 (x) = x .
n=0
(2n)! n=0
(2n + 1)!

Since the coefficient functions in the differential equation do not have any singularities in the finite complex plane, the
radius of convergence of the series is infinite.
If = n is a positive even integer, then the first solution, y1 , is a polynomial of order n. If = n is a positive odd
integer, then the second solution, y2 , is a polynomial of order n. For = 0, 1, 2, 3, we have
H0 (x) = 1
H1 (x) = x
H2 (x) = 1 2x2
2
H3 (x) = x x3
3

1226
Solution 23.2
1. First we write the differential equation in the standard form.

1 x2 y 00 2xy 0 + ( + 1)y = 0

(23.2)

2x 0 ( + 1)
y 00 y + y = 0. (23.3)
1 x2 1 x2
Since the coefficients of y 0 and y are analytic in a neighborhood of x = 0, We can find two Taylor series solutions
about that point. We find the Taylor series for y and its derivatives.

X
y= an x n
n=0

X
y0 = nan xn1
n=1
X
y 00 = (n 1)nan xn2
n=2

X
= (n + 1)(n + 2)an+2 xn
n=0

Here we used index shifting to explicitly write the two forms that we will need for y 00 . Note that we can take the
lower bound of summation to be n = 0 for all above sums. The terms added by this operation are zero. We
substitute the Taylor series into Equation 23.2.

X
X
X
X
n n n
(n + 1)(n + 2)an+2 x (n 1)nan x 2 nan x + ( + 1) an x n = 0
n=0 n=0 n=0 n=0

X  
(n + 1)(n + 2)an+2 (n 1)n + 2n ( + 1) an xn = 0
n=0

1227
We equate coefficients of xn to obtain a recurrence relation.

(n + 1)(n + 2)an+2 = (n(n + 1) ( + 1))an


n(n + 1) ( + 1)
an+2 = an , n 0
(n + 1)(n + 2)

We can solve this difference equation to determine the an s. (a0 and a1 are arbitrary.)

n2
a0 Y 
k(k + 1) ( + 1) , even n,


n! k=0



even k
an = n2
a1 Y 
k(k + 1) ( + 1) , odd n



n! k=1


odd k

We will find the fundamental set of solutions at x = 0, that is the set {y1 , y2 } that satisfies

y1 (0) = 1 y10 (0) = 0


y2 (0) = 0 y20 (0) = 1.

For y1 we take a0 = 1 and a1 = 0; for y2 we take a0 = 0 and a1 = 1. The rest of the coefficients are determined
from the recurrence relation.

n2
X 1 Y
k(k + 1) ( + 1) xn

y1 =
n! k=0

n=0
even n even k

n2
X 1 Y
k(k + 1) ( + 1) xn

y2 =
n!

n=1 k=1
odd n odd k

1228
2. We determine the radius of convergence of the series solutions with the ratio test.
an+2 xn+2

lim <1
n an x n
n(n+1)(+1)
n+2

(n+1)(n+2) a n x
lim <1

n an x n

n(n + 1) ( + 1) 2
lim x < 1
n (n + 1)(n + 2)
2
x < 1

Thus we see that the radius of convergence of the series is 1. We knew that the radius of convergence would be
at least one, because the nearest singularities of the coefficients of (23.3) occur at x = 1, a distance of 1 from
the origin. This implies that the solutions of the equation are analytic in the unit circle about x = 0. The radius
of convergence of the Taylor series expansion of an analytic function is the distance to the nearest singularity.

3. If = 2n then a2n+2 = 0 in our first solution. From the recurrence relation, we see that all subsequent coefficients
are also zero. The solution becomes an even polynomial.

2n m2
X 1 Y
k(k + 1) ( + 1) xm

y1 =
m!

m=0 k=0
even m even k

4. If = 2n + 1 then a2n+3 = 0 in our second solution. From the recurrence relation, we see that all subsequent
coefficients are also zero. The solution becomes an odd polynomial.

2n+1 m2
X 1 Y
k(k + 1) ( + 1) xm

y2 =
m!

m=1 k=1
odd m odd k

1229
Figure 23.4: The First Four Legendre Polynomials

5. From our solutions above, the first four polynomials are

1
x
1 3x2
5
x x3
3

To obtain the Legendre polynomials we normalize these to have value unity at x = 1

P0 = 1
P1 = x
1
3x2 1

P2 =
2
1
5x3 3x

P3 =
2

These four Legendre polynomials are plotted in Figure 23.4.

1230
6. We note that the first two terms in the Legendre equation form an exact derivative. Thus the Legendre equation
can also be written as 0
(1 x2 )y 0 = ( + 1)y.
Pn and Pm are solutions of the Legendre equation.
0 0
(1 x2 )Pn0 = n(n + 1)Pn , (1 x2 )Pm0 = m(m + 1)Pm (23.4)

We multiply the first relation of Equation 23.4 by Pm and integrate by parts.


0
(1 x2 )Pn0 Pm = n(n + 1)Pn Pm
Z 1 Z 1
2 0 0

(1 x )Pn Pm dx = n(n + 1) Pn Pm dx
1 1
Z 1 Z 1
 2 0
 1 2 0 0
(1 x )Pn Pm 1 (1 x )Pn Pm dx = n(n + 1) Pn Pm dx
1 1
Z 1 Z 1
2 0 0
(1 x )Pn Pm dx = n(n + 1) Pn Pm dx
1 1

We Rmultiply the secord relation of Equation 23.4 by Pn and integrate by parts. To obtain a different expression
1
for 1 (1 x2 )Pm0 Pn0 dx.
Z 1 Z 1
2 0 0
(1 x )Pm Pn dx = m(m + 1) Pm Pn dx
1 1
R1
We equate the two expressions for 1
(1 x2 )Pm0 Pn0 dx. to obtain an orthogonality relation.
Z 1
(n(n + 1) m(m + 1)) Pn Pm dx = 0
1
Z 1
Pn (x)Pm (x) dx = 0 if n 6= m.
1

1231
We verify that for the first four polynomials the value of the integral is 2/(2n + 1) for n = m.
Z 1 Z 1
P0 (x)P0 (x) dx = 1 dx = 2
1 1
1 1 1
x3
Z Z 
2 2
P1 (x)P1 (x) dx = x dx = =
1 1 3 1 3
Z 1 Z 1   5 1
1 4 2
 1 9x 3 2
P2 (x)P2 (x) dx = 9x 6x + 1 dx = 2x + x =
1 1 4 4 5 1 5
Z 1 Z 1 1
1 25x7
  
1 2
25x6 30x4 + 9x2 dx = 6x5 + 3x3

P3 (x)P3 (x) dx = =
1 1 4 4 7 1 7
Solution 23.3
The indicial equation for this problem is
2 + 1 = 0.
Since the two roots 1 = i and 2 = i are distinct and do not differ by an integer, there are two solutions in the
Frobenius form.
X X
i n i
w1 = z an z , w1 = z bn z n
n=0 n=0
However, these series are not real-valued on the positive real axis. Recalling that
z i = ei log z = cos(log z) + i sin(log z), and z i = ei log z = cos(log z) i sin(log z),
we can write a new set of solutions that are real-valued on the positive real axis as linear combinations of w1 and w2 .
1 1
u1 = (w1 + w2 ), u2 = (w1 w2 )
2 2i

X
X
u1 = cos(log z) cn z n , u1 = sin(log z) dn z n
n=0 n=0

1232
Solution 23.4
Consider the equation w00 + w0 /(z 1) + 2w = 0.
We see that there is a regular singular point at z = 1. All other finite values of z are ordinary points of the equation.
To examine the point at infinity we introduce the transformation z = 1/t, w(z) = u(t). Writing the derivatives with
respect to z in terms of t yields
d d d2 4 d
2
d
= t2 , 2
= t 2
+ 2t3 .
dz dt dz dt dt
Substituting into the differential equation gives us
t2 u0
t4 u00 + 2t3 u0 + 2u = 0
1/t 1
 
00 2 1 2
u + u0 + 4 u = 0.
t t(1 t) t
Since t = 0 is an irregular singular point in the equation for u(t), z = is an irregular singular point in the equation
for w(z).

Solution 23.5
Find the series expansions about z = 0 for
5 0 z1
w00 +w + w = 0.
4z 8z 2
We see that z = 0 is a regular singular point of the equation. The indicial equation is
1 1
2 + = 0
 4 8 
1 1
+ = 0.
2 4
Since the roots are distinct and do not differ by an integer, there will be two solutions in the Frobenius form.

X
X
1/4 n 1/2
w1 = z an (1 )z , w2 = z an (2 )z n
n=0 n=0

1233
We multiply the differential equation by 8z 2 to put it in a better form. Substituting a Frobenius series into the
differential equation,

X
X
X
8z 2 (n + )(n + 1)an z n+2 + 10z (n + )an z n+1 + (z 1) an z n+
n=0 n=0 n=0

X
X
X
X
8 (n + )(n + 1)an z n + 10 (n + )an z n + an1 z n an z n .
n=0 n=0 n=1 n=0

Equating coefficients of powers of z,


[8(n + )(n + 1) + 10(n + ) 1] an = an1
an1
an = 2
.
8(n + ) + 2(n + ) 1

The First Solution. Setting = 1/4 in the recurrence formula,


an1
an (1 ) = 2
8(n + 1/4) + 2(n + 1/4) 1
an1
an (1 ) = .
2n(4n + 3)
Thus the first solution is
 
1/4
X
n 1/4 1 1 2
w1 = z an (1 )z = a0 z 1 z+ z + .
n=0
14 616

The Second Solution. Setting = 1/2 in the recurrence formula,


an1
an = 2
8(n 1/2) + 2(n 1/2) 1
an1
an =
2n(4n 3)

1234
Thus the second linearly independent solution is
 
1/2
X
n 1/2 1 1
w2 = z an (2 )z = a0 z 1 z + z2 + .
n=0
2 40

Solution 23.6
We consider the series solutions of,
w00 + zw0 + w = 0.
We would like to find the expansions of the fundamental set of solutions about z = 0. Since z = 0 is a regular
point, (the coefficient functions are analytic there), we expand the solutions in Taylor series. Differentiating the series
expansions for w(z),

X
w= an z n
n=0
X
w0 = nan z n1
n=1

X
w00 = n(n 1)an z n2
n=2

X
= (n + 2)(n + 1)an+2 z n
n=0

We may take the lower limit of summation to be zero without changing the sums. Substituting these expressions into
the differential equation,

X
X
X
n n
(n + 2)(n + 1)an+2 z + nan z + an z n = 0
n=0 n=0 n=0

X
(n + 2)(n + 1)an+2 + (n + 1)an z n = 0.

n=0

1235
Equating the coefficient of the z n term gives us

(n + 2)(n + 1)an+2 + (n + 1)an = 0, n0


an
an+2 = , n 0.
n+2
a0 and a1 are arbitrary. We determine the rest of the coefficients from the recurrence relation. We consider the cases
for even and odd n separately.
a2n2
a2n =
2n
a2n4
=
(2n)(2n 2)
a0
= (1)n
(2n)(2n 2) 4 2
a0
= (1)n Qn , n0
m=1 2m

a2n1
a2n+1 =
2n + 1
a2n3
=
(2n + 1)(2n 1)
a1
= (1)n
(2n + 1)(2n 1) 5 3
a1
= (1)n Qn , n0
m=1 (2m + 1)

If {w1 , w2 } is the fundamental set of solutions, then the initial conditions demand that w1 = 1 + 0 z + and
w2 = 0 + z + . We see that w1 will have only even powers of z and w2 will have only odd powers of z.

X (1)n 2n X (1)n
w1 = Qn z , w2 = Qn z 2n+1
n=0 m=1 2m n=0 m=1 (2m + 1)

1236
Since the coefficient functions in the differential equation are entire, (analytic in the finite complex plane), the radius
of convergence of these series solutions is infinite.

Solution 23.7

1 0 1
w00 + w + w = 0.
2z z

We can find the indicial equation by substituting w = z + O(z +1 ) into the differential equation.

1
( 1)z 2 + z 2 + z 1 = O(z 1 )
2

Equating the coefficient of the z 2 term,

1
( 1) + = 0
2
1
= 0, .
2

Since the roots are distinct and do not differ by an integer, the solutions are of the form


X
X
n 1/2
w1 = an z , w2 = z bn z n .
n=0 n=0

1237
Differentiating the series for the first solution,

X
w1 = an z n
n=0

X
w10 = nan z n1
n=1

X
= (n + 1)an+1 z n
n=0
X
w100 = n(n + 1)an+1 z n1 .
n=1

Substituting this series into the differential equation,



X
n1 1 X n 1X
n(n + 1)an+1 z + (n + 1)an+1 z + an z n = 0
n=1
2z n=0 z n=0
 
X 1 1 1
n(n + 1)an+1 + (n + 1)an+1 + an z n1 + a1 + a0 = 0.
n=1
2 2z z
Equating powers of z,
a1
z 1 : + a0 = 0 a1 = 2a0
2 
1 an
z n1 : n+ (n + 1)an+1 + an = 0 an+1 = .
2 (n + 1/2)(n + 1)
We can combine the above two equations for an .
an
an+1 = , for n 0
(n + 1/2)(n + 1)

1238
Solving this difference equation for an ,
n1
Y 1
an = a0
j=0
(j + 1/2)(j + 1)

n1
(1)n Y 1
an = a0
n! j=0 j + 1/2

Now lets find the second solution. Differentiating w2 ,



X
w20 = (n + 1/2)bn z n1/2
n=0

X
w200 = (n + 1/2)(n 1/2)bn z n3/2 .
n=0

Substituting these expansions into the differential equation,



X 1X X
(n + 1/2)(n 1/2)bn z n3/2 + (n + 1/2)bn z n3/2 + bn1 z n3/2 = 0.
n=0
2 n=0 n=1

Equating the coefficient of the z 3/2 term,


 
1 1 11
b0 + b0 = 0,
2 2 22
we see that b0 is arbitrary. Equating the other coefficients of powers of z,
1
(n + 1/2)(n 1/2)bn + (n + 1/2)bn + bn1 = 0
2
bn1
bn =
n(n + 1/2)

1239
Calculating the bn s,
b0
b1 =
1 32
b0
b2 =
1 2 23 25
(1)n 2n b0
bn =
n! 3 5 (2n + 1)
Thus the second solution is

1/2
X (1)n 2n z n
w2 = b0 z .
n=0
n! 3 5 (2n + 1)

Solution 23.8
 
00 2 3 1
w + + 2 w0 + 2 w = 0.
z z z
In order to analyze the behavior at infinity we make the change of variables t = 1/z, u(t) = w(z) and examine the
point t = 0. Writing the derivatives with respect to z in terms if t yields
1
z=
t
1
dz = 2 dt
t
d d
= t2
dz dt
d2
 
2 d 2 d
= t t
dz 2 dt dt
2
d d
= t4 2 + 2t3 .
dt dt

1240
The equation for u is then

t4 u00 + 2t3 u0 + (2t + 3t2 )(t2 )u0 + t2 u = 0


1
u00 + 3u0 + 2 u = 0
t
We see that t = 0 is a regular singular point. To find the indicial equation, we substitute u = t + O(t+1 ) into the
differential equation.

( 1)t2 3t1 + t2 = O(t1 )

Equating the coefficients of the t2 terms,

( 1) + 1 = 0

1i 3
=
2
Since the roots of the indicial equation are distinct and do not differ by an integer, a set of solutions has the form
(
)
X X
t(1+i 3)/2 an tn , t(1i 3)/2 bn tn .
n=0 n=0

Noting that
! !
i 3 i 3
(1+i 3)/2
t = t1/2 exp log t , and t (1i 3)/2
= t1/2 exp log t .
2 2
We can take the sum and difference of the above solutions to obtain the form
! !
3 X 3 X
u1 = t1/2 cos log t an tn , u1 = t1/2 sin log t bn tn .
2 n=0
2 n=0

1241
Putting the answer in terms of z, we have the form of the two Frobenius expansions about infinity.
! !
3 X an 3 X bn
w1 = z 1/2 cos log z , w 1 = z 1/2
sin log z .
2 n=0
zn 2 n=0
zn

Solution 23.9
1. We write the equation in the standard form.
bx 0 a
y 00 + y y=0
x x
Since bx
x
has no worse than a first order pole and xa has no worse than a second order pole at x = 0, that is a
regular singular point. Since the coefficient functions have no other singularities in the finite complex plane, all
the other points in the finite complex plane are regular points.
Now to examine the point at infinity. We make the change of variables u() = y(x), = 1/x.
d d 1
y0 = u = 2 u0 = 2 u0
dx d x
 
d d
y 00 = 2 2 u = 4 u00 + 2 3 u0
d d
The differential equation becomes
xy 00 + (b x)y 0 ay
 
1 4 00 3 0 1
2 u0 au = 0
 
u + 2 u + b

u + (2 b) + u0 au = 0
3 00 2

 
00 2b 1 a
u + + 2 3u = 0

Since this equation has an irregular singular point at = 0, the equation for y(x) has an irregular singular point
at infinity.

1242
2. The coefficient functions are


1X 1
p(x) pn xn = (b x),
x n=1 x

1 X 1
q(x) 2 qn xn = 2 (0 ax).
x n=1 x

The indicial equation is

2 + (p0 1) + q0 = 0
2 + (b 1) + 0 = 0
( + b 1) = 0.

3. Since one of the roots of the indicial equation is zero, and the other root is not a negative integer, one of the

1243
solutions of the differential equation is a Taylor series.
X
y1 = ck xk
k=0

X
y10 = kck xk1
k=1
X
= (k + 1)ck+1 xk
k=0

X
= kck xk1
k=0
X
y100 = k(k 1)ck xk2
k=2

X
= (k + 1)kck+1 xk1
k=1

X
= (k + 1)kck+1 xk1
k=0

We substitute the Taylor series into the differential equation.


xy 00 + (b x)y 0 ay = 0

X
X X
X
(k + 1)kck+1 xk + b (k + 1)ck+1 xk kck xk a ck xk = 0
k=0 k=0 k=0 k=0

We equate coefficients to determine a recurrence relation for the coefficients.


(k + 1)kck+1 + b(k + 1)ck+1 kck ack = 0
k+a
ck+1 = ck
(k + 1)(k + b)

1244
For c0 = 1, the recurrence relation has the solution

(a)k xk
ck = .
(b)k k!
Thus one solution is

X (a)k k
y1 (x) = x .
k=0
(b)k k!

4. If a = m, where m is a non-negative integer, then (a)k = 0 for k > m. This makes y1 a polynomial:
m
X (a)k k
y1 (x) = x .
k=0
(b)k k!

5. If b = n + 1, where n is a non-negative integer, the indicial equation is

( + n) = 0.

For the case n = 0, the indicial equation has a double root at zero. Thus the solutions have the form:
m
X (a)k k X
y1 (x) = x , y2 (x) = y1 (x) log x + dk xk
k=0
(b)k k!
k=0

For the case n > 0 the roots of the indicial equation differ by an integer. The solutions have the form:
m
X (a)k k n
X
y1 (x) = x , y2 (x) = d1 y1 (x) log x + x dk xk
k=0
(b)k k! k=0

The form of the solution for y2 can be substituted into the equation to determine the coefficients dk .

1245
Solution 23.10
We write the equation in the standard form.
xy 00 + 2xy 0 + 6 ex y = 0
ex
y 00 + 2y 0 + 6 y = 0
x
We see that x = 0 is a regular singular point. The indicial equation is
2 = 0
= 0, 1.
The first solution has the Frobenius form.
y1 = x + a2 x2 + a3 x3 + O(x4 )
We substitute y1 into the differential equation and equate coefficients of powers of x.
xy 00 + 2xy 0 + 6 ex y = 0

x(2a2 + 6a3 x + O(x2 )) + 2x(1 + 2a2 x + 3a3 x2 + O(x3 ))


+ 6(1 + x + x2 /2 + O(x3 ))(x + a2 x2 + a3 x3 + O(x4 )) = 0

(2a2 x + 6a3 x2 ) + (2x + 4a2 x2 ) + (6x + 6(1 + a2 )x2 ) = O(x3 ) = 0


17
a2 = 4, a3 =
3
17
y1 = x 4x2 + x3 + O(x4 )
3
Now we see if the second solution has the Frobenius form. There is no a1 x term because y2 is only determined up to
an additive constant times y1 .
y2 = 1 + O(x2 )

1246
We substitute y2 into the differential equation and equate coefficients of powers of x.

xy 00 + 2xy 0 + 6 ex y = 0
O(x) + O(x) + 6(1 + O(x))(1 + O(x2 )) = 0
6 = O(x)

The substitution y2 = 1 + O(x) has yielded a contradiction. Since the second solution is not of the Frobenius form, it
has the following form:
y2 = y1 ln(x) + a0 + a2 x2 + O(x3 )
The first three terms in the solution are

y2 = a0 + x ln x 4x2 ln x + O(x2 ).

We calculate the derivatives of y2 .

y20 = ln(x) + O(1)


1
y200 = + O(ln(x))
x
We substitute y2 into the differential equation and equate coefficients.

xy 00 + 2xy 0 + 6 ex y = 0
(1 + O(x ln x)) + 2 (O(x ln x)) + 6 (a0 + O(x ln x)) = 0
1 + 6a0 = 0
1
y2 = + x ln x 4x2 ln x + O(x2 )
6

1247
23.8 Quiz
Problem 23.1
Write the definition of convergence of the series
P
n=1 an .
Solution
Problem 23.2
What is the Cauchy convergence criterion for series?
Solution
Problem 23.3
Define absolute convergence and uniform convergence. What is the relationship between the two?
Solution
Problem 23.4
Write the geometric series and the function to which it converges. For what values of the variable does the series
converge?
Solution
Problem 23.5
For what real values of a does the series a
P
n=1 n converge?
Solution
Problem 23.6
State the ratio and root convergence tests.
Solution
Problem 23.7
State the integral convergence test.
Solution

1248
23.9 Quiz Solutions
Solution 23.1
The series
P PN
n=1 an converges if the sequence of partial sums, SN = n=1 an , converges. That is,

N
X
lim SN = lim an = constant.
N N
n=1

Solution 23.2
A series converges if and only if for any  > 0 there exists an N such that |Sn Sm | <  for all n, m > N .

Solution 23.3
The series
P P P
n=1 an converges absolutely if n=1 |an | converges. If the rate of convergence of n=1 an (z) is indepen-
dent of z then the series is uniformly convergent. The series is uniformly convergent in a domain if for any given  > 0
there exists an N , independent of z, such that

N
X
|f (z) SN (z)| = f (z) an (z) < 


n=1

for all z in the domain.


There is no relationship between absolute convergence and uniform convergence.

Solution 23.4


1 X
= zn for |z| < 1.
1z n=0

Solution 23.5
The series converges for a < 1.

1249
Solution 23.6
The series
P
n=1 an converges absolutely if
an+1
lim < 1.
n an

If the limit is greater


P than unity, then the series diverges. If the limit is unity, the test fails.
The series n=1 an converges absolutely if

lim |an |1/n < 1.


n

If the limit is greater than unity, then the series diverges. If the limit is unity, the test fails.
Solution 23.7
If the coefficients an of a series
P
n=1 an are monotonically decreasing and can be extended to a monotonically decreasing
function of the continuous variable x:
a(x) = an for integer x,
then the sum converges or diverges with the integral:
Z
a(x) dx.
1

1250
Chapter 24

Asymptotic Expansions

The more you sweat in practice, the less you bleed in battle.

-Navy Seal Saying

24.1 Asymptotic Relations


The  and symbols. First we will introduce two new symbols used in asymptotic relations.

f (x)  g(x) as x x0 ,

is read, f (x) is much smaller than g(x) as x tends to x0 . This means

f (x)
lim = 0.
xx0 g(x)

The notation
f (x) g(x) as x x0 ,

1251
is read f (x) is asymptotic to g(x) as x tends to x0 ; which means
f (x)
lim = 1.
xx0 g(x)
A few simple examples are
ex  x as x +
sin x x as x 0
1/x  1 as x +
e1/x  xn as x 0+ for all n
An equivalent definition of f (x) g(x) as x x0 is
f (x) g(x)  g(x) as x x0 .
Note that it does not make sense to say that a function f (x) is asymptotic to zero. Using the above definition this
would imply
f (x)  0 as x x0 .
If you encounter an expression like f (x) + g(x) 0, take this to mean f (x) g(x).

The Big O and Little o Notation. If |f (x)| m|g(x)| for some constant m in some neighborhood of the point
x = x0 , then we say that
f (x) = O(g(x)) as x x0 .
We read this as f is big O of g as x goes to x0 . If g(x) does not vanish, an equivalent definition is that f (x)/g(x)
is bounded as x x0 .
If for any given positive there exists a neighborhood of x = x0 in which |f (x)| |g(x)| then
f (x) = o(g(x)) as x x0 .
This is read, f is little o of g as x goes to x0 .
For a few examples of the use of this notation,

1252
ex = o(xn ) as x for any n.

sin x = O(x) as x 0.

cos x 1 = o(1) as x 0.

log x = o(x ) as x + for any positive .

Operations on Asymptotic Relations. You can perform the ordinary arithmetic operations on asymptotic rela-
tions. Addition, multiplication, and division are valid.
You can always integrate an asymptotic relation. Integration is a smoothing operation. However, it is necessary to
exercise some care.

Example 24.1.1 Consider


1
f 0 (x) as x .
x2
This does not imply that
1
f (x)
as x .
x
We have forgotten the constant of integration. Integrating the asymptotic relation for f 0 (x) yields
1
f (x) + c as x .
x
If c is nonzero then
f (x) c as x .

It is not always valid to differentiate an asymptotic relation.


1 1
Example 24.1.2 Consider f (x) = x
+ x2
sin(x3 ).
1
f (x) as x .
x

1253
Differentiating this relation yields
1
f 0 (x) as x .
x2
However, this is not true since
1 2
f 0 (x) = 2
3 sin(x3 ) + 2 cos(x3 )
x x
1
6 2 as x .

x

The Controlling Factor. The controlling factor is the most rapidly varying factor in an asymptotic relation.
Consider a function f (x) that is asymptotic to x2 ex as x goes to infinity. The controlling factor is ex . For a few
examples of this,

x log x has the controlling factor x as x .

x2 e1/x has the controlling factor e1/x as x 0.

x1 sin x has the controlling factor sin x as x .

The Leading Behavior. Consider a function that is asymptotic to a sum of terms.

f (x) a0 (x) + a1 (x) + a2 (x) + , as x x0 .

where
a0 (x)  a1 (x)  a2 (x)  , as x x0 .
The first term in the sum is the leading order behavior. For a few examples,

For sin x x x3 /6 + x5 /120 as x 0, the leading order behavior is x.

For f (x) ex (1 1/x + 1/x2 ) as x , the leading order behavior is ex .

1254
24.2 Leading Order Behavior of Differential Equations
It is often useful to know the leading order behavior of the solutions to a differential equation. If we are considering
a regular point or a regular singular point, the approach is straight forward. We simply use a Taylor expansion or the
Frobenius method. However, if we are considering an irregular singular point, we will have to be a little more creative.
Instead of an all encompassing theory like the Frobenius method which always gives us the solution, we will use a
heuristic approach that usually gives us the solution.

Example 24.2.1 Consider the Airy equation


y 00 = xy.

We 1 would like to know how the solutions of this equation behave as x +. First we need to classify the point at
infinity. The change of variables

1 d d d2 4 d
2
d
x= , y(x) = u(t), = t2 , 2
= t 2
+ 2t3
t dx dt dx dt dt

yields

1
t4 u00 + 2t3 u0 = u
t
00 2 0 1
u + u 5 u = 0.
t t

Since the equation for u has an irregular singular point at zero, the equation for y has an irregular singular point at
infinity.

1
Using We may be a bit presumptuous on my part. Even if you dont particularly want to know how the solutions behave, I
urge you to just play along. This is an interesting section, I promise.

1255
The Controlling Factor. Since the solutions at irregular singular points often have exponential behavior, we make
the substitution y = es(x) into the differential equation for y.

d2  s 
2
e = x es
dx
 00
s + (s0 )2 es = x es


s00 + (s0 )2 = x

The Dominant Balance. Now we have a differential equation for s that appears harder to solve than our equation
for y. However, we did not introduce the substitution in order to obtain an equation that we could solve exactly. We
are looking for an equation that we can solve approximately in the limit as x . If one of the terms in the equation
for s is much smaller that the other two as x , then dropping that term and solving the simpler equation may give
us an approximate solution. If one of the terms in the equation for s is much smaller than the others then we say that
the remaining terms form a dominant balance in the limit as x .
Assume that the s00 term is much smaller that the others, s00  (s0 )2 , x as x . This gives us

(s0 )2 x

s0 x
2
s x3/2 as x .
3

Now lets
check our assumption that the s00 term is small. Assuming that we can differentiate the asymptotic relation
s0 x, we obtain s00 12 x1/2 as x .

s00  (s0 )2 , x x1/2  x as x

Thus we see that the behavior we found for s is consistent with our assumption. The controlling factors for solutions
to the Airy equation are exp( 23 x3/2 ) as x .

1256
The Leading Order Behavior of the Decaying Solution. Lets find the leading order behavior as x goes to
infinity of the solution with the controlling factor exp( 32 x3/2 ). We substitute
2
s(x) = x3/2 + t(x), where t(x)  x3/2 as x
3
into the differential equation for s.
s00 + (s0 )2 = x
1
x1/2 + t00 + (x1/2 + t0 )2 = x
2
1
t00 + (t0 )2 2x1/2 t0 x1/2 = 0
2
Assume that we can differentiate t  x3/2 to obtain
t0  x1/2 , t00  x1/2 as x .
Since t00  12 x1/2 we drop the t00 term. Also, t0  x1/2 implies that (t0 )2  2x1/2 t0 , so we drop the (t0 )2 term.
This gives us
1
2x1/2 t0 x1/2 0
2
1
t x1
0
4
1
t log x + c
4
1
t log x as x .
4
Checking our assumptions about t,
t0  x1/2 x1  x1/2
t00  x1/2 x2  x1/2

1257
we see that the behavior of t is consistent with our assumptions.
So far we have  
2 3/2 1
y(x) exp x log x + u(x) as x ,
3 4
where u(x)  log x as x . To continue, we substitute t(x) = 41 log x + u(x) into the differential equation for
t(x).
1
t00 + (t0 )2 2x1/2 t0 x1/2 = 0
2
 2  
1 2 00 1 1 0 1 1 1
x +u + x +u 2x 1/2
x + u x1/2 = 0
0
4 4 4 2
 
1 5
u00 + (u0 )2 + x1 2x1/2 u0 + x2 = 0
2 16
Assume that we can differentiate the asymptotic relation for u to obtain
u0  x1 , u00  x2 as x .
We know that 12 x1 u0  2x1/2 u0 . Using our assumptions,
5 2
u00  x2 u00  x
16
5
u0  x1 (u0 )2  x2 .
16
Thus we obtain
5 2
2x1/2 u0 + x 0
16
5 5/2
u0 x
32
5
u x3/2 + c
48
u c as x .

1258
Since u = c + o(1), eu = ec +o(1). The behavior of y is
 
1/4 2 3/2
yx exp x (ec +o(1)) as x .
3
Thus the full leading order behavior of the decaying solution is
 
1/4 2 3/2
y (const)x exp x as x .
3
You can show that the leading behavior of the exponentially growing solution is
 
1/4 2 3/2
y (const)x exp x as x .
3

Example 24.2.2 The Modified Bessel Equation. Consider the modified Bessel equation
x2 y 00 + xy 0 (x2 + 2 )y = 0.
We would like to know how the solutions of this equation behave as x +. First we need to classify the point at
infinity. The change of variables x = 1t , y(x) = u(t) yields
 
1 4 00 3 0 1 2 0 1 2
(t u + 2t u ) + (t u ) 2 + u = 0
t2 t t
2
 
1 1
u00 + u0 4 + 2 u = 0
t t t
Since u(t) has an irregular singular point at t = 0, y(x) has an irregular singular point at infinity.

The Controlling Factor. Since the solutions at irregular singular points often have exponential behavior, we make
the substitution y = es(x) into the differential equation for y.
x2 (s00 + (s0 )2 ) es +xs0 es (x2 + 2 ) es = 0
1 2
s00 + (s0 )2 + s0 (1 + 2 ) = 0
x x

1259
We make the assumption that s00  (s0 )2 as x and we know that 2 /x2  1 as x . Thus we drop these
two terms from the equation to obtain an approximate equation for s.
1
(s0 )2 + s0 1 0
x
0
This is a quadratic equation for s , so we can solve it exactly. However, let us try to simplify the equation even further.
Assume that as x goes to infinity one of the three terms is much smaller that the other two. If this is the case, there
will be a balance between the two dominant terms and we can neglect the third. Lets check the three possibilities.
1.
1 1
1 is small. (s0 )2 + s0 0 s0 , 0
x x
1
1 6 x2 , 0 as x so this balance is inconsistent.
2.
1 0
s is small. (s0 )2 1 0 s0 1
x
This balance is consistent as x1  1 as x .
3.
1 0
(s0 )2 is small. s 10 s0 x
x
This balance is not consistent as x2 6 1 as x .
The only dominant balance that makes sense leads to s0 1 as x . Integrating this relationship,
s x + c
x as x .
Now lets see if our assumption that we made to get the simplified equation for s is valid. Assuming that we can
differentiate s0 1, s00  (s0 )2 becomes
d   2
1 + o(1)  1 + o(1)
dx
0 + o(1/x)  1

1260
Thus we see that the behavior we obtained for s is consistent with our initial assumption.
We have found two controlling factors, ex and ex . This is a good sign as we know that there must be two linearly
independent solutions to the equation.

Leading Order Behavior. Now lets find the full leading behavior of the solution with the controlling factor ex .
In order to find a better approximation for s, we substitute s(x) = x + t(x), where t(x)  x as x , into the
differential equation for s.
2
 
00 0 2 1 0
s + (s ) + s 1 + 2 = 0
x x
2
 
00 0 2 1 0
t + (1 + t ) + (1 + t ) 1 + 2 = 0
x x
2
   
00 0 2 1 0 1
t + (t ) + 2 t + =0
x x x2
1 2 1
We know that x
 2 and x2
 x
as x . Dropping these terms from the equation yields
1
t00 + (t0 )2 2t0 0.
x
Assuming that we can differentiate the asymptotic relation for t, we obtain t0  1 and t00  1
x
as x . We can
drop t00 . Since t0 vanishes as x goes to infinity, (t0 )2  t0 . Thus we are left with
1
2t0 0, as x .
x
Integrating this relationship,
1
t log x + c
2
1
log x as x .
2

1261
Checking our assumptions about the behavior of t,
1
t0  1 1
2x
1 1 1
t00  
x 2x2 x
we see that the solution is consistent with our assumptions.
The leading order behavior to the solution with controlling factor ex is
 
1
y(x) exp x log x + u(x) = x1/2 ex+u(x) as x ,
2
where u(x)  log x. We substitute t = 21 log x + u(x) into the differential equation for t in order to find the
asymptotic behavior of u.
1 2
   
00 0 2 1 0
t + (t ) + 2 t + =0
x x x2
2 
1 2
    
1 00 1 0 1 1 0
+u + +u + 2 +u + =0
2x2 2x x 2x x x2
1 2
u00 + (u0 )2 2u0 + 2 2 = 0
4x x
Assuming that we can differentiate the asymptotic relation for u, u  x1 and u00  x12 as x . Thus we see that
0

we can neglect the u00 and (u0 )2 terms.  


0 1 2 1
2u + 0
4 x2
 
0 1 1 2 1
u
2 4 x2
 
1 1 1
u 2 +c
2 4 x
u c as x

1262
Since u = c + o(1), we can expand eu as ec +o(1). Thus we can write the leading order behavior as

y x1/2 ex (ec +o(1)).

Thus the full leading order behavior is

y (const)x1/2 ex as x .

You can verify that the solution with the controlling factor ex has the leading order behavior

y (const)x1/2 ex as x .

Two linearly independent solutions to the modified Bessel equation are the modified Bessel functions, I (x) and
K (x). These functions have the asymptotic behavior

1
I (x) ex , K (x) ex as x .
2x 2x

x
In Figure 24.1 K0 (x) is plotted in a solid line and 2x e is plotted in a dashed line. We see that the leading order
behavior of the solution as x goes to infinity gives a good approximation to the behavior even for fairly small values of
x.

24.3 Integration by Parts


Example 24.3.1 The complementary error function
Z
2 2
erfc(x) = et dt
x

1263
2

1.75

1.5

1.25

0.75

0.5

0.25

0 1 2 3 4 5

Figure 24.1: Plot of K0 (x) and its leading order behavior.

is used in statistics for its relation to the normal probability distribution. We would like to find an approximation to
erfc(x) for large x. Using integration by parts,

Z  
2 1 2

erfc(x) = 2t et dt
x 2t
  Z
2 1 t2 2 1 2 t2
= e t e dt
2t x x 2
Z
1 1 x2 1 2
= x e t2 et dt.
x

1264
We examine the residual integral in this expression.
Z
1 3
Z
1 2 t2 2
t e dt x 2t et dt
x 2 x
1 3 x2
= x e .
2
Thus we see that Z
1 2 1 2
x1 ex  t2 et dt as x .
x
Therefore,
1 2
erfc(x) x1 ex as x ,

2
and we expect that 1 x1 ex would be a good approximation to erfc(x) for large x. In Figure 24.2 log(erfc(x)) is
 
2
graphed in a solid line and log 1 x1 ex is graphed in a dashed line. We see that this first approximation to the
error function gives very good results even for moderate values of x. Table 24.1 gives the error in this first approximation
for various values of x.
If we continue integrating by parts, we might get a better approximation to the complementary error function.
Z
1 1 x2 1 2
erfc(x) = x e t2 et dt
x
  Z
1 1 x2 1 1 3 t2 1 3 4 t2
= x e t e + t e dt
2 x x 2
  Z
1 x2 1 1 3 1 3 4 t2
= e x x + t e dt
2 x 2
    Z
1 x2 1 1 3 1 3 5 t2 1 15 6 t2
= e x x + t e t e dt
2 4 x x 4
  Z
1 x2 1 1 3 3 5 1 15 6 t2
= e x x + x t e dt
2 4 x 4

1265
2

0.5 1 1.5 2 2.5 3


-2

-4

-6

-8

-10

Figure 24.2: Logarithm of the Approximation to the Complementary Error Function.

The error in approximating erfc(x) with the first three terms is given in Table 24.1. We see that for x 2 the three
terms give a much better approximation to erfc(x) than just the first term.

At this point you might guess that you could continue this process indefinitely. By repeated application of integration
by parts, you can obtain the series expansion


X (1)n (2n)!
2 2
erfc(x) = ex .
n=0
n!(2x)2n+1

1266
x erfc(x) One Term Relative Error Three Term Relative Error
1 0.157 0.3203 0.6497
2 0.00468 0.1044 0.0182
3 2.21 105 0.0507 0.0020
4 1.54 108 0.0296 3.9 104
5 1.54 1012 0.0192 1.1 104
6 2.15 1017 0.0135 3.7 105
7 4.18 1023 0.0100 1.5 105
8 1.12 1029 0.0077 6.9 106
9 4.14 1037 0.0061 3.4 106
10 2.09 1045 0.0049 1.8 106

Table 24.1:

This is a Taylor expansion about infinity. Lets find the radius of convergence.

(1)n+1 (2(n + 1))! n!(2x)2n+1



an+1 (x)
lim < 1 lim <1
n an (x) n (n + 1)!(2x)2(n+1)+1 (1)n (2n)!

(2n + 2)(2n + 1)
lim <1
n (n + 1)(2x)2

2(2n + 1)
lim <1
n (2x)2

1
= 0
x

Thus we see that our series diverges for all x. Our conventional mathematical sense would tell us that this series is
useless, however we will see that this series is very useful as an asymptotic expansion of erfc(x).

1267
Say we are working with a convergent series expansion of some function f (x).

X
f (x) = an (x)
n=0

For fixed x = x0 ,
N
X
f (x0 ) an (x0 ) 0 as N .
n=0
P
For an asymptotic series we have a quite different behavior. If g(x) is asymptotic to n=0 bn (x) as x x0 then for
fixed N ,
XN
g(x) bn (x)  bN (x) as x x0 .
0

For the complementary error function,


N
X (1)n (2n)!
2 2
For fixed N , erfc(x) ex 2n+1
 x2N 1 as x .
n=0
n!(2x)

We say that the error function is asymptotic to the series as x goes to infinity.

X (1)n (2n)!
2 2
erfc(x) ex as x
n=0
n!(2x)2n+1

In Figure 24.3 the logarithm of the difference between the one term, ten term and twenty term approximations and
the complementary error function are graphed in coarse, medium, and fine dashed lines, respectively.

*Optimal Asymptotic Series. Of the three approximations, the one term is best for x . 2, the ten term is
best for 2 . x . 4, and the twenty term is best for 4 . x. This leads us to the concept of an optimal asymptotic
approximation. An optimal asymptotic approximation contains the number of terms in the series that best approximates
the true behavior.

1268
1 2 3 4 5 6

-20

-40

-60

Figure 24.3: log(error in approximation)

In Figure 24.4 we see a plot of the number of terms in the approximation versus the logarithm of the error for x = 3.
Thus we see that the optimal asymptotic approximation is the first nine terms. After nine terms the error gets larger.
It was inevitable that the error would start to grow after some point as the series diverges for all x.

A good rule of thumb for finding the optimal series is to find the smallest term in the series and take all of the
terms up to but not including the smallest term as the optimal approximation. This makes sense, because the nth term
is an approximation of the error incurred by using the first n 1 terms. In Figure 24.5 there is a plot of n versus the
logarithm of the nth term in the asymptotic expansion of erfc(3). We see that the tenth term is the smallest. Thus, in
this case, our rule of thumb predicts the actual optimal series.

1269
-12

-14

-16

-18

5 10 15 20 25

Figure 24.4: The logarithm of the error in using n terms.

24.4 Asymptotic Series


P
A function f (x) has an asymptotic series expansion about x = x0 , n=0 an (x), if
N
X
f (x) an (x)  aN (x) as x x0 for all N.
n=0

An asymptotic series may be convergent or divergent. Most of the asymptotic series you encounter will be divergent.
If the series is convergent, then we have that
N
X
f (x) an (x) 0 as N for fixed x.
n=0

1270
-12

-14

-16

5 10 15 20 25

Figure 24.5: The logarithm of the nth term in the expansion for x = 3.

Let n (x) be some set of gauge functions. The example that we are most familiar with is n (x) = xn . If we say that


X
X
an n (x) bn n (x),
n=0 n=0

then this means that an = bn .

1271
24.5 Asymptotic Expansions of Differential Equations
24.5.1 The Parabolic Cylinder Equation.
Controlling Factor. Let us examine the behavior of the bounded solution of the parabolic cylinder equation as
x +.  
00 1 1 2
y + + x y=0
2 4
This equation has an irregular singular point at infinity. With the substitution y = es , the equation becomes
1 1
s00 + (s0 )2 + + x2 = 0.
2 4
We know that
1 1
+  x2 as x +
2 4
so we drop this term from the equation. Let us make the assumption that
s00  (s0 )2 as x +.
Thus we are left with the equation
1 2
(s0 )2 x
4
1
s0 x
2
1
s x2 + c
4
1
s x2 as x +
4
Now lets check if our assumption is consistent. Substituting into s00  (s0 )2 yields 1/2  x2 /4 as x + which
is true. Since the equation for y is second order, we would expect that there are two different behaviors as x +.
This is confirmed by the fact that we found two behaviors for s. s x2 /4 corresponds to the solution that is bounded
2
at +. Thus the controlling factor of the leading behavior is ex /4 .

1272
Leading Order Behavior. Now we attempt to get a better approximation to s. We make the substitution
s = 41 x2 + t(x) into the equation for s where t  x2 as x +.
1 1 1 1
+ t00 + x2 xt0 + (t0 )2 + + x2 = 0
2 4 2 4
t00 xt0 + (t0 )2 + = 0
Since t  x2 , we assume that t0  x and t00  1 as x +. Note that this in only an assumption since it is not
always valid to differentiate an asymptotic relation. Thus (t0 )2  xt0 and t00  xt0 as x +; we drop these terms
from the equation.

t0
x
t log x + c
t log x as x +
Checking our assumptions for the derivatives of t,
1 1
t0  x x t00  1  1,
x x2
we see that they were consistent. Now we wish to refine our approximation for t with the substitution t(x) =
log x + u(x). So far we have that
 2   2 
x x
y exp + log x + u(x) = x exp + u(x) as x +.
4 4
We can try and determine u(x) by substituting the expression t(x) = log x + u(x) into the equation for t.
00 0 2 2 0
2
+ u ( + xu ) + 2
+ u + (u0 )2 + = 0
x x x
After suitable simplification, this equation becomes
2
u0 as x +
x3

1273
Integrating this asymptotic relation,
2
u + c as x +.
2x2
2
Notice that
2x2
 c as x +; thus this procedure fails to give us the behavior of u(x). Further refinements to
our approximation for s go to a constant value as x +. Thus we have that the leading behavior is
 2
x
y cx exp as x +
4

Asymptotic Expansion Since we have factored off the singular behavior of y, we might expect that what is left
over is well behaved enough to be expanded in a Taylor series about infinity. Let us assume that we can expand the
solution for y in the form
 2  2X
x x

y(x) x exp
(x) = x exp an xn as x +
4 4 n=0
 2
where a0 = 1. Differentiating y = x exp x4 (x),
 
0 1 +1 x2 /4 2
y = x 1
x e (x) + x ex /4 0 (x)
2
   
00 2 1 1 1 +2 x2 /4 1 1 +1 x2 /4 0
y = ( 1)x x ( + 1)x + x e (x) + 2 x x e (x)
2 2 4 2
2 /4
+ x ex 00 (x).
Substituting this into the differential equation for y,
     
2 1 1 2 1 1 0 00 1 1 2
( 1)x ( + ) + x (x) + 2 x x (x) + (x) + + x (x) = 0
2 4 2 2 4
00 1 0 2
(x) + (2x x) (x) + ( 1)x = 0
x2 00 (x) + (2x x3 ) 0 (x) + ( 1)(x) = 0.

1274
Differentiating the expression for (x),

X
(x) = an xn
n=0
X
X
0 (x) = nan xn1 = (n + 2)an+2 xn3
n=1 n=1

X
00 (x) = n(n + 1)an xn2 .
n=1

Substituting this into the differential equation for (x),



X
X
X
X
n(n + 1)an xn + 2 nan xn (n + 2)an+2 xn + ( 1) an xn = 0.
n=1 n=1 n=1 n=0

Equating the coefficient of x1 to zero yields

a1 x = 0 a1 = 0.

Equating the coefficient of x0 ,


1
2a2 + ( 1)a0 = 0 a2 = ( 1).
2
From the coefficient of xn for n > 0,

n(n + 1)an 2nan + (n + 2)an+2 + ( 1)an = 0


(n + 2)an+2 = [n(n + 1) 2n + ( 1)]an
(n + 2)an+2 = [n2 + n 2n + ( 1)]an
(n + 2)an+2 = (n )(n + 1)an .

1275
Thus the recursion formula for the an s is
(n )(n + 1)
an+2 = an , a0 = 1, a1 = 0.
n+2
The first few terms in (x) are

( 1) 2 ( 1)( 2)( 3) 4
(x) 1 x + x as x +
21 1! 22 2!
If we check the radius of convergence of this series
an+2 xn2

(n )(n + 1) 2
lim <1 lim x < 1
n an xn n n+2
1
=0
x
we see that the radius of convergence is zero. Thus if 6= 0, 1, 2, . . . our asymptotic expansion for y
 
x2 /4 ( 1) 2 ( 1)( 2)( 3) 4
yx e 1 x + x
21 1! 22 2!

diverges for all x. However this solution is still very useful. If we only use a finite number of terms, we will get a very
good numerical approximation for large x.
In Figure 24.6 the one term, two term, and three term asymptotic approximations are shown in rough, medium, and
fine dashing, respectively. The numerical solution is plotted in a solid line.

1276
6

1 2 3 4 5 6

-2

Figure 24.6: Asymptotic Approximations to the Parabolic Cylinder Function.

1277
Chapter 25

Hilbert Spaces

An expert is a man who has made all the mistakes which can be made, in a narrow field.

- Niels Bohr

WARNING: UNDER HEAVY CONSTRUCTION.

In this chapter we will introduce Hilbert spaces. We develop the two important examples: l2 , the space of square
summable infinite vectors and L2 , the space of square integrable functions.

25.1 Linear Spaces


A linear space is a set of elements {x, y, z, . . .} that is closed under addition and scalar multiplication. By closed under
addition we mean: if x and y are elements, then z = x + y is an element. The addition is commutative and associative.

x+y =y+x
(x + y) + z = x + (y + z)

1278
Scalar multiplication is associative and distributive. Let a and b be scalars, a, b C.

(ab)x = a(bx)
(a + b)x = ax + bx
a(x + y) = ax + ay

All the linear spaces that we will work with have additional properties: The zero element 0 is the additive identity.

x+0=x

Multiplication by the scalar 1 is the multiplicative identity.

1x = x

Each element x and the additive inverse, x.


x + (x) = 0
Consider a set of elements {x1 , x2 , . . .}. Let the ci be scalars. If

y = c1 x1 + c2 x2 +

then y is a linear combination of the xi . A set of elements {x1 , x2 , . . .} is linearly independent if the equation

c1 x1 + c2 x2 + = 0

has only the trivial solution c1 = c2 = = 0. Otherwise the set is linearly dependent.
Let {e1 , e2 , } be a linearly independent set of elements. If every element x can be written as a linear combination
of the ei then the set {ei } is a basis for the space. The ei are called base elements.
X
x= ci ei
i

The set {ei } is also called a coordinate system. The scalars ci are the coordinates or components of x. If the set {ei }
is a basis, then we say that the set is complete.

1279
25.2 Inner Products
hx|yi is an inner product of two elements x and y if it satisfies the properties:
1. Conjugate-commutative.
hx|yi = hx|yi

2. Linearity in the second argument.


hx|ay + bzi = ahx|yi + bhx|yi

3. Positive definite.

hx|xi 0
hx|xi = 0 if and only if x = 0

From these properties one can derive the properties:


1. Conjugate linearity in the first argument.

hax + by|zi = ahx|zi + bhx|zi

2. Schwarz Inequality.
|hx|yi|2 hx|xihy|yi

One inner product of vectors is the Euclidean inner product.


n
X
hx|yi x y = xi yi .
i=0

One inner product of functions defined on (a . . . b) is


Z b
hu|vi = u(x)v(x) dx.
a

1280
If (x) is a positive-valued function, then we can define the inner product:
Z b
hu||vi = u(x)(x)v(x) dx.
a

This is called the inner product with respect to the weighting function (x). It is also denoted hu|vi .

25.3 Norms
A norm is a real-valued function on a space which satisfies the following properties.

1. Positive.
kxk 0

2. Definite.
kxk = 0 if and only if x = 0

3. Multiplication my a scalar, c C.
kcxk = |c|kxk

4. Triangle inequality.
kx + yk kxk + kyk

Example 25.3.1 Consider a vector space, (finite or infinite dimension), with elements x = (x1 , x2 , x3 , . . .). Here are
some common norms.

Norm generated by the inner product. p


kxk = hx|xi

1281
The lp norm.

!1/p
X
kxkp = |xk |p
k=1
There are three common cases of the lp norm.
Euclidian norm, or l2 norm. v
u
uX
kxk2 = t |xk |2
k=1

l1 norm.

X
kxk1 = |xk |
k=1

l norm.
kxk = max |xk |
k

Example 25.3.2 Consider a space of functions defined on the interval (a . . . b). Here are some common norms.
Norm generated by the inner product. p
kuk = hu|ui

The Lp norm.
Z b 1/p
p
kukp = |u(x)| dx
a
There are three common cases of the Lp norm.
Euclidian norm, or L2 norm. s
Z b
kuk2 = |u(x)|2 dx
a

1282
L1 norm. Z b
kuk1 = |u(x)| dx
a
L norm.
kuk = lim sup |u(x)|
x(a...b)

Distance. Using the norm, we can define the distance between elements u and v.
d(u, v) ku vk
Note that d(u, v) = 0 does not necessarily imply that u = v. CONTINUE.

25.4 Linear Independence.


25.5 Orthogonality
Orthogonality.
hj |k i = 0 if j 6= k
Orthonormality.
hj |k i = jk

Example 25.5.1 Infinite vectors. ej has all zeros except for a 1 in the j th position.
ej = (0, 0, . . . 0, 1, 0, . . .)
Example 25.5.2 L2 functions on (0 . . . 2).
1
j = ejx , j Z
2
1 (1) 1 (1) 1
0 = , j = cos(jx), j = sin(jx), j Z+
2

1283
25.6 Gramm-Schmidt Orthogonalization
Let {1 (x), . . . , n (x)} be a set of linearly independent functions. Using the Gramm-Schmidt orthogonalization process
we can construct a set of orthogonal functions {1 (x), . . . , n (x)} that has the same span as the set of n s with the
formulas

1 = 1
h1 |2 i
2 = 2 1
k1 k2
h1 |3 i h2 |3 i
3 = 3 2
1 2
k1 k k2 k2

n1
X hj |n i
n = n j .
j=1
kj k2

You could verify that the m are orthogonal with a proof by induction.

Example 25.6.1 Suppose we would like a polynomial approximation to cos(x) in the domain [1, 1]. One way to
do this is to find the Taylor expansion of the function about x = 0. Up to terms of order x4 , this is

(x)2 (x)4
cos(x) = 1 + + O(x6 ).
2 24

In the first graph of Figure 25.1 cos(x) and this fourth degree polynomial are plotted. We see that the approximation
is very good near x = 0, but deteriorates as we move away from that point. This makes sense because the Taylor
expansion only makes use of information about the functions behavior at the point x = 0.
As a second approach, we could find the least squares fit of a fourth degree polynomial to cos(x). The set of
functions {1, x, x2 , x3 , x4 } is independent, but not orthogonal in the interval [1, 1]. Using Gramm-Schmidt orthogo-

1284
nalization,
0 = 1
h1|xi
1 = x =x
h1|1i
h1|x2 i hx|x2 i 1
2 = x2 x = x2
h1|1i hx|xi 3
3
3 = x3 x
5
4 6 2 3
4 = x x
7 35
A widely used set of functions in mathematics is the set of Legendre polynomials {P0 (x), P1 (x), . . .}. They differ
from the n s that we generated only by constant factors. The first few are
P0 (x) = 1
P1 (x) = x
3x2 1
P2 (x) =
2
3
5x 3x
P3 (x) =
2
35x4 30x2 + 3
P4 (x) = .
8
Expanding cos(x) in Legendre polynomials
4
X
cos(x) cn Pn (x),
n=0

and calculating the generalized Fourier coefficients with the formula


hPn | cos(x)i
cn = ,
hPn |Pn i

1285
yields

15 45(2 2 21)
cos(x) P 2 (x) + P4 (x)
2 4
105
= 4 [(315 30 2 )x4 + (24 2 270)x2 + (27 2 2 )]
8

The cosine and this polynomial are plotted in the second graph in Figure 25.1. The least squares fit method uses
information about the function on the entire interval. We see that the least squares fit does not give as good an ap-
proximation close to the point x = 0 as the Taylor expansion. However, the least squares fit gives a good approximation
on the entire interval.
In order to expand a function in a Taylor series, the function must be analytic in some domain. One advantage of
using the method of least squares is that the function being approximated does not even have to be continuous.

1 1

0.5 0.5

-1 -0.5 0.5 1 -1 -0.5 0.5 1

-0.5 -0.5

-1 -1

Figure 25.1: Polynomial Approximations to cos(x).

1286
25.7 Orthonormal Function Expansion
Let {j } be an orthonormal set of functions on the interval (a, b). We expand a function f (x) in the j .
X
f (x) = cj j
j

We choose the coefficients to minimize the norm of the error.


2 * +
X X X
f cj j = f cj j f cj j


j j j
* + * + *X X +
X X
= kf k2 f cj j cj j f + cj j cj j
j j j j
X X X
= kf k2 + |cj |2 cj hf |j i cj hj |f i
j j j

2
X X X X
f cj j = kf k2 + |cj |2 cj hj |f i cj hj |f i (25.1)


j j j j

P
To complete the square, we add the constant j hj |f ihj |f i. We see the values of cj which minimize
X
kf k2 + |cj hj |f i|2 .
j

Clearly the unique minimum occurs for


cj = hj |f i.

1287
We substitute this value for cj into the right side of Equation 25.1 and note that this quantity, the squared norm of the
error, is non-negative.
X X X
kf k2 + |cj |2 |cj |2 |cj |2 0
j j j
X
kf k2 |cj |2
j

This is known as Bessels Inequality. If the set of {j } is complete then the norm of the error is zero and we obtain
Bessels Equality. X
kf k2 = |cj |2
j

25.8 Sets Of Functions


Orthogonality. Consider two complex valued functions of a real variable 1 (x) and 2 (x) defined on the interval
a x b. The inner product of the two functions is defined
Z b
h1 |2 i = 1 (x)2 (x) dx.
a
p
The two functions are orthogonal if h1 |2 i = 0. The L2 norm of a function is defined kk = h|i.
Let {1 , 2 , 3 , . . .} be a set of complex valued functions. The set of functions is orthogonal if each pair of functions
is orthogonal. That is,
hn |m i = 0 if n 6= m.
If in addition the norm of each function is 1, then the set is orthonormal. That is,
(
1 if n = m
hn |m i = nm =
0 if n 6= m.

1288
Example 25.8.1 The set of functions
(r r r )
2 2 2
sin(x), sin(2x), sin(3x), . . .

is orthonormal on the interval [0, ]. To verify this,


*r r +
2 2
Z
2 2

sin(nx) sin(nx) = sin (nx) dx
0
=1
If n 6= m then
*r r +
2
Z
2 2

sin(nx) sin(mx) = sin(nx) sin(mx) dx
0
1
Z
= (cos[(n m)x] cos[(n + m)x]) dx
0
= 0.
Example 25.8.2 The set of functions
1 1 1 1
{. . . , ex , , ex , e2x , . . .},
2 2 2 2
is orthonormal on the interval [, ]. To verify this,
* + Z
1 nx 1 nx 1
e e = enx enx dx
2 2 2
Z
1
= dx
2
= 1.

1289
If n 6= m then
* + Z
1 nx 1 mx 1
e e = enx emx dx
2 2 2
Z
1
= e(mn)x dx
2
= 0.

Orthogonal with Respect to a Weighting Function. Let (x) be a real-valued, positive function on the
interval [a, b]. We introduce the notation
Z b
hn ||m i n m dx.
a

If the set of functions {1 , 2 , 3 , . . .} satisfy


hn ||m i = 0 if n 6= m
then the functions are orthogonal with respect to the weighting function (x).
If the functions satisfy
hn ||m i = nm
then the set is orthonormal with respect to (x).

Example 25.8.3 We know that the set of functions


(r r r )
2 2 2
sin(x), sin(2x), sin(3x), . . .

is orthonormal on the interval [0, ]. That is,


Z r r
2 2
sin(nx) sin(mx) dx = nm .
0

1290

If we make the change of variables x = t in this integral, we obtain
Z 2 r

r

1 2 2
sin(n t) sin(m t) dt = nm .
0 2 t
Thus the set of functions (r )

r

r
1 1 1
sin( t), sin(2 t), sin(3 t), . . .

1
is orthonormal with respect to (t) =
2 t
on the interval [0, 2 ].

Orthogonal Series. Suppose that a function f (x) defined on [a, b] can be written as a uniformly convergent sum
of functions that are orthogonal with respect to (x).

X
f (x) = cn n (x)
n=1

We can solve for the cn by taking the inner product of m (x) and each side of the equation with respect to (x).
* +
X
hm ||f i = m cn n


n=1

X
hm ||f i = cn hm ||n i
n=1
hm ||f i = cm hm ||m i
hm ||f i
cm =
hm ||m i
The cm are known as Generalized Fourier coefficients. If the functions in the expansion are orthonormal, the formula
simplifies to
cm = hm ||f i.

1291
Example 25.8.4 The function f (x) = x( x) has a uniformly convergent series expansion in the domain [0, ] of
the form
r
X 2
x( x) = cn sin(nx).
n=1

The Fourier coefficients are


*r +
2
cn = sin(nx) x( x)


r Z
2
= x( x) sin(nx) dx
0
r
2 2
= (1 (1)n )
n3
(q
2 4
n3
for odd n
=
0 for even n

Thus the expansion is



X 8
x( x) = sin(nx) for x [0, ].
n=1
n3
oddn

In the first graph of Figure 25.2 the first term in the expansion is plotted in a dashed line and x( x) is plotted
in a solid line. The second graph shows the two term approximation.

Example 25.8.5 The set {. . . , 1/ 2 ex , 1/ 2, 1/ 2 ex , 1/ 2 e2x , . . .} is orthonormal on the interval [, ].

1292
2 2

1 1

1 2 3 1 2 3

Figure 25.2: Series Expansions of x( x).

f (x) = sign(x) has the expansion


* +
X 1 1
sign(x) en sign() enx

n=
2 2
Z
1 X
= en sign() d enx
2 n=
Z 0 Z 
1 X n n
= e d + e d enx
2 n= 0

1 X 1 (1)n nx
= e .
n= n

1293
In terms of real functions, this is

1 X 1 (1)n
= (cos(nx) + sin(nx))
n= n

2 X 1 (1)n
= sin(nx)
n=1 n


4X1
sign(x) sin(nx).
n=1 n
oddn

25.9 Least Squares Fit to a Function and Completeness


Let {1 , 2 , 3 , . . .} be a set of real, square integrable functions that are orthonormal with respect to the weighting
function (x) on the interval [a, b]. That is,
hn ||m i = nm .
Let f (x) be some square integrable function defined on the same interval. We would like to approximate the function
f (x) with a finite orthonormal series.
N
X
f (x) n n (x)
n=1

f (x) may or may not have a uniformly convergent expansion in the orthonormal functions.
We would like to choose the n so that we get the best possible approximation to f (x). The most common measure
of how well a series approximates a function is the least squares measure. The error is defined as the integral of the
weighting function times the square of the deviation.
Z b N
!2
X
E= (x) f (x) n n (x) dx
a n=1

1294
The best fit is found by choosing the n that minimize E. Let cn be the Fourier coefficients of f (x).

cn = hn ||f i

we expand the integral for E.

N
!2
Z b X
E() = (x) f (x) n n (x) dx
a n=1
 N
X
N
X 

= f n n

f
n n
n=1 n=1
N N
N
X  X X 

= hf ||f i 2 n n f + n n n n
n=1 n=1 n=1
N
X N
XX N
= hf ||f i 2 n hn ||f i + n m hn ||m i
n=1 n=1 m=1
XN N
X
= hf ||f i 2 n cn + n2
n=1 n=1
N
X N
X
2
= hf ||f i + (n cn ) c2n
n=1 n=1

Each term involving n in non-negative and is minimized for n = cn . The Fourier coefficients give the least squares
approximation to a function. The least squares fit to f (x) is thus

N
X
f (x) hn ||f i n (x).
n=1

1295
Result 25.9.1 If {1 , 2 , 3 , . . .} is a set of real, square integrable functions that are orthog-
onal with respect to (x) then the least squares fit of the first N orthogonal functions to the
square integrable function f (x) is
N
X hn ||f i
f (x) n (x).
n=1
h n ||n i

If the set is orthonormal, this formula reduces to


N
X
f (x) hn ||f i n (x).
n=1

Since the error in the approximation E is a nonnegative number we can obtain on inequality on the sum of the
squared coefficients.
N
X
E = hf ||f i c2n
n=1
N
X
c2n hf ||f i
n=1

equation is known as Bessels Inequality. Since hf ||f i is just a nonnegative number, independent of N , the
This P
sum 2
n=1 cn is convergent and cn 0 as n

Convergence in the Mean. If the error E goes to zero as N tends to infinity


Z b N
!2
X
lim (x) f (x) cn n (x) dx = 0,
N a n=1

1296
then the sum converges in the mean to f (x) relative to the weighting function (x). This implies that

N
!
X
lim hf ||f i c2n =0
N
n=1

X
c2n = hf ||f i.
n=1

This is known as Parsevals identity.

Completeness. Consider a set of functions {1 , 2 , 3 , . . .} that is orthogonal with respect to the weighting function
(x). If every function f (x) that is square integrable with respect to (x) has an orthogonal series expansion


X
f (x) cn n (x)
n=1

that converges in the mean to f (x), then the set is complete.

25.10 Closure Relation


Let {1 , 2 , . . .} be an orthonormal, complete set on the domain [a, b]. For any square integrable function f (x) we can
write
X
f (x) cn n (x).
n=1

1297
Here the cn are the generalized Fourier coefficients and the sum converges in the mean to f (x). Substituting the
expression for the Fourier coefficients into the sum yields

X
f (x) hn |f in (x)
n=1
X Z b 
= n ()f () d n (x).
n=1 a

Since the sum is not necessarily uniformly convergent, we are not justified in exchanging the order of summation and
integration. . . but what the heck, lets do it anyway.


!
Z b X
= n ()f ()n (x) d
a n=1

!
Z b X
= n ()n (x) f () d
a n=1

The sum behaves like a Dirac delta function. Recall that (x ) satisfies the equation
Z b
f (x) = (x )f () d for x (a, b).
a

Thus we could say that the sum is a representation of (x ). Note that a series representation of the delta function
could not be convergent, hence the necessity of throwing caution to the wind when we interchanged the summation
and integration in deriving the series. The closure relation for an orthonormal, complete set states

X
n (x)n () (x ).
n=1

1298
Alternatively, you can derive the closure relation by computing the generalized Fourier coefficients of the delta
function.
X
(x ) cn n (x)
n=1

cn = hn |(x )i
Z b
= n (x)(x ) dx
a
= n ()

X
(x ) n (x)n ()
n=1

Result 25.10.1 If {1 , 2 , . . .} is an orthogonal, complete set on the domain [a, b], then

X n (x)n ()
(x ).
n=1
kn k2

If the set is orthonormal, then



X
n (x)n () (x ).
n=1

Example 25.10.1 The integral of the Dirac delta function is the Heaviside function. On the interval x (, )
Z x (
1 for 0 < x <
(t) dt = H(x) =
0 for < x < 0.

1299
Consider the orthonormal, complete set {. . . , 12 ex , 12 , 12 ex , . . .} on the domain [, ]. The delta function
has the series

X 1 1 1 X nt
(t) ent en0 = e .
n=
2 2 2 n=

We will find the series expansion of the Heaviside function first by expanding directly and then by integrating the
expansion for the delta function.

Finding the series expansion of H(x) directly. The generalized Fourier coefficients of H(x) are

Z
1
c0 = H(x) dx
2
Z
1
= dx
2 0
r

=
2
Z
1
cn = enx H(x) dx
2
Z
1
= enx dx
2 0
1 (1)n
= .
n 2

1300
Thus the Heaviside function has the expansion
r
1 X 1 (1)n 1 nx
H(x) + e
2 2 n= n 2 2
n6=0

1 1 X 1 (1)n
= + sin(nx)
2 n=1 n


1 2X1
H(x) + sin(nx).
2 n=1 n
oddn

Integrating the series for (t).


Z x Z x X
1
(t) dt ent dt
2 n=

 x
1 X 1
= (x + ) + ent

2 in

n=
n6=0


1 X 1 nx
e (1)n

= (x + ) +
2 n=
n
n6=0

x 1 1 X 1 nx
e enx (1)n + (1)n

= + +
2 2 2 n=1
n

x 1 1X1
= + + sin(nx)
2 2 n=1 n

1301
x
Expanding 2
in the orthonormal set,

x X 1
cn enx .
2 n= 2

Z
1 x
c0 = dx = 0
2 2
Z

1 x (1)n
cn = enx dx =
2 2 n 2

x X (1)n 1 nx 1X
e = (1)n sin(nx)
2 n= n 2 2 n=1
n6=0
x
Substituting the series for 2
into the expression for the integral of the delta function,
x
1 1 X 1 (1)n
Z
(t) dt + sin(nx)
2 n=1 n

Z x
1 2X1
(t) dt + sin(nx).
2 n=1 n
oddn

Thus we see that the series expansions of the Heaviside function and the integral of the delta function are the same.

25.11 Linear Operators

1302
25.12 Exercises
Exercise 25.1
1. Suppose {k (x)}k=0 is an orthogonal system on [a, b]. Show that any finite set of the j (x) is a linearly
independent set on [a, b]. That is, if {j1 (x), j2 (x), . . . , jn (x)} is the set and all the j are distinct, then

a1 j1 (x) + a2 j2 (x) + + an jn (x) = 0 on a x b

is true iff: a1 = a2 = = an = 0.
RL
2. Show that the complex functions k (x) ekx/L , k = 0, 1, 2, . . . are orthogonal in the sense that L
k (x)n (x) dx =
0, for n 6= k. Here n (x) is the complex conjugate of n (x).
Hint, Solution

1303
25.13 Hints
Hint 25.1

1304
25.14 Solutions
Solution 25.1
1.

a1 j1 (x) + a2 j2 (x) + + an jn (x) = 0


X n
ak jk (x) = 0
k=1

Rb
We take the inner product with j for any = 1, . . . , n. (h, i a
(x) (x) dx.)
* n +
X
ak jk , j =0
k=1

We interchange the order of summation and integration.

n
X
ak hjk , j i = 0
k=1

hjk j i = 0 for j 6= .

a hj j i = 0

hj j i =
6 0.

a = 0

Thus we see that a1 = a2 = = an = 0.

1305
2. For k 6= n, hk , n i = 0.
Z L
hk , n i k (x)n (x) dx
L
Z L
= ekx/L enx/L dx
L
Z L
= e(kn)x/L dx
L
L
e(kn)x/L

=
(k n)/L L
e(kn) e(kn)
=
(k n)/L
2L sin((k n))
=
(k n)
=0

1306
Chapter 26

Self Adjoint Linear Operators

26.1 Adjoint Operators

The adjoint of an operator, L , satisfies

hv|Lui hL v|ui = 0

for all elements u an v. This is known as Greens Identity.

The adjoint of a matrix. For vectors, one can represent linear operators L with matrix multiplication.

Lx Ax

1307
Let B = A be the adjoint of the matrix A. We determine the adjoint of A from Greens Identity.

hx|Ayi hBx|yi = 0
x Ay = Bx y
T
xT Ay = Bx y
T
xT Ay = xT B y
T T
yT A x = yT BxB = A

T
Thus we see that the adjoint of a matrix is the conjugate transpose of the matrix, A = A . The conjugate transpose
is also called the Hermitian transpose and is denoted AH .

The adjoint of a differential operator. Consider a second order linear differential operator acting on C 2 functions
defined on (a . . . b) which satisfy certain boundary conditions.

Lu p2 (x)u00 + p1 (x)u0 + p0 (x)u

26.2 Self-Adjoint Operators


T
Matrices. A matrix is self-adjoint if it is equal to its conjugate transpose A = AH A . Such matrices are called
Hermitian. For a Hermitian matrix H, Greens identity is

hy|Hxi = hHy|xi
y Hx = Hy x

1308
The eigenvalues of a Hermitian matrix are real. Let x be an eigenvector with eigenvalue .

hx|Hxi = hHx|xi
hx|xi hx|xi = 0
( )hx|xi = 0
=

The eigenvectors corresponding to distinct eigenvalues are distinct. Let x and y be eigenvectors with distinct eigenvalues
and .

hy|Hxi = hHy|xi
hy|xi hy|xi = 0
( )hy|xi = 0
( )hy|xi = 0
hy|xi = 0

Furthermore, all Hermitian matrices are similar to a diagonal matrix and have a complete set of orthogonal eigenvectors.

Trigonometric Series. Consider the problem

y 00 = y, y(0) = y(2), y 0 (0) = y 0 (2).

1309
d2
We verify that the differential operator L = dx 2 with periodic boundary conditions is self-adjoint.

hv|Lui = hv| u00 i


2
= [vu0 ]0 hv 0 | u0 i
= hv 0 |u0 i
 2
= v 0 u 0 hv 00 |ui
= hv 00 |ui
= hLv|ui

The eigenvalues and eigenfunctions of this problem are

0 = 0, 0 = 1
n = n 2 , (1)
n = cos(nx), (2)
n = sin(nx), n Z+

1310
26.3 Exercises

1311
26.4 Hints

1312
26.5 Solutions

1313
Chapter 27

Self-Adjoint Boundary Value Problems

Seize the day and throttle it.

-Calvin

27.1 Summary of Adjoint Operators


The adjoint of the operator
dn y dn1 y
L[y] = pn + p n1 + + p0 y,
dxn dxn1
is defined
dn n1 d
n1
L [y] = (1)n (p n y) + (1) (pn1 y) + + p0 y
dxn dxn1
If each of the pk is k times continuously differentiable and u and v are n times continuously differentiable on some
interval, then on that interval Lagranges identity states

d
vL[u] uL [v] = B[u, v]
dx

1314
where B[u, v] is the bilinear form
n
X X
B[u, v] = (1)j u(k) (pm v)(j) .
m=1 j+k=m1
j0,k0

If L is a second order operator then

vL[u] uL [v] = u00 p2 v + u0 p1 v + u p2 v 00 + (2p02 + p1 )v 0 + (p002 + p01 )v .


 

Integrating Lagranges identity on its interval of validity gives us Greens formula.


Z b 
vL[u] uL [v] dx = hv|L[u]i hL [v]|ui = B[u, v] x=b B[u, v] x=a

a

27.2 Formally Self-Adjoint Operators


Example 27.2.1 The linear operator
L[y] = x2 y 00 + 2xy 0 + 3y

has the adjoint operator

d2 2 d
L [y] = 2 (x y) (2xy) + 3y
dx dx
= x2 y 00 + 4xy 0 + 2y 2xy 0 2y + 3y
= x2 y 00 + 2xy 0 + 3y.

In Example 27.2.1, the adjoint operator is the same as the operator. If L = L , the operator is said to be formally
self-adjoint.

1315
Most of the differential equations that we study in this book are second order, formally self-adjoint, with real-valued
coefficient functions. Thus we wish to find the general form of this operator. Consider the operator

L[y] = p2 y 00 + p1 y 0 + p0 y,

where the pj s are real-valued functions. The adjoint operator then is

d2 d
L [y] = 2
(p2 y) (p1 y) + p0 y
dx dx
= p2 y 00 + 2p02 y 0 + p002 y p1 y 0 p01 y + p0 y
= p2 y 00 + (2p02 p1 )y 0 + (p002 p01 + p0 )y.

Equating L and L yields the two equations,

2p02 p1 = p1 , p002 p01 + p0 = p0


p02 = p1 , p002 = p01 .

Thus second order, formally self-adjoint operators with real-valued coefficient functions have the form

L[y] = p2 y 00 + p02 y 0 + p0 y,

which is equivalent to the form


d
L[y] = (py 0 ) + qy.
dx

Any linear differential equation of the form

L[y] = y 00 + p1 y 0 + p0 y = f (x),

where each pj is j times continuously differentiable and real-valued, can be written as a formally self adjoint equation.
We just multiply by the factor, Z x
P (x)
e = exp( p1 () d)

1316
to obtain
exp [P (x)] (y 00 + p1 y 0 + p0 y) = exp [P (x)] f (x)
d
(exp [P (x)] y 0 ) + exp [P (x)] p0 y = exp [P (x)] f (x).
dx
Example 27.2.2 Consider the equation
1
y 00 + y 0 + y = 0.
x
Multiplying by the factor Z x 
1
exp d = elog x = x

will make the equation formally self-adjoint.
xy 00 + y 0 + xy = 0
d
(xy 0 ) + xy = 0
dx

Result 27.2.1 If L = L then the linear operator L is formally self-adjoint. Second order
formally self-adjoint operators have the form
d
L[y] = (py 0 ) + qy.
dx
Any differential equation of the form

L[y] = y 00 + p1 y 0 + p0 y = f (x),

where each pj is j times continuously differentiable and real-valued, can Rbe written as a
x
formally self adjoint equation by multiplying the equation by the factor exp( p1 () d).

1317
27.3 Self-Adjoint Problems
Consider the nth order formally self-adjoint equation L[y] = 0, on the domain a x b subject to the boundary
conditions, Bj [y] = 0 for j = 1, . . . , n. where the boundary conditions can be written
n
X
Bj [y] = jk y (k1) (a) + jk y (k1) (b) = 0.
k=1

If the boundary conditions are such that Greens formula reduces to


hv|L[u]i hL[v]|ui = 0
then the problem is self-adjoint

Example 27.3.1 Consider the formally self-adjoint equation y 00 = 0, subject to the boundary conditions y(0) =
y() = 0. Greens formula is
hv| u00 i hv 00 |ui = [u0 (v) u(v)0 ]0
= [uv 0 u0 v]0
= 0.
Thus this problem is self-adjoint.

27.4 Self-Adjoint Eigenvalue Problems


Associated with the self-adjoint problem
L[y] = 0, subject to Bj [y] = 0,
is the eigenvalue problem
L[y] = y, subject to Bj [y] = 0.
This is called a self-adjoint eigenvalue problem. The values of for which there exist nontrivial solutions to this problem
are called eigenvalues. The functions that satisfy the equation when is an eigenvalue are called eigenfunctions.

1318
Example 27.4.1 Consider the self-adjoint eigenvalue problem
y 00 = y, subject to y(0) = y() = 0.
First consider the case = 0. The general solution is
y = c1 + c2 x.
Only the trivial solution satisfies the boundary conditions. = 0 is not an eigenvalue. Now consider 6= 0. The
general solution is    
y = c1 cos x + c2 sin x .
The solution that satisfies the left boundary condition is
 
y = c sin x .
For non-trivial solutions, we must have  
sin = 0,
= n2 , n N.
Thus the eigenvalues n and eigenfunctions n are
n = n 2 , n = sin(nx), for n = 1, 2, 3, . . .
Self-adjoint eigenvalue problems have a number a interesting properties. We will devote the rest of this section to
developing some of these properties.

Real Eigenvalues. The eigenvalues of a self-adjoint problem are real. Let be an eigenvalue with the eigenfunction
. Greens formula states
h|L[]i hL[]|i = 0
h|i h|i = 0
( )h|i = 0
Since 6 0, h|i > 0. Thus = and is real.

1319
Orthogonal Eigenfunctions. The eigenfunctions corresponding to distinct eigenvalues are orthogonal. Let n and
m be distinct eigenvalues with the eigenfunctions n and m . Using Greens formula,
hn |L[m ]i hL[n ]|m i = 0
hn |m m i hn n |m i = 0
(m n )hn |m i = 0.

Since the eigenvalues are real,

(m n )hn |m i = 0.
Since the two eigenvalues are distinct, hn |m i = 0 and thus n and m are orthogonal.

*Enumerable Set of Eigenvalues. The eigenvalues of a self-adjoint eigenvalue problem form an enumerable set
with no finite cluster point. Consider the problem
L[y] = y on a x b, subject to Bj [y] = 0.
Let {1 , 2 , . . . , n } be a fundamental set of solutions at x = x0 for some a x0 b. That is,
(k1)
j (x0 ) = jk .
The key to showing that the eigenvalues are enumerable, is that the j are entire functions of . That is, they are
analytic functions of for all finite . We will not prove this.
The boundary conditions are
n
X
jk y (k1) (a) + jk y (k1) (b) = 0.
 
Bj [y] =
k=1

The eigenvalue problem has a solution for a given value of if y = nk=1 ck k satisfies the boundary conditions. That
P
is, " n #
X Xn
Bj ck k = ck Bj [k ] = 0 for j = 1, . . . , n.
k=1 k=1

1320
Define an n n matrix M such that Mjk = Bk [j ]. Then if ~c = (c1 , c2 , . . . , cn ), the boundary conditions can be
written in terms of the matrix equation M~c = 0. This equation has a solution if and only if the determinant of the
matrix is zero. Since the j are entire functions of , [M ] is an entire function of . The eigenvalues are real, so
[M ] has only real roots. Since [M ] is an entire function, (that is not identically zero), with only real roots, the
roots of [M ] can only cluster at infinity. Thus the eigenvalues of a self-adjoint problem are enumerable and can only
cluster at infinity.
An example of a function whose roots have a finite cluster point is sin(1/x). This function, (graphed in Figure 27.1),
is clearly not analytic at the cluster point x = 0.

-1 1

Figure 27.1: Graph of sin(1/x).

1321
Infinite Number of Eigenvalues. Though we will not show it, self-adjoint problems have an infinite number of
eigenvalues. Thus the eigenfunctions form an infinite orthogonal set.

Eigenvalues of Second Order Problems. Consider the second order, self-adjoint eigenvalue problem

L[y] = (py 0 )0 + qy = y, on a x b, subject to Bj [y] = 0.

Let n be an eigenvalue with the eigenfunction n .

hn |L[n ]i = hn |n n i
hn |(p0n )0 + qn i = n hn |n i
Z b
n (p0n )0 dx + hn |q|n i = n hn |n i
a
Z b
0 b 0
n p0n dx + hn |q|n i = n hn |n i
 
n pn a
a

[pn 0n ]ba h0n |p|0n i + hn |q|n i


n =
hn |n i

Thus we can express each eigenvalue in terms of its eigenfunction. You might think that this formula is just a
shade less than worthless. When solving an eigenvalue problem you have to find the eigenvalues before you determine
the eigenfunctions. Thus this formula could not be used to compute the eigenvalues. However, we can often use the
formula to obtain information about the eigenvalues before we solve a problem.

Example 27.4.2 Consider the self-adjoint eigenvalue problem

y 00 = y, y(0) = y() = 0.

1322
The eigenvalues are given by the formula
b
(1)0 a h0n |(1)|0n i + hn |0|n i

n =
hn |n i
0 0
0 + hn |n i + 0
= .
hn |n i

We see that n 0. If n = 0 then h0n |0n i = 0,which implies that n = const. The only constant that satisfies the
boundary conditions is n = 0 which is not an eigenfunction since it is the trivial solution. Thus the eigenvalues are
positive.

27.5 Inhomogeneous Equations


Let the problem,
L[y] = 0, Bk [y] = 0,
be self-adjoint. If the inhomogeneous problem,

L[y] = f, Bk [y] = 0,

has a solution, then we we can write this solution in terms of the eigenfunction of the associated eigenvalue problem,

L[y] = y, Bk [y] = 0.

We denote the eigenvalues as n and the eigenfunctions as n for n Z+ . For the moment we assume that
= 0 is not an eigenvalue and that the eigenfunctions are real-valued. We expand the function f (x) in a series of the
eigenfunctions.
X hn |f i
f (x) = fn n (x), fn =
kn k

1323
We expand the inhomogeneous solution in a series of eigenfunctions and substitute it into the differential equation.

L[y] = f
hX i X
L yn n (x) = fn n (x)
X X
n yn n (x) = fn n (x)
fn
yn =
n

The inhomogeneous solution is


X hn |f i
y(x) = n (x). (27.1)
n kn k

As a special case we consider the Green function problem,

L[G] = (x ), Bk [G] = 0,

We expand the Dirac delta function in an eigenfunction series.

X hn |i X n ()n (x)
(x ) = n (x) =
kn k kn k

The Green function is


X n ()n (x)
G(x|) = .
n kn k

1324
We corroborate Equation 27.1 by solving the inhomogeneous equation in terms of the Green function.
Z b
y= G(x|)f () d
a
Z bX
n ()n (x)
y= f () d
a n kn k
X b n ()f () d
R
a
y= n (x)
n kn k
X hn |f i
y= n (x)
n kn k

Example 27.5.1 Consider the Green function problem

G00 + G = (x ), G(0|) = G(1|) = 0.

First we examine the associated eigenvalue problem.

00 + = , (0) = (1) = 0
00 + (1 ) = 0, (0) = (1) = 0
n = 1 (n)2 , n = sin(nx), n Z+

We write the Green function as a series of the eigenfunctions.



X sin(n) sin(nx)
G(x|) = 2
n=1
1 (n)2

1325
27.6 Exercises
Exercise 27.1
Show that the operator adjoint to

Ly = y (n) + p1 (z)y (n1) + p2 (z)y (n2) + + pn (z)y

is given by
M y = (1)n u(n) + (1)n1 (p1 (z)u)(n1) + (1)n2 (p2 (z)u)(n2) + + pn (z)u.
Hint, Solution

1326
27.7 Hints
Hint 27.1

1327
27.8 Solutions
Solution 27.1
Consider u(x), v(x) C n . (C n is the set of n times continuously differentiable functions). First we prove the preliminary
result
n1
(n) n (n) d X
uv (1) u v = (1)k u(k) v (nk1) (27.2)
dx k=0
by simplifying the right side.
n1 n1
d X X
(1)k u(k) v (nk1) = (1)k u(k) v (nk) + u(k+1) v (nk1)

dx k=0 k=0
n1
X n1
X
k (k) (nk)
= (1) u v (1)k+1 u(k+1) v (nk1)
k=0 k=0
n1
X Xn
= (1)k u(k) v (nk) (1)k u(k) v (nk)
k=0 k=1
0 (0) n0 n (n) (nn)
= (1) u v (1) u v
= uv (n) (1)n u(n) v
We define p0 (x) = 1 so that we can write the operators in a nice form.
Xn Xn
Ly = pm (z)y (nm) , M u = (1)m (pm (z)u)(nm)
m=0 m=0
Now we show that M is the adjoint to L.
n
X n
X
(nm)
uLy yM u = u pm (z)y y (1)m (pm (z)u)(nm)
m=0 m=0
n
X
upm (z)y (nm) (pm (z)u)(nm) y

=
m=0

1328
We use Equation 27.2.

n nm1
X d X
= (1)k (upm (z))(k) y (nmk1)
m=0
dz k=0

n nm1
d X X
uLy yM u = (1)k (upm (z))(k) y (nmk1)
dz m=0 k=0

1329
Chapter 28

Fourier Series

Every time I close my eyes


The noise inside me amplifies
I cant escape
I relive every moment of the day
Every misstep I have made
Finds a way it can invade
My every thought
And this is why I find myself awake

-Failure
-Tom Shear (Assemblage 23)

28.1 An Eigenvalue Problem.


A self adjoint eigenvalue problem. Consider the eigenvalue problem

y 00 + y = 0, y() = y(), y 0 () = y 0 ().

1330
We rewrite the equation so the eigenvalue is on the right side.
L[y] y 00 = y
We demonstrate that this eigenvalue problem is self adjoint.
hv|L[u]i hL[v]|ui = hv| u00 i hv 00 |ui
= [vu0 ] + hv 0 |u0 i [v 0 u] hv 0 |u0 i
= v()u0 () + v()u0 () + v 0 ()u() v 0 ()u()
= v()u0 () + v()u0 () + v 0 ()u() v 0 ()u()
=0
Since Greens Identity reduces to hv|L[u]i hL[v]|ui = 0, the problem is self adjoint. This means that the eigenvalues
are real and that eigenfunctions corresponding to distinct eigenvalues are orthogonal. We compute the Rayleigh quotient
for an eigenvalue with eigenfunction .
[0 ] + h0 |0 i
=
h|i
()0 () + ()0 () + h0 |0 i
=
h|i
()0 () + ()0 () + h0 |0 i
=
h|i
0 0
h | i
=
h|i
We see that the eigenvalues are non-negative.

Computing the eigenvalues and eigenfunctions. Now we find the eigenvalues and eigenfunctions. First we
consider the case = 0. The general solution of the differential equation is
y = c1 + c2 x.

1331
The solution that satisfies the boundary conditions is y = const.
Now consider > 0. The general solution of the differential equation is
   
y = c1 cos x + c2 sin x .
We apply the first boundary condition.
y() = y()
       
c1 cos + c2 sin = c1 cos + c2 sin
       
c1 cos c2 sin = c1 cos + c2 sin
 
c2 sin = 0
Then we apply the second boundary condition.
y 0 () = y 0 ()
       
c1 sin + c2 cos = c1 sin + c2 cos
       
c1 sin + c2 cos = c1 sin + c2 cos
 
c1 sin = 0
 
To satisify the two boundary conditions either c1 = c2 = 0 or sin = 0. The former yields the trivial solution.
The latter gives us the eigenvalues n = n2 , n Z+ . The corresponding solution is
yn = c1 cos(nx) + c2 sin(nx).
There are two eigenfunctions for each of the positive eigenvalues.
We choose the eigenvalues and eigenfunctions.
1
0 = 0, 0 =
2
2
n = n , 2n1 = cos(nx), 2n = sin(nx), for n = 1, 2, 3, . . .

1332
Orthogonality of Eigenfunctions. We know that the eigenfunctions of distinct eigenvalues are orthogonal. In
addition, the two eigenfunctions of each positive eigenvalue are orthogonal.
Z  
1 2
cos(nx) sin(nx) dx = sin (nx) =0
2n

Thus the eigenfunctions { 21 , cos(x), sin(x), cos(2x), sin(2x)} are an orthogonal set.

28.2 Fourier Series.


A series of the eigenfunctions

1
0 = , (1)
n = cos(nx), (2)
n = sin(nx), for n 1
2

is

1 X 
a0 + an cos(nx) + bn sin(nx) .
2 n=1

This is known as a Fourier series. (We choose 0 = 21 so all of the eigenfunctions have the same norm.) A fairly general
class of functions can be expanded in Fourier series. Let f (x) be a function defined on < x < . Assume that
f (x) can be expanded in a Fourier series


1 X 
f (x) a0 + an cos(nx) + bn sin(nx) . (28.1)
2 n=1

Here the means has the Fourier series. We have not said if the series converges yet. For now lets assume that
the series converges uniformly so we can replace the with an =.

1333
We integrate Equation 28.1 from to to determine a0 .
Z Z Z
1 X
f (x) dx = a0 dx + an cos(nx) + bn sin(nx) dx
2 n=1
Z 
X Z Z 
f (x) dx = a0 + an cos(nx) dx + bn sin(nx) dx
n=1
Z
f (x) dx = a0

1
Z
a0 = f (x) dx

Multiplying by cos(mx) and integrating will enable us to solve for am .


Z Z
1
f (x) cos(mx) dx = a0 cos(mx) dx
2

X Z Z 
+ an cos(nx) cos(mx) dx + bn sin(nx) cos(mx) dx
n=1

All but one of the terms on the right side vanishes due to the orthogonality of the eigenfunctions.
Z Z
f (x) cos(mx) dx = am cos(mx) cos(mx) dx

Z Z  
1
f (x) cos(mx) dx = am + cos(2mx) dx
2
Z
f (x) cos(mx) dx = am

1
Z
am = f (x) cos(mx) dx.

1334
Note that this formula is valid for m = 0, 1, 2, . . ..
Similarly, we can multiply by sin(mx) and integrate to solve for bm . The result is
1
Z
bm = f (x) sin(mx) dx.

an and bn are called Fourier coefficients.
Although we will not show it, Fourier series converge for a fairly general class of functions. Let f (x ) denote the
left limit of f (x) and f (x+ ) denote the right limit.

Example 28.2.1 For the function defined


(
0 for x < 0,
f (x) =
x+1 for x 0,
the left and right limits at x = 0 are
f (0 ) = 0, f (0+ ) = 1.

R
Result 28.2.1 Let f (x) be a 2-periodic function for which |f (x)| dx exists. Define the
Fourier coefficients
1 1
Z Z
an = f (x) cos(nx) dx, bn = f (x) sin(nx) dx.

If x is an interior point of an interval on which f (x) has limited total fluctuation, then the
Fourier series of f (x)

a0 X 
+ an cos(nx) + bn sin(nx) ,
2 n=1

converges to 21 (f (x ) + f (x+ )). If f is continuous at x, then the series converges to f (x).

1335
Periodic Extension of a Function. Let g(x) be a function that is arbitrarily defined on x < . The
Fourier series of g(x) will represent the periodic extension of g(x). The periodic extension, f (x), is defined by the two
conditions:
f (x) = g(x) for x < ,
f (x + 2) = f (x).
The periodic extension of g(x) = x2 is shown in Figure 28.1.

10

-5 5 10
-2

Figure 28.1: The Periodic Extension of g(x) = x2 .

Limited Fluctuation. A function that has limited total fluctuation can be written f (x) = + (x) (x), where
+ and are bounded, nondecreasing functions. An example of a function that does not have limited total fluctuation

1336
is sin(1/x), whose fluctuation is unlimited at the point x = 0.

Functions with Jump Discontinuities. Let f (x) be a discontinuous function that has a convergent Fourier
series. Note that the series does not necessarily converge to f (x). Instead it converges to f(x) = 12 (f (x ) + f (x+ )).

Example 28.2.2 Consider the function defined by


(
x for x < 0
f (x) =
2x for 0 x < .
The Fourier series converges to the function defined by


0 for x =

x for <x<0
f(x) =


/2 for x=0
2x

for 0 < x < .

The function f(x) is plotted in Figure 28.2.

28.3 Least Squares Fit


Approximating a function with a Fourier series. Suppose we want to approximate a 2-periodic function
f (x) with a finite Fourier series.
N
a0 X
f (x) + (an cos(nx) + bn sin(nx))
2 n=1
Here the coefficients are computed with the familiar formulas. Is this the best approximation to the function? That is,
is it possible to choose coefficients n and n such that
N
0 X
f (x) + (n cos(nx) + n sin(nx))
2 n=1

1337
3

-3 -2 -1 1 2 3

-1

-2

-3

Figure 28.2: Graph of f(x).

would give a better approximation?

Least squared error fit. The most common criterion for finding the best fit to a function is the least squares fit.
The best approximation to a function is defined as the one that minimizes the integral of the square of the deviation.
Thus if f (x) is to be approximated on the interval a x b by a series

N
X
f (x) cn n (x), (28.2)
n=1

1338
the best approximation is found by choosing values of cn that minimize the error E.
Z b 2
XN
E f (x) cn n (x) dx

a n=1

Generalized Fourier coefficients. We consider the case that the n are orthogonal. For simplicity, we also
assume that the n are real-valued. Then most of the terms will vanish when we interchange the order of integration
and summation.
Z b N N N
!
X X X
E= f 2 2f cn n + cn n cm m dx
a n=1 n=1 m=1
Z b N
X Z b N
XXN Z b
2
E= f dx 2 cn f n dx + cn cm n m dx
a n=1 a n=1 m=1 a
Z b N
X Z b N
X Z b
2
E= f dx 2 cn f n dx + c2n 2n dx
a n=1 a n=1 a
Z b N 
X Z b Z b 
2
E= f dx + c2n 2n dx 2cn f n dx
a n=1 a a

We complete the square for each term.


Rb !2 Rb !2
b N b
f n dx f n dx
Z X Z
E= f 2 dx + 2n dx cn Ra b Ra b
a n=1 a
a n
2 dx a n
2 dx

Each term involving cn is non-negative, and is minimized for


Rb
f n dx
cn = Ra b . (28.3)
2 dx
a n

1339
We call these the generalized Fourier coefficients.
For such a choice of the cn , the error is

Z b N
X Z b
2
E= f dx c2n 2n dx.
a n=1 a

Since the error is non-negative, we have

Z b N
X Z b
2
f dx c2n 2n dx.
a n=1 a

This is known as Bessels Inequality. If the series in Equation 28.2 converges in the mean to f (x), lim N E = 0,
then we have equality as N .
Z b X Z b
2 2
f dx = cn 2n dx.
a n=1 a

This is Parsevals equality.

Fourier coefficients. Previously we showed that if the series,


a0 X
f (x) = + (an cos(nx) + bn sin(nx),
2 n=1

converges uniformly then the coefficients in the series are the Fourier coefficients,
Z Z
1 1
an = f (x) cos(nx) dx, bn = f (x) sin(nx) dx.

1340
Now we show that by choosing the coefficients to minimize the squared error, we obtain the same result. We apply
Equation 28.3 to the Fourier eigenfunctions.
R
f 1 dx
Z
2 1
a0 = R 1 = f (x) dx
4
dx
R
f cos(nx) dx
Z
1
an = R
= f (x) cos(nx) dx

cos2 (nx) dx
R
f sin(nx) dx
Z
1
bn = R
= f (x) sin(nx) dx

sin2 (nx) dx

28.4 Fourier Series for Functions Defined on Arbitrary Ranges


If f (x) is defined on c d x < c + d and f (x + 2d) = f (x), then f (x) has a Fourier series of the form
   
a0 X n(x + c) n(x + c)
f (x) + an cos + bn sin .
2 n=1
d d

Since
Z c+d   Z c+d  
2 n(x + c) 2 n(x + c)
cos dx = sin dx = d,
cd d cd d
the Fourier coefficients are given by the formulas
Z c+d 
1 n(x + c)
an = f (x) cos dx
d cd d
1 c+d
Z  
n(x + c)
bn = f (x) sin dx.
d cd d

1341
Example 28.4.1 Consider the function defined by

x + 1
for 1 x < 0
f (x) = x for 0 x < 1

3 2x for 1 x < 2.

This function is graphed in Figure 28.3.


The Fourier series converges to f(x) = (f (x ) + f (x+ ))/2,


12 for x = 1

x + 1 for 1<x<0




f (x) = 21
for x=0

x for 0<x<1





3 2x for 1 x < 2.

f(x) is also graphed in Figure 28.3.


The Fourier coefficients are
Z 2  
1 2n(x + 1/2)
an = f (x) cos dx
3/2 1 3
2 5/2
Z  
2nx
= f (x 1/2) cos dx
3 1/2 3
2 1/2 2 3/2
Z   Z  
2nx 2nx
= (x + 1/2) cos dx + (x 1/2) cos dx
3 1/2 3 3 1/2 3
2 5/2
Z  
2nx
+ (4 2x) cos dx
3 3/2 3
 
1 2n h n
 n i
= sin 2(1) n + 9 sin
(n)2 3 3

1342
1 1

0.5 0.5

-1 -0.5 0.5 1 1.5 2 -1 -0.5 0.5 1 1.5 2

-0.5 -0.5

-1 -1

Figure 28.3: A Function Defined on the range 1 x < 2 and the Function to which the Fourier Series Converges.
Z 2  
1 2n(x + 1/2)
bn = f (x) sin dx
3/2 1 3
2 5/2
Z  
2nx
= f (x 1/2) sin dx
3 1/2 3
2 1/2 2 3/2
Z   Z  
2nx 2nx
= (x + 1/2) sin dx + (x 1/2) sin dx
3 1/2 3 3 1/2 3
2 5/2
Z  
2nx
+ (4 2x) sin dx
3 3/2 3
2 2 n
 h
n
 n   n i
= sin 2(1) n + 4n cos 3 sin
(n)2 3 3 3

1343
28.5 Fourier Cosine Series
If f (x) is an even function, (f (x) = f (x)), then there will not be any sine terms in the Fourier series for f (x). The
Fourier sine coefficient is
1
Z
bn = f (x) sin(nx) dx.

Since f (x) is an even function and sin(nx) is odd, f (x) sin(nx) is odd. bn is the integral of an odd function from
to and is thus zero. We can rewrite the cosine coefficients,

1
Z
an = f (x) cos(nx) dx

2
Z
= f (x) cos(nx) dx.
0

Example 28.5.1 Consider the function defined on [0, ) by


(
x for 0 x < /2
f (x) =
x for /2 x < .

The Fourier cosine coefficients for this function are


Z /2 Z
2 2
an = x cos(nx) dx + ( x) cos(nx) dx
0 /2
(

4
for n = 0,
= 8 n
 2 n

n2
cos 2
sin 4
for n 1.

In Figure 28.4 the even periodic extension of f (x) is plotted in a dashed line and the sum of the first five nonzero terms
in the Fourier cosine series are plotted in a solid line.

1344
1.5

1.25

0.75

0.5

0.25

-3 -2 -1 1 2 3

Figure 28.4: Fourier Cosine Series.

28.6 Fourier Sine Series

If f (x) is an odd function, (f (x) = f (x)), then there will not be any cosine terms in the Fourier series. Since
f (x) cos(nx) is an odd function, the cosine coefficients will be zero. Since f (x) sin(nx) is an even function,we can
rewrite the sine coefficients

Z
2
bn = f (x) sin(nx) dx.
0

1345
Example 28.6.1 Consider the function defined on [0, ) by
(
x for 0 x < /2
f (x) =
x for /2 x < .
The Fourier sine coefficients for this function are
2 /2 2
Z Z
bn = x sin(nx) dx + ( x) sin(nx) dx
0 /2
16  n 
3 n
 
= cos sin
n2 4 4
In Figure 28.5 the odd periodic extension of f (x) is plotted in a dashed line and the sum of the first five nonzero terms
in the Fourier sine series are plotted in a solid line.

28.7 Complex Fourier Series and Parsevals Theorem


By writing sin(nx) and cos(nx) in terms of enx and enx we can obtain the complex form for a Fourier series.
 
a0 X  a0 X 1 nx nx 1 nx nx
+ an cos(nx) + bn sin(nx) = + an (e + e ) + bn (e e )
2 n=1
2 n=1
2 2
 
a0 X 1 nx 1 nx
= + (an bn ) e + (an + bn ) e
2 n=1
2 2

X
= cn enx
n=

where
1
2 (an bn )
for n 1
a0
cn = 2
for n = 0

1
(a
2 n
+ bn ) for n 1.

1346
1.5

0.5

-3 -2 -1 1 2 3

-0.5

-1

-1.5

Figure 28.5: Fourier Sine Series.

The functions {. . . , ex , 1, ex , e2x , . . .}, satisfy the relation


Z Z
nx mx
e e dx = e(nm)x dx

(
2 for n = m
=
0 for n 6= m.

Starting with the complex form of the Fourier series of a function f (x),

X
f (x) cn enx ,

1347
we multiply by emx and integrate from to to obtain
Z Z
X
mx
f (x) e dx = cn enx emx dx

Z
1
cm = f (x) emx dx
2

If f (x) is real-valued then


Z Z
1 1
cm = f (x) emx dx = f (x)(emx ) dx = cm
2 2

where z denotes the complex conjugate of z.


Assume that f (x) has a uniformly convergent Fourier series.


!
!
Z Z X X
f 2 (x) dx = cm emx cn enx dx
m= n=

X
= 2 cn cn
n=
1    !
X 1 a0 a0 X 1
= 2 (an + bn )(an bn ) + + (an bn )(an + bn )
n=
4 2 2 n=1
4

!
a20 1 X 2
= 2 + (a + b2n )
4 2 n=1 n

This yields a result known as Parsevals theorem which holds even when the Fourier series of f (x) is not uniformly
convergent.

1348
Result 28.7.1 Parsevals Theorem. If f (x) has the Fourier series

a0 X
f (x) + (an cos(nx) + bn sin(nx)),
2 n=1

then
Z
2 2 X
f (x) dx = a0 + (a2n + b2n ).
2 n=1

28.8 Behavior of Fourier Coefficients


Before we jump hip-deep into the grunge involved in determining the behavior of the Fourier coefficients, lets take a
step back and get some perspective on what we should be looking for.
One of the important questions is whether the Fourier series converges uniformly. From Result 12.2.1 we know that
a uniformly convergent series represents a continuous function. Thus we know that the Fourier series of a discontinuous
function cannot be uniformly convergent. From Section 12.2 we know that a series is uniformly convergent if it can be

bounded by a series of positive
Pterms. If the Fourier coefficients, an and bn , are O(1/n ) where > 1 then the series
can be bounded by (const) n=1 1/n and will thus be uniformly convergent.

Let f (x) be a function that meets the conditions for having a Fourier series and in addition is bounded. Let
(, p1 ), (p1 , p2 ), (p2 , p3 ), . . . , (pm , ) be a partition into a finite number of intervals of the domain, (, ) such that
on each interval f (x) and all its derivatives are continuous. Let f (p ) denote the left limit of f (p) and f (p+ ) denote
the right limit.
f (p ) = lim+ f (p ), f (p+ ) = lim+ f (p + )
0 0

Example 28.8.1 The function shown in Figure 28.6 would be partitioned into the intervals
(2, 1), (1, 0), (0, 1), (1, 2).

1349
1

0.5

-2 -1 1 2

-0.5

-1

Figure 28.6: A Function that can be Partitioned.

Suppose f (x) has the Fourier series



a0 X
f (x) + an cos(nx) + bn sin(nx).
2 n=1

We can use the integral formula to find the an s.


1
Z
an = f (x) cos(nx) dx

Z p1 Z p2 Z 
1
= f (x) cos(nx) dx + f (x) cos(nx) dx + + f (x) cos(nx) dx
p1 pm

1350
Using integration by parts,

h i 
1 ip1 h ip2 h
= f (x) sin(nx) + f (x) sin(nx) + + f (x) sin(nx)
n p1 pm
Z p1 Z p2 Z 
1 0 0 0
f (x) sin(nx) dx + f (x) sin(nx) dx + f (x) sin(nx) dx
n p1 pm
1 n +
  +
 o
= f (p1 ) f (p1 ) sin(np1 ) + + f (pm ) f (pm ) sin(npm )
n
11 0
Z
f (x) sin(nx) dx
n
1 1
= An b0n
n n

where
m
1X
sin(npj ) f (p +
 
An = j ) f (pj )
j=1

and the b0n are the sine coefficients of f 0 (x).


Since f (x) is bounded, An = O(1). Since f 0 (x) is bounded,

Z
1
b0n = f 0 (x) sin(nx) dx = O(1).

Thus an = O(1/n) as n . (Actually, from the Riemann-Lebesgue Lemma, b0n = O(1/n).)

1351
Now we repeat this analysis for the sine coefficients.

1
Z
bn = f (x) sin(nx) dx

Z p1 Z p2 Z 
1
= f (x) sin(nx) dx + f (x) sin(nx) dx + + f (x) sin(nx) dx
p1 pm
1 n p1  p2   o
= f (x) cos(nx) + f (x) cos(nx) p1 + + f (x) cos(nx) pm
n Z p1 Z p2 Z 
1 0 0 0
+ f (x) cos(nx) dx + f (x) cos(nx) dx + f (x) cos(nx) dx
n p1 pm
1 1
= Bn + a0n
n n
where
m
(1)n   1X
cos(npj ) f (p +
 
Bn = f () f () j ) f (p j )
j=1

and the a0n are the cosine coefficients of f 0 (x).


Since f (x) and f 0 (x) are bounded, Bn , a0n = O(1) and thus bn = O(1/n) as n .
With integration by parts on the Fourier coefficients of f 0 (x) we could find that

1 0 1
a0n = An b00n
n n
Pm
where A0n = 1
j=1 sin(npj )[f 0 (p 0 + 00 00
j ) f (pj )] and the bn are the sine coefficients of f (x), and

1 1
b0n = Bn0 + a00n
n n
(1)n Pm
where Bn0 =
[f 0 () f 0 ()] 1 j=1 cos(npj )[f 0 (p 0 + 00 00
j ) f (pj )] and the an are the cosine coefficients of f (x).

1352
Now we can rewrite an and bn as
1 1 1
an = An + 2 Bn0 2 a00n
n n n
1 1 0 1
bn = Bn + 2 An 2 b00n .
n n n
(j) (j)
Continuing this process we could define An and Bn so that
1 1 1 1
an = An + 2 Bn0 3 A00n 4 Bn000 +
n n n n
1 1 0 1 00 1
bn = Bn + 2 An + 3 Bn 4 A000 .
n n n n n
For any bounded function, the Fourier coefficients satisfy an , bn = O(1/n) as n . If An and Bn are zero
then the Fourier coefficients will be O(1/n2 ). A sufficient condition for this is that the periodic extension of f (x) is
continuous. We see that if the periodic extension of f 0 (x) is continuous then A0n and Bn0 will be zero and the Fourier
coefficients will be O(1/n3 ).

Result 28.8.1 Let f (x) be a bounded function for which there is a partition of the range
(, ) into a finite number of intervals such that f (x) and all its derivatives are continuous
on each of the intervals. If f (x) is not continuous then the Fourier coefficients are O(1/n).
If f (x), f 0 (x), . . . , f (k2) (x) are continuous then the Fourier coefficients are O(1/nk ).
2
P
If the periodic extension
P of f (x) is continuous, then the Fourier coefficients will be O(1/n ). The series n=1 |an cos(nx)
2
can be bounded by M n=1 1/n where M = max(|an | + |bn |). Thus the Fourier series converges to f (x) uniformly.
n

Result 28.8.2 If the periodic extension of f (x) is continuous then the Fourier series of f (x)
will converge uniformly for all x.
If the periodic extension of f (x) is not continuous, we have the following result.

1353
Result 28.8.3 If f (x) is continuous in the interval c < x < d, then the Fourier series is
uniformly convergent in the interval c + x d for any > 0.

Example 28.8.2 Different Rates of Convergence.

A Discontinuous Function. Consider the function defined by


(
1 for 1 < x < 0
f1 (x) =
1, for 0 < x < 1.

This function has jump discontinuities, so we know that the Fourier coefficients are O(1/n).
Since this function is odd, there will only be sine terms in its Fourier expansion. Furthermore, since the function is
symmetric about x = 1/2, there will be only odd sine terms. Computing these terms,
Z 1
bn = 2 sin(nx) dx
0
 1
1
=2 cos(nx)
n 0
(1)n 1
 
=2
n n
(
4
for odd n
= n
0 for even n.

The function and the sum of the first three terms in the expansion are plotted, in dashed and solid lines respectively,
in Figure 28.7. Although the three term sum follows the general shape of the function, it is clearly not a good
approximation.

1354
1 0.4

0.5 0.2

-1 -0.5 0.5 1 -1 -0.5 0.5 1

-0.5 -0.2

-1 -0.4

Figure 28.7: Three Term Approximation for a Function with Jump Discontinuities and a Continuous Function.

A Continuous Function. Consider the function defined by


x 1
for 1 < x < 1/2
f2 (x) = x for 1/2 < x < 1/2

x + 1 for 1/2 < x < 1.

1355
1 0.5
0.2

0.1 1 1

-1 -0.5 0.5 1 0.25 0.1

-0.1
1
-0.2
0
0.1

Figure 28.8: Three Term Approximation for a Function with Continuous First Derivative and Comparison of the
Rates of Convergence.

Since this function is continuous, the Fourier coefficients will be O(1/n2 ). Also we see that there will only be odd sine
terms in the expansion.
Z 1/2 Z 1/2 Z 1
bn = (x 1) sin(nx) dx + x sin(nx) dx + (x + 1) sin(nx) dx
1 1/2 1/2
Z 1/2 Z 1
=2 x sin(nx) dx + 2 (1 x) sin(nx) dx
0 1/2
4
= sin(n/2)
(n)2
(
4
(n)2
(1)(n1)/2 for odd n
=
0 for even n.
1356
The function and the sum of the first three terms in the expansion are plotted, in dashed and solid lines respectively,
in Figure 28.7. We see that the convergence is much better than for the function with jump discontinuities.

A Function with a Continuous First Derivative. Consider the function defined by


(
x(1 + x) for 1 < x < 0
f3 (x) =
x(1 x) for 0 < x < 1.

Since the periodic extension of this function is continuous and has a continuous first derivative, the Fourier coefficients
will be O(1/n3 ). We see that the Fourier expansion will contain only odd sine terms.

Z 0 Z 1
bn = x(1 + x) sin(nx) dx + x(1 x) sin(nx) dx
1 0
Z 1
=2 x(1 x) sin(nx) dx
0
4(1 (1)n )
=
(n)3
(
4
(n)3
for odd n
=
0 for even n.

The function and the sum of the first three terms in the expansion are plotted in Figure 28.8. We see that the first
three terms give a very good approximation to the function. The plots of the function, (in a dashed line), and the three
term approximation, (in a solid line), are almost indistinguishable.
In Figure 28.8 the convergence of the of the first three terms to f1 (x), f2 (x), and f3 (x) are compared. In the last
graph we see a closeup of f3 (x) and its Fourier expansion to show the error.

1357
28.9 Gibbs Phenomenon
The Fourier expansion of
(
1 for 0 x < 1
f (x) =
1 for 1 x < 0

is

4X1
f (x) sin(nx).
n=1 n

For any fixed x, the series converges to 21 (f (x ) + f (x+ )). For any > 0, the convergence is uniform in the intervals
1 + x and x 1 . How will the nonuniform convergence at integral values of x affect the Fourier
series? Finite Fourier series are plotted in Figure 28.9 for 5, 10, 50 and 100 terms. (The plot for 100 terms is closeup
of the behavior near x = 0.) Note that at each discontinuous point there is a series of overshoots and undershoots
that are pushed closer to the discontinuity by increasing the number of terms, but do not seem to decrease in height.
In fact, as the number of terms goes to infinity, the height of the overshoots and undershoots does not vanish. This is
known as Gibbs phenomenon.

28.10 Integrating and Differentiating Fourier Series


Integrating Fourier Series. Since integration is a smoothing operation, any convergent Fourier series can be
integrated term by term to yield another convergent Fourier series.

Example 28.10.1 Consider the step function


(
for 0 x <
f (x) =
for x < 0.

1358
1 1

1 1

1 1.2

1 0.1

0.8

Figure 28.9:

Since this is an odd function, there are no cosine terms in the Fourier series.
2
Z
bn = sin(nx) dx
0
 
1
= 2 cos(nx)
n 0
2
= (1 (1)n )
(n
4
for odd n
= n
0 for even n.

1359

X 4
f (x) sin nx
n=1
n
oddn

Integrating this relation,


Z x Z x
X 4
f (t) dt sin(nt) dt
n=1
n
oddn
Z x
X 4
F (x) sin(nt) dt
n=1
n
oddn
 x
X 4 1
= cos(nt)
n=1
n n
oddn

X 4
= ( cos(nx) + (1)n )
n=1
n2
oddn

X 1 X cos(nx)
=4 2
4
n=1
n n=1
n2
oddn oddn

Since this series converges uniformly,



(
X 1 X cos(nx) x for x < 0
4 2
4 2
= F (x) =
n=1
n n=1
n x for 0 x < .
oddn oddn

The value of the constant term is


1 2
Z
X 1
4 2
= F (x) dx = .
n=1
n 0
oddn

1360
Thus

(
1 X cos(nx) x for x < 0
4 2
=
n=1
n x for 0 x < .
oddn

Differentiating Fourier Series. Recall that in general, a series can only be differentiated if it is uniformly conver-
gent. The necessary and sufficient condition that a Fourier series be uniformly convergent is that the periodic extension
of the function is continuous.
Result 28.10.1 The Fourier series of a function f (x) can be differentiated only if the periodic
extension of f (x) is continuous.

Example 28.10.2 Consider the function defined by


(
for 0 x <
f (x) =
for x < 0.

f (x) has the Fourier series



X 4
f (x) sin nx.
n=1
n
oddn

The function has a derivative except at the points x = n. Differentiating the Fourier series yields

X
0
f (x) 4 cos(nx).
n=1
oddn

For x 6= n, this implies



X
0=4 cos(nx),
n=1
oddn

1361
which is false. The series does not converge. This is as we expected since the Fourier series for f (x) is not uniformly
convergent.

1362
28.11 Exercises
Exercise 28.1
1. Consider a 2 periodic function f (x) expressed as a Fourier series with partial sums
N
a0 X
SN (x) = + an cos(nx) + bn sin(nt).
2 n=1

Assuming that the Fourier series converges in the mean, i.e.


Z
lim (f (x) SN (x))2 dx = 0,
N

show
a20 X 2
Z
1
+ an + b2n = f (x)2 dx.
2 n=1

This is called Parsevals equation.

2. Find the Fourier series for f (x) = x on x < (and repeating periodically). Use this to show

X 1 2
2
= .
n=1
n 6

3. Similarly, by choosing appropriate functions f (x), use Parsevals equation to determine



X 1 X 1
and .
n=1
n4 n=1
n6

Exercise 28.2
Consider the Fourier series of f (x) = x on x < as found above. Investigate the convergence at the points of
discontinuity.

1363
1. Let SN be the sum of the first N terms in the Fourier series. Show that

N + 21 x
 
dSN N cos
= 1 (1) .
cos x2

dx

2. Now use this to show that


1
x
 
sin N+ ( )
Z
2
x SN =
 d.
0 sin 2

3. Finally investigate the maxima of this difference around x = and provide an estimate (good to two decimal
places) of the overshoot in the limit N .

Exercise 28.3
Consider the boundary value problem on the interval 0 < x < 1

y 00 + 2y = 1 y(0) = y(1) = 0.

1. Choose an appropriate periodic extension and find a Fourier series solution.

2. Solve directly and find the Fourier series of the solution (using the same extension). Compare the result to the
previous step and verify the series agree.

Exercise 28.4
Consider the boundary value problem on 0 < x <

y 00 + 2y = sin x y 0 (0) = y 0 () = 0.

1. Find a Fourier series solution.

2. Suppose the ODE is slightly modified: y 00 + 4y = sin x with the same boundary conditions. Attempt to find a
Fourier series solution and discuss in as much detail as possible what goes wrong.

1364
Exercise 28.5
Find the Fourier cosine and sine series for f (x) = x2 on 0 x < . Are the series differentiable?

Exercise 28.6
Find the Fourier series of cosn (x).

Exercise 28.7
For what values of x does the Fourier series

2 X (1)n
+4 2
cos nx = x2
3 n=1
n

converge? What is the value of the above Fourier series for all x? From this relation show that

X 1 2
=
n=1
n2 6

X (1)n+1 2
=
n=1
n2 12

Exercise 28.8
1. Compute the Fourier sine series for the function

2x
f (x) = cos x 1 + , 0 x .

2. How fast do the Fourier coefficients an where



X
f (x) = an sin nx
n=1

decrease with increasing n? Explain this rate of decrease.

1365
Exercise 28.9
Determine the cosine and sine series of
f (x) = x sin x, (0 < x < ).
Estimate before doing the calculation the rate of decrease of Fourier coefficients, an , bn , for large n.
Exercise 28.10
Determine the Fourier cosine series of the function
f (x) = cos(x), 0 x ,
where is an arbitrary real number. From this series deduce the following identities for non-integer .
 
1 X n 1 1
= + (1) +
sin() n=1 n +n
 
1 X 1 1
cot() = + +
n=1 n + n

Integrate the last formula from = 0 to = , (0 < < 1), to show that

2

sin() Y
= 1 2 .
n=1
n

Exercise 28.11
1. Show that

  x  X (1)n
ln cos = ln 2 cos(nx), < x < .
2 n=1
n
Use properties of Fourier series to conclude that

x X (1)n
ln cos = ln 2 cos(nx), x 6= (2k + 1), k Z.

2 n=1
n

1366
Hint: use the identity

X zn
Log(1 z) = for |z| 1, z 6= 1.
n=1
n

2. From this series deduce that Z  x


ln cos dx = ln 2.
0 2
3. Show that
1 sin((x + )/2) X sin(nx) sin(n)
ln = , x 6= + 2k.
2 sin((x )/2) n=1 n

Exercise 28.12
Solve the problem
y 00 + y = f (x), y(a) = y(b) = 0,
with an eigenfunction expansion. Assume that 6= n/(b a), n N.

Exercise 28.13
Solve the problem
y 00 + y = f (x), y(a) = A, y(b) = B,
with an eigenfunction expansion. Assume that 6= n/(b a), n N.

Exercise 28.14
Find the trigonometric series and the simple closed form expressions for A(r, x) and B(r, x) where z = r ex and |r| < 1.
1
a) A + B = 1 + z2 + z4 +
1 z2
1 1
b) A + B log(1 + z) = z z 2 + z 3
2 3
Find An and Bn , and the trigonometric sum for them where:
c) An + Bn = 1 + z + z 2 + + z n .

1367
Exercise 28.15
1. Is the trigonometric system
{1, sin x, cos x, sin 2x, cos 2x, . . .}
orthogonal on the interval [0, ]? Is the system orthogonal on any interval of length ? Why, in each case?
2. Show that each of the systems
{1, cos x, cos 2x, . . .}, and {sin x, sin 2x, . . .}
are orthogonal on [0, ]. Make them orthonormal too.

Exercise 28.16
Let SN (x) be the N th partial sum of the Fourier series for f (x) |x| on < x < . Find N such that |f (x)
SN (x)| < 101 on |x| < .

Exercise 28.17
The set {sin(nx)}
n=1 is orthogonal and complete on [0, ].

1. Find the Fourier sine series for f (x) 1 on 0 x .


2. Find a convergent series for g(x) = x on 0 x by integrating the series for part (a).
3. Apply Parsevals relation to the series in (a) to find:

X 1
n=1
(2n 1)2
Check this result by evaluating the series in (b) at x = .

Exercise 28.18
1. Show that the Fourier cosine series expansion on [0, ] of:


1, 0 x < 2 ,

f (x) 12 , x = 2 ,

0, 2 < x ,

1368
is
1 2 X (1)n
S(x) = + cos((2n + 1)x).
2 n=0 2n + 1

2. Show that the N th partial sum of the series in (a) is


Z x/2
1 1 sin((2(N + 1)t)
SN (x) = dt.
2 0 sin t
P2N +1 PN
( Hint: Consider the difference of n=1 (ey )n and n=1 (e
2y n
) , where y = x /2.)
n
3. Show that dSN (x)/dx = 0 at x = xn = 2(N +1)
for n = 0, 1, . . . , N, N + 2, . . . , 2N + 2.

4. Show that at x = xN , the maximum of SN (x) nearest to /2 in (0, /2) is


Z N
1 1 2(N +1) sin(2(N + 1)t)
SN (xN ) = + dt.
2 0 sin t

Clearly xN /2 as N .

5. Show that also in this limit, Z


1 1 sin t
SN (xN ) + dt 1.0895.
2 0 t
How does this compare with f (/2 0)? This overshoot is the Gibbs phenomenon that occurs at each disconti-
nuity. It is a manifestation of the non-uniform convergence of the Fourier series for f (x) on [0, ].

Exercise 28.19
Prove the Isoperimetric Inequality: L2 4A where L is the length of the perimeter and A the area of any piecewise
smooth plane figure. Show that equality is attained only for the circle. (Hints: The closed curve is represented
parametrically as
x = x(s), y = y(s), 0 s L

1369
where s is the arclength. In terms of t = 2s/L we have
 2  2   2
dx dy L
+ = .
dt dt 2
Integrate this relation over [0, 2]. The area is given by
Z 2
dy
A= x dt.
0 dt
Express x(t) and y(t) as Fourier series and use the completeness and orthogonality relations to show that L2 4A 0.)

Exercise 28.20
1. Find the Fourier sine series expansion and the Fourier cosine series expansion of

g(x) = x(1 x), on 0 x 1.

Which is better and why over the indicated interval?


2. Use these expansions to show that:

X 1 2 X (1)k 2 X (1)k 3
i) 2
= , ii) = , iii) = .
k=1
k 6 k=1
k2 12 k=1
(2k 1)2 32

Note: Some useful integration by parts formulas are:


Z Z
1 x 1 x
x sin(nx) = 2 sin(nx) cos(nx); x cos(nx) = 2 cos(nx) + sin(nx)
n n n n
2 2
n x 2
Z
2x
x2 sin(nx) = 2 sin(nx) cos(nx)
n n3
n2 x2 2
Z
2x
x2 cos(nx) = 2 cos(nx) + sin(nx)
n n3

1370
28.12 Hints
Hint 28.1

Hint 28.2

Hint 28.3

Hint 28.4

Hint 28.5

Hint 28.6
Expand
 n
n 1 x x
cos (x) = (e + e )
2
Using Newtons binomial formula.

Hint 28.7

Hint 28.8

Hint 28.9

1371
Hint 28.10

Hint 28.11

Hint 28.12

Hint 28.13

Hint 28.14

Hint 28.15

Hint 28.16

Hint 28.17

Hint 28.18

Hint 28.19

Hint 28.20

1372
28.13 Solutions
Solution 28.1
1. We start by assuming that the Fourier series converges in the mean.


!2
Z
a0 X
f (x) (an cos(nx) + bn sin(nx)) = 0
2 n=1

We interchange the order of integration and summation.

Z Z 
X Z Z 
2
(f (x)) dx a0 f (x) dx 2 an f (x) cos(nx) dx + bn f (x) sin(nx)
n=1
Z
a20 X
+ + a0 (an cos(nx) + bn sin(nx)) dx
2 n=1
X X Z
+ (an cos(nx) + bn sin(nx))(am cos(mx) + bm sin(mx)) dx = 0
n=1 m=1

Most of the terms vanish because the eigenfunctions are orthogonal.

Z Z 
X Z Z 
2
(f (x)) dx a0 f (x) dx 2 an f (x) cos(nx) dx + bn f (x) sin(nx)
n=1
Z
a20 X
+ + (a2n cos2 (nx) + b2n sin2 (nx)) dx = 0
2 n=1

1373
We use the definition of the Fourier coefficients to evaluate the integrals in the last sum.
Z
2 2
X
2 2
 a20 X
a2n + b2n = 0

(f (x)) dx a0 2 an + b n + +
n=1
2 n=1

a20 X 2  1
Z
+ an + b2n = f (x)2 dx
2 n=1

2. We determine the Fourier coefficients for f (x) = x. Since f (x) is odd, all of the an are zero.
1
Z
b0 = x sin(nx) dx

  Z
1 1 1
= x cos(nx) + cos(nx) dx
n n
2(1)n+1
=
n
The Fourier series is
X 2(1)n+1
x= sin(nx) for x ( . . . ).
n=1
n
We apply Parsevals theorem for this series to find the value of 1
P
n=1 n2
.

1 2
Z
X 4
2
= x dx
n=1
n

X 4 2 2
=
n=1
n2 3

X 1 2
=
n=1
n2 6

1374
3. Consider f (x) = x2 . Since the function is even, there are no sine terms in the Fourier series. The coefficients in
the cosine series are

2 2
Z
a0 = x dx
0
2 2
=
3Z
2 2
an = x cos(nx) dx
0
4(1)n
= .
n2

Thus the Fourier series is


2 2 X (1)n
x = +4 cos(nx) for x ( . . . ).
3 n=1
n2

P 1
We apply Parsevals theorem for this series to find the value of n=1 n4 .


2 4 1 4
Z
X 1
+ 16 4
= x dx
9 n=1
n

2 4 X 1 2 4
+ 16 =
9 n=1
n4 5

X 1 4
=
n=1
n4 90

1375
Now we integrate the series for f (x) = x2 .

x x
2 (1)n
Z   X Z
2
d = 4 cos(n) d
0 3 n=1
n2 0
3 2
x X (1)n
x=4 sin(nx)
3 3 n=1
n3

P 1
We apply Parsevals theorem for this series to find the value of n=1 n6 .

2
1 x3 2
Z 
X 1
16 6
= x dx
n=1
n 3 3

X 1 16 6
16 =
n=1
n6 945

X 1 6
=
n=1
n6 945

1376
Solution 28.2
1. We differentiate the partial sum of the Fourier series and evaluate the sum.

N
X 2(1)n+1
SN = sin(nx)
n=1
n
XN
0
SN =2 (1)n+1 cos(nx)
n=1
N
!
X
0 n+1 nx
SN = 2< (1) e
n=1
1 (1)N +2 e(N +1)x
 
0
SN = 2<
1 + ex
1 + ex (1)N e(N +1)x (1)N eN x
 
0
SN = <
1 + cos(x)
0 cos((N + 1)x) + cos(N x)
SN = 1 (1)N
1 + cos(x)
1 x
  
0 cos N + x cos
SN = 1 (1)N 2 2
cos2 x2


1
 
dSN cos N + x
= 1 (1)N x
2
dx cos 2

1377
0
2. We integrate SN .
1
 
x
(1)N cos N+
Z
2
SN (x) SN (0) = x
 d
0 cos 2
1
x
 
sin N+ ( )
Z
2
x SN =
 d
0 sin 2

3. We find the extrema of the overshoot E = x SN with the first derivative test.
sin N + 12 (x )
 
0
E = =0
sin x

2

We look for extrema in the range ( . . . ).


 
1
N+ (x ) = n
2
 
n
x= 1 , n [1 . . . 2N ]
N + 1/2
The closest of these extrema to x = is
 
1
x= 1
N + 1/2
Let E0 be the overshoot at this point. We approximate E0 for large N .
sin N + 21 ( )
Z (11/(N +1/2))  
E0 = d
sin

0 2

We shift the limits of integration.


1

 
sin N+
Z
2
E0 =
 d
/(N +1/2) sin 2

1378
We add and subtract an integral over [0 . . . /(N + 1/2)].

1 1

  /(N +1/2)
 
sin N+ sin N+
Z Z
2 2
E0 =
 d
 d
0 sin 2 0 sin 2

We can evaluate the first integral with contour integration on the unit circle C.

1

 
sin N+
Z Z
2 sin ((2N + 1) )

 d = d
0 sin 2 0 sin ()
Z
1 sin ((2N + 1) )
= d
2 sin ()

= z 2N +1
Z
1 dz
=
2 C (z 1/z)/(2) z
z 2N +1
Z 
== 2 dz
C (z 1)
z 2N +1 z 2N +1
    
= = Res , 1 + Res , 1
(z + 1)(z 1) (z + 1)(z 1)
 2N +1
(1)2N +1

1
= < +
2 2
=

1379
We approximate the second integral.

1
/(N +1/2)
  Z
sin N+
Z
2 2 sin(x)
d =  dx
2N + 1 0 sin 2Nx+1

0 sin 2
Z
sin(x)
2 dx
0 x
Z X
1 (1)n x2n+1
=2 dx
0 x n=0 (2n + 1)!
Z
X (1)n x2n
=2 dx
n=0 0 (2n + 1)!

X (1)n 2n+1
=2 dx
n=0
(2n + 1)(2n + 1)!
3.70387

In the limit as N , the overshoot is

| 3.70387| 0.56.

Solution 28.3
1. The eigenfunctions of the self-adjoint problem

y 00 = y, y(0) = y(1) = 0,

are
n = sin(nx), n Z+

1380
We find the series expansion of the inhomogeneity f (x) = 1.

X
1= fn sin(nx)
n=1
Z 1
fn = 2 sin(nx) dx
0
 1
cos(nx)
fn = 2
n 0
2
fn = (1 (1)n )
(n
4
n
for odd n
fn =
0 for even n
We expand the solution in a series of the eigenfunctions.

X
y= an sin(nx)
n=1

We substitute the series into the differential equation.


y 00 + 2y = 1

X
2 2
X X 4
an n sin(nx) + 2 an sin(nx) = sin(nx)
n=1 n=1 n=1
n
odd n
(
4
n(2 2 n2 )
for odd n
an =
0 for even n

X 4
y= sin(nx)
n=1
n(2 2 n2 )
odd n

1381
2. Now we solve the boundary value problem directly.
y 00 + 2y = 1 y(0) = y(1) = 0
The general solution of the differential equation is
    1
y = c1 cos 2x + c2 sin 2x + .
2
We apply the boundary conditions to find the solution.
1     1
c1 + = 0, c1 cos 2 + c2 sin 2 + =0
2  2
1 cos 2 1
c 1 = , c2 = 
2 2 sin 2
  cos 2 1
!
1  
y= 1 cos 2x +  sin 2x
2 sin 2

We find the Fourier sine series of the solution.



X
y= an sin(nx)
n=1
Z 1
an = 2 y(x) sin(nx) dx
0
  cos 2 1
!
Z 1  
an = 1 cos 2x +  sin 2x sin(nx) dx
0 sin 2
2(1 (1)2
an =
n(2 2 n2 )
(
4
n(2 2 n2 )
for odd n
an =
0 for even n

1382
We obtain the same series as in the first part.

Solution 28.4
1. The eigenfunctions of the self-adjoint problem

y 00 = y, y 0 (0) = y 0 () = 0,

are
1
0 = , n = cos(nx), n Z+
2
We find the series expansion of the inhomogeneity f (x) = sin(x).

f0 X
f (x) = + fn cos(nx)
2 n=1
2
Z
f0 = sin(x) dx
0
4
f0 =

2
Z
fn = sin(x) cos(nx) dx
0
2(1 + (1)n )
fn =
(1 n2 )
(
4
2 for even n
fn = (1n )
0 for odd n

We expand the solution in a series of the eigenfunctions.



a0 X
y= + an cos(nx)
2 n=1

1383
We substitute the series into the differential equation.

y 00 + 2y = sin(x)

X
2
X 2 X 4
an n cos(nx) + a0 + 2 an cos(nx) = + cos(nx)
n=1 n=1
n=2
(1 n2 )
even n

1 X 4
y= + cos(nx)
n=2
(1 n2 )(2 n2 )
even n

2. We expand the solution in a series of the eigenfunctions.



a0 X
y= + an cos(nx)
2 n=1

We substitute the series into the differential equation.

y 00 + 4y = sin(x)

X
2
X 2 X 4
an n cos(nx) + 2a0 + 4 an cos(nx) = + cos(nx)
n=1 n=1
n=2
(1 n2 )
even n

It is not possible to solve for the a2 coefficient. That equation is


4
(0)a2 = .
3
This problem is to be expected, as this boundary value problem does not have a solution. The solution of the
differential equation is
1
y = c1 cos(2x) + c2 sin(2x) + sin(x)
3

1384
The boundary conditions give us an inconsistent set of constraints.

y 0 (0) = 0, y 0 () = 0
1 1
c2 + = 0, c2 = 0
3 3

Thus the problem has no solution.

Solution 28.5
Cosine Series. The coefficients in the cosine series are

2 2
Z
a0 = x dx
0
2 2
=
3Z
2 2
an = x cos(nx) dx
0
4(1)n
= .
n2

Thus the Fourier cosine series is



2 X 4(1)n
f (x) = + cos(nx).
3 n=1
n2

In Figure 28.10 the even periodic extension of f (x) is plotted in a dashed line and the sum of the first five terms in the
Fourier series is plotted in a solid line. Since the even periodic extension is continuous, the cosine series is differentiable.

1385
10 10

8
5

-3 -2 -1 1 2 3
4

-5
2

-3 -2 -1 1 2 3 -10

Figure 28.10: The Fourier Cosine and Sine Series of f (x) = x2 .

Sine Series. The coefficients in the sine series are

Z
2
bn = x2 sin(nx) dx
0
2(1)n 4(1 (1)n )
=
( n n3
n
2(1)
n

for even n
= n
2(1) 8
n n3 for odd n.

1386
Thus the Fourier sine series is

2(1)n 4(1 (1)n )
X 
f (x) + sin(nx).
n=1
n n3

In Figure 28.10 the odd periodic extension of f (x) and the sum of the first five terms in the sine series are plotted.
Since the odd periodic extension of f (x) is not continuous, the series is not differentiable.

Solution 28.6
We could find the expansion by integrating to find the Fourier coefficients, but it is easier to expand cosn (x) directly.
 n
n 1 x x
cos (x) = (e + e )
2
        
1 n nx n (n2)x n (n2)x n nx
= n e + e + + e + e
2 0 1 n1 n
If n is odd,
"   
1 n nx n
cosn (x) = n nx
(e + e )+ (e(n2)x + e(n2)x ) +
2 0 1
  #
n
+ (ex + ex )
(n 1)/2
      
1 n n n
= n 2 cos(nx) + 2 cos((n 2)x) + + 2 cos(x)
2 0 1 (n 1)/2
(n1)/2  
1 X n
= n1 cos((n 2m)x)
2 m=0
m
n  
1 X n
= n1 cos(kx).
2 k=1
(n k)/2
odd k

1387
If n is even,

"   
1 n nx n
cosn (x) = n nx
(e + e )+ (e(n2)x + e(n2)x ) +
2 0 1
   #
n n
+ (e2x + ei2x ) +
n/2 1 n/2
       
1 n n n n
= n 2 cos(nx) + 2 cos((n 2)x) + + 2 cos(2x) +
2 0 1 n/2 1 n/2
  (n2)/2  
1 n 1 X n
= n + n1 cos((n 2m)x)
2 n/2 2 m=0
m
  n  
1 n 1 X n
= n + n1 cos(kx).
2 n/2 2 k=2
(n k)/2
even k

We may denote,

n
n a0 X
cos (x) = ak cos(kx),
2 k=1

where

1 + (1)nk 1
 
n
ak = .
2 2n1 (n k)/2

1388
Solution 28.7
We expand f (x) in a cosine series. The coefficients in the cosine series are

2 2
Z
a0 = x dx
0
2 2
=
3Z
2 2
an = x cos(nx) dx
0
4(1)n
= .
n2

Thus the Fourier cosine series is



2 X (1)n
f (x) = +4 2
cos(nx).
3 n=1
n

The Fourier series converges to the even periodic extension of

f (x) = x2 for 0 < x < ,

which is
  2
x+
f(x) = x 2 .
2

(bc denotes the floor or greatest integer function.) This periodic extension is a continuous function. Since x2 is an
even function, we have

2 X (1)n
+4 2
cos nx = x2 for x .
3 n=1
n

1389
We substitute x = into the Fourier series.

2 X (1)n
+4 2
cos(n) = 2
3 n=1
n

X 1 2
=
n=1
n2 6

We substitute x = 0 into the Fourier series.



2 X (1)n
+4 =0
3 n=1
n2

X (1)n+1 2
=
n=1
n2 12

Solution 28.8
1. We compute the Fourier sine coefficients.

2
Z
an = f (x) sin(nx) dx
0
2
Z  
2x
= cos x 1 + sin(nx) dx
0
2(1 + (1)n )
=
(n3 n)
(
4
(n3 n)
for even n
an =
0 for odd n

1390
2. From our work in the previous part, we see that the Fourier coefficients decay as 1/n3 . The Fourier sine series
converges to the odd periodic extension of the function, f(x). We can determine the rate of decay of the Fourier
coefficients from the smoothness of f(x). For < x < , the odd periodic extension of f (x) is defined
(
f (x) = cos(x) 1 + 2x
0 x < ,
f (x) = 2x
f (x) = cos(x) + 1 + x < 0.

Since
f(0+ ) = f(0 ) = 0 and f() = f() = 0
f(x) is continuous, C 0 . Since
2 2
f0 (0+ ) = f0 (0 ) = and f0 () = f0 () =

f(x) is continuously differentiable, C 1 . However, since
f00 (0+ ) = 1, and f00 (0 ) = 1
f(x) is not C 2 . Since f(x) is C 1 we know that the Fourier coefficients decay as 1/n3 .

Solution 28.9
Cosine Series. The even periodic extension of f (x) is a C 0 , continuous, function (See Figure 28.11. Thus the
coefficients in the cosine series will decay as 1/n2 . The Fourier cosine coefficients are
2
Z
a0 = x sin x dx
0
=2
Z
2
a1 = x sin x cos x dx
0
1
=
2

1391
2
Z
an = x sin x cos(nx) dx
0
2(1)n+1
= , for n 2
n2 1
The Fourier cosine series is

1 X 2(1)n
f(x) = 1 cos x 2 21
cos(nx).
2 n=2
n

-5 5

Figure 28.11: The even periodic extension of x sin x.

Sine Series. The odd periodic extension of f (x) is a C 1 , continuously differentiable, function (See Figure 28.12.
Thus the coefficients in the cosine series will decay as 1/n3 . The Fourier sine coefficients are
1
Z
a1 = x sin x sin x dx
0

=
2

1392
Z
2
an = x sin x sin(nx) dx
0
4(1 + (1)n )n
= , for n 2
(n2 1)2

The Fourier sine series is



4 X (1 + (1)n )n
f(x) = sin x cos(nx).
2 n=2 (n2 1)2

-5 5

Figure 28.12: The odd periodic extension of x sin x.

Solution 28.10
If = n is an integer, then the Fourier cosine series is the single term cos(|n|x). We assume that 6= n.
We note that the even periodic extension of cos(x) is C 0 so that the series converges to cos(x) for x

1393
and the coefficients decay as 1/n2 . We compute the Fourier cosine coefficients.
2
Z
a0 = cos(x) dx
0
2 sin()
=

2
Z
an = cos(x) cos(nx) dx
0
 
n 1 1
= (1) + sin()
n +n
The Fourier cosine series is
 
sin() X 1 1
cos(x) = + (1)n + sin() cos(nx).
n=1
n +n

We substitute x = 0 into the Fourier cosine series.


 
sin() X n 1 1
1= + (1) + sin()
n=1
n +n
 
1 X n 1 1
= + (1) +
sin n=1 n +n

Next we substitute x = into the Fourier cosine series.


 
sin() X n 1 1
cos() = + (1) + sin()(1)n
n=1
n + n
 
1 X 1 1
cot = + +
n=1 n + n

1394
Note that neither cot() nor 1/ is integrable at = 0. We write the last formula so each side is integrable.
 
1 X 1 1
cot = +
n=1
n +n

We integrate from = 0 to = < 1.


   X 
sin() 
ln = [ln(n )]0 + [ln(n + )]0
0 n=1
      
sin() X n n+
ln ln = ln + ln
n=1
n n

2
  X  
sin()
ln = ln 1 2
n=1
n
 !
2
 
sin() Y
ln = ln 1 2
n=1
n

2

sin() Y
= 1 2
n=1
n

Solution 28.11
1. We will consider the principal branch of the logarithm, < =(Log z) . For < x < , cos(x/2) is
positive so that ln(cos(x/2)) is well-defined. At x = , ln(cos(x/2)) is singular. However, the function is
integrable so it has a Fourier series which converges except at x = (2k + 1), k Z.
+ ex/2
 x/2 
 x e
ln cos = ln
2 2
= ln 2 + ln ex/2 (1 + ex )

x
= ln 2 + Log (1 + ex )
2

1395
Since | ex | 1 and ex 6= 1 for =(x) 0, x 6= (2k + 1), we can expand the last term in a Taylor series in
that domain.

x X (1)n x n
= ln 2 (e )
2 n=1 n

!
X (1)n x X (1)n
= ln 2 cos(nx) + sin(nx)
n=1
n 2 n=1 n

For < x < , ln(cos(x/2)) is real-valued. We equate the real parts of the equation on this domain to obtain
the desired Fourier series.

  x  X (1)n
ln cos = ln 2 cos(nx), < x < .
2 n=1
n

The domain of convergence for this series is =(x) = 0, x 6= (2k + 1). The Fourier series converges to the
periodic extension of the function.

x X (1)n
ln cos = ln 2 cos(nx), x 6= (2k + 1), k Z

2 n=1
n

2. Now we integrate the function from 0 to .



Z  Z !
x X (1)n
ln cos dx = ln 2 cos(nx) dx
0 2 0 n=1
n

(1)n
X Z
= ln 2 cos(nx) dx
n=1
n 0

(1)n sin(nx)
X 
= ln 2
n=1
n n 0

1396
Z   x 
ln cos dx = ln 2
0 2

3. We expand the logorithm.



1 sin((x + )/2) 1 1
ln = ln |sin((x + )/2)| ln |sin((x )/2)|
2 sin((x )/2) 2 2
Consider the function ln | sin(y/2)|. Since sin(x) = cos(x /2), we can use the result of part (a) to obtain,
 
 y  y
ln sin = ln cos

2 2

X (1)n
= ln 2 cos(n(y ))
n=1
n

X 1
= ln 2 cos(ny), for y 6= 2k, k Z.
n=1
n

We return to the original function:



!
1 sin((x + )/2) 1 X 1 X 1
ln = ln 2 cos(n(x + )) + ln 2 + cos(n(x )) ,
2 sin((x )/2) 2 n=1
n n=1
n

for x 6= 2k, k Z.

1 sin((x + )/2) X sin(nx) sin(n)
ln = , x 6= + 2k
2 sin((x )/2) n=1 n

Solution 28.12
The eigenfunction problem associated with this problem is

00 + 2 = 0, (a) = (b) = 0,

1397
which has the solutions,  
n n(x a)
n = , n = sin , n N.
ba ba
We expand the solution and the inhomogeneity in the eigenfunctions.
 
X n(x a)
y(x) = yn sin
n=1
ba

  b  
n(x a) n(x a)
Z
X 2
f (x) = fn sin , fn = f (x) sin dx
n=1
ba ba a ba
Since the solution y(x) satisfies the same homogeneous boundary conditions as the eigenfunctions, we can differentiate
the series. We substitute the series expansions into the differential equation.

y 00 + y = f (x)

X X
2

yn n + sin (n x) = fn sin (n x)
n=1 n=1
fn
yn =
2n

Thus the solution of the problem has the series representation,


 
X n(x a)
2n

y(x) = sin .
n=1
ba

Solution 28.13
The eigenfunction problem associated with this problem is

00 + 2 = 0, (a) = (b) = 0,

1398
which has the solutions,  
n n(x a)
n = , n = sin , n N.
ba ba
We expand the solution and the inhomogeneity in the eigenfunctions.
 
X n(x a)
y(x) = yn sin
n=1
ba
  Z b  
X n(x a) 2 n(x a)
f (x) = fn sin , fn = f (x) sin dx
n=1
b a b a a b a
Since the solution y(x) does not satisfy the same homogeneous boundary conditions as the eigenfunctions, we can
differentiate the series. We multiply the differential equation by an eigenfunction and integrate from a to b. We use
integration by parts to move derivatives from y to the eigenfunction.
y 00 + y = f (x)
Z b Z b Z b
00
y (x) sin(m x) dx + y(x) sin(m x) dx = f (x) sin(m x) dx
a a a
Z b
0 b ba ba
[y sin(m x)]a y 0 m cos(m x) dx + ym = fm
a 2 2
Z b
b ba ba
[ym cos(m x)]a y2m sin(m x) dx + ym = fm
a 2 2
ba ba
Bm (1)m + Am (1)m+1 2m ym + ym = fm
2 2
fm + (1)m m (A + B)
ym =
2m
Thus the solution of the problem has the series representation,

fm + (1)m m (A + B)
 
X n(x a)
y(x) = 2
sin .
n=1
m b a

1399
Solution 28.14
1.

1
A + B =
1 z2
X
= z 2n
n=0

X
= r2n e2nx
n=0

X
X
2n
= r cos(2nx) + r2n sin(2nx)
n=0 n=1


X
X
A= r2n cos(2nx), B= r2n sin(2nx)
n=0 n=1

1
A + B =
1 z2
1
=
1 r2 e2x
1
=
1 cos(2x) r2 sin(2x)
r2
1 r2 cos(2x) + r2 sin(2x)
=
(1 r2 cos(2x))2 + (r2 sin(2x))2

1 r2 cos(2x) r2 sin(2x)
A= , B=
1 2r2 cos(2x) + r4 1 2r2 cos(2x) + r4

1400
2. We consider the principal branch of the logarithm.

A + B = log(1 + z)

X (1)n+1 n
= z
n=1
n

X (1)n+1
= rn enx
n=1
n

X (1)n+1 n 
= r cos(nx) + sin(nx)
n=1
n


X (1)n+1 n
X (1)n+1
A= r cos(nx), B= rn sin(nx)
n=1
n n=1
n

A + B = log(1 + z)
= log (1 + r ex )
= log (1 + r cos x + r sin x)
= log |1 + r cos x + r sin x| + arg (1 + r cos x + r sin x)
p
= log (1 + r cos x)2 + (r sin x)2 + arctan (1 + r cos x, r sin x)

1
log 1 + 2r cos x + r2 ,

A= B = arctan (1 + r cos x, r sin x)
2

1401
3.
n
X
An + Bn = zk
k=1
1 z n+1
=
1z
1 rn+1 e(n+1)x
=
1 r ex
1 r ex rn+1 e(n+1)x +rn+2 enx
=
1 2r cos x + r2
1 r cos x rn+1 cos((n + 1)x) + rn+2 cos(nx)
An =
1 2r cos x + r2
r sin x rn+1 sin((n + 1)x) + rn+2 sin(nx)
Bn =
1 2r cos x + r2
n
X
An + Bn = zk
k=1
n
X
= rk ekx
k=1

n
X n
X
k
An = r cos(kx), Bn = rk sin(kx)
k=1 k=1

Solution 28.15
1. Z
1 sin x dx = [ cos x]0 = 2
0

1402
Thus the system is not orthogonal on the interval [0, ]. Consider the interval [a, a + ].
Z a+
1 sin x dx = [ cos x]a+
a = 2 cos a
Za a+
1 cos x dx = [sin x]a+
a = 2 sin a
a

Since there is no value of a for which both cos a and sin a vanish, the system is not orthogonal for any interval
of length .

2. First note that Z


cos nx dx = 0 for n N.
0

If n 6= m, n 1 and m 0 then
Z
1
Z

cos nx cos mx dx = cos((n m)x) + cos((n + m)x) dx = 0
0 2 0

Thus the set {1, cos x, cos 2x, . . .} is orthogonal on [0, ]. Since
Z
dx =
0
Z

cos2 (nx) dx = ,
0 2

the set
(r r r )
1 2 2
, cos x, cos 2x, . . .

is orthonormal on [0, ].

1403
If n 6= m, n 1 and m 1 then
Z Z
1 
sin nx sin mx dx = cos((n m)x) cos((n + m)x) dx = 0
0 2 0

Thus the set {sin x, sin 2x, . . .} is orthogonal on [0, ]. Since

Z

sin2 (nx) dx = ,
0 2

the set
(r r )
2 2
sin x, sin 2x, . . .

is orthonormal on [0, ].

Solution 28.16
Since the periodic extension of |x| in [, ] is an even function its Fourier series is a cosine series. Because of the
anti-symmetry about x = /2 we see that except for the constant term, there will only be odd cosine terms. Since the
periodic extension is a continuous function, but has a discontinuous first derivative, the Fourier coefficients will decay
as 1/n2 .

X
|x| = an cos(nx), for x [, ]
n=0

 
1 x2
Z
1
a0 = x dx = =
0 2 0 2

1404
2
Z
an = x cos(nx) dx
0

2 sin(nx)
 Z
2 sin(nx)
= x dx
n 0 0 n
 
2 cos(nx)
=
n2 0
2
= 2 (cos(n) 1)
n
2(1 (1)n )
=
n2

4 X 1
|x| = + cos(nx) for x [, ]
2 n=1 n2
odd n

Define RN (x) = f (x) SN (x). We seek an upper bound on |RN (x)|.





4 X 1
|RN (x)| = cos(nx)

n 2


n=N +1
odd n

4 X 1

n=N +1 n2
odd n
N
4 X 1 4 X 1
=
n=1
n2 n=1 n2
odd n odd n

Since
X 1 2
=
n=1
n2 8
odd n

1405
We can bound the error with,
N
4 X 1
|RN (x)| .
2 n=1 n2
odd n

N = 7 is the smallest number for which our error bound is less than 101 . N 7 is sufficient to make the error less
that 0.1.  
4 1 1 1
|R7 (x)| 1+ + + 0.079
2 9 25 49
N 7 is also necessary because.

4 X 1
|RN (0)| = .
n=N +1 n2
odd n

Solution 28.17
1.

X
1 an sin(nx), 0x
n=1
Since the odd periodic extension of the function is discontinuous, the Fourier coefficients will decay as 1/n.
Because of the symmetry about x = /2, there will be only odd sine terms.
2
Z
an = 1 sin(nx) dx
0
2
= ( cos(n) + cos(0))
n
2
= (1 (1)n )
n

4 X sin(nx)
1
n=1 n
odd n

1406
2. Its always OK to integrate a Fourier series term by term. We integrate the series in part (a).
x Z
4 X x sin(n)
Z
1 dx dx
a n=1 a n
odd n

4 X cos(na) cos(nx)
xa
n=1 n2
odd n

Since the series converges uniformly, we can replace the with =.



4 X cos(na) 4 X cos(nx)
xa=
n=1 n2 n=1 n2
odd n odd n

Now we have a Fourier cosine series. The first sum on the right is the constant term. If we choose a = /2 this
sum vanishes since cos(n/2) = 0 for odd integer n.

4 X cos(nx)
x=
2 n=1 n2
odd n

3. If f (x) has the Fourier series



a0 X
f (x) + (an cos(nx) + bn sin(nx)),
2 n=1

then Parsevals theorem states that


Z
2 2 X
f (x) dx = a0 + (a2n + b2n ).
2 n=1

1407
We apply this to the Fourier sine series from part (a).
Z  2
2
X 4
f (x) dx =
n=1
n
odd n
Z 0 Z
2 2 16 X 1
(1) dx + (1) dx =
0 n=1 (2n 1)2

X 1 2
=
n=1
(2n 1)2 8

We substitute x = in the series from part (b) to corroborate the result.



4 X cos((2n 1)x)
x=
2 n=1 (2n 1)2

4 X cos((2n 1))
=
2 n=1 (2n 1)2

X 1 2
=
n=1
(2n 1)2 8

Solution 28.18
1.

X
f (x) a0 + an cos(nx)
n=1

Since the periodic extension of the function is discontinuous, the Fourier coefficients will decay like 1/n. Because
of the anti-symmetry about x = /2, there will be only odd cosine terms.
1
Z
1
a0 = f (x) dx =
0 2

1408
2
Z
an = f (x) cos(nx) dx
0
2 /2
Z
= cos(nx) dx
0
2
= sin(n/2)
n
(
2
n
(1)(n1)/2 , for odd n
=
0 for even n

The Fourier cosine series of f (x) is


1 2 X (1)n
f (x) + cos((2n + 1)x).
2 n=0 2n + 1

2. The N th partial sum is

N
1 2 X (1)n
SN (x) = + cos((2n + 1)x).
2 n=0 2n + 1

We wish to evaluate the sum from part (a). First we make the change of variables y = x /2 to get rid of the

1409
(1)n factor.


X (1)n
cos((2n + 1)x)
n=0
2n + 1
N
X (1)n
= cos((2n + 1)(y + /2))
n=0
2n + 1
N
X (1)n
= (1)n+1 sin((2n + 1)y)
n=0
2n + 1
N
X 1
= sin((2n + 1)y)
n=0
2n + 1

1410
We write the summand as an integral and interchange the order of summation and integration to get rid of the
1/(2n + 1) factor.
XN Z y
= cos((2n + 1)t) dt
n=0 0
Z N
yX
= cos((2n + 1)t) dt
0 n=0
2N +1 N
!
Z y X X
= cos(nt) cos(2nt) dt
0 n=1 n=1
2N +1 N
!
Z y X X
= < ent 2nt
e dt
0 n=1 n=1
y
et (2N +2)t
e2t e2(N +1)t
 
e
Z
= < dt
0 1 et 1 e2t
Z y  t
(e e2(N +1)t )(1 e2t ) (e2t e2(N +1)t )(1 et )

= < dt
0 (1 et )(1 e2t )
Z y  t
e e2t + e(2N +4)t e(2N +3)t

= < dt
0 (1 et )(1 e2t )
Z y  t
e e(2N +3)t

= < dt
0 1 e2t
Z y  (2N +2)t 
e 1
= < dt
0 et et
Z y 
e2(N +1)t +

= < dt
0 2 sin t
1 y sin(2(N + 1)t)
Z
= dt
2 0 sin t
1 x/2 sin(2(N + 1)t)
Z
= dt
2 0 sin t
1411
Now we have a tidy representation of the partial sum.

1 1 x/2 sin(2(N + 1)t)


Z
SN (x) = dt
2 0 sin t

dSN (x)
3. We solve dx
= 0 to find the relative extrema of SN (x).
0
SN (x) = 0
1 sin(2(N + 1)(x /2))
=0
sin(x /2)
(1)N +1 sin(2(N + 1)x)
=0
cos(x)
sin(2(N + 1)x)
=0
cos(x)
n
x = xn = , n = 0, 1, . . . , N, N + 2, . . . , 2N + 2
2(N + 1)

Note that xN +1 = /2 is not a solution as the denominator vanishes there. The function has a removable
singularity at x = /2 with limiting value (1)N .
4. N
Z /2
1 1 2(N +1) sin(2(N + 1)t)
SN (xN ) = dt
2 0 sin t
We note that the integrand is even.
Z N /2 Z 2(N+1) Z
2(N +1) 2(N +1)
= =
0 0 0
Z
1 1 2(N +1) sin(2(N + 1)t)
SN (xN ) = + dt
2 0 sin t

1412
5. We make the change of variables 2(N + 1)t t.
Z
1 1 sin(t)
SN (xN ) = + dt
2 0 2(N + 1) sin(t/(2(N + 1)))

Note that
sin(t) t cos(t)
lim = lim =t
0  0 1
1 1 sin(t)
Z
SN (xN ) + dt 1.0895 as N
2 0 t

This is not equal to the limiting value of f (x), f (/2 0) = 1.

Solution 28.19
With the parametrization in t, x(t) and y(t) are continuous functions on the range [0, 2]. Since the curve is closed,
we have x(0) = x(2) and y(0) = y(2). This means that the periodic extensions of x(t) and y(t) are continuous
functions. Thus we can differentiate their Fourier series. First we define formal Fourier series for x(t) and y(t).

a0 X 
x(t) = + an cos(nt) + bn sin(nt)
2 n=1

c0 X 
y(t) = + cn cos(nt) + dn sin(nt)
2 n=1

X
0

x (t) = nbn cos(nt) nan sin(nt)
n=1
X
y 0 (t) =

ndn cos(nt) ncn sin(nt)
n=1

In this problem we will be dealing with integrals on [0, 2] of products of Fourier series. We derive a general formula

1413
for later use.

Z 2
! !
Z 2
a0 X  c0 X 
xy dt = + an cos(nt) + bn sin(nt) + cn cos(nt) + dn sin(nt) dt
0 0 2 n=1
2
Z 2
!n=1
a0 c 0 X
an cn cos2 (nt) + bn dn sin2 (nt)

= + dt
0 4 n=1

!
1 X
= a0 c 0 + (an cn + bn dn )
2 n=1

In the arclength parametrization we have


 2  2
dx dy
+ = 1.
ds ds
In terms of t = 2s/L this is
 2  2  2
dx dy L
+ = .
dt dt 2
We integrate this identity on [0, 2].
Z 2 2 2 !
L2
 
dx dy
= + dt
2 0 dt dt

!
X  X
2 2
(ndn )2 + (ncn )2

= (nbn ) + (nan ) +
n=1 n=1

X
= n2 (a2n + b2n + c2n + d2n )
n=1

X
L2 = 2 2 n2 (a2n + b2n + c2n + d2n )
n=1

1414
We assume that the curve is parametrized so that the area is positive. (Reversing the orientation changes the sign
of the area as defined above.) The area is
Z 2
dy
A= x dt
0 dt

Z 2 ! !
a0 X  X 
= + an cos(nt) + bn sin(nt) ndn cos(nt) ncn sin(nt) dt
0 2 n=1 n=1

X
= n(an dn bn cn )
n=1

Now we find an upper bound on the area. We will use the inequality |ab| 21 |a2 + b2 |, which follows from expanding
(a b)2 0.

X
n a2n + b2n + c2n + d2n

A
2 n=1

X 2 2
n an + b2n + c2n + d2n


2 n=1

We can express this in terms of the perimeter.

L2
=
4
L2 4A

Now we determine the curves for which L2 = 4A. To do this we find conditions for which A is equal to the upper
bound we obtained for it above. First note that
X
2 2 2 2
 X
n2 a2n + b2n + c2n + d2n

n an + bn + cn + dn =
n=1 n=1

1415
implies that all the coefficients except a0 , c0 , a1 , b1 , c1 and d1 are zero. The constraint,

X X
n a2n + b2n + c2n + d2n

n(an dn bn cn ) =
n=1
2 n=1
then becomes
a1 d1 b1 c1 = a21 + b21 + c21 + d21 .
This implies that d1 = a1 and c1 = b1 . a0 and c0 are arbitrary. Thus curves for which L2 = 4A have the
parametrization
a0 c0
x(t) = + a1 cos t + b1 sin t, y(t) = b1 cos t + a1 sin t.
2 2
Note that  a0  2  c 0 2
x(t) + y(t) = a21 + b21 .
2 2
p
2 2
The curve is a circle of radius a1 + b1 and center (a0 /2, c0 /2).

Solution 28.20
1. The Fourier sine series has the form

X
x(1 x) = an sin(nx).
n=1
The norm of the eigenfunctions is Z 1
1
sin2 (nx) dx = .
0 2
The coefficients in the expansion are
Z 1
an = 2 x(1 x) sin(nx) dx
0
2
= (2 2 cos(n) n sin(n))
3 n3
4
= (1 (1)n ).
3 n3

1416
Thus the Fourier sine series is

8 X sin(nx) 8 X sin((2n 1)x)
x(1 x) = 3 = 3 .
n=1 n3 n=1 (2n 1)3
odd n

The Fourier cosine series has the form



X
x(1 x) = an cos(nx).
n=0

The norm of the eigenfunctions is


Z 1 Z 1
2 1
1 dx = 1, cos2 (nx) dx = .
0 0 2
The coefficients in the expansion are Z 1
1
a0 = x(1 x) dx = ,
0 6
Z 1
an = 2 x(1 x) cos(nx) dx
0
2 4 sin(n) n cos(n)
= +
2 n2 3 n3
2
= (1 + (1)n )
2 n2
Thus the Fourier cosine series is

1 4 X cos(nx) 1 1 X cos(2nx)
x(1 x) = 2 = 2 .
6 n=1 n2 6 n=1 n2
even n

1417
0.2 0.2
0.1 0.1

-1 -0.5 0.5 1 -1 -0.5 0.5 1


-0.1 -0.1
-0.2 -0.2

Figure 28.13: The odd and even periodic extension of x(1 x), 0 x 1.

The Fourier sine series converges to the odd periodic extension of the function. Since this function is C 1 ,
continuously differentiable, we know that the Fourier coefficients must decay as 1/n3 . The Fourier cosine series
converges to the even periodic extension of the function. Since this function is only C 0 , continuous, the Fourier
coefficients must decay as 1/n2 . The odd and even periodic extensions are shown in Figure 28.13. The sine series
is better because of the faster convergence of the series.

2. (a) We substitute x = 0 into the cosine series.


1 1 X 1
0= 2
6 n=1 n2

X 1 2
=
n=1
n2 6

1418
(b) We substitute x = 1/2 into the cosine series.

1 1 1 X cos(n)
= 2
4 6 n=1 n2

X (1)n 2
=
n=1
n2 12

(c) We substitute x = 1/2 into the sine series.



1 8 X sin((2n 1)/2)
= 3
4 n=1 (2n 1)3

X (1)n 3
=
n=1
(2n 1)3 32

1419
Chapter 29

Regular Sturm-Liouville Problems

I learned there are troubles


Of more than one kind.
Some come from ahead
And some come from behind.

But Ive bought a big bat.


Im all ready, you see.
Now my troubles are going
To have troubles with me!
-I Had Trouble in Getting to Solla Sollew
-Theodor S. Geisel, (Dr. Suess)

29.1 Derivation of the Sturm-Liouville Form


Consider the eigenvalue problem on the finite interval [a . . . b],
p2 (x)y 00 + p1 (x)y 0 + p0 (x)y = y,

1420
subject to the homogeneous unmixed boundary conditions
1 y(a) + 2 y 0 (a) = 0, 1 y(b) + 2 y 0 (b) = 0.
Here the coefficient functions pj are real and continuous and p2 > 0 on the interval [a . . . b]. (Note that if p2 were
negative we could multiply the equation by (1) and replace by .) The parameters j and j are real.
We would like to write this problem in a form that can be used to obtain qualitative information about the problem.
First we will write the operator in self-adjoint form. We divide by p2 since it is non-vanishing.
p1 p0
y 00 + y 0 + y = y.
p2 p2 p2
We multiply by an integrating factor.

Z 
p1
I = exp dx eP (x)
p2
 
P (x) 00 p1 0 p0
e y + y + y = eP (x) y
p2 p2 p2
P (x) 0 0 P (x) p0 P (x)

e y +e y=e y
p2 p2
For notational convenience, we define new coefficient functions and parameters.
p0 1
p = eP (x) , q = eP (x) , = eP (x) , = .
p2 p2
Since the pj are continuous and p2 is positive, p, q, and are continuous. p and are positive functions. The problem
now has the form,
(py 0 )0 + qy + y = 0,
subject to the same boundary conditions,
1 y(a) + 2 y 0 (a) = 0, 1 y(b) + 2 y 0 (b) = 0.
This is known as a Regular Sturm-Liouville problem. We will devote much of this chapter to studying the properties of
this problem. We will encounter many results that are analogous to the properties of self-adjoint eigenvalue problems.

1421
Example 29.1.1
 
d dy
ln x + xy = 0, y(1) = y(2) = 0
dx dx

is not a regular Sturm-Liouville problem since ln x vanishes at x = 1.

Result 29.1.1 Any eigenvalue problem of the form

p2 y 00 + p1 y 0 + p0 y = y, for a x b,
1 y(a) + 2 y 0 (a) = 0, 1 y(b) + 2 y 0 (b) = 0,

where the pj are real and continuous and p2 > 0 on [a, b], and the j and j are real can be
written in the form of a regular Sturm-Liouville problem,

(py 0 )0 + qy + y = 0, on a x b,
1 y(a) + 2 y 0 (a) = 0, 1 y(b) + 2 y 0 (b) = 0.

29.2 Properties of Regular Sturm-Liouville Problems

Self-Adjoint. Consider the Regular Sturm-Liouville equation.

L[y] (py 0 )0 + qy = y.

1422
We see that the operator is formally self-adjoint. Now we determine if the problem is self-adjoint.

hv|L[u]i hL[v]|ui = hv|(pu0 )0 + qui h(pv 0 )0 + qv|ui


= [vpu0 ]ba hv 0 |pu0 i + hv|qui [pv 0 u]ba + hpv 0 |u0 i hqv|ui
= [vpu0 ]ba [pv 0 u]ba
= p(b) v(b)u0 (b) v 0 (b)u(b) + p(a) v(a)u0 (a) v 0 (a)u(a)
 
     
1 1
= p(b) v(b) u(b) v(b)u(b)
2 2
     
1 1
+ p(a) v(a) u(a) v(a)u(a)
2 2
=0

Above we used the fact that the i and i are real.


       
1 1 1 1
= , =
2 2 2 2

Thus L[y] subject to the boundary conditions is self-adjoint.

Real Eigenvalues. Let be an eigenvalue with the eigenfunction . We start with Greens formula.

h|L[]i hL[]|i = 0
h| i h|i = 0
h||i + h||i = 0
( )h||i = 0

Since h||i > 0, = 0. Thus the eigenvalues are real.

1423
Infinite Number of Eigenvalues. There are an infinite of eigenvalues which have no finite cluster point. This
result is analogous to the result that we derived for self-adjoint eigenvalue problems. When we cover the Rayleigh
quotient, we will find that there is a least eigenvalue. Since the eigenvalues are distinct and have no finite cluster point,
n as n . Thus the eigenvalues form an ordered sequence,
1 < 2 < 3 < .

Orthogonal Eigenfunctions. Let and be two distinct eigenvalues with the eigenfunctions and . Greens
formula states
h|L[]i hL[]|i = 0.
h| i h|i = 0
h||i + h||i = 0
( )h||i = 0
Since the eigenvalues are distinct, h||i = 0. Thus eigenfunctions corresponding to distinct eigenvalues are orthogonal
with respect to .

Unique Eigenfunctions. Let be an eigenvalue. Suppose and are two independent eigenfunctions corre-
sponding to .
L[] + = 0, L[] + = 0
We take the difference of times the first equation and times the second equation.
L[] L[] = 0
(p0 )0 (p 0 )0 = 0
(p(0 0 ))0 = 0
p(0 0 ) = const
In order to satisfy the boundary conditions, the constant must be zero.
p(0 0 ) = 0

1424
Since p > 0 the second factor vanishes.

0 0 = 0
0 0
2 =0

 
d
=0
dx

= const

and are not independent. Thus each eigenvalue has a unique, (to within a multiplicative constant), eigenfunction.

Real Eigenfunctions. If is an eigenvalue with eigenfunction , then

(p0 )0 + q + = 0.

We take the complex conjugate of this equation.

0 0
 
p + q + = 0.

Thus is also an eigenfunction corresponding to . Are and independent functions, or do they just differ by a
multiplicative constant? (For example, ex and ex are independent functions, but x and x are dependent.) From
our argument on unique eigenfunctions, we see that

= (const).

Since and only differ by a multiplicative constant, the eigenfunctions can be chosen so that they are real-valued
functions.

1425
Rayleighs Quotient. Let be an eigenvalue with the eigenfunction .

h|L[]i = h| i
h|(p0 )0 + qi = h||i
b
p0 a h0 |p|0 i + h|q|i = h||i


b
p0 a + h0 |p|0 i h|q|i

=
h||i

This is known as Rayleighs quotient. It is useful for obtaining qualitative information about the eigenvalues.

Minimum Property of Rayleighs Quotient. Note that since p, q, and are bounded functions, the Rayleigh
quotient is bounded below. Thus there is a least eigenvalue. If we restrict u to be a real continuous function that
satisfies the boundary conditions, then

[puu0 ]ba + hu0 |p|u0 i hu|q|ui


1 = min ,
u hu||ui
where 1 is the least eigenvalue. This form allows us to get upper and lower bounds on 1 .
To derive this formula, we first write it in terms of the operator L.
hu|L[u]i
1 = min
u hu||ui
Since u is continuous and satisfies the boundary conditions, we can expand u in a series of the eigenfunctions.

P P
hu|L[u]i n=1 cn n L [ m=1 cm m ]

=
P P
hu||ui n=1 c n n

m=1 cm m

P P
n=1 cn n m=1 cm m m

=
P P
n=1 c n n

m=1 cm m

1426
We assume that we can interchange summation and integration.
P P
n=1 P m=1 cn cm n hm ||n i
= P
cn cm hm ||n i
Pn=1 2m=1
n=1 |cn | n hn ||n i
= P
|cn |2 hn ||n i
Pn=1
2
n=1 |cn | hn ||n i
1
P 2
n=1 |cn | hn ||n i
= 1

We see that the minimum value of Rayleighs quotient is 1 . The minimum is attained when cn = 0 for all n 2, that
is, when u = c1 1 .

Completeness. The set of the eigenfunctions of a regular Sturm-Liouville problem is complete. That is, any piecewise
continuous function defined on [a, b] can be expanded in a series of the eigenfunctions,

X
f (x) cn n (x),
n=1

where the cn are the generalized Fourier coefficients,

hn ||f i
cn = .
hn ||n i

Here the sum is convergent in the mean. For any fixed x, the sum converges to 21 (f (x )+f (x+ )). If f (x) is continuous
and satisfies the boundary conditions, then the convergence is uniform.

1427
Result 29.2.1 Properties of regular Sturm-Liouville problems.
The eigenvalues are real.
There are an infinite number of eigenvalues

1 < 2 < 3 < .

There is a least eigenvalue 1 but there is no greatest eigenvalue, (n as n ).


For each eigenvalue, there is one unique, (to within a multiplicative constant), eigenfunc-
tion n . The eigenfunctions can be chosen to be real-valued. (Assume the n following
are real-valued.) The eigenfunction n has exactly n 1 zeros in the open interval
a < x < b.
The eigenfunctions are orthogonal with respect to the weighting function (x).
Z b
n (x)m (x)(x) dx = 0 if n 6= m.
a

The eigenfunctions are complete. Any piecewise continuous function f (x) defined on
a x b can be expanded in a series of eigenfunctions

X
f (x) cn n (x),
n=1

where Rb
a f (x)n (x)(x) dx
cn = Rb .
2 (x)(x) dx
a n
1428
The sum converges to 1 (f (x ) + f (x+ )).
Example 29.2.1 A simple example of a Sturm-Liouville problem is
 
d dy
+ y = 0, y(0) = y() = 0.
dx dx

Bounding The Least Eigenvalue. The Rayleigh quotient for the first eigenvalue is
R 0 2
( ) dx
1 = 0R 12 .
0
1 dx
R
Immediately we see that the eigenvalues are non-negative. If 0 (01 )2 dx = 0 then = (const). The only constant that
satisfies the boundary conditions is = 0. Since the trivial solution is not an eigenfunction, = 0 is not an eigenvalue.
Thus all the eigenvalues are positive.
Now we get an upper bound for the first eigenvalue.
R 0 2
(u ) dx
1 = min R0 2
u
0
u dx

where u is continuous and satisfies the boundary conditions. We choose u = x(x ) as a trial function.
R 0 2
(u ) dx
1 R0 2
u dx
R 0
(2x )2 dx
= R 0 2
0
(x x)2 dx
3 /3
= 5
/30
10
= 2

1.013

1429
Finding the Eigenvalues and Eigenfunctions. We consider the cases of negative, zero, and positive eigenvalues
to check our results above.

< 0. The general solution is


x
y = ce +d e x
.
The only solution that satisfies the boundary conditions is the trivial solution, y = 0. Thus there are no negative
eigenvalues.

= 0. The general solution is


y = c + dx.
Again only the trivial solution satisfies the boundary conditions, so = 0 is not an eigenvalue.

> 0. The general solution is


y = c cos( x) + d sin( x).
We apply the boundary conditions.

y(0) = 0 c=0

y() = 0 d sin( ) = 0

The nontrivial solutions are


= n = 1, 2, 3, . . . y = d sin(n).

Thus the eigenvalues and eigenfunctions are

n = n 2 , n = sin(nx), for n = 1, 2, 3, . . .

We can verify that this example satisfies all the properties listed in Result 29.2.1. Note that there are an infinite number
of eigenvalues. There is a least eigenvalue 1 = 1 but there is no greatest eigenvalue. For each eigenvalue, there is one
eigenfunction. The nth eigenfunction sin(nx) has n 1 zeroes in the interval 0 < x < .

1430
Since a series of the eigenfunctions is the familiar Fourier sine series, we know that the eigenfunctions are orthogonal
and complete. We check Rayleighs quotient.
Z  
dn dn 2
 2
pn dx + p dx qn dx
0 0
n = R 2
0
n dx
Z  2 
d(sin(nx)) d(sin(nx))
sin(nx) dx + dx
dx
0 0
= R 2
0
sin (nx)dx
R 2
n cos2 (nx) dx
= 0
/2
2
=n

Example 29.2.2 Consider the eigenvalue problem

x2 y 00 + xy 0 + y = y, y(1) = y(2) = 0.

Since x2 > 0 on [1 . . . 2], we can write this problem in terms of a regular Sturm-Liouville eigenvalue problem. We divide
by x2 .
1 1
y 00 + y 0 + 2 (1 )y = 0
x x
R 1
We multiply by the integrating factor exp( x dx) = exp(ln x) = x and make the substitution, = 1 to obtain
the Sturm-Liouville form.
1
xy 00 + y 0 + y = 0
x
1
(xy 0 )0 + y = 0
x
We see that the eigenfunctions will be orthogonal with respect to the weighting function = 1/x.

1431
The Rayleigh quotient is
b
p0 a + h0 |x|0 i

=
h| x1 |i
h0 |x|0 i
= .
h| x1 |i
If 0 = 0, then only the trivial solution, = 0, satisfies the boundary conditions. Thus the eigenvalues are positive.
Returning to the original problem, we see that the eigenvalues, , satisfy < 1. Since this is an Euler equation, we
can find solutions with the substitution y = x .
( 1) + + 1 = 0
2 + 1 = 0

Note that < 1.


p
= 1
The general solution is
y = c1 x 1
+ c2 x 1
.
We know that the eigenfunctions can be written as real functions. We rewrite the solution.

y = c 1 e 1 ln x
+c2 e 1 ln x

An equivalent form is p p
y = c1 cos( 1 ln x) + c2 sin( 1 ln x).
We apply the boundary conditions.
y(1) = 0 c1 = 0
p
y(2) = 0 sin( 1 ln 2) = 0
p
1 ln 2 = n, for n = 1, 2, . . .

1432
Thus the eigenvalues and eigenfunctions are
 n 2  
ln x
n = 1 , n = sin n for n = 1, 2, . . .
ln 2 ln 2

29.3 Solving Differential Equations With Eigenfunction Expansions


Linear Algebra. Consider the eigenvalue problem,
Ax = x.
If the matrix A has a complete, orthonormal set of eigenvectors {xik } with eigenvalues {k } then we can represent
any vector as a linear combination of the eigenvectors.
n
X
y= ak xik , ak = xik y
k=1
n
X
y= (xik y) xik
k=1

This property allows us to solve the inhomogeneous equation


Ax x = b. (29.1)
Before we try to solve this equation, we should consider the existence/uniqueness of the solution. If is not an
eigenvalue, then the range of L A is Rn . The problem has a unique solution. If is an eigenvalue, then the
null space of L is the span of the eigenvectors of . That is, if = i , then nullspace(L) = span(xii1 , xii2 , . . . , xiim ).
({xii1 , xii2 , . . . , xiim } are the eigenvalues of i .) If b is orthogonal to nullspace(L) then Equation 29.1 has a solution,
but it is not unique. If y is a solution then we can add any linear combination of {xiij } to obtain another solution.
Thus the solutions have the form m
X
x=y+ cj xiij .
j=1

1433
If b is not orthogonal to nullspace(L) then Equation 29.1 has no solution.
Now we solve Equation 29.1. We assume that is not an eigenvalue. We expand the solution x and the inhomo-
geneity in the orthonormal eigenvectors.
Xn Xn
x= ak xik , b= bk xik
k=1 k=1
We substitute the expansions into Equation 29.1.
Xn n
X n
X
A ak xik ak xik = bk xik
k=1 k=1 k=1
n
X n
X n
X
ak k xik ak xik = bk xik
k=1 k=1 k=1
bk
ak =
k
The solution is n
X bk
x= xik .
k=1
k

Inhomogeneous Boundary Value Problems. Consider the self-adjoint eigenvalue problem,


Ly = y, a < x < b,
B1 [y] = B2 [y] = 0.
If the problem has a complete, orthonormal set of eigenfunctions {k } with eigenvalues {k } then we can represent
any square-integrable function as a linear combination of the eigenfunctions.
X Z b
f= fk k , fk = hk |f i = k (x)f (x) dx
k a
X
f= hk |f ik
k

1434
This property allows us to solve the inhomogeneous differential equation

Ly y = f, a < x < b, (29.2)


B1 [y] = B2 [y] = 0.

Before we try to solve this equation, we should consider the existence/uniqueness of the solution. If is not an
eigenvalue, then the range of L is the space of square-integrable functions. The problem has a unique solution. If
is an eigenvalue, then the null space of L is the span of the eigenfunctions of . That is, if = i , then nullspace(L) =
span(i1 , i2 , . . . , im ). ({i1 , i2 , . . . , im } are the eigenvalues of i .) If f is orthogonal to nullspace(L ) then
Equation 29.2 has a solution, but it is not unique. If u is a solution then we can add any linear combination of {ij }
to obtain another solution. Thus the solutions have the form
m
X
y =u+ cj ij .
j=1

If f is not orthogonal to nullspace(L ) then Equation 29.2 has no solution.


Now we solve Equation 29.2. We assume that is not an eigenvalue. We expand the solution y and the inhomo-
geneity in the orthonormal eigenfunctions.
X X
y= yk k , f= fk k
k k

It would be handy if we could substitute the expansions into Equation 29.2. However, the expansion of a function is
not necessarily differentiable. Thus we demonstrate that since y is C 2 (a . . . b) and satisfies the boundary conditions
B1 [y] = B2 [y] = 0, we are justified in substituting it into the differential equation. In particular, we will show that
" #
X X X
L[y] = L yk k = yk L [k ] = yk k k .
k k k

To do this we will use Greens identity. If u and v are C 2 (a . . . b) and satisfy the boundary conditions B1 [y] = B2 [y] = 0
then
hu|L[v]i = hL[u]|vi.

1435
First we assume that we can differentiate y term-by-term.
X
L[y] = yk k k
k

Now we directly expand L[y] and show that we get the same result.
X
L[y] = ck k
k

ck = hk |L[y]i
= hL[k ]|yi
= hk k |yi
= k hk |yi
= k y k
X
L[y] = yk k
k
The series representation of y may not be differentiable, but we are justified in applying L term-by-term.
Now we substitute the expansions into Equation 29.2.
" #
X X X
L yk k yk k = fk k
k k k
X X X
k yk k yk k = fk k
k k k
fk
yk =
k
The solution is
X fk
y= k
k
k

1436
Consider a second order, inhomogeneous problem.

L[y] = f (x), B1 [y] = b1 , B2 [y] = b2

We will expand the solution in an orthogonal basis.


X
y= an n
n

We would like to substitute the series into the differential equation, but in general we are not allowed to differentiate
such series. To get around this, we use integration by parts to move derivatives from the solution y, to the n .

Example 29.3.1 Consider the problem,

y 00 + y = f (x), y(0) = a, y() = b,

where 6= n2 , n Z+ . We expand the solution in a cosine series.

r
y0 X 2
y(x) = + yn cos(nx)
n=1

We also expand the inhomogeneous term.

r
f0 X 2
f (x) = + fn cos(nx)
n=1

We multiply the differential equation by the orthonormal functions and integrate over the interval. We neglect the

1437

special case 0 = 1/ for now.
Z r Z r Z r
2 2 2
cos(nx)y 00 dx + cos(nx)y dx = f (x) dx
0 0 0
"r # Z r

2 2
cos(nx)y 0 (x) + n sin(nx)y 0 (x) dx + yn = fn
0
r "r 0 # Z r

2 2 2 2
((1)n y 0 () y 0 (0)) + n sin(nx)y(x) n cos(nx)y(x) dx + yn = fn
0
0
r
2
((1)n y 0 () y 0 (0)) n2 yn + yn = fn

Unfortunately we dont know the values of y 0 (0) and y 0 ().


CONTINUE HERE

1438
29.4 Exercises
Exercise 29.1
Find the eigenvalues and eigenfunctions of

y 00 + 2y 0 + y = 0, y(a) = y(b) = 0,

where a < b.
Write the problem in Sturm Liouville form. Verify that the eigenvalues and eigenfunctions satisfy the properties of
regular Sturm-Liouville problems. Find the coefficients in the expansion of an arbitrary function f (x) in a series of the
eigenfunctions.
Hint, Solution
Exercise 29.2
Find the eigenvalues and eigenfunctions of the boundary value problem


y 00 + y=0
(x + 1)2

on the interval 1 x 2 with boundary conditions y(1) = y(2) = 0. Discuss how the results satisfy the properties of
Sturm-Liouville problems.
Hint, Solution
Exercise 29.3
Find the eigenvalues and eigenfunctions of

2 + 1 0
y 00 + y + 2 y = 0, y(a) = y(b) = 0,
x x
where 0 < a < b. Write the problem in Sturm Liouville form. Verify that the eigenvalues and eigenfunctions satisfy the
properties of regular Sturm-Liouville problems. Find the coefficients in the expansion of an arbitrary function f (x) in a
series of the eigenfunctions.
Hint, Solution

1439
Exercise 29.4
Find the eigenvalues and eigenfunctions of

y 00 y 0 + y = 0, y(0) = y(1) = 0.

Find the coefficients in the expansion of an arbitrary, f (x), in a series of the eigenfunctions.
Hint, Solution
Exercise 29.5
Consider
y 00 + y = f (x), y(0) = 0, y(1) + y 0 (1) = 0. (29.3)
The associated eigenvalue problem is

y 00 + y = y y(0) = 0 y(1) + y 0 (1) = 0.

Find the eigenfunctions for this problem and the equation which the eigenvalues must satisfy.
To do this, consider the eigenvalues and eigenfunctions for,

y 00 + y = 0, y(0) = 0, y(1) + y 0 (1) = 0.

Show that the transcendental equation for has infinitely many roots 1 < 2 < 3 < . Find the limit of n as
n . How is this limit approached?
Give the general solution of Equation 29.3 in terms of the eigenfunctions.
Hint, Solution
Exercise 29.6
Consider
y 00 + y = f (x) y(0) = 0 y(1) + y 0 (1) = 0.
Find the eigenfunctions for this problem and the equation which the eigenvalues satisfy. Give the general solution in
terms of these eigenfunctions.
Hint, Solution

1440
Exercise 29.7
Show that the eigenvalue problem,

y 00 + y = 0, y(0) = 0, y 0 (0) y(1) = 0,

(note the mixed boundary condition), has only one real eigenvalue. Find it and the corresponding eigenfunction. Show
that this problem is not self-adjoint. Thus the proof, valid for unmixed, homogeneous boundary conditions, that all
eigenvalues are real fails in this case.
Hint, Solution
Exercise 29.8
Determine the Rayleigh quotient, R[] for,
1
y 00 + y 0 + y = 0, |y(0)| < , y(1) = 0.
x
x in R[] to deduce that the smallest zero of J0 (x), the Bessel function of the first kind
Use the trial function = 1
and order zero, is less than 6.
Hint, Solution
Exercise 29.9
Discuss the eigenvalues of the equation

y 00 + q(z)y = 0, y(0) = y() = 0

where (
a > 0, 0 z l
q(z) =
b > 0, l < z .
This is an example that indicates that the results we obtained in class for eigenfunctions and eigenvalues with q(z)
continuous and bounded also hold if q(z) is simply integrable; that is
Z
|q(z)| dz
0

1441
is finite.
Hint, Solution
Exercise 29.10
1. Find conditions on the smooth real functions p(x), q(x), r(x) and s(x) so that the eigenvalues, , of:

Lv (p(x)v 00 (x))00 (q(x)v 0 (x))0 + r(x)v(x) = s(x)v(x), a<x<b


v(a) = v 00 (a) = 0
v 00 (b) = 0, p(b)v 000 (b) q(b)v 0 (b) = 0

are positive. Prove the assertion.

2. Show that for any smooth p(x), q(x), r(x) and s(x) the eigenfunctions belonging to distinct eigenvalues are
orthogonal relative to the weight s(x). That is:
Z b
vm (x)vk (x)s(x) dx = 0 if k 6= m .
a

3. Find the eigenvalues and eigenfunctions for:

d4 n(0) = 00 (0) = 0,
= ,
dx4 (1) = 00 (1) = 0.

Hint, Solution

1442
29.5 Hints
Hint 29.1

Hint 29.2

Hint 29.3

Hint 29.4
Write the problem in Sturm-Liouville form to show that the eigenfunctions are orthogonal with respect to the weighting
function = ex .
Hint 29.5
Note that the solution is a regular Sturm-Liouville problem and thus the eigenvalues are real. Use the Rayleigh quotient
to show that there are only positive eigenvalues. Informally show that there are an infinite number of eigenvalues with
a graph.
Hint 29.6

Hint 29.7
Find the solution for = 0, < 0 and > 0. A problem is self-adjoint if it satisfies Greens identity.

Hint 29.8
Write the equation in self-adjoint form. The Bessel equation of the first kind and order zero satisfies the problem,

1
y 00 + y 0 + y = 0, |y(0)| < , y(r) = 0,
x
where r is a positive root of J0 (x). Make the change of variables = x/r, u() = y(x).

1443
Hint 29.9

Hint 29.10

1444
29.6 Solutions
Solution 29.1
Recall that constant coefficient equations are shift invariant. If u(x) is a solution, then so is u(x c).
We substitute y = ex into the constant coefficient equation.
y 00 + 2y 0 + y = 0
2 + 2 + = 0

= 2
First we consider the case = 2 . A set of solutions of the differential equation is
 x
e , x ex

The homogeneous solution that satisfies the left boundary condition y(a) = 0 is
y = c(x a) ex .
Since only the trivial solution with c = 0 satisfies the right boundary condition, = 2 is not an eigenvalue.
Next we consider the case 6= 2 . We write

= 2 .

Note that <( 2 ) 0. A set of solutions of the differential equation is
n o
( 2 )x
e

By taking the sum and difference of these solutions we obtain a new set of linearly independent solutions.
n    o
x 2 x 2
e cos x ,e sin x

The solution which satisfies the left boundary condition is


 
x 2
y=c e sin (x a) .

1445
For nontrivial solutions, the right boundary condition y(b) = 0 imposes the constraint
 
eb sin 2 (b a) = 0

2 (b a) = n, n Z

We have the eigenvalues


 2
2 n
n = + , nZ
ba
with the eigenfunctions
 
x xa
n = e sin n .
ba
To write the problem in Sturm-Liouville form, we multiply by the integrating factor
R
2 dx
e = e2x .
0
e2x y 0 + e2x y = 0, y(a) = y(b) = 0
Now we verify that the Sturm-Liouville properties are satisfied.
The eigenvalues  2
2 n
n = + , nZ
ba
are real.

There are an infinite number of eigenvalues

1 < 2 < 3 < ,


 2  2  2
2 3
2 + 2
< + < + 2
< .
ba ba ba

1446
There is a least eigenvalue
 2
2
1 = + ,
ba
but there is no greatest eigenvalue, (n as n ).

For each eigenvalue, we found one unique, (to within a multiplicative constant), eigenfunction n . We were able
to choose the eigenfunctions to be real-valued. The eigenfunction
 
x xa
n = e sin n .
ba

has exactly n 1 zeros in the open interval a < x < b.

The eigenfunctions are orthogonal with respect to the weighting function (x) = e2ax .
b b    
x a x x a 2ax
Z Z
x
n (x)m (x)(x) dx = e sin n e sin m e dx
a a ba ba
Z b    
xa xa
= sin n sin m dx
a ba ba
ba
Z
= sin(nx) sin(mx) dx
0
ba
Z
= (cos((n m)x) cos((n + m)x)) dx
2 0
= 0 if n 6= m

The eigenfunctions are complete. Any piecewise continuous function f (x) defined on a x b can be expanded
in a series of eigenfunctions
X
f (x) cn n (x),
n=1

1447
where
Rb
a
f (x)n (x)(x) dx
cn = Rb .
2 (x)(x) dx
a n

The sum converges to 12 (f (x ) + f (x+ )). (We do not prove this property.)

The eigenvalues can be related to the eigenfunctions with the Rayleigh quotient.

b R b  dn 2 
pn d
 2
n
dx a
+ a
p dx
q n dx
n = Rb
2 dx
a n

R b 2x x n 2 
xa xa

a
e e ba
cos n ba
sin n ba
dx
= Rb 2
a
ex sin n xa ba
e2x dx
 
n 2
Rb  2 xa
 n xa
 xa
 2 2 xa
a ba
cos n ba 2 ba cos n ba sin n ba + sin n ba dx
= Rb 2
sin n xa

a ba
dx
 
n 2
R  2 n 2 2
0 ba
cos (x) 2 ba
cos(x) sin(x) + sin (x) dx
= R 2
0
sin (x) dx
 2
n
= 2 +
ba

Now we expand a function f (x) in a series of the eigenfunctions.

 
X
x xa
f (x) cn e sin n ,
n=1
ba

1448
where
Rb
f (x)n (x)(x) dx
a
cn = Rb
2 (x)(x) dx
a n
Z b  
2n x xa
= f (x) e sin n dx
ba a ba

Solution 29.2
This is an Euler equation. We substitute y = (x + 1) into the equation.


y 00 + y=0
(x + 1)2
( 1) + = 0

1 1 4
=
2
First consider the case = 1/4. A set of solutions is
n o
x + 1, x + 1 ln(x + 1) .

Another set of solutions is



  
x+1
x + 1, x + 1 ln .
2
The solution which satisfies the boundary condition y(1) = 0 is


 
x+1
y = c x + 1 ln .
2

Since only the trivial solution satisfies the y(2) = 0, = 1/4 is not an eigenvalue.

1449
Now consider the case 6= 1/4. A set of solutions is
n o
(x + 1)(1+ 14)/2 , (x + 1)(1 14)/2 .

We can write this in terms of the exponential and the logarithm.


 

  
4 1 4 1
x + 1 exp ln(x + 1) , x + 1 exp ln(x + 1) .
2 2
Note that  

    
4 1 x+1 4 1 x+1
x + 1 exp ln , x + 1 exp ln .
2 2 2 2
is also a set of solutions. The new factor of 2 in the logarithm just multiplies the solutions by a constant. We write
the solution in terms of the cosine and sine.
 

    
4 1 x+1 4 1 x+1
x + 1 cos ln , x + 1 sin ln .
2 2 2 2
The solution of the differential equation which satisfies the boundary condition y(1) = 0 is


 
1 4 x+1
y = c x + 1 sin ln .
2 2
Now we use the second boundary condition to find the eigenvalues.
y(2) = 0
  
4 1 3
sin ln =0
2 2
 
4 1 3
ln = n, n Z
2 2
 2 !
1 2n
= 1+ , nZ
4 ln(3/2)

1450
n = 0 gives us a trivial solution, so we discard it. Discarding duplicate solutions, The eigenvalues and eigenfunctions
are
2

  
1 n ln((x + 1)/2)
n = + , yn = x + 1 sin n , n Z+ .
4 ln(3/2) ln(3/2)
Now we verify that the eigenvalues and eigenfunctions satisfy the properties of regular Sturm-Liouville problems.
The eigenvalues are real.
There are an infinite number of eigenvalues
1 < 2 < 3 <
 2  2  2
1 1 2 1 3
+ < + < + <
4 ln(3/2) 4 ln(3/2) 4 ln(3/2)
There is a least least eigenvalue  2
1
1 = + ,
4 ln(3/2)
but there is no greatest eigenvalue.
The eigenfunctions are orthogonal with respect to the weighting function (x) = 1/(x + 1)2 . Let n 6= m.
Z 2
yn (x)ym (x)(x) dx
1
ln((x + 1)/2)
Z 2    
ln((x + 1)/2) 1
= x + 1 sin n x + 1 sin m dx
1 ln(3/2) ln(3/2) (x + 1)2
Z
ln(3/2)
= sin(nx) sin(mx) dx
0
ln(3/2)
Z
= (cos((n m)x) cos((n + m)x)) dx
2 0
=0

1451
The eigenfunctions are complete. A function f (x) defined on (1 . . . 2) has the series representation


 
X X ln((x + 1)/2)
f (x) cn yn (x) = cn x + 1 sin n ,
n=1 n=1
ln(3/2)

where
2
hyn |1/(x + 1)2 |f i
Z  
2 ln((x + 1)/2) 1
cn = = sin n f (x) dx
hyn |1/(x + 1)2 |yn i ln(3/2) 1 ln(3/2) (x + 1)3/2

Solution 29.3
Recall that Euler equations are scale invariant. If u(x) is a solution, then so is u(cx) for any nonzero constant c.
We substitute y = x into the Euler equation.
2 + 1 0
y 00 + y + 2y = 0
x x
( 1) + (2 + 1) + = 0
2 + 2 + = 0

= 2

First we consider the case = 2 . A set of solutions of the differential equation is



x , x ln x

The homogeneous solution that satisfies the left boundary condition y(a) = 0 is
x
y = cx (ln x ln a) = cx ln .
a
Since only the trivial solution with c = 0 satisfies the right boundary condition, = 2 is not an eigenvalue.
Next we consider the case 6= 2 . We write

= 2 .

1452

Note that <( 2 ) 0. A set of solutions of the differential equation is
n o
2
x
n o
2
x e ln x .

By taking the sum and difference of these solutions we obtain a new set of linearly independent solutions.
n     o
x cos 2 ln x , x sin 2 ln x ,

The solution which satisfies the left boundary condition is


  x 
2
y = cx sin ln .
a
For nontrivial solutions, the right boundary condition y(b) = 0 imposes the constraint

  
b
b sin 2 ln
a

 
b
2 ln = n, n Z
a
We have the eigenvalues
 2
2 n
n = + , nZ
ln(b/a)
with the eigenfunctions
 
ln(x/a)
n = x sin n .
ln(b/a)
To write the problem in Sturm-Liouville form, we multiply by the integrating factor
R
(2+1)/x dx
e = e(2+1) ln x = x2+1 .

1453
0
x2+1 y 0 + x21 y = 0, y(a) = y(b) = 0

Now we verify that the Sturm-Liouville properties are satisfied.

The eigenvalues
 2
2 n
n = + , nZ
ln(b/a)
are real.

There are an infinite number of eigenvalues

1 < 2 < 3 < ,


 2  2  2
2 3
2 + 2
< + < + 2
<
ln(b/a) ln(b/a) ln(b/a)

There is a least eigenvalue


 2
2
1 = + ,
ln(b/a)
but there is no greatest eigenvalue, (n as n ).

For each eigenvalue, we found one unique, (to within a multiplicative constant), eigenfunction n . We were able
to choose the eigenfunctions to be real-valued. The eigenfunction
 
ln(x/a)
n = x sin n .
ln(b/a)

has exactly n 1 zeros in the open interval a < x < b.

1454
The eigenfunctions are orthogonal with respect to the weighting function (x) = x21 .

Z b Z b    
ln(x/a) ln(x/a)
n (x)m (x)(x) dx = x sin n x sin m x21 dx
a a ln(b/a) ln(b/a)
Z b    
ln(x/a) ln(x/a) 1
= sin n sin m dx
a ln(b/a) ln(b/a) x
ln(b/a)
Z
= sin(nx) sin(mx) dx
0
ln(b/a)
Z
= (cos((n m)x) cos((n + m)x)) dx
2 0
= 0 if n 6= m

The eigenfunctions are complete. Any piecewise continuous function f (x) defined on a x b can be expanded
in a series of eigenfunctions

X
f (x) cn n (x),
n=1

where
Rb
a
f (x)n (x)(x) dx
cn = Rb .
2 (x)(x) dx
a n

The sum converges to 12 (f (x ) + f (x+ )). (We do not prove this property.)

1455
The eigenvalues can be related to the eigenfunctions with the Rayleigh quotient.
b R b  dn 2 
pn d
 2
n
dx a
+ a
p dx
qn dx
n = Rb
2 dx
a n
      2 
Rb 2+1 1 n ln(x/a) ln(x/a)
a
x x ln(b/a)
cos n ln(b/a) sin n ln(b/a) dx
= Rb  2
x sin n ln(x/a) x21 dx
a ln(b/a)
 
R b  n 2
a ln(b/a)
cos () 2 ln(b/a) cos () sin () + sin () x1 dx
2 n 2 2

= R b 2  ln(x/a) 
a
sin n ln(b/a) x1 dx
 
R  n 2 2 n 2 2
0 ln(b/a)
cos (x) 2 ln(b/a) cos(x) sin(x) + sin (x) dx
= R 2
0
sin (x) dx
 2
n
= 2 +
ln(b/a)

Now we expand a function f (x) in a series of the eigenfunctions.


 
X
ln(x/a)
f (x) cn x sin n ,
n=1
ln(b/a)

where
Rb
f (x)n (x)(x) dx
a
cn = Rb
2 (x)(x) dx
a n
Z b  
2n 1 ln(x/a)
= f (x)x sin n dx
ln(b/a) a ln(b/a)

1456
Solution 29.4

y 00 y 0 + y = 0, y(0) = y(1) = 0.
The factor that will put this equation in Sturm-Liouville form is
Z x 
F (x) = exp 1 dx = ex .

The differential equation becomes


d x 0 
e y + ex y = 0.
dx
Thus we see that the eigenfunctions will be orthogonal with respect to the weighting function = ex .
Substituting y = ex into the differential equation yields

2 + = 0

1 1 4
=
2
1 p
= 1/4 .
2
If < 1/4 then the solutions to the differential equation are exponential and only the trivial solution satisfies the
boundary conditions.
If = 1/4 then the solution is y = c1 ex/2 +c2 x ex/2 and again only the trivial solution satisfies the boundary
conditions.
Now consider the case that > 1/4.
1 p
= 1/4
2
The solutions are p p
ex/2 cos( 1/4 x), ex/2 sin( 1/4 x).
The left boundary condition gives us p
y = c ex/2 sin( 1/4 x).

1457
The right boundary condition demands that
p
1/4 = n, n = 1, 2, . . .

Thus we see that the eigenvalues and eigenfunctions are


1
n = + (n)2 , yn = ex/2 sin(nx).
4
If f (x) is a piecewise continuous function then we can expand it in a series of the eigenfunctions.

X
f (x) = an ex/2 sin(nx)
n=1

The coefficients are


R1
f (x) ex ex/2 sin(nx) dx
an = 0R 1
ex (ex/2 sin(nx))2 dx
R 10
f (x) ex/2 sin(nx) dx
= 0 R1 2
sin (nx) dx
Z 1 0
=2 f (x) ex/2 sin(nx) dx.
0

Solution 29.5
Consider the eigenvalue problem
y 00 + y = 0 y(0) = 0 y(1) + y 0 (1) = 0.
Since this is a Sturm-Liouville problem, there are only real eigenvalues. By the Rayleigh quotient, the eigenvalues are
1 R   
d 1 d 2
dx + 0 dx
dx
0
= R1 ,
0
2 dx

1458
R 1  d 2 
2 (1) + 0 dx
dx
= R1 .
0
2 dx
This demonstrates that there are only positive eigenvalues. The general solution of the differential equation for positive,
real is    
y = c1 cos x + c2 sin x .
The solution that satisfies the left boundary condition is
 
y = c sin x .

For nontrivial solutions we must have    


sin + cos =0
 
= tan .
 
The positive solutions of this equation are eigenvalues with corresponding eigenfunctions sin x . In Figure 29.1
we plot the functions x and tan(x) and draw vertical lines at x = (n 1/2), n N.
From this we see that there are an infinite number of eigenvalues, 1 < 2 < 3 < . In the limit as n ,
n (n 1/2). The limit is approached from above.
Now consider the eigenvalue problem

y 00 + y = y y(0) = 0 y(1) + y 0 (1) = 0.

From above we see that the eigenvalues satisfy


p p 
1 = tan 1

and that there are an infinite number of eigenvalues. For large n, n 1 (n 1/2). The eigenfunctions are
p 
n = sin 1 n x .

1459
Figure 29.1: x and tan(x).

To solve the inhomogeneous problem, we expand the solution and the inhomogeneity in a series of the eigenfunctions.
R1
X f (x)n (x) dx
f= fn n , fn = 0 R 1
n=1 2 (x) dx
0 n

X
y= yn n
n=1

We substitite the expansions into the differential equation to determine the coefficients.
y 00 + y = f

X
X
n yn n = fn n
n=1 n=1

X fn p 
y= sin 1 n x

n=1 n

1460
Solution 29.6
Consider the eigenvalue problem
y 00 + y = y y(0) = 0 y(1) + y 0 (1) = 0.
From Exercise 29.5 we see that the eigenvalues satisfy
p p 
1 = tan 1

and that there are an infinite number of eigenvalues. For large n, n 1 (n 1/2). The eigenfunctions are
p 
n = sin 1 n x .

To solve the inhomogeneous problem, we expand the solution and the inhomogeneity in a series of the eigenfunctions.
R1
X f (x)n (x) dx
f= fn n , fn = 0 R 1
n=1 2 (x) dx
0 n

X
y= yn n
n=1

We substitite the expansions into the differential equation to determine the coefficients.

y 00 + y = f

X
X
n yn n = fn n
n=1 n=1

X fn p 
y= sin 1 n x

n=1 n

Solution 29.7
First consider = 0. The general solution is
y = c1 + c2 x.

1461
y = cx satisfies the boundary conditions. Thus = 0 is an eigenvalue.
Now consider negative real . The general solution is
   
y = c1 cosh x + c2 sinh x .

The solution that satisfies the left boundary condition is


 
y = c sinh x .

For nontrivial solutions of the boundary value problem, there must be negative real solutions of
 
sinh = 0.

Since x = sinh x has no nonzero real solutions, this equation has no solutions for negative real . There are no negative
real eigenvalues.
Finally consider positive real . The general solution is
   
y = c1 cos x + c2 sin x .

The solution that satisfies the left boundary condition is


 
y = c sin x .

For nontrivial solutions of the boundary value problem, there must be positive real solutions of
 
sin = 0.

Since x = sin x has no nonzero real solutions, this equation has no solutions for positive real . There are no positive
real eigenvalues.
There is only one real eigenvalue, = 0, with corresponding eigenfunction = x.

1462
The difficulty with the boundary conditions, y(0) = 0, y 0 (0) y(1) = 0 is that the problem is not self-adjoint. We
demonstrate this by showing that the problem does not satisfy Greens identity. Let u and v be two functions that
satisfy the boundary conditions, but not necessarily the differential equation.
hu, L[v]i hL[u], vi = hu, v 00 i hu00 , vi
1 1
= [uv 0 ]0 hu0 , v 0 i hu0 , v 0 i [u0 v]0 + hu0 , v 0 i hu0 , v 0 i
= u(1)v 0 (1) u0 (1)v(1)
Greens identity is not satisfied,
hu, L[v]i hL[u], vi =
6 0;
The problem is not self-adjoint.
Solution 29.8
First we write the equation in formally self-adjoint form,
L[y] (xy 0 )0 = xy, |y(0)| < , y(1) = 0.
Let be an eigenvalue with corresponding eigenfunction . We derive the Rayleigh quotient for .
h, L[]i = h, xi
h, (x0 )0 i = h, xi
1
[x0 ]0 h0 , x0 i = h, xi

We apply the boundary conditions and solve for .

h0 , x0 i
=
h, xi

The Bessel equation of the first kind and order zero satisfies the problem,
1
y 00 + y 0 + y = 0, |y(0)| < , y(r) = 0,
x

1463
where r is a positive root of J0 (x). We make the change of variables = x/r, u() = y(x) to obtain the problem
1 00 1 1 0
2
u + u + u = 0, |u(0)| < , u(1) = 0,
r r r
1
u00 + u0 + r2 u = 0, |u(0)| < , u(1) = 0.

Now r2 is the eigenvalue of the problem for u(). From the Rayleigh quotient, the minimum eigenvalue obeys the
inequality
h0 , x0 i
r2 ,
h, xi
where is any test function that satisfies the boundary conditions. Taking = 1 x we obtain,
R1
2 (1)x(1) dx
r R1 0 = 6,
0
(1 x)x(1 x) dx

r 6

Thus the smallest zero of J0 (x) is less than or equal to 6 2.4494. (The smallest zero of J0 (x) is approximately
2.40483.)

Solution 29.9
We assume that 0 < l < .
Recall that the solution of a second order differential equation with piecewise continuous coefficient functions is
piecewise C 2 . This means that the solution is C 2 except for a finite number of points where it is C 1 .
First consider the case = 0. A set of linearly independent solutions of the differential equation is {1, z}. The
solution which satisfies y(0) = 0 is y1 = c1 z. The solution which satisfies y() = 0 is y2 = c2 ( z). There is a
solution for the problem if there are there are values of c1 and c2 such that y1 and y2 have the same position and slope
at z = l.
y1 (l) = y2 (l), y10 (l) = y20 (l)
c1 l = c2 ( l), c1 = c2

1464
Since there is only the trivial solution, c1 = c2 = 0, = 0 is not an eigenvalue.
Now consider 6= 0. For 0 z l a set of linearly independent solutions is
n o
cos( az), sin( az) .

The solution which satisfies y(0) = 0 is


y1 = c1 sin( az).
For l < z a set of linearly independent solutions is
n o
cos( bz), sin( bz) .

The solution which satisfies y() = 0 is


y2 = c2 sin( b( z)).
6= 0 is an eigenvalue if there are nontrivial solutions of

y1 (l) = y2 (l), y10 (l) = y20 (l)



c1 sin( al) = c2 sin( b( l)), c1 a cos( al) = c2 b cos( b( l))
p
We divide the second equation by () since 6= 0 and write this as a linear algebra problem.
    
sin( al)
sin( b( l)) c1 0
=
a cos( al) b sin( b( l)) c 2 0

This system of equations has nontrivial solutions if and only if the determinant of the matrix is zero.

b sin( al) sin( b( l)) + a cos( al) sin( b( l)) = 0

We can use trigonometric identities to write this equation as


   
( b a) sin (l a ( l) b) + ( b + a) sin (l a + ( l) b) = 0

1465
Clearly this equation has an infinite number of solutions for real, positive . However, it is not clear that this equation
does not have non-real solutions. In order to prove that, we will show that the problem is self-adjoint. Before going on
to that we note that the eigenfunctions have the form

( 
sin an z 0zl
n (z) = 
sin bn ( z) l < z .

Now we prove that the problem is self-adjoint. We consider the class of functions which are C 2 in (0 . . . ) except
at the interior point x = l where they are C 1 and which satisfy the boundary conditions y(0) = y() = 0. Note that
the differential operator is not defined at the point x = l. Thus Greens identity,

hu|q|Lvi = hLu|q|vi

is not well-defined. To remedy this we must define a new inner product. We choose

Z l Z
hu|vi uv dx + uv dx.
0 l

This new inner product does not require differentiability at the point x = l.
The problem is self-adjoint if Greens indentity is satisfied. Let u and v be elements of our class of functions. In

1466
addition to the boundary conditions, we will use the fact that u and v satisfy y(l ) = y(l+ ) and y 0 (l ) = y 0 (l+ ).

Z l Z
00
hv|Lui = vu dx + vu00 dx
0 l
Z l Z
0 l 0 0 0
= [vu ]0 v u dx + [vu ]l v 0 u0 dx
0 l
Z l Z
0 0 0 0
= v(l)u (l) v u dx v(l)u (l) v 0 u0 dx
0 l
Z l Z
0 0
= v u dx v 0 u0 dx
0 l
Z l Z
0 l 00 0
= [v u]0 + v u dx [v u]l + v 00 u dx
0 l
Z l Z
0 00 0
= v (l)u(l) + v u dx + v (l)u(l) + v 00 u dx
0 l
Z l Z
= v 00 u dx + v 00 u dx
0 l
= hLv|Lui

The problem is self-adjoint. Hence the eigenvalues are real. There are an infinite number of positive, real eigenvalues
n .

Solution 29.10
1. Let v be an eigenfunction with the eigenvalue . We start with the differential equation and then take the inner
product with v.

(pv 00 )00 (qv 0 )0 + rv = sv


hv, (pv 00 )00 (qv 0 )0 + rvi = hv, svi

1467
We use integration by parts and utilize the homogeneous boundary conditions.
b b
[v(pv 00 )0 ]a hv 0 , (pv 00 )0 i [vqv 0 ]a + hv 0 , qv 0 i + hv, rvi = hv, svi
b
[v 0 pv 00 ]a + hv 00 , pv 00 i + hv 0 , qv 0 i + hv, rvi = hv, svi
hv 00 , pv 00 i + hv 0 , qv 0 i + hv, rvi
=
hv, svi
We see that if p, q, r, s 0 then the eigenvalues will be positive. (Of course we assume that p and s are not
identically zero.)
2. First we prove that this problem is self-adjoint. Let u and v be functions that satisfy the boundary conditions,
but do not necessarily satsify the differential equation.
hv, L[u]i hL[v], ui = hv, (pu00 )00 (qu0 )0 + rui h(pv 00 )00 (qv 0 )0 + rv, ui

Following our work in part (a) we use integration by parts to move the derivatives.

= (hv 00 , pu00 i + hv 0 , qu0 i + hv, rui) (hpv 00 , u00 i + hqv 0 , u0 i + hrv, ui)
=0
This problem satisfies Greens identity,
hv, L[u]i hL[v], ui = 0,
and is thus self-adjoint.
Let vk and vm be eigenfunctions corresponding to the distinct eigenvalues k and m . We start with Greens
identity.
hvk , L[vm ]i hL[vk ], vm i = 0
hvk , m svm i hk svk , vm i = 0
(m k )hvk , svm i = 0
hvk , svm i = 0
The eigenfunctions are orthogonal with respect to the weighting function s.

1468
3. From part (a) we know that there are only positive eigenvalues. The general solution of the differential equation
is
= c1 cos(1/4 x) + c2 cosh(1/4 x) + c3 sin(1/4 x) + c4 sinh(1/4 x).
Applying the condition (0) = 0 we obtain

= c1 (cos(1/4 x) cosh(1/4 x)) + c2 sin(1/4 x) + c3 sinh(1/4 x).

The condition 00 (0) = 0 reduces this to

= c1 sin(1/4 x) + c2 sinh(1/4 x).

We substitute the solution into the two right boundary conditions.

c1 sin(1/4 ) + c2 sinh(1/4 ) = 0
c1 1/2 sin(1/4 ) + c2 1/2 sinh(1/4 ) = 0

We see that sin(1/4 ) = 0. The eigenvalues and eigenfunctions are

n = (n)4 , n = sin(nx), n N.

1469
Chapter 30

Integrals and Convergence

Never try to teach a pig to sing. It wastes your time and annoys the pig.
-?

30.1 Uniform Convergence of Integrals


Consider the improper integral Z
f (x, t) dt.
c
The integral is convergent to S(x) if, given any  > 0, there exists T (x, ) such that
Z


f (x, t) dt S(x) <  for all > T (x, ).

c

The sum is uniformly convergent if T is independent of x.


Similar to the Weierstrass M-test for infinite sums we have
R a uniform convergence test forR
integrals. If there exists
a continuous function M (t) such that |f (x, t)| M (t) and c M (t) dt is convergent, then c f (x, t) dt is uniformly
convergent.

1470
R
If c
f (x, t) dt is uniformly convergent, we have the following properties:
If f (x, t) is continuous for x [a, b] and t [c, ) then for a < x0 < b,
Z Z  
lim f (x, t) dt = lim f (x, t) dt.
xx0 c c xx0

If a x1 < x2 b then we can interchange the order of integration.


Z x2 Z  Z Z x2 
f (x, t) dt dx = f (x, t) dx dt
x1 c c x1

f
If x
is continuous, then Z Z
d
f (x, t) dt = f (x, t) dt.
dx c c x

30.2 The Riemann-Lebesgue Lemma


Rb
Result 30.2.1 If a |f (x)| dx exists, then
Z b
f (x) sin(x) dx 0 as .
a
Before we try to justify the Riemann-Lebesgue lemma, we will need a preliminary result. Let be a positive constant.
Z b  b
1
sin(x) dx = cos(x)


a a

2
.

1471
We will prove the Riemann-Lebesgue lemma for the case when f (x) has limited total fluctuation on the interval
(a, b). We can express f (x) as the difference of two functions

f (x) = + (x) (x),

where + and are positive, increasing, bounded functions.


From the mean value theorem for positive, increasing functions, there exists an x0 , a x0 b, such that
Z b Z b


+ (x) sin(x) dx =

+ (b) sin(x) dx

a x0
2
|+ (b)| .

Similarly, Z b
| (b)| 2 .

(x) sin(x) dx

a

Thus
Z b
2
f (x) sin(x) dx (|+ (b)| + | (b)|)

a
0 as .

30.3 Cauchy Principal Value


30.3.1 Integrals on an Infinite Domain
R
The improper integral
f (x) dx is defined
Z Z 0 Z b
f (x) dx = lim f (x) dx + lim f (x) dx,
a a b 0

1472
when these limits exist. The Cauchy principal value of the integral is defined
Z Z a
PV f (x) dx = lim f (x) dx.
a a

The principal value may exist when the integral diverges.


R
Example 30.3.1 x dx diverges, but
Z Z a
PV x dx = lim x dx = lim (0) = 0.
a a a

If the improper integral converges, then the Cauchy principal value exists and is equal to the value of the integral.
The principal value of the integral of an odd function is zero. If the principal value of the integral of an even function
exists, then the integral converges.

30.3.2 Singular Functions


Let f (x) have a singularity at x = 0. Let a and b satisfy a < 0 < b. The integral of f (x) is defined
Z b Z 1 Z b
f (x) dx = lim f (x) dx + lim+ f (x) dx,
a 1 0 a 2 0 2

when the limits exist. The Cauchy principal value of the integral is defined
Z b Z  Z b 
PV f (x) dx = lim+ f (x) dx + f (x) dx ,
a 0 a 

when the limit exists.

Example 30.3.2 The integral Z 2


1
dx
1 x

1473
diverges, but the principal value exists.
Z 2 Z  Z 2 
1 1 1
PV dx = lim+ dx + dx
1 x 0 1 x  x
 Z 1 Z 2 
1 1
= lim+ dx + dx
0  x  x
Z 2
1
= dx
1 x
= log 2

1474
Chapter 31

The Laplace Transform

31.1 The Laplace Transform


The Laplace transform of the function f (t) is defined
Z
L[f (t)] = est f (t) dt,
0

for all values of s for which the integral exists. The Laplace transform of f (t) is a function of s which we will denote
f(s). 1
A function f (t) is of exponential order if there exist constants t0 and M such that

|f (t)| < M et , for all t > t0 .


Rt
If 0 0 f (t) dt exists and f (t) is of exponential order then the Laplace transform f(s) exists for <(s) > .
Here are a few examples of these concepts.
sin t is of exponential order 0.
1
Denoting the Laplace transform of f (t) as F (s) is also common.

1475
t e2t is of exponential order for any > 2.
2
et is not of exponential order for any .

tn is of exponential order for any > 0.

t2 does not have a Laplace transform as the integral diverges.

Example 31.1.1 Consider the Laplace transform of f (t) = 1. Since f (t) = 1 is of exponential order for any > 0,
the Laplace transform integral converges for <(s) > 0.
Z

f (s) = est dt
0 
1 st
= e
s 0
1
=
s
Example 31.1.2 The function f (t) = t et is of exponential order for any > 1. We compute the Laplace transform
of this function.
Z

f (s) = est t et dt
Z0
= t e(1s)t dt
0  Z
1 (1s)t 1
= te e(1s)t dt
1s 1 s
 0
 0
1
= e(1s)t
(1 s)2 0
1
= for <(s) > 1.
(1 s)2

1476
Example 31.1.3 Consider the Laplace transform of the Heaviside function,
(
0 for t < c
H(t c) =
1 for t > c,

where c > 0.
Z
L[H(t c)] = est H(t c) dt
0
Z
= est dt
c st 
e
=
s c
e cs
= for <(s) > 0
s
Example 31.1.4 Next consider H(t c)f (t c).
Z
L[H(t c)f (t c)] = est H(t c)f (t c) dt
Z0
= est f (t c) dt
Zc
= es(t+c) f (t) dt
0
=e cs
f(s)

31.2 The Inverse Laplace Transform


The inverse Laplace transform in denoted
f (t) = L1 [f(s)].

1477
We compute the inverse Laplace transform with the Mellin inversion formula.
Z +
1
f (t) = est f(s) ds
2

Here is a real constant that is to the right of the singularities of f(s).


To see why the Mellin inversion formula is correct, we take the Laplace transform of it. Assume that f (t) is of
exponential order . Then will be to the right of the singularities of f(s).
 Z + 
1 1 zt
L[L [f (s)]] = L e f (z) dz
2
Z Z +
st 1
= e ezt f(z) dz dt
0 2

We interchange the order of integration.


Z + Z
1
= f(z) e(zs)t dt dz
2 0

Since <(z) = , the integral in t exists for <(s) > .

+
f(z)
Z
1
= dz
2 sz
We would like to evaluate this integral by closing the path of integration with a semi-circle of radius R in the right half
plane and applying the residue theorem. However, in order for the integral along the semi-circle to vanish as R ,
f(z) must vanish as |z| . If f(z) vanishes we can use the maximum modulus bound to show that the integral
along the semi-circle vanishes. This we assume that f(z) vanishes at infinity.
Consider the integral,
I
1 f (z)
dz,
2 C s z

1478
Im(z)
+iR

s
Re(z)

-iR

Figure 31.1: The Laplace Transform Pair Contour.

where C is the contour that starts at R, goes straight up to + R, and then follows a semi-circle back down to
R. This contour is shown in Figure 31.1.
If s is inside the contour then I
1 f (z)
dz = f(s).
2 C s z
Note that the contour is traversed in the negative direction. Since f(z) decays as |z| , the semicircular contribution
to the integral will vanish as R . Thus
Z +
1 f (z)
dz = f(s).
2 s z
Therefore, we have shown than
L[L1 [f(s)]] = f(s).
f (t) and f(s) are known as Laplace transform pairs.

1479
31.2.1 f(s) with Poles

Example 31.2.1 Consider the inverse Laplace transform of 1/s2 . s = 1 is to the right of the singularity of 1/s2 .

  Z 1+
1 1 1 1
L 2
= est 2 ds
s 2 1 s

Let BR be the contour starting at 1 R and following a straight line to 1 + R; let CR be the contour starting at
1 + R and following a semicircular path down to 1 R. Let C be the combination of BR and CR . This contour is
shown in Figure 31.2.

Im(s)
+iR
CR
BR
Re(s)

-iR

Figure 31.2: The Path of Integration for the Inverse Laplace Transform.

1480
Consider the line integral on C for R > 1.
I  
1 1 st st 1
e 2 ds = Res e 2 , 0
2 C s s
d st
= e
ds s=0
=t

If t 0, the integral along CR vanishes as R . We parameterize s.

3
s = 1 + R e ,
2 2
st t(1+R e )
e = e = et etR cos et

Z Z
1 1
est 2 ds est ds
s2

CR s CR
1
R et
(R 1)2
0 as R

Thus the inverse Laplace transform of 1/s2 is


 
1 1
L = t, for t 0.
s2

Let f(s) be analytic except for isolated poles at s1 , s2 , . . . , sN and let be to the right of these poles. Also, let

f (s) 0 as |s| . Define BR to be the straight line from R to + R and CR to be the semicircular path

1481
from + R to R. If R is large enough to enclose all the poles, then
I N
1 X
e f(s) ds =
st
Res(est f(s), sn )
2 BR +CR n=1
Z N Z
1 X 1
e f(s) ds =
st
Res(e f(s), sn )
st
est f(s) ds.
2 BR n=1
2 CR

Now lets examine the integral along CR . Let the maximum of |f(s)| on CR be MR . We can parameterize the
contour with s = + R e , /2 < < 3/2.
Z Z 3/2

st t(+R ei )

e f (s) ds = e f ( + R e )R e d


CR /2
Z 3/2
et etR cos RMR d
/2
Z
= RMR e t
etR sin d
0

If t 0 we can use Jordans Lemma to obtain,


< RMR et .
tR

= MR et
t

We use that MR 0 as R .

0 as R

1482
Thus we have an expression for the inverse Laplace transform of f(s).
Z + N
1 X
e f(s) ds =
st
Res(est f(s), sn )
2 n=1
N
X
L1 [f(s)] = Res(est f(s), sn )
n=1

Result 31.2.1 If f(s) is analytic except for poles at s1 , s2 , . . . , sN and f(s) 0 as |s|
then the inverse Laplace transform of f(s) is
N
X
f (t) = L [f(s)] =
1
Res(est f(s), sn ), for t > 0.
n=1

1
Example 31.2.2 Consider the inverse Laplace transform of s3 s 2.

First we factor the denominator.


1 1 1
3 2
= 2 .
s s s s1
Taking the inverse Laplace transform,
     
1 1 st 1 1 st 1 1
L = Res e 2 , 0 + Res e 2 ,1
s3 s3 s s1 s s1
d est

= + et
ds s 1 s=0
1 t
= 2
+ + et
(1) 1

1483
Thus we have that  
1 1
L = et t 1, for t > 0.
s3 s2

Example 31.2.3 Consider the inverse Laplace transform of


s2 + s 1
.
s3 2s2 + s 2
We factor the denominator.
s2 + s 1
.
(s 2)(s )(s + )
Then we take the inverse Laplace transform.
s2 + s 1 s2 + s 1 s2 + s 1
     
1 st st
L = Res e , 2 + Res e ,
s3 2s2 + s 2 (s 2)(s )(s + ) (s 2)(s )(s + )
s2 + s 1
 
st
+ Res e ,
(s 2)(s )(s + )
1 1
= e2t + et + et
2 2
Thus we have
s2 + s 1
 
1
L 3 2
= sin t + e2t , for t > 0.
s 2s + s 2

31.2.2 f(s) with Branch Points



Example 31.2.4 Consider the inverse Laplace transform of 1 . s denotes the principal branch of s1/2 . There is a
s
branch cut from s = 0 to s = and
1 e/2
= , for < < .
s r

1484
Let be any positive number. The inverse Laplace transform of 1 is
s
Z +
1 1
f (t) = est ds.
2 s
We will evaluate the integral by deforming it to wrap around the branch cut. Consider the integral on the contour
shown in Figure 31.3. CR+ and CR are circular arcs of radius R. B is the vertical line at <(s) = joining the two arcs.
C is a semi-circle in the right half plane joining  and . L+ and L are lines joining the circular arcs at =(s) = .

CR+
B

/2
L+ C
L- /2+

CR-


Figure 31.3: Path of Integration for 1/ s

Since there are no residues inside the contour, we have


Z Z Z Z Z Z !
1 1
+ + + + + est ds = 0.
2 B +
CR L+ C L
CR s

We will evaluate the inverse Laplace transform for t > 0.

1485
First we will show that the integral along CR+ vanishes as R .
Z Z /2 Z
ds = d + d.
+
CR /2 /2

The first integral vanishes by the maximum modulus bound. Note that the length of the path of integration is less than
2.
Z !
/2
1
d max+ est (2)

s

/2 sCR

1
= et (2)
R
0 as R

The second integral vanishes by Jordans Lemma. A parameterization of CR+ is s = R e .


Z Z
R e t 1 R e t 1
e d

e
d

R e R e
/2 /2
Z
1
eR cos()t d
R /2
Z /2
1
eRt sin() d
R 0
1
<
R 2Rt
0 as R

We could show that the integral along CR vanishes by the same method. Now we have
Z Z Z Z 
1 1
+ + + est ds = 0.
2 B L+ C L s

1486
We can show that the integral along C vanishes as  0 with the maximum modulus bound.
Z  
st 1 st 1
e ds max e ()

C s sC s
1
< et 

0 as  0
Now we can express the inverse Laplace transform in terms of the integrals along L+ and L .
Z + Z Z
1 st 1 1 st 1 1 1
f (t) e ds = e ds est ds
2 s 2 L+ s 2 L s
On L+ , s = r e , ds = e dr = dr; on L , s = r e , ds = e dr = dr. We can combine the integrals along
the top and bottom of the branch cut.
Z 0 Z
1 rt 1
f (t) = e (1) dr ert (1) dr
2 r 2 0 r
Z
1 2
= ert dr
2 0 r

We make the change of variables x = rt.


Z
1 1
= ex dx
t 0 x

We recognize this integral as (1/2).


1
= (1/2)
t
1
=
t

1487
Thus the inverse Laplace transform of 1 is
s

1
f (t) = , for t > 0.
t

31.2.3 Asymptotic Behavior of f(s)


Consider the behavior of Z
f(s) = est f (t) dt
0
as s +. Assume that f (t) is analytic in a neighborhood of t = 0. Only the behavior of the integrand near t = 0
will make a significant contribution to the value of the integral. As you move away from t = 0, the est term dominates.
Thus we could approximate the value of f(s) by replacing f (t) with the first few terms in its Taylor series expansion
about the origin. Z
t2 00
 

f (s) e st 0
f (0) + tf (0) + f (0) + dt as s +
0 2
Using
n!
L [tn ] = n+1
s
we obtain
f (0) f 0 (0) f 00 (0)
f(s) + 2 + 3 + as s +.
s s s

Example 31.2.5 The Taylor series expansion of sin t about the origin is
t3
sin t = t + O(t5 ).
6
Thus the Laplace transform of sin t has the behavior
1 1
L[sin t] + O(s6 ) as s +.
s2 s4

1488
We corroborate this by expanding L[sin t].

1
L[sin t] =
s2 + 1
s2
=
1 + s2
X
= s2 (1)n s2n
n=0
1 1
= 2 4 + O(s6 )
s s

31.3 Properties of the Laplace Transform


In this section we will list several useful properties of the Laplace transform. If a result is not derived, it is shown in
the Problems section. Unless otherwise stated, assume that f (t) and g(t) are piecewise continuous and of exponential
order .

L[af (t) + bg(t)] = aL[f (t)] + bL[g(t)]

L[ect f (t)] = f(s c) for s > c +


dn
L[tn f (t)] = (1)n ds n [f (s)] for n = 1, 2, . . .
R f (t)
If 0 t
dt exists for positive then
  Z
f (t)
L = f() d.
t s

hR i
t f(s)
L 0
f ( ) d = s

1489
f (t) = sf(s) f (0)
d 
L dt
h i
d2
L dt2
f (t) = s2 f(s) sf (0) f 0 (0)

To derive these formulas,

  Z
d
L f (t) = est f 0 (t) dt
dt 0
 st 
Z
= e f (t) 0 s est f (t) dt
0
= f (0) + sf(s)

d2
 
L f (t) = sL[f 0 (t)] f 0 (0)
dt2
= s2 f(s) sf (0) f 0 (0)

Let f (t) and g(t) be continuous. The convolution of f (t) and g(t) is defined

Z t Z t
h(t) = (f g) = f ( )g(t ) d = f (t )g( ) d
0 0

The convolution theorem states

h(s) = f(s)g(s).

1490
To show this,
Z Z t
st
h(s) = e f ( )g(t ) d dt
0 0
Z Z
= est f ( )g(t ) dt d
Z0 Z
s
= e f ( ) es(t ) g(t ) dt d
Z0
Z
s
= e f ( ) d es g() d
0 0
= f(s)g(s)

If f (t) is periodic with period T then


RT
0
est f (t) dt
L[f (t)] = .
1 esT
1
Example 31.3.1 Consider the inverse Laplace transform of s3 s2
. First we factor the denominator.
1 1 1
= 2
s 2 s3
s s1
We know the inverse Laplace transforms of each term.
   
1 1 1 1
L = t, L = et
s2 s1
We apply the convolution theorem.
  Z t
1 1 1
L = et d
s2 s 1 0
Z t
t
t t
e d
 
= e e 0 e
0
t
= t 1 + e

1491
 
1 1 1
L = et t 1.
s2 s 1

Example 31.3.2 We can find the inverse Laplace transform of

s2 + s 1
s3 2s2 + s 2

with the aid of a table of Laplace transform pairs. We factor the denominator.

s2 + s 1
(s 2)(s )(s + )

We expand the function in partial fractions and then invert each term.

s2 + s 1 1 /2 /2
= +
(s 2)(s )(s + ) s2 s s+
2
s +s1 1 1
= + 2
(s 2)(s )(s + ) s2 s +1
 
1 1
L1 + 2 = e2t + sin t
s2 s +1

31.4 Constant Coefficient Differential Equations


Example 31.4.1 Consider the differential equation

y 0 + y = cos t, for t > 0, y(0) = 1.

1492
We take the Laplace transform of this equation.
s
sy(s) y(0) + y(s) =
+1 s2
s 1
y(s) = +
(s + 1)(s2 + 1) s + 1
1/2 1 s+1
y(s) = + 2
s+1 2s +1
Now we invert y(s).
1 t 1 1
y(t) = e + cos t + sin t, for t > 0
2 2 2
Notice that the initial condition was included when we took the Laplace transform.

One can see from this example that taking the Laplace transform of a constant coefficient differential equation
reduces the differential equation for y(t) to an algebraic equation for y(s).

Example 31.4.2 Consider the differential equation

y 00 + y = cos(2t), for t > 0, y(0) = 1, y 0 (0) = 0.

We take the Laplace transform of this equation.


s
s2 y(s) sy(0) y 0 (0) + y(s) =
s2 +4
s s
y(s) = + 2
(s2 2
+ 1)(s + 4) s + 1

From the table of Laplace transform pairs we know


   
1 s 1 1 1
L = cos t, L = sin(2t).
s2 + 1 2
s +4 2

1493
We use the convolution theorem to find the inverse Laplace transform of y(s).
Z t
1
y(t) = sin(2 ) cos(t ) d + cos t
0 2
1 t
Z
= sin(t + ) + sin(3 t) d + cos t
4 0
 t
1 1
= cos(t + ) cos(3 t) + cos t
4 3 0
 
1 1 1
= cos(2t) + cos t cos(2t) + cos(t) + cos t
4 3 3
1 4
= cos(2t) + cos(t)
3 3
Alternatively, we can find the inverse Laplace transform of y(s) by first finding its partial fraction expansion.
s/3 s/3 s
y(s) = 2 + 2
s2
+1 s +4 s +1
s/3 4s/3
= 2 + 2
s +4 s +1
1 4
y(t) = cos(2t) + cos(t)
3 3
Example 31.4.3 Consider the initial value problem
y 00 + 5y 0 + 2y = 0, y(0) = 1, y 0 (0) = 2.
Without taking a Laplace transform, we know that since
y(t) = 1 + 2t + O(t2 )
the Laplace transform has the behavior
1 2
y(s) + 2 + O(s3 ), as s +.
s s

1494
31.5 Systems of Constant Coefficient Differential Equations
The Laplace transform can be used to transform a system of constant coefficient differential equations into a system
of algebraic equations. This should not be surprising, as a system of differential equations can be written as a single
differential equation, and vice versa.

Example 31.5.1 Consider the set of differential equations

y10 = y2
y20 = y3
y30 = y3 y2 y1 + t3

with the initial conditions


y1 (0) = y2 (0) = y3 (0) = 0.

We take the Laplace transform of this system.

sy1 y1 (0) = y2
sy2 y2 (0) = y3
6
sy3 y3 (0) = y3 y2 y1 +
s4

The first two equations can be written as

y3
y1 =
s2
y3
y2 = .
s

1495
We substitute this into the third equation.
y3 y3 6
sy3 = y3 2+ 4
s s s
6
(s3 + s2 + s + 1)y3 = 2
s
6
y3 = 2 3 .
s (s + s2 + s + 1)

We solve for y1 .
6
y1 =
s4 (s3
+ s2 + s + 1)
1 1 1 1s
y1 = 4 3 + +
s s 2(s + 1) 2(s2 + 1)

We then take the inverse Laplace transform of y1 .

t3 t2 1 t 1 1
y1 = + e + sin t cos t.
6 2 2 2 2
We can find y2 and y3 by differentiating the expression for y1 .

t2 1 1 1
y2 = t et + cos t + sin t
2 2 2 2
1 t 1 1
y3 = t 1 + e sin t + cos t
2 2 2

1496
31.6 Exercises
Exercise 31.1
Find the Laplace transform of the following functions:
1. f (t) = eat
2. f (t) = sin(at)
3. f (t) = cos(at)
4. f (t) = sinh(at)
5. f (t) = cosh(at)
sin(at)
6. f (t) =
t
Z t
sin(au)
7. f (t) = du
0 u
(
1, 0 t <
8. f (t) =
0, t < 2
and f (t + 2) = f (t) for t > 0. That is, f (t) is periodic for t > 0.
Hint, Solution
Exercise 31.2
Show that L[af (t) + bg(t)] = aL[f (t)] + bL[g(t)].
Hint, Solution
Exercise 31.3
Show that if f (t) is of exponential order ,

L[ect f (t)] = f(s c) for s > c + .

1497
Hint, Solution
Exercise 31.4
Show that
dn
L[tn f (t)] = (1)n [f (s)] for n = 1, 2, . . .
dsn
Hint, Solution
Exercise 31.5
R f (t)
Show that if 0 t
dt exists for positive then
  Z
f (t)
L = f() d.
t s

Hint, Solution
Exercise 31.6
Show that
t
f(s)
Z 
L f ( ) d = .
0 s
Hint, Solution
Exercise 31.7
Show that if f (t) is periodic with period T then
RT
0
est f (t) dt
L[f (t)] = .
1 esT

Hint, Solution

1498
Exercise 31.8
The function f (t) t 0, is periodic with period 2T ; i.e. f (t + 2T ) f (t), and is also odd with period T ; i.e.
f (t + T ) = f (t). Further,
Z T
f (t) est dt = g(s).
0

Show that the Laplace transform of f (t) is f(s) = g(s)/(1 + esT ). Find f (t) such that f(s) = s1 tanh(sT /2).
Hint, Solution
Exercise 31.9
Find the Laplace transform of t , > 1 by two methods.

1. Assume that s is complex-valued. Make the change of variables z = st and use integration in the complex plane.

2. Show that the Laplace transform of t is an analytic function for <(s) > 0. Assume that s is real-valued. Make
the change of variables x = st and evaluate the integral. Then use analytic continuation to extend the result to
complex-valued s.

Hint, Solution
Exercise 31.10 (mathematica/ode/laplace/laplace.nb)
Show that the Laplace transform of f (t) = ln t is
Z
Log s
f(s) = , where = et ln t dt.
s s 0

[ = 0.5772 . . . is known as Eulers constant.]


Hint, Solution
Exercise 31.11
Find the Laplace transform of t ln t. Write the answer in terms of the digamma function, () = 0 ()/(). What
is the answer for = 0?
Hint, Solution

1499
Exercise 31.12
Find the inverse Laplace transform of
1
f(s) =
s3 2s2 +s2
with the following methods.

1. Expand f(s) using partial fractions and then use the table of Laplace transforms.

2. Factor the denominator into (s 2)(s2 + 1) and then use the convolution theorem.

3. Use Result 31.2.1.

Hint, Solution
Exercise 31.13
Solve the differential equation

y 00 + y 0 + y = sin t, y(0) = y 0 (0) = 0, 0<1

using the Laplace transform. This equation represents a weakly damped, driven, linear oscillator.
Hint, Solution
Exercise 31.14
Solve the problem,
y 00 ty 0 + y = 0, y(0) = 0, y 0 (0) = 1,
with the Laplace transform.
Hint, Solution
Exercise 31.15
Prove the following relation between the inverse Laplace transform and the inverse Fourier transform,

1 ct 1
L1 [f(s)] = e F [f (c + )],
2

1500
where c is to the right of the singularities of f(s).
Hint, Solution
Exercise 31.16 (mathematica/ode/laplace/laplace.nb)
Show by evaluating the Laplace inversion integral that if
 1/2 1/2
f(s) = e2(as) , s1/2 = s for s > 0,
s

then f (t) = ea/t
R / ax
t. Hint: cut theps-plane along the negative real axis and deform the contour onto the cut.
2 2
Remember that 0 e cos(bx) dx = /4a eb /4a .
Hint, Solution
Exercise 31.17 (mathematica/ode/laplace/laplace.nb)
Use Laplace transforms to solve the initial value problem

d4 y
y = t, y(0) = y 0 (0) = y 00 (0) = y 000 (0) = 0.
dt4
Hint, Solution
Exercise 31.18 (mathematica/ode/laplace/laplace.nb)
Solve, by Laplace transforms,
Z t
dy
= sin t + y( ) cos(t ) d, y(0) = 0.
dt 0

Hint, Solution
Exercise 31.19 (mathematica/ode/laplace/laplace.nb)
Suppose u(t) satisfies the difference-differential equation

du
+ u(t) u(t 1) = 0, t 0,
dt

1501
and the initial condition u(t) = u0 (t), 1 t 0, where u0 (t) is given. Show that the Laplace transform u(s) of
u(t) satisfies
Z 0
u0 (0) es
u(s) = s
+ s
est u0 (t) dt.
1+s e 1+s e 1

Find u(t), t 0, when u0 (t) = 1. Check the result.


Hint, Solution

Exercise 31.20
Let the function f (t) be defined by
(
1 0t<
f (t) =
0 t < 2,

and for all positive values of t so that f (t + 2) = f (t). That is, f (t) is periodic with period 2. Find the solution of
the intial value problem
d2 y
y = f (t); y(0) = 1, y 0 (0) = 0.
dt2
Examine the continuity of the solution at t = n, where n is a positive integer, and verify that the solution is continuous
and has a continuous derivative at these points.
Hint, Solution

Exercise 31.21
Use Laplace transforms to solve
Z t
dy
+ y( ) d = et , y(0) = 1.
dt 0

Hint, Solution

1502
Exercise 31.22
An electric circuit gives rise to the system

di1
L + Ri1 + q/C = E0
dt
di2
L + Ri2 q/C = 0
dt
dq
= i 1 i2
dt
with initial conditions
E0
i1 (0) = i2 (0) =
, q(0) = 0.
2R
Solve the system by Laplace transform methods and show that
E0 E0 t
i1 = + e sin(t)
2R 2L
where
R 2
= and 2 = 2 .
2L LC
Hint, Solution
Exercise 31.23
Solve the initial value problem,

y 00 + 4y 0 + 4y = 4 et , y(0) = 2, y 0 (0) = 3.

Hint, Solution

1503
31.7 Hints
Hint 31.1
Use the differentiation and integration properties of the Laplace transform where appropriate.

Hint 31.2

Hint 31.3

Hint 31.4
g
If the integral is uniformly convergent and s
is continuous then

Z b Z b
d
g(s, t) dt = g(s, t) dt
ds a a s

Hint 31.5

Z
1 sx
etx dt = e
s x

Hint 31.6
Use integration by parts.

Hint 31.7

Z Z (n+1)T
X
st
e f (t) dt = est f (t) dt
0 n=0 nT

1504
The sum can be put in the form of a geometric series.

X 1
n = , for || < 1
n=0
1

Hint 31.8

Hint 31.9
Write the answer in terms of the Gamma function.

Hint 31.10

Hint 31.11

Hint 31.12

Hint 31.13

Hint 31.14

Hint 31.15

Hint 31.16

1505
Hint 31.17

Hint 31.18

Hint 31.19

Hint 31.20

Hint 31.21

Hint 31.22

Hint 31.23

1506
31.8 Solutions
Solution 31.1
1.
Z
L eat = est eat dt
 
0
Z
= e(sa)t dt
0 (sa)t

e
= for <(s) > <(a)
sa 0

1
L eat =
 
for <(s) > <(a)
sa

2.
Z
L[sin(at)] = est sin(at) dt
0
1 (s+a)t
Z
e(sa)t dt

= e
2 0

1 e(s+a)t e(sa)t

= + , for <(s) > 0
2 s a s + a 0
 
1 1 1
=
2 s a s + a

a
L[sin(at)] = for <(s) > 0
s2 + a2

1507
3.


d sin(at)
L[cos(at)] = L
dt a
 
sin(at)
= sL sin(0)
a
s
L[cos(at)] = for <(s) > 0
s2 + a2
4.
Z
L[sinh(at)] = est sinh(at) dt
0
1 (s+a)t
Z
e(sa)t dt

= e
2 0

1 e(s+a)t e(sa)t

= + for <(s) > |<(a)|
2 sa s+a 0
 
1 1 1
=
2 sa s+a
a
L[sinh(at)] = for <(s) > |<(a)|
s2 a2
5.
 
d sinh(at)
L[cosh(at)] = L
dt a
 
sinh(at)
= sL sinh(0)
a
s
L[cosh(at)] = for <(s) > |<(a)|
s2 a2

1508
6. First note that   Z
sin(at)
L (s) = L[sin(at)]() d.
t s
Now we use the Laplace transform of sin(at) to compute the Laplace transform of sin(at)/t.
  Z
sin(at) a
L = d
t s + a2
2
Z
1 d
= 2
(/a) + 1 a
hs  i
= arctan
a  s
s
= arctan
2 a
 
sin(at) a
L = arctan
t s

7. Z t   
sin(a ) 1 sin(at)
L d = L
0 s t
Z t 
sin(a ) 1 a
L d = arctan
0 s s

8.
R 2
est f (t) dt
0
L[f (t)] =
e2s
R 1 st
e dt
= 0 2s
1e
1 es
=
s(1 e2s )

1509
1
L[f (t)] =
s(1 + es )

Solution 31.2

Z
est af (t) + bg(t) dt

L[af (t) + bg(t)] =
0
Z Z
st
=a e f (t) dt + b est g(t) dt
0 0
= aL[f (t)] + bL[g(t)]

Solution 31.3
If f (t) is of exponential order , then ect f (t) is of exponential order c + .

Z
ct
L[e f (t)] = est ect f (t) dt
Z0
= e(sc)t f (t) dt
0
= f(s c) for s > c +

Solution 31.4
First consider the Laplace transform of t0 f (t).

L[t0 f (t)] = f(s)

1510
Now consider the Laplace transform of tn f (t) for n 1.
Z
n
L[t f (t)] = est tn f (t) dt
0
d st n1
Z
= e t f (t) dt
ds 0
d
= L[tn1 f (t)]
ds
Thus we have a difference equation for the Laplace transform of tn f (t) with the solution

n dn
n
L[t f (t)] = (1) n
L[t0 f (t)] for n Z0+ ,
ds

dn
L[tn f (t)] = (1)n n
f (s) for n Z0+ .
ds

Solution
R f (t) 31.5
If 0 t dt exists for positive and f (t) is of exponential order then the Laplace transform of f (t)/t is defined for
s > .
  Z
f (t) 1
L = est f (t) dt
t t
Z0 Z
= et d f (t) dt
Z0 Zs
= et f (t) dt d
Zs 0
= f() d
s

1511
Solution 31.6

Z t  Z Z t
st
L f ( ) d = e f ( ) d dx
0 0 0
 st t  Z t
est d
Z 
e
Z
= f ( ) d f ( ) d dt
s s dt
Z 0 0 0 0
1
= est f (t) dt
s 0
1
= f(s)
s

1512
Solution 31.7
f (t) is periodic with period T .

Z
L[f (t)] = est f (t) dt
0
Z T Z 2T
st
= e f (t) dt + est f (t) dt +
0 T
Z
X (n+1)T
= est f (t) dt
n=0 nT
X Z T
= es(t+nT ) f (t + nT ) dt
n=0 0

X Z T
snT
= e est f (t) dt
n=0 0
Z T
X
= est f (t) dt esnT
0 n=0
RT
0
est
f (t) dt
=
1 esT

1513
Solution 31.8

Z
f(s) = est f (t) dt
0
n Z
X (n+1)T
= est f (t) dt
0 nT
n
X T
Z
= es(t+nT ) f (t + nT ) dt
0 0
n
X Z T
snT
= e est (1)n f (t) dt
0 0
Z T n
st
X n
= e f (t) dt (1)n esT
0 0

g(s)
f(s) = , for <(s) > 0
1 + esT

Consider f(s) = s1 tanh(sT /2).

esT /2 esT /2
s1 tanh(sT /2) = s1
esT /2 + esT /2
1 esT
= s1
1 + esT

We have
T
1 est
Z
g(s) f (t) est dt = .
0 s

1514
By inspection we see that this is satisfied for f (t) = 1 for 0 < t < T . We conclude:

(
1 for t [2nT . . . (2n + 1)T ),
f (t) =
1 for t [(2n + 1)T . . . (2n + 2)T ),

where n Z.
Solution 31.9
The Laplace transform of t , > 1 is
Z
f(s) = est t dt.
0

Assume s is complex-valued. The integral converges for <(s) > 0 and > 1.

Method 1. We make the change of variables z = st.


Z  z  1
f(s) = ez dz
C s s
Z
(+1)
=s ez z dz
C

C is the path from 0 to along arg(z) = arg(s). (Shown in Figure 31.4).


Since the integrand is analytic in the domain  < r < R, 0 < < arg(s), the integral along the boundary of this
domain vanishes. !
Z R Z R e arg(s) Z  e arg(s) Z 
+ + + ez z dz = 0
 R R e arg(s)  e arg(s)

We show that the integral along CR , the circular arc of radius R, vanishes as R with the maximum modulus

1515
Im(z)

arg(s)
Re(z)

Figure 31.4: The Path of Integration.

integral bound.
Z
z
z dz R| arg(s)| max ez z


e

CR zCR

= R| arg(s)| eR cos(arg(s)) R
0 as R .

The integral along C , the circular arc of radius , vanishes as  0. We demonstrate this with the maximum modulus
integral bound.
Z
z
z dz | arg(s)| max ez z


e

C zC

= | arg(s)| e cos(arg(s)) 
0 as  0.

1516
Taking the limit as  0 and R , we see that the integral along C is equal to the integral along the real axis.
Z Z
z
e z dz = ez z dz
C 0

We can evaluate the Laplace transform of t in terms of this integral.


Z
(+1)

L [t ] = s et t dt
0

( + 1)
L [t ] =
s+1
In the case that is a non-negative integer = n > 1 we can write this in terms of the factorial.
n!
L [tn ] =
sn+1

Method 2. First note that the integral Z


f(s) = est t dt
0
exists for <(s) > 0. It converges uniformly for <(s) c > 0. On this domain of uniform convergence we can
interchange differentiation and integration.

df d st
Z
= e t dt
ds ds 0
Z
st 
= e t dt
s
Z0
= t est t dt
0
Z
= est t+1 dt
0

1517
Since f0 (s) is defined for <(s) > 0, f(s) is analytic for <(s) > 0.
Let be real and positive. We make the change of variables x = t.
Z  x  1

f () = ex dx
0
Z
(+1)
= ex x dx
0
( + 1)
=
+1
Note that the function
( + 1)
f(s) =
s+1
is the analytic continuation of f(). Thus we can define the Laplace transform for all complex s in the right half plane.
( + 1)
f(s) =
s+1
Solution 31.10
Note that f(s) is an analytic function for <(s) > 0. Consider real-valued s > 0. By definition, f(s) is
Z

f (s) = est ln t dt.
0
We make the change of variables x = st.
Z  x  dx
f(s) = ex ln
0 s s
Z
1
= ex (ln x ln s) dx
s 0
ln |s| x 1 x
Z Z
= e dx + e ln x dx
s 0 s 0
ln s
= , for real s > 0
s s

1518
The analytic continuation of f(s) into the right half-plane is

Log s
f(s) = .
s s

Solution 31.11
Define

Z
f(s) = L[t ln t] =

est t ln t dt.
0

This integral defines f(s) for <(s) > 0. Note that the integral converges uniformly for <(s) c > 0. On this domain
we can interchange differentiation and integration.

Z Z
0 st
t est t Log t dt

f (s) = e t ln t dt =
0 s 0

Since f0 (s) also exists for <(s) > 0, f(s) is analytic in that domain.

1519
Let be real and positive. We make the change of variables x = t.

f() = L [t ln t]
Z
= et t ln t dt
Z0  x  x 1
= ex ln dx
0
Z
1
= +1 ex x (ln x ln ) dx

0Z Z 
1 x x
= +1 e x ln x dx ln e x dx
0 0
Z 
1 x 
= +1 e x dx ln ( + 1)
0
 Z 
1 d x
= +1 e x dx ln ( + 1)
d 0
 
1 d
= +1 ( + 1) ln ( + 1)
d
 0 
1 ( + 1)
= +1 ( + 1) ln
( + 1)
1
= +1 ( + 1) (( + 1) ln )

Note that the function
1
f(s) = ( + 1) (( + 1) ln s)
s+1
is an analytic continuation of f(). Thus we can define the Laplace transform for all s in the right half plane.

1
L[t ln t] = ( + 1) (( + 1) ln s) for <(s) > 0.
s+1

1520
For the case = 0, we have
1
L[ln t] = (1) ((1) ln s)
s1
ln s
L[ln t] = ,
s
where is Eulers constant Z
= ex ln x dx = 0.5772156629 . . .
0

Solution 31.12
Method 1. We factor the denominator.
1 1
f(s) = 2
=
(s 2)(s + 1) (s 2)(s )(s + )
We expand the function in partial fractions and simplify the result.
1 1/5 (1 2)/10 (1 + 2)/10
=
(s 2)(s )(s + ) s2 s s+
1 1 1 s+2
f(s) = 2
5s2 5s +1
We use a table of Laplace transforms to do the inversion.
1 s 1
L[e2t ] = , L[cos t] = , L[sin t] =
s2 s2 +1 s2 +1
1 2t 
f (t) = e cos t 2 sin t
5
Method 2. We factor the denominator.
1 1
f(s) = 2
s2s +1

1521
From a table of Laplace transforms we note
1 1
L[e2t ] = , L[sin t] = .
s2 s2 +1
We apply the convolution theorem.
Z t
f (t) = sin e2(t ) d
0
1 2t 
f (t) = e cos t 2 sin t
5

Method 3. We factor the denominator.


1
f(s) =
(s 2)(s )(s + )

f(s) is analytic except for poles and vanishes at infinity.

est
X  
f (t) = Res , sn
s =2,,
(s 2)(s )(s + )
n

e2t et et
= + +
(2 )(2 + ) ( 2)(2) ( 2)(2)
e2t (1 + 2) et (1 2) et
= + +
5 10 10
e 2t t
e +e t t
e e t
= + +
5 10 5

1 2t 
f (t) = e cos t 2 sin t
5

1522
Solution 31.13

y 00 + y 0 + y = sin t, y(0) = y 0 (0) = 0, 0<1


We take the Laplace transform of this equation.

(s2 y(s) sy(0) y 0 (0)) + (sy(s) y(0)) + y(s) = L[sin(t)]


(s2 + s + 1)y(s) = L[sin(t)]
1
y(s) = 2 L[sin(t)]
s + s + 1
1
y(s) = 2 L[sin(t)]
(s + 2 ) + 1 4
 2

We use a table of Laplace transforms to find the inverse Laplace transform of the first term.
" # r !
1 1  2
L1  2 2
=q et/2 sin 1 t
(s + 2 ) + 1 4 1 4 2 4

We define r
2
= 1
4
to get rid of some clutter. Now we apply the convolution theorem to invert 2 ys.
Z t
1  /2
y(t) = e sin ( ) sin(t ) d
0
 
t/2 1 1 1
y(t) = e cos (t) + sin (t) cos t
 2 

The solution is plotted in Figure 31.5 for  = 0.05.


2
Evaluate the convolution integral by inspection.

1523
15

10

20 40 60 80 100
-5

-10

-15

Figure 31.5: The Weakly Damped, Driven Oscillator

Solution 31.14
We consider the solutions of
y 00 ty 0 + y = 0, y(0) = 0, y 0 (0) = 1
which are of exponential order for any > 0. We take the Laplace transform of the differential equation.
d
s2 y 1 + (sy) + y = 0
 ds 
2 1
y 0 + s + y =
s s
2
1 es /2
y(s) = 2 + c 2
s s

1524
We use that
y(0) y 0 (0)
y(s) + 2 +
s s
to conclude that c = 0.
1
y(s) =
s2
y(t) = t

Solution 31.15
Z c+
1
L [f(s)] =
1
est f(s) ds
2 c
First we make the change of variable s = c + .
Z
1 ct
L [f(s)] =
1
e et f(c + ) d
2

Then we make the change of variable = .


Z
1
L [f(s)] =
1
e ct
et f(c + ) d
2
1
L1 [f(s)] = ect F 1 [f(c + )]
2
Solution 31.16
We assume that <(a) 0. We are considering the principal branch of the square root: s1/2 = s. There is a branch
cut on the negative real axis. f(s) is singular at s = 0 and along the negative real axis. Let be any positive number.
1/2 2(as)1/2
The inverse Laplace transform of s e is
Z +  1/2
1 1/2
f (t) = est e2(as) ds.
2 s

1525
We will evaluate the integral by deforming it to wrap around the branch cut. Consider the integral on the contour
shown in Figure 31.6. CR+ and CR are circular arcs of radius R. B is the vertical line at <(s) = joining the two arcs.
C is a semi-circle in the right half plane joining  and . L+ and L are lines joining the circular arcs at =(s) = .

CR+
B

/2
L+ C
L- /2+

CR-

Figure 31.6: Path of Integration

Since there are no residues inside the contour, we have


Z Z Z Z Z Z !  1/2
1 1/2
+ + + + + est e2(as) ds = 0.
2 B CR+
L+ C L
CR s

We will evaluate the inverse Laplace transform for t > 0.


First we will show that the integral along CR+ vanishes as R . We parametrize the path of integration with
s = R e and write the integral along CR+ as the sum of two integrals.
Z Z /2 Z
ds = d + d
+
CR /2 /2

1526
The first integral vanishes by the maximum modulus bound. Note that the length of the path of integration is less than
2.
Z 
/2   
st 1/2 2(as)1/2
d max e e (2)

s

/2 [/2.../2]

t
= e (2)
R
0 as R

The second integral vanishes by Jordans Lemma.

Z
2aR e
Z
/2
R e t
R e t 2 a R e
e e d e e d
R e R e

/2 /2
Z

eR cos()t d
R /2
Z /2

eRt sin() d
R 0


<
R 2Rt
0 as R

We could show that the integral along CR vanishes by the same method.
Now we have
Z Z Z Z 
1  1/2 1/2
+ + + est e2(as) ds = 0.
2 B L+ C L s

1527
We show that the integral along C vanishes as  0 with the maximum modulus bound.

Z  1/2    
st 2(as)1/2
st 1/2 2(as)1/2
e e ds max e
e ()

C s sC s


et 

0 as  0.

Now we can express the inverse Laplace transform in terms of the integrals along L+ and L

Z +  1/2
1 1/2
f (t) est e2(as) ds
2 s
Z Z
1  1/2
2(as)1/2 1 st
 1/2 1/2
= e st
e ds e e2(as) ds.
2 L+ s 2 L s

On L+ , s = r e , ds = e dr = dr; on L , s = r e , ds = e dr = dr. We can combine the integrals along


the top and bottom of the branch cut.

Z 0 Z
1 rt 2ar 1 rt
f (t) = e e ( dr) e e2 a r ( dr)
2 r 2 0 r
Z
1 1  
= ert e2 a r + e2 a r dr
2 0 r
Z
1 1 
= ert 2 cos 2 a r dr
2 0 r

1528

We make the change of variables x = r.
Z
1 1 tx2 
= e cos 2 ax 2x dx
0 x
Z
2 2 
= etx cos 2 ax dx
0
r
2 4a/(4t)
= e
4t
ea/t
=
t
Thus the inverse Laplace transform is
ea/t
f (t) =
t

Solution 31.17
We consider the problem
d4 y
y = t, y(0) = y 0 (0) = y 00 (0) = y 000 (0) = 0.
dt4
We take the Laplace transform of the differential equation.
1
s4 y(s) s3 y(0) s2 y 0 (0) sy 00 (0) y 000 (0) y(s) =
s2
1
s4 y(s) y(s) =
s2
1
y(s) =
s2 (s4 1)

There are several ways in which we could carry out the inverse Laplace transform to find y(t). We could expand the
right side in partial fractions and then use a table of Laplace transforms. Since the function is analytic except for

1529
isolated singularities and vanishes as s we could use the result,
N
X  
L [f(s)] =
1 st
Res e f (s), sn ,
n=1

where {sk }nk=1 are the singularities of f(s). Since we can write the function as a product of simpler terms we could
also apply the convolution theorem.
We will first do the inverse Laplace transform by expanding the function in partial fractions to obtain simpler rational
functions.
1 1
= 2
s2 (s4 1) s (s 1)(s + 1)(s )(s + )
a b c d e f
= 2+ + + + +
s s s1 s+1 s s+

 
1
a= 4 = 1
s 1 s=0
 
d 1
b= =0
ds s4 1 s=0
 
1 1
c= 2 =
s (s + 1)(s )(s + ) s=1 4
 
1 1
d= 2 =
s (s 1)(s )(s + ) s=1 4
 
1 1
e= 2 =
s (s 1)(s + 1)(s + ) s= 4
 
1 1
f= 2 =
s (s 1)(s + 1)(s ) s= 4

1530
Now we have simple functions that we can look up in a table.
1 1/4 1/4 1/2
y(s) = 2 + + 2
 s s 1 s + 1 s + 1
1 1 1
y(t) = t + et et + sin t H(t)
4 4 2
 
1
y(t) = t + (sinh t + sin t) H(t)
2
We can also do the inversion with the convolution theorem.
1 1 1 1
2 4
= 2 2 2
s (s 1) s s +1s 1
From a table of Laplace transforms we know,
 
1 1
L = t,
s2
 
1 1
L = sin t,
s2 + 1
 
1 1
L = sinh t.
s2 1
Now we use the convolution theorem to find the solution for t > 0.
  Z t
1 1
L = sinh( ) sin(t ) d
s4 1 0
1
= (sinh t sin t)
2
  Z t
1 1
L1 2 4 = (sinh sin ) (t ) d
s (s 1) 0 2
1
= t + (sinh t + sin t)
2

1531
Solution 31.18

Z t
dy
= sin t + y( ) cos(t ) d
dt 0
1 s
sy(s) y(0) = + y(s)
s2 + 1 s2 + 1
3
(s + s)y(s) sy(s) = 1
1
y(s) = 3
s
t2
y(t) =
2

Solution 31.19
The Laplace transform of u(t 1) is

Z
L[u(t 1)] = est u(t 1) dt
Z0
= es(t+1) u(t) dt
1
Z 0 Z
s st s
=e e u(t) dt + e est u(t) dt
1 0
Z 0
= es est u0 (t) dt + es u(s).
1

1532
We take the Laplace transform of the difference-differential equation.
Z 0
s
su(s) u(0) + u(s) e est u0 (t) dt + es u(s) = 0
1
Z 0
s s
(1 + s e )u(s) = u0 (0) + e est u0 (t) dt
1
0
es
Z
u0 (0)
u(s) = + est u0 (t) dt
1 + s es 1 + s es 1

Consider the case u0 (t) = 1.


Z 0
1 es
u(s) = + est dt
1 + s es 1 + s es 1
es
 
1 1 1 s
u(s) = + + e
1 + s es 1 + s es s s
s
1/s + 1 e /s
u(s) =
1 + s es
1
u(s) =
s
u(t) = 1
Clearly this solution satisfies the difference-differential equation.
Solution 31.20
We consider the problem,
d2 y
2
y = f (t), y(0) = 1, y 0 (0) = 0,
dt
where f (t) is periodic with period 2 and is defined by,
(
1 0 t < ,
f (t) =
0 t < 2.

1533
We take the Laplace transform of the differential equation.
s2 y(s) sy(0) y 0 (0) y(s) = f(s)
s2 y(s) s y(s) = f(s)
s f(s)
y(s) = +
s2 1 s2 1
By inspection, (of a table of Laplace transforms), we see that
 
1 s
L = cosh(t)H(t),
s2 1
 
1 1
L = sinh(t)H(t).
s2 1
Now we use the convolution theorem.
" # Z
f(s) t
L1 = f ( ) sinh(t ) d
s2 1 0

The solution for positive t is


Z t
y(t) = cosh(t) + f ( ) sinh(t ) d.
0

Clearly the solution is continuous because the integral of a bounded function is continuous. The first derivative of the
solution is
Z t
0
y (t) = sinh t + f (t) sinh(0) + f ( ) cosh(t ) d
0
Z t
0
y (t) = sinh t + f ( ) cosh(t ) d
0

We see that the first derivative is also continuous.

1534
Solution 31.21
We consider the problem
Z t
dy
+ y( ) d = et , y(0) = 1.
dt 0
We take the Laplace transform of the equation and solve for y.
y 1
sy y(0) + =
s s+1
s(s + 2)
y =
(s + 1)(s2 + 1)
We expand the right side in partial fractions.
1 1 + 3s
y = +
2(s + 1) 2(s2 + 1)
We use a table of Laplace transforms to do the inversion.

1 1
y = et + (sin(t) + 3 cos(t))
2 2
Solution 31.22
We consider the problem

di1
L + Ri1 + q/C = E0
dt
di2
L + Ri2 q/C = 0
dt
dq
= i 1 i2
dt
E0
i1 (0) = i2 (0) = , q(0) = 0.
2R

1535
We take the Laplace transform of the system of differential equations.
 
E0 q E0
L si1 + Ri1 + =
2R C s
 
E0 q
L si2 + Ri2 = 0
2R C
sq = i1 i2

We solve for i1 , i2 and q.


 
E0 1 1/L
i1 = +
2 Rs s2 + Rs/L + 2/(CL)
 
E0 1 1/L
i2 =
2 Rs s2 + Rs/L + 2/(CL)
 
CE0 1 s + R/L
q =
2 s s2 + Rs/L + 2/(CL)

We factor the polynomials in the denominators.


 
E0 1 1/L
i1 = +
2 Rs (s + )(s + + )
 
E0 1 1/L
i2 =
2 Rs (s + )(s + + )
 
CE0 1 s + 2
q =
2 s (s + )(s + + )

Here we have defined


R 2
= and 2 = 2 .
2L LC

1536
We expand the functions in partial fractions.
  
E0 1 1 1
i1 = +
2 Rs 2L s + + s +
  
E0 1 1 1
i2 =
2 Rs 2L s + + s +
  
CE0 1 +
q = +
2 s 2 s + s + +

Now we can do the inversion with a table of Laplace transforms.


 
E0 1 ()t (+)t

i1 = + e e
2 R 2L
 
E0 1 ()t (+)t

i2 = e e
2 R 2L
CE0  (+)t ()t

q= 1+ ( + ) e ( ) e
2 2
We simplify the expressions to obtain the solutions.
 
E0 1 1 t
i1 = + e sin(t)
2 R L
 
E0 1 1 t
i2 = e sin(t)
2 R L
CE0  t
 
q= 1e cos(t) + sin(t)
2
Solution 31.23
We consider the problem
y 00 + 4y 0 + 4y = 4 et , y(0) = 2, y 0 (0) = 3

1537
We take the Laplace transform of the differential equation and solve for y(s).
4
s2 y sy(0) y 0 (0) + 4sy 4y(0) + 4y =
s+1
4
s2 y 2s + 3 + 4sy 8 + 4y =
s+1
4 2s + 5
y = +
(s + 1)(s + 2)2 (s + 2)2
4 2 3
y =
s + 1 s + 2 (s + 2)2

We take the inverse Laplace transform to determine the solution.

y = 4 et (2 + 3t) e2t

1538
Chapter 32

The Fourier Transform

32.1 Derivation from a Fourier Series


Consider the eigenvalue problem

y 00 + y = 0, y(L) = y(L), y 0 (L) = y 0 (L).

The eigenvalues and eigenfunctions are


 n 2
n = for n Z0+
L
nx/L
n = e , for n Z
L

The eigenfunctions form an orthogonal set. A piecewise continuous function defined on [L . . . L] can be expanded in
a series of the eigenfunctions.

X
f (x) cn enx/L
n=
L

1539
The Fourier coefficients are
D E

enx/L f (x)

L
cn = D E

enx/L L enx/L

L
Z L
1
= enx/L f (x) dx.
2 L

We substitute the expression for cn into the series for f (x).


 Z L 
X 1 n/L
f (x) e f () d enx/L .
n=
2L L

We let n = n/L and = /L.


 Z L 
X 1 n
f (x) e f () d en x .
=
2 L
n

In the limit as L , (and thus 0), the sum becomes an integral.


Z  Z 
1
f (x) e f () d ex d.
2

Thus the expansion of f (x) for finite L



X nx/L
f (x) cn e
n=
L
Z L
1
cn = enx/L f (x) dx
2 L

1540
in the limit as L becomes
Z
f (x) f() ex d

Z
1
f() = f (x) ex dx.
2

Of course this derivation is only heuristic. In the next section we will explore these formulas more carefully.

32.2 The Fourier Transform


R
Let f (x) be piecewise continuous and let
|f (x)| dx exist. We define the function I(x, L).
Z L Z 
1
I(x, L) = f () e
d ex d.
2 L

Since the integral in parentheses is uniformly convergent, we can interchange the order of integration.
Z Z L 
1 (x)
= f () e d d
2 L
Z  L
1 e(x)
= f () d
2 ( x) L
Z
1 1
eL(x) eL(x) d

= f ()
2 ( x)
Z
1 sin(L( x))
= f () d
x
1
Z
sin(L)
= f ( + x) d.

1541
In Example 32.3.3 we will show that
Z
sin(L)
d = .
0 2

Continuous Functions. Suppose that f (x) is continuous.

1
Z
sin(L)
f (x) = f (x) d

1 f (x + ) f (x)
Z
I(x, L) f (x) = sin(L) d.

R f (x+)f (x)
If f (x) has a left and right derivative at x then f (x+)f

(x)
is bounded and d < . We use the
Riemann-Lebesgue lemma to show that the integral vanishes as L .

f (x + ) f (x)
Z
1
sin(L) d 0 as L .

Now we have an identity for f (x).


Z Z 
1
f (x) = f () e
d ex d.
2

Piecewise Continuous Functions. Now consider the case that f (x) is only piecewise continuous.

f (x+ ) 1
Z
sin(L)
= f (x+ ) d
2 0
f (x ) 1 0
Z
sin(L)
= f (x ) d
2

1542
0
f (x+ ) + f (x ) f (x + ) f (x )
Z  
I(x, L) = sin(L) d
2
Z 
f (x + ) f (x+ )

sin(L) d
0

If f (x) has a left and right derivative at x, then

f (x + ) f (x )
is bounded for 0, and

f (x + ) f (x+ )
is bounded for 0.

Again using the Riemann-Lebesgue lemma we see that


f (x+ ) + f (x )
Z Z 
1
= f () e
d ex d.
2 2

1543
R
Result 32.2.1 Let f (x) be piecewise continuous with |f (x)| dx < . The Fourier
transform of f (x) is defined
Z
1
f() = F[f (x)] = f (x) ex dx.
2
We see that the integral is uniformly convergent. The inverse Fourier transform is defined
Z
f (x+ ) + f (x ) 1
= F [f ()] = f() ex d.
2

If f (x) is continuous then this reduces to


Z
f (x) = F 1
[f()] = f() ex d.

32.2.1 A Word of Caution


Other texts may define the Fourier transform differently. The important relation is
Z  Z 
1
f (x) = f () e d ex d.
2

Multiplying the right side of this equation by 1 = 1 yields


Z  Z 
1
f (x) = f () e d ex d.
2

1544

Setting = 2 and choosing sign in the exponentials gives us the Fourier transform pair
Z
1
f() = f (x) ex dx
2
Z
1
f (x) = f() ex d.
2
Other equally valid pairs are
Z

f () = f (x) ex dx

Z
1
f (x) = f() ex d,
2
and
Z
f() = f (x) ex dx

Z
1
f (x) = f() ex d.
2

Be aware of the different definitions when reading other texts or consulting tables of Fourier transforms.

32.3 Evaluating Fourier Integrals


32.3.1 Integrals that Converge
If the Fourier integral Z
1
F[f (x)] = f (x) ex dx,
2
converges for real , then finding the transform of a function is just a matter of direct integration. We will consider
several examples of such garden variety functions in this subsection. Later on we will consider the more interesting
cases when the integral does not converge for real .

1545
Example 32.3.1 Consider the Fourier transform of ea|x| , where a > 0. Since the integral of ea|x| is absolutely
convergent, we know that the Fourier transform integral converges for real . We write out the integral.

Z
 a|x|  1
F e = ea|x| ex dx
2
Z 0 Z
1 1
= e axx
dx + eaxx dx
2 2 0
Z 0 Z
1 (a<()+=())x 1
= e dx + e(a<()+=())x dx
2 2 0

The integral converges for |=()| < a. This domain is shown in Figure 32.1.

Im(z)

Re(z)

Figure 32.1: The Domain of Convergence

1546
Now We do the integration.
Z 0 Z
1 1
F ea|x| = (a)x
e(a+)x dx
 
e dx +
2 2 0
 (a)x 0  (a+)x 
1 e 1 e
= +
2 a 2 a + 0
 
1 1 1
= +
2 a a +
1 a
= , for |=()| < a
( + a2 )
2

We can extend the domain of the Fourier transform with analytic continuation.
a
F ea|x| =
 
, for 6= a
( + a2 )
2

1
Example 32.3.2 Consider the Fourier transform of f (x) = x , > 0.
  Z
1 1 1
F = ex dx
x 2 x
The integral converges for =() = 0. We will evaluate the integral for positive and negative real values of .

For > 0, we will close the path of integration in the lower half-plane. Let CR be the contour from x = R to
x = R following a semicircular path in the lower half-plane. The integral along CR vanishes as R by Jordans
Lemma. Z
1
ex dx 0 as R .
CR x
Since the integrand is analytic in the lower half-plane the integral vanishes.
 
1
F =0
x

1547
For < 0, we will close the path of integration in the upper half-plane. Let CR denote the semicircular contour from
x = R to x = R in the upper half-plane. The integral along CR vanishes as R goes to infinity by Jordans Lemma.
We evaluate the Fourier transform integral with the Residue Theorem.
   x 
1 1 e
F = 2i Res , i
x 2 x i
= e

We combine the results for positive and negative values of .


  (
1 0 for > 0,
F =
x e for < 0

32.3.2 Cauchy Principal Value and Integrals that are Not Absolutely Convergent.
That the integral of f (x) is Rabsolutely convergent is a sufficient but not
R a necessary condition that the Fourier transform
x
of f (x) exists. The integral f (x) e dx may converge even if |f (x)| dx does not. Furthermore, if the Fourier
transform integral diverges, its principal value may exist. We will say that the Fourier transform of f (x) exists if the
principal value of the integral exists. Z
F[f (x)] = f (x) ex dx

Example 32.3.3 Consider the Fourier transform of f (x) = 1/x.


Z
1 1 x
f() = e dx
2 x

If > 0, we can close the contour in the lower half-plane. The integral along the semi-circle vanishes due to Jordans
Lemma. Z
1 x
lim e dx = 0
R C x
R

1548
We can evaluate the Fourier transform with the Residue Theorem.
   
1 1 1
f() = (2i) Res e x
,0
2 2 x

f() = , for > 0.
2
The factor of 1/2 in the above derivation arises because the path of integration is in the negative, (clockwise),
direction and the path of integration crosses through the first order pole at x = 0. The path of integration is shown in
Figure 32.2.

Im(z)

Re(z)

Figure 32.2: The Path of Integration

If < 0, we can close the contour in the upper half plane to obtain

f() = , for < 0.
2
1
For = 0 the integral vanishes because x
is an odd function.
Z
1 1
f (0) = = dx = 0
2 x

1549
We collect the results in one formula.

f() = sign()
2
We write the integrand for > 0 as the sum of an odd and and even function.
Z
1 1 x
e dx =
2 x 2
Z Z
1
cos(x) dx + sin(x) dx =
x x

The principal value of the integral of any odd function is zero.


Z
1
sin(x) dx =
x

If the principal value of the integral of an even function exists, then the integral converges.
Z
1
sin(x) dx =
x
Z
1
sin(x) dx =
0 x 2
Thus we have evaluated an integral that we used in deriving the Fourier transform.

32.3.3 Analytic Continuation


Consider the Fourier transform of f (x) = 1. The Fourier integral is not convergent, and its principal value does not
exist. Thus we will have to be a little creative in order to define the Fourier transform. Define the two functions

1
for x > 0 0
for x > 0
f+ (x) = 1/2 for x = 0 , f (x) = 1/2 for x = 0 .

0 for x < 0 1 for x < 0

1550
Note that 1 = f (x) + f+ (x).

The Fourier transform of f+ (x) converges for =() < 0.


Z
1
F[f+ (x)] = ex dx
2 0
Z
1
= e(<()+=())x dx.
2 0

1 ex

=
2 0

= for =() < 0
2
Using analytic continuation, we can define the Fourier transform of f+ (x) for all except the point = 0.

F[f+ (x)] =
2

We follow the same procedure for f (x). The integral converges for =() > 0.
Z 0
1
F[f (x)] = ex dx
2
Z 0
1
= e(<()+=())x dx
2
0
1 ex

=
2

= .
2
Using analytic continuation we can define the transform for all nonzero .

F[f (x)] =
2

1551
Now we are prepared to define the Fourier transform of f (x) = 1.

F[1] = F[f (x)] + F[f+ (x)]



= +
2 2
= 0, for 6= 0

When = 0 the integral diverges. When we consider the closure relation for the Fourier transform we will see that

F[1] = ().

32.4 Properties of the Fourier Transform


In this section we will explore various properties of the Fourier Transform. I would like to avoid stating assumptions on
various functions at the beginning of each subsection. Unless otherwise indicated, assume that the integrals converge.

32.4.1 Closure Relation.


Recall the closure relation for an orthonormal set of functions {1 , 2 , . . .},

X
n (x)n () (x ).
n=1

There is a similar closure relation for Fourier integrals. We compute the Fourier transform of (x ).
Z
1
F[(x )] = (x ) ex dx
2
1
= e
2

1552
Next we take the inverse Fourier transform.
Z
1 x
(x ) e e d
2
Z
1
(x ) e(x) d.
2
Note that the integral is divergent, but it would be impossible to represent (x ) with a convergent integral.

32.4.2 Fourier Transform of a Derivative.


Consider the Fourier transform of y 0 (x).
Z
0 1
F[y (x)] = y 0 (x) ex dx
2
  Z
1 x 1
= y(x) e ()y(x) ex dx
2 2
Z
1
= y(x) ex dx
2
= F[y(x)]
Next consider y 00 (x).
 
00 d 0
F[y (x)] = F (y (x))
dx
= F[y 0 (x)]
= ()2 F[y(x)]
= 2 F[y(x)]
In general,
F y (n) (x) = ()n F[y(x)].
 

1553
Example 32.4.1 The Dirac delta function can be expressed as the derivative of the Heaviside function.
(
0 for x < c,
H(x c) =
1 for x > c

Thus we can express the Fourier transform of H(x c) in terms of the Fourier transform of the delta function.

F[(x c)] = F[H(x c)]


Z
1
(x c) ex dx = F[H(x c)]
2
1 c
e = F[H(x c)]
2
1 c
F[H(x c)] = e
2

32.4.3 Fourier Convolution Theorem.


Consider the Fourier transform of a product of two functions.
Z
1
F[f (x)g(x)] = f (x)g(x) ex dx
2
Z Z 
1
= f() e d g(x) ex dx
x
2
Z Z 
1 ()x
= f ()g(x) e dx d
2
Z  Z 
1 ()x
= f () g(x) e dx d
2
Z
= f()G( ) d

1554
The convolution of two functions is defined
Z
f g(x) = f ()g(x ) d.

Thus Z
F[f (x)g(x)] = f g() = f()g( ) d.

Now consider the inverse Fourier Transform of a product of two functions.


Z
1
F [f ()g()] = f()g() ex d
Z  Z 
1
= f () e d g() ex d
2
Z Z 
1 (x)
= f ()g() e d d
2
Z Z 
1 (x)
= f () g() e d d
2
Z
1
= f ()g(x ) d
2

Thus
Z
1 1
F 1
[f()g()] = f g(x) = f ()g(x ) d,
2 2

F[f g(x)] = 2 f()g().

These relations are known as the Fourier convolution theorem.

1555
Example 32.4.2 Using the convolution theorem and the table of Fourier transform pairs in the appendix, we can find
the Fourier transform of
1
f (x) = 4 .
x + 5x2 + 4
We factor the fraction.
1
f (x) =
(x2 + 1)(x2 + 4)
From the table, we know that
 
2c
F 2 = ec|| for c > 0.
x + c2
We apply the convolution theorem.
 
1 2 4
F[f (x)] = F
8 x2 + 1 x2 + 4
Z 
1 || 2||
= e e d
8
Z 0 Z 
1 2|| 2||
= e e d + e e d
8 0

First consider the case > 0.


Z 0 Z Z 
1 2+3 2+ 23
F[f (x)] = e d + e d + e d
8 0
 
1 1 2 2 1
= e +e e + e
8 3 3
1 1
= e e2
6 12

1556
Now consider the case < 0.
Z Z 0 Z 
1 2+3 2 23
F[f (x)] = e d + e d + e d
8 0
 
1 1 2 1 2
= e e +e + e
8 3 3
1 1 2
= e e
6 12
We collect the result for positive and negative .
1 || 1 2||
F[f (x)] = e e
6 12
A better way to find the Fourier transform of
1
f (x) =
x4 + 5x2 + 4
is to first expand the function in partial fractions.
1/3 1/3
f (x) = 2
x2
+1 x +4
   
1 2 1 4
F[f (x)] = F 2 F 2
6 x +1 12 x +4
1 || 1 2||
= e e
6 12

32.4.4 Parsevals Theorem.


P
Recall Parsevals theorem for Fourier series. If f (x) is a complex valued function with the Fourier series n= cn enx
then Z
X
2
2 |cn | = |f (x)|2 dx.
n=

1557
Analogous to this result is Parsevals theorem for Fourier transforms.

Let f (x) be a complex valued function that is both absolutely integrable and square integrable.
Z Z
|f (x)| dx < and |f (x)|2 dx <

The Fourier transform of f (x) is f().


Z
h i 1
F f (x) = f (x) ex dx
2
Z
1
= f (x) ex dx
2
Z
1
= f (x) ex dx
2
= f()
We apply the convolution theorem.
Z
F 1
[2 f()f()] = f ()f ((x )) d

Z Z
2 f()f() ex d = f ()f ( x) d

We set x = 0.
Z Z
2 f()f() d = f ()f () d

Z Z
2 |f()|2 d = |f (x)|2 dx

This is known as Parsevals theorem.

1558
32.4.5 Shift Property.
The Fourier transform of f (x + c) is
Z
1
F[f (x + c)] = f (x + c) ex dx
2
Z
1
= f (x) e(xc) dx
2

F[f (x + c)] = ec f()

The inverse Fourier transform of f( + c) is


Z
F 1
[f( + c)] = f( + c) ex d
Z

= f() e(c)x d

F 1 [f( + c)] = ecx f (x)

32.4.6 Fourier Transform of x f(x).


The Fourier transform of xf (x) is
Z
1
F[xf (x)] = xf (x) ex dx
2
Z
1
= f (x) (ex ) dx
2
 Z 
1 x
= f (x) e dx
2

1559
f
F[xf (x)] = .

Similarly, you can show that
n f
F[xn f (x)] = (i)n .
n

32.5 Solving Differential Equations with the Fourier Transform


The Fourier transform is useful in solving some differential equations on the domain ( . . . ) with homogeneous
boundary conditions at infinity. We take the Fourier transform of the differential equation L[y] = f and solve for y. We
take the inverse transform to determine the solution y. Note that this process is only applicable if the Fourier transform
of y exists. Hence the requirement for homogeneous boundary conditions at infinity.
We will use the table of Fourier transforms in the appendix in solving the examples in this section.

Example 32.5.1 Consider the problem


y 00 y = e|x| , y() = 0, > 0, 6= 1.
We take the Fourier transform of this equation.
/
2 y() y() =
2 + 2
We take the inverse Fourier transform to determine the solution.
/
y() =
( 2
+ 2 )( 2 + 1)
 
1 1 1
=
2 1 2 + 1 2 + 2
 
1 / 1/
= 2 2
1 2 + 2 +1

1560
e|x| e|x|
y(x) =
2 1

Example 32.5.2 Consider the Green function problem


G00 G = (x ), y() = 0.
We take the Fourier transform of this equation.
2 G G = F[(x )]
1
G = 2 F[(x )]
+1
We use the Table of Fourier transforms.

G = F e|x| F[(x )]
 

We use the convolution theorem to do the inversion.


Z
1
G = e|x| ( ) d
2
1
G(x|) = e|x|
2
The inhomogeneous differential equation
y 00 y = f (x), y() = 0,
has the solution Z
1
y= f () e|x| d.
2

When solving the differential equation L[y] = f with the Fourier transform, it is quite common to use the convolution
theorem. With this approach we have no need to compute the Fourier transform of the right side. We merely denote
it as F[f ] until we use f in the convolution integral.

1561
32.6 The Fourier Cosine and Sine Transform
32.6.1 The Fourier Cosine Transform
Suppose f (x) is an even function. In this case the Fourier transform of f (x) coincides with the Fourier cosine transform
of f (x).
Z
1
F[f (x)] = f (x) ex dx
2
Z
1
= f (x)(cos(x) sin(x)) dx
2
Z
1
= f (x) cos(x) dx
2
1
Z
= f (x) cos(x) dx
0
The Fourier cosine transform is defined:
Z
1
Fc [f (x)] = fc () = f (x) cos(x) dx.
0

Note that fc () is an even function. The inverse Fourier cosine transform is


Z
1
Fc [fc ()] = fc () ex d
Z

= fc ()(cos(x) + sin(x)) d
Z

= fc () cos(x) d

Z
=2 fc () cos(x) d.
0

1562
Thus we have the Fourier cosine transform pair
Z Z
1 1
f (x) = Fc [fc ()] = 2 fc () cos(x) d, fc () = Fc [f (x)] = f (x) cos(x) dx.
0 0

32.6.2 The Fourier Sine Transform


Suppose f (x) is an odd function. In this case the Fourier transform of f (x) coincides with the Fourier sine transform
of f (x).
Z
1
F[f (x)] = f (x) ex dx
2
Z
1
= f (x)(cos(x) sin(x)) dx
2

Z
= f (x) sin(x) dx
0

Note that f() = F[f (x)] is an odd function of . The inverse Fourier transform of f() is
Z
1
F [f ()] = f() ex d

Z
= 2 f() sin(x) d.
0

Thus we have that




Z Z 
f (x) = 2 f (x) sin(x) dx sin(x) d
0 0
Z  Z 
1
=2 f (x) sin(x) dx sin(x) d.
0 0

1563
This gives us the Fourier sine transform pair
Z Z
1
f (x) = Fs1 [fs ()] =2 fs () sin(x) d, fs () = Fs [f (x)] = f (x) sin(x) dx.
0 0

Result 32.6.1 The Fourier cosine transform pair is defined:


Z
f (x) = Fc1 [fc ()] = 2 fc () cos(x) d
Z 0
1
fc () = Fc [f (x)] = f (x) cos(x) dx
0
The Fourier sine transform pair is defined:
Z
f (x) = Fs1 [fs ()]
=2 fs () sin(x) d
Z 0
1
fs () = Fs [f (x)] = f (x) sin(x) dx
0

32.7 Properties of the Fourier Cosine and Sine Transform

32.7.1 Transforms of Derivatives


Cosine Transform. Using integration by parts we can find the Fourier cosine transform of derivatives. Let y be a
function for which the Fourier cosine transform of y and its first and second derivatives exists. Further assume that y

1564
and y 0 vanish at infinity. We calculate the transforms of the first and second derivatives.

1 0
Z
0
Fc [y ] = y cos(x) dx
0

Z
1
= y cos(x) 0 + y sin(x) dx
0
1
= yc () y(0)
Z
1
Fc [y 00 ] = y 00 cos(x) dx
0
 0
Z
1 0
= y cos(x) 0 + y sin(x) dx
0
 2
Z
1 0 
= y (0) + y sin(x) 0 y cos(x) dx
0
1
= 2 fc () y 0 (0)

Sine Transform. You can show, (see Exercise 32.3), that the Fourier sine transform of the first and second derivatives
are

Fs [y 0 ] = fc ()

Fs [y 00 ] = 2 yc () + y(0).

1565
32.7.2 Convolution Theorems

Cosine Transform of a Product. Consider the Fourier cosine transform of a product of functions. Let f (x) and
g(x) be two functions defined for x 0. Let Fc [f (x)] = fc (), and Fc [g(x)] = gc ().

1
Z
Fc [f (x)g(x)] = f (x)g(x) cos(x) dx
0
1
Z  Z 
= 2 fc () cos(x) d g(x) cos(x) dx
0 0
2
Z Z
= fc ()g(x) cos(x) cos(x) dx d
0 0

We use the identity cos a cos b = 21 (cos(a b) + cos(a + b)).

1
Z Z

= fc ()g(x) cos(( )x) + cos(( + )x) dx d

Z 0 0
 Z Z 
1 1
= fc () g(x) cos(( )x) dx + g(x) cos(( + )x) dx d
0 0 0
Z
fc () gc ( ) + gc ( + ) d

=
0

gc () is an even function. If we have only defined gc () for positive argument, then gc () = gc (||).

Z
fc () gc (| |) + gc ( + ) d

=
0

1566
Inverse Cosine Transform of a Product. Now consider the inverse Fourier cosine transform of a product of
functions. Let Fc [f (x)] = fc (), and Fc [g(x)] = gc ().
Z
Fc1 [fc ()gc ()] =2 fc ()gc () cos(x) d
0
Z  Z 
1
=2 f () cos() d gc () cos(x) d
0 0
2
Z Z
= f ()gc () cos() cos(x) d d
0
Z Z0
1 
= f ()gc () cos((x )) + cos((x + )) d d
0
Z 0  Z Z 
1
= f () 2 gc () cos((x )) d + 2 gc () cos((x + )) d d
2 0 0 0
Z
1 
= f () g(|x |) + g(x + ) d
2 0

Sine Transform of a Product. You can show, (see Exercise 32.5), that the Fourier sine transform of a product
of functions is
Z
fs () gc (| |) gc ( + ) d.

Fs [f (x)g(x)] =
0

Inverse Sine Transform of a Product. You can also show, (see Exercise 32.6), that the inverse Fourier sine
transform of a product of functions is
Z
1
Fs1 [fs ()gc ()]

= f () g(|x |) g(x + ) d.
2 0

1567
Result 32.7.1 The Fourier cosine and sine transform convolution theorems are
Z
fc () gc (| |) + gc ( + ) d
 
Fc [f (x)g(x)] =
0 Z
1 1 
Fc [fc ()gc ()] = f () g(|x |) + g(x + ) d
2
Z 0
fs () gc (| |) gc ( + ) d

Fs [f (x)g(x)] =
0 Z
1
Fs1 [fs ()gc ()] =

f () g(|x |) g(x + ) d
2 0

32.7.3 Cosine and Sine Transform in Terms of the Fourier Transform


We can express the Fourier cosine and sine transform in terms of the Fourier transform. First consider the Fourier
cosine transform. Let f (x) be an even function.
1
Z
Fc [f (x)] = f (x) cos(x) dx
0
We extend the domain integration because the integrand is even.
Z
1
= f (x) cos(x) dx
2
R
Note that f (x) sin(x) dx = 0 because the integrand is odd.
Z
1
= f (x) ex dx
2
= F[f (x)]

1568
Fc [f (x)] = F[f (x)], for even f (x).
For general f (x), use the even extension, f (|x|) to write the result.

Fc [f (x)] = F[f (|x|)]

There is an analogous result for the inverse Fourier cosine transform.


h i h i
Fc1 f() = F 1 f(||)

For the sine series, we have


h i h i
Fs [f (x)] = F [sign(x)f (|x|)] Fs1 f() = F 1 sign()f(||)

Result 32.7.2 The results:


h i h i
Fc [f (x)] = F[f (|x|)] Fc1
f () = F 1
f (||)
h i h i
Fs [f (x)] = F[sign(x)f (|x|)] Fs1
f () = F 1
sign()f (||)
allow us to evaluate Fourier cosine and sine transforms in terms of the Fourier transform.
This enables us to use contour integration methods to do the integrals.

32.8 Solving Differential Equations with the Fourier Cosine and


Sine Transforms
Example 32.8.1 Consider the problem

y 00 y = 0, y(0) = 1, y() = 0.

1569
Since the initial condition is y(0) = 1 and the sine transform of y 00 is 2 yc () + y(0) we take the Fourier sine
transform of both sides of the differential equation.

2 yc () + y(0) yc () = 0

2
( + 1)yc () =


yc () =
( 2 + 1)
We use the table of Fourier Sine transforms.

y = ex
Example 32.8.2 Consider the problem
y 00 y = e2x , y 0 (0) = 0, y() = 0.
Since the initial condition is y 0 (0) = 0, we take the Fourier cosine transform of the differential equation. From the table
of cosine transforms, Fc [e2x ] = 2/(( 2 + 4)).
1 0 2
2 yc () y (0) yc () = 2
( + 4)

2
yc () =
( 2
+ 4)( 2 + 1)
 
2 1/3 1/3
=
2 + 1 2 + 4
1 2/ 2 1/
=
3 + 4 3 2 + 1
2

1 2x 2 x
y= e e
3 3

1570
32.9 Exercises
Exercise 32.1
Show that
sin(c)
H(x + c) H(x c) = .

Hint, Solution
Exercise 32.2
Using contour integration, find the Fourier transform of
1
f (x) = ,
x2 + c2
where <(c) 6= 0
Hint, Solution
Exercise 32.3
Find the Fourier sine transforms of y 0 (x) and y 00 (x).
Hint, Solution
Exercise 32.4
Prove the following identities.
1. F[f (x a)] = ea f()
1  
2. F[f (ax)] = f
|a| a
Hint, Solution
Exercise 32.5
Show that Z
fs () gc (| |) gc ( + ) d.

Fs [f (x)g(x)] =
0

1571
Hint, Solution
Exercise 32.6
Show that Z
1
Fs1 [fs ()gc ()]

= f () g(|x |) g(x + ) d.
2 0
Hint, Solution
Exercise 32.7
Let fc () = Fc [f (x)], fc () = Fs [f (x)], and assume the cosine and sine transforms of xf (x) exist. Express Fc [xf (x)]
and Fs [xf (x)] in terms of fc () and fc ().
Hint, Solution
Exercise 32.8
Solve the problem
y 00 y = e2x , y(0) = 1, y() = 0,
using the Fourier sine transform.
Hint, Solution
Exercise 32.9
Prove the following relations between the Fourier sine transform and the Fourier transform.

Fs [f (x)] = F[sign(x)f (|x|)]


h i h i
1 1
Fs f () = F sign()f (||)

Hint, Solution
Exercise 32.10
Let fc () = Fc [f (x)] and fc () = Fs [f (x)]. Show that

1. Fc [xf (x)] = f ()
c

1572

2. Fs [xf (x)] = fc ()

3. Fc [f (cx)] = 1c fc

c
for c > 0

4. Fs [f (cx)] = 1c fc

c
for c > 0.

Hint, Solution
Exercise 32.11
Solve the integral equation,
Z
2 2
u() ea(x) d = ebx ,

where a, b > 0, a 6= b, with the Fourier transform.


Hint, Solution
Exercise 32.12
Evaluate
Z
1 1 cx
e sin(x) dx,
0 x
where is a positive, real number and <(c) > 0.
Hint, Solution
Exercise 32.13
Use the Fourier transform to solve the equation

y 00 a2 y = ea|x|

on the domain < x < with boundary conditions y() = 0.


Hint, Solution

1573
Exercise 32.14
1. Use the cosine transform to solve

y 00 a2 y = 0 on x 0 with y 0 (0) = b, y() = 0.

2. Use the cosine transform to show that the Green function for the above with b = 0 is
1 a|x| 1 a(x)
G(x, ) = e e .
2a 2a
Hint, Solution
Exercise 32.15
1. Use the sine transform to solve

y 00 a2 y = 0 on x 0 with y(0) = b, y() = 0.

2. Try using the Laplace transform on this problem. Why isnt it as convenient as the Fourier transform?

3. Use the sine transform to show that the Green function for the above with b = 0 is
1 a(x)
ea|x+|

g(x; ) = e
2a
Hint, Solution
Exercise 32.16
1. Find the Green function which solves the equation

y 00 + 2y 0 + ( 2 + 2 )y = (x ), > 0, > 0,

in the range < x < with boundary conditions y() = y() = 0.

1574
2. Use this Greens function to show that the solution of
y 00 + 2y 0 + ( 2 + 2 )y = g(x), > 0, > 0, y() = y() = 0,
with g() = 0 in the limit as 0 is
Z x
1
y= g() sin[(x )]d.

You may assume that the interchange of limits is permitted.


Hint, Solution
Exercise 32.17
Using Fourier transforms, find the solution u(x) to the integral equation
Z
u() 1
2 2
d = 2 0 < a < b.
[(x ) + a ] x + b2
Hint, Solution
Exercise 32.18
The Fourer cosine transform is defined by
Z
1
fc () = f (x) cos(x) dx.
0

1. From the Fourier theorem show that the inverse cosine transform is given by
Z
f (x) = 2 fc () cos(x) d.
0

2. Show that the cosine transform of f 00 (x) is


f 0 (0)
2 fc () .

1575
3. Use the cosine transform to solve the following boundary value problem.
y 00 a2 y = 0 on x > 0 with y 0 (0) = b, y() = 0

Hint, Solution
Exercise 32.19
The Fourier sine transform is defined by
Z
1
fs () = f (x) sin(x) dx.
0

1. Show that the inverse sine transform is given by


Z
f (x) = 2 fs () sin(x) d.
0

2. Show that the sine transform of f 00 (x) is



f (0) 2 fs ().

3. Use this property to solve the equation
y 00 a2 y = 0 on x > 0 with y(0) = b, y() = 0.

4. Try using the Laplace transform on this problem. Why isnt it as convenient as the Fourier transform?
Hint, Solution
Exercise 32.20
Show that
1
F[f (x)] = (Fc [f (x) + f (x)] Fs [f (x) f (x)])
2
where F, Fc and Fs are respectively the Fourier transform, Fourier cosine transform and Fourier sine transform.
Hint, Solution

1576
Exercise 32.21
Find u(x) as the solution to the integral equation:
Z
u() 1
2 2
d = 2 , 0 < a < b.
(x ) + a x + b2

Use Fourier transforms and the inverse transform. Justify the choice of any contours used in the complex plane.
Hint, Solution

1577
32.10 Hints
Hint 32.1
(
1 for |x| < c,
H(x + c) H(x c) =
0 for |x| > c

Hint 32.2
Consider the two cases <() < 0 and <() > 0, closing the path of integration with a semi-circle in the lower or upper
half plane.

Hint 32.3

Hint 32.4

Hint 32.5

Hint 32.6

Hint 32.7

Hint 32.8

Hint 32.9

1578
Hint 32.10

Hint 32.11
2
The left side is the convolution of u(x) and eax .

Hint 32.12

Hint 32.13

Hint 32.14

Hint 32.15

Hint 32.16

Hint 32.17

Hint 32.18

Hint 32.19

Hint 32.20

1579
Hint 32.21

1580
32.11 Solutions
Solution 32.1

Z
1
F[H(x + c) H(x c)] = (H(x + c) H(x c)) ex dx
2
Z
c
1
= ex dx
2
cx c
1 e
=
2
 c c c 
1 e e
=
2

sin(c)
F[H(x + c) H(x c)] =

Solution 32.2

  Z
1 1 1
F 2 = ex dx
x + c2 2 x2 + c2
Z
1 ex
= dx
2 (x c)(x + c)

If <() < 0 then we close the path of integration with a semi-circle in the upper half plane.

ex
   
1 1 1 c
F 2 2
= 2i Res , x = c = e
x +c 2 (x c)(x + c) 2c

1581
If > 0 then we close the path of integration in the lower half plane.

ex
   
1 1 1 c
F 2 2
= 2i Res , c = e
x +c 2 (x c)(x + c) 2c

Thus we have that


 
1 1 c||
F 2 2
= e , for <(c) 6= 0.
x +c 2c

Solution 32.3

1 0
Z
0
Fs [y ] = y sin(x) dx
0
1h i Z
= y sin(x) y cos(x) dx
0 0
= yc ()
1 00
Z
00
Fs [y ] = y sin(x) dx
0
1h 0 i Z
= y sin(x) y 0 cos(x) dx
0 0
h i 2 Z
= y cos(x) y sin(x) dx
0 0

= 2 ys () + y(0).

1582
Solution 32.4
1.
Z
1
F[f (x a)] = f (x a) ex dx
2
Z
1
= f (x) e(x+a) dx
2
Z
a 1
=e f (x) ex dx
2

F[f (x a)] = ea f()


2. If a > 0, then
Z
1
F[f (ax)] = f (ax) ex dx
2
Z
1 1
= f () e/a d
2 a
1  
= f .
a a
If a < 0, then
Z
1
F[f (ax)] = f (ax) ex dx
2
Z
1 1
= e/a d
2 a
1  
= f .
a a
Thus
1  
F[f (ax)] = f .
|a| a

1583
Solution 32.5

1
Z
Fs [f (x)g(x)] = f (x)g(x) sin(x) dx
0
1
Z  Z 
= 2
fs () sin(x) d g(x) sin(x) dx
0 0
2
Z Z
= fs ()g(x) sin(x) sin(x) dx d
0 0

Use the identity, sin a sin b = 12 [cos(a b) cos(a + b)].

Z Z
1
h i
= fs ()g(x) cos(( )x) cos(( + )x) dx d
0 0
 Z
1
Z Z 
1
= fs () g(x) cos(( )x) dx g(x) cos(( + )x) dx d
0 0 0

Z
fs () Gc (| |) Gc ( + ) d
 
Fs [f (x)g(x)] =
0

1584
Solution 32.6

Z
Fs1 [fs ()Gc ()] =2 fs ()Gc () sin(x) d
Z0  Z 
1
=2 f () sin() d Gc () sin(x) d
0 0
2
Z Z
= f ()Gc () sin() sin(x) d d
0 0
1
Z Z h i
= f ()Gc () cos((x )) cos((x + )) d d
0
Z 0  Z Z 
1
= f () 2 Gc () cos((x )) d 2 Gc () cos((x + )) d) d
2 0 0 0
Z
1
= f ()[g(x ) g(x + )] d
2 0
Z
1 1  
Fs [fs ()Gc ()] = f () g(|x |) g(x + ) d
2 0

Solution 32.7

1
Z
Fc [xf (x)] = xf (x) cos(x) dx
0
1
Z

= f (x) (sin(x)) dx
0
Z
1
= f (x) sin(x) dx
0

= fs ()

1585
1
Z
Fs [xf (x)] = xf (x) sin(x) dx
0
1
Z

= f (x) ( cos(x)) dx
0
Z
1
= f (x) cos(x) dx
0

= fc ()

Solution 32.8

y 00 y = e2x , y(0) = 1, y() = 0


We take the Fourier sine transform of the differential equation.
2/
2 ys () + y(0) ys () = 2
+4
/ /
ys () = + 2
( 2 2
+ 4)( + 1) ( + 1)
/(3) /(3) /
= 2 2 + 2
+4 +1 +1
2 / 1 /
= +
3 + 1 3 2 + 4
2

2 x 1 2x
y= e + e
3 3

Solution 32.9
Consider the Fourier sine transform. Let f (x) be an odd function.
1
Z
Fs [f (x)] = f (x) sin(x) dx
0

1586
Extend the integration because the integrand is even.
Z
1
= f (x) sin(x) dx
2
R
Note that
f (x) cos(x) dx = 0 as the integrand is odd.
Z
1
= f (x) ex dx
2
= F[f (x)]
Fs [f (x)] = F[f (x)], for odd f (x).
For general f (x), use the odd extension, sign(x)f (|x|) to write the result.
Fs [f (x)] = F[sign(x)f (|x|)]
Now consider the inverse Fourier sine transform. Let f() be an odd function.
h i Z
1
Fs f () = 2 f() sin(x) d
0

Extend the integration because the integrand is even.


Z
= f() sin(x) d

R
Note that
f() cos(x) d = 0 as the integrand is odd.
Z
= f()(i) ex d

h i
= F 1 f()

1587
h i h i
Fs1 f() = F 1 f() , for odd f().

For general f(), use the odd extension, sign()f(||) to write the result.
h i h i
Fs1
f () = F 1
sign()f (||)

Solution 32.10

1
Z
Fc [xf (x)] = xf (x) cos(x) dx
0
Z
1
= f (x) sin(x) dx
0
1
Z
= f (x) sin(x) dx
0

= fs ()

1
Z
Fs [xf (x)] = xf (x) sin(x) dx
0
1
Z

= f (x) ( cos(x)) dx
0
Z
1
= f (x) cos(x) dx
0

= fc ()

1588
1
Z
Fc [f (cx)] = f (cx) cos(x) dx
0
1
Z   d
= f () cos
0 c c
1  
= fc
c c

1
Z
Fs [f (cx)] = f (cx) sin(x) dx
0
1
Z   d
= f () sin
0 c c
1  
= fs
c c
Solution 32.11
Z
2 2
u() ea(x) d = ebx

We take the Fourier transform and solve for U ().


h 2
i h 2
i
2U ()F eax = F ebx
1 2 1 2
2U () e /(4a) = e /(4b)
4a 4b
r
1 a 2 (ab)/(4ab)
U () = e
2 b
Now we take the inverse Fourier transform.
r p
1 a 4ab/(a b) 2 (ab)/(4ab)
U () = p e
2 b 4ab/(a b)

1589
a 2 /(ab)
u(x) = p eabx
(a b)

Solution 32.12

Z
1 1 cx
I= e sin(x) dx
0 x
Z Z 
1 zx
= e dz sin(x) dx
0 c
Z Z
1
= ezx sin(x) dx dz

Zc 0
1
= dz
c z + 2
2

1 h  z i
= arctan
c
1   c 
= arctan
2
1  
= arctan
c
Solution 32.13
We consider the differential equation
y 00 a2 y = ea|x|
on the domain < x < with boundary conditions y() = 0. We take the Fourier transform of the differential
equation and solve for y().
a
2 y a2 y =
( + a2 )
2
a
y() =
( + a2 )2
2

1590
We take the inverse Fourier transform to find the solution of the differential equation.
Z
a
y(x) = ex d
( 2 + a2 )2

Note that since y() is a real-valued, even function, y(x) is a real-valued, even function. Thus we only need to evaluate
the integral for positive x. If we replace x by |x| in this expression we will have the solution that is valid for all x.
For x 0, we evaluate the integral by closing the path of integration in the upper half plane and using the Residue
Theorem and Jordans Lemma.
Z
a 1
y(x) = ex d
( +a)2 ( a)2
 
a 1 x
= 2 Res e , = a
( a)2 ( + a)2
ex
 
d
= 2a lim
a d ( + a)2
x ex 2 ex
 
= 2a lim
a ( + a)2 ( + a)3
x eax 2 eax
 
= 2a
4a2 8a3
ax
(1 + ax) e
=
2a2

The solution of the differential equation is

1
y(x) = (1 + a|x|) ea|x| .
2a2

1591
Solution 32.14
1. We take the Fourier cosine transform of the differential equation.

b
2 y() a2 y() = 0

b
y() =
( + a2 )
2

Now we take the inverse Fourier cosine transform. We use the fact that y() is an even function.
 
1 b
y(x) = Fc
( 2 + a2 )
 
1 b
=F
( 2 + a2 )
 
b 1 x
= 2 Res e , = a
2 + a2
 x 
e
= 2b lim , for x 0
a + a

b
y(x) = eax
a
2. The Green function problem is

G00 a2 G = (x ) on x, > 0, G0 (0; ) = 0, G(; ) = 0.

We take the Fourier cosine transform and solve for G(; ).

2 G a2 G = Fc [(x )]
1
G(; ) = 2 Fc [(x )]
+ a2

1592
We express the right side as a product of Fourier cosine transforms.

G(; ) = Fc [eax ]Fc [(x )]
a
Now we can apply the Fourier cosine convolution theorem.
Z
1 1 
Fc [Fc [f (x)]Fc [g(x)]] = f (t) g(|x t|) + g(x + t) dt
2 0
Z
1
(t ) ea|xt| + ea(x+t) dt

G(x; ) =
a 2 0
1 a|x|
+ ea(x+)

G(x; ) = e
2a
Solution 32.15
1. We take the Fourier sine transform of the differential equation.
b
2 y() + a2 y() = 0

b
y() =
( + a2 )
2

Now we take the inverse Fourier sine transform. We use the fact that y() is an odd function.
 
1 b
y(x) = Fs
( 2 + a2 )
 
1 b
= F
( 2 + a2 )
 
b x
= 2 Res e , = a
2 + a2
 x 
e
= 2b lim
a + a
ax
= be for x 0

1593
y(x) = b eax

2. Now we solve the differential equation with the Laplace transform.

y 00 a2 y = 0
s2 y(s) sy(0) y 0 (0) a2 y(s) = 0

We dont know the value of y 0 (0), so we treat it as an unknown constant.

bs + y 0 (0)
y(s) =
s 2 a2
y 0 (0)
y(x) = b cosh(ax) + sinh(ax)
a
In order to satisfy the boundary condition at infinity we must choose y 0 (0) = ab.

y(x) = b eax

We see that solving the differential equation with the Laplace transform is not as convenient, because the boundary
condition at infinity is not automatically satisfied. We had to find a value of y 0 (0) so that y() = 0.

3. The Green function problem is

G00 a2 G = (x ) on x, > 0, G(0; ) = 0, G(; ) = 0.

We take the Fourier sine transform and solve for G(; ).

2 G a2 G = Fs [(x )]
1
G(; ) = 2 Fs [(x )]
+ a2

1594
We write the right side as a product of Fourier cosine transforms and sine transforms.


G(; ) = Fc [eax ]Fs [(x )]
a

Now we can apply the Fourier sine convolution theorem.


Z
1
Fs1

[Fs [f (x)]Fc [g(x)]] = f (t) g(|x t|) g(x + t) dt
2 0
Z
1
(t ) ea|xt| ea(x+t) dt

G(x; ) =
a 2 0
1 a(x)
ea|x+|

G(x; ) = e
2a

Solution 32.16
1. We take the Fourier transform of the differential equation, solve for G and then invert.

G00 + 2G0 + 2 + 2 G = (x )


e
2 G + 2 G + 2 + 2 G =

2
e
G =
2 ( 2 2 2 )
2
Z
e ex
G= d
2( 2 2 2 2 )
Z
1 e(x)
G= d
2 ( + )( )

For x > we close the path of integration in the upper half plane and use the Residue theorem. There are two
simple poles in the upper half plane. For x < we close the path of integration in the lower half plane. Since

1595
the integrand is analytic there, the integral is zero. G(x; ) = 0 for x < . For x > we have

e(x)
 
1
G(x; ) = 2 Res , = +
2 ( + )( )
!
e(x)

+ Res , =
( + )( )

e(+)(x) e(+)(x)
 
G(x; ) = +
2 2
1
G(x; ) = e(x) sin((x )).

Thus the Green function is
1 (x)
G(x; ) = e sin((x ))H(x ).

2. We use the Green function to find the solution of the inhomogeneous equation.
y 00 + 2y 0 + 2 + 2 y = g(x), y() = y() = 0

Z
y(x) = g()G(x; ) d

Z
1
y(x) = g() e(x) sin((x ))H(x ) d

Z x
1
y(x) = g() e(x) sin((x )) d

We take the limit 0.
Z x
1
y= g() sin((x )) d

1596
Solution 32.17
First we consider the Fourier transform of f (x) = 1/(x2 + c2 ) where <(c) > 0.

 
1
f() = F
x + c2
2
Z
1 1
= ex dx
2 x + c2
2
Z
1 ex
= dx
2 (x c)(x + c)

If < 0 then we close the path of integration with a semi-circle in the upper half plane.

ex
 
1
f () = 2i Res , x = c
2 (x c)(x + c)
ec
= , for < 0
2c

Note that f (x) = 1/(x2 + c2 ) is an even function of x so that f() is an even function of . If f() = g() for < 0
then f () = g(||) for all . Thus
 
1 1 c||
F 2 2
= e .
x +c 2c

Now we consider the integral equation

Z
u() 1
2 2
d = 2 0 < a < b.
[(x ) + a ] x + b2

1597
We take the Fourier transform, utilizing the convolution theorem.

ea|| eb||
2u() =
2a 2b
(ba)||
ae
u() =
2b
a 1
u(x) = 2(b a) 2
2b x + (b a)2
a(b a)
u(x) =
b(x2 + (b a)2 )

Solution 32.18
1. Note that fc () is an even function. We compute the inverse Fourier cosine transform.

h i
f (x) = Fc1 fc ()
Z
= fc () ex d
Z

= fc ()(cos(x) + sin(x)) d
Z

= fc () cos(x) d

Z
=2 fc () cos(x) d
0

1598
2.
1 00
Z
00
Fc [y ] = y cos(x) dx
0
0
Z
1 0
= [y cos(x)]0 + y sin(x) dx
0
2
Z
1 0
= y (0) + [y sin(x)]0 y cos(x) dx
0
y 0 (0)
Fc [y 00 ] = 2 yc ()

3. We take the Fourier cosine transform of the differential equation.
b
2 y() a2 y() = 0

b
y() =
( 2 + a2 )
Now we take the inverse Fourier cosine transform. We use the fact that y() is an even function.
 
1 b
y(x) = Fc
( 2 + a2 )
 
1 b
=F
( 2 + a2 )
 
b 1 x
= 2 Res e , = a
2 + a2
 x 
e
= 2b lim , for x 0
a + a
b
y(x) = eax
a

1599
Solution 32.19
1. Suppose f (x) is an odd function. The Fourier transform of f (x) is
Z
1
F[f (x)] = f (x) ex dx
2
Z
1
= f (x)(cos(x) sin(x)) dx
2

Z
= f (x) sin(x) dx.
0

Note that f() = F[f (x)] is an odd function of . The inverse Fourier transform of f() is
Z
F 1
[f()] = f() ex d

Z
= 2 f() sin(x) d.
0

Thus we have that




Z Z  
f (x) = 2 f (x) sin(x) dx sin(x) d
0 0
Z  Z 
1
=2 f (x) sin(x) dx sin(x) d.
0 0

This gives us the Fourier sine transform pair


Z Z
1
f (x) = 2 fs () sin(x) d, fs () = f (x) sin(x) dx.
0 0

1600
2.
1 00
Z
00
Fs [y ] = y sin(x) dx
0
1h 0 i Z
= y sin(x) y 0 cos(x) dx
0 0
h i 2 Z
= y cos(x) y sin(x) dx
0 0

Fs [y 00 ] = 2 ys () + y(0)

3. We take the Fourier sine transform of the differential equation.
b
2 y() + a2 y() = 0

b
y() =
( + a2 )
2

Now we take the inverse Fourier sine transform. We use the fact that y() is an odd function.
 
1 b
y(x) = Fs
( 2 + a2 )
 
1 b
= F
( 2 + a2 )
 
b x
= 2 Res e , = a
2 + a2
 x 
e
= 2b lim
a + a
ax
= be for x 0

y(x) = b eax

1601
4. Now we solve the differential equation with the Laplace transform.
y 00 a2 y = 0
s2 y(s) sy(0) y 0 (0) a2 y(s) = 0

We dont know the value of y 0 (0), so we treat it as an unknown constant.

bs + y 0 (0)
y(s) =
s 2 a2
y 0 (0)
y(x) = b cosh(ax) + sinh(ax)
a
In order to satisfy the boundary condition at infinity we must choose y 0 (0) = ab.

y(x) = b eax

We see that solving the differential equation with the Laplace transform is not as convenient, because the boundary
condition at infinity is not automatically satisfied. We had to find a value of y 0 (0) so that y() = 0.

Solution 32.20
The Fourier, Fourier cosine and Fourier sine transforms are defined:
Z
1
F[f (x)] = f (x) ex dx,
2
1
Z
F[f (x)]c = f (x) cos(x) dx,
0
1
Z
F[f (x)]s = f (x) sin(x) dx.
0
We start with the right side of the identity and apply the usual tricks of integral calculus to reduce the expression to
the left side.
1
(Fc [f (x) + f (x)] Fs [f (x) f (x)])
2

1602
Z Z Z Z 
1
f (x) cos(x) dx + f (x) cos(x) dx f (x) sin(x) dx + f (x) sin(x) dx
2 0 0 0 0
Z Z Z Z 
1
f (x) cos(x) dx f (x) cos(x) dx f (x) sin(x) dx f (x) sin(x) dx
2 0 0 0 0
Z Z 0 Z Z 0 
1
f (x) cos(x) dx + f (x) cos(x) dx f (x) sin(x) dx f (x) sin(x) dx
2 0 0

Z Z 
1
f (x) cos(x) dx f (x) sin(x) dx
2
Z
1
f (x) ex dx
2
F[f (x)]

Solution 32.21
1
We take the Fourier transform of the integral equation, noting that the left side is the convolution of u(x) and x2 +a2
.

   
1 1
2u()F 2 =F 2
x + a2 x + b2

We find the Fourier transform of f (x) = 1


x2 +c2
. Note that since f (x) is an even, real-valued function, f() is an
even, real-valued function.
  Z
1 1 1
F 2 2
= ex dx
x +c 2 x + c2
2

1603
For x > 0 we close the path of integration in the upper half plane and apply Jordans Lemma to evaluate the integral
in terms of the residues.

ex
 
1
= 2 Res , x = c
2 (x c)(x + c)
ec
=
2c
1 c
= e
2c

Since f() is an even function, we have  


1 1 c||
F 2 2
= e .
x +c 2c
Our equation for u() becomes,
1 a|| 1 b||
2u() e = e
2a 2b
a (ba)||
u() = e .
2b
We take the inverse Fourier transform using the transform pair we derived above.

a 2(b a)
u(x) =
2b x + (b a)2
2

a(b a)
u(x) =
b(x2 + (b a)2 )

1604
Chapter 33

The Gamma Function

33.1 Eulers Formula


For non-negative, integral n the factorial function is

n! = n(n 1) (1), with 0! = 1.

We would like to extend the factorial function so it is defined for all complex numbers.
Consider the function (z) defined by Eulers formula
Z
(z) = et tz1 dt.
0

(Here we take the principal value of tz1 .) The integral converges for <(z) > 0. If <(z) 0 then the integrand will
be at least as singular as 1/t at t = 0 and thus the integral will diverge.

1605
Difference Equation. Using integration by parts,
Z
(z + 1) = et tz dt
0
h i Z
t z
= e t et ztz1 dt.
0 0

Since <(z) > 0 the first term vanishes.


Z
=z et tz1 dt
0
= z(z)

Thus (z) satisfies the difference equation


(z + 1) = z(z).
For general z it is not possible to express the integral in terms of elementary functions. However, we can evaluate
the integral for some z. The value z = 1 looks particularly simple to do.
Z h i
(1) = et dt = et = 1.
0 0

Using the difference equation we can find the value of (n) for any positive, integral n.

(1) = 1
(2) = 1
(3) = (2)(1) = 2
(4) = (3)(2)(1) = 6
=
(n + 1) = n!.

1606
Thus the Gamma function, (z), extends the factorial function to all complex z in the right half-plane. For non-
negative, integral n we have
(n + 1) = n!.

Analyticity. The derivative of (z) is


Z
0
(z) = et tz1 log t dt.
0

Since this integral converges for <(z) > 0, (z) is analytic in that domain.

33.2 Hankels Formula


We would like to find the analytic continuation of the Gamma function into the left half-plane. We accomplish this
with Hankels formula
Z
1
(z) = et tz1 dt.
2 sin(z) C

Here C is the contour starting at below the real axis, enclosing the origin and returning to above the real
axis. A graph of this contour is shown in Figure 33.1. Again we use the principle value of tz1 so there is a branch cut
on the negative real axis.
The integral in Hankels formula converges for all complex z. For non-positive, integral z the integral does not
vanish. Thus because of the sine term the Gamma function has simple poles at z = 0, 1, 2, . . .. For positive,
integral z, the integrand is entire and thus the integral vanishes. Using LHospitals rule you can show that the points,
z = 1, 2, 3, . . . are removable singularities and the Gamma function is analytic at these points. Since the only zeroes of
sin(z) occur for integral z, (z) is analytic in the entire plane except for the points, z = 0, 1, 2, . . ..

1607
Figure 33.1: The Hankel Contour.

Difference Equation. Using integration by parts we can derive the difference equation from Hankels formula.
Z
1
(z + 1) = et tz dt
2 sin((z + 1)) C
h i+0 Z 
1 t z t z1
= e t e zt dt
2 sin(z) 0 C
Z
1
= z et tz1 dt
2 sin(z) C
= z(z).

Evaluating (1),
R
et tz1 dt
C
(1) = lim .
z1 2 sin(z)

1608
Both the numerator and denominator vanish. Using LHospitals rule,
R t z1
e t log t dt
= lim C
z1 2 cos(z)
R t
e log t dt
= C
2
Let Cr be the circle of radius r starting at radians and going to radians.
Z r Z Z 
1 t t t
= e [log(t) i] dt + e log t dt + e [log(t) + i] dt
2 Cr r
Z Z Z 
1 t t t
= e [ log(t) + i] dt + e [log(t) + i] dt + e log t dt
2 r r Cr
Z Z 
1 t t
= e 2 dt + e log t dt
2 r Cr

The integral on Cr vanishes as r 0.


Z
1
= 2 et dt
2 0
= 1.
Thus we obtain the same value as with Eulers formula. It can be shown that Hankels formula is the analytic continuation
of the Gamma function into the left half-plane.

33.3 Gauss Formula


Gauss defined the Gamma function as an infinite product. This form is useful in deriving some of its properties. We
can obtain the product form from Eulers formula. First recall that
 n
t t
e = lim 1 .
n n

1609
Substituting this into Eulers formula,
Z
(z) = et tz1 dt
0
Z n n
t
= lim 1 tz1 dt.
n 0 n

With the substitution = t/n,


Z 1
= lim (1 )n nz1 z1 n d
n
0
Z 1
z
= lim n (1 )n z1 d.
n 0

Let n be an integer. Using integration by parts we can evaluate the integral.


1 1 Z 1
(1 )n z z
Z 
n z1
(1 ) d = n(1 )n1 d
0 z 0 0 z
Z 1
n
= (1 )n1 z d
z 0
n(n 1) 1
Z
= (1 )n2 z+1 d
z(z + 1) 0
Z 1
n(n 1) (1)
= z+n1 d
z(z + 1) (z + n 1) 0
 z+n 1
n(n 1) (1)
=
z(z + 1) (z + n 1) z + n 0
n!
=
z(z + 1) (z + n)

1610
Thus we have that
n!
(z) = lim nz
n z(z + 1) (z + n)
1 (1)(2) (n)
= lim nz
z n (z + 1)(z + 2) (z + n)
1 1
= lim nz
z n (1 + z)(1 + z/2) (1 + z/n)
1 1 2z 3z nz
= lim
z n (1 + z)(1 + z/2) (1 + z/n) 1z 2z (n 1)z

(n+1)z
Since limn nz
= 1 we can multiply by that factor.

1 1 2z 3z (n + 1)z
= lim
z n (1 + z)(1 + z/2) (1 + z/n) 1z 2z nz

(n + 1)z

1Y 1
=
z n=1
1 + z/n nz

Thus we have Gauss formula for the Gamma function


 z 
1Y 1  z 1
(z) = 1+ 1+ .
z n=1 n n

We derived this formula from Eulers formula which is valid only in the left half-plane. However, the product formula
is valid for all z except z = 0, 1, 2, . . ..

33.4 Weierstrass Formula

1611
The Euler-Mascheroni Constant. Before deriving Weierstrass product formula for the Gamma function we will
need to define the Euler-Mascheroni constant

  
1 1 1
= lim 1 + + + + log n = 0.5772 .
n 2 3 n

In deriving the Euler product formula, we had the equation

 
z n!
(z) = lim n .
n z(z + 1) (z + n)
 
1
 z 1  z 1  z 1 z
= lim z 1+ 1+ 1 + n
n 1 2 n
1 h  z z  z  z log n i
= lim z 1 + 1+ 1 + e
(z) n 1 2 n
    
z  z  z  z/2  z  z/n 1 1
= lim z 1 + e 1+ e 1 + e exp 1 + + + log n z
n 1 2 n 2 n

Weierstrass formula for the Gamma function is then

h
1 z
Y z  z/n i
=z e 1+ e .
(z) n=1
n

Since the product is uniformly convergent, 1/(z) is an entire function. Since 1/(z) has no singularities, we see
that (z) has no zeros.

1612
Result 33.4.1 Eulers formula for the Gamma function is valid for <(z) > 0.
Z
(z) = et tz1 dt
0

Hankels formula defines the (z) for the entire complex plane except for the points z =
0, 1, 2, . . .. Z
1
(z) = et tz1 dt
2 sin(z) C
Gauss and Weierstrass product formulas are, respectively
 z 
1Y 1  z 1
(z) = 1+ 1+ and
z n=1 n n
h
1 z
Y z  z/n i
= ze 1+ e .
(z) n=1
n

33.5 Stirlings Approximation


In this section we will try to get an approximation to the Gamma function for large positive argument. Eulers formula
is Z
(x) = et tx1 dt.
0

We could first try to approximate the integral by only looking at the domain where the integrand is large. In Figure 33.2
the integrand in the formula for (10), et t9 , is plotted.

1613
40000

30000

20000

10000

5 10 15 20 25 30

Figure 33.2: Plot of the integrand for (10)

We see that the important part of the integrand is the hump centered around x = 9. If we find where the
integrand of (x) has its maximum

d t x1 
e t =0
dx
et tx1 + (x 1) et tx2 = 0
(x 1) t = 0
t = x 1,

we see that the maximum varies with x. This could complicate our analysis. To take care of this problem we introduce

1614
the change of variables t = xs.
Z
(x) = exs (xs)x1 x ds
0
Z
=x x
exs sx s1 ds
Z0
= xx ex(slog s) s1 ds
0

The integrands, (ex(slog s) s1 ), for (5) and (20) are plotted in Figure 33.3.

0.007 210
-9
0.006
-9
0.005 1.510
0.004 -9
110
0.003
0.002 -10
510
0.001
1 2 3 4 1 2 3 4

Figure 33.3: Plot of the integrand for (5) and (20).

We see that the important part of the integrand is the hump that seems to be centered about s = 1. Also note
that the the hump becomes narrower with increasing x. This makes sense as the ex(slog s) term is the most rapidly
varying term. Instead of integrating from zero to infinity, we could get a good approximation to the integral by just
integrating over some small neighborhood centered at s = 1. Since s log s has a minimum at s = 1, ex(slog s)
has a maximum there. Because the important part of the integrand is the small area around s = 1, it makes sense to

1615
approximate s log s with its Taylor series about that point.
1
s log s = 1 + (s 1)2 + O (s 1)3
 
2
Since the hump becomes increasingly narrow with increasing x, we will approximate the 1/s term in the integrand with
its value at s = 1. Substituting these approximations into the integral, we obtain
Z 1+
2
(x) x x
ex(1+(s1) /2) ds
1
Z 1+
x x 2
=x e ex(s1) /2 ds
1

As x both of the integrals


Z 1 Z
x(s1)2 /2 2 /2
e ds and ex(s1) ds
1+

are exponentially small. Thus instead of integrating from 1  to 1 +  we can integrate from to .
Z
x x 2
(x) x e ex(s1) /2 ds
Z
2
= xx ex exs /2 ds
r
2
= xx ex
x

(x) 2xx1/2 ex as x .
This is known as Stirlings approximation to the Gamma function. In the table below, we see that the approximation
is pretty good even for relatively small argument.

1616

n (n) 2xx1/2 ex relative error
5 24 23.6038 0.0165
15 8.71783 1010 8.66954 1010 0.0055
25 6.20448 1023 6.18384 1023 0.0033
35 2.95233 1038 2.94531 1038 0.0024
45 2.65827 1054 2.65335 1054
0.0019
In deriving Stirlings approximation to the Gamma function we did a lot of hand waving. However, all of the steps can
be justified and better approximations can be obtained by using Laplaces method for finding the asymptotic behavior
of integrals.

1617
33.6 Exercises
Exercise 33.1
Given that Z
2
ex dx = ,

deduce the value of (1/2). Now find the value of (n + 1/2).

Exercise R33.2
3
Evaluate 0 ex dx in terms of the gamma function.

Exercise 33.3
Show that
Z
() + ()
ex sin(log x) dx = .
0 2

1618
33.7 Hints
Hint 33.1
Use the change of variables, = x2 in the integral. To find the value of (n + 1/2) use the difference relation.

Hint 33.2
Make the change of variable = x3 .

Hint 33.3

1619
33.8 Solutions
Solution 33.1

Z
2
ex dx =

Z
x2
e dx =
0 2

Make the change of variables = x2 .


Z
1 1/2

e d =
0 2 2

(1/2) =

Recall the difference relation for the Gamma function (z + 1) = z(z).

(n + 1/2) = (n 1/2)(n 1/2)


2n 1
= (n 1/2)
2
(2n 3)(2n 1)
= (n 3/2)
22
(1)(3)(5) (2n 1)
= (1/2)
2n

(1)(3)(5) (2n 1)
(n + 1/2) =
2n

1620
Solution 33.2
We make the change of variable = x3 , x = 1/3 , dx = 13 2/3 d.
Z Z
x3 1
e dx = e 2/3 d
0 0 3
 
1 1
=
3 3

Solution 33.3

Z Z
x 1 log x
ex e log x dx

e sin(log x) dx = e
0 0 2
Z
1
ex x x dx

=
2 0
1
= ((1 + ) (1 ))
2
1
= (() ()())
2
() + ()
=
2

1621
Chapter 34

Bessel Functions

Ideas are angels. Implementations are a bitch.

34.1 Bessels Equation


A commonly encountered differential equation in applied mathematics is Bessels equation

2
 
1 0
00
y + y + 1 2 y = 0.
z z

For our purposes, we will consider R0+ . This equation arises when solving certain partial differential equations
with the method of separation of variables in cylindrical coordinates. For this reason, the solutions of this equation are
sometimes called cylindrical functions.
This equation cannot be solved directly. However, we can find series representations of the solutions. There is
a regular singular point at z = 0, so the Frobenius method is applicable there. The point at infinity is an irregular
singularity, so we will look for asymptotic series about that point. Additionally, we will use Laplaces method to find
definite integral representations of the solutions.

1622
Note that Bessels equation depends only on 2 and not alone. Thus if we find a solution, (which of course
depends on this parameter), y (z) we know that y (z) is also a solution. For this reason, we will consider R0+ .
Whether or not y (z) and y (z) are linearly independent, (distinct solutions), remains to be seen.

Example 34.1.1 Consider the differential equation

1 2
y 00 + y 0 + 2 y = 0
z z
One solution is y (z) = z . Since the equation depends only on 2 , another solution is y (z) = z . For 6= 0, these
two solutions are linearly independent.
Now consider the differential equation
y 00 + 2 y = 0
One solution is y (z) = cos(z). Therefore, another solution is y (z) = cos(z) = cos(z). However, these two
solutions are not linearly independent.

34.2 Frobeneius Series Solution about z = 0


We note that z = 0 is a regular singular point, (the only singular point of Bessels equation in the finite complex plane.)
We will use the Frobenius method at that point to analyze the solutions. We assume that 0.
The indicial equation is

( 1) + 2 = 0
= .

If do not differ by an integer, (that is if is not a half-integer), then there will be two series solutions of the
Frobenius form.

X X
k
y1 (z) = z ak z , y2 (z) = z bk z k
k=0 k=0

1623
If is a half-integer, the second solution may or may not be in the Frobenius form. In any case, then will always be at
least one solution in the Frobenius form. We will determine that series solution. y(z) and it derivatives are

X
X
X
y= ak z k+ , y0 = (k + )ak z k+1 , y 00 = (k + )(k + 1)ak z k+2 .
k=0 k=0 k=0

We substitute the Frobenius series into the differential equation.

z 2 y 00 + zy 0 + z 2 2 y = 0


X X X
X
k+ k+ k++2
(k + )(k + 1)ak z + (k + )ak z + ak z 2 ak z k+ = 0
k=0 k=0 k=0 k=0

X
X
k 2 + 2k ak z k + ak2 z k = 0

k=0 k=2

We equate powers of z to obtain equations that determine the coefficients. The coefficient of z 0 is the equation
0 a0 = 0. This corroborates that a0 is arbitrary, (but non-zero). The coefficient of z 1 is the equation

(1 + 2)a1 = 0
a1 = 0

The coefficient of z k for k 2 gives us

k 2 + 2k ak + ak2 = 0.

ak2 ak2
ak = 2 =
k + 2k k(k + 2)

From the recurrence relation we see that all the odd coefficients are zero, a2k+1 = 0. The even coefficients are

a2k2 (1)k a0
a2k = = 2k
4k(k + ) 2 k!(k + + 1)

1624
Thus we have the series solution
X (1)k
y(z) = a0 z 2k .
k=0
22k k!(k + + 1)

a0 is arbitrary. We choose a0 = 2 . We call this solution the Bessel function of the first kind and order and denote
it with J (z).

X (1)k  z 2k+
J (z) =
k=0
k!(k + + 1) 2
Recall that the Gamma function is non-zero and finite for all real arguments except non-positive integers. (x)
has singularities at x = 0, 1, 2, . . .. Therefore, J (z) is well-defined when is not a positive integer. Since
J (z) z at z = 0, J (z) is clear linearly independent to J (z) for non-integer . In particular we note that there
are two solutions of the Frobenius form when is a half odd integer.

X (1)k  z 2k
J (z) = , for 6 Z+
k=0
k!(k + 1) 2

Of course for = 0, J (z) and J (z) are identical. Consider the case that = n is a positive integer. Since
(x) + as x 0, 1, 2, . . . we see the the coefficients in the series for Jnu (z) vanish for k = 0, . . . , n 1.

X (1)k  z 2kn
Jn (z) =
k=n
k!(k n + 1) 2

X (1)k+n  z 2k+n
Jn (z) =
k=0
(k + n)!(k + 1) 2

n
X (1)k  z 2k+n
Jn (z) = (1)
k=0
k!(k + n)! 2
Jn (z) = (1)n Jn (z)

Thus we see that Jn (z) and Jn (z) are not linearly independent for integer n.

1625
34.2.1 Behavior at Infinity
With the change of variables z = 1/, w(z) = u() Bessels equation becomes
4 u00 + 2 3 u0 + 2 u0 + 1 2 2 u = 0
 

2
 
00 1 0 1
u + u + u = 0.
4 2
The point = 0 and hence the point z = is an irregular singular point. We will find the leading order asymptotic
behavior of the solutions as z +.

Controlling Factor. We starti with Bessels equation for real argument.


2
 
00 1 0
y + y + 1 2 y =0
x x
We make the substitution y = es(x) .
1 2
s00 + (s0 )2 + s0 + 1 2 = 0
x x
2 00 0 2
We know that x2
 1 as x ; we will assume that s  (s ) as x .
1
(s0 )2 + s0 + 1 0 as x
x
To simplify the equation further, we will try the possible two-term balances.
1. (s0 )2 + x1 s0 0 s0 x1 This balance is not consistent as it violates the assumption that 1 is smaller
than the other terms.
2. (s0 )2 + 1 0 s0 This balance is consistent.
1 0
3. x
s +10 s0 x This balance is inconsistent as (s0 )2 isnt smaller than the other terms.
Thus the only dominant balance is s0 . This balance is consistent with our initial assumption that s00  (s0 )2 .
Thus s x and the controlling factor is ex .

1626
Leading Order Behavior. In order to find the leading order behavior, we substitute s = x+t(x) where t(x)  x
as x into the differential equation for s. We first consider the case s = x + t(x). We assume that t0  1 and
t00  1/x.

00 0 2 1 0 2
t + ( + t ) + ( + t ) + 1 2 = 0
x x
2
1
t00 + 2t0 + (t0 )2 + + t0 2 = 0
x x x

We use our assumptions about the behavior of t0 and t00 .



2t0 + 0
x
1
t0
2x
1
t ln x as x .
2
This asymptotic behavior is consistent with our assumptions.
Substituting s = x + t(x) will also yield t 12 ln x. Thus the leading order behavior of the solutions is
1
y c ex 2 ln x+u(x) = cx1/2 ex+u(x) as x ,

where u(x)  ln x as x .
By substituting t = 12 ln x+u(x) into the differential equation for t, you could show that u(x) const as x .
Thus the full leading order behavior of the solutions is

y cx1/2 ex+u(x) as x

where u(x) 0 as x . Writing this in terms of sines and cosines yields

y1 x1/2 cos(x + u1 (x)), y2 x1/2 sin(x + u2 (x)), as x ,

1627
where u1 , u2 0 as x .

Result 34.2.1 Bessels equation for real argument is


2
 
00 1 0
y + y + 1 2 y = 0.
x x
If is not an integer then the solutions behave as linear combinations of

y1 = x , and y2 = x

at x = 0. If is an integer, then the solutions behave as linear combinations of

y1 = x , and y2 = x + cx log x

at x = 0. The solutions are asymptotic to a linear combination of

y1 = x1/2 sin(x + u1 (x)), and y2 = x1/2 cos(x + u2 (x))

as x +, where u1 , u2 0 as x .

34.3 Bessel Functions of the First Kind


Consider the function exp( 12 z(t 1/t)). We can expand this function in a Laurent series in powers of t,


1
X
e 2 z(t1/t) = Jn (z)tn ,
n=

1628
where the coefficient functions Jn (z) are
I
1 1
Jn (z) = n1 e 2 z( 1/ ) d.
2
Here the path of integration is any positive closed path around the origin. exp( 21 z(t 1/t)) is the generating function
for Bessel function of the first kind.

34.3.1 The Bessel Function Satisfies Bessels Equation


We would like to expand Jn (z) in powers of z. The first step in doing this is to make the substitution = 2t/z.
I  n1   
1 2t 1 2t z 2
Jn (z) = exp z dt
2 z 2 z 2t z
I
1  z n 2
= tn1 etz /4t dt
2 2
We differentiate the expression for Jn (z).
1 nz n1
 
2z tz2 /4t
I I
0 n1 tz 2 /4t 1  z n n1
Jn (z) = t e dt + t e dt
2 2n 2 2 4t
I
1  z n  n z  n1 tz2 /4t
= t e dt
2 2 z 2t

I     
1  z n n n z n 1 z n z  n1 tz2 /4t
Jn00 (z) = + 2 t e dt
2 2 z z 2t z 2t 2t z 2t
I  2
z 2 n1 tz2 /4t

1  z n n nz n 1 nz
= + t e dt
2 2 z2 2zt z 2 2t 2zt 4t2
z 2 n1 tz2 /4t
I  
1  z n n(n 1) 2n + 1
= + 2 t e dt
2 2 z2 2t 4t

1629
We substitute Jn (z) into Bessels equation.

n2
 
00 1 0
Jn + J n + 1 2 Jn
z z
z2 n2
     
n(n 1) 2n + 1
I
1 z n
  n 1 n1 tz 2 /4t
= + + + 1 t e dt
2 2 z2 2t 4t2 z 2 2t z2
z 2 n1 tz2 /4t
I  
1  z n n+1
= 1 + 2 t e dt
2 2 t 4t
I
1  z n d  n1 tz2 /4t 
= t e dt
2 2 dt
2 /4t
Since tn1 etz is analytic in 0 < |t| < when n is an integer, the integral vanishes.

= 0.

Thus for integer n, Jn (z) satisfies Bessels equation.

Jn (z) is called the Bessel function of the first kind. The subscript is the order. Thus J1 (z) is a Bessel function
of order 1. J0 (x) and J1 (x) are plotted in the first graph in Figure 34.1. J5 (x) is plotted in the second graph in
Figure 34.1. Note that for non-negative, integer n, Jn (z) behaves as z n at z = 0.

34.3.2 Series Expansion of the Bessel Function


We expand exp(z 2 /4t) in the integral expression for Jn .
I
1  z n 2
Jn (z) = tn1 etz /4t dt
2 2

!
2 m

z
I
1  z n X 1
= tn1 et dt
2 2 m=0
4t m!

1630
1

0.8 0.3

0.6 0.2

0.4 0.1

0.2
5 10 15 20
2 4 6 8 10 12 14 -0.1
-0.2
-0.2
-0.4

Figure 34.1: Plots of J0 (x), J1 (x) and J5 (x).

For the path of integration, we are free to choose any contour that encloses the origin. Consider the circular path on
|t| = 1. Since the integral is uniformly convergent, we can interchange the order of integration and summation.


1  z n X (1)m z 2m
I
Jn (z) = tnm1 et dt
2 2 m=0 22m m!

1631
Let n be a non-negative integer.
dn+m z
I  
1 nm1 t 1
t e dt = lim (e )
2 z0 (n + m)! dz n+m
1
=
(n + m)!
We have the series expansion

X (1)m  z n+2m
Jn (z) = for n 0.
m=0
m!(n + m)! 2

Now consider Jn (z), (n positive).



1  z n X (1)m z 2m
I
Jn (z) = 2m m!
tnm1 et dt
2 2 m=1
2
For m n, the integrand has a pole of order m n + 1 at the origin.
(
1
for m n
I
1
tnm1 et dt = (mn)!
2 0 for m < n
The expression for Jn is then

X (1)m  z n+2m
Jn (z) =
m=n
m!(m n)! 2

X (1)m+n  z n+2m
=
m=0
(m + n)!m! 2
= (1)n Jn (z).
Thus we have that
Jn (z) = (1)n Jn (z) for integer n.

1632
34.3.3 Bessel Functions of Non-Integer Order
The generalization of the factorial function is the Gamma function. For integer values of n, n! = (n + 1). The Gamma
function is defined for all complex-valued arguments. Thus one would guess that if the Bessel function of the first kind
were defined for non-integer order, it would have the definition,

X (1)m  z +2m
J (z) = .
m=0
m!( + m + 1) 2

The Integrand for Non-Integer . Recall the definition of the Bessel function
I
1  z  2
J (z) = t1 etz /4t dt.
2 2
When is an integer, the integrand is single valued. Thus if you start at any point and follow any path around the
origin, the integrand will return to its original value. This property was the key to Jn satisfying Bessels equation. If
is not an integer, then this property does not hold for arbitrary paths around the origin.

A New Contour. First, since the integrand is multiple-valued, we need to define what branch of the function we
are talking about. We will take the principal value of the integrand and introduce a branch cut on the negative real
axis. Let C be a contour that starts at z = below the branch cut, circles the origin, and returns to the point
z = above the branch cut. This contour is shown in Figure 34.2.
Thus we define I
1  z  2
J (z) = t1 etz /4t dt.
2 2 C

Bessels Equation. Substituting J (z) into Bessels equation yields


2
  I
00 1 0 1  z  d  1 tz2 /4t 
J + J + 1 2 J = t e dt.
z z 2 2 C dt
2
Since t1 etz /4t is analytic in 0 < |z| < and | arg(z)| < , and it vanishes at z = , the integral is zero.
Thus the Bessel function of the first kind satisfies Bessels equation for all complex orders.

1633
Figure 34.2: The Contour of Integration.

Series Expansion. Because of the et factor in the integrand, the integral defining J converges uniformly. Expanding
2
ez /4t in a Taylor series yields


1  z  X (1)m z 2m
I
J (z) = tm1 et dt
2 2 m=0 22m m! C

Since
I
1 1
= t1 et dt,
() 2 C

we have the series expansion of the Bessel function


X (1)m  z +2m
J (z) = .
m=0
m!( + m + 1) 2

1634
Linear Independence. We use Abels formula to compute the Wronskian of Bessels equation.
 Z z 
1 1
W (z) = exp d = e log z =
z

Thus to within a function of , the Wronskian of any two solutions is 1/z. For any given , there are two linearly
independent solutions. Note that Bessels equation is unchanged under the transformation . Thus both J and
J satisfy Bessels equation. Now we must determine if they are linearly independent. We have already shown that
for integer values of they are not independent. (Jn = (1)n Jn .) Assume that is not an integer. We compute the
Wronskian of J and J .

J J
W [J , J ] = 0 0

J J
0
= J J J J0

We substitute in the expansion for J


!

!
X (1)m  z +2m X (1)n ( + 2n)  z +2n1
=
m=0
m!( + m + 1) 2 n=0
n!( + n + 1)2 2

!
!
X (1)m  z +2m X (1)n ( + 2n)  z +2n1

m=0
m!( + m + 1) 2 n=0
n!( + n + 1)2 2

Since the Wronskian is a function of times 1/z the coefficients of all of the powers of z except 1/z must vanish.


=
z( + 1)( + 1) z( + 1)( + 1)
2
=
z()(1 )

1635
Using an identity for the Gamma function simplifies this expression.

2
= sin()
z
Since the Wronskian is nonzero for non-integer , J and J are independent functions when is not an integer. In
this case, the general solution of Bessels equation is aJ + bJ .

34.3.4 Recursion Formulas


In showing that J satisfies Bessels equation for arbitrary complex , we obtained
I
d  tz2 /4t 
t e dt = 0.
C dt

Expanding the integral,

z 2 2
I  
1 2
t + t t etz /4t dt = 0.
C 4
z 2 2
I  
1 z
 
1 2
t + t t etz /4t dt = 0.
2 2 C 4

2 /4t
1
t1 etz
H
Since J (z) = 2
(z/2) C
dt,
"  #
1   2
2 2 z
J1 + J+1 J = 0.
z z 4
2
J1 + J+1 = J
z

1636
Differentiating the integral expression for J ,

1 z 1
I I
1 tz 2 /4t 1  z   z 2
J0 (z) =
t e dt + t1
etz /4t dt
2 2 C I 2 2 C 2t
I
1 z   2 1 z  +1 2
J0 (z) = t1 etz /4t dt t2 etz /4t dt
z 2 2 C 2 2 C

J0 = J J+1
z

From the two relations we have derived you can show that

1
J0 = (J1 + J+1 ) and J0 = J1 J .
2 z

1637
Result 34.3.1 The Bessel function of the first kind, J (z), is defined,
I
1  z  2
J (z) = t1 etz /4t dt.
2 2 C

The Bessel function has the expansion,



X (1)m  z +2m
J (z) = .
m=0
m!( + m + 1) 2

The asymptotic behavior for large argument is


r  
2  |=(z)| 1

J (z) cos z +e O |z| as |z| , | arg(z)| < .
z 2 4
The Wronskian of J (z) and J (z) is
2
W (z) = sin().
z
Thus J (z) and J (z) are independent when is not an integer. The Bessel functions satisfy
the recursion relations,
2
J1 + J+1 = J J0 = J J+1
z z
1
J0 = (J1 J+1 ) J0 = J1 J .
2 z

1638
34.3.5 Bessel Functions of Half-Integer Order
Consider J1/2 (z). Start with the series expansion


X (1)m  z 1/2+2m
J1/2 (z) = .
m=0
m!(1/2 + m + 1) 2

(1)(3)(2n1)
Use the identity (n + 1/2) = 2n
.


X (1)m 2m+1  z 1/2+2m
=
m=0
m!(1)(3) (2m + 1) 2
 1/2+m
X (1)m 2m+1 1
= z 1/2+2m
m=0
(2)(4) (2m) (1)(3) (2m + 1) 2
 1/2 X
2 (1)m 2m+1
= z
z m=0
(2m + 1)!

We recognize the sum as the Taylor series expansion of sin z.

 1/2
2
= sin z
z

Using the recurrence relations,


J+1 = J J0 and J1 = J + J0 ,
z z

we can find Jn+1/2 for any integer n.

1639
Example 34.3.1 To find J3/2 (z),
1/2 0
J3/2 (z) = J1/2 (z) J1/2 (z)
z
 1/2    1/2  1/2
1/2 2 1/2 1 2 3/2 2
= z sin z z sin z z 1/2 cos z
z 2
= 21/2 1/2 z 3/2 sin z + 21/2 1/2 z 3/2 sin z 21/2 1/2 cos z
 1/2  1/2
2 3/2 2
= z sin z z 1/2 cos z

 1/2
2
z 3/2 sin z z 1/2 cos z .

=

You can show that  1/2
2
J1/2 (z) = cos z.
z

Note that at a first glance it appears that J3/2 z 1/2 as z 0. However, if you expand the sine and cosine you
will see that the z 1/2 and z 1/2 terms vanish and thus J3/2 (z) z 3/2 as z 0 as we showed previously.
Recall that we showed the asymptotic behavior as x + of Bessel functions to be linear combinations of
x1/2 sin(x + U1 (x)) and x1/2 cos(x + U2 (x))
where U1 , U2 0 as x +.

34.4 Neumann Expansions


Consider expanding an analytic function in a series of Bessel functions of the form

X
f (z) = an Jn (z).
n=0

1640
If f (z) is analytic in the disk |z| r then we can write
I
1 f ()
f (z) = d,
2 z
1
where the path of integration is || = r and |z| < r. If we were able to expand the function z in a series of Bessel
functions, then we could interchange the order of summation and integration to get a Bessel series expansion of f (z).

1
The Expansion of 1/( z). Assume that z
has the uniformly convergent expansion

1 X
= c0 ()J0 (z) + 2 cn ()Jn (z),
z n=1

where each cn () is analytic. Note that


 
1 1 1
+ = 2
+ = 0.
z z ( z) ( z)2

Thus we have
 "
#
X
+ c0 ()J0 (z) + 2 cn ()Jn (z) = 0
z n=1
"
# "
#
X X
c00 J0 + 2 c0n Jn + c0 J00 + 2 cn Jn0 = 0.
n=1 n=1

Using the identity 2Jn0 = Jn1 Jn+1 ,


"
# "
#
X X
c00 J0 + 2 c0n Jn + c0 (J1 ) + cn (Jn1 Jn+1 ) = 0.
n=1 n=1

1641
Collecting coefficients of Jn ,

X
(c00 + c1 )J0 + (2c0n + cn+1 cn1 )Jn = 0.
n=1

Equating the coefficients of Jn , we see that the cn are given by the relations,

c1 = c00 , and cn+1 = cn1 2c0n .

We can evaluate c0 (). Setting z = 0,



1 X
= c0 ()J0 (0) + 2 cn ()Jn (0)
n=1
1
= c0 ().

Using the recurrence relations we can calculate the cn s. The first few are:
1 1
c1 = 2
= 2

1 2 1 4
c2 = 2 3 = + 3

 
1 1 12 3 24
c3 = 2 2 2
4 = 2 + 4.

We see that cn is a polynomial of degree n + 1 in 1/. One can show that


 
2n1 n! 2 4 n
n+1
1 + 2(2n2) + 24(2n2)(2n4) + + 24n(2n2)(2nn) for even n
cn () =  
2 n+1n! 1 + 2 +
n1 4
+ + n1
for odd n
2(2n2) 24(2n2)(2n4) 24(n1)(2n2)(2n(n1))

1642
1
Uniform Convergence of the Series. We assumed before that the series expansion of z
is uniformly convergent.
The behavior of cn and Jn are

2n1 n! zn
cn () = + O( n ), Jn (z) = + O(z n+1 ).
n+1 2n n!

This gives us
 n  n+1 !
1 z 1 z
cn ()Jn (z) = +O .
2

If z = < 1 we can bound the series with the geometric series
P n
. Thus the series is uniformly convergent.

Neumann Expansion of an Analytic Function. Let f (z) be a function that is analytic in the disk |z| r.
Consider |z| < r and the path of integration along || = r. Cauchys integral formula tells us that
I
1 f ()
f (z) = d.
2 z

1
Substituting the expansion for z
,


I !
1 X
= f () co ()J0 (z) + 2 cn ()Jn (z) d
2 n=1
I I
1 f () X Jn (z)
= J0 (z) d + cn ()f () d
2 n=1

I
X Jn (z)
= J0 (z)f (0) + cn ()f () d.
n=1

1643
Result 34.4.1 let f (z) be analytic in the disk, |z| r. Consider |z| < r and the path of
integration along || = r. f (z) has the Bessel function series expansion
I
X Jn (z)
f (z) = J0 (z)f (0) + cn ()f () d,
n=1

where the cn satisfy



1 X
= c0 ()J0 (z) + 2 cn ()Jn (z).
z n=1

34.5 Bessel Functions of the Second Kind

When is an integer, J and J are not linearly independent. In order to find an second linearly independent solution,
we define the Bessel function of the second kind, (also called Webers function),

(J
(z) cos()J (z)
sin()
when is not an integer
Y =
lim J (z) cos()J
sin()
(z)
when is an integer.

J and Y are linearly independent for all .


In Figure 34.3 Y0 and Y1 are plotted in solid and dashed lines, respectively.

1644
0.5

0.25

5 10 15 20
-0.25

-0.5

-0.75

-1

Figure 34.3: Bessel Functions of the Second Kind

Result 34.5.1 The Bessel function of the second kind, Y (z), is defined,
( J (z) cos()J (z)

sin() when is not an integer
Y = J (z) cos()J (z)
lim sin() when is an integer.

The Wronskian of J (z) and Y (z) is


2
W [J , Y ] = .
z
Thus J (z) and Y (z) are independent for all . The Bessel functions of the second kind
satisfy the recursion relations,
1645
2
34.6 Hankel Functions
Another set of solutions to Bessels equation is the Hankel functions,

H(1) (z) = J (z) + Y (z),


H(2) (z) = J (z) Y (z)

Result 34.6.1 The Hankel functions are defined

H(1) (z) = J (z) + Y (z),


H(2) (z) = J (z) Y (z)
(1) (2)
The Wronskian of H (z) and H (z) is
4
W [H(1) , H(2) ] = .
z
The Hankel functions are independent for all . The Hankel functions satisfy the same
recurrence relations as the other Bessel functions.

34.7 The Modified Bessel Equation


The modified Bessel equation is
2
 
1 0
00
w + w 1 + 2 w = 0.
z z

1646
This equation is identical to the Bessel equation except for a sign change in the last term. If we make the change of
variables = z, u() = w(z) we obtain the equation

2
 
00 1 0
u u 1 2 u = 0

2
 
00 1 0
u + u + 1 2 u = 0.

This is the Bessel equation. Thus J (z) is a solution to the modified Bessel equation. This motivates us to define the
modified Bessel function of the first kind
I (z) = J (z).
Since J and J are linearly independent solutions when is not an integer, I and I are linearly independent
solutions to the modified Bessel equation when is not an integer.
The Taylor series expansion of I (z) about z = 0 is

I (z) = J (z)


X (1)m  z +2m
=
m=0
m!( + m + 1) 2

X (1)m 2m  z +2m
=
m=0
m!( + m + 1) 2

X 1  z +2m
=
m=0
m!( + m + 1) 2

Modified Bessel Functions of the Second Kind. In order to have a second linearly independent solution when
is an integer, we define the modified Bessel function of the second kind
( I I

when is not an integer,
K (z) = 2 sin() I I
lim 2 sin() when is an integer.

1647
10

1 2 3 4

Figure 34.4: Modified Bessel Functions

I and K are linearly independent for all . In Figure 34.4 I0 and K0 are plotted in solid and dashed lines, respectively.

1648
Result 34.7.1 The modified Bessel functions of the first and second kind, I (z) and K (z),
are defined,
I (z) = J (z).
( I I

when is not an integer,
K (z) = 2 sin() I I
lim 2 sin() when is an integer.
The modified Bessel function of the first kind has the expansion,

X 1  z +2m
I (z) =
m=0
m!( + m + 1) 2

The Wronskian of I (z) and I (z) is


2
W [I , I ] = sin().
z
I (z) and I (z) are linearly independent when is not an integer. The Wronskian of I (z)
and K (z) is
1
W [I , K ] = .
z
I (z) and K (z) are independent for all . The modified Bessel functions satisfy the recursion
relations,
2
A1 A+1 = A A0 = A+1 + A
z z
1
A0 = (A1 + A+1 ) A0 = A1 A .
2 z
where A stands for either I or K.
1649
34.8 Exercises
Exercise 34.1
Consider Bessels equation
z 2 y 00 (z) + zy 0 (z) + z 2 2 y = 0


where 0. Find the Frobenius series solution that is asymptotic to t as t 0. By multiplying this solution by a
constant, define the solution

X (1)k  z 2k+
J (z) = .
k=1
k!(k + + 1) 2
This is called the Bessel function of the first kind and order . Clearly J (z) is defined and is linearly independent to
J (z) if is not an integer. What happens when is an integer?

Exercise 34.2
Consider Bessels equation for integer n,

z 2 y 00 + zy 0 + z 2 n2 y = 0.


Using the kernel


1 1
K(z, t) = e 2 z(t t ) ,
find two solutions of Bessels equation. (For n = 0 you will find only one solution.) Are the two solutions linearly
independent? Define the Bessel function of the first kind and order n,
I
1 1
Jn (z) = tn1 e 2 z(t1/t) dt,
2 C
where C is a simple, closed contour about the origin. Verify that

1
X
e 2 z(t1/t) = Jn (z)tn .
n=

This is the generating function for the Bessel functions.

1650
Exercise 34.3
Use the generating function

1
X
z(t1/t)
e 2 = Jn (z)tn
n=

to show that Jn satisfies Bessels equation

z 2 y 00 + zy 0 + z 2 n2 y = 0.


Exercise 34.4
Using
2n n
Jn1 + Jn+1 = Jn and Jn0 = Jn Jn+1 ,
z z
show that
1 n
Jn0 = (Jn1 Jn+1 ) and Jn0 = Jn1 Jn .
2 z
Exercise 34.5
Find the general solution of  
001 0 1
w + w + 1 2 w = z.
z 4z

Exercise 34.6
Show that J (z) and Y (z) are linearly independent for all .

Exercise 34.7
Compute W [I , I ] and W [I , K ].

Exercise 34.8
Using the generating function,
   +
z 1 X
exp t = Jn (z)tn ,
2 t n=

1651
verify the following identities:
1.
2n
Jn (z) = Jn1 (z) + Jn+1 (z).
z
This relation is useful for recursively computing the values of the higher order Bessel functions.
2.
1
Jn0 (z) = (Jn1 Jn+1 ) .
2
This relation is useful for computing the derivatives of the Bessel functions once you have the values of the Bessel
functions of adjacent order.
3.
d n
z Jn (z) = z n Jn+1 (z).

dz
Exercise 34.9
Use the Wronskian of J (z) and J (z),
2 sin
W [J (z), J (z)] = ,
z
to derive the identity
2
J+1 (z)J (z) + J (z)J1 (z) = sin .
z
Exercise 34.10
Show that, using the generating function or otherwise,
J0 (z) + 2J2 (z) + 2J4 (z) + 2J6 (z) + = 1
J0 (z) 2J2 (z) + 2J4 (z) 2J6 (z) + = cos z
2J1 (z) 2J3 (z) + 2J5 (z) = sin z
J0 (z) + 2J12 (z) + 2J22 (z) + 2J32 (z) + = 1
2

1652
Exercise 34.11
It is often possible to solve certain ordinary differential equations by converting them into the Bessel equation by
means of various transformations. For example, show that the solution of

y 00 + xp2 y = 0,

can be written in terms of Bessel functions.


   
1/2 2 p/2 1/2 2 p/2
y(x) = c1 x J1/p x + c2 x Y1/p x
p p

Here c1 and c2 are arbitrary constants. Thus show that the Airy equation,

y 00 + xy = 0,

can be solved in terms of Bessel functions.


Exercise 34.12
The spherical Bessel functions are defined by

r

jn (z) = Jn+1/2 (z),
2z
r

yn (z) = Yn+1/2 (z),
2z
r

kn (z) = Kn+1/2 (z),
2z
r

in (z) = In+1/2 (z).
2z

1653
Show that
sin z cos z
j1 (z) = ,
z2 z
sinh z
i0 (z) = ,
z

k0 (z) = exp(z).
2z
Exercise 34.13
Show that as x ,
ex 4n2 1 (4n2 1)(4n2 9)
 
Kn (x) 1+ + + .
x 8x 128x2

1654
34.9 Hints
Hint 34.2

Hint 34.3

Hint 34.4
Use the generating function

1
X
z(t1/t)
e 2 = Jn (z)tn
n=

to show that Jn satisfies Bessels equation


z 2 y 00 + zy 0 + z 2 n2 y = 0.


Hint 34.6
Use variation of parameters and the Wronskian that was derived in the text.

Hint 34.7
Compute the Wronskian of J (z) and Y (z). Use the relation
2
W [J , J ] = sin()
z
Hint 34.8
Derive W [I , I ] from the value of W [J , J ]. Derive W [I , K ] from the value of W [I , I ].

Hint 34.9

Hint 34.10

1655
Hint 34.11

Hint 34.12

Hint 34.13

Hint 34.14

1656
34.10 Solutions
Solution 34.1
Bessels equation is
L[y] z 2 y 00 + zy 0 + z 2 n2 y = 0.


We consider a solution of the form Z


1
y(z) = e 2 z(t1/t) v(t) dt.
C

We substitute the form of the solution into Bessels equation.


Z h 1 i
L e 2 z(t1/t) v(t) dt = 0
C
Z  2  2 !
1 1 1 1 1
z2 + z 2 n2 e 2 z(t1/t) v(t) dt = 0

t+ +z t (34.1)
C 4 t 2 t

By considering
   
d 1 z(t1/t) 1 1 1
te 2 = x t+ + 1 e 2 z(t1/t)
dt 2 t
2 !
d2 2 1 z(t1/t)
  
1 2 1 1 1
t e2 = x t+ + x 2t + + 2 e 2 z(t1/t)
dt2 4 t t

we see that
d2 2
 
h 1
z(t1/t)
i d 2
 1 z(t1/t)
L e 2 = t 3 t+ 1n e2 .
dt2 dt
Thus Equation 34.1 becomes

d2 2 1 z(t1/t)
Z  
d 1 z(t1/t) 2 1
z(t1/t)
t e2 3 t e 2 +(1 n ) e 2 v(t) dt = 0
C dt2 dt

1657
We apply integration by parts to move derivatives from the kernel to v(t).
h 1 i h 1 i h i Z
1 1
0
2 2 z(t1/t) z(t1/t) z(t1/t)
e 2 z(t1/t) t2 v 00 (t) + 3tv(t) + 1 n2 v(t) dt =
 
t e v(t) t e 2 v (t) + 3t e 2 v(t) +
C C C C
h 1 i Z
1
e 2 z(t1/t) (t2 3t)v(t) tv 0 (t) e 2 z(t1/t) t2 v 00 (t) + 3tv(t) + (1 n2 )v(t) dt = 0
 
+
C C

In order that the integral vanish, v(t) must be a solution of the differential equation
t2 v 00 + 3tv + 1 n2 v = 0.


This is an Euler equation with the solutions {tn1 , tn1 } for non-zero n and {t1 , t1 log t} for n = 0.
Consider the case of non-zero n. Since
1
e 2 z(t1/t) t2 3t v(t) tv 0 (t)
 

is single-valued and analytic for t 6= 0 for the functions v(t) = tn1 and v(t) = tn1 , the boundary term will vanish if
C is any closed contour that that does not pass through the origin. Note that the integrand in our solution,
1
e 2 z(t1/t) v(t),
is analytic and single-valued except at the origin and infinity where it has essential singularities. Consider a simple
closed contour that does not enclose the origin. The integral along such a path would vanish and give us y(z) = 0.
This is not an interesting solution. Since
1
e 2 z(t1/t) v(t),
has non-zero residues for v(t) = tn1 and v(t) = tn1 , choosing any simple, positive, closed contour about the origin
will give us a non-trivial solution of Bessels equation. These solutions are
Z Z
n1 12 z(t1/t) 1
y1 (t) = t e dt, y2 (t) = tn1 e 2 z(t1/t) dt.
C C

Now consider the case n = 0. The two solutions above concide and we have the solution
Z
1
y(t) = t1 e 2 z(t1/t) dt.
C

1658
Choosing v(t) = t1 log t would make both the boundary terms and the integrand multi-valued. We do not pursue the
possibility of a solution of this form.
The solution y1 (t) and y2 (t) are not linearly independent. To demonstrate this we make the change of variables
t 1/t in the integral representation of y1 (t).
Z
1
y1 (t) = tn1 e 2 z(t1/t) dt
ZC
1 1
= (1/t)n1 e 2 z(1/t+t) dt
t2
ZC
1
= (1)n tn1 e 2 z(t1/t) dt
C
= (1)n y2 (t)

Thus we see that a solution of Bessels equation for integer n is


Z
1
y(t) = tn1 e 2 z(t1/t) dt
C

where C is any simple, closed contour about the origin.


Therefore, the Bessel function of the first kind and order n,
I
1 1
Jn (z) = tn1 e 2 z(t1/t) dt
2 C
1
is a solution of Bessels equation for integer n. Note that Jn (z) is the coefficient of tn in the Laurent series of e 2 z(t1/t) .
This establishes the generating function for the Bessel functions.

1
X
z(t1/t)
e 2 = Jn (z)tn
n=

1659
Solution 34.2
The generating function is

z
X
(t1/t)
e 2 = Jn (z)tn .
n=

In order to show that Jn satisfies Bessels equation we seek to show that



X
z 2 Jn00 (z) + zJn (z) + (z 2 n2 )Jn (z) tn = 0.

n=

To get the appropriate terms in the sum we will differentiate the generating function with respect to z and t. First we
differentiate it with respect to z.
 
1 1 z
X
t e 2
(t1/t)
= Jn0 (z)tn
2 t n=
 2
1 1 z
X
t e 2
(t1/t)
= Jn00 (z)tn
4 t n=

Now we differentiate with respect to t and multiply by t get the n2 Jn term.


 
z 1 z
(t1/t)
X
1+ 2 e 2 = nJn (z)tn1
2 t n=
 
z 1 z
X
t+ e 2 (t1/t) = nJn (z)tn
2 t n=
  2
 2
z 1 z
(t1/t) z 1 z
(t1/t)
X
1 2 e2 + t+ e2 = n2 Jn (z)tn1
2 t 4 t n=
  2
 2
z 1 z
(t1/t) z 1 z
(t1/t)
X
t e2 + t+ e2 = n2 Jn (z)tn
2 t 4 t n=

1660
Now we can evaluate the desired sum.

X
z 2 Jn00 (z) + zJn (z) + z 2 n2 Jn (z) tn
 
n=
2 2 !
z2 z2
     
1 z 1 z 1 1 z
= t + t + z2 t t+ e 2 (t1/t)
4 t 2 t 2 t 4 t


X
z 2 Jn00 (z) + zJn (z) + z 2 n2 Jn (z) tn = 0
 
n=

z 2 Jn00 (z) + zJn (z) + z 2 n2 Jn (z) = 0




Thus Jn satisfies Bessels equation.


Solution 34.3

n
Jn0 = Jn Jn+1
z
1
= (Jn1 + Jn+1 ) Jn+1
2
1
= (Jn1 Jn+1 )
2

n
Jn0 = Jn Jn+1
z  
n 2n
= Jn Jn Jn1
z z
n
= Jn1 Jn
z

1661
Solution 34.4
The linearly independent homogeneous solutions are J1/2 and J1/2 . The Wronskian is

2 2
W [J1/2 , J1/2 ] = sin(/2) = .
z z

Using variation of parameters, a particular solution is

z Z z
J1/2 () J1/2 ()
Z
yp = J1/2 (z) d + J1/2 (z) d
2/ 2/
Z z Z z
2
= J1/2 (z) J1/2 () d J1/2 (z) 2 J1/2 () d.
2 2

Thus the general solution is

Z z Z z
2
y = c1 J1/2 (z) + c2 J1/2 (z) + J1/2 (z) J1/2 () d J1/2 (z) 2 J1/2 () d.
2 2

We could substitute
 1/2  1/2
2 2
J1/2 (z) = sin z and J1/2 = cos z
z z

into the solution, but we cannot evaluate the integrals in terms of elementary functions. (You can write the solution in
terms of Fresnel integrals.)

1662
Solution 34.5


J J cot() J csc()
W [J , Y ] = 0
J J0 cot() J 0
csc()

J J J J
= cot() 0
csc() 0

J J0 0
J J
2
= csc() sin()
z
2
=
z

Since the Wronskian does not vanish identically, the functions are independent for all values of .

Solution 34.6

I (z) = J (z)


I I
W [I , I ] = 0
0
I I

J (z) J (z)
= 0
0

J (z) J (z)

J (z) J (z)
= 0
0

J (z) J (z)
2
= sin()
z
2
= sin()
z

1663

I csc()(I I )
W [I , K ] = 0 2 0

I 2 csc()(I I0 )
 
I I I I
= csc() 0 0 0

2 I I I I0
2
= csc() sin()
2 z
1
=
z

Solution 34.7
1. We diferentiate the generating function with respect to t.


z
X
(t1/t)
e 2 = Jn (z)tn
n=
 
z 1 z
(t1/t)
X
1+ 2 e 2 = Jn (z)ntn1
2 t n=
  X
1 n 2 X
1+ 2 Jn (z)t = Jn (z)ntn1
t n= z n=

X
n
X
n2 2 X
Jn (z)t + Jn (z)t = Jn (z)ntn1
n= n=
z n=

X X 2 X
Jn1 (z)tn1 + Jn+1 (z)tn1 = Jn (z)ntn1
n= n=
z n=
2
Jn1 (z) + Jn+1 (z) = Jn (z)n
z
2n
Jn (z) = Jn1 (z) + Jn+1 (z)
z

1664
2. We diferentiate the generating function with respect to z.

z
X
(t1/t)
e 2 = Jn (z)tn
n=
 
1 1 z
X
t e 2
(t1/t)
= Jn0 (z)tn
2 t n=
  X
1 1 X
t Jn (z)tn = Jn0 (z)tn
2 t n= n=

!
1 X X X
Jn (z)tn+1
Jn (z)tn1
= Jn0 (z)tn
2 n= n= n=

!
1 X X X
Jn1 (z)t n
Jn+1 (z)t n
= Jn0 (z)tn
2 n= n= n=
1
(Jn1 (z) Jn+1 (z)) = Jn0 (z)
2
1
Jn0 (z) = (Jn1 Jn+1 )
2

3.
d n
z Jn (z) = nz n1 Jn (z) + z n Jn0 (z)

dz
1 2n 1
= z n Jn (z) + z n (Jn1 (z) Jn+1 (z))
2 z 2
1 n 1
= z (Jn+1 (z) + Jn1 (z)) + z n (Jn1 (z) Jn+1 (z))
2 2
d n
z Jn (z) = z n Jn+1 (z)

dz

1665
Solution 34.8
For this part we will use the identities

J0 (z) = J (z) J+1 (z), J0 (z) = J1 (z) J (z).
z z

= 2 sin()
J (z) J (z)
0 0
J (z) J (z) z

J (z) J (z) 2 sin()
J1 (z) J J (z) J+1 (z) = z


z z

J (z) J (z) J (z) J (z) 2 sin()
J1 (z) J+1 (z) z J (z) J (z) = z

2 sin()
J+1 (z)J (z) J (z)J1 (z) =
z
2
J+1 (z)J (z) + J (z)J1 (z) = sin
z
Solution 34.9
The generating function for the Bessel functions is

1
X
z(t1/t)
e 2 = Jn (z)tn . (34.2)
n=

1. We substitute t = 1 into the generating function.



X
Jn (z) = 1
n=

X
X
J0 (z) + Jn (z) + Jn (z) = 1
n=1 n=1

1666
We use the identity Jn = (1)n Jn .

X
J0 (z) + (1 + (1)n ) Jn (z) = 1
n=1

X
J0 (z) + 2 Jn (z) = 1
n=2
even n


X
J0 (z) + 2 J2n (z) = 1
n=1

2. We substitute t = into the generating function.



X
Jn (z)n = ez
n=

X
X
J0 (z) + n
Jn (z) + Jn (z)n = ez
n=1 n=1

X
X
J0 (z) + Jn (z)n + (1)n Jn (z)()n = ez
n=1 n=1


X
J0 (z) + 2 Jn (z)n = ez (34.3)
n=1

Next we substitute t = into the generating function.



X
J0 (z) + 2 (1)n Jn (z)n = ez (34.4)
n=1

1667
Dividing the sum of Equation 34.3 and Equation 34.4 by 2 gives us the desired identity.


X
J0 (z) + (1 + (1)n ) Jn (z)n = cos z
n=1

X
J0 (z) + 2 Jn (z)n = cos z
n=2
even n

X
J0 (z) + 2 (1)n/2 Jn (z) = cos z
n=2
even n

X
J0 (z) + 2 (1)n J2n (z) = cos z
n=1

3. Dividing the difference of Equation 34.3 and Equation 34.4 by 2 gives us the other identity.


X
(1 (1)n ) Jn (z)n = sin z
n=1

X
2 Jn (z)n1 = sin z
n=1
odd n

X
2 (1)(n1)/2 Jn (z) = sin z
n=1
odd n

X
2 (1)n J2n+1 (z) = sin z
n=0

1668
4. We substitute t for t in the generating function.

12 z(t1/t)
X
e = Jn (z)(t)n . (34.5)
n=

We take the product of Equation 34.2 and Equation 34.5 to obtain the final identity.


!
!
1 1
X X
Jn (z)tn Jm (z)(t)m = e 2 z(t1/t) e 2 z(t1/t) = 1
n= m=

Note that the coefficients of all powers of t except t0 in the product of sums must vanish.

X
Jn (z)tn Jn (z)(t)n = 1
n=

X
Jn2 (z) = 1
n=

X
J02 (z) + 2 Jn2 (z) = 1
n=1

Solution 34.10
First we make the change of variables y(x) = x1/2 v(x). We compute the derivatives of y(x).

1
y 0 = x1/2 v 0 + x1/2 v,
2
1
y 00 = x1/2 v 00 + x1/2 v 0 x3/2 v.
4

1669
We substitute these into the differential equation for y.

y 00 + xp2 y = 0
1
x1/2 v 00 + x1/2 v 0 x3/2 v + xp3/2 v = 0
4 
2 00 0 p 1
x v + xv + x v=0
4

Then we make the change of variables v(x) = u(), = p2 xp/2 . We write the derivatives in terms of .

d d d d p d
x =x = xxp/21 =
dx dx d d 2 d
2
d d d d p d p d p2 d2 p2 d
x2 2 + x =x x = = 2 2 +
dx dx dx dx 2 d 2 d 4 d 4 d

We write the differential equation for u().

p2 2 00 p2 0
 2 
p 2 1
u + u + u=0
4 4 4 4
 
00 1 0 1
u + u + 1 2 2 u=0
p

This is the Bessel equation of order 1/p. We can write the general solution for u in terms of Bessel functions of the
first kind if p 6= 1. Otherwise, we use a Bessel function of the second kind.

u() = c1 J1/p () + c2 J1/p () for p 6= 0, 1


u() = c1 J1/p () + c2 Y1/p () for p 6= 0

1670
We write the solution in terms of y(x).


  
2 p/2 2 p/2
y(x) = c1 xJ1/p x + c2 xJ1/p x for p 6= 0, 1
p p

   
2 p/2 2 p/2
y(x) = c1 xJ1/p x + c2 xY1/p x for p 6= 0
p p

The Airy equation y 00 + xy = 0 is the case p = 3. The general solution of the Airy equation is


   
2 3/2 2 3/2
y(x) = c1 xJ1/3 x + c2 xJ1/3 x .
3 3

Solution 34.11
Consider J1/2 (z). We start with the series expansion.

X (1)m  z 1/2+2m
J1/2 (z) = .
m=0
m!(1/2 + m + 1) 2

(1)(3)(2n1)
Use the identity (n + 1/2) = 2n
.


X (1)m 2m+1  z 1/2+2m
=
m=0
m!(1)(3) (2m + 1) 2
 1/2+m
X (1)m 2m+1 1
= z 1/2+2m
m=0
(2)(4) (2m) (1)(3) (2m + 1) 2
 1/2 X
2 (1)m 2m+1
= z
z m=0
(2m + 1)!

1671
We recognize the sum as the Taylor series expansion of sin z.
 1/2
2
= sin z
z

Using the recurrence relations,



J+1 = J J0 and J1 = J + J0 ,
z z
we can find Jn+1/2 for any integer n.
We need J3/2 (z) to determine j1 (z). To find J3/2 (z),

1/2 0
J3/2 (z) = J1/2 (z) J1/2 (z)
z
 1/2    1/2  1/2
1/2 2 1/2 1 2 3/2 2
= z sin z z sin z z 1/2 cos z
z 2
= 21/2 1/2 z 3/2 sin z + 21/2 1/2 z 3/2 sin z 21/2 1/2 cos z
 1/2  1/2
2 3/2 2
= z sin z z 1/2 cos z

 1/2
2
z 3/2 sin z z 1/2 cos z .

=

The spherical Bessel function j1 (z) is


sin z cos z
j1 (z) = .
z2 z
The modified Bessel function of the first kind is

I (z) = J (z).

1672
We can determine I1/2 (z) from J1/2 (z).
r
2
I1/2 (z) = 1/2 sin(z)
z
r
2
= sinh(z)
z
r
2
= sinh(z)
z
The spherical Bessel function i0 (z) is
sinh z
i0 (z) = .
z
The modified Bessel function of the second kind is
I I
K (z) = lim
2 sin()
Thus K1/2 (z) can be determined in terms of I1/2 (z) and I1/2 (z).

K1/2 (z) = I1/2 I1/2
2
We determine I1/2 with the recursion relation

I1 (z) = I0 (z) + I (z).
z

0 1
I1/2 (z) = I1/2 (z) + I1/2 (z)
r 2z r r
2 1/2 1 2 3/2 1 2 1/2
= z cosh(z) z sinh(z) + z sinh(z)
2 2z
r
2
= cosh(z)
z

1673
Now we can determine K1/2 (z).
r r !
2 2
K1/2 (z) = cosh(z) sinh(z)
2 z z
r
z
= e
2z
The spherical Bessel function k0 (z) is
z
k0 (z) = e .
2z

Solution 34.12
The Point at Infinity. With the change of variables z = 1/, w(z) = u() the modified Bessel equation becomes

n2
 
00 1 0
w + w 1+ 2 w =0
z z
4 00 3 0
 0
u + 2 u + u 1 + n2 2 u = 0
2


n2
 
00 1 0 1
u + u 2 u = 0.
4
The point = 0 and hence the point z = is an irregular singular point. We will find the leading order asymptotic
behavior of the solutions as z +.
Controlling Factor. Starting with the modified Bessel equation for real argument

n2
 
00 1 0
y + y 1 + 2 y = 0,
x x

we make the substitution y = es(x) to obtain

1 n2
s00 + (s0 )2 + s0 1 2 = 0.
x x

1674
n2
We know that x2
 1 as x ; we will assume that s00  (s0 )2 as x . This gives us
1
(s0 )2 + s0 1 0 as x .
x
To simplify the equation further, we will try the possible two-term balances.
1. (s0 )2 + x1 s0 0 s0 x1 This balance is not consistent as it violates the assumption that 1 is smaller
than the other terms.

2. (s0 )2 1 0 s0 1 This balance is consistent.


1 0
3. x
s 10 s0 x This balance is inconsistent as (s0 )2 isnt smaller than the other terms.
Thus the only dominant balance is s0 1. This balance is consistent with our initial assumption that s00  (s0 )2 .
Thus s x and the controlling factor is ex . We are interested in the decaying solution, so we will work with the
controlling factor ex .
Leading Order Behavior. In order to find the leading order behavior, we substitute s = x+t(x) where t(x)  x
as x into the differential equation for s. We assume that t0  1 and t00  1/x.

1 n2
t00 + (1 + t0 )2 + (1 + t0 ) 1 2 = 0
x x
2
1 1 n
t00 2t0 + (t0 )2 + t0 2 = 0
x x x
Using our assumptions about the behavior of t0 and t00 ,

1
2t0 0
x
1
t0
2x
1
t ln x as x .
2

1675
This asymptotic behavior is consistent with our assumptions.
Thus the leading order behavior of the decaying solution is
1
y c ex 2 ln x+u(x) = cx1/2 ex+u(x) as x ,

where u(x)  ln x as x .
By substituting t = 12 ln x+u(x) into the differential equation for t, you could show that u(x) const as x .
Thus the full leading order behavior of the decaying solution is

y cx1/2 ex as x

where u(x) 0 as x . It turns out that the asymptotic behavior of the modified Bessel function of the second
kind is r
x
Kn (x) e as x
2x
Asymptotic Series. Now we find the full asymptotic series for Kn (x) as x . We substitute

ex
r
x
Kn (x) e w(x)Kn (x)
2x x
into the modified Bessel equation, where w(x) is a Taylor series about x = , i.e.,
r
x X
Kn (x) e ak xk , a0 = 1.
2x k=0

First we differentiate the expression for Kn (x).


r    
0 x 0 1
Kn (x) e w 1+ w
2x 2x
r      
00 x 00 1 0 1 3
Kn (x) e w 2+ w + 1+ + 2 w
2x x x 4x

1676
We substitute these expressions into the modified Bessel equation.

x2 y 00 + xy 0 x2 + n2 y = 0

   
2 00 2
 0 2 3 0 1
w x2 + n2 w = 0

x w 2x + x w + x + x + w + xw x +
4 2
 
1
x2 w00 2x2 w0 + n2 w = 0
4

We compute the derivatives of the Taylor series.


X
0
w = (k)ak xk1
k=1
X
= (k 1)ak+1 xk2
k=0

X
w00 = (k)(k 1)ak xk2
k=1

X
= (k)(k 1)ak xk2
k=0

We substitute these expression into the differential equation.

X

X
k2
X 1 k2
x2
k(k + 1)ak x + 2x 2
(k + 1)ak+1 x + n 2
ak xk = 0
k=0 k=0
4 k=0
 X
X X 1
k(k + 1)ak xk + 2 (k + 1)ak+1 xk + n2 ak xk = 0
k=0 k=0
4 k=0

1677
We equate coefficients of x to obtain a recurrence relation for the coefficients.
 
1 2
k(k + 1)ak + 2(k + 1)ak+1 + n ak = 0
4
n2 1/4 k(k + 1)
ak+1 = ak
2(k + 1)
n2 (k + 1/2)2
ak+1 = ak
2(k + 1)
4n2 (2k + 1)2
ak+1 = ak
8(k + 1)
We set a0 = 1. We use the recurrence relation to determine the rest of the coefficients.
Qk 2 2
j=1 (4n (2j 1) )
ak =
8k k!
Now we have the asymptotic expansion of the modified Bessel function of the second kind.
Qk
x X j=1 (4n2 (2j 1)2 ) k
r
Kn (x) e k k!
x , as x
2x k=0
8

Convergence. We determine the domain of convergence of the series with the ratio test. The Taylor series about
infinity will converge outside of some circle.

ak+1 (x)
lim <1
k ak (x)
ak+1 xk1

lim <1
k ak xk
2
4n (2k + 1)2 1

lim |x| < 1
k 8(k + 1)
< |x|

1678
The series does not converge for any x in the finite complex plane. However, if we take only a finite number of terms
in the series, it gives a good approximation of Kn (x) for large, positive x. At x = 10, the one, two and three term
approximations give relative errors of 0.01, 0.0006 and 0.00006, respectively.

1679
Part V

Partial Differential Equations

1680
Chapter 35

Transforming Equations

Im about two beers away from fine.


Let {xi } denote rectangular coordinates. Let {ai } be unit basis vectors in the orthogonal coordinate system {i }.
The distance metric coefficients hi can be defined
s 2  2  2
x1 x2 x3
hi = + + .
i i i

The gradient, divergence, etc., follow.


a1 u a2 u a3 u
u = + +
h1 1 h2 2 h3 3
 
1
v = (h2 h3 v1 ) + (h3 h1 v2 ) + (h1 h2 v3 )
h1 h2 h3 1 2 3
      
2 1 h2 h3 u h3 h1 u h1 h2 u
u= + +
h1 h2 h3 1 h1 1 2 h2 2 3 h3 3

1681
35.1 Exercises
Exercise 35.1
Find the Laplacian in cylindrical coordinates (r, , z).

x = r cos , y = r sin , z

Hint, Solution
Exercise 35.2
Find the Laplacian in spherical coordinates (r, , ).

x = r sin cos , y = r sin sin , z = r cos

Hint, Solution

1682
35.2 Hints
Hint 35.1

Hint 35.2

1683
35.3 Solutions
Solution 35.1

p
h1 = (cos )2 + (sin )2 + 0 = 1
p
h2 = (r sin )2 + (r cos )2 + 0 = r

h3 = 0 + 0 + 12 = 1

      
2 1 u 1 u u
u= r + + r
r r r r z z
2 2
 
1 u 1 u u
2 u = r + 2 2 + 2
r r r r z

Solution 35.2

p
h1 = (sin cos )2 + (sin sin )2 + (cos )2 = 1
p
h2 = (r cos cos )2 + (r cos sin )2 + (r sin )2 = r
p
h3 = (r sin sin )2 + (r sin cos )2 + 0 = r sin

      
2 1 2 u u 1 u
u= 2 r sin + sin +
r sin r r sin
1 2u
   
1 u 1 u
2 u = 2 r2 + 2 sin + 2
r r r r sin r sin 2

1684
Chapter 36

Classification of Partial Differential Equations

36.1 Classification of Second Order Quasi-Linear Equations


Consider the general second order quasi-linear partial differential equation in two variables.

a(x, y)uxx + 2b(x, y)uxy + c(x, y)uyy = F (x, y, u, ux , uy ) (36.1)

We classify the equation by the sign of the discriminant. At a given point x0 , y0 , the equation is classified as one of
the following types:
b2 ac > 0 : hyperbolic
2
b ac = 0 : parabolic
2
b ac < 0 : elliptic
If an equation has a particular type for all points x, y in a domain then the equation is said to be of that type in the
domain. Each of these types has a canonical form that can be obtained through a change of independent variables.
The type of an equation indicates much about the nature of its solution.
We seek a change of independent variables, (a different coordinate system), such that Equation 36.1 has a simpler
form. We will find that a second order quasi-linear partial differential equation in two variables can be transformed to

1685
one of the canonical forms:

u = G(, , u, u , u ), hyperbolic
u = G(, , u, u , u ), parabolic
u + u = G(, , u, u , u ), elliptic

Consider the change of independent variables

= (x, y), = (x, y).

We calculate the partial derivatives of u.

ux = x u + x u
uy = y u + y u
uxx = x2 u + 2x x u + x2 u + xx u + xx u
uxy = x y u + (x y + y x )u + x y u + xy u + xy u
uyy = y2 u + 2y y u + y2 u + yy u + yy u

Substituting these into Equation 36.1 yields an equation in and .

ax2 + 2bx y + cy2 u + 2 (ax x + b(x y + y x ) + cy y ) u




+ ax2 + 2bx y + cy2 u = H(, , u, u , u )




(, )u + (, )u + (, )u = H(, , u, u , u ) (36.2)

36.1.1 Hyperbolic Equations


We start with a hyperbolic equation, (b2 ac > 0). We seek a change of independent variables that will put Equation 36.1
in the form
u = G(, , u, u , u ) (36.3)

1686
We require that the u and u terms vanish. That is = = 0 in Equation 36.2. This gives us two constraints on
and .

ax2 + 2bx y + cy2 = 0, ax2 + 2bx y + cy2 = 0 (36.4)



x b + b2 ac x b b2 ac
= , =
y a y a

b b2 ac b + b2 ac
x + y = 0, x + y = 0
a a
Here we chose the signs in the quadratic formulas to get different solutions for and .
Now we have first order quasi-linear partial differential equations for the coordinates and . We solve these
equations with the method of characteristics. The characteristic equations for are

dy b b2 ac d
= , (x, y(x)) = 0
dx a dx
Solving the differential equation for y(x) determines (x, y). We just write the solution for y(x) in the form F (x, y(x)) =
const. Since the solution of the differential equation for is (x, y(x)) = const, we then have = F (x, y). Upon
solving for and we divide Equation 36.2 by (, ) to obtain the canonical form.

Note that we could have solved for y /x in Equation 36.4.



dx y b b2 ac
= =
dy x c

This form is useful if a vanishes.

Another canonical form for hyperbolic equations is

u u = K(, , u, u , u ). (36.5)

1687
We can transform Equation 36.3 to this form with the change of variables

= + , = .

Equation 36.3 becomes  


+
u u = G , , u, u + u , u u .
2 2

Example 36.1.1 Consider the wave equation with a source.

utt c2 uxx = s(x, t)

Since 0 (1)(c2 ) > 0, the equation is hyperbolic. We find the new variables.
dx
= c, x = ct + const, = x + ct
dt
dx
= c, x = ct + const, = x ct
dt
Then we determine t and x in terms of and .
+
t= , x=
2c 2
We calculate the derivatives of and .

t = c x = 1
t = c x = 1

Then we calculate the derivatives of u.

utt = c2 u 2c2 u + c2 u
uxx = u + u

1688
Finally we transform the equation to canonical form.
 
2 +
2c u = s ,
2 2c
 
1 +
u = 2s ,
2c 2 2c

If s(x, t) = 0, then the equation is u = 0 we can integrate with respect to and to obtain the solution,
u = f () + g(). Here f and g are arbitrary C 2 functions. In terms of t and x, we have

u(x, t) = f (x + ct) + g(x ct).

To put the wave equation in the form of Equation 36.5 we make a change of variables

= + = 2x, = = 2ct
utt c2 uxx = s(x, t)
 
4c2 u 4c2 u = s ,
2 2c
1  
u u = 2 s ,
4c 2 2c

Example 36.1.2 Consider


y 2 uxx x2 uyy = 0.
For x 6= 0 and y 6= 0 this equation is hyperbolic. We find the new variables.
p
dy y 2 x2 x y2 x2
= = , y dy = x dx, = + const, = y 2 + x2
dx y2 y 2 2
p
dy y 2 x2 x y2 x2
= = , y dy = x dx, = + const, = y 2 x2
dx y2 y 2 2

1689
We calculate the derivatives of and .

x = 2x y = 2y
x = 2x y = 2y

Then we calculate the derivatives of u.

ux = 2x(u u )
uy = 2y(u + u )
2
uxx = 4x (u 2u + u ) + 2(u u )
uyy = 4y 2 (u + 2u + u ) + 2(u + u )

Finally we transform the equation to canonical form.

y 2 uxx x2 uyy = 0
8x2 y 2 u 8x2 y 2 u + 2y 2 (u u ) + 2x2 (u + u ) = 0
1 1
16 ( ) ( + )u = 2u 2u
2 2
u u
u =
2( 2 2 )

Example 36.1.3 Consider Laplaces equation.


uxx + uyy = 0
Since 0 (1)(1) < 0, the equation is elliptic. We will transform this equation to the canical form of Equation 36.3.
We find the new variables.
dy
= , y = x + const, = x + y
dx
dy
= , y = x + const, = x y
dx

1690
We calculate the derivatives of and .

x = 1 y =
x = 1 y =

Then we calculate the derivatives of u.

uxx = u + 2u + u
uyy = u + 2u u

Finally we transform the equation to canonical form.

4u = 0
u = 0

We integrate with respect to and to obtain the solution, u = f () + g(). Here f and g are arbitrary C 2
functions. In terms of x and y, we have

u(x, y) = f (x + y) + g(x y).

This solution makes a lot of sense, because the real and imaginary parts of an analytic function are harmonic.

36.1.2 Parabolic equations


Now we consider a parabolic equation, (b2 ac = 0). We seek a change of independent variables that will put
Equation 36.1 in the form
u = G(, , u, u , u ). (36.6)
We require that the u and u terms vanish. That is = = 0 in Equation 36.2. This gives us two constraints on
and .
ax x + b(x y + y x ) + cy y = 0, ax2 + 2bx y + cy2 = 0

1691
We consider the case a 6= 0. The latter constraint allows us to solve for x /y .

x b b2 ac b
= =
y a a
With this information, the former constraint is trivial.
ax x + b(x y + y x ) + cy y = 0
ax (b/a) + b(x + y (b/a)) + cy = 0
(ac b2 )y = 0
0=0
Thus we have a first order partial differential equation for the coordinate which we can solve with the method of
characteristics.
b
x + y = 0
a
The coordinate is chosen to be anything linearly independent of . The characteristic equations for are
dy b d
= , (x, y(x)) = 0
dx a dx
Solving the differential equation for y(x) determines (x, y). We just write the solution for y(x) in the form F (x, y(x)) =
const. Since the solution of the differential equation for is (x, y(x)) = const, we then have = F (x, y). Upon
solving for and choosing a linearly independent , we divide Equation 36.2 by (, ) to obtain the canonical form.
In the case that a = 0, we would instead have the constraint,
b
x + y = 0.
c

36.1.3 Elliptic Equations


We start with an elliptic equation, (b2 ac < 0). We seek a change of independent variables that will put Equation 36.1
in the form
u + u = G(, , u, u , u ) (36.7)

1692
If we make the change of variables determined by

x b + ac b2 x b ac b2
= , = ,
y a y a

the equation will have the form


u = G(, , u, u , u ).
and are complex-valued. If we then make the change of variables

+
= , =
2 2
we will obtain the canonical form of Equation 36.7. Note that since and are complex conjugates, and are
real-valued.

Example 36.1.4 Consider


y 2 uxx + x2 uyy = 0. (36.8)
For x 6= 0 and y 6= 0 this equation is elliptic. We find new variables that will put this equation in the form u = G().
From Example 36.1.2 we see that they are
p
dy y 2 x2 x y2 x2
= = , y dy = x dx, = + const, = y 2 + x2
dx y2 y 2 2
p
dy y 2 x2 x y2 x2
= = , y dy = x dx, = + const, = y 2 x2
dx y2 y 2 2

The variables that will put Equation 36.8 in canonical form are

+
= = y2, = = x2
2 2

1693
We calculate the derivatives of and .
x = 0 y = 2y
x = 2x y = 0
Then we calculate the derivatives of u.
ux = 2xu
uy = 2yu
uxx = 4x2 u + 2u
uyy = 4y 2 u + 2u
Finally we transform the equation to canonical form.
y 2 uxx + x2 uyy = 0
(4 u + 2u ) + (4u + 2u ) = 0
1 1
u + u = u u
2 2

36.2 Equilibrium Solutions


Example 36.2.1 Consider the equilibrium solution for the following problem.
ut = uxx , u(x, 0) = x, ux (0, t) = ux (1, t) = 0
Setting ut = 0 we have an ordinary differential equation.
d2 u
=0
dx2
This equation has the solution,
u = ax + b.

1694
Applying the boundary conditions we see that
u = b.
To determine the constant, we note that the heat energy in the rod is constant in time.
Z 1 Z 1
u(x, t) dx = u(x, 0) dx
0 0
Z 1 Z 1
b dx = x dx
0 0

Thus the equilibrium solution is


1
u(x) = .
2

1695
36.3 Exercises
Exercise 36.1
Classify and transform the following equation into canonical form.

uxx + (1 + y)2 uyy = 0

Hint, Solution
Exercise 36.2
Classify as hyperbolic, parabolic, or elliptic in a region R each of the equations:

1. ut = (pux )x

2. utt = c2 uxx u

3. (qux )x + (qut )t = 0

where p(x), c(x, t), q(x, t), and (x) are given functions that take on only positive values in a region R of the (x, t)
plane.
Hint, Solution
Exercise 36.3
Transform each of the following equations for (x, y) into canonical form in appropriate regions

1. xx y 2 yy + x + x2 = 0

2. xx + xyy = 0

The equation in part (b) is known as Tricomis equation and is a model for transonic fluid flow in which the flow speed
changes from supersonic to subsonic.
Hint, Solution

1696
36.4 Hints
Hint 36.1

Hint 36.2

Hint 36.3

1697
36.5 Solutions
Solution 36.1
For y = 1, the equation is parabolic. For this case it is already in the canonical form, uxx = 0.
For y 6= 1, the equation is elliptic. We find new variables that will put the equation in the form u =
G(, , u, u , u ).
dy p
= (1 + y)2 = (1 + y)
dx
dy
= dx
1+y
log(1 + y) = x + c
1 + y = c ex
(1 + y) ex = c
= (1 + y) ex
= = (1 + y) ex
The variables that will put the equation in canonical form are
+
= = (1 + y) cos x, = = (1 + y) sin x.
2 2
We calculate the derivatives of and .
x = (1 + y) sin x y = cos x
x = (1 + y) cos x y = sin x
Then we calculate the derivatives of u.
ux = (1 + y) sin(x)u + (1 + y) cos(x)u
uy = cos(x)u + sin(x)u
uxx = (1 + y) sin (x)u + (1 + y)2 cos2 (x)u (1 + y) cos(x)u (1 + y) sin(x)u
2 2

uyy = cos2 (x)u + sin2 (x)u

1698
We substitute these results into the differential equation to obtain the canonical form.

uxx + (1 + y)2 uyy = 0


(1 + y)2 (u + u ) (1 + y) cos(x)u (1 + y) sin(x)u = 0
2 + 2 (u + u ) u u = 0


u + u
u + u =
2 + 2

Solution 36.2
1.

ut = (pux )x
puxx + 0uxt + 0utt + px ux ut = 0

Since 02 p0 = 0, the equation is parabolic.

2.

utt = c2 uxx u
utt + 0utx c2 uxx + u = 0

Since 02 (1)(c2 ) > 0, the equation is hyperbolic.

3.

(qux )x + (qut )t = 0
quxx + 0uxt + qutt + qx ux + qt ut = 0

Since 02 qq < 0, the equation is elliptic.

1699
Solution 36.3
1. For y 6= 0, the equation is hyperbolic. We find the new independent variables.
p
dy y2
= = y, y = c ex , ex y = c, = ex y
dx p1
dy y2
= = y, y = c ex , ex y = c, = ex y
dx 1
Next we determine x and y in terms of and .
p
= y 2 , y =
 
p p 1
= ex , ex = /, x = log
2
We calculate the derivatives of and .

x = ex y =
p
y = ex = /
x = ex y =
p
y = ex = /

Then we calculate the derivatives of .


s s

= + , = +
x y
s s

x = + , y = +


xx = 2 2 + 2 + + , yy = + 2 +

1700
Finally we transform the equation to canonical form.

xx y 2 yy + x + x2 = 0
 

4 + + + + log =0

 
1
= + log
2

For y = 0 we have the ordinary differential equation

xx + x + x2 = 0.

2. For x < 0, the equation is hyperbolic. We find the new independent variables.
dy 2 2
= x, y = x x + c, = x x y
dx 3 3
dy 2 2
= x, y = x x + c, = x x + y
dx 3 3
Next we determine x and y in terms of and .
 1/3
3
x= ( + ) , y=
4 2
We calculate the derivatives of and .
1/6


3
x = x = ( + ) , y = 1
4
 1/6
3
x = ( + ) , y = 1
4

1701
Then we calculate the derivatives of .
 1/6
3
x = ( + ) ( + )
4
y = +
 1/3
3
xx = ( + ) ( + ) + (6( + ))1/3 + (6( + ))2/3 ( + )
4
yy = 2 +

Finally we transform the equation to canonical form.

xx + xyy = 0
(6( + )) 1/3
+ (6( + ))1/3 + (6( + ))2/3 ( + ) = 0
+
=
12( + )

For x > 0, the equation is elliptic. The variables we defined before are complex-valued.
2 2
= x3/2 y, = x3/2 + y
3 3
We choose the new real-valued variables.

= , = ( + )

We write the derivatives in terms of and .

=
=
=

1702
We transform the equation to canonical form.
+
=
12( + )
2
=
12

+ =
6

1703
Chapter 37

Separation of Variables

37.1 Eigensolutions of Homogeneous Equations

37.2 Homogeneous Equations with Homogeneous Boundary Condi-


tions
The method of separation of variables is a useful technique for finding special solutions of partial differential equations.
We can combine these special solutions to solve certain problems. Consider the temperature of a one-dimensional rod
of length h 1 . The left end is held at zero temperature, the right end is insulated and the initial temperature distribution
is known at time t = 0. To find the temperature we solve the problem:

u 2u
= 2, 0 < x < h, t > 0
t x
u(0, t) = ux (h, t) = 0
u(x, 0) = f (x)
1
Why h? Because l looks like 1 and we use L to denote linear operators

1704
We look for special solutions of the form, u(x, t) = X(x)T (t). Substituting this into the partial differential equation
yields
X(x)T 0 (t) = X 00 (x)T (t)
T 0 (t) X 00 (x)
=
T (t) X(x)
Since the left side is only dependent on t, the right side in only dependent on x, and the relation is valid for all t and
x, both sides of the equation must be constant.
T0 X 00
= =
T X
Here is an arbitrary constant. (Youll see later that this form is convenient.) u(x, t) = X(x)T (t) will satisfy the
partial differential equation if X(x) and T (t) satisfy the ordinary differential equations,
T 0 = T and X 00 = X.
Now we see how lucky we are that this problem happens to have homogeneous boundary conditions 2 . If the left
boundary condition had been u(0, t) = 1, this would imply X(0)T (t) = 1 which tells us nothing very useful about
either X or T . However the boundary condition u(0, t) = X(0)T (t) = 0, tells us that either X(0) = 0 or T (t) = 0.
Since the latter case would give us the trivial solution, we must have X(0) = 0. Likewise by looking at the right
boundary condition we obtain X 0 (h) = 0.

We have a regular Sturm-Liouville problem for X(x).


X 00 + X = 0, X(0) = X 0 (h) = 0
The eigenvalues and orthonormal eigenfunctions are
 2 r  
(2n 1) 2 (2n 1)
n = , Xn = sin x , n Z+ .
2h h 2h
2
Actually luck has nothing to do with it. I planned it that way.

1705
Now we solve the equation for T (t).
T 0 = n T
T = c en t
The eigen-solutions of the partial differential equation that satisfy the homogeneous boundary conditions are
r
2 p 
un (x, t) = sin n x en t .
h
We seek a solution of the problem that is a linear combination of these eigen-solutions.
r
X 2 p 
u(x, t) = an sin n x en t
n=1
h

We apply the initial condition to find the coefficients in the expansion.


r
X 2 p 
u(x, 0) = an sin n x = f (x)
n=1
h
r Z h
2 p 
an = sin n x f (x) dx
h 0

37.3 Time-Independent Sources and Boundary Conditions


Consider the temperature in a one-dimensional rod of length h. The ends are held at temperatures and ,
respectively, and the initial temperature is known at time t = 0. Additionally, there is a heat source, s(x), that is
independent of time. We find the temperature by solving the problem,
ut = uxx + s(x), u(0, t) = , u(h, t) = , u(x, 0) = f (x). (37.1)
Because of the source term, the equation is not separable, so we cannot directly apply separation of variables. Fur-
thermore, we have the added complication of inhomogeneous boundary conditions. Instead of attacking this problem
directly, we seek a transformation that will yield a homogeneous equation and homogeneous boundary conditions.

1706
Consider the equilibrium temperature, (x). It satisfies the problem,

s(x)
00 (x) = = 0, (0) = , (h) = .

The Green function for this problem is,


x< (x> h)
G(x; ) = .
h
The equilibrium temperature distribution is
Z h
xh x 1
(x) = + x< (x> h)s() d,
h h h 0
 Z x Z h 
x 1
(x) = + ( ) (x h) s() d + x ( h)s() d .
h h 0 x

Now we substitute u(x, t) = v(x, t) + (x) into Equation 37.1.

2
(v + (x)) = 2 (v + (x)) + s(x)
t x
vt = vxx + 00 (x) + s(x)
vt = vxx (37.2)

Since the equilibrium solution satisfies the inhomogeneous boundary conditions, v(x, t) satisfies homogeneous boundary
conditions.
v(0, t) = v(h, t) = 0.

The initial value of v is


v(x, 0) = f (x) (x).

1707
We seek a solution for v(x, t) that is a linear combination of eigen-solutions of the heat equation. We substitute the
separation of variables, v(x, t) = X(x)T (t) into Equation 37.2
T0 X 00
= =
T X
This gives us two ordinary differential equations.
X 00 + X = 0, X(0) = X(h) = 0
0
T = T.
The Sturm-Liouville problem for X(x) has the eigenvalues and orthonormal eigenfunctions,
 n 2 r
2  nx 
n = , Xn = sin , n Z+ .
h h h
We solve for T (t).
2
Tn = c e(n/h) t .
The eigen-solutions of the partial differential equation are
r
2  nx  2
vn (x, t) = sin e(n/h) t .
h h
The solution for v(x, t) is a linear combination of these.
r
X 2  nx  2
v(x, t) = an sin e(n/h) t
n=1
h h
We determine the coefficients in the series with the initial condition.
r
X 2  nx 
v(x, 0) = an sin = f (x) (x)
n=1
h h
r Z h
2  nx 
an = sin (f (x) (x)) dx
h 0 h

1708
The temperature of the rod is
r
X 2  nx  2
u(x, t) = (x) + an sin e(n/h) t
n=1
h h

37.4 Inhomogeneous Equations with Homogeneous Boundary Con-


ditions
Now consider the heat equation with a time dependent source, s(x, t).
ut = uxx + s(x, t), u(0, t) = u(h, t) = 0, u(x, 0) = f (x). (37.3)
In general we cannot transform the problem to one with a homogeneous differential equation. Thus we cannot represent
the solution in a series of the eigen-solutions of the partial differential equation. Instead, we will do the next best thing
and expand the solution in a series of eigenfunctions in Xn (x) where the coefficients depend on time.

X
u(x, t) = un (t)Xn (x)
n=1

We will find these eigenfunctions with the separation of variables, u(x, t) = X(x)T (t) applied to the homogeneous
equation, ut = uxx , which yields, r
2  nx 
Xn (x) = sin , n Z+ .
h h
We expand the heat source in the eigenfunctions.
r
X 2  nx 
s(x, t) = sn (t) sin
n=1
h h
r Z h
2  nx 
sn (t) = sin s(x, t) dx,
h 0 h

1709
We substitute the series solution into Equation 37.3.
r  n 2 r 2 r
X
0 2  nx  X  nx  X 2  nx 
un (t) sin = un (t) sin + sn (t) sin
n=1
h h n=1
h h h n=1
h h
 n 2
u0n (t) + un (t) = sn (t)
h
Now we have a first order, ordinary differential equation for each of the un (t). We obtain initial conditions from the
initial condition for u(x, t).
r
X 2  nx 
u(x, 0) = un (0) sin = f (x)
n=1
h h
r Z h
2  nx 
un (0) = sin f (x) dx fn
h 0 h
The temperature is given by
r
X 2  nx 
u(x, t) = un (t) sin ,
n=1
h h
Z t
(n/h)2 t 2 (t )
un (t) = fn e + e(n/h) sn ( ) d.
0

37.5 Inhomogeneous Boundary Conditions


Consider the temperature of a one-dimensional rod of length h. The left end is held at the temperature (t), the
heat flow at right end is specified, there is a time-dependent source and the initial temperature distribution is known
at time t = 0. To find the temperature we solve the problem:
ut = uxx + s(x, t), 0 < x < h, t > 0 (37.4)
u(0, t) = (t), ux (h, t) = (t) u(x, 0) = f (x)

1710
Transformation to a homogeneous equation. Because of the inhomogeneous boundary conditions, we cannot
directly apply the method of separation of variables. However we can transform the problem to an inhomogeneous
equation with homogeneous boundary conditions. To do this, we first find a function, (x, t) which satisfies the
boundary conditions. We note that
(x, t) = (t) + x(t)
does the trick. We make the change of variables

u(x, t) = v(x, t) + (x, t)

in Equation 37.4.

vt + t = (vxx + xx ) + s(x, t)
vt = vxx + s(x, t) t

The boundary and initial conditions become

v(0, t) = 0, vx (h, t) = 0, v(x, 0) = f (x) (x, 0).

Thus we have a heat equation with the source s(x, t) t (x, t). We could apply separation of variables to find a
solution of the form r  
X 2 (2n 1)x
u(x, t) = (x, t) + un (t) sin .
n=1
h 2h

Direct eigenfunction expansion. Alternatively we could seek a direct eigenfunction expansion of u(x, t).
r  
X 2 (2n 1)x
u(x, t) = un (t) sin .
n=1
h 2h

Note that the eigenfunctions satisfy the homogeneous boundary conditions while u(x, t) does not. If we choose any
fixed time t = t0 and form the periodic extension of the function u(x, t0 ) to define it for x outside the range (0, h), then

1711
this function will have jump discontinuities. This means that our eigenfunction expansion will not converge uniformly.
We are not allowed to differentiate the series with respect to x. We cant just plug the series into the partial differential
equation to determine the coefficients. Instead, we will multiply Equation 37.4, by an eigenfunction and integrate from
x = 0 to x = h. To avoid differentiating the series with respect to 
x, we will use integration by parts to move derivatives
 2
(2n1)
from u(x, t) to the eigenfunction. (We will denote n = 2h
.)

r Z h r Z h
2 p 2 p
sin( n x)(ut uxx ) dx = sin( n x)s(x, t) dx
h 0 h 0
r h r Z h
0 2 p i h 2 p p
un (t) ux sin( n x) + n ux cos( n x) dx = sn (t)
h 0 h 0
r r r Z h
0 2 n 2 p h p i h 2 p
un (t) (1) ux (h, t) + n u cos( n x) + n u sin( n x) dx = sn (t)
h h 0 h 0
r r
2 2 p
u0n (t) (1)n (t) n u(0, t) + n un (t) = sn (t)
h h
r 
2 p 
u0n (t) + n un (t) = n (t) + (1)n (t) + sn (t)
h

Now we have an ordinary differential equation for each of the un (t). We obtain initial conditions for them using the
initial condition for u(x, t).

r
X 2 p
u(x, 0) = un (0) sin( n x) = f (x)
n=1
h
r Z h
2 p
un (0) = sin( n x)f (x) dx fn
h 0

1712
Thus the temperature is given by
r
2X p
u(x, t) = un (t) sin( n x),
h n=1
r Z t
n t 2 p 
un (t) = fn e + en (t ) n ( ) + (1)n ( ) d.
h 0

37.6 The Wave Equation


Consider an elastic string with a free end at x = 0 and attached to a massless spring at x = 1. The partial differential
equation that models this problem is

utt = uxx
ux (0, t) = 0, ux (1, t) = u(1, t), u(x, 0) = f (x), ut (x, 0) = g(x).

We make the substitution u(x, t) = (x)(t) to obtain

00 00
= = .

First we consider the problem for .

00 + = 0, 0 (0) = (1) + 0 (1) = 0.

To find the eigenvalues we consider the following three cases:

< 0. The general solution is


= a cosh( x) + b sinh( x).

1713
0 (0) = 0 b = 0.

(1) + 0 (1) = 0 a cosh( ) + a sinh( ) = 0
a = 0.

Since there is only the trivial solution, there are no negative eigenvalues.

= 0. The general solution is


= ax + b.

0 (0) = 0 a = 0.
(1) + 0 (1) = 0 b + 0 = 0.

Thus = 0 is not an eigenvalue.

> 0. The general solution is


= a cos( x) + b sin( x).

0 (0) b = 0.
0

(1) + (1) = 0 a cos( ) a sin( ) = 0

cos( ) = sin( )

= cot( )

By looking at Figure 37.1, (the plot shows the functions f (x) = x, f (x) = cot x and has lines at x = n), we
see that there are an infinite number of positive eigenvalues and that

n (n)2 as n .

The eigenfunctions are p


n = cos( n x).

1714
10

2 4 6 8 10

-2

Figure 37.1: Plot of x and cot x.

The solution for is p p


n = an cos( n t) + bn sin( n t).
Thus the solution to the differential equation is

X p p p
u(x, t) = cos( n x)[an cos( n t) + bn sin( n t)].
n=1

Let

X p
f (x) = fn cos( n x)
n=1

X p
g(x) = gn cos( n x).
n=1

1715
From the initial value we have

X p
X p
cos( n x)an = fn cos( n x)
n=1 n=1
an = fn .

The initial velocity condition gives us



X p p
X p
cos( n x) n bn = gn cos( n x)
n=1 n=1
gn
bn = .
n
Thus the solution is
 
X p p gn p
u(x, t) = cos( n x) fn cos( n t) + sin( n t) .
n=1
n

37.7 General Method


Here is an outline detailing the method of separation of variables for a linear partial differential equation for u(x, y, z, . . .).

1. Substitute u(x, y, z, . . .) = X(x)Y (y)Z(z) into the partial differential equation. Separate the equation into
ordinary differential equations.

2. Translate the boundary conditions for u into boundary conditions for X, Y , Z, . . .. The continuity of u may give
additional boundary conditions and boundedness conditions.

3. Solve the differential equation(s) that determine the eigenvalues. Make sure to consider all cases. The eigen-
functions will be determined up to a multiplicative constant.

1716
4. Solve the rest of the differential equations subject to the homogeneous boundary conditions. The eigenvalues will
be a parameter in the solution. The solutions will be determined up to a multiplicative constant.

5. The eigen-solutions are the product of the solutions of the ordinary differential equations. n = Xn Yn Zn .
The solution of the partial differential equation is a linear combination of the eigen-solutions.
X
u(x, y, z, . . .) = an n

6. Solve for the coefficients, an using the inhomogeneous boundary conditions.

1717
37.8 Exercises
Exercise 37.1
Solve the following problem with separation of variables.

ut (uxx + uyy ) = q(x, y, t), 0 < x < a, 0 < y < b


u(x, y, 0) = f (x, y), u(0, y, t) = u(a, y, t) = u(x, 0, t) = u(x, b, t) = 0

Hint, Solution
Exercise 37.2
Consider a thin half pipe of unit radius laying on the ground. It is heated by radiation from above. We take the initial
temperature of the pipe and the temperature of the ground to be zero. We model this problem with a heat equation
with a source term.

ut = uxx + A sin(x)
u(0, t) = u(, t) = 0, u(x, 0) = 0

Hint, Solution
Exercise 37.3
Consider Laplaces Equation 2 u = 0 inside the quarter circle of radius 1 (0 2 , 0 r 1). Write the problem
in polar coordinates u = u(r, ) and use separation of variables to find the solution subject to the following boundary
conditions.

1.
u  
(r, 0) = 0, u r, = 0, u(1, ) = f ()
2
2.
u u   u
(r, 0) = 0, r, = 0, (1, ) = g()
2 r
Under what conditions does this solution exist?

1718
Hint, Solution
Exercise 37.4
Consider the 2-D heat equation
ut = (uxx + uyy ),
on a square plate 0 < x < 1, 0 < y < 1 with two sides insulated

ux (0, y, t) = 0 ux (1, y, t) = 0,

two sides with fixed temperature


u(x, 0, t) = 0 u(x, 1, t) = 0,
and initial temperature
u(x, y, 0) = f (x, y).

1. Reduce this to a set of 3 ordinary differential equations using separation of variables.

2. Find the corresponding set of eigenfunctions and give the solution satisfying the given initial condition.

Hint, Solution
Exercise 37.5
Solve the 1-D heat equation
ut = uxx ,
on the domain 0 < x < subject to conditions that the ends are insulated (i.e. zero flux)

ux (0, t) = 0 ux (, t) = 0,

and the initial temperature distribution is u(x, 0) = x.


Hint, Solution

1719
Exercise 37.6
Obtain Poissons formula to solve the Dirichlet problem for the circular region 0 r < R, 0 < 2. That is,
determine a solution (r, ) to Laplaces equation
2 = 0
in polar coordinates given (R, ). Show that
Z 2
1 R2 r 2
(r, ) = (R, ) 2 d
2 0 R + r2 2Rr cos( )
Hint, Solution
Exercise 37.7
Consider the temperature of a ring of unit radius. Solve the problem
ut = u , u(, 0) = f ()
with separation of variables.
Hint, Solution
Exercise 37.8
Solve the Laplaces equation by separation of variables.
u uxx + uyy = 0, 0 < x < 1, 0 < y < 1,
u(x, 0) = f (x), u(x, 1) = 0, u(0, y) = 0, u(1, y) = 0
Here f (x) is an arbitrary function which is known.
Hint, Solution
Exercise 37.9
Solve Laplaces equation in the unit disk with separation of variables.
u = 0, 0 < r < 1
u(1, ) = f ()

1720
The Laplacian in cirular coordinates is
2 u 1 u 1 2u
u + + .
r2 r r r2 2
Hint, Solution
Exercise 37.10
Find the normal modes of oscillation of a drum head of unit radius. The drum head obeys the wave equation with zero
displacement on the boundary.
1 2v 1 2v
 
1 v
v r + 2 2 = 2 2, v(1, , t) = 0
r r r r c t
Hint, Solution
Exercise 37.11
Solve the equation
t = a2 xx , 0 < x < l, t>0
with boundary conditions (0, t) = (l, t) = 0, and initial conditions
(
x, 0 x l/2,
(x, 0) =
l x, l/2 < x l.

Comment on the differentiability ( that is the number of finite derivatives with respect to x ) at time t = 0 and at time
t = , where  > 0 and   1.
Hint, Solution
Exercise 37.12
Consider a one-dimensional rod of length L with initial temperature distribution f (x). The temperatures at the left
and right ends of the rod are held at T0 and T1 , respectively. To find the temperature of the rod for t > 0, solve
ut = uxx , 0 < x < L, t > 0
u(0, t) = T0 , u(L, t) = T1 , u(x, 0) = f (x),

1721
with separation of variables.
Hint, Solution
Exercise 37.13
For 0 < x < l solve the problem
t = a2 xx + w(x, t) (37.5)
(0, t) = 0, x (l, t) = 0, (x, 0) = f (x)
by means of a series expansion involving the eigenfunctions of
d2 (x)
+ (x) = 0,
dx2
(0) = 0 (l) = 0.
Here w(x, t) and f (x) are prescribed functions.
Hint, Solution
Exercise 37.14
Solve the heat equation of Exercise 37.13 with the same initial conditions but with the boundary conditions
(0, t) = 0, c(l, t) + x (l, t) = 0.
Here c > 0 is a constant. Although it is not possible to solve for the eigenvalues in closed form, show that the
eigenvalues assume a simple form for large values of .
Hint, Solution
Exercise 37.15
Use a series expansion technique to solve the problem
t = a2 xx + 1, t > 0, 0<x<l
with boundary and initial conditions given by
(x, 0) = 0, (0, t) = t, x (l, t) = c(l, t)

1722
where c > 0 is a constant.
Hint, Solution
Exercise 37.16
Let (x, t) satisfy the equation
t = a2 xx
for 0 < x < l, t > 0 with initial conditions (x, 0) = 0 for 0 < x < l, with boundary conditions (0, t) = 0 for t > 0,
and (l, t) + x (l, t) = 1 for t > 0. Obtain two series solutions for this problem, one which is useful for large t and the
other useful for small t.
Hint, Solution
Exercise 37.17
A rod occupies the portion 1 < x < 2 of the x-axis. The thermal conductivity depends on x in such a manner that the
temperature (x, t) satisfies the equation
t = A2 (x2 x )x (37.6)
where A is a constant. For (1, t) = (2, t) = 0 for t > 0, with (x, 0) = f (x) for 1 < x < 2, show that the
appropriate series expansion involves the eigenfunctions
 
1 n ln x
n (x) = sin .
x ln 2
Work out the series expansion for the given boundary and initial conditions.
Hint, Solution
Exercise 37.18
Consider a string of length L with a fixed left end a free right end. Initially the string is at rest with displacement f (x).
Find the motion of the string by solving,

utt = c2 uxx , 0 < x < L, t > 0,


u(0, t) = 0, ux (L, t) = 0,
u(x, 0) = f (x), ut (x, 0) = 0,

1723
with separation of variables.
Hint, Solution
Exercise 37.19
Consider the equilibrium temperature distribution in a two-dimensional block of width a and height b. There is a heat
source given by the function f (x, y). The vertical sides of the block are held at zero temperature; the horizontal sides
are insulated. To find this equilibrium temperature distribution, solve the potential equation,

uxx + uyy = f (x, y), 0 < x < a, 0 < y < b,


u(0, y) = u(a, y) = 0, uy (x, 0) = uy (x, b) = 0,

with separation of variables.


Hint, Solution
Exercise 37.20
Consider the vibrations of a stiff beam of length L. More precisely, consider the transverse vibrations of an unloaded
beam, whose weight can be neglected compared to its stiffness. The beam is simply supported at x = 0, L. (That is,
it is resting on fulcrums there. u(0, t) = 0 means that the beam is resting on the fulcrum; uxx (0, t) = 0 indicates that
there is no bending force at that point.) The beam has initial displacement f (x) and velocity g(x). To determine the
motion of the beam, solve

utt + a2 uxxxx = 0, 0 < x < L, t > 0,


u(x, 0) = f (x), ut (x, 0) = g(x),
u(0, t) = uxx (0, t) = 0, u(L, t) = uxx (L, t) = 0,

with separation of variables.


Hint, Solution
Exercise 37.21
The temperature along a magnet winding of length L carrying a current I satisfies, (for some > 0):

ut = uxx + I 2 u.

1724
The ends of the winding are kept at zero, i.e.,

u(0, t) = u(L, t) = 0;

and the initial temperature distribution is


u(x, 0) = g(x).
Find u(x, t) and determine the critical current ICR which is defined as the least current at which the winding begins to
heat up exponentially. Suppose that < 0, so that the winding has a negative coefficient of resistance with respect to
temperature. What can you say about the critical current in this case?
Hint, Solution
Exercise 37.22
The e-folding time of a decaying function of time is the time interval, e , in which the magnitude of the function
is reduced by at least 1e . Thus if u(x, t) = et f (x) + et g(x) with > > 0 then e = 1 . A body with heat
conductivity has its exterior surface maintained at temperature zero. Initially the interior of the body is at the uniform
temperature T > 0. Find the e-folding time of the body if it is:

a) An infinite slab of thickness a.

b) An infinite cylinder of radius a.

c) A sphere of radius a.

Note that in (a) the temperature varies only in the z direction and in time; in (b) and (c) the temperature varies only
in the radial direction and in time.

u
d) What are the e-folding times if the surfaces are perfectly insulated, (i.e., n
= 0, where n is the exterior normal
at the surface)?

Hint, Solution

1725
Exercise 37.23
Solve the heat equation with a time-dependent diffusivity in the rectangle 0 < x < a, 0 < y < b. The top and bottom
sides are held at temperature zero; the lateral sides are insulated. We have the initial-boundary value problem:

ut = (t) (uxx + uyy ) , 0 < x < a, 0 < y < b, t > 0,


u(x, 0, t) = u(x, b, t) = 0,
ux (0, y, t) = ux (a, y, t) = 0,
u(x, y, 0) = f (x, y).

The diffusivity, (t), is a known, positive function.


Hint, Solution
Exercise 37.24
A semi-circular rod of infinite extent is maintained at temperature T = 0 on the flat side and at T = 1 on the curved
surface:
x2 + y 2 = 1, y > 0.
Find the steady state temperature in a cross section of the rod using separation of variables.
Hint, Solution
Exercise 37.25
Use separation of variables to find the steady state temperature u(x, y) in a slab: x 0, 0 y 1, which has zero
temperature on the faces y = 0 and y = 1 and has a given distribution: u(y, 0) = f (y) on the edge x = 0, 0 y 1.
Hint, Solution
Exercise 37.26
Find the solution of Laplaces equation subject to the boundary conditions.

u = 0, 0 < < , a < r < b,


u(r, 0) = u(r, ) = 0, u(a, ) = 0, u(b, ) = f ().

Hint, Solution

1726
Exercise 37.27

a) A piano string of length L is struck, at time t = 0, by a flat hammer of width 2d centered at a point , having
velocity v. Find the ensuing motion, u(x, t), of the string for which the wave speed is c.
b) Suppose the hammer is curved, rather than flat as above, so that the initial velocity distribution is
(  
(x)
v cos 2d
, |x | < d
ut (x, 0) =
0 |x | > d.

Find the ensuing motion.


c) Compare the kinetic energies of each harmonic in the two solutions. Where should the string be struck in order
to maximize the energy in the nth harmonic in each case?
Hint, Solution

Exercise 37.28
If the striking hammer is not perfectly rigid, then its effect must be included as a time dependent forcing term of the
form:  
(
v cos (x) sin t

2d
, for |x | < d, 0 < t < ,
s(x, t) =
0 otherwise.

Find the motion of the string for t > . Discuss the effects of the width of the hammer and duration of the blow with
regard to the energy in overtones.
Hint, Solution

Exercise 37.29
Find the propagating modes in a square waveguide of side L for harmonic signals of frequency when the propagation
speed of the medium is c. That is, we seek those solutions of

utt c2 u = 0,

1727
where u = u(x, y, z, t) has the form u(x, y, z, t) = v(x, y, z) et , which satisfy the conditions:

u(x, y, z, t) = 0 for x = 0, L, y = 0, L, z > 0,


lim |u| =
6 and 6= 0.
z

Indicate in terms of inequalities involving k = /c and appropriate eigenvalues, n,m say, for which n and m the
solutions un,m satisfy the conditions.
Hint, Solution
Exercise 37.30
Find the modes of oscillation and their frequencies for a rectangular drum head of width a and height b. The modes of
oscillation are eigensolutions of

utt = c2 u, 0 < x < a, 0 < y < b,


u(0, y) = u(a, y) = u(x, 0) = u(x, b) = 0.

Hint, Solution
Exercise 37.31
Using separation of variables solve the heat equation

t = a2 (xx + yy )

in the rectangle 0 < x < lx , 0 < y < ly with initial conditions

(x, y, 0) = 1,

and boundary conditions


(0, y, t) = (lx , y, t) = 0, y (x, 0, t) = y (x, ly , t) = 0.
Hint, Solution

1728
Exercise 37.32
Using polar coordinates and separation of variables solve the heat equation

t = a2 2

in the circle 0 < r < R0 with initial conditions


(r, , 0) = V
where V is a constant, and boundary conditions

(R0 , , t) = 0.

1. Show that for t > 0,



a2 j0,n
2
 
X J0 (j0,n r/R0 )
(r, , t) = 2V exp 2 t ,
n=1
R 0 j 0,n J 1 (j 0,n )
where j0,n are the roots of J0 (x):
J0 (j0,n ) = 0, n = 1, 2, . . .
Hint: The following identities may be of some help:
Z R0
rJ0 (j0,n r/R0 ) J0 (j0,m r/R0 ) dr = 0, m 6= n,
0
Z R0
R2
rJ02 (j0,n r/R0 ) dr = 0 J12 (j0,n ),
2
Z0 r
r
rJ0 (r)dr = J1 (r) for any .
0

2. For any fixed r, 0 < r < R0 , use the asymptotic approximation for the Jn Bessel functions for large argument
(this can be found in any standard math tables) to determine the rate of decay of the terms of the series solution
for at time t = 0.
Hint, Solution

1729
Exercise 37.33
Consider the solution of the diffusion equation in spherical coordinates given by

x = r sin cos ,
y = r sin sin ,
z = r cos ,

where r is the radius, is the polar angle, and is the azimuthal angle. We wish to solve the equation on the surface
of the sphere given by r = R, 0 < < , and 0 < < 2. The diffusion equation for the solution (, , t) in these
coordinates on the surface of the sphere becomes
a2 1 2
   
1
= 2 sin + . (37.7)
t R sin sin2 2
where a is a positive constant.
1. Using separation of variables show that a solution can be found in the form

(, , t) = T (t)()(),

where T ,, obey ordinary differential equations in t,, and respectively. Derive the ordinary differential
equations for T and , and show that the differential equation obeyed by is given by
d2
c = 0,
d2
where c is a constant.

2. Assuming that (, , t) is determined over the full range of the azimuthal angle, 0 < < 2, determine the
allowable values of the separation constant c and the corresponding allowable functions . Using these values
of c and letting x = cos rewrite in terms of the variable x the differential equation satisfied by . What are
appropriate boundary conditions for ? The resulting equation is known as the generalized or associated Legendre
equation.

1730
3. Assume next that the initial conditions for are chosen such that

(, , t = 0) = f (),

where f () is a specified function which is regular at the north and south poles (that is = 0 and = ).
Note that the initial condition is independent of the azimuthal angle . Show that in this case the method of
separation of variables gives a series solution for of the form

X
(, t) = Al exp(2l t)Pl (cos ),
l=0

where Pl (x) is the lth Legendre polynomial, and determine the constants l as a function of the index l.

4. Solve for (, t), t > 0 given that f () = 2 cos2 1.


Useful facts:  
d 2 dPl (x)
(1 x ) + l(l + 1)Pl (x) = 0
dx dx

P0 (x) = 1
P1 (x) = x
3 2 1
P2 (x) = x
2 2
if l 6= m
(
Z 1 0
dxPl (x)Pm (x) = 2
1 if l = m
2l + 1
Hint, Solution
Exercise 37.34
Let (x, y) satisfy Laplaces equation
xx + yy = 0

1731
in the rectangle 0 < x < 1, 0 < y < 2, with (x, 2) = x(1 x), and with = 0 on the other three sides. Use a series
solution to determine inside the rectangle. How many terms are required to give ( 21 , 1) with about 1% (also 0.1%)
accuracy; how about x ( 12 , 1)?
Hint, Solution
Exercise 37.35
Let (r, , ) satisfy Laplaces equation in spherical coordinates in each of the two regions r < a, r > a, with 0
as r . Let

lim (r, , ) lim (r, , ) = 0,


ra+ ra
lim r (r, , ) lim r (r, , ) = Pnm (cos ) sin(m),
ra+ ra

where m and n m are integers. Find in r < a and r > a. In electrostatics, this problem corresponds to that of
determining the potential of a spherical harmonic type charge distribution over the surface of the sphere. In this way
one can determine the potential due to an arbitrary surface charge distribution since any charge distribution can be
expressed as a series of spherical harmonics.
Hint, Solution
Exercise 37.36
Obtain a formula analogous to the Poisson formula to solve the Neumann problem for the circular region 0 r < R,
0 < 2. That is, determine a solution (r, ) to Laplaces equation

2 = 0

in polar coordinates given r (R, ). Show that

R 2 r2
Z  
2r
(r, ) = r (R, ) ln 1 cos( ) + 2 d
2 0 R R

within an arbitrary additive constant.


Hint, Solution

1732
Exercise 37.37
Investigate solutions of
t = a2 xx
obtained by setting the separation constant C = ( + )2 in the equations obtained by assuming = X(x)T (t):

T0 X 00 C
= C, = 2.
T X a
Hint, Solution

1733
37.9 Hints
Hint 37.1

Hint 37.2

Hint 37.3

Hint 37.4

Hint 37.5

Hint 37.6

Hint 37.7
Impose the boundary conditions
u(0, t) = u(2, t), u (0, t) = u (2, t).

Hint 37.8
Apply the separation of variables u(x, y) = X(x)Y (y). Solve an eigenvalue problem for X(x).

Hint 37.9

Hint 37.10

1734
Hint 37.11

Hint 37.12
There are two ways to solve the problem. For the first method, expand the solution in a series of the form


X  nx 
u(x, t) = an (t) sin .
n=1
L

Because of the inhomogeneous boundary conditions, the convergence of the series will not be uniform. You can
differentiate the series with respect to t, but not with respect to x. Multiply the partial differential equation by the
eigenfunction sin(nx/L) and integrate from x = 0 to x = L. Use integration by parts to move derivatives in x from
u to the eigenfunctions. This process will yield a first order, ordinary differential equation for each of the an s.
For the second method: Make the change of variables v(x, t) = u(x, t) (x), where (x) is the equilibrium
temperature distribution to obtain a problem with homogeneous boundary conditions.

Hint 37.13

Hint 37.14

Hint 37.15

Hint 37.16

Hint 37.17

1735
Hint 37.18
Use separation of variables to find eigen-solutions of the partial differential equation that satisfy the homogeneous
boundary conditions. There will be two eigen-solutions for each eigenvalue. Expand u(x, t) in a series of the eigen-
solutions. Use the two initial conditions to determine the constants.
Hint 37.19
Expand the solution in a series of eigenfunctions in x. Determine these eigenfunctions by using separation of variables
on the homogeneous partial differential equation. You will find that the answer has the form,

X  nx 
u(x, y) = un (y) sin .
n=1
a

Substitute this series into the partial differential equation to determine ordinary differential equations for each of the
un s. The boundary conditions on u(x, y) will give you boundary conditions for the un s. Solve these ordinary differential
equations with Green functions.
Hint 37.20
Solve this problem by expanding the solution in a series of eigen-solutions that satisfy the partial differential equation
and the homogeneous boundary conditions. Use the initial conditions to determine the coefficients in the expansion.
Hint 37.21
Use separation of variables to find eigen-solutions that satisfy the partial differential equation and the homogeneous
boundary conditions. The solution is a linear combination of the eigen-solutions. The whole solution will be exponentially
decaying if each of the eigen-solutions is exponentially decaying.
Hint 37.22
For parts (a), (b) and (c) use separation of variables. For part (b) the eigen-solutions will involve Bessel functions. For
part (c) the eigen-solutions will involve spherical Bessel functions. Part (d) is trivial.

Hint 37.23
The solution is a linear combination of eigen-solutions of the partial differential equation that satisfy the homogeneous
boundary conditions. Determine the coefficients in the expansion with the initial condition.

1736
Hint 37.24
The problem is
1 1
urr + ur + 2 u = 0, 0 < r < 1, 0 < <
r r
u(r, 0) = u(r, ) = 0, u(0, ) = 0, u(1, ) = 1
The solution is a linear combination of eigen-solutions that satisfy the partial differential equation and the three
homogeneous boundary conditions.
Hint 37.25

Hint 37.26

Hint 37.27

Hint 37.28

Hint 37.29

Hint 37.30

Hint 37.31

Hint 37.32

1737
Hint 37.33

Hint 37.34

Hint 37.35

Hint 37.36

Hint 37.37

1738
37.10 Solutions

Solution 37.1
We expand the solution in eigenfunctions in x and y which satify the boundary conditions.


X  mx   ny 
u= umn (t) sin sin
m,n=1
a b

We expand the inhomogeneities in the eigenfunctions.


X  mx   ny 
q(x, y, t) = qmn (t) sin sin
m,n=1
a b
Z a Z b
4  mx   ny 
qmn (t) = q(x, y, t) sin sin dy dx
ab 0 0 a b

X  mx   ny 
f (x, y) = fmn sin sin
m,n=1
a b
Z aZ b
4  mx   ny 
fmn = f (x, y) sin sin dy dx
ab 0 0 a b

1739
We substitute the expansion of the solution into the diffusion equation and the initial condition to determine initial
value problems for the coefficients in the expansion.

ut (uxx + uyy ) = q(x, y, t)


   
X
0 m 2  n 2  mx   ny  X  mx   ny 
umn (t) + + umn (t) sin sin = qmn (t) sin sin
m,n=1
a b a b m,n=1
a b
 
m 2  n 2
u0mn (t) + + umn (t) = qmn (t)
a b
u(x, y, 0) = f (x, y)

X  mx   ny  X  mx   ny 
umn (0) sin sin = fmn sin sin
m,n=1
a b m,n=1
a b
umn (0) = fmn

We solve the ordinary differential equations for the coefficients umn (t) subject to their initial conditions.

Z t        
m 2  n 2 m 2  n 2
umn (t) = exp + (t ) qmn ( ) d + fmn exp + t
0 a b a b

Solution 37.2
After looking at this problem for a minute or two, it seems like the answer would have the form

u = sin(x)T (t).

This form satisfies the boundary conditions. We substitute it into the heat equation and the initial condition to determine

1740
T

sin(x)T 0 = sin(x)T + A sin(x), T (0) = 0


T 0 + T = A, T (0) = 0
A
T = + c et

A
1 et

T =

Now we have the solution of the heat equation.

A
sin(x) 1 et

u=

Solution 37.3
First we write the Laplacian in polar coordinates.

1 1
urr + ur + 2 u = 0
r r
1. We introduce the separation of variables u(r, ) = R(r)().

1 1
R00 + R0 + 2 R00 = 0
r r
00 0
R R 00
r2 +r = =
R R
We have a regular Sturm-Liouville problem for and a differential equation for R.

00 + = 0, 0 (0) = (/2) = 0 (37.8)


r2 R00 + rR0 R = 0, R is bounded

1741
First we solve the problem for to determine the eigenvalues and eigenfunctions. The Rayleigh quotient is
R /2
(0 )2 d
= 0R /2
0
2 d

Immediately we see that the eigenvalues are non-negative. If 0 = 0, then the right boundary condition implies
that = 0. Thus = 0 is not an eigenvalue. We find the general solution of Equation 37.8 for positive .
   
= c1 cos + c2 sin

The solution that satisfies the left boundary condition is


 
= c cos .

We apply the right boundary condition to determine the eigenvalues.


 
cos =0
2
n = (2n 1)2 , n = cos ((2n 1)) , n Z+

Now we solve the differential equation for R. Since this is an Euler equation, we make the substitition R = r .

r2 Rn00 + rRn0 (2n 1)2 Rn = 0


( 1) + (2n 1)2 = 0
= (2n 1)
Rn = c1 r2n1 + c2 r12n

The solution which is bounded in 0 r 1 is

Rn = r2n1 .

1742
The solution of Laplaces equation is a linear combination of the eigensolutions.

X
u= un r2n1 cos ((2n 1))
n=1

We use the boundary condition at r = 1 to determine the coefficients.



X
u(1, ) = f () = un cos ((2n 1))
n=1
Z /2
4
un = f () cos ((2n 1)) d
0

2. We introduce the separation of variables u(r, ) = R(r)().


1 1
R00 + R0 + 2 R00 = 0
r r
00 0
2R R 00
r +r = =
R R
We have a regular Sturm-Liouville problem for and a differential equation for R.

00 + = 0, 0 (0) = 0 (/2) = 0 (37.9)


r2 R00 + rR0 R = 0, R is bounded

First we solve the problem for to determine the eigenvalues and eigenfunctions. We recognize this problem as
the generator of the Fourier cosine series.

n = (2n)2 , n Z0+ ,
1
0 = , n = cos (2n) , n Z+
2

1743
Now we solve the differential equation for R. Since this is an Euler equation, we make the substitition R = r .

r2 Rn00 + rRn0 (2n)2 Rn = 0


( 1) + (2n)2 = 0
= 2n
R0 = c1 + c2 ln(r), Rn = c1 r2n + c2 r2n , n Z+

The solutions which are bounded in 0 r 1 are

Rn = r2n .

The solution of Laplaces equation is a linear combination of the eigensolutions.



u0 X
u= + un r2n cos (2n)
2 n=1

We use the boundary condition at r = 1 to determine the coefficients.



X
ur (1, ) = 2nun cos(2n) = g()
n=1

Note that the constant term is missing in this cosine series. g() has such a series expansion only if
Z /2
g() d = 0.
0

This is the condition for the existence of a solution of the problem. If this is satisfied, we can solve for the
coefficients in the expansion. u0 is arbitrary.

4 /2
Z
un = g() cos (2n) d, n Z+
0

1744
Solution 37.4
1.

ut = (uxx + uyy )
XY T 0 = (X 00 Y T + XY 00 T )
T0 X 00 Y 00
= + =
T X Y
X 00 Y 00
= =
X Y
We have boundary value problems for X(x) and Y (y) and a differential equation for T (t).

X 00 + X = 0, X 0 (0) = X 0 (1) = 0
Y 00 + ( )Y = 0, Y (0) = Y (1) = 0
T 0 = T

2. The solutions for X(x) form a cosine series.


1
m = m 2 2 , m Z0+ , X0 = , Xm = cos(mx)
2
The solutions for Y (y) form a sine series.

mn = (m2 + n2 ) 2 , n Z+ , Yn = sin(nx)

We solve the ordinary differential equation for T (t).


2 +n2 ) 2 t
Tmn = e(m

We expand the solution of the heat equation in a series of the eigensolutions.


X
1X n2 2 t
X 2 2 2
u(x, y, t) = u0n sin(ny) e + umn cos(mx) sin(ny) e(m +n ) t
2 n=1 m=1 n=1

1745
We use the initial condition to determine the coefficients.
X
1X X
u(x, y, 0) = f (x, y) = u0n sin(ny) + umn cos(mx) sin(ny)
2 n=1 m=1 n=1
Z 1Z 1
umn = 4 f (x, y) cos(mx) sin(ny) dx dy
0 0

Solution 37.5
We use the separation of variables u(x, t) = X(x)T (t) to find eigensolutions of the heat equation that satisfy the
boundary conditions at x = 0, .

ut = uxx
XT 0 = X 00 T
T0 X 00
= =
T X
The problem for X(x) is
X 00 + X = 0, X 0 (0) = X 0 () = 0.
The eigenfunctions form the familiar cosine series.
1
n = n 2 , n Z0+ , X0 = , Xn = cos(nx)
2
Next we solve the differential equation for T (t).

Tn0 = n2 Tn
2t
T0 = 1, Tn = en

We expand the solution of the heat equation in a series of the eigensolutions.



1 X 2
u(x, t) = u0 + un cos(nx) en t
2 n=1

1746
We use the initial condition to determine the coefficients in the series.

1 X
u(x, 0) = x = u0 + un cos(nx)
2 n=1
2
Z
u0 = x dx =
0
(
2
Z
0 even n
un = x cos(nx) dx = 4
0 n2 odd n

X 4 2
u(x, t) = 2
cos(nx) en t
2 n=1
n
odd n

Solution 37.6
We expand the solution in a Fourier series.

1 X X
= a0 (r) + an (r) cos(n) + bn (r) sin(n)
2 n=1 n=1

We substitute the series into the Laplaces equation to determine ordinary differential equations for the coefficients.
1 2
 

r + 2 2 =0
r r r
1 1 1
a000 + a00 = 0, a00n + a0n n2 an = 0, b00n + b0n n2 bn = 0
r r r
The solutions that are bounded at r = 0 are, (to within multiplicative constants),
a0 (r) = 1, an (r) = rn , bn (r) = rn .
Thus (r, ) has the form

1 X
n
X
(r, ) = c0 + cn r cos(n) + dn rn sin(n)
2 n=1 n=1

1747
We apply the boundary condition at r = R.

1 X
n
X
(R, ) = c0 + cn R cos(n) + dn Rn sin(n)
2 n=1 n=1

The coefficients are


1 2 2 2
Z Z Z
1 1
c0 = (R, ) d, cn = (R, ) cos(n) d, dn = (R, ) sin(n) d.
0 Rn 0 Rn 0

We substitute the coefficients into our series solution.


Z 2
1 X  r n 2
Z
1
(r, ) = (R, ) d + (R, ) cos(n( )) d
2 0 n=1 R 0
 
Z 2 !
1 2
Z
1 X r n n()
(r, ) = (R, ) d + (R, )< e d
2 0 0 n=1
R
!
Z 2 r ()
1 2 e
Z
1 R
(r, ) = (R, ) d + (R, )< d
2 0 0 1 Rr e()
r 2
!
r ()
Z 2 
1 2 e
Z
1 R R
(r, ) = (R, ) d + (R, )< 2 d
2 0 0 r
1 2 R cos( ) + Rr
Z 2
1 2 Rr cos( ) r2
Z
1
(r, ) = (R, ) d + (R, ) 2 d
2 0 0 R + r2 2Rr cos( )
Z 2
1 R2 r 2
(r, ) = (R, ) 2 d
2 0 R + r2 2Rr cos( )
Solution 37.7
In order that the solution is continuously differentiable, (which it must be in order to satisfy the differential equation),
we impose the boundary conditions
u(0, t) = u(2, t), u (0, t) = u (2, t).

1748
We apply the separation of variables u(, t) = ()T (t).
ut = u
T 0 = 00 T
T0 00
= =
T
We have the self-adjoint eigenvalue problem

00 + = 0, (0) = (2), 0 (0) = 0 (2)

which has the eigenvalues and orthonormal eigenfunctions


1
n = n 2 , n = en , n Z.
2
Now we solve the problems for Tn (t) to obtain eigen-solutions of the heat equation.
Tn0 = n2 Tn
2 t
Tn = en
The solution is a linear combination of the eigen-solutions.

X 1 2
u(, t) = un en en t
n=
2

We use the initial conditions to determine the coefficients.



X 1
u(, 0) = un en = f ()
n=
2
Z 2
1
un = en f () d
2 0

1749
Solution 37.8
Substituting u(x, y) = X(x)Y (y) into the partial differential equation yields
X 00 Y 00
= = .
X Y
With the homogeneous boundary conditions, we have the two problems
X 00 + X = 0, X(0) = X(1) = 0,
Y 00 Y = 0, Y (1) = 0.
The eigenvalues and orthonormal eigenfunctions for X(x) are

n = (n)2 , Xn = 2 sin(nx).
The general solution for Y is
Yn = a cosh(ny) + b sinh(ny).
The solution for that satisfies the right homogeneous boundary condition, (up to a multiplicative constant), is
Yn = sinh(n(1 y))
u(x, y) is a linear combination of the eigen-solutions.

X
u(x, y) = un 2 sin(nx) sinh(n(1 y))
n=1

We use the inhomogeneous boundary condition to determine coefficients.



X
u(x, 0) = un 2 sin(nx) sinh(n) = f (x)
n=1

Z 1
un = 2 sin(n)f () d
0

1750
Solution 37.9
We substitute u(r, ) = R(r)() into the partial differential equation.

2 u 1 u 1 2u
+ + =0
r2 r r r2 2
1 1
R00 + R0 + 2 R00 = 0
r r
00 0
R R 00
r2 +r = =
R R
r2 R00 + rR0 R = 0, 00 + = 0

We assume that u is a strong solution of the partial differential equation and is thus twice continuously differentiable,
(u C 2 ). In particular, this implies that R and are bounded and that is continuous and has a continuous first
derivative along = 0. This gives us a boundary value problem for and a differential equation for R.

00 + = 0, (0) = (2), 0 (0) = 0 (2)


r2 R00 + rR0 R = 0, R is bounded

The eigensolutions for form the familiar Fourier series.

n = n 2 , n Z0+
(1) 1
0 = , (1) n = cos(n), n Z+
2
(2)
n = sin(n), n Z+

Now we find the bounded solutions for R. The equation for R is an Euler equation so we use the substitution
R = r .

r2 Rn00 + rRn0 n Rn = 0
( 1) + n = 0
p
= n

1751
First we consider the case 0 = 0. The solution is

R = a + b ln r.

Boundedness demands that b = 0. Thus we have the solution

R = 1.

Now we consider the case n = n2 > 0. The solution is

Rn = arn + brn .

Boundedness demands that b = 0. Thus we have the solution

Rn = r n .

The solution for u is a linear combination of the eigensolutions.



a0 X
u(r, ) = + (an cos(n) + bn sin(n)) rn
2 n=1

The boundary condition at r = 1 determines the coefficients in the expansion.



a0 X
u(1, ) = + [an cos(n) + bn sin(n)] = f ()
2 n=1
1 2 1 2
Z Z
an = f () cos(n) d, bn = f () sin(n) d
0 0
Solution 37.10
A normal mode of frequency is periodic in time.

v(r, , t) = u(r, ) et

1752
We substitute this form into the wave equation to obtain a Helmholtz equation, (also called a reduced wave equation).

1 2u 2
 
1 u
r + 2 2 = 2 u, u(1, ) = 0,
r r r r c
2 u 1 u 1 2u
+ + + k 2 u = 0, u(1, ) = 0
r2 r r r2 2
Here we have defined k = c . We apply the separation of variables u = R(r)() to the Helmholtz equation.

r2 R00 + rR0 + R00 + k 2 r2 R = 0,


R00 R0 00
r2 + r + k2 r2 = = 2
R R
Now we have an ordinary differential equation for R(r) and an eigenvalue problem for ().

2
 
00 1 0 2
R + R + k 2 R = 0, R(0) is bounded, R(1) = 0,
r r
+ = 0, () = (), 0 () = 0 ().
00 2

We compute the eigenvalues and eigenfunctions for .

n = n, n Z0+
1
0 = , (1) n = cos(n), (2)
n = sin(n), n Z+
2
The differential equations for the Rn are Bessel equations.
n2
 
00 1 0 2
Rn + Rn + k 2 Rn = 0, Rn (0) is bounded, Rn (1) = 0
r r
The general solution is a linear combination of order n Bessel functions of the first and second kind.

Rn (r) = c1 Jn (kr) + c2 Yn (kr)

1753
Since the Bessel function of the second kind, Yn (kr), is unbounded at r = 0, the solution has the form

Rn (r) = cJn (kr).

Applying the second boundary condition gives us the admissable frequencies.

Jn (k) = 0
knm = jnm , Rnm = Jn (jnm r), n Z0+ , m Z+

Here jnm is the mth positive root of Jn . We combining the above results to obtain the normal modes of oscillation.
1
v0m = J0 (j0m r) ecj0m t , m Z+
2
vnm = cos(n + )Jnm (jnm r) ecjnm t , n, m Z+

Some normal modes are plotted in Figure 37.2. Note that cos(n + ) represents a linear combination of cos(n) and
sin(n). This form is preferrable as it illustrates the circular symmetry of the problem.

Solution 37.11
We will expand the solution in a complete, orthogonal set of functions {Xn (x)}, where the coefficients are functions
of t. X
= Tn (t)Xn (x)
n

We will use separation of variables to determine a convenient set {Xn }. We substitite = T (t)X(x) into the diffusion
equation.

t = a2 xx
XT 0 = a2 X 00 T
T0 X 00
= =
a2 T X
T 0 = a2 T, X 00 + X = 0

1754
Note that in order to satisfy (0, t) = (l, t) = 0, the Xn must satisfy the same homogeneous boundary conditions,
Xn (0) = Xn (l) = 0. This gives us a Sturm-Liouville problem for X(x).

X 00 + X = 0, X(0) = X(l) = 0
 n 2  nx 
n = , Xn = sin , n Z+
l l

Thus we seek a solution of the form


X  nx 
= Tn (t) sin . (37.10)
n=1
l

This solution automatically satisfies the boundary conditions. We will assume that we can differentiate it. We will
substitite this form into the diffusion equation and the initial condition to determine the coefficients in the series, Tn (t).
First we substitute Equation 37.10 into the partial differential equation for to determine ordinary differential equations
for the Tn .

t = a2 xx
 nx  
X
0 2
X n 2  nx 
Tn (t) sin = a Tn (t) sin
n=1
l n=1
l l
 an 2
Tn0 = Tn
l

1755
Now we substitute Equation 37.10 into the initial condition for to determine initial conditions for the Tn .
X  nx 
Tn (0) sin = (x, 0)
n=1
l
Rl
sin nx

0 l
(x, 0) dx
Tn (0) = R l 2 nx 
0
sin l
dx
Z l
2  nx 
Tn (0) = sin (x, 0) dx
l 0 l
2 l/2 2 l/2
Z  nx  Z  nx 
Tn (0) = sin x dx + sin (l x) dx
l 0 l l 0 l
4l  n 
Tn (0) = 2 2 sin
n 2
4l
T2n1 (0) = (1)n , T2n (0) = 0, n Z+
(2n 1)2 2
We solve the ordinary differential equations for Tn subject to the initial conditions.
 2 !
4l a(2n 1)
T2n1 (t) = (1)n exp t , T2n (t) = 0, n Z+
(2n 1)2 2 l
This determines the series representation of the solution.
 2  2 !  
4X l a(2n 1) (2n 1)x
= (1)n exp t sin
l n=1 (2n 1) l l

From the initial condition, we know that the the solution at t = 0 is C 0 . That is, it is continuous, but not
differentiable. The series representation of the solution at t = 0 is
 2  
4X n l (2n 1)x
= (1) sin .
l n=1 (2n 1) l

1756
That the coefficients decay as 1/n2 corroborates that (x, 0) is C 0 .
The derivatives of with respect to x are
2m3 2 !
2m1 4(1)m+1 X
   
n (2n 1) a(2n 1) (2n 1)x
= (1) exp t cos
x2m1 l n=1
l l l
2m2 2 !
2m 4(1)m X
   
n (2n 1) a(2n 1) (2n 1)x
= (1) exp t sin
x2m l n=1
l l l
n
For any fixed t > 0, the coefficients in the series for x
decay exponentially. These series are uniformly convergent in
x. Thus for any fixed t > 0, is C in x.
Solution 37.12

ut = uxx , 0 < x < L, t > 0


u(0, t) = T0 , u(L, t) = T1 , u(x, 0) = f (x),

Method 1. We solve this problem with an eigenfunction expansion in x. To find an appropriate set of eigenfunctions,
we apply the separation of variables, u(x, t) = X(x)T (t) to the partial differential equation with the homogeneous
boundary conditions, u(0, t) = u(L, t) = 0.

(XT )t = (XT )xx


XT 0 = X 00 T
T0 X 00
= = 2
T X
We have the eigenvalue problem,
X 00 + 2 X = 0, X(0) = X(L) = 0,
which has the solutions,
nx  nx 
n = , Xn = sin , n N.
L L

1757
We expand the solution of the partial differential equation in terms of these eigenfunctions.


X  nx 
u(x, t) = an (t) sin
n=1
L

Because of the inhomogeneous boundary conditions, the convergence of the series will not be uniform. We can
differentiate the series with respect to t, but not with respect to x. We multiply the partial differential equation by
an eigenfunction and integrate from x = 0 to x = L. We use integration by parts to move derivatives from u to the
eigenfunction.

ut uxx = 0
Z L  mx 
(ut uxx ) sin dx = 0
0 L

Z L X !
m L
 nx   mx  h  mx iL Z  mx 
0
an (t) sin sin dx ux sin + ux cos dx = 0
0 n=1
L L L 0 L 0 L
L 0 m h  mx iL  m 2 Z L  mx 
am (t) + u cos + u sin dx = 0
2 L L 0 L 0 L

!
L 0 m  m 2 Z L X  nx   mx 
am (t) + ((1)m u(L, t) u(0, t)) + an (t) sin sin dx = 0
2 L L 0 n=1
L L
L 0 m L  m 2
am (t) + ((1)m T1 T0 ) + am (t) = 0
2 L 2 L
 m 2 2m
a0m (t) + am (t) = 2 (T0 (1)m T1 )
L L

1758
Now we have a first order differential equation for each of the an s. We obtain initial conditions for each of the an s
from the initial condition for u(x, t).

u(x, 0) = f (x)

X  nx 
an (0) sin = f (x)
n=1
L
2 L
Z  nx 
an (0) = f (x) sin dx fn
L 0 L

By solving the first order differential equation for an (t), we obtain

2(T0 (1)n T1 ) 2(T0 (1)n T1 )


 
(n/L)2 t
an (t) = +e fn .
n n

Note that the series does not converge uniformly due to the 1/n term.
Method 2. For our second method we transform the problem to one with homogeneous boundary conditions so
that we can use the partial differential equation to determine the time dependence of the eigen-solutions. We make the
change of variables v(x, t) = u(x, t) (x) where (x) is some function that satisfies the inhomogeneous boundary
conditions. If possible, we want (x) to satisfy the partial differential equation as well. For this problem we can choose
(x) to be the equilibrium solution which satisfies

00 (x) = 0, (0)T0 , (L) = T1 .

This has the solution


T1 T0
(x) = T0 + x.
L
With the change of variables,  
T1 T0
v(x, t) = u(x, t) T0 + x ,
L

1759
we obtain the problem

vt = vxx , 0 < x < L, t>0


 
T1 T0
v(0, t) = 0, v(L, t) = 0, v(x, 0) = f (x) T0 + x .
L
Now we substitute the separation of variables v(x, t) = X(x)T (t) into the partial differential equation.

(XT )t = (XT )xx


T0 X 00
= = 2
T X
Utilizing the boundary conditions at x = 0, L we obtain the two ordinary differential equations,

T 0 = 2 T,
X 00 = 2 X, X(0) = X(L) = 0.

The problem for X is a regular Sturm-Liouville problem and has the solutions
n  nx 
n = , Xn = sin , n N.
L L
The ordinary differential equation for T becomes,
 n 2
Tn0 = Tn ,
L
which, (up to a multiplicative constant), has the solution,
2
Tn = e(n/L) t .

Thus the eigenvalues and eigen-solutions of the partial differential equation are,
n  nx  2
n = , vn = sin e(n/L) t , n N.
L L

1760
Let v(x, t) have the series expansion,
 nx  2
X
v(x, t) = an sin e(n/L) t .
n=1
L

We determine the coefficients in the expansion from the initial condition,


 
X  nx  T1 T0
v(x, 0) = an sin = f (x) T0 + x .
n=1
L L

The coefficients in the expansion are the Fourier sine coefficients of f (x) T0 + T1 T

L
0
x .

2 L
Z   
T1 T0  nx 
an = f (x) T0 + x sin dx
L 0 L L
2(T0 (1)n T1 )
an = fn
n
With the coefficients defined above, the solution for u(x, t) is

2(T0 (1)n T1 )

T1 T0 X  nx  2
u(x, t) = T0 + x+ fn sin e(n/L) t .
L n=1
n L

Since the coefficients in the sum decay exponentially for t > 0, we see that the series is uniformly convergent for positive
t. It is clear that the two solutions we have obtained are equivalent.
Solution 37.13
First we solve the eigenvalue problem for (x), which is the problem we would obtain if we applied separation of
variables to the partial differential equation, t = xx . We have the eigenvalues and orthonormal eigenfunctions
 2 r  
(2n 1) 2 (2n 1)x
n = , n (x) = sin , n Z+ .
2l l 2l

1761
We expand the solution and inhomogeneity in Equation 37.5 in a series of the eigenvalues.

X
(x, t) = Tn (t)n (x)
n=1

X Z l
w(x, t) = wn (t)n (x), wn (t) = n (x)w(x, t) dx
n=1 0

Since satisfies the same homgeneous boundary conditions as , we substitute the series into Equation 37.5 to
determine differential equations for the Tn (t).

X
X
X
Tn0 (t)n (x) = a2 Tn (t)(n )n (x) + wn (t)n (x)
n=1 n=1 n=1
 2
(2n 1)
Tn0 (t) = a2 Tn (t) + wn (t)
2l
Now we substitute the series for into its initial condition to determine initial conditions for the Tn .

X
(x, 0) = Tn (0)n (x) = f (x)
n=1
Z l
Tn (0) = n (x)f (x) dx
0

We solve for Tn (t) to determine the solution, (x, t).


 2 ! Z t  2 ! !
(2n 1)a (2n 1)a
Tn (t) = exp t Tn (0) + wn ( ) exp d
2l 0 2l

Solution 37.14
Separation of variables leads to the eigenvalue problem
00 + = 0, (0) = 0, (l) + c 0 (l) = 0.

1762
First we consider the case = 0. A set of solutions of the differential equation is {1, x}. The solution that satisfies
the left boundary condition is (x) = x. The right boundary condition imposes the constraint l + c = 0. Since c is
positive, this has no solutions. = 0 is not an eigenvalue.
Now we consider 6= 0. A set of solutions ofthe differential equation is {cos( x), sin( x)}. The solution that
satisfies the left boundary condition is = sin( x). The right boundary condition imposes the constraint
   
c sin l + cos l = 0
 

tan l =
c

For large , the we can determine approximate solutions.

p (2n 1)
n l , n Z+
2
 2
(2n 1)
n , n Z+
2l

The eigenfunctions are



sin n x
n (x) = qR , n Z+ .
l
sin2

0
n x dx

We expand (x, t) and w(x, t) in series of the eigenfunctions.


X
(x, t) = Tn (t)n (x)
n=1

X Z l
w(x, t) = wn (t)n (x), wn (t) = n (x)w(x, t) dx
n=1 0

1763
Since satisfies the same homgeneous boundary conditions as , we substitute the series into Equation 37.5 to
determine differential equations for the Tn (t).

X
X
X
Tn0 (t)n (x) =a 2
Tn (t)(n )n (x) + wn (t)n (x)
n=1 n=1 n=1
Tn0 (t) = a2 n Tn (t) + wn (t)

Now we substitute the series for into its initial condition to determine initial conditions for the Tn .

X
(x, 0) = Tn (0)n (x) = f (x)
n=1
Z l
Tn (0) = n (x)f (x) dx
0

We solve for Tn (t) to determine the solution, (x, t).


 Z t 
2 2
 
Tn (t) = exp a n t Tn (0) + wn ( ) exp a n d
0

Solution 37.15
First we seek a function u(x, t) that satisfies the boundary conditions u(0, t) = t, ux (l, t) = cu(l, t). We try a
function of the form u = (ax + b)t. The left boundary condition imposes the constraint b = 1. We then apply the
right boundary condition no determine u.

at = c(al + 1)t
c
a=
 1 + cl 
cx
u(x, t) = 1 t
1 + cl

1764
Now we define to be the difference of and u.

(x, t) = (x, t) u(x, t)

satisfies an inhomogeneous diffusion equation with homogeneous boundary conditions.

( + u)t = a2 ( + u)xx + 1
t = a2 xx + 1 + a2 uxx ut
cx
t = a2 xx +
1 + cl

The initial and boundary conditions for are

(x, 0) = 0, (0, t) = 0, x (l, t) = c(l, t).

We solved this system in problem 2. Just take


cx
w(x, t) = , f (x) = 0.
1 + cl

The solution is

X
(x, t) = Tn (t)n (x),
n=1
Z t
wn exp a2 n (t ) d,

Tn (t) =
0
Z l
cx
wn (t) = n (x) dx.
0 1 + cl

This determines the solution for .

1765
Solution 37.16
First we solve this problem with a series expansion. We transform the problem to one with homogeneous boundary
conditions. Note that
x
u(x) =
l+1

satisfies the boundary conditions. (It is the equilibrium solution.) We make the change of variables = u. The
problem for is

t = a2 xx ,
x
(0, t) = (l, t) + x (l, t) = 0, (x, 0) = .
l+1

This is a particular case of what we solved in Exercise 37.14. We apply the result of that problem. The solution for
(x, t) is


x X
(x, t) = + Tn (t)n (x)
l + 1 n=1

sin n x
n (x) = qR , n Z+
l 2

0
sin n x dx
 
tan l =
Tn (t) = Tn (0) exp a2 n t

Z l
x
Tn (0) = n (x) dx
0 l+1

This expansion is useful for large t because the coefficients decay exponentially with increasing t.

1766
Now we solve this problem with the Laplace transform.

t = a2 xx , (0, t) = 0, (l, t) + x (l, t) = 1, (x, 0) = 0


1
s = a2 xx , (0, s) = 0, (l, s) + x (l, s) =
s
s 1
xx 2 = 0, (0, s) = 0, (l, s) + x (l, s) =
a s

The solution that satisfies the left boundary condition is

 
sx
= c sinh .
a

We apply the right boundary condition to determine the constant.

 
sinh asx
=     
s sinh asl + as cosh asl

1767
We expand this in a series of simpler functions of s.
 
2 sinh asx
=          
s exp asl exp asl + as exp asl + exp asl
 
2 sinh asx 1
=      
sl
s exp a 1 + a 1 a exp 2 asl
s s

   
exp asx exp asx 1
=        
s 1 + as exp asl 1 1
1+ s/a
s/a
exp 2 asl
   
exp s(xl)
exp s(xl)  n  
a a X 1 s/a 2 sln
=   exp
s 1+ s
n=0
1 + s/a a
a


(1 s/a)n

1 X s((2n + 1)l x)
= exp
s n=0
(1 + s/a)n+1 a
 !
X (1 s/a)n s((2n + 1)l + x)
n+1
exp
n=0
(1 + s/a) a

By expanding
(1 s/a)n

(1 + s/a)n+1
in binomial series all the terms would be of the form
 
m/23/2 s((2n 1)l x)
s exp .
a

1768
Taking the first term in each series yields
    
a s(l x) s(l + x)
3/2 exp exp , as s .
s a a
We take the inverse Laplace transform to obtain an appoximation of the solution for t  1.
   
(lx)2 (l+x)2

exp 4a2 t
exp 4a2 t
(x, t) 2a2 t
lx l+x
    
lx l+x
erfc erfc , for t  1
2a t 2a t
Solution 37.17
We apply the separation of variables (x, t) = X(x)T (t).
t = A2 x2 x

x
0 2 2 0 0

XT = T A x X
T0 (x2 X 0 )0
= =
A2 T X
This gives us a regular Sturm-Liouville problem.
0
x2 X 0 + X = 0, X(1) = X(2) = 0
This is an Euler equation. We make the substitution X = x to find the solutions.
x2 X 00 + 2xX 0 + X = 0 (37.11)
( 1) + 2 + = 0

1 1 4
=
2
1 p
= 1/4
2

1769
First we consider the case of a double root when = 1/4. The solutions of Equation 37.11 are {x1/2 , x1/2 ln x}.
The solution that satisfies the left boundary condition is X = x1/2 ln x. Since this does not satisfy the right boundary
condition, = 1/4 is not an eigenvalue.
Now we consider 6= 1/4. The solutions of Equation 37.11 are
 
1 p  1 p
cos 1/4 ln x , sin 1/4 ln x .
x x
The solution that satisfies the left boundary condition is
1 p 
sin 1/4 ln x .
x
The right boundary condition imposes the constraint
p
1/4 ln 2 = n, n Z+ .
This gives us the eigenvalues and eigenfunctions.
 
1  n 2 1 n ln x
n = + , Xn (x) = sin , n Z+ .
4 ln 2 x ln 2
We normalize the eigenfunctions.
Z 2   Z 1
1 n ln x ln 2
sin2 dx = ln 2 sin2 (n) d =
1 x ln 2 0 2
r  
2 1 n ln x
Xn (x) = sin , n Z+ .
ln 2 x ln 2
From separation of variables, we have differential equations for the Tn .
 
0 2 1  n 2
Tn = A + Tn
4 ln 2
   
2 1  n 2
Tn (t) = exp A + t
4 ln 2

1770
We expand in a series of the eigensolutions.

X
(x, t) = n Xn (x)Tn (t)
n=1

We substitute the expansion for into the initial condition to determine the coefficients.

X
(x, 0) = n Xn (x) = f (x)
n=1
Z 2
n = Xn (x)f (x) dx
1

Solution 37.18

utt = c2 uxx , 0 < x < L, t > 0,


u(0, t) = 0, ux (L, t) = 0,
u(x, 0) = f (x), ut (x, 0) = 0,

We substitute the separation of variables u(x, t) = X(x)T (t) into the partial differential equation.

(XT )tt = c2 (XT )xx


T 00 X 00
= = 2
c2 T X
With the boundary conditions at x = 0, L, we have the ordinary differential equations,

T 00 = c2 2 T,
X 00 = 2 X, X(0) = X 0 (L) = 0.

1771
The problem for X is a regular Sturm-Liouville eigenvalue problem. From the Rayleigh quotient,
0 L
RL 0 2 RL 0 2
[ ] 0 + ( ) dx ( ) dx
2 = RL 0 = R0 L
0
2 dx 0
2 dx

we see that there are only positive eigenvalues. For 2 > 0 the general solution of the ordinary differential equation is

X = a1 cos(x) + a2 sin(x).

The solution that satisfies the left boundary condition is

X = a sin(x).

For non-trivial solutions, the right boundary condition imposes the constraint,

cos (L) = 0,
 
1
= n , n N.
L 2
The eigenvalues and eigenfunctions are
 
(2n 1) (2n 1)x
n = , Xn = sin , n N.
2L 2L

The differential equation for T becomes  2


00 2 (2n 1)
T = c T,
2L
which has the two linearly independent solutions,
   
(1) (2n 1)ct (2n 1)ct
Tn = cos , Tn(2) = sin .
2L 2L

1772
The eigenvalues and eigen-solutions of the partial differential equation are,
(2n 1)
n = , n N,
2L
       
(2n 1)x (2n 1)ct (2n 1)x (2n 1)ct
u(1)
n = sin cos (2)
, un = sin sin .
2L 2L 2L 2L
We expand u(x, t) in a series of the eigen-solutions.
     
X (2n 1)x (2n 1)ct (2n 1)ct
u(x, t) = sin an cos + bn sin .
n=1
2L 2L 2L

We impose the initial condition ut (x, 0) = 0,


 
X (2n 1)c (2n 1)x
ut (x, 0) = bn sin = 0,
n=1
2L 2L

bn = 0.
The initial condition u(x, 0) = f (x) allows us to determine the remaining coefficients,
 
X (2n 1)x
u(x, 0) = an sin = f (x),
n=1
2L

L  
(2n 1)x
Z
2
an = f (x) sin dx.
L 0 2L
The series solution for u(x, t) is,
   
X (2n 1)x (2n 1)ct
u(x, t) = an sin cos .
n=1
2L 2L

1773
Solution 37.19

uxx + uyy = f (x, y), 0 < x < a, 0 < y < b,


u(0, y) = u(a, y) = 0, uy (x, 0) = uy (x, b) = 0
We will solve this problem with an eigenfunction expansion in x. To determine a suitable set of eigenfunctions, we
substitute the separation of variables u(x, y) = X(x)Y (y) into the homogeneous partial differential equation.
uxx + uyy = 0
(XY )xx + (XY )yy = 0
X 00 Y 00
= = 2
X Y
With the boundary conditions at x = 0, a, we have the regular Sturm-Liouville problem,
X 00 = 2 X, X(0) = X(a) = 0,
which has the solutions,
n  nx 
n = , Xn = sin , n Z+ .
a a
We expand u(x, y) in a series of the eigenfunctions.

X  nx 
u(x, y) = un (y) sin
n=1
a

We substitute this series into the partial differential equation and boundary conditions at y = 0, b.
   nx 
X n 2  nx 
00
un (y) sin + un (y) sin = f (x)
n=1
a a a
X  nx  X  nx 
u0n (0) sin = u0n (b) sin =0
n=1
a n=1
a

1774
We expand f (x, y) in a Fourier sine series.

X  nx 
f (x, y) = fn (y) sin
n=1
a
Z a
2  nx 
fn (y) = f (x, y) sin dx
a 0 a
We obtain the ordinary differential equations for the coefficients in the expansion.
 n 2
u00n (y) un (y) = fn (y), u0n (0) = u0n (b) = 0, n Z+ .
a
We will solve these ordinary differential equations with Green functions.
Consider the Green function problem,
 n 2
gn00 (y; ) gn (y; ) = (y ), gn0 (0; ) = gn0 (b; ) = 0.
a
The homogeneous solutions  
 ny  n(y b)
cosh and cosh
a a
satisfy the left and right boundary conditions, respectively. We compute the Wronskian of these two solutions.

cosh(ny/a) cosh(n(y b)/a)
W (y) = n

a
sinh(ny/a) n a
sinh(n(y b)/a)
    
n  ny  n(y b)  ny  n(y b)
= cosh sinh sinh cosh
a a a a a
 
n nb
= sinh
a a
The Green function is
a cosh(ny< /a) cosh(n(y> b)/a)
gn (y; ) = .
n sinh(nb/a)

1775
The solutions for the coefficients in the expansion are
Z b
un (y) = gn (y; )fn () d.
0

Solution 37.20

utt + a2 uxxxx = 0, 0 < x < L, t > 0,


u(x, 0) = f (x), ut (x, 0) = g(x),
u(0, t) = uxx (0, t) = 0, u(L, t) = uxx (L, t) = 0,

We will solve this problem by expanding the solution in a series of eigen-solutions that satisfy the partial differential
equation and the homogeneous boundary conditions. We will use the initial conditions to determine the coefficients in
the expansion. We substitute the separation of variables, u(x, t) = X(x)T (t) into the partial differential equation.

(XT )tt + a2 (XT )xxxx = 0


T 00 X 0000
2
= = 4
aT X
Here we make the assumption that 0 arg() < /2, i.e., lies in the first quadrant of the complex plane. Note that
4 covers the entire complex plane. We have the ordinary differential equation,

T 00 = a2 4 T,

and with the boundary conditions at x = 0, L, the eigenvalue problem,

X 0000 = 4 X, X(0) = X 00 (0) = X(L) = X 00 (L) = 0.

For = 0, the general solution of the differential equation is

X = c1 + c2 x + c3 x2 + c4 x3 .

1776
Only the trivial solution satisfies the boundary conditions. = 0 is not an eigenvalue. For 6= 0, a set of linearly
independent solutions is
{ex , ex , ex , ex }.
Another linearly independent set, (which will be more useful for this problem), is

{cos(x), sin(x), cosh(x), sinh(x)}.

Both sin(x) and sinh(x) satisfy the left boundary conditions. Consider the linear combination c1 cos(x)+c2 cosh(x).
The left boundary conditions impose the two constraints c1 + c2 = 0, c1 c2 = 0. Only the trivial linear combination
of cos(x) and cosh(x) can satisfy the left boundary condition. Thus the solution has the form,

X = c1 sin(x) + c2 sinh(x).

The right boundary conditions impose the constraints,


(
c1 sin(L) + c2 sinh(L) = 0,
c1 2 sin(L) + c2 2 sinh(L) = 0
(
c1 sin(L) + c2 sinh(L) = 0,
c1 sin(L) + c2 sinh(L) = 0

This set of equations has a nontrivial solution if and only if the determinant is zero,

sin(L) sinh(L)
sin(L) sinh(L) = 2 sin(L) sinh(L) = 0.

Since sinh(z) is nonzero in 0 arg(z) < /2, z 6= 0, and sin(z) has the zeros z = n, n N in this domain, the
eigenvalues and eigenfunctions are,
n  nx 
n = , Xn = sin , n N.
L L

1777
The differential equation for T becomes,  n 4
00 2
T = a T,
L
which has the solutions,         
n 2 n 2
cos a t , sin a t .
L L
The eigen-solutions of the partial differential equation are,
 nx         
(1) n 2 (2)
 nx  n 2
un = sin cos a t , un = sin sin a t , n N.
L L L L
We expand the solution of the partial differential equation in a series of the eigen-solutions.
 nx          
X n 2 n 2
u(x, t) = sin cn cos a t + dn sin a t
n=1
L L L

The initial condition for u(x, t) and ut (x, t) allow us to determine the coefficients in the expansion.

X  nx 
u(x, 0) = cn sin = f (x)
n=1
L

X  n 2  nx 
ut (x, 0) = dn a sin = g(x)
n=1
L L

cn and dn are coefficients in Fourier sine series.


Z L
2  nx 
cn = f (x) sin dx
L 0 L
Z L
2L  nx 
dn = 2 2 g(x) sin dx
a n 0 L

1778
Solution 37.21

ut = uxx + I 2 u, 0 < x < L, t > 0,


u(0, t) = u(L, t) = 0, u(x, 0) = g(x).

We will solve this problem with an expansion in eigen-solutions of the partial differential equation. We substitute the
separation of variables u(x, t) = X(x)T (t) into the partial differential equation.

(XT )t = (XT )xx + I 2 XT


T0 I 2 X 00
= = 2
T X
Now we have an ordinary differential equation for T and a Sturm-Liouville eigenvalue problem for X. (Note that we
have followed the rule of thumb that the problem will be easier if we move all the parameters out of the eigenvalue
problem.)

T 0 = 2 I 2 T


X 00 = 2 X, X(0) = X(L) = 0

The eigenvalues and eigenfunctions for X are


n  nx 
n = , Xn = sin , n N.
L L
The differential equation for T becomes,
   
n 2
Tn0 = 2
I Tn ,
L
which has the solution,      
n 2 2
Tn = c exp I t .
L

1779
From this solution, we see that the critical current is
r

ICR = .
L

If I is greater that this, then the eigen-solution for n = 1 will be exponentially growing. This would make the whole
solution exponentially growing. For I < ICR , each of the Tn is exponentially decaying. The eigen-solutions of the
partial differential equation are,
     
n 2 2
 nx 
un = exp I t sin , n N.
L L

We expand u(x, t) in its eigen-solutions, un .

     
X n 2 2
 nx 
u(x, t) = an exp I t sin
n=1
L L

We determine the coefficients an from the initial condition.



X  nx 
u(x, 0) = an sin = g(x)
n=1
L
Z L
2  nx 
an = g(x) sin dx.
L 0 L

If < 0, then the solution is exponentially decaying regardless of current. Thus there is no critical current.

Solution 37.22

1780
a) The problem is

ut (x, y, z, t) = u(x, y, z, t), < x < , < y < , 0 < z < a, t > 0,
u(x, y, z, 0) = T, u(x, y, 0, t) = u(x, y, a, t) = 0.

Because of symmetry, the partial differential equation in four variables is reduced to a problem in two variables,

ut (z, t) = uzz (z, t), 0 < z < a, t > 0,


u(z, 0) = T, u(0, t) = u(a, t) = 0.

We will solve this problem with an expansion in eigen-solutions of the partial differential equation that satisfy the
homogeneous boundary conditions. We substitute the separation of variables u(z, t) = Z(z)T (t) into the partial
differential equation.

ZT 0 = Z 00 T
T0 Z 00
= = 2
T Z
With the boundary conditions at z = 0, a we have the Sturm-Liouville eigenvalue problem,

Z 00 = 2 Z, Z(0) = Z(a) = 0,

which has the solutions,


n  nz 
n = , Zn = sin , n N.
a a
The problem for T becomes,
 n 2
Tn0 = Tn ,
a
with the solution, 
 n 2 
Tn = exp t .
a

1781
The eigen-solutions are 
 nz   n 2 
un (z, t) = sin exp t .
a a
The solution for u is a linear combination of the eigen-solutions. The slowest decaying eigen-solution is
 z    2 
u1 (z, t) = sin exp t .
a a

Thus the e-folding time is


a2
e = .
2

b) The problem is

ut (r, , z, t) = u(r, , z, t), 0 < r < a, 0 < < 2, < z < , t > 0,
u(r, , z, 0) = T, u(0, , z, t) is bounded, u(a, , z, t) = 0.

The Laplacian in cylindrical coordinates is

1 1
u = urr + ur + 2 u + uzz .
r r
Because of symmetry, the solution does not depend on or z.
 
1
ut (r, t) = urr (r, t) + ur (r, t) , 0 < r < a, t > 0,
r
u(r, 0) = T, u(0, t) is bounded, u(a, t) = 0.

We will solve this problem with an expansion in eigen-solutions of the partial differential equation that satisfy
the homogeneous boundary conditions at r = 0 and r = a. We substitute the separation of variables u(r, t) =

1782
R(r)T (t) into the partial differential equation.
 
0 1 0
00
RT = R T + R T
r
0 00 0
T R R
= + = 2
T R rR
We have the eigenvalue problem,
1
R00 + R0 + 2 R = 0, R(0) is bounded, R(a) = 0.
r
Recall that the Bessel equation,
2
 
001 0 2
y + y + 2 y = 0,
x x
has the general solution y = c1 J (x) + c2 Y (x). We discard the Bessel function of the second kind, Y , as it
is unbounded at the origin. The solution for R(r) is

R(r) = J0 (r).

Applying the boundary condition at r = a, we see that the eigenvalues and eigenfunctions are
 
n n r
n = , Rn = J 0 , n N,
a a
where {n } are the positive roots of the Bessel function J0 .
The differential equation for T becomes,  2
n
Tn0 = Tn ,
a
which has the solutions,
 2 !
n
Tn = exp t .
a

1783
The eigen-solutions of the partial differential equation for u(r, t) are,
   2 !
n r n
un (r, t) = J0 exp t .
a a

The solution u(r, t) is a linear combination of the eigen-solutions, un . The slowest decaying eigenfunction is,
   2 !
1 r 1
u1 (r, t) = J0 exp t .
a a

Thus the e-folding time is


a2
e = .
12

c) The problem is

ut (r, , , t) = u(r, , , t), 0 < r < a, 0 < < 2, 0 < < , t > 0,
u(r, , , 0) = T, u(0, , , t) is bounded, u(a, , , t) = 0.

The Laplacian in spherical coordinates is,

2 1 cos 1
u = urr + ur + 2 u + 2 u + 2 2 u .
r r r sin r sin
Because of symmetry, the solution does not depend on or .
 
2
ut (r, t) = urr (r, t) + ur (r, t) , 0 < r < a, t > 0,
r
u(r, 0) = T, u(0, t) is bounded, u(a, t) = 0

1784
We will solve this problem with an expansion in eigen-solutions of the partial differential equation that satisfy
the homogeneous boundary conditions at r = 0 and r = a. We substitute the separation of variables u(r, t) =
R(r)T (t) into the partial differential equation.
 
0 00 2 0
RT = R T + R T
r
0 00 0
T R 2R
= + = 2
T R rR
We have the eigenvalue problem,
2
R00 + R0 + 2 R = 0, R(0) is bounded, R(a) = 0.
r
Recall that the equation,  
002 0 2 ( + 1)
y + y + y = 0,
x x2
has the general solution y = c1 j (x) + c2 y (x), where j and y are the spherical Bessel functions of the first
and second kind. We discard y as it is unbounded at the origin. (The spherical Bessel functions are related to
the Bessel functions by r

j (x) = J+1/2 (x).)
2x
The solution for R(r) is
Rn = j0 (r).
Applying the boundary condition at r = a, we see that the eigenvalues and eigenfunctions are
n  r
n
n = , Rn = j0 , n N.
a a
The problem for T becomes  2
n
Tn0 = Tn ,
a

1785
which has the solutions,  2 
n
Tn = exp t .
a
The eigen-solutions of the partial differential equation are,
 r   2 
n n
un (r, t) = j0 exp t .
a a

The slowest decaying eigen-solution is,


 r   2 
1 1
u1 (r, t) = j0 exp t .
a a

Thus the e-folding time is


a2
e = .
12

d) If the edges are perfectly insulated, then no heat escapes through the boundary. The temperature is constant for
all time. There is no e-folding time.
Solution 37.23
We will solve this problem with an eigenfunction expansion. Since the partial differential equation is homogeneous, we
will find eigenfunctions in both x and y. We substitute the separation of variables u(x, y, t) = X(x)Y (y)T (t) into the
partial differential equation.

XY T 0 = (t) (X 00 Y T + XY 00 T )
T0 X 00 Y 00
= + = 2
(t)T X Y
X 00 Y 00
= 2 = 2
X Y

1786
First we have a Sturm-Liouville eigenvalue problem for X,
X 00 = 2 X, X 0 (0) = X 0 (a) = 0,
which has the solutions,
m  mx 
m =
, Xm = cos , m = 0, 1, 2, . . . .
a a
Now we have a Sturm-Liouville eigenvalue problem for Y ,
  m 2 
00 2
Y = Y, Y (0) = Y (b) = 0,
a
which has the solutions,
r
m 2  n 2  ny 
mn = + , Yn = sin , m = 0, 1, 2, . . . , n = 1, 2, 3, . . . .
a b b
ny
A few of the eigenfunctions, cos mx
 
a
sin b
, are shown in Figure 37.3.
The differential equation for T becomes,
 
0 m 2  n 2
Tmn = + (t)Tmn ,
a b
which has the solutions,   Z t 
m 2  n 2
Tmn = exp + ( ) d .
a b 0
The eigen-solutions of the partial differential equation are,
 mx   ny    Z t 
m 2  n 2
umn = cos sin exp + ( ) d .
a b a b 0

The solution of the partial differential equation is,


X
 mx   ny    Z t 
X m 2  n 2
u(x, y, t) = cmn cos sin exp + ( ) d .
m=0 n=1
a b a b 0

1787
We determine the coefficients from the initial condition.
X
X  mx   ny 
u(x, y, 0) = cmn cos sin = f (x, y)
m=0 n=1
a b

Z a Z b
2  n 
c0n = f (x, y) sin dy dx
ab 0 0 b
Z a Z b
4  m   n 
cmn = f (x, y) cos sin dy dx
ab 0 0 a b

Solution 37.24
The steady state temperature satisfies Laplaces equation, u = 0. The Laplacian in cylindrical coordinates is,
1 1
u(r, , z) = urr + ur + 2 u + uzz .
r r
Because of the homogeneity in the z direction, we reduce the partial differential equation to,
1 1
urr + ur + 2 u = 0, 0 < r < 1, 0 < < .
r r
The boundary conditions are,

u(r, 0) = u(r, ) = 0, u(0, ) = 0, u(1, ) = 1.

We will solve this problem with an eigenfunction expansion. We substitute the separation of variables u(r, ) = R(r)T ()
into the partial differential equation.
1 1
R00 T + R0 T + 2 RT 00 = 0
r r
00 0
R R T 00
r2 +r = = 2
R R T

1788
We have the regular Sturm-Liouville eigenvalue problem,
T 00 = 2 T, T (0) = T () = 0,
which has the solutions,
n = n, Tn = sin(n), n N.
The problem for R becomes,
r2 R00 + rR0 n2 R = 0, R(0) = 0.
This is an Euler equation. We substitute R = r into the differential equation to obtain,
( 1) + n2 = 0,
= n.
The general solution of the differential equation for R is
Rn = c1 rn + c2 rn .
The solution that vanishes at r = 0 is
Rn = crn .
The eigen-solutions of the differential equation are,
un = rn sin(n).
The solution of the partial differential equation is

X
u(r, ) = an rn sin(n).
n=1

We determine the coefficients from the boundary condition at r = 1.



X
u(1, ) = an sin(n) = 1
n=1
Z
2 2
an = sin(n) d = (1 (1)n )
0 n

1789
The solution of the partial differential equation is

4 X n
u(r, ) = r sin(n).
n=1
odd n

Solution 37.25
The problem is

uxx + uyy = 0, 0 < x, 0 < y < 1,


u(x, 0) = u(x, 1) = 0, u(0, y) = f (y).

We substitute the separation of variables u(x, y) = X(x)Y (y) into the partial differential equation.

X 00 Y + XY 00 = 0
X 00 Y 00
= = 2
X Y
We have the regular Sturm-Liouville problem,

Y 00 = 2 Y, Y (0) = Y (1) = 0,

which has the solutions,


n = n, Yn = sin(ny), n N.
The problem for X becomes,
Xn00 = (n)2 X,
which has the general solution,
Xn = c1 enx +c2 enx .
The solution that is bounded as x is,
Xn = c enx .

1790
The eigen-solutions of the partial differential equation are,

un = enx sin(ny), n N.

The solution of the partial differential equation is,


X
u(x, y) = an enx sin(ny).
n=1

We find the coefficients from the boundary condition at x = 0.



X
u(0, y) = an sin(ny) = f (y)
n=1

Z 1
an = 2 f (y) sin(ny) dy
0

Solution 37.26
The Laplacian in polar coordinates is
1 1
u urr + ur + 2 u .
r r
Since we have homogeneous boundary conditions at = 0 and = , we will solve this problem with an eigenfunction
expansion. We substitute the separation of variables u(r, ) = R(r)() into Laplaces equation.

1 1
R00 + R0 + 2 R00 = 0
r r
00 0
R R 00
r2 +r = = 2 .
R R

1791
We have a regular Sturm-Liouville eigenvalue problem for .

00 = 2 , (0) = () = 0
 
n n
n = , n = sin , n Z+ .

We have Euler equations for Rn . We solve them with the substitution R = r .
 n 2
2 00 0
r Rn + rRn Rn = 0, Rn (a) = 0

 n 2
( 1) + =0

n
=

Rn = c1 r n/
+ c2 rn/ .

The solution, (up to a multiplicative constant), that vanishes at r = a is

Rn = rn/ a2n/ rn/ .

Thus the series expansion of our solution is,


 
X
n/ 2n/ n/
 n
u(r, ) = un r a r sin .
n=1

We determine the coefficients from the boundary condition at r = b.


 
X
n/ 2n/ n/ n

u(b, ) = un b a b sin = f ()
n=1

Z  
2 n
un = f () sin d
(bn/ a2n/ bn/ ) 0

1792
Solution 37.27

a) The mathematical statement of the problem is

utt = c2 uxx , 0 < x < L, t > 0,


u(0, t) = u(L, t) = 0,
(
v for |x | < d
u(x, 0) = 0, ut (x, 0) =
0 for |x | > d.

Because we are interest in the harmonics of the motion, we will solve this problem with an eigenfunction expansion in
x. We substitute the separation of variables u(x, t) = X(x)T (t) into the wave equation.

XT 00 = c2 X 00 T
T 00 X 00
= = 2
c2 T X
The eigenvalue problem for X is,
X 00 = 2 X, X(0) = X(L) = 0,
which has the solutions,
n  nx 
n = , Xn = sin , n N.
L L
The ordinary differential equation for the Tn are,
 nc 2
Tn00 = Tn ,
L
which have the linearly independent solutions,
   
nct nct
cos , sin .
L L

1793
The solution for u(x, t) is a linear combination of the eigen-solutions.
 nx      
X nct nct
u(x, t) = sin an cos + bn sin
n=1
L L L

Since the string initially has zero displacement, each of the an are zero.
 nx   
X nct
u(x, t) = bn sin sin
n=1
L L

Now we use the initial velocity to determine the coefficients in the expansion. Because the position is a continuous
function of x, and there is a jump discontinuity in the velocity as a function of x, the coefficients in the expansion will
decay as 1/n2 .

(
X nc  nx  v for |x | < d
ut (x, 0) = bn sin =
n=1
L L 0 for |x | > d.
Z L
nc 2  nx 
bn = ut (x, 0) sin dx
L L 0 L

Z +d
2  nx 
bn = v sin dx
nc d L
   
4Lv nd n
= 2 2 sin sin
n c L L

The solution for u(x, t) is,

     
4Lv X 1 nd n  nx  nct
u(x, t) = 2 sin sin sin sin .
c n=1 n2 L L L L

1794
b) The form of the solution is again,
 nx   
X nct
u(x, t) = bn sin sin
n=1
L L

We determine the coefficients in the expansion from the initial velocity.


 

(
(x)
X nc  nx  v cos 2d
for |x | < d
ut (x, 0) = bn sin =
n=1
L L 0 for |x | > d.
Z L
nc 2  nx 
bn = ut (x, 0) sin dx
L L 0 L

Z +d  
2 (x )  nx 
bn = v cos sin dx
nc d 2d L
(
8dL2 v nd n L
 
2 2 2 2 cos sin for d 6= ,
bn = nv c(L 4d n ) L L 2n
2nd + L sin 2nd sin n L
 
n2 2 c L L
for d = 2n

The solution for u(x, t) is,



8dL2 v X
     
1 nd n  nx  nct L
u(x, t) = cos sin sin sin for d 6= ,
2 c n=1 n(L2 4d2 n2 ) L L L L 2n
      
v X 1 2nd n  nx  nct L
u(x, t) = 2 2nd + L sin sin sin sin for d = .
c n=1 n2 L L L L 2n

c) The kinetic energy of the string is


Z L
1
E= (ut (x, t))2 dx,
2 0

1795
where is the density of the string per unit length.
Flat Hammer. The nth harmonic is
     
4Lv nd n  nx  nct
un = 2 2 sin sin sin sin .
n c L L L L

The kinetic energy of the nth harmonic is


L 2
4Lv 2
Z       
un nd n nct
En = dx = 2 2 sin2 sin 2
cos2
.
2 0 t n L L L

This will be maximized if



n
2
sin = 1,
L
n (2m 1)
= , m = 1, . . . , n,
L 2
(2m 1)L
= , m = 1, . . . , n
2n

We note that the kinetic energies of the nth harmonic decay as 1/n2 .
L
Curved Hammer. We assume that d 6= 2n . The nth harmonic is

8dL2 v
     
nd n  nx  nct
un = cos sin sin sin .
n 2 c(L2 4d2 n2 ) L L L L

The kinetic energy of the nth harmonic is


L 2
16d2 L3 v 2
Z       
un nd n nct
En = dx = 2 2 2 2 2
cos2 sin 2
cos2
.
2 0 t (L 4d n ) L L L

1796
This will be maximized if
 
2 n
sin = 1,
L
(2m 1)L
= , m = 1, . . . , n
2n

We note that the kinetic energies of the nth harmonic decay as 1/n4 .

Solution 37.28
In mathematical notation, the problem is

utt c2 uxx = s(x, t), 0 < x < L, t > 0,


u(0, t) = u(L, t) = 0,
u(x, 0) = ut (x, 0) = 0.

Since this is an inhomogeneous partial differential equation, we will expand the solution in a series of eigenfunctions in
x for which the coefficients are functions of t. The solution for u has the form,

X  nx 
u(x, t) = un (t) sin .
n=1
L

Substituting this expression into the inhomogeneous partial differential equation will give us ordinary differential equa-
tions for each of the un .
  2 
2 n
X  nx 
00
un + c un sin = s(x, t).
n=1
L L
We expand the right side in a series of the eigenfunctions.

X  nx 
s(x, t) = sn (t) sin .
n=1
L

1797
For 0 < t < we have
2 L
Z  nx 
sn (t) = s(x, t) sin dx
L 0 L
2 L
   
(x )
Z
t  nx 
= v cos sin sin dx
L 0 2d L
     
8dLv nd n t
= 2 2 2
cos sin sin .
(L 4d n ) L L
For t > , sn (t) = 0. Substituting this into the partial differential equation yields,
(
8dLv
cos nd sin n sin t
  
00
 nc 2
(L2 4d2 n2 ) L L
, for t < ,
un + un =
L 0 for t > .

Since the initial position and velocity of the string is zero, we have

un (0) = u0n (0) = 0.

First we solve the differential equation on the range 0 < t < . The homogeneous solutions are
   
nct nct
cos , sin .
L L
Since the right side of the ordinary differential equation is a constant times sin(t/), which is an eigenfunction of the
differential operator, we can guess the form of a particular solution, pn (t).
 
t
pn (t) = d sin

We substitute this into the ordinary differential equation to determine the multiplicative constant d.
8d 2 L3 v
     
nd n t
pn (t) = 3 2 2 2 2 2 2 2
cos sin sin
(L c n )(L 4d n ) L L

1798
The general solution for un (t) is

8d 2 L3 v
         
nct nct nd n t
un (t) = a cos + b sin 3 2 2 2 2 2 2 2
cos sin sin .
L L (L c n )(L 4d n ) L L

We use the initial conditions to determine the constants a and b. The solution for 0 < t < is
8d 2 L3 v
       
nd n L nct t
un (t) = 3 2 2 2 2 2 2 2
cos sin sin sin .
(L c n )(L 4d n ) L L cn L

The solution for t > , the solution is a linear combination of the homogeneous solutions. This linear combination is
determined by the position and velocity at t = . We use the above solution to determine these quantities.

8d 2 L4 v
     
nd n nc
un () = 3 cos sin sin
cn(L2 c2 2 n2 )(L2 4d2 n2 ) L L L
2 3
     
0 8d L v nd n nc
un () = 2 cos sin 1 + cos
(L2 c2 2 n2 )(L2 4d2 n2 ) L L L

The fundamental set of solutions at t = is


    
nc(t ) L nc(t )
cos , sin
L nc L

From the initial conditions at t = , we see that the solution for t > is

8d 2 L3 v
   
nd n
un (t) = 3 2 cos sin
(L c2 2 n2 )(L2 4d2 n2 ) L L
         
L nc nc(t ) nc nc(t )
sin cos + 1 + cos sin .
cn L L L L

1799
Width of the Hammer. The nth harmonic has the width dependent factor,
 
d nd
cos .
L2 4d2 n2 L

Differentiating this expression and trying to find zeros to determine extrema would give us an equation with both
algebraic and transcendental terms. Thus we dont attempt to find the maxima exactly. We know that d < L. The
cosine factor is large when

nd
m, m = 1, 2, . . . , n 1,
L
mL
d , m = 1, 2, . . . , n 1.
n
Substituting d = mL/n into the width dependent factor gives us

d
(1)m .
L2 (1 4m2 )

Thus we see that the amplitude of the nth harmonic and hence its kinetic energy will be maximized for

L
d
n

The cosine term in the width dependent factor vanishes when

(2m 1)L
d= , m = 1, 2, . . . , n.
2n
The kinetic energy of the nth harmonic is minimized for these widths.
L L
For the lower harmonics, n  2d , the kinetic energy is proportional to d2 ; for the higher harmonics, n  2d
, the
kinetic energy is proportional to 1/d2 .

1800
Duration of the Blow. The nth harmonic has the duration dependent factor,
2
         
L nc nc(t ) nc nc(t )
sin cos + 1 + cos sin .
L2 n2 c2 2 nc L L L L
If we assume that is small, then  
L nc
sin .
nc L
and   
nc 2
1 + cos .
L
Thus the duration dependent factor is about,
 
nc(t )
sin .
L n2 c2 2
2 L
L
Thus for the lower harmonics, (those satisfying n  c ), the amplitude is proportional to , which means that the
L
kinetic energy is proportional to 2 . For the higher harmonics, (those with n  c ), the amplitude is proportional to
2
1/, which means that the kinetic energy is proportional to 1/ .

Solution 37.29
Substituting u(x, y, z, t) = v(x, y, z) et into the wave equation will give us a Helmholtz equation.

2 v et c2 (vxx + vyy + vzz ) et = 0


vxx + vyy + vzz + k 2 v = 0.

We find the propagating modes with separation of variables. We substitute v = X(x)Y (y)Z(z) into the Helmholtz
equation.

X 00 Y Z + XY 00 Z + XY Z 00 + k 2 XY Z = 0
X 00 Y 00 Z 00
= + + k2 = 2
X Y Z

1801
The eigenvalue problem in x is
X 00 = 2 X, X(0) = X(L) = 0,
which has the solutions,
n  nx 
n = , Xn = sin .
L L
We continue with the separation of variables.

Y 00 Z 00  n 2
= + k2 = 2
Y Z L
The eigenvalue problem in y is
Y 00 = 2 Y, Y (0) = Y (L) = 0,
which has the solutions,
m  my 
n = , Ym = sin .
L L
Now we have an ordinary differential equation for Z,
  2 
00 2 2 2

Z + k n +m Z = 0.
L

We define the eigenvalues,


 2
2n,m 2
n2 + m2 .

=k
L
2

If k 2 L
(n2 + m2 ) < 0, then the solutions for Z are,
s  !
 2
exp (n2 + m2 ) k 2 z .
L

We discard this case, as the solutions are not bounded as z .

1802
2

If k 2 L
(n2 + m2 ) = 0, then the solutions for Z are,

{1, z}

The solution Z = 1 satisfies the boundedness and nonzero condition at infinity. This corresponds to a standing wave.
2
If k 2 L (n2 + m2 ) > 0, then the solutions for Z are,

en,m z .
2

These satisfy the boundedness and nonzero conditions at infinity. For values of n, m satisfying k 2 L
(n2 + m2 ) 0,
there are the propagating modes,
 nx   my 
un,m = sin sin e(tn,m z) .
L L
Solution 37.30

utt = c2 u, 0 < x < a, 0 < y < b, (37.12)


u(0, y) = u(a, y) = u(x, 0) = u(x, b) = 0.

We substitute the separation of variables u(x, y, t) = X(x)Y (y)T (t) into Equation 37.12.
T 00 X 00 Y 00
= + =
c2 T X Y
X 00 Y 00
= =
X Y
This gives us differential equations for X(x), Y (y) and T (t).

X 00 = X, X(0) = X(a) = 0
Y 00 = ( )Y, Y (0) = Y (b) = 0
T 00 = c2 T

1803
First we solve the problem for X.  m 2  mx 
m = , Xm = sin
a a
Then we solve the problem for Y .
 m 2  n 2  ny 
m,n = + , Ym,n = sin
a b b
Finally we determine T . r !
cos m 2  n 2
Tm,n = c + t
sin a b
The modes of oscillation are
r  !
 mx   ny  cos m 2  n 2
um,n = sin sin c + t .
a b sin a b

The frequencies are r


m 2  n 2
m,n = c + .
a b
Figure 37.4 shows a few of the modes of oscillation in surface and density plots.
Solution 37.31
We substitute the separation of variables = X(x)Y (y)T (t) into the differential equation.

t = a2 (xx + yy ) (37.13)
XY T 0 = a2 (X 00 Y T + XY 00 T )
T0 X 00 Y 00
= + =
a2 T X Y
T0 X 00 Y 00
= , = =
a2 T X Y

1804
First we solve the eigenvalue problem for X.
X 00 + X = 0, X(0) = X(lx ) = 0
 2  
m mx
m = , Xm (x) = sin , m Z+
lx lx
Then we solve the eigenvalue problem for Y .
Y 00 + ( m )Y = 0, Y 0 (0) = Y 0 (ly ) = 0
 2  
n ny
mn = m + , Ymn (y) = cos , n Z0+
ly ly
Next we solve the differential equation for T , (up to a multiplicative constant).
T 0 = a2 mn T
T (t) = exp a2 mn t


The eigensolutions of Equation 37.13 are


sin(m x) cos(mn y) exp a2 mn t , m Z+ , n Z0+ .


We choose the eigensolutions mn to be orthonormal on the xy domain at t = 0.


s
2
sin(m x) exp a2 mn t , m Z+

m0 (x, y, t) =
lx ly
2
sin(m x) cos(mn y) exp a2 mn t , m Z+ , n Z+

mn (x, y, t) = p
lx ly
The solution of Equation 37.13 is a linear combination of the eigensolutions.

X
(x, y, t) = cmn mn (x, y, t)
m=1
n=0

1805
We determine the coefficients from the initial condition.
(x, y, 0) = 1

X
cmn mn (x, y, 0) = 1
m=1
n=0
Z lx Z ly
cmn = mn (x, y, 0) dy dx
0 0
s Z lx Z ly
2
cm0 = sin(m x) dy dx
lx ly 0 0
p 1 (1)m
cm0 = 2lx ly , m Z+
m
Z l x Z ly
2
cmn =p sin(m x) cos(mn y) dy dx
lx ly 0 0
cmn = 0, m Z+ , n Z+

X
(x, y, t) = cm0 m0 (x, y, t)
m=1
p
X 2 2lx ly
sin(m x) exp a2 mn t

(x, y, t) =
m=1
m
odd m

Addendum. Note that an equivalent problem to the one specified is


t = a2 (xx + yy ) , 0 < x < lx , < y < ,
(x, y, 0) = 1, (0, y, t) = (ly , y, t) = 0.
Here we have done an even periodic continuation of the problem in the y variable. Thus the boundary conditions
y (x, 0, t) = y (x, ly , t) = 0

1806
are automatically satisfied. Note that this problem does not depend on y. Thus we only had to solve

t = a2 xx , 0 < x < lx
(x, 0) = 1, (0, t) = (ly , t) = 0.

Solution 37.32
1. Since the initial and boundary conditions do not depend on , neither does . We apply the separation of variables
= u(r)T (t).

t = a2 (37.14)
1
t = a2 (rr )r (37.15)
r
T0 1
= (ru0 )0 = (37.16)
a2 T r

We solve the eigenvalue problem for u(r).

(ru0 )0 + u = 0, u(0) bounded, u(R) = 0

First we write the general solution.


   
u(r) = c1 J0 r + c2 Y0 r

The Bessel function of the second kind, Y0 , is not bounded at r = 0, so c2 = 0. We use the boundary condition
at r = R to determine the eigenvalues.
 2  
j0,n j0,n r
n = , un (r) = cJ0
R R

1807
We choose the constant c so that the eigenfunctions are orthonormal with respect to the weighting function r.

 
j0,n r
J0 R
un (r) = r  
RR j0,n r
0
rJ02 R
 
2 j0,n r
= J0
RJ1 (j0,n ) R

Now we solve the differential equation for T .

T 0 = a2 n T
 2 !
aj0,n
Tn = exp t
R2

The eigensolutions of Equation 37.14 are

   2 !
2 j0,n r aj0,n
n (r, t) = J0 exp t
RJ1 (j0,n ) R R2

The solution is a linear combination of the eigensolutions.

   2 !
X 2 j0,n r aj0,n
= cn J0 exp t
n=1
RJ1 (j0,n ) R R2

1808
We determine the coefficients from the initial condition.
(r, , 0) = V
 
X 2 j0,n r
cn J0 =V
n=1
RJ1 (j 0,n ) R
Z R  
2 j0,n r
cn = Vr J0 dr
0 RJ1 (j0,n ) R

2 R
cn = V J1 (j0,n )
RJ1 (j0,n ) j0,n /R

2 VR
cn =
j0,n
 
J j0,n r  2 !
X 0 R aj0,n
(r, , t) = 2V exp t
j J (j )
n=1 0,n 1 0,n
R2

2.
r
2  
J (r) cos r , r +
r  2 4

1
j,n n +
2 4
For large n, the terms in the series solution at t = 0 are
  q  
j0,n r
J0 j0,n R
r 2R
j0,n r
cos R

4
q
j0,n J1 (j0,n ) j0,n j20,n cos j0,n 3

4
 
(n1/4)r
R cos R
4
.
r(n 1/4) cos ((n 1))

1809
The coefficients decay as 1/n.

Solution 37.33
1. We substitute the separation of variables = T (t)()() into Equation 37.7
a2
 
0 1 0 1 00
T = 2 (sin T ) + T
R sin sin2
R2 T 0 1 00
 
1 0 0
= (sin ) + =
a2 T sin sin2
sin 00
(sin 0 )0 + sin2 = =

We have differential equations for each of T , and .
a2 1  
T 0 = T, (sin 0 )0 + = 0, 00 + = 0
R2 sin sin2
2. In order that the solution be continuously differentiable, we need the periodic boundary conditions
(0) = (2), 0 (0) = 0 (2).
The eigenvalues and eigenfunctions for are
1
n = n2 , n = en , n Z.
2
Now we deal with the equation for .
d 1 d
x = cos , () = P (x), sin2 = 1 x2 , =
dx sin d
1 2 1 0 0
 
(sin ) + =0
sin sin  sin2
 0 n2
1 x2 P 0 + P =0
1 x2
P (x) should be bounded at the endpoints, x = 1 and x = 1.

1810

3. If the solution does not depend on , then the only one of the n that will appear in the solution is 0 = 1/ 2.
The equations for T and P become
 0
1 x2 P 0 + P = 0, P (1) bounded,
a2
T 0 = 2 T.
R

The solutions for P are the Legendre polynomials.

l = l(l + 1), Pl (cos ), l Z0+

We solve the differential equation for T .

a2
T 0 = l(l + 1) 2 T
 2 R 
a l(l + 1)
Tl = exp t
R2

The eigensolutions of the partial differential equation are

a2 l(l + 1)
 
l = Pl (cos ) exp t .
R2

The solution is a linear combination of the eigensolutions.


a2 l(l + 1)
X  
= Al Pl (cos ) exp t
l=0
R2

1811
4. We determine the coefficients in the expansion from the initial condition.

(, 0) = 2 cos2 1

X
Al Pl (cos ) = 2 cos2 1
l=0
 
3 2 1
A0 + A1 cos + A2 cos + = 2 cos2 1
2 2
1 4
A0 = , A1 = 0, A2 = , A3 = A4 = = 0
3 3
6a2
 
1 4
(, t) = P0 (cos ) + P2 (cos ) exp 2 t
3 3 R
6a2
   
1 2 2
(, t) = + 2 cos exp 2 t
3 3 R

Solution 37.34
Since we have homogeneous boundary conditions at x = 0 and x = 1, we will expand the solution in a series of
eigenfunctions in x. We determine a suitable set of eigenfunctions with the separation of variables, = X(x)Y (y).

xx + yy = 0 (37.17)
X 00 Y 00
= =
X Y
We have differential equations for X and Y .

X 00 + X = 0, X(0) = X(1) = 0
Y 00 Y = 0, Y (0) = 0

The eigenvalues and orthonormal eigenfunctions for X are



n = (n)2 , Xn (x) = 2 sin(nx), n Z+ .

1812
The solutions for Y are, (up to a multiplicative constant),

Yn (y) = sinh(ny).

The solution of Equation 37.17 is a linear combination of the eigensolutions.


X
(x, y) = an 2 sin(nx) sinh(ny)
n=1

We determine the coefficients from the boundary condition at y = 2.


X
x(1 x) = an 2 sin(nx) sinh(n2)
n=1
Z 1
an sinh(2n) = 2 x(1 x) sin(nx) dx
0

2 2(1 (1)n )
an = 3 3
n sinh(2n)

8 X 1 sinh(ny)
(x, y) = 3 3
sin(nx)
n=1 n sinh(2n)
odd n

The solution at x = 1/2, y = 1 is



8 X 1 sinh(n)
(1/2, 1) = 3 .
n=1 n3 sinh(2n)
odd n

1813
Let Rk be the relative error at that point incurred by taking k terms.

83 P

1 sinh(n)
n=k+2 3
n sinh(2n)
odd n
Rk = 8 P 1 sinh(n)

3 n=1 n3 sinh(2n)
odd n
P 1 sinh(n)
n=k+2 n3 sinh(2n)
Rk = Podd

n
1 sinh(n)
n=1 n3 sinh(2n)
odd n

Since R1 0.0000693169 we see that one term is sufficient for 1% or 0.1% accuracy.
Now consider x (1/2, 1).


8 X 1 sinh(ny)
x (x, y) = 2 cos(nx)
n=1 n2 sinh(2n)
odd n
x (1/2, 1) = 0

Since all the terms in the series are zero, accuracy is not an issue.

Solution 37.35
The solution has the form
(
rn1 Pnm (cos ) sin(m), r > a
=
rn Pnm (cos ) sin(m), r < a.

The boundary condition on at r = a gives us the constraint

an1 an = 0
= a2n1 .

1814
Then we apply the boundary condition on r at r = a.
(n + 1)an2 na2n1 an1 = 1
an+2
=
2n + 1
( n+2
a
2n+1 rn1 Pnm (cos ) sin(m), r > a
= n+1
a2n+1 rn Pnm (cos ) sin(m), r<a

Solution 37.36
We expand the solution in a Fourier series.

1 X X
= a0 (r) + an (r) cos(n) + bn (r) sin(n)
2 n=1 n=1

We substitute the series into the Laplaces equation to determine ordinary differential equations for the coefficients.
1 2
 

r + 2 2 =0
r r r
1 1 1
a000 + a00 = 0, a00n + a0n n2 an = 0, b00n + b0n n2 bn = 0
r r r
The solutions that are bounded at r = 0 are, (to within multiplicative constants),
a0 (r) = 1, an (r) = rn , bn (r) = rn .
Thus (r, ) has the form

1 X
n
X
(r, ) = c0 + cn r cos(n) + dn rn sin(n)
2 n=1 n=1
We apply the boundary condition at r = R.

X
X
n1
r (R, ) = ncn R cos(n) + ndn Rn1 sin(n)
n=1 n=1

1815
In order that r (R, ) have a Fourier series of this form, it is necessary that
Z 2
r (R, ) d = 0.
0

In that case c0 is arbitrary in our solution. The coefficients are


Z 2 Z 2
1 1
cn = r (R, ) cos(n) d, dn = r (R, ) sin(n) d.
nRn1 0 nRn1 0
We substitute the coefficients into our series solution to determine it up to the additive constant.

R X 1  r n 2
Z
(r, ) = r (R, ) cos(n( )) d
n=1 n R 0

R 2
Z X 1  r n
(r, ) = r (R, ) cos(n( )) d
0 n=1
n R
Z 2 Z r n1
R X n()

(r, ) = r (R, ) d< e d
0 n=1 0
Rn

!
R 2
Z r X
n n()
Z
1
(r, ) = r (R, )< n
e d d
0 0 n=1 R
!
R 2 1 R e()
Z Z r
(r, ) = r (R, )< () d d
0 0 1 R e

R 2
Z   r 
(r, ) = r (R, )< ln 1 e() d
0 R
R 2
Z r
(r, ) = r (R, ) ln 1 e() d

0 R
R 2 r2
Z  
r
(r, ) = r (R, ) ln 1 2 cos( ) + 2 d
2 0 R R

1816
Solution 37.37
We will assume that both and are nonzero. The cases of real and pure imaginary have already been covered.
We solve the ordinary differential equations, (up to a multiplicative constant), to find special solutions of the diffusion
equation.

T0 X 00 ( + )2
= ( + )2 , = 2
T X a 
+
T = exp ( + )2 t ,

X = exp x
a
 
2 2
 
T = exp t + 2t , X = exp x x
a a
  

= exp 2 2 t x + 2t x

a a

We take the sum and difference of these solutions to obtain


 

2 2
  cos
= exp t x 2t x
a sin a

1817
1818
m=0, n=1 m=0, n=2 m=0, n=3

m=1, n=1 m=1, n=2 m=1, n=3

m=2, n=1 m=2, n=2 m=2, n=3

mx ny
 
Figure 37.3: The eigenfunctions cos a
sin b

1819
m=1,n=1 m=1,n=2 m=1,n=3 m=1,n=1 m=1,n=2 m=1,n=3

m=2,n=1 m=2,n=2 m=2,n=3 m=2,n=1 m=2,n=2 m=2,n=3

m=3,n=1 m=3,n=2 m=3,n=3 m=3,n=1 m=3,n=2 m=3,n=3

Figure 37.4: The modes of oscillation of a rectangular drum head.

1820
Chapter 38

Finite Transforms

Example 38.0.1 Consider the problem

1 2u
u 2 2 = (x )(y ) et on < x < , 0 < y < b,
c t
with
uy (x, 0, t) = uy (x, b, t) = 0.
Substituting u(x, y, t) = v(x, y) et into the partial differential equation yields the problem

v + k 2 v = (x )(y ) on < x < , 0 < y < b,

with
vy (x, 0) = vy (x, b) = 0.
We assume that the solution has the form

1 X  ny 
v(x, y) = c0 (x) + cn (x) cos , (38.1)
2 n=1
b

1821
and apply a finite cosine transform in the y direction. Integrating from 0 to b yields
Z b Z b
2
vxx + vyy + k v dy = (x )(y ) dy,
0 0

 b
Z b
vy 0 + vxx + k 2 v dy = (x ),
0
Z b
vxx + k 2 v dy = (x ).
0

Substituting in Equation 38.1 and using the orthogonality of the cosines gives us

2
c000 (x) + k 2 c0 (x) = (x ).
b

Multiplying by cos(ny/b) and integrating form 0 to b yields


Z b  ny  Z b  ny 
2

vxx + vyy + k v cos dy = (x )(y ) cos dy.
0 b 0 b

The vyy term becomes


Z b  ny   ny ib Z b
h n  ny 
vyy cos dy = vy cos vy sin dy
0 b b 0 0 b b
h n  ny ib Z b  n 2  ny 
= v sin v cos dy.
b b 0 0 b b

The right-hand-side becomes


Z b  ny   n 
(x )(y ) cos dy = (x ) cos .
0 b b

1822
Thus the partial differential equation becomes
Z b  n 2   ny   n 
2
vxx v + k v cos dy = (x ) cos .
0 b b b
Substituting in Equation 38.1 and using the orthogonality of the cosines gives us
  n 2 
00 2 2  n 
cn (x) + k cn (x) = (x ) cos .
b b b

Now we need to solve for the coefficients in the expansion of v(x, y). The homogeneous solutions for c0 (x) are
ekx . The solution for u(x, y, t) must satisfy the radiation condition. The waves at x = travel to the left and the
waves at x = + travel to the right. The two solutions of that will satisfy these conditions are, respectively,

y1 = ekx , y2 = ekx .

The Wronskian of these two solutions is 2k. Thus the solution for c0 (x) is

ekx< ekx>
c0 (x) =
bk
We need to consider three cases for the equation for cn .
p
k > n/b Let = k 2 (n/b)2 . The homogeneous solutions that satisfy the radiation condition are

y1 = ex , y2 = ex .

The Wronskian of the two solutions is 2. Thus the solution is


ex< ex>  n 
cn (x) = cos .
b b
n

In the case that cos b
= 0 this reduces to the trivial solution.

1823
k = n/b The homogeneous solutions that are bounded at infinity are

y1 = 1, y2 = 1.

If the right-hand-side is nonzero there is no wayto combine these solutions to satisfy both the continuity and
the derivative jump conditions. Thus if cos nb
6 0 there is no bounded solution. If cos n
= b
= 0 then the
solution is not unique.
cn (x) = const.
p
k < n/b Let = (n/b)2 k 2 . The homogeneous solutions that are bounded at infinity are

y1 = ex , y2 = ex .

The Wronskian of these solutions is 2. Thus the solution is

ex< ex>  n 
cn (x) = cos
b b
n

In the case that cos b
= 0 this reduces to the trivial solution.

1824
38.1 Exercises
Exercise 38.1
A slab is perfectly insulated at the surface x = 0 and has a specified time varying temperature f (t) at the surface
x = L. Initially the temperature is zero. Find the temperature u(x, t) if the heat conductivity in the slab is = 1.

Exercise 38.2
Solve

uxx + uyy = 0, 0 < x < L, y > 0,


u(x, 0) = f (x), u(0, y) = g(y), u(L, y) = h(y),

with an eigenfunction expansion.

1825
38.2 Hints
Hint 38.1

Hint 38.2

1826
38.3 Solutions
Solution 38.1
The problem is

ut = uxx , 0 < x < L, t > 0,


ux (0, t) = 0, u(L, t) = f (t), u(x, 0) = 0.

We will solve this problem with an eigenfunction expansion. We find these eigenfunction by replacing the inhomogeneous
boundary condition with the homogeneous one, u(L, t) = 0. We substitute the separation of variables v(x, t) =
X(x)T (t) into the homogeneous partial differential equation.

XT 0 = X 00 T
T0 X 00
= = 2 .
T X
This gives us the regular Sturm-Liouville eigenvalue problem,

X 00 = 2 X, X 0 (0) = X(L) = 0,

which has the solutions,  


(2n 1) (2n 1)x
n = , Xn = cos , n N.
2L 2L
Our solution for u(x, t) will be an eigenfunction expansion in these eigenfunctions. Since the inhomogeneous boundary
condition is a function of t, the coefficients will be functions of t.

X
u(x, t) = an (t) cos(n x)
n=1

Since u(x, t) does not satisfy the homogeneous boundary conditions of the eigenfunctions, the series is not uniformly
convergent and we are not allowed to differentiate it with respect to x. We substitute the expansion into the partial

1827
differential equation, multiply by the eigenfunction and integrate from x = 0 to x = L. We use integration by parts to
move derivatives from u to the eigenfunctions.
ut = uxx
Z L Z L
ut cos(m x) dx = uxx cos(m x) dx
0 0

!
Z L X Z L
a0n (t) cos(n x) cos(m x) dx = [ux cos(m x)]L0 + ux m sin(m x) dx
0 n=1 0
Z L
L 0 L
am (t) = [um sin(m x)]0 u2m cos(m x) dx
2 0

Z L X !
L 0 2
a (t) = m u(L, t) sin(m L) m an (t) cos(n x) cos(m x) dx
2 m 0 n=1
L 0 L
am (t) = m (1)n f (t) 2m am (t)
2 2
0 2 n
am (t) + m am (t) = (1) m f (t)
From the initial condition u(x, 0) = 0 we see that am (0) = 0. Thus we have a first order differential equation and an
initial condition for each of the am (t).
a0m (t) + 2m am (t) = (1)n m f (t), am (0) = 0
This equation has the solution,
Z t
2
n
am (t) = (1) m em (t ) f ( ) d.
0

Solution 38.2

uxx + uyy = 0, 0 < x < L, y > 0,


u(x, 0) = f (x), u(0, y) = g(y), u(L, y) = h(y),

1828
We seek a solution of the form,

X  nx 
u(x, y) = un (y) sin .
n=1
L

Since we have inhomogeneous boundary conditions at x = 0, L, we cannot differentiate the series representation with
respect to x. We multiply Laplaces equation by the eigenfunction and integrate from x = 0 to x = L.
Z L  mx 
(uxx + uyy ) sin dx = 0
0 L

We use integration by parts to move derivatives from u to the eigenfunctions.

m L
 mx iL Z  mx 
h L
ux sin ux cos dx + u00m (y) = 0
L 0 L 0 L 2
h m  mx iL  m 2 Z L  mx  L
u cos u sin dx + u00m (y) = 0
L L 0 L 0 L 2
m m L  m 2 L
h(y)(1)m + g(y) um (y) + u00m (y) = 0
L L 2 L 2
 m 2
00 m
um (y) um (y) = 2m ((1) h(y) g(y))
L
Now we have an ordinary differential equation for the un (y). In order that the solution is bounded, we require that each
un (y) is bounded as y . We use the boundary condition u(x, 0) = f (x) to determine boundary conditions for the
um (y) at y = 0.

X  nx 
u(x, 0) = un (0) sin = f (x)
n=1
L
Z L
2  nx 
un (0) = fn f (x) sin dx
L 0 L

1829
Thus we have the problems,
 n 2
00
un (y) un (y) = 2n ((1)n h(y) g(y)) , un (0) = fn , un (+) bounded,
L
for the coefficients in the expansion. We will solve these with Green functions. Consider the associated Green function
problem  n 2
G00n (y; ) Gn (y; ) = (y ), Gn (0; ) = 0, Gn (+; ) bounded.
L
The homogeneous solutions that satisfy the boundary conditions are
 ny 
sinh and eny/L ,
L
respectively. The Wronskian of these solutions is
sinh ny
 ny/L

e n 2ny/L
sinh ny n eny/L = L e
n L  .
L L L

Thus the Green function is


L sinh ny
 ny /L
L
<
e >

Gn (y; ) = .
n e2n/L
Using the Green function we determine the un (y) and thus the solution of Laplaces equation.
Z
ny/L
un (y) = fn e +2n Gn (y; ) ((1)n h() g()) d
0

X  nx 
u(x, y) = un (y) sin .
n=1
L

1830
Chapter 39

The Diffusion Equation

1831
39.1 Exercises
Exercise 39.1
Is the solution of the Cauchy problem for the heat equation unique?

ut uxx = q(x, t), < x < , t>0


u(x, 0) = f (x)

Exercise 39.2
Consider the heat equation with a time-independent source term and inhomogeneous boundary conditions.

ut = uxx + q(x)
u(0, t) = a, u(h, t) = b, u(x, 0) = f (x)

Exercise 39.3
Is the Cauchy problem for the backward heat equation

ut + uxx = 0, u(x, 0) = f (x) (39.1)

well posed?
Exercise 39.4
Derive the heat equation for a general 3 dimensional body, with non-uniform density (x), specific heat c(x), and
conductivity k(x). Show that
u(x, t) 1
= (ku(x, t))
t c
where u is the temperature, and you may assume there are no internal sources or sinks.
Exercise 39.5
Verify Duhamels Principal: If u(x, t, ) is the solution of the initial value problem:

ut = uxx , u(x, 0, ) = f (x, ),

1832
then the solution of
wt = wxx + f (x, t), w(x, 0) = 0
is Z t
w(x, t) = u(x, t , ) d.
0

Exercise 39.6
Modify the derivation of the diffusion equation

k
t = a2 xx , a2 = , (39.2)
c
so that it is valid for diffusion in a non-homogeneous medium for which c and k are functions of x and and so that
it is valid for a geometry in which A is a function of x. Show that Equation (39.2) above is in this case replaced by

cAt = (kAx )x .

Recall that c is the specific heat, k is the thermal conductivity, is the density, is the temperature and A is the
cross-sectional area.

1833
39.2 Hints
Hint 39.1

Hint 39.2

Hint 39.3

Hint 39.4

Hint 39.5
Check that the expression for w(x, t) satisfies the partial differential equation and initial condition. Recall that
Z x Z x

h(x, ) d = hx (x, ) d + h(x, x).
x a a

Hint 39.6

1834
39.3 Solutions
Solution 39.1
Let u and v both be solutions of the Cauchy problem for the heat equation. Let w be the difference of these solutions.
w satisfies the problem

wt wxx = 0, < x < , t > 0,


w(x, 0) = 0.

We can solve this problem with the Fourier transform.

wt + 2 w = 0, w(, 0) = 0
w = 0
w=0

Since u v = 0, we conclude that the solution of the Cauchy problem for the heat equation is unique.
Solution 39.2
Let (x) be the equilibrium temperature. It satisfies an ordinary differential equation boundary value problem.

q(x)
00 = , (0) = a, (h) = b

To solve this boundary value problem we find a particular solution p that satisfies homogeneous boundary conditions
and then add on a homogeneous solution h that satisfies the inhomogeneous boundary conditions.
q(x)
00p = , p (0) = p (h) = 0

00h = 0, h (0) = a, h (h) = b

We find the particular solution p with the method of Green functions.

G00 = (x ), G(0|) = G(h|) = 0.

1835
We find homogeneous solutions which respectively satisfy the left and right homogeneous boundary conditions.
y1 = x, y2 = h x
Then we compute the Wronskian of these solutions and write down the Green function.

x h x
W = = h
1 1
1
G(x|) = x< (h x> )
h
The homogeneous solution that satisfies the inhomogeneous boundary conditions is
ba
h = a + x
h
Now we have the equilibrium temperature.
Z h  
ba 1 q()
=a+ x+ x< (h x> ) d
h 0 h
hx x
Z h
ba
Z
x
=a+ x+ q() d + (h )q() d
h h 0 h x
Let v denote the deviation from the equilibrium temperature.
u=+v
v satisfies a heat equation with homogeneous boundary conditions and no source term.
vt = vxx , v(0, t) = v(h, t) = 0, v(x, 0) = f (x) (x)
We solve the problem for v with separation of variables.
v = X(x)T (t)
XT 0 = X 00 T
T0 X 00
= =
T X

1836
We have a regular Sturm-Liouville problem for X and a differential equation for T .
X 00 + X = 0, X(0) = X() = 0
 n 2  nx 
n = , Xn = sin , n Z+
h h
T 0 = T
  n 2 
Tn = exp t
h
v is a linear combination of the eigensolutions.

X  nx    n 2 
v= vn sin exp t
n=1
h h
The coefficients are determined from the initial condition, v(x, 0) = f (x) (x).
2 h
Z  nx 
vn = (f (x) (x)) sin dx
h 0 h
We have determined the solution of the original problem in terms of the equilibrium temperature and the deviation
from the equilibrium. u = + v.
Solution 39.3
A problem is well posed if there exists a unique solution that depends continiously on the nonhomogeneous data.
First we find some solutions of the differential equation with the separation of variables u = X(x)T (t).
ut + uxx = 0, > 0
XT 0 + X 00 T = 0
T0 X 00
= =
T X
X 00 + X = 0, T 0 = T
   
u = cos x et , u = sin x et

1837
Note that  
u =  cos x et
satisfies the Cauchy problem  
ut + uxx = 0, u(x, 0) =  cos x
Consider   1. The initial condition is small, it satisfies |u(x, 0)| < . However the solution for any positive time
can be made arbitrarily large by choosing a sufficiently large, positive value of . We can make the solution exceed the
value M at time t by choosing a value of such that
 et > M
 
1 M
> ln .
t 
Thus we see that Equation 39.1 is ill posed because the solution does not depend continuously on the initial data. A
small change in the initial condition can produce an arbitrarily large change in the solution for any fixed time.
Solution 39.4
Consider a Region of material, R. Let u be the temperature and be the heat flux. The amount of heat energy in
the region is Z
cu dx.
R
We equate the rate of change of heat energy in the region with the heat flux across the boundary of the region.
Z Z
d
cu dx = n ds
dt R R

We apply the divergence theorem to change the surface integral to a volume integral.
Z Z
d
cu dx = dx
dt R R
Z  
u
c + dx = 0
R t

1838
Since the region is arbitrary, the integral must vanish identically.

u
c =
t

We apply Fouriers law of heat conduction, = ku, to obtain the heat equation.

u 1
= (ku)
t c

Solution 39.5
We verify Duhamels principal by showing that the integral expression for w(x, t) satisfies the partial differential equation
and the initial condition. Clearly the initial condition is satisfied.
Z 0
w(x, 0) = u(x, 0 , ) d = 0
0

Now we substitute the expression for w(x, t) into the partial differential equation.

t Z t
2
Z

u(x, t , ) d = 2 u(x, t , ) d + f (x, t)
t 0 x 0
Z t Z t
u(x, t t, t) + ut (x, t , ) d = uxx (x, t , ) d + f (x, t)
0 0
Z t Z t
f (x, t) + ut (x, t , ) d = uxx (x, t , ) d + f (x, t)
0 0
Z t
(ut (x, t , ) d uxx (x, t , )) d
0

Since ut (x, t , ) d uxx (x, t , ) = 0, this equation is an identity.

1839
Solution 39.6
We equate the rate of change of thermal energy in the segment ( . . . ) with the heat entering the segment through
the endpoints.
Z
t cA dx = k(, ())A()x (, t) k(, ())A()x (, t)

Z
t cA dx = [kAx ]

Z Z
t cA dx = (kAx )x dx

Z
cAt (kAx )x dx = 0

Since the domain is arbitrary, we conclude that

cAt = (kAx )x .

1840
Chapter 40

Laplaces Equation

40.1 Introduction
Laplaces equation in n dimensions is
u = 0
where
2 2
= + + .
x21 x2n
The inhomogeneous analog is called Poissons Equation.
u = f (x)
CONTINUE

40.2 Fundamental Solution


The fundamental solution of Poissons equation in Rn satisfies
G = (x ).

1841
40.2.1 Two Dimensional Space
If n = 2 then the fundamental solution satisfies
 2
2


+ G = (x )(y ).
x2 y 2

Since the product of delta functions, (x )(y


p ) is circularly symmetric about the point (, ), we look for a
solution in the form u(x, y) = v(r) where r = ((x )2 + (y )2 ).
CONTINUE

1842
40.3 Exercises
Exercise 40.1
Is the solution of the following Dirichlet problem unique?
uxx + uyy = q(x, y), < x < , y>0
u(x, 0) = f (x)
Exercise 40.2
Is the solution of the following Dirichlet problem unique?
uxx + uyy = q(x, y), < x < , y > 0
u(x, 0) = f (x), u bounded as x2 + y 2
Exercise 40.3
Not all combinations of boundary conditions/initial conditions lead to so called well-posed problems. Essentially, a well
posed problem is one where the solutions depend continuously on the boundary data. Otherwise it is considered ill
posed.
Consider Laplaces equation on the unit-square
uxx + uyy = 0,
with u(0, y) = u(1, y) = 0 and u(x, 0) = 0, uy (x, 0) =  sin(nx).
1. Show that even as  0, you can find n so that the solution can attain any finite value for any y > 0. Use this
to then show that this problem is ill posed.
2. Contrast this with the case where u(0, y) = u(1, y) = 0 and u(x, 0) = 0, u(x, 1) =  sin(nx). Is this well
posed?
Exercise 40.4
Use the fundamental solutions for the Laplace equation
2 G = (x )

1843
in three dimensions
1
G(x|) =
4|x |
to derive the mean value theorem for harmonic functions
Z
1
u(p) = u() dA ,
4R2 SR

that relates the value of any harmonic function u(x) at the point x = p to the average of its value on the boundary
of the sphere of radius R with center at p, (SR ).

Exercise 40.5
Use the fundamental solutions for the modified Helmholz equation

2 u u = (x )

in three dimensions
1
u (x|) = e |x| ,
4|x |
to derive a generalized mean value theorem:
 
sinh R 1
Z
u(p) = u(x) dA
R 4R2 S

that relates the value of any solution u(x) at a point P to the average of its value on the sphere of radius R (S) with
center at P.
Exercise 40.6
Consider the uniqueness of solutions of 2 u(x) = 0 in a two dimensional region R with boundary curve C and a
boundary condition n u(x) = a(x)u(x) on C. State a non-trivial condition on the function a(x) on C for which
solutions are unique, and justify your answer.

1844
Exercise 40.7
Solve Laplaces equation on the surface of a semi-infinite cylinder of unit radius, 0 < < 2, z > 0, where the solution,
u(, z) is prescribed at z = 0: u(, 0) = f ().

Exercise 40.8
Solve Laplaces equation in a rectangle.

wxx + wyy = 0, 0 < x < a, 0 < y < b,


w(0, y) = f1 (y), w(a, y) = f2 (y),
wy (x, 0) = g1 (x), w(x, b) = g2 (x)

Proceed by considering w = u + v where u and v are harmonic and satisfy

u(0, y) = u(a, y) = 0, uy (x, 0) = g1 (x), u(x, b) = g2 (x),


v(0, y) = f1 (y), v(a, y) = f2 (y), vy (x, 0) = v(x, b) = 0.

1845
40.4 Hints
Hint 40.1

Hint 40.2

Hint 40.3

Hint 40.4

Hint 40.5

Hint 40.6

Hint 40.7

Hint 40.8

1846
40.5 Solutions
Solution 40.1
Let u and v both be solutions of the Dirichlet problem. Let w be the difference of these solutions. w satisfies the
problem

wxx + wyy = 0, < x < , y>0


w(x, 0) = 0.

Since w = cy is a solution. We conclude that the solution of the Dirichlet problem is not unique.

Solution 40.2
Let u and v both be solutions of the Dirichlet problem. Let w be the difference of these solutions. w satisfies the
problem

wxx + wyy = 0, < x < , y > 0


w(x, 0) = 0, w bounded as x2 + y 2 .

We solve this problem with a Fourier transform in x.

2 w + wyy = 0, w(, 0) = 0, w bounded as y


(
c1 cosh y + c2 sinh(y), 6= 0
w =
c1 + c2 y, =0
w = 0
w=0

Since u v = 0, we conclude that the solution of the Dirichlet problem is unique.

1847
Solution 40.3
1. We seek a solution of the form u(x, y) = sin(nx)Y (y). This form satisfies the boundary conditions at x = 0, 1.
uxx + uyy = 0
(n) Y + Y 00 = 0, Y (0) = 0
2

Y = c sinh(ny)
Now we apply the inhomogeneous boundary condition.
uy (x, 0) =  sin(nx) = cn sin(nx)

u(x, y) = sin(nx) sinh(ny)
n
For  = 0 the solution is u = 0. Now consider any  > 0. For any y > 0 and any finite value M , we can choose
a value of n such that the solution along y = 0 takes on all values in the range [M . . . M ]. We merely choose
a value of n such that
sinh(ny) M
.
n 
Since the solution does not depend continuously on boundary data, this problem is ill posed.
2. We seek a solution of the form u(x, y) = c sin(nx) sinh(ny). This form satisfies the differential equation and
the boundary conditions at x = 0, 1 and at y = 0. We apply the inhomogeneous boundary condition at y = 1.
u(x, 1) =  sin(nx) = c sin(nx) sinh(n)
sinh(ny)
u(x, y) =  sin(nx)
sinh(n)
For  = 0 the solution is u = 0. Now consider any  > 0. Note that |u|  for (x, y) [0 . . . 1] [0 . . . 1]. The
solution depends continuously on the given boundary data. This problem is well posed.
Solution 40.4
The Green function problem for a sphere of radius R centered at the point is

G = (x ), G |x|=R = 0. (40.1)

1848
We will solve Laplaces equation, u = 0, where the value of u is known on the boundary of the sphere of radius R in
terms of this Green function.
First we solve for u(x) in terms of the Green function.
Z Z
(uG Gu) d = u(x ) d = u(x)
S S

Z Z  
G u
(uG Gu) d = u G dA
S n n
ZS
G
= u dA
S n

Z
G
u(x) = u dA
S n
We are interested in the value of u at the center of the sphere. Let = |p |
Z
G
u(p) = u() (p|) dA
S

We do not need to compute the general solution of Equation 40.1. We only need the Green function at the point
x = p. We know that the general solution of the equation G = (x ) is

1
G(x|) = + v(x),
4|x |

where v(x) is an arbitrary harmonic function. The Green function at the point x = p is

1
G(p|) = + const.
4|p |

1849
We add the constraint that the Green function vanishes at = R. This determines the constant.
1 1
G(p|) = +
4|p | 4R
1 1
G(p|) = +
4 4R
1
G (p|) =
42

Now we are prepared to write u(p) in terms of the Green function.


Z
1
u(p) = u() dA
S 42
Z
1
u(p) = u() dA
4R2 S
This is the Mean Value Theorem for harmonic functions.
Solution 40.5
The Green function problem for a sphere of radius R centered at the point is

G G = (x ), G |x|=R = 0. (40.2)

We will solve the modified Helmholtz equation,

u u = 0,

where the value of u is known on the boundary of the sphere of radius R in terms of this Green function.
in terms of this Green function.
Let L[u] = u u. Z Z
(uL[G] GL[u]) d = u(x ) d = u(x)
S S

1850
Z Z
(uL[G] GL[u]) d = (uG Gu) d
S S
Z  
G u
= u G dA
S n n
Z
G
= u dA
S n

Z
G
u(x) = u dA
S n

We are interested in the value of u at the center of the sphere. Let = |p |

Z
G
u(p) = u() (p|) dA
S

We do not need to compute the general solution of Equation 40.2. We only need the Green function at the point
x = p. We know that the Green function there is a linear combination of the fundamental solutions,

1 1
G(p|) = c1 e |p| +c2 e |p| ,
4|p | 4|p |

such that c1 + c2 = 1. The Green function is symmetric with respect to x and . We add the constraint that the Green

1851
function vanishes at = R. This gives us two equations for c1 and c2 .
c1 R c2 R
c1 + c2 = 1, e e =0
4R 4R
1 e2 R
c1 = , c2 =
e2 R 1  e2 R1

sinh ( R)
G(p|) =  
4 sinh R
   
cosh ( R) sinh ( R)
G (p|) =    
4 sinh R 2
4 sinh R


G (p|) ||=R =  
4R sinh R

Now we are prepared to write u(p) in terms of the Green function.


Z

u(p) = u()   dA
S 4 sinh R
Z

u(p) = u(x)   dA
S 4R sinh R

Rearranging this formula gives us the generalized mean value theorem.


 
sinh R 1
Z
u(p) = u(x) dA
R 4R2 S

1852
Solution 40.6
First we think of this problem in terms of the the equilibrium solution of the heat equation. The boundary condition
expresses Newtons law of cooling. Where a = 0, the boundary is insulated. Where a > 0, the rate of heat loss is
proportional to the temperature. The case a < 0 is non-physical and we do not consider this scenario further. We
know that if the boundary is entirely insulated, a = 0, then the equilibrium temperature is a constant that depends on
the initial temperature distribution. Thus for a = 0 the solution of Laplaces equation is not unique. If there is any
point on the boundary where a is positive then eventually, all of the heat will flow out of the domain. The equilibrium
temperature is zero, and the solution of Laplaces equation is unique, u = 0. Therefore the solution of Laplaces
equation is unique if a is continuous, non-negative and not identically zero.
Now we prove our assertion. First note that if we substitute f = vu in the divergence theorem,
Z Z
f dx = f n ds,
R R

we obtain the identity, Z Z


u
(vu + vu) dx = v ds. (40.3)
R R n
Let u be a solution of Laplaces equation subject to the Robin boundary condition with our restrictions on a. We take
v = u in Equation 40.3. Z Z Z
2 u
(u) dx = u ds = au2 ds
R C n C

Since the first integral is non-negative and the last is non-positive, the integrals vanish. This implies that u = 0. u is
a constant. In order to satisfy the boundary condition where a is non-zero, u must be zero. Thus the unique solution
in this scenario is u = 0.
Solution 40.7
The mathematical statement of the problem is

u u + uzz = 0, 0 < < 2, z > 0,


u(, 0) = f ().

1853
We have the implicit boundary conditions,

u(0, z) = u(2, z), u (0, z) = u (0, z)

and the boundedness condition,


u(, +) bounded.
We expand the solution in a Fourier series. (This ensures that the boundary conditions at = 0, 2 are satisfied.)

X
u(, z) = un (z) en
n=

We substitute the series into the partial differential equation to obtain ordinary differential equations for the un .

n2 un (z) + u00n (z) = 0

The general solutions of this equation are


(
c1 + c2 z, for n = 0,
un (z) =
c1 enz +c2 enz 6 0.
for n =

The bounded solutions are


nz
c e , for n > 0,

un (z) = c, for n = 0, = c e|n|z .

nz
ce , for n < 0,
We substitute the series into the initial condition at z = 0 to determine the multiplicative constants.

X
u(, 0) = un (0) en = f ()
n=
Z 2
1
un (0) = f () en d fn
2 0

1854
Thus the solution is

X
u(, z) = fn en e|n|z .
n=

Note that Z 2
1
u(, z) f0 = f () d
2 0
as z +.
Solution 40.8
The decomposition of the problem is shown in Figure 40.1.
w=g2(x) u=g2(x) v=0

w=f1(y) w=0 w=f2(y) = u=0 u=0 u=0 + v=f1(y) v=0 v=f2(y)

wy=g1(x) uy=g1(x) vy=0

Figure 40.1: Decomposition of the problem.


First we solve the problem for u.
uxx + uyy = 0, 0 < x < a, 0 < y < b,
u(0, y) = u(a, y) = 0,
uy (x, 0) = g1 (x), u(x, b) = g2 (x)
We substitute the separation of variables u(x, y) = X(x)Y (y) into Laplaces equation.
X 00 Y 00
= = 2
X Y

1855
We have the eigenvalue problem,

X 00 = 2 X, X(0) = X(a) = 0,

which has the solutions,


n  nx 
n = , Xn = sin , n N.
a a

The equation for Y (y) becomes,


 n 2
Yn00 = Yn ,
a

which has the solutions,


n  ny   ny o
eny/a , eny/a

or cosh , sinh .
a a

It will be convenient to choose solutions that satisfy the conditions, Y (b) = 0 and Y 0 (0) = 0, respectively.

    ny 
n(b y)
sinh , cosh
a a

The solution for u(x, y) has the form,

 nx      ny 
X n(b y)
u(x, y) = sin n sinh + n cosh .
n=1
a a a

We determine the coefficients from the inhomogeneous boundary conditions. (Here we see how our choice of solutions

1856
for Y (y) is convenient.)
 
X n  nx  nb
uy (x, 0) = n sin cosh = g1 (x)
n=1
a a a
nb 2 a
  Z
a  nx 
n = sech g1 (x) sin dx
n a a 0 a
X  nx   ny 
u(x, y) = n sin cosh
a a
 n=1  Z a
nb 2  nx 
n = sech g2 (x) sin dx
a a 0 a
Now we solve the problem for v.
vxx + vyy = 0, 0 < x < a, 0 < y < b,
v(0, y) = f1 (y), v(a, y) = f2 (y),
vy (x, 0) = 0, v(x, b) = 0
We substitute the separation of variables u(x, y) = X(x)Y (y) into Laplaces equation.
X 00 Y 00
= = 2
X Y
We have the eigenvalue problem,
Y 00 = 2 Y, Y 0 (0) = Y (b) = 0,
which has the solutions,  
(2n 1) (2n 1)y
n = , Yn = cos , n N.
2b 2b
The equation for X(y) becomes,
 2
(2n 1)
Xn00 = Xn .
2b

1857
We choose solutions that satisfy the conditions, X(a) = 0 and X(0) = 0, respectively.
    
(2n 1)(a x) (2n 1)x
sinh , sinh
2b 2b

The solution for v(x, y) has the form,


     
X (2n 1)y (2n 1)(a x) (2n 1)x
v(x, y) = cos n sinh + n sinh .
n=1
2b 2b 2b

We determine the coefficients from the inhomogeneous boundary conditions.


   
X (2n 1)y (2n 1)a
v(0, y) = n cos sinh = f1 (y)
n=1
2b 2b
(2n 1)a 2 b
  Z  
(2n 1)y
n = csch f1 (y) cos dy
2b b 0 2b
   
X (2n 1)y (2n 1)a
v(a, y) = n cos sinh = f2 (y)
n=1
2b 2b
(2n 1)a 2 b
  Z  
(2n 1)y
n = csch f2 (y) cos dy
2b b 0 2b

With u and v determined, the solution of the original problem is w = u + v.

1858
Chapter 41

Waves

1859
41.1 Exercises
Exercise 41.1
Consider the 1-D wave equation
utt uxx = 0
on the domain 0 < x < 4 with initial displacement
(
1, 1 < x < 2
u(x, 0) =
0, otherwise,

initial velocity ut (x, 0) = 0, and subject to the following boundary conditions

1.
u(0, t) = u(4, t) = 0

2.
ux (0, t) = ux (4, t) = 0

In each case plot u(x, t) for t = 12 , 1, 32 , 2 and combine onto a general plot in the x, t plane (up to a sufficiently large
time) so the behavior of u is clear for arbitrary x, t.

Exercise 41.2
Sketch the solution to the wave equation:
Z x+ct
1 1
u(x, t) = (u(x + ct, 0) + u(x ct, 0)) + ut (, 0) d,
2 2c xct

for various values of t corresponding to the initial conditions:

1. u(x, 0) = 0, ut (x, 0) = sin x where is a constant,

1860

1
for 0 < x < 1
2. u(x, 0) = 0, ut (x, 0) = 1 for 1 < x < 0

0 for |x| > 1.

Exercise 41.3
1. Consider the solution of the wave equation for u(x, t):

utt = c2 uxx

on the infinite interval < x < with initial displacement of the form
(
h(x) for x > 0,
u(x, 0) =
h(x) for x < 0,

and with initial velocity


ut (x, 0) = 0.
Show that the solution of the wave equation satisfying these initial conditions also solves the following semi-infinite
problem: Find u(x, t) satisfying the wave equation utt = c2 uxx in 0 < x < , t > 0, with initial conditions
u(x, 0) = h(x), ut (x, 0) = 0, and with the fixed end condition u(0, t) = 0. Here h(x) is any given function with
h(0) = 0.
2. Use a similar idea to explain how you could use the general solution of the wave equation to solve the finite
interval problem (0 < x < l) in which u(0, t) = u(l, t) = 0 for all t, with u(x, 0) = h(x) and ut (x, 0) = 0. Take
h(0) = h(l) = 0.

Exercise 41.4
The deflection u(x, T ) = (x) and velocity ut (x, T ) = (x) for an infinite string (governed by utt = c2 uxx ) are
measured at time T , and we are asked to determine what the initial displacement and velocity profiles u(x, 0) and
ut (x, 0) must have been. An alert student suggests that this problem is equivalent to that of determining the solution
of the wave equation at time T when initial conditions u(x, 0) = (x), ut (x, 0) = (x) are prescribed. Is she correct?
If not, can you rescue her idea?

1861
Exercise 41.5
In obtaining the general solution of the wave equation the interval was chosen to be infinite in order to simplify the
evaluation of the functions () and () in the general solution
u(x, t) = (x + ct) + (x ct).
But this general solution is in fact valid for any interval be it infinite or finite. We need only choose appropriate functions
(), () to satisfy the appropriate initial and boundary conditions. This is not always convenient but there are other
situations besides the solution for u(x, t) in an infinite domain in which the general solution is of use. Consider the
whip-cracking problem,
utt = c2 uxx ,
(with c a constant) in the domain x > 0, t > 0 with initial conditions
u(x, 0) = ut (x, 0) = 0 x > 0,
and boundary conditions
u(0, t) = (t)
prescribed for all t > 0. Here (0) = 0. Find and so as to determine u for x > 0, t > 0.

Hint: (From physical considerations conclude that you can take () = 0. Your solution will corroborate this.)
Use the initial conditions to determine () and () for > 0. Then use the initial condition to determine () for
< 0.
Exercise 41.6
Let u(x, t) satisfy the equation
utt = c2 uxx ;
(with c a constant) in some region of the (x, t) plane.
1. Show that the quantity (ut cux ) is constant along each straight line defined by x ct = constant, and that
(ut + cux ) is constant along each straight line of the form x + ct = constant. These straight lines are called
characteristics; we will refer to typical members of the two families as C+ and C characteristics, respectively.
Thus the line x ct = constant is a C+ characteristic.

1862
2. Let u(x, 0) and ut (x, 0) be prescribed for all values of x in < x < , and let (x0 , t0 ) be some point in the
(x, t) plane, with t0 > 0. Draw the C+ and C characteristics through (x0 , t0 ) and let them intersect the x-axis
at the points A,B. Use the properties of these curves derived in part (a) to determine ut (x0 , t0 ) in terms of initial
data at points A and B. Using a similar technique to obtain ut (x0 , ) with 0 < < t, determine u(x0 , t0 ) by
integration with respect to , and compare this with the solution derived in class:
1 x+ct
Z
1
u(x, t) = (u(x + ct, 0) + u(x ct, 0)) + ut (, 0)d.
2 2c xct
Observe that this method of characteristics again shows that u(x0 , t0 ) depends only on that part of the initial
data between points A and B.
Exercise 41.7
The temperature u(x, t) at a depth x below the Earths surface at time t satisfies
ut = uxx .
The surface x = 0 is heated by the sun according to the periodic rule:
u(0, t) = T cos(t).
Seek a solution of the form
u(x, t) = < A etx .


a) Find u(x, t) satisfying u 0 as x +, (i.e. deep into the Earth).


b) Find the temperature variation at a fixed depth, h, below the surface.
c) Find the phase lag (x) such that when the maximum temperature occurs at t0 on the surface, the maximum at
depth x occurs at t0 + (x).
d) Show that the seasonal, (i.e. yearly), temperature changes and daily temperature changes penetrate to depths in
the ratio:
xyear
= 365,
xday
where xyear and xday are the depths of same temperature variation caused by the different periods of the source.

1863
Exercise 41.8
An infinite cylinder of radius a produces an external acoustic pressure field u satisfying:

utt = c2 u,

by a pure harmonic oscillation of its surface at r = a. That is, it moves so that

u(a, , t) = f () et

where f () is a known function. Note that the waves must be outgoing at infinity, (radiation condition at infinity).
Find the solution, u(r, , t). We seek a periodic solution of the form,

u(r, , t) = v(r, ) et .

Exercise 41.9
Plane waves are incident on a soft cylinder of radius a whose axis is parallel to the plane of the waves. Find the
field scattered by the cylinder. In particular, examine the leading term of the solution when a is much smaller than the
wavelength of the incident waves. If v(x, y, t) is the scattered field it must satisfy:

Wave Equation: vtt = c2 v, x2 + y 2 > a2 ;


Soft Cylinder: v(x, y, t) = e(ka cos t) , on r = a, 0 < 2;
Scattered: v is outgoing as r .

Here k = /c. Use polar coordinates in the (x, y) plane.

Exercise 41.10
Consider the flow of electricity in a transmission line. The current, I(x, t), and the voltage, V (x, t), obey the telegra-
phers system of equations:

Ix = CVt + GV,
Vx = LIt + RI,

1864
where C is the capacitance, G is the conductance, L is the inductance and R is the resistance.
a) Show that both I and V satisfy a damped wave equation.
b) Find the relationship between the physical constants, C, G, L and R such that there exist damped traveling
wave solutions of the form:
V (x, t) = et (f (x at) + g(x + at)).
What is the wave speed?

1865
41.2 Hints
Hint 41.1

Hint 41.2

Hint 41.3

Hint 41.4

Hint 41.5
From physical considerations conclude that you can take () = 0. Your solution will corroborate this. Use the initial
conditions to determine () and () for > 0. Then use the initial condition to determine () for < 0.

Hint 41.6

Hint 41.7

a) Substitute u(x, t) = <(A etx ) into the partial differential equation and solve for . Assume that has
positive real part so that the solution vanishes as x +.

Hint 41.8
Seek a periodic solution of the form,
u(r, , t) = v(r, ) et .

1866
Solve the Helmholtz equation for v with a Fourier series expansion,

X
v(r, ) = vn (r) en .
n=

You will find that the vn satisfy Bessels equation. Choose the vn so that u satisfies the boundary condition at r = a
and the radiation condition at infinity.
The Bessel functions have the asymptotic behavior,
r
2
Jn () cos( n/2 /4), as ,

r
2
Yn () sin( n/2 /4), as ,

r
(1) 2 i(n/2/4)
Hn () e , as ,

r
(2) 2 i(n/2/4)
Hn () e , as .

Hint 41.9

Hint 41.10

1867
41.3 Solutions
Solution 41.1
1. The initial position is
 
1 3
u(x, 0) = H x .
2 2
We extend the domain of the problem to ( . . . ) and add image sources in the initial condition so that
u(x, 0) is odd about x = 0 and x = 4. This enforces the boundary conditions at these two points.

utt uxx = 0, x ( . . . ), t (0 . . . )
    
X 1 3 1 13
u(x, 0) = H x 8n H x 8n , ut (x, 0) = 0
n=
2 2 2 2

We use DAlemberts solution to solve this problem.

    
1 X 1 3 1 3
u(x, t) = H x 8n t + H
x 8n + t
2 n= 2 2 2 2
   
1 13 1 13
H x 8n t H x 8n + t
2 2 2 2

The solution for several times is plotted in Figure 41.1. Note that the solution is periodic in time with period 8.
Figure 41.3 shows the solution in the phase plane for 0 < t < 8. Note the odd reflections at the boundaries.

2. The initial position is


 
1 3
u(x, 0) = H x .
2 2
We extend the domain of the problem to ( . . . ) and add image sources in the initial condition so that

1868
0.4 0.4
0.2 0.2
1 2 3 4 1 2 3 4
-0.2 -0.2
-0.4 -0.4

0.4 0.4
0.2 0.2
1 2 3 4 1 2 3 4
-0.2 -0.2
-0.4 -0.4

Figure 41.1: The solution at t = 1/2, 1, 3/2, 2 for the boundary conditions u(0, t) = u(4, t) = 0.

u(x, 0) is even about x = 0 and x = 4. This enforces the boundary conditions at these two points.

utt uxx = 0, x ( . . . ), t (0 . . . )
    
X 1 3 1 13
u(x, 0) = H x 8n + H x 8n , ut (x, 0) = 0
n=
2 2 2 2

We use DAlemberts solution to solve this problem.

    
1 X 1 3 1 3
u(x, t) = H x 8n t + H
x 8n + t
2 n= 2 2 2 2
   
1 13 1 13
+H x 8n t + H
x 8n + t
2 2 2 2

1869
The solution for several times is plotted in Figure 41.2. Note that the solution is periodic in time with period 8.
Figure 41.3 shows the solution in the phase plane for 0 < t < 8. Note the even reflections at the boundaries.

1 1
0.8 0.8
0.6 0.6
0.4 0.4
0.2 0.2
1 2 3 4 1 2 3 4
1 1
0.8 0.8
0.6 0.6
0.4 0.4
0.2 0.2
1 2 3 4 1 2 3 4

Figure 41.2: The solution at t = 1/2, 1, 3/2, 2 for the boundary conditions ux (0, t) = ux (4, t) = 0.

Solution 41.2
1.

1 x+ct
Z
1
u(x, t) = (u(x + ct, 0) + u(x ct, 0)) + ut (, 0) d
2 2c xct
1 x+ct
Z
u(x, t) = sin( ) d
2c xct
sin(x) sin(ct)
u(x, t) =
c
Figure 41.4 shows the solution for c = = 1.

1870
t t
0 1 0 1
1/2 0 1/2 0

1/2 1/2
0 1
0 0
0 1
-1/2 1/2
-1/2 1/2

0 -1 0 0 1 0

-1/2 1/2
-1/2 1/2
0 1
0 0
0 1
1/2 1/2
1/2 1/2
0 0
0 1 0 1
x x

Figure 41.3: The solution in the phase plane for the boundary conditions u(0, t) = u(4, t) = 0 and ux (0, t) =
ux (4, t) = 0.

2. We can write the initial velocity in terms of the Heaviside function.


1
for 0 < x < 1
ut (x, 0) = 1 for 1 < x < 0

0 for |x| > 1.

ut (x, 0) = H(x + 1) + 2H(x) H(x 1)

1871
1
0.5
u 6
0
-0.5
-1 4
-5 t
2
0
x
5 0

Figure 41.4: Solution of the wave equation.

We integrate the Heaviside function.



Z b 0
for b < c
H(x c) dx = b a for a > c
a
b c otherwise

If a < b, we can express this as


Z b
H(x c) dx = min(b a, max(b c, 0)).
a

1872
Now we find an expression for the solution.

1 x+ct
Z
1
u(x, t) = (u(x + ct, 0) + u(x ct, 0)) + ut (, 0) d
2 2c xct
1 x+ct
Z
u(x, t) = (H( + 1) + 2H( ) H( 1)) d
2c xct
u(x, t) = min(2ct, max(x + ct + 1, 0)) + 2 min(2ct, max(x + ct, 0)) min(2ct, max(x + ct 1, 0))

Figure 41.5 shows the solution for c = 1.

1
0.5
u 3
0
-0.5
-1 2
-4 t
-2 1
0
x 2
4 0

Figure 41.5: Solution of the wave equation.

1873
Solution 41.3
1. The solution on the interval ( . . . ) is
1
u(x, t) = (h(x + ct) + h(x ct)).
2
Now we solve the problem on (0 . . . ). We define the odd extension of h(x).
(
h(x) for x > 0,
h(x) = = sign(x)h(|x|)
h(x) for x < 0,

Note that
d
h0 (0 ) = (h(x)) x0+ = h0 (0+ ) = h0 (0+ ).

dx
Thus h(x) is piecewise C 2 . Clearly
1
u(x, t) = (h(x + ct) + h(x ct))
2
satisfies the differential equation on (0 . . . ). We verify that it satisfies the initial condition and boundary
condition.
1
u(x, 0) = (h(x) + h(x)) = h(x)
2
1 1
u(0, t) = (h(ct) + h(ct)) = (h(ct) h(ct)) = 0
2 2
2. First we define the odd extension of h(x) on the interval (l . . . l).
h(x) = sign(x)h(|x|), x (l . . . l)
Then we form the odd periodic extension of h(x) defined on ( . . . ).
      
x+l x + l
h(x) = sign x 2l h x 2l , x ( . . . )
2l 2l

1874
We note that h(x) is piecewise C 2 . Also note that h(x) is odd about the points x = nl, n Z. That is,
h(nl x) = h(nl + x). Clearly
1
u(x, t) = (h(x + ct) + h(x ct))
2
satisfies the differential equation on (0 . . . l). We verify that it satisfies the initial condition and boundary condi-
tions.
1
u(x, 0) = (h(x) + h(x))
2
u(x, 0) = h(x)
      
x+l x + l
u(x, 0) = sign x 2l h x 2l
2l 2l
u(x, 0) = h(x)
1 1
u(0, t) = (h(ct) + h(ct)) = (h(ct) h(ct)) = 0
2 2
1 1
u(l, t) = (h(l + ct) + h(l ct)) = (h(l + ct) h(l + ct)) = 0
2 2
Solution 41.4
Change of Variables. Let u(x, t) be the solution of the problem with deflection u(x, T ) = (x) and velocity
ut (x, T ) = (x). Define
v(x, ) = u(x, T ).
We note that u(x, 0) = v(x, T ). v( ) satisfies the wave equation.

v = c2 vxx

The initial conditions for v are

v(x, 0) = u(x, T ) = (x), v (x, 0) = ut (x, T ) = (x).

Thus we see that the student was correct.

1875
Direct Solution. DAlemberts solution is valid for all x and t. We formally substitute t T for t in this solution
to solve the problem with deflection u(x, T ) = (x) and velocity ut (x, T ) = (x).

1 x+c(tT )
Z
1
u(x, t) = ((x + c(t T )) + (x c(t T ))) + ( ) d
2 2c xc(tT )
This satisfies the wave equation, because the equation is shift-invariant. It also satisfies the initial conditions.
1 x
Z
1
u(x, T ) = ((x) + (x)) + ( ) d = (x)
2 2c x
1 1
ut (x, t) = (c0 (x + c(t T )) c0 (x c(t T ))) + ((x + c(t T )) + (x c(t T )))
2 2
1 1
ut (x, T ) = (c0 (x) c0 (x)) + ((x) + (x)) = (x)
2 2
Solution 41.5
Since the solution is a wave moving to the right, we conclude that we could take () = 0. Our solution will corroborate
this.
The form of the solution is
u(x, t) = (x + ct) + (x ct).
We substitute the solution into the initial conditions.

u(x, 0) = () + () = 0, > 0
ut (x, 0) = c0 () c 0 () = 0, > 0

We integrate the second equation to obtain the system

() + () = 0, > 0,
() () = 2k, > 0,

which has the solution


() = k, () = k, > 0.

1876
Now we substitute the solution into the initial condition.
u(0, t) = (ct) + (ct) = (t), t > 0
() + () = (/c), > 0
() = (/c) k, < 0
This determines u(x, t) for x > 0 as it depends on () only for > 0. The constant k is arbitrary. Changing k does
not change u(x, t). For simplicity, we take k = 0.
u(x, t) = (x ct)
(
0 for x ct < 0
u(x, t) =
(t x/c) for x ct > 0
u(x, t) = (t x/c)H(ct x)

Solution 41.6
1. We write the value of u along the line x ct = k as a function of t: u(k + ct, t). We differentiate ut cux with
respect to t to see how the quantity varies.
d
(ut (k + ct, t) cux (k + ct, t)) = cuxt + utt c2 uxx cuxt
dt
= utt c2 uxx
=0
Thus ut cux is constant along the line x ct = k. Now we examine ut + cux along the line x + ct = k.
d
(ut (k ct, t) + cux (k ct, t)) = cuxt + utt c2 uxx + cuxt
dt
= utt c2 uxx
=0
ut + cux is constant along the line x + ct = k.

1877
2. From part (a) we know
ut (x0 , t0 ) cux (x0 , t0 ) = ut (x0 ct0 , 0) cux (x0 ct0 , 0)
ut (x0 , t0 ) + cux (x0 , t0 ) = ut (x0 + ct0 , 0) + cux (x0 + ct0 , 0).
We add these equations to find ut (x0 , t0 ).
1
ut (x0 , t0 ) = (ut (x0 ct0 , 0) cux (x0 ct0 , 0)ut (x0 + ct0 , 0) + cux (x0 + ct0 , 0))
2
Since t0 was arbitrary, we have
1
ut (x0 , ) = (ut (x0 c, 0) cux (x0 c, 0)ut (x0 + c, 0) + cux (x0 + c, 0))
2
for 0 < < t0 . We integrate with respect to to determine u(x0 , t0 ).
Z t0
1
u(x0 , t0 ) = u(x0 , 0) + (ut (x0 c, 0) cux (x0 c, 0)ut (x0 + c, 0) + cux (x0 + c, 0)) d
0 2
1 t0
Z
= u(x0 , 0) + (cux (x0 c, 0) + cux (x0 + c, 0)) d
2 0
1 t0
Z
+ (ut (x0 c, 0) + ut (x0 + c, 0)) d
2 0
1
= u(x0 , 0) + (u(x0 ct0 , 0) u(x0 , 0) + u(x0 + ct0 , 0) u(x0 , 0))
Z x20 ct0
1 x0 +ct0
Z
1
+ ut (, 0) d + ut (, 0) d
2c x0 2c x0
1 x0 +ct0
Z
1
= (u(x0 ct0 , 0) + u(x0 + ct0 , 0)) + ut (, 0) d
2 2c x0 ct0
We have DAlemberts solution.
Z x+ct
1 1
u(x, t) = (u(x ct, 0) + u(x + ct, 0)) + ut (, 0) d
2 2c xct

1878
Solution 41.7

a) We substitute u(x, t) = A etx into the partial differential equation and take the real part as the solution. We
assume that has positive real part so the solution vanishes as x +.
A etx = 2 A etx
= 2
r

= (1 + )
2
A solution of the partial differential equation is,
  r 

u(x, t) = < A exp t (1 + ) x ,
2
 r   r 

u(x, t) = A exp x cos t x .
2 2
Applying the initial condition, u(0, t) = T cos(t), we obtain,
 r   r 

u(x, t) = T exp x cos t x .
2 2

b) At a fixed depth x = h, the temperature is


 r   r 

u(h, t) = T exp h cos t h .
2 2
Thus the temperature variation is
 r   r 

T exp h u(h, t) T exp h .
2 2

1879
p
c)
The solution is an exponentially decaying, traveling wave that propagates into the Earth with speed / /(2) =
2. More generally, the wave
ebt cos(t ax)
travels in the positive direction with speed /a. Figure 41.6 shows such a wave for a sequence of times.

Figure 41.6: An Exponentially Decaying, Traveling Wave

The phase lag, (x) is the time that it takes for the wave to reach a depth of x. It satisfies,
r

(x) x = 0,
2
x
(x) = .
2
d) Let year be the frequency for annual temperature variation, then day = 365year . If xyear is the depth that a
particular yearly temperature variation reaches and xday is the depth that this same variation in daily temperature
reaches, then  r   r 
year day
exp xyear = exp xday ,
2 2

1880
r r
year day
xyear = xday ,
2 2
xyear
= 365.
xday

Solution 41.8
We seek a periodic solution of the form,
u(r, , t) = v(r, ) et .
Substituting this into the wave equation will give us a Helmholtz equation for v.

2 v = c2 v
1 1 2
vrr + vr + 2 v + 2 v = 0
r r c
We have the boundary condition v(a, ) = f () and the radiation condition at infinity. We expand v in a Fourier series
in in which the coefficients are functions of r. You can check that en are the eigenfunctions obtained with separation
of variables.
X
v(r, ) = vn (r) en
n=

We substitute this expression into the Helmholtz equation to obtain ordinary differential equations for the coefficients
vn .
  2
n2
 
X
00 1 0
vn + vn + 2
2 vn en = 0
n=
r c r

The differential equations for the vn are

2 n2
 
1
vn00 + vn0 + 2 vn = 0.
r c2 r

1881
which has as linearly independent solutions the Bessel and Neumann functions,
 r   r 
Jn , Yn ,
c c

or the Hankel functions,


 r   r 
Hn(1) , Hn(2) .
c c

The functions have the asymptotic behavior,


r
2
Jn () cos( n/2 /4), as ,

r
2
Yn () sin( n/2 /4), as ,

r
(1) 2 i(n/2/4)
Hn () e , as ,

r
(2) 2 i(n/2/4)
Hn () e , as .

u(r, , t) will be an outgoing wave at infinity if it is the sum of terms of the form ei(tconstr) . Thus the vn must have
the form
(2) r
 
vn (r) = bn Hn
c
for some constants, bn . The solution for v(r, ) is

X  r 
v(r, ) = bn Hn(2) en .
n=
c

1882
We determine the constants bn from the boundary condition at r = a.

X  a 
v(a, ) = bn Hn(2) en = f ()
n=
c
Z 2
1
bn = (2)
f () en d
2Hn (a/c) 0


X  r 
u(r, , t) = et bn Hn(2) en
n=
c

Solution 41.9
We substitute the form v(x, y, t) = u(r, ) et into the wave equation to obtain a Helmholtz equation.

c2 u + 2 u = 0
1 1
urr + ur + 2 u + k 2 u = 0
r r
We solve the Helmholtz equation with separation of variables. We expand u in a Fourier series.

X
u(r, ) = un (r) en
n=

We substitute the sum into the Helmholtz equation to determine ordinary differential equations for the coefficients.

n2
 
00 1 0 2
u n + un + k 2 un = 0
r r

This is Bessels equation, which has as solutions the Bessel and Neumann functions, {Jn (kr), Yn (kr)} or the Hankel
(1) (2)
functions, {Hn (kr), Hn (kr)}.

1883
Recall that the solutions of the Bessel equation have the asymptotic behavior,
r
2
Jn () cos( n/2 /4), as ,

r
2
Yn () sin( n/2 /4), as ,

r
(1) 2 i(n/2/4)
Hn () e , as ,

r
2 i(n/2/4)
Hn(2) () e , as .

From this we see that only the Hankel function of the first kink will give us outgoing waves as . Our solution
for u becomes,
X
u(r, ) = bn Hn(1) (kr) en .
n=

We determine the coefficients in the expansion from the boundary condition at r = a.



X
u(a, ) = bn Hn(1) (ka) en = eka cos
n=
Z 2
1
bn = (1)
eka cos en d
2Hn (ka) 0

We evaluate the integral with the identities,


Z 2
1
Jn (x) = n
ex cos en d,
2i 0
Jn (x) = (1)n Jn (x).

1884
Thus we obtain,

X ()n Jn (ka)
u(r, ) = (1)
Hn(1) (kr) en .
n= Hn (ka)

When a  1/k, i.e. ka  1, the Bessel function has the behavior,

(ka/2)n
Jn (ka) .
n!
In this case, the n 6= 0 terms in the sum are much smaller than the n = 0 term. The approximate solution is,
(1)
H0 (kr)
u(r, ) (1)
,
H0 (ka)
(1)
H0 (kr)
v(r, , t) (1)
et .
H0 (ka)

Solution 41.10

a) (
Ix = CVt + GV,
Vx = LIt + RI

First we derive a single partial differential equation for I. We differentiate the two partial differential equations with
respect to x and t, respectively and then eliminate the Vxt terms.
(
Ixx = CVtx + GVx ,
Vxt = LItt + RIt
Ixx + LCItt + RCIt = GVx

1885
We use the initial set of equations to write Vx in terms of I.

Ixx + LCItt + RCIt + G(LIt + RI) = 0


RC + GL GR 1
Itt + It + I Ixx = 0
LC LC LC
Now we derive a single partial differential equation for V . We differentiate the two partial differential equations
with respect to t and x, respectively and then eliminate the Ixt terms.
(
Ixt = CVtt + GVt ,
Vxx = LItx + RIx
Vxx = RIx LCVtt LGVt

We use the initial set of equations to write Ix in terms of V .

LCVtt + LGVt Vxx + R(CVt + GV ) = 0


RC + LG RG 1
Vtt + Vt + V Vxx = 0.
LC LC LC
Thus we see that I and V both satisfy the same damped wave equation.
b) We substitute V (x, t) = et (f (x at) + g(x + at)) into the damped wave equation for V .
   
RC + LG RG t RC + LG
2
+ e (f + g) + 2 + a et (f 0 + g 0 )
LC LC LC
1 t 00
+ a2 et (f 00 + g 00 ) e (f + g 00 ) = 0
LC
Since f and g are arbitrary functions, the coefficients of et (f + g), et (f 0 + g 0 ) and et (f 00 + g 00 ) must vanish.
This gives us three constraints.
1 RC + LG RC + LG RG
a2 = 0, 2 + = 0, 2 + =0
LC LC LC LC

1886

The first equation determines the wave speed to be a = 1/ LC. We substitute the value of from the second
equation into the third equation.
RC + LG RG
= , 2 + =0
2LC LC
In order for damped waves to propagate, the physical constants must satisfy,
 2
RG RC + LG
= 0,
LC 2LC
4RGLC (RC + LG)2 = 0,
(RC LG)2 = 0,
RC = LG.

1887
Chapter 42

Similarity Methods

Introduction. Consider the partial differential equation (not necessarily linear)


 
u u
F , , u, t, x = 0.
t x
Say the solution is
t1/2
 
x
u(x, t) = sin .
t x1/2
Making the change of variables = x/t, f () = u(x, t), we could rewrite this equation as

f () = sin 1/2 .


We see now that if we had guessed that the solution of this partial differential equation was only dependent on powers
of x/t we could have changed variables to and f and instead solved the ordinary differential equation
 
df
G , f, = 0.
d
By using similarity methods one can reduce the number of independent variables in some PDEs.

1888
Example 42.0.1 Consider the partial differential equation
u u
x +t u = 0.
t x
One way to find a similarity variable is to introduce a transformation to the temporary variables u0 , t0 , x0 , and the
parameter .

u = u0
t = t0 m
x = x 0 n

where n and m are unknown. Rewriting the partial differential equation in terms of the temporary variables,
u0 1m 0
0 m u 1n
x 0 n + t u0 = 0
t0 x0
u0 u0
x0 0 m+n + t0 0 mn u0 = 0
t x
There is a similarity variable if can be eliminated from the equation. Equating the coefficients of the powers of in
each term,
m + n = m n = 0.
This has the solution m = n. The similarity variable, , will be unchanged under the transformation to the temporary
variables. One choice is
t t0 n t0
= = 0 m = 0.
x x x
Writing the two partial derivative in terms of ,
d 1 d
= =
t t d x d
d t d
= = 2
x x d x d

1889
The partial differential equation becomes

du du
2 u=0
d d
du u
=
d 1 2

Thus we have reduced the partial differential equation to an ordinary differential equation that is much easier to solve.
Z 
d
u() = exp
1 2
Z 
1/2 1/2
u() = exp + d
1 1+
 
1 1
u() = exp log(1 ) + log(1 + )
2 2
u() = (1 )1/2 (1 + )1/2
 1/2
1 + t/x
u(x, t) =
1 t/x

Thus we have found a similarity solution to the partial differential equation. Note that the existence of a similarity
solution does not mean that all solutions of the differential equation are similarity solutions.

Another Method. Another method is to substitute = x t and determine if there is an that makes a similarity
variable. The partial derivatives become

d d
= = x
t t d d
d d
= = x1 t
x x d d

1890
The partial differential equation becomes
du du
x+1 + x1 t2 u = 0.
d d
If there is a value of such that we can write this equation in terms of , then = x t is a similarity variable. If
= 1 then the coefficient of the first term is trivially in terms of . The coefficient of the second term then becomes
x2 t2 . Thus we see = x1 t is a similarity variable.

Example 42.0.2 To see another application of similarity variables, any partial differential equation of the form
 ut ux 
F tx, u, , =0
x t
is equivalent to the ODE  
du du
F , u, , =0
d d
where = tx. Performing the change of variables,
1 u 1 du 1 du du
= = x =
x t x t d x d d
1 u 1 du 1 du du
= = t = .
t x t x d t d d
For example the partial differential equation
u x u
u + + tx2 u = 0
t t x
which can be rewritten
1 u 1 u
u + + txu = 0,
x t t x
is equivalent to
du du
u + + u = 0
d d
where = tx.

1891
42.1 Exercises
Exercise 42.1
Consider the 1-D heat equation
ut = uxx
Assume that there exists a function (x, t) such that it is possible to write u(x, t) = F ((x, t)). Re-write the PDE
in terms of F (), its derivatives and (partial) derivatives of . By guessing that this transformation takes the form
= xt , find a value of so that this reduces to an ODE for F () (i.e. x and t are explicitly removed). Find the
general solution and use this to find the corresponding solution u(x, t). Is this the general solution of the PDE?

Exercise 42.2
With = x t, find such that for some function f , = f () is a solution of

t = a2 xx .

Find f () as well.

1892
42.2 Hints
Hint 42.1

Hint 42.2

1893
42.3 Solutions
Solution 42.1
We write the derivatives of u(x, t) in terms of derivatives of F ().

ut = xt1 F 0 = F 0
t
0
ux = t F
2
uxx = t2 F 00 = 2 F 00
x
We substitite these expressions into the heat equation.
2
F 0 = 2 F 00
t x
2
x 1 0
F 00 = F
t
We can write this equation in terms of F and only if = 1/2. We make this substitution and solve the ordinary
differential equation for F ().
F 00
0
=
F 2
2
log(F 0 ) = + c
4 2 

F 0 = c exp
4
Z  2

F = c1 exp d + c2
4
We can write F in terms of the error function.
 

F = c1 erf + c2
2

1894
We write this solution in terms of x and t.
 
x
u(x, t) = c1 erf + c2
2 t

This is not the general solution of the heat equation. There are many other solutions. Note that since x and t do not
explicitly appear in the heat equation,
!
x x0
u(x, t) = c1 erf p + c2
2 (t t0 )

is a solution.
Solution 42.2
We write the derivatives of in terms of f .


t = f = x f 0 = t1 f 0
t

x = f = x1 tf 0
x

xx = f 0 x1 t + x1 tx1 t f 0

x
2 22 2 00 2 0
xx = x t f + ( 1)x tf
2
2 2 f 00 + ( 1)f 0

xx = x

We substitute these expressions into the diffusion equation.

f 0 = x2 t 2 2 f 00 + ( 1)f 0


In order for this equation to depend only on the variable , we must have = 2. For this choice we obtain an ordinary

1895
differential equation for f ().

f 0 = 4 2 f 00 + 6f 0
f 00 1 3
0
= 2
f 4 2
1 3
log(f 0 ) = log + c
4 2
f = c1 3/2 e1/(4)
0
Z
f () = c1 t3/2 e1/(4t) dt + c2
Z 1/(2)
2
f () = c1 et dt + c2
 
1
f () = c1 erf + c2
2

1896
Chapter 43

Method of Characteristics

43.1 First Order Linear Equations


Consider the following first order wave equation.

ut + cux = 0 (43.1)

Let x(t) be some path in the phase plane. Perhaps x(t) describes the position of an observer who is noting the value
of the solution u(x(t), t) at their current location. We differentiate with respect to t to see how the solution varies for
the observer.
d
u(x(t), t) = ut + x0 (t)ux (43.2)
dt
We note that if the observer is moving with velocity c, x0 (t) = c, then the solution at their current location does not
change because ut + cux = 0. We will examine this more carefully.
By comparing Equations 43.1 and 43.2 we obtain ordinary differential equations representing the position of an
observer and the value of the solution at that position.
dx du
= c, =0
dt dt

1897
Let the observer start at the position x0 . Then we have an initial value problem for x(t).
dx
= c, x(0) = x0
dt
x(t) = x0 + ct

These lines x(t) are called characteristics of Equation 43.1.


Let the initial condition be u(x, 0) = f (x). We have an initial value problem for u(x(t), t).
du
= 0, u(0) = f (x0 )
dt
u(x(t), t) = f (x0 )

Again we see that the solution is constant along the characteristics. We substitute the equation for the characteristics
into this expression.

u(x0 + ct, t) = f (x0 )


u(x, t) = f (x ct)

Now we see that the solution of Equation 43.1 is a wave moving with velocity c. The solution at time t is the initial
condition translated a distance of ct.

43.2 First Order Quasi-Linear Equations


Consider the following quasi-linear equation.

ut + a(x, t, u)ux = 0 (43.3)

We will solve this equation with the method of characteristics. We differentiate the solution along a path x(t).
d
u(x(t), t) = ut + x0 (t)ux (43.4)
dt

1898
By comparing Equations 43.3 and 43.4 we obtain ordinary differential equations for the characteristics x(t) and the
solution along the characteristics u(x(t), t).

dx du
= a(x, t, u), =0
dt dt

Suppose an initial condition is specified, u(x, 0) = f (x). Then we have ordinary differential equation, initial value
problems.

dx
= a(x, t, u), x(0) = x0
dt
du
= 0, u(0) = f (x0 )
dt
We see that the solution is constant along the characteristics. The solution of Equation 43.3 is a wave moving with
velocity a(x, t, u).

Example 43.2.1 Consider the inviscid Burger equation,

ut + uux = 0, u(x, 0) = f (x).

We write down the differential equations for the solution along a characteristic.

dx
= u, x(0) = x0
dt
du
= 0, u(0) = f (x0 )
dt

First we solve the equation for u. u = f (x0 ). Then we solve for x. x = x0 + f (x0 )t. This gives us an implicit solution
of the Burger equation.
u(x0 + f (x0 )t, t) = f (x0 )

1899
43.3 The Method of Characteristics and the Wave Equation
Consider the one dimensional wave equation,
utt = c2 uxx .
We make the change of variables, a = ux , b = ut , to obtain a coupled system of first order equations.

at b x = 0
b t c 2 ax = 0

We write this as a matrix equation.


    
a 0 1 a
+ 2 =0
b t c 0 b x
The eigenvalues and eigenvectors of the matrix are
   
1 1
1 = c, 2 = c, 1 = , 2 = .
c c

The matrix is diagonalized by a similarity transformation.


   1   
c 0 1 1 0 1 1 1
=
0 c c c c2 0 c c

We make a change of variables to diagonalize the system.


    
a 1 1
=
b c c
      
1 1 0 1 1 1
+ 2 =0
c c t c 0 c c x

1900
Now we left multiply by the inverse of the matrix of eigenvectors to obtain an uncoupled system that we can solve
directly.
    
c 0
+ = 0.
t 0 c x
(x, t) = p(x + ct), (x, t) = q(x ct),

Here p, q C 2 are arbitrary functions. We change variables back to a and b.

a(x, t) = p(x + ct) + q(x ct), b(x, t) = cp(x + ct) cq(x ct)

We could integrate either a = ux or b = ut to obtain the solution of the wave equation.

u = F (x ct) + G(x + ct)

Here F, G C 2 are arbitrary functions. We see that u(x, t) is the sum of a waves moving to the right and left with
speed c. This is the general solution of the one-dimensional wave equation. Note that for any given problem, F and
G are only determined to whithin an additive constant. For any constant k, adding k to F and subtracting it from G
does not change the solution.
u = (F (x ct) + k) + (G(x ct) k)

43.4 The Wave Equation for an Infinite Domain


Consider the Cauchy problem for the wave equation on < x < .

utt = c2 uxx , < x < , t > 0


u(x, 0) = f (x), ut (x, 0) = g(x)

We know that the solution is the sum of right-moving and left-moving waves.

u(x, t) = F (x ct) + G(x + ct) (43.5)

1901
The initial conditions give us two constraints on F and G.

F (x) + G(x) = f (x), cF 0 (x) + cG0 (x) = g(x).

We integrate the second equation. Z


1
F (x) + G(x) = g(x) dx + const
c
R
Here Q(x) = q(x) dx. We solve the system of equations for F and G.
Z Z
1 1 1 1
F (x) = f (x) g(x) dx k, G(x) = f (x) + g(x) dx + k
2 2c 2 2c

Note that the value of the constant k does not affect the solution, u(x, t). For simplicity we take k = 0. We substitute
F and G into Equation 43.5 to determine the solution.
Z x+ct Z xct 
1 1
u(x, t) = (f (x ct) + f (x + ct)) + g(x) dx g(x) dx
2 2c
1 x+ct
Z
1
u(x, t) = (f (x ct) + f (x + ct)) + g() d
2 2c xct
1 x+ct
Z
1
u(x, t) = (u(x ct, 0) + u(x + ct, 0)) + ut (, 0) d
2 2c xct

43.5 The Wave Equation for a Semi-Infinite Domain


Consider the wave equation for a semi-infinite domain.

utt = c2 uxx , 0 < x < , t > 0


u(x, 0) = f (x), ut (x, 0) = g(x), u(0, t) = h(t)

1902
Again the solution is the sum of a right-moving and a left-moving wave.
u(x, t) = F (x ct) + G(x + ct)
For x > ct, the boundary condition at x = 0 does not affect the solution. Thus we know the solution in this domain
from our work on the wave equation in the infinite domain.
1 x+ct
Z
1
u(x, t) = (f (x ct) + f (x + ct)) + g() d, x > ct
2 2c xct
From this, F () and G() are determined for > 0.
Z
1 1
F () = f () g() d, >0
2 2c
Z
1 1
G() = f () + g() d, >0
2 2c

In order to determine the solution u(x, t) for x, t > 0 we also need to determine F () for < 0. To do this, we
substitute the form of the solution into the boundary condition at x = 0.
u(0, t) = h(t), t > 0
F (ct) + G(ct) = h(t), t > 0
F () = G() + h(/c), < 0
1
Z
1
F () = f () g() d + h(/c), <0
2 2c
We determine the solution of the wave equation for x < ct.
u(x, t) = F (x ct) + G(x + ct)
1 x+ct 1 x+ct
Z Z
1 1
u(x, t) = f (x + ct) g() d + h(t x/c) + f (x + ct) + g() d, x < ct
2 2c 2 2c
1 x+ct
Z
1
u(x, t) = (f (x + ct) + f (x + ct)) + g() d + h(t x/c), x < ct
2 2c x+ct

1903
Finally, we collect the solutions in the two domains.
( R x+ct
1 1
2
(f (x ct) + f (x + ct)) + 2c xct
g() d, x > ct
u(x, t) = 1 1
R x+ct
2
(f (x + ct) + f (x + ct)) + 2c x+ct g() d + h(t x/c), x < ct

43.6 The Wave Equation for a Finite Domain


Consider the wave equation for the infinite domain.

utt = c2 uxx , < x < , t > 0


u(x, 0) = f (x), ut (x, 0) = g(x)

If f (x) and g(x) are odd about x = 0, (f (x) = f (x), g(x) = g(x)), then u(x, t) is also odd about x = 0. We
can demonstrate this with DAlemberts solution.
1 x+ct
Z
1
u(x, t) = (f (x ct) + f (x + ct)) + g() d
2 2c xct

1 x+ct
Z
1
u(x, t) = (f (x ct) + f (x + ct)) g() d
2 2c xct
1 xct
Z
1
= (f (x + ct) + f (x ct)) g() (d)
2 2c x+ct
1 x+ct
Z
1
= (f (x ct) + f (x + ct)) + g() d
2 2c xct
= u(x, t)

Thus if the initial conditions f (x) and g(x) are odd about a point then the solution of the wave equation u(x, t) is
also odd about that point. The analogous result holds if the initial conditions are even about a point. These results
are useful in solving the wave equation on a finite domain.

1904
Consider a string of length L with fixed ends.

utt = c2 uxx , 0 < x < L, t > 0


u(x, 0) = f (x), ut (x, 0) = g(x), u(0, t) = u(L, t) = 0

We extend the domain of the problem to x ( . . . ). We form the odd periodic extensions f and g which are
odd about the points x = 0, L.
If a function h(x) is defined for positive x, then sign(x)h(|x|) is the odd extension of the function. If h(x) is defined
for x (L . . . L) then its periodic extension is
  
x+L
h x 2L .
2L

We combine these two formulas to form odd periodic extensions.


      
x+L x + L
f (x) = sign x 2L f x 2L

2L 2L
      
x+L x + L
g(x) = sign x 2L g x 2L

2L 2L

Now we can write the solution for the vibrations of a string with fixed ends.
Z x+ct
1  1
u(x, t) = f (x ct) + f(x + ct) + g() d
2 2c xct

43.7 Envelopes of Curves


Consider the tangent lines to the parabola y = x2 . The slope of the tangent at the point (x, x2 ) is 2x. The set of
tangents form a one parameter family of lines,

f (x, t) = t2 + (x t)2t = 2tx t2 .

1905
1

-1 1

-1

Figure 43.1: A parabola and its tangents.

The parabola and some of its tangents are plotted in Figure 43.1.
The parabola is the envelope of the family of tangent lines. Each point on the parabola is tangent to one of the
lines. Given a curve, we can generate a family of lines that envelope the curve. We can also do the opposite, given
a family of lines, we can determine the curve that they envelope. More generally, given a family of curves, we can
determine the curve that they envelope. Let the one parameter family of curves be given by the equation F (x, y, t) = 0.
For the example of the tangents to the parabola this equation would be y 2tx + t2 = 0.
Let y(x) be the envelope of F (x, y, t) = 0. Then the points on y(x) must lie on the family of curves. Thus y(x)
must satisfy the equation F (x, y, t) = 0. The points that lie on the envelope have the property,

F (x, y, t) = 0.
t
We can solve this equation for t in terms of x and y, t = t(x, y). The equation for the envelope is then
F (x, y, t(x, y)) = 0.

1906
Consider the example of the tangents to the parabola. The equation of the one-parameter family of curves is
F (x, y, t) y 2tx + t2 = 0.
The condition Ft (x, y, t) = 0 gives us the constraint,
2x + 2t = 0.
Solving this for t gives us t(x, y) = x. The equation for the envelope is then,
y 2xx + x2 = 0,
y = x2 .

Example 43.7.1 Consider the one parameter family of curves,


(x t)2 + (y t)2 1 = 0.
These are circles of unit radius and center (t, t). To determine the envelope of the family, we first use the constraint
Ft (x, y, t) to solve for t(x, y).
Ft (x, y, t) = 2(x t) 2(y t) = 0
x+y
t(x, y) =
2
Now we substitute this into the equation F (x, y, t) = 0 to determine the envelope.
   2  2
x+y x+y x+y
F x, y, = x + y 1=0
2 2 2
 2  2
xy yx
+ 1=0
2 2
(x y)2 = 2

y =x 2
The one parameter family of curves and its envelope is shown in Figure 43.2.

1907
3

-3 -2 -1 1 2 3

-1

-2

-3

Figure 43.2: The envelope of (x t)2 + (y t)2 1 = 0.

43.8 Exercises
Exercise 43.1
Consider the small transverse vibrations of a composite string of infinite extent, made up of two homogeneous strings
of different densities joined at x = 0. In each region 1) x < 0, 2) x > 0 we have

utt c2j uxx = 0 j = 1, 2 c1 6= c2 ,

and we require continuity of u and ux at x = 0. Suppose for t < 0 a wave approaches the junction x = 0 from the
left, i.e. as t approaches 0 from negative values:
(
F (x c1 t) x < 0, t 0
u(x, t) =
0 x > 0, t 0

As t increases further, the wave reaches x = 0 and gives rise to reflected and transmitted waves.

1908
1. Formulate the appropriate initial values for u at t = 0.

2. Solve the initial-value problem for < x < , t > 0.

3. Identify the incident, reflected and transmitted waves in your solution and determine the reflection and transmission
coefficients for the junction in terms of c1 and c2 . Comment also on their values in the limit c1 c2 .

Exercise 43.2
Consider a semi-infinite string, x > 0. For all time the end of the string is displaced according to u(0, t) = f (t). Find
the motion of the string, u(x, t) with the method of characteristics and then with a Fourier transform in time. The
wave speed is c.
Exercise 43.3
Solve using characteristics:
x
uux + uy = 1, u x=y = .
2
Exercise 43.4
Solve using characteristics:
(y + u)ux + yuy = x y, u y=1 = 1 + x.

1909
43.9 Hints
Hint 43.1

Hint 43.2
1. Because the left end of the string is being displaced, there will only be right-moving waves. Assume a solution of
the form
u(x, t) = F (x ct).

2. Take a Fourier transform in time. Use that there are only outgoing waves.
Hint 43.3

Hint 43.4

1910
43.10 Solutions
Solution 43.1
1.
(
F (x), x < 0
u(x, 0) =
0, x>0
(
c1 F 0 (x), x < 0
ut (x, 0) =
0, x>0

2. Regardless of the initial condition, the solution has the following form.
(
f1 (x c1 t) + g1 (x + c1 t), x < 0
u(x, t) =
f2 (x c2 t) + g1 (x + c2 t), x > 0

For x < 0, the right-moving wave is F (x c1 t) and the left-moving wave is zero for x < c1 t. For x > 0, there
is no left-moving wave and the right-moving wave is zero for x > c2 t. We apply these restrictions to the solution.
(
F (x c1 t) + g(x + c1 t), x < 0
u(x, t) =
f (x c2 t), x>0

We use the continuity of u and ux at x = 0 to solve for f and g.


F (c1 t) + g(c1 t) = f (c2 t)
F 0 (c1 t) + g 0 (c1 t) = f 0 (c2 t)
We integrate the second equation.
F (t) + g(t) = f (c2 t/c1 )
c1
F (t) + g(t) = f (c2 t/c1 ) + a
c2

1911
We solve for f for x < c2 t and for g for x > c1 t.
2c2 c2 c1
f (c2 t/c1 ) = F (t) + b, g(t) = F (t) + b
c1 + c2 c1 + c2
By considering the case that the solution is continuous, F (0) = 0, we conclude that b = 0 since f (0) = g(0) = 0.
2c2 c2 c1
f (t) = F (c1 t/c2 ), g(t) = F (t)
c1 + c2 c1 + c2
Now we can write the solution for u(x, t) for t > 0.
c1
(
F (x c1 t) + cc12 +c F (x c1 t)H(x + c1 t), x < 0
2 
u(x, t) = 2c2 c1
c1 +c2
F c2 (x c2 t) H(c2 t x), x>0

3. The incident, reflected and transmitted waves are, respectively,


 
c2 c1 2c2 c1
F (x c1 t), F (x c1 t)H(x + c1 t), F (x c2 t) H(c2 t x).
c1 + c2 c1 + c2 c2

The reflection and transmission coefficients are, respectively,


c1 c2 2c2
, .
c1 + c2 c1 + c2
In the limit as c1 c2 , the reflection coefficient vanishes and the transmission coefficient tends to unity.
Solution 43.2
1. Method of characteristics. The problem is

utt c2 uxx = 0, x > 0, < t < ,


u(0, t) = f (t).

1912
Because the left end of the string is being displaced, there will only be right-moving waves. The solution has the
form
u(x, t) = F (x ct).
We substitute this into the boundary condition.

F (ct) = f (t)
 

F () = f
c
u(x, t) = f (t x/c)

2. Fourier transform. We take the Fourier transform in time of the wave equation and the boundary condition.

utt = c2 uxx , u(0, t) = f (t)


2 u = c2 uxx , u(0, ) = f()
2
uxx + 2 u = 0, u(0, ) = f()
c
The general solution of this ordinary differential equation is

u(x, ) = a() ex/c +b() ex/c .

The radiation condition, (u(x, t) must be a wave traveling in the positive direction), and the boundary condition
at x = 0 will determine the constants a and b. Consider the solution u(x, t) we will obtain by taking the inverse
Fourier transform of u.
Z
a() ex/c +b() ex/c et d

u(x, t) =
Z

a() e(t+x/c) +b() e(tx/c) d

u(x, t) =

1913
The first and second terms in the integrand are left and right traveling waves, respectively. In order that u is a
right traveling wave, it must be a superposition of right traveling waves. We conclude that a() = 0. We apply
the boundary condition at x = 0, we solve for u.

u(x, ) = f() ex/c

Finally we take the inverse Fourier transform.


Z
u(x, t) = f() e(tx/c) d

u(x, t) = f (t x/c)

Solution 43.3
x
uux + uy = 1, u x=y = (43.6)
2
du
We form dy
.
du dx
= ux + uy
dy dy
We compare this with Equation 43.6 to obtain differential equations for x and u.

dx du
= u, = 1. (43.7)
dy dy

The initial data is



x(y = ) = , u(y = ) = . (43.8)
2
We solve the differenial equation for u (43.7) subject to the initial condition (43.8).

u(x(y), y) = y
2

1914
The differential equation for x becomes
dx
=y .
dy 2
We solve this subject to the initial condition (43.8).

1
x(y) = (y 2 + (2 y))
2
This defines the characteristic starting at the point (, ). We solve for .

y 2 2x
=
y2

We substitute this value for into the solution for u.


y(y 4) + 2x
u(x, y) =
2(y 2)

This solution is defined for y 6= 2. This is because at (x, y) = (2, 2), the characteristic is parallel to the line x = y.
Figure 43.3 has a plot of the solution that shows the singularity at y = 2.

Solution 43.4

(y + u)ux + yuy = x y, u y=1 = 1 + x (43.9)
We differentiate u with respect to s.
du dx dy
= ux + uy
ds ds ds
We compare this with Equation 43.9 to obtain differential equations for x, y and u.

dx dy du
= y + u, = y, =xy
ds ds ds

1915
x -2
0
2
10
u 0
-10

-2
0
y 2

Figure 43.3: The solution u(x, y).

We parametrize the initial data in terms of s.

x(s = 0) = , y(s = 0) = 1, u(s = 0) = 1 +

We solve the equation for y subject to the inital condition.

y(s) = es

This gives us a coupled set of differential equations for x and u.


dx du
= es +u, = x es
ds ds
The solutions subject to the initial conditions are

x(s) = ( + 1) es es , u(s) = es + es .

1916
We substitute y(s) = es into these solutions.
1 1
x(s) = ( + 1)y , u(s) = y +
y y
We solve the first equation for and substitute it into the second equation to obtain the solution.
2 + xy y 2
u(x, y) =
y
This solution is valid for y > 0. The characteristic passing through (, 1) is

x(s) = ( + 1) es es , y(s) = es .

Hence we see that the characteristics satisfy y(s) 0 for all real s. Figure 43.4 shows some characteristics in the (x, y)
plane with starting points from (5, 1) to (5, 1) and a plot of the solution.

2 -2
-1
1.75 x 0
1.5 1
1.25 2
15
1 10
0.75 u 5
0.5
0
0.25
0.5 1 1.5 2
-10-7.5 -5 -2.5 2.5 5 7.5 10 y

Figure 43.4: Some characteristics and the solution u(x, y).

1917
Chapter 44

Transform Methods

44.1 Fourier Transform for Partial Differential Equations


Solve Laplaces equation in the upper half plane

2 u = 0 < x < , y > 0


u(x, 0) = f (x) <x<

Taking the Fourier transform in the x variable of the equation and the boundary condition,

2u 2u
 
F + = 0, F [u(x, 0)] = F [f (x)]
x2 y 2
2
2 U (, y) + 2 U (, y) = 0, U (, 0) = F ().
y

The general solution to the equation is


U (, y) = a ey +b ey .

1918
Remember that in solving the differential equation here we consider to be a parameter. Requiring that the solution
be bounded for y [0, ) yields
U (, y) = a e||y .

Applying the boundary condition,


U (, y) = F () e||y .

The inverse Fourier transform of e||y is


2y
F 1 e||y =
 
.
x2 + y 2
Thus
 
2y
U (, y) = F () F 2
x + y2
 
2y
F [u(x, y)] = F [f (x)] F 2 .
x + y2

Recall that the convolution theorem is


 Z 
1
F f (x )g() d = F ()G().
2

Applying the convolution theorem to the equation for U ,



f (x )2y
Z
1
u(x, y) = d
2 2 + y2


f (x )
Z
y
u(x, y) = d.
2 + y2

1919
44.2 The Fourier Sine Transform
Consider the problem
ut = uxx , x > 0, t > 0
u(0, t) = 0, u(x, 0) = f (x)
Since we are given the position at x = 0 we apply the Fourier sine transform.
 
2 2
ut = u + u(0, t)

ut = 2 u
2t
u(, t) = c() e
The initial condition is
u(, 0) = f().
We solve the first order differential equation to determine u.
2
u(, t) = f() e t
 
1 x2 /(4t)
u(, t) = f ()Fc e
4t
We take the inverse sine transform with the convolution theorem.
Z
1  2 2

u(x, t) = 3/2 f () e|x| /(4t) e(x+) /(4t) d
4 t 0

44.3 Fourier Transform


Consider the problem
u u
+ u = 0, < x < , t > 0,
t x

1920
u(x, 0) = f (x).
Taking the Fourier Transform of the partial differential equation and the initial condition yields
U
U + U = 0,
t
Z
1
U (, 0) = F () = f (x) ex dx.
2
Now we have a first order differential equation for U (, t) with the solution

U (, t) = F () e(1+)t .

Now we apply the inverse Fourier transform.


Z
u(x, t) = F () e(1+)t ex d

Z
t
u(x, t) = e F () e(x+t) d

u(x, t) = et f (x + t)

1921
44.4 Exercises
Exercise 44.1
Find an integral representation of the solution u(x, y), of
uxx + uyy = 0 in < x < , 0 < y < ,
subject to the boundary conditions:
u(x, 0) = f (x), < x < ;
u(x, y) 0 as x2 + y 2 .
Exercise 44.2
Solve the Cauchy problem for the one-dimensional heat equation in the domain < x < , t > 0,
ut = uxx , u(x, 0) = f (x),
with the Fourier transform.
Exercise 44.3
Solve the Cauchy problem for the one-dimensional heat equation in the domain < x < , t > 0,
ut = uxx , u(x, 0) = f (x),
with the Laplace transform.
Exercise 44.4
1. In Exercise ?? above, let f (x) = f (x) for all x and verify that (x, t) so obtained is the solution, for x > 0,
of the following problem: find (x, t) satisfying
t = a2 xx
in 0 < x < , t > 0, with boundary condition (0, t) = 0 and initial condition (x, 0) = f (x). This technique,
in which the solution for a semi-infinite interval is obtained from that for an infinite interval, is an example of
what is called the method of images.

1922
2. How would you modify the result of part (a) if the boundary condition (0, t) = 0 was replaced by x (0, t) = 0?

Exercise 44.5
Solve the Cauchy problem for the one-dimensional wave equation in the domain < x < , t > 0,

utt = c2 uxx , u(x, 0) = f (x), ut (x, 0) = g(x),

with the Fourier transform.


Exercise 44.6
Solve the Cauchy problem for the one-dimensional wave equation in the domain < x < , t > 0,

utt = c2 uxx , u(x, 0) = f (x), ut (x, 0) = g(x),

with the Laplace transform.


Exercise 44.7
Consider the problem of determining (x, t) in the region 0 < x < , 0 < t < , such that

t = a2 xx , (44.1)

with initial and boundary conditions

(x, 0) = 0 for all x > 0,


(0, t) = f (t) for all t > 0,

where f (t) is a given function.

1. Obtain the formula for the Laplace transform of (x, t), (x, s) and use the convolution theorem for Laplace
transforms to show that Z t
x2
 
x 1
(x, t) = f (t ) 3/2 exp 2 d.
2a 0 4a

1923
2. Discuss the special case obtained by setting f (t) = 1 and also that in which f (t) = 1 for 0 < t < T , with
f (t) = 0 for t > T . Here T is some positive constant.

Exercise 44.8
Solve the radiating half space problem:
ut = uxx , x > 0, t > 0,
ux (0, t) u(0, t) = 0, u(x, 0) = f (x).
To do this, define
v(x, t) = ux (x, t) u(x, t)
and find the half space problem that v satisfies. Solve this problem and then show that
Z
u(x, t) = e(x) v(, t) d.
x

Exercise 44.9
Show that
Z
c 2 x x2 /(4c)
e sin(x) d = 3/2 e .
0 4c
Use the sine transform to solve:
ut = uxx , x > 0, t > 0,
u(0, t) = g(t), u(x, 0) = 0.
Exercise 44.10
Use the Fourier sine transform to find the steady state temperature u(x, y) in a slab: x 0, 0 y 1, which has zero
temperature on the faces y = 0 and y = 1 and has a given distribution: u(y, 0) = f (y) on the edge x = 0, 0 y 1.

Exercise 44.11
Find a harmonic function u(x, y) in the upper half plane which takes on the value g(x) on the x-axis. Assume that u
and ux vanish as |x| . Use the Fourier transform with respect to x. Express the solution as a single integral by
using the convolution formula.

1924
Exercise 44.12
Find the bounded solution of

ut = uxx a2 u, 0 < x < , t > 0,


ux (0, t) = f (t), u(x, 0) = 0.

Exercise 44.13
The left end of a taut string of length L is displaced according to u(0, t) = f (t). The right end is fixed, u(L, t) = 0.
Initially the string is at rest with no displacement. If c is the wave speed for the string, find its motion for all t > 0.
Exercise 44.14
Let 2 = 0 in the (x, y)-plane region defined by 0 < y < l, < x < , with (x, 0) = (x ), (x, l) = 0, and
0 as |x| . Solve for using Fourier transforms. You may leave your answer in the form of an integral but in
fact it is possible to use techniques of contour integration to show that
 
1 sin(y/l)
(x, y|) = .
2l cosh[(x )/l] cos(y/l)

Note that as l we recover the result derived in class:


1 y
,
(x )2 + y 2

which clearly approaches (x ) as y 0.

1925
44.5 Hints
Hint 44.1 R
The desired solution form is: u(x, y) = K(x , y)f () d. You must find the correct K. Take the Fourier
transform with respect to x and solve for u(, y) recalling that uxx = 2 u. By uxx we denote the Fourier transform
with respect to x of uxx (x, y).

Hint 44.2
Use the Fourier convolution theorem and the table of Fourier transforms in the appendix.

Hint 44.3

Hint 44.4

Hint 44.5
Use the Fourier convolution theorem. The transform pairs,

F[((x + ) + (x ))] = cos( ),


sin( )
F[(H(x + ) H(x ))] = ,

will be useful.
Hint 44.6

Hint 44.7

Hint 44.8
v(x, t) satisfies the same partial differential equation. You can solve the problem for v(x, t) with the Fourier sine

1926
transform. Use the convolution theorem to invert the transform.
To show that Z
u(x, t) = e(x) v(, t) d,
x
find the solution of
ux u = v
that is bounded as x .
Hint 44.9
Note that Z Z
c 2 2
e sin(x) d = ec cos(x) d.
0 x 0
Write the integral as a Fourier transform.
Take the Fourier sine transform of the heat equation to obtain a first order, ordinary differential equation for u(, t).
Solve the differential equation and do the inversion with the convolution theorem.
Hint 44.10

Hint 44.11

Hint 44.12

Hint 44.13

Hint 44.14

1927
44.6 Solutions
Solution 44.1
1. We take the Fourier transform of the integral equation, noting that the left side is the convolution of u(x) and
1
x2 +a2
.
   
1 1
2u()F 2 =F 2
x + a2 x + b2
1
We find the Fourier transform of f (x) = x2 +c
2 . Note that since f (x) is an even, real-valued function, f () is an

even, real-valued function.


  Z
1 1 1
F 2 2
= ex dx
x +c 2 x + c2
2

For x > 0 we close the path of integration in the upper half plane and apply Jordans Lemma to evaluate the
integral in terms of the residues.
ex
 
1
= 2 Res , x = c
2 (x c)(x + c)
ec
=
2c
1 c
= e
2c
Since f() is an even function, we have
 
1 1 c||
F 2 2
= e .
x +c 2c
Our equation for u() becomes,
1 a|| 1 b||
2u() e = e
2a 2b
a (ba)|omega|
u() = e .
2b

1928
We take the inverse Fourier transform using the transform pair we derived above.
a 2(b a)
u(x) =
2b x + (b a)2
2

a(b a)
u(x) =
b(x2 + (b a)2 )

2. We take the Fourier transform of the partial differential equation and the boundary condtion.

uxx + uyy = 0, u(x, 0) = f (x)


2
u(, y) + uyy (, y) = 0, u(, 0) = f()

This is an ordinary differential equation for u in which is a parameter. The general solution is

u = c1 ey +c2 ey .

We apply the boundary conditions that u(, 0) = f() and u 0 and y .

u(, y) = f() ey

We take the inverse transform using the convolution theorem.


Z
1
u(x, y) = e(x)y f () d
2

Solution 44.2

ut = uxx , u(x, 0) = f (x),


We take the Fourier transform of the heat equation and the initial condition.

ut = 2 u, u(, 0) = f()

1929
This is a first order ordinary differential equation which has the solution,
2
u(, t) = f() e t .

Using a table of Fourier transforms we can write this in a form that is conducive to applying the convolution theorem.
r 
x2 /(4t)
u(, t) = f ()F e
t
Z
1 2
u(x, t) = e(x) /(4t) f () d
2 t

Solution 44.3
We take the Laplace transform of the heat equation.

ut = uxx
su u(x, 0) = uxx
s f (x)
uxx u = (44.2)

The Green function problem for Equation 44.2 is
s
G00 G = (x ), G(; ) is bounded.

The homogeneous solutions that satisfy the left and right boundary conditions are, respectively,
   
sa sa
exp , exp .
x x
We compute the Wronskian of these solutions.
   
s
exp a x exp as x
r
  = 2 s

W = s  
sa

a exp x a exp xsa
s

1930
The Green function is

ps
exp s x>
p  
exp x
<
G(x; ) =
2 s
p
 r 
s
G(x; ) = exp |x | .
2 s

Now we solve Equation 44.2 using the Green function.

Z
f ()
u(x, s) = G(x; ) d

Z  r 
1 s
u(x, s) = f () exp |x | d
2 s

Finally we take the inverse Laplace transform to obtain the solution of the heat equation.


(x )2
Z  
1
u(x, t) = f () exp d
2 t 4t

Solution 44.4
1. Clearly the solution satisfies the differential equation. We must verify that it satisfies the boundary condition,

1931
(0, t) = 0.

(x )2
Z  
1
(x, t) = f () exp d
2a t 4a2 t
Z 0 Z
(x )2 (x )2
   
1 1
(x, t) = f () exp d + f () exp d
2a t 4a2 t 2a t 0 4a2 t
Z Z
(x + )2 (x )2
   
1 1
(x, t) = f () exp d + f () exp d
2a t 0 4a2 t 2a t 0 4a2 t
Z Z
(x + )2 (x )2
   
1 1
(x, t) = f () exp d + f () exp d
2a t 0 4a2 t 2a t 0 4a2 t
Z
(x )2 (x + )2
    
1
(x, t) = f () exp exp d
2a t 0 4a2 t 4a2 t
Z  2
x + 2
    
1 x x
(x, t) = f () exp exp exp 2 d
2a t 0 4a2 t 2a2 t 2a t
Z  2
x + 2
  
1 x
(x, t) = f () exp sinh d
a t 0 4a2 t 2a2 t

Since the integrand is zero for x = 0, the solution satisfies the boundary condition there.
2. For the boundary condition x (0, t) = 0 we would choose f (x) to be even. f (x) = f (x). The solution is
Z  2
x + 2
  
1 x
(x, t) = f () exp cosh d
a t 0 4a2 t 2a2 t

The derivative with respect to x is


Z  2
x + 2
    
1 x x
x (x, t) = 3 3/2 f () exp sinh x cosh d.
2a t 0 4a2 t 2a2 t 2a2 t
Since the integrand is zero for x = 0, the solution satisfies the boundary condition there.

1932
Solution 44.5

utt = c2 uxx , u(x, 0) = f (x), ut (x, 0) = g(x),


With the change of variables
t 1
= ct, = = , v(x, ) = u(x, t),
t c t
the problem becomes
1
v = vxx , v(x, 0) = f (x), v (x, 0) = g(x).
c
(This change of variables isnt necessary, it just gives us fewer constants to carry around.) We take the Fourier transform
in x of the equation and the initial conditions, (we consider to be a parameter).
1
v (, ) = 2 v(, ), v(, ) = f(), v (, ) = g()
c
Now we have an ordinary differential equation for v(, ), (now we consider to be a parameter). The general solution
of this constant coefficient differential equation is,
v(, ) = a() cos( ) + b() sin( ),
where a and b are constants that depend on the parameter . We applying the initial conditions to obtain v(, ).
1
v(, ) = f() cos( ) + g() sin( )
c
With the Fourier transform pairs
F[((x + ) + (x ))] = cos( ),
sin( )
F[(H(x + ) H(x ))] = ,

we can write v(, ) in a form that is conducive to applying the Fourier convolution theorem.
1
v(, ) = F[f (x)]F[((x + ) + (x ))] + F[g(x)]F[(H(x + ) H(x ))]
c

1933
Z
1
v(x, ) = f ()((x + ) + (x )) d
2
Z
1 1
+ g()(H(x + ) H(x )) d
c 2

Z x+
1 1
v(x, ) = (f (x + ) + f (x )) + g() d
2 2c x

Finally we make the change of variables t = /c, u(x, t) = v(x, ) to obtain DAlemberts solution of the wave equation,
Z x+ct
1 1
u(x, t) = (f (x ct) + f (x + ct)) + g() d.
2 2c xct

Solution 44.6
With the change of variables
t 1
= ct, = = , v(x, ) = u(x, t),
t c t
the problem becomes
1
v = vxx , v (x, 0) = g(x).
v(x, 0) = f (x),
c
We take the Laplace transform in of the equation, (we consider x to be a parameter),

s2 V (x, s) sv(x, 0) v (x, 0) = Vxx (x, s),

1
Vxx (x, s) s2 V (x, s) = sf (x) g(x),
c
Now we have an ordinary differential equation for V (x, s), (now we consider s to be a parameter). We impose the
boundary conditions that the solution is bounded at x = . Consider the Greens function problem

gxx (x; ) s2 g(x; ) = (x ), g(; ) bounded.

1934
esx is a homogeneous solution that is bounded at x = . esx is a homogeneous solution that is bounded at
x = +. The Wronskian of these solutions is
sx sx

e e
W (x) = sx = 2s.
se s esx

Thus the Greens function is


(
1 sx s
2s e e for x < , 1
g(x; ) = 1 s sx
= es|x| .
2s e e for x > , 2s

The solution for V (x, s) is


1 s|x|
Z
1
V (x, s) = e (sf () g()) d,
2s c
Z Z
1 1
V (x, s) = es|x| f () d + es|x| g()) d,
2 2cs
1 s|| 1 es||
Z Z
V (x, s) = e f (x ) d + g(x )) d.
2 2c s
Now we take the inverse Laplace transform and interchange the order of integration.
Z  Z s|| 
1 1 s|| 1 1 e
v(x, ) = L e f (x ) d + L g(x )) d
2 2c s

1 1  s||  1 1 es||
Z Z  
v(x, ) = L e f (x ) d + L g(x )) d
2 2c s
1 1
Z Z
v(x, ) = ( ||)f (x ) d + H( ||)g(x )) d
2 2c
1
Z
1
v(x, ) = (f (x ) + f (x + )) + g(x ) d
2 2c

1935
Z x+
1 1
v(x, ) = (f (x ) + f (x + )) + g() d
2 2c x
Z x+
1 1
v(x, ) = (f (x ) + f (x + )) + g() d
2 2c x

Now we write make the change of variables t = /c, u(x, t) = v(x, ) to obtain DAlemberts solution of the wave
equation,
1 x+ct
Z
1
u(x, t) = (f (x ct) + f (x + ct)) + g() d.
2 2c xct

Solution 44.7
1. We take the Laplace transform of Equation 44.1.

s (x, 0) = a2 xx
s
xx 2 = 0 (44.3)
a

We take the Laplace transform of the initial condition, (0, t) = f (t), and use that (x, s) vanishes as x
to obtain boundary conditions for (x, s).

(0, s) = f(s), (, s) = 0

The solutions of Equation 44.3 are


 
s
exp x .
a
The solution that satisfies the boundary conditions is
 
s
(x, s) = f(s) exp x .
a

1936
We write this as the product of two Laplace transforms.

2
  
x x
(x, s) = f(s)L exp 2
2a t3/2 4a t

We invert using the convolution theorem.

t
x2
Z  
x 1
(x, t) = f (t ) exp 2 d.
2a 0 3/2 4a

2. Consider the case f (t) = 1.

Z t
x2
 
x 1
(x, t) = exp 2 d
2a 0 3/2 4a
x x
= , d =
2a 4a 3/2

Z x/(2a t)
2 2
(x, t) = e d

 
x
(x, t) = erfc
2a t

Now consider the case in which f (t) = 1 for 0 < t < T , with f (t) = 0 for t > T . For t < T , is the same as
before.
 
x
(x, t) = erfc , for 0 < t < T
2a t

1937
Consider t > T .
t
x2
Z  
x 1
(x, t) = exp 2 d
2a tT 3/2 4a
Z x/(2at)
2 2
(x, t) =
e d
x/(2a tT )
   
x x
(x, t) = erf erf
2a t T 2a t

Solution 44.8

ut = uxx , x > 0, t > 0,


ux (0, t) u(0, t) = 0, u(x, 0) = f (x).

First we find the partial differential equation that v satisfies. We start with the partial differential equation for u,

ut = uxx .

Differentiating this equation with respect to x yields,

utx = uxxx .

Subtracting times the former equation from the latter yields,

utx ut = uxxx uxx ,


2
(ux u) = 2 (ux u) ,
t x
vt = vxx .

1938
Thus v satisfies the same partial differential equation as u. This is because the equation for u is linear and homogeneous
and v is a linear combination of u and its derivatives. The problem for v is,

vt = vxx , x > 0, t > 0,


v(0, t) = 0, v(x, 0) = f 0 (x) f (x).

With this new boundary condition, we can solve the problem with the Fourier sine transform. We take the sine transform
of the partial differential equation and the initial condition.
 
2 1
vt (, t) = v(, t) + v(0, t) ,

0
v(, 0) = Fs [f (x) f (x)]

vt (, t) = 2 v(, t)
v(, 0) = Fs [f 0 (x) f (x)]

Now we have a first order, ordinary differential equation for v. The general solution is,
2
v(, t) = c e t .

The solution subject to the initial condition is,


2
v(, t) = Fs [f 0 (x) f (x)] e t .

Now we take the inverse sine transform to find v. We utilize the Fourier cosine transform pair,
h 2
i r 2
Fc1 e t = ex /(4t) ,
t
to write v in a form that is suitable for the convolution theorem.
r 
0 x2 /(4t)
v(, t) = Fs [f (x) f (x)] Fc e
t

1939
Recall that the Fourier sine convolution theorem is,
 Z 
1
Fs f () (g(|x |) g(x + )) d = Fs [f (x)]Fc [g(x)].
2 0

Thus v(x, t) is
Z
1 0

|x|2 /(4t) (x+)2 /(4t)

v(x, t) = (f () f ()) e e d.
2 t 0

With v determined, we have a first order, ordinary differential equation for u,

ux u = v.

We solve this equation by multiplying by the integrating factor and integrating.

x 
e u = ex v
x Z
x
ex u = e v(x, t) d + c(t)
Z x
u= e(x) v(x, t) d + ex c(t)

The solution that vanishes as x is

Z
u(x, t) = e(x) v(, t) d.
x

1940
Solution 44.9

Z Z
c 2 2
e sin(x) d = ec cos(x) d
0 x 0
Z
1 2
= ec cos(x) d
2 x
Z
1 2
= ec +x d
2 x
Z
1 2 2
= ec(+x/(2c)) ex /(4c) d
2 x
1 x2 /(4c) c2
Z
= e e d
2 x
r
1 x2 /(4c)
= e
2 c x

x x2 /(4c)
= 3/2 e
4c

ut = uxx , x > 0, t > 0,


u(0, t) = g(t), u(x, 0) = 0.
We take the Fourier sine transform of the partial differential equation and the initial condition.

ut (, t) = 2 u(, t) + g(t), u(, 0) = 0

Now we have a first order, ordinary differential equation for u(, t).
 2 t  2
e ut (, t) = g(t) e t
t
2 t t
Z
2 2
u(, t) = e g( ) e d + c() e t
0

1941
The initial condition is satisfied for c() = 0.
Z t
2 (t )
u(, t) = g( ) e d
0

We take the inverse sine transform to find u.


 Z t 
2 (t )
u(x, t) = Fs1 g( ) e d
0

Z t h i
2 (t )
u(x, t) = g( )Fs1 e d
0
Z t
x 2
u(x, t) = g( ) 3/2
ex /(4(t )) d
0 2 (t )
t 2
ex /(4(t ))
Z
x
u(x, t) = g( ) d
2 0 (t )3/2

Solution 44.10
The problem is

uxx + uyy = 0, 0 < x, 0 < y < 1,


u(x, 0) = u(x, 1) = 0, u(0, y) = f (y).

We take the Fourier sine transform of the partial differential equation and the boundary conditions.

k
2 u(, y) + u(0, y) + uyy (, y) = 0

k
uyy (, y) 2 u(, y) = f (y), u(, 0) = u(, 1) = 0

1942
This is an inhomogeneous, ordinary differential equation that we can solve with Green functions. The homogeneous
solutions are
{cosh(y), sinh(y)}.

The homogeneous solutions that satisfy the left and right boundary conditions are

y1 = sinh(y), y2 = sinh((y 1)).

The Wronskian of these two solutions is,



sinh(y) sinh((y 1))
W (x) =
cosh(y) cosh((y 1))
= (sinh(y) cosh((y 1)) cosh(y) sinh((y 1)))
= sinh().

The Green function is


sinh(y< ) sinh((y> 1))
G(y|) = .
sinh()
The solution of the ordinary differential equation for u(, y) is

1
Z
u(, y) = f ()G(y|) d
0
1 y sinh() sinh((y 1)) 1 1 sinh(y) sinh(( 1))
Z Z
= f () d f () d.
0 sinh() y sinh()

With some uninteresting grunge, you can show that,



sinh() sinh((y 1))
Z
sin() sin(y)
2 sin(x) d = 2 .
0 sinh() (cosh(x) cos((y )))(cosh(x) cos((y + )))

1943
Taking the inverse Fourier sine transform of u(, y) and interchanging the order of integration yields,
Z y
2 sin() sin(y)
u(x, y) = f () d
0 (cosh(x) cos((y )))(cosh(x) cos((y + )))
2 1
Z
sin(y) sin()
+ f () d.
y (cosh(x) cos(( y)))(cosh(x) cos(( + y)))

Z 1
2 sin() sin(y)
u(x, y) = f () d
0 (cosh(x) cos((y )))(cosh(x) cos((y + )))

Solution 44.11
The problem for u(x, y) is,

uxx + uyy = 0, < x < , y > 0,


u(x, 0) = g(x).

We take the Fourier transform of the partial differential equation and the boundary condition.

2 u(, y) + uyy (, y) = 0, u(, 0) = g().

This is an ordinary differential equation for u(, y). So far we only have one boundary condition. In order that u
is bounded we impose the second boundary condition u(, y) is bounded as y . The general solution of the
differential equation is (
c1 () ey +c2 () ey , for 6= 0,
u(, y) =
c1 () + c2 ()y, for = 0.

Note that ey is the bounded solution for < 0, 1 is the bounded solution for = 0 and ey is the bounded solution
for > 0. Thus the bounded solution is
u(, y) = c() e||y .

1944
The boundary condition at y = 0 determines the constant of integration.

u(, y) = g() e||y

Now we take the inverse Fourier transform to obtain the solution for u(x, y). To do this we use the Fourier transform
pair,
 
2c
F 2 2
= ec|| ,
x +c

and the convolution theorem,


 Z 
1
F f ()g(x ) d = f()g().
2

Z
1 2y
u(x, y) = g() d.
2 (x )2 + y 2

Solution 44.12
Since the derivative of u is specified at x = 0, we take the cosine transform of the partial differential equation and the
initial condition.
 
1
ut (, t) = u(, t) ux (0, t) a2 u(, t), u(, 0) = 0
2


ut + 2 + a2 u = f (t), u(, 0) = 0


This first order, ordinary differential equation for u(, t) has the solution,
Z t
2 +a2 )(t )
u(, t) = e( f ( ) d.
0

1945
We take the inverse Fourier cosine transform to find the solution u(x, t).
Z t 
1 ( 2 +a2 )(t )
u(x, t) = Fc e f ( ) d
0
t 1 h 2 (t ) i a2 (t )
Z
u(x, t) = F e e f ( ) d
0 c
t
Z r
2 2
u(x, t) = ex /(4(t )) ea (t ) f ( ) d
0 (t )
r Z t x2 /(4(t ))a2 (t )
e
u(x, t) = f ( ) d
0 t

Solution 44.13
Mathematically stated we have
utt = c2 uxx , 0 < x < L, t > 0,
u(x, 0) = ut (x, 0) = 0,
u(0, t) = f (t), u(L, t) = 0.
We take the Laplace transform of the partial differential equation and the boundary conditions.
s2 u(x, s) su(x, 0) ut (x, 0) = c2 uxx (x, s)
s2
uxx = 2 u, u(0, s) = f(s), u(L, s) = 0
c
Now we have an ordinary differential equation. A set of solutions is
n  sx   sx o
cosh , sinh .
c c
The solution that satisfies the right boundary condition is
 
s(L x)
u = a sinh .
c

1946
The left boundary condition determines the multiplicative constant.

sinh(s(L x)/c)
u(x, s) = f(s)
sinh(sL/c)

If we can find the inverse Laplace transform of

sinh(s(L x)/c)
u(x, s) =
sinh(sL/c)

then we can use the convolution theorem to write u in terms of a single integral. We proceed by expanding this function
in a sum.

sinh(s(L x)/c) es(Lx)/c es(Lx)/c


=
sinh(sL/c) esL/c esL/c
esx/c es(2Lx)/c
=
1 e2sL/c

sx/c s(2Lx)/c
X
= e e e2nsL/c
n=0

X
X
= es(2nL+x)/c es(2(n+1)Lx)/c
n=0 n=0

X
X
= es(2nL+x)/c es(2nLx)/c
n=0 n=1

Now we use the Laplace transform pair:


L[(x a)] = esa .
  X
1 sinh(s(L x)/c) X
L = (t (2nL + x)/c) (t (2nL x)/c)
sinh(sL/c) n=0 n=1

1947
We write u in the form,
"
#
X X
u(x, s) = L[f (t)]L (t (2nL + x)/c) (t (2nL x)/c) .
n=0 n=1

By the convolution theorem we have



!
Z t X X
u(x, t) = f ( ) (t (2nL + x)/c) (t (2nL x)/c) d.
0 n=0 n=1

We can simplify this a bit. First we determine which Dirac delta functions have their singularities in the range (0..t).
For the first sum, this condition is
0 < t (2nL + x)/c < t.
The right inequality is always satisfied. The left inequality becomes

(2nL + x)/c < t,


ct x
n< .
2L
For the second sum, the condition is
0 < t (2nL x)/c < t.
Again the right inequality is always satisfied. The left inequality becomes
ct + x
n< .
2L
We change the index range to reflect the nonzero contributions and do the integration.
ctx
b ct+x

Z t b 2L c 2L
c
X X
u(x, t) = f ( ) (t (2nL + x)/c) (t (2nL x)/c) d.
0 n=0 n=1

1948
b ctx
2L
c b ct+x
2L
c
X X
u(x, t) = f (t (2nL + x)/c) f (t (2nL x)/c)
n=0 n=1

Solution 44.14
We take the Fourier transform of the partial differential equation and the boundary conditions.

1
2 + yy = 0, (, 0) = e , (, l) = 0
2
We solve this boundary value problem.

(, y) = c1 cosh((l y)) + c2 sinh((l y))


1 sinh((l y))
(, y) = e
2 sinh(l)

We take the inverse Fourier transform to obtain an expression for the solution.

sinh((l y))
Z
1
(x, y) = e(x) d
2 sinh(l)

1949
Chapter 45

Green Functions

45.1 Inhomogeneous Equations and Homogeneous Boundary Con-


ditions
Consider a linear differential equation on the domain subject to homogeneous boundary conditions.
L[u(x)] = f (x) for x , B[u(x)] = 0 for x (45.1)
For example, L[u] might be
L[u] = ut u, or L[u] = utt c2 u.
and B[u] might be u = 0, or u n = 0.

If we find a Green function G(x; xi) that satisfies


L[G(x; xi)] = (x xi), B[G(x; xi)] = 0
then the solution to Equation 45.1 is Z
u(x) = G(x; xi)f (xi) dxi.

1950
We verify that this solution satisfies the equation and boundary condition.
Z
L[u(x)] = L[G(x; xi)]f (xi) dxi
Z
= (x xi)f (xi) dxi

= f (x)
Z
B[u(x)] = B[G(x; xi)]f (xi) dxi
Z
= 0 f (xi) dxi

=0

45.2 Homogeneous Equations and Inhomogeneous Boundary Con-


ditions
Consider a homogeneous linear differential equation on the domain subject to inhomogeneous boundary conditions,

L[u(x)] = 0 for x , B[u(x)] = h(x) for x . (45.2)

If we find a Green function g(x; xi) that satisfies

L[g(x; xi)] = 0, B[g(x; xi)] = (x xi)

then the solution to Equation 45.2 is Z


u(x) = g(x; xi)h(xi) dxi.

1951
We verify that this solution satisfies the equation and boundary condition.
Z
L[u(x)] = L[g(x; xi)]h(xi) dxi
Z
= 0 h(xi) dxi

=0
Z
B[u(x)] = B[g(x; xi)]h(xi) dxi
Z

= (x xi)h(xi) dxi

= h(x)

Example 45.2.1 Consider the Cauchy problem for the homogeneous heat equation.

ut = uxx , < x < , t > 0


u(x, 0) = h(x), u(, t) = 0

We find a Green function that satisfies

gt = gxx , < x < , t > 0


g(x, 0; ) = (x ), g(, t; ) = 0.

Then we write the solution Z


u(x, t) = g(x, t; )h() d.

To find the Green function for this problem, we apply a Fourier transform to the equation and boundary condition

1952
for g.

gt = 2 g, g(, 0; ) = F[(x )]
2
g(, t; ) = F[(x )] e t
r
x2
 

g(, t; ) = F[(x )]F exp
t 4t
We invert using the convolution theorem.

r
(x )2
Z  
1
g(x, t; ) = ( ) exp d
2 t 4t
(x )2
 
1
= exp
4t 4t

The solution of the heat equation is


(x )2
Z  
1
u(x, t) = exp h() d.
4t 4t

45.3 Eigenfunction Expansions for Elliptic Equations


Consider a Green function problem for an elliptic equation on a finite domain.

L[G] = (x xi), x (45.3)


B[G] = 0, x

Let the set of functions {n } be orthonormal and complete on . (Here n is the multi-index n = n1 , . . . , nd .)
Z
n (x)m (x) dx = nm

1953
In addition, let the n be eigenfunctions of L subject to the homogeneous boundary conditions.

L [n ] = n n , B [n ] = 0

We expand the Green function in the eigenfunctions.


X
G= gn n (x)
n

Then we expand the Dirac Delta function.


X
(x xi) = dn n (x)
n
Z
dn = n (x)(x xi) dx

dn = n (xi)

We substitute the series expansions for the Green function and the Dirac Delta function into Equation 45.3.
X X
gn n n (x) = n (xi)n (x)
n n

We equate coefficients to solve for the gn and hence determine the Green function.

n (xi)
gn =
n
X n (xi)n (x)
G(x; xi) =
n
n

Example 45.3.1 Consider the Green function for the reduced wave equation, u k 2 u in the rectangle, 0 x a,
0 y b, and vanishing on the sides.

1954
First we find the eigenfunctions of the operator L = k 2 = 0. Note that = X(x)Y (y) is an eigenfunction of
2 2
L if X is an eigenfunction of x 2 and Y is an eigenfunction of y 2 . Thus we consider the two regular Sturm-Liouville

eigenvalue problems:

X 00 = X, X(0) = X(a) = 0
Y 00 = Y, Y (0) = Y (b) = 0

This leads us to the eigenfunctions  mx   ny 


mn = sin sin .
a b
We use the orthogonality relation Z 2  mx   nx  a
sin sin dx = mn
0 a a 2
to make the eigenfunctions orthonormal.
2  mx   ny 
mn = sin sin , m, n Z+
ab a b
The mn are eigenfunctions of L.
 
m 2  n 2 2
L [mn ] = + + k mn
a b
By expanding the Green function and the Dirac Delta function in the mn and substituting into the differential equation
we obtain the solution.
2 sin m sin n 2 sin mx sin ny
   
a b a b
X ab ab
G=  
m 2 n 2
 
+ b +k 2
m,n=1 a
m ny n
sin mx
   
X
a
sin a
sin b
sin b
G(x, y; , ) = 4ab 2 + (na)2 + (kab)2
m,n=1
(mb)

1955
Example 45.3.2 Consider the Green function for Laplaces equation, u = 0 in the disk, |r| < a, and vanishing at
r = a.
First we find the eigenfunctions of the operator
2 1 1 2
= + + .
r2 r r r2 2
d2
We will look for eigenfunctions of the form = ()R(r). We choose the to be eigenfunctions of d2
subject to
the periodic boundary conditions in .

00 = , (0) = (2), 0 (0) = 0 (2)


n = ein , nZ

We determine R(r) by requiring that be an eigenfunction of .

=
1 1
(n R)rr + (n R)r + 2 (n R) = n R
r r
1 1
n R00 + n R0 + 2 (n2 )n R = R
r r

For notational convenience, we denote = 2 .

n2
 
00 1 0 2
R + R + 2 R = 0, R(0) bounded, R(a) = 0
r r
The general solution for R is
R = c1 Jn (r) + c2 Yn (r).
The left boundary condition demands that c2 = 0. The right boundary condition determines the eigenvalues.
 
jn,m r jn,m
Rnm = Jn , nm =
a a

1956
Here jn,m is the mth positive root of Jn . This leads us to the eigenfunctions
 
in jn,m r
nm = e Jn
a

We use the orthogonality relations


Z 2
eim ein d = 2mn ,
0
Z 1
1 0 2
rJ (j,m r)J (j,n r) dr = (J (j,n )) mn
0 2

to make the eigenfunctions orthonormal.


 
1 in jn,m r
nm = e Jn , n Z, m Z+
a|J 0 n (jn,m )| a

The nm are eigenfunctions of L.


 2
jn,m
nm = nm
a
By expanding the Green function and the Dirac Delta function in the nm and substituting into the differential equation
we obtain the solution.
   
1 in jn,m 1 in jn,m r
X X
0
a|J n (jn,m )|
e J n a
0
a|J n (jn,m )|
e J n a
G=  2
n= m=1 jn,m a
   
X X 1 in() jn,m jn,m r
G(r, ; , ) = 0 (j 2
e Jn Jn
n= m=1
(j n,m J n n,m )) a a

1957
45.4 The Method of Images
Consider Poissons equation in the upper half plane.

2 u = f (x, y), < x < , y > 0


u(x, 0) = 0, u(x, y) 0 as x2 + y 2

The associated Green function problem is

2 G = (x )(y ), < x < , y > 0


G(x, 0|, ) = 0, G(x, y|, ) 0 as x2 + y 2 .

We will solve the Green function problem with the method of images. We expand the domain to include the lower
half plane. We place a negative image of the source in the lower half plane. This will make the Green function odd
about y = 0, i.e. G(x, 0|, ) = 0.

2 G = (x )(y ) (x )(y + ), < x < , y>0


G(x, y|, ) 0 as x2 + y 2

Recall that the infinite space Green function which satisfies F = (x )(y ) is
1
ln (x )2 + (y )2 .

F (x, y|, ) =
4
We solve for G by using the infinite space Green function.

G = F (x, y|, ) F (x, y|, )


1 1
ln (x )2 + (y )2 ln (x )2 + (y + )2
 
=
4   4
1 (x )2 + (y )2
= ln
4 (x )2 + (y + )2

1958
We write the solution of Poissons equation using the Green function.
Z Z
u(x, y) = G(x, y|, )f (, ) d d
0

(x )2 + (y )2
Z Z  
1
u(x, y) = ln f (, ) d d
0 4 (x )2 + (y + )2

1959
45.5 Exercises
Exercise 45.1
Consider the Cauchy problem for the diffusion equation with a source.

ut uxx = s(x, t), u(x, 0) = f (x), u 0 as x

Find the Green function for this problem and use it to determine the solution.

Exercise 45.2
Consider the 2-dimensional wave equation
utt c2 (uxx + uyy ) = 0.

1. Determine the fundamental solution for this equation. (i.e. response to source at t = , x = ). You may find
the following information useful about the Bessel function:

1 x cos
Z
J0 (x) = e d,
0
Z (
0, 0<b<a
J0 (ax) sin(bx) dx = 1
0
b2 a2
, 0<a<b

2. Use the method of descents to recover the 1-D fundamental solution.


Exercise 45.3
Consider the linear wave equation
utt = c2 uxx ,
with constant c, on the infinite domain < x < .

1. By using the Fourier transform find the solution of Gtt = c2 Gxx subject to initial conditions G(x, 0) = 0,
Gt (x, 0) = (x ).

1960
2. Now use this to find u in the case where c = 1, u(x, 0) = 0, and
(
0 |x| > 1
ut (x, 0) =
1 |x| |x| < 1

Sketch the solution in x for fixed times t < 1 and t > 1 and also indicate on the x, t (t > 0) plane the regions of
qualitatively different behavior of u.
Exercise 45.4
Consider a generalized Laplace equation with non-constant coefficients of the form:

2 u + A(x) u + h(x)u = q(x),

on a region V with u = 0 on the boundary S. Suppose we find a Green function which satisfies

2 G + A(x) G + h(x)G = (x ).

Use the divergence theorem to derive an appropriate generalized Greens identity and show that
Z
u() 6= G(x|)q(x) dx.
V

What equation should the Green function satisfy? Note: this equation is called the adjoint of the original partial
differential equation.
Exercise 45.5
Consider Laplaces equation in the infinite three dimensional domain with two sources of equal strength C, opposite
sign and separated by a distance .
2 u = C(x + ) C(x ),
where = ( 2 , 0, 0).

1. Find the solution in terms of the fundamental solutions.

1961
2. Now consider the limit in which the distance between sources goes to zero ( 0) and the strength increases in
such a way that C = D remains fixed. Show that the solution can be written
Dx
u= ,
4r3
where r = |x|. This is called the response to a dipole located at the origin, with strength D, and oriented in the
positive x direction.

3. Show that in general the response to a unit (D = 1) dipole at an arbitrary point 0 and oriented in the direction
of the unit vector a is  
1 1
u(x) = ~a
4 |x | =0

Exercise 45.6
Consider Laplaces equation
2 u = 0,
inside the unit circle with boundary condition u = f (). By using the Green function for the Dirichlet problem on the
circle:  
1 |x |
G(x|) = ln ,
2 |||x |
where and have the same polar angle and | | = ||
1
, show that the solution may be expressed in polar coordinates
as
1 r2 2
Z
f ()
u(r, ) = 2
d.
2 0 1 + r 2r cos( )

Exercise 45.7
Consider an alternate derivation of the fundamental solution of Laplaces equation

2 u = (x),

with u 0 as |x| in three dimensions.

1962
1. Convert this equation to spherical coordinates. You may define a new delta function
Z (
1 if B contains the origin
3 (r) = (x)(y)(z) such that 3 (r) dV =
B 0 otherwise

2. Show, by symmetry, that this can be reduced to an ordinary differential equation. Solve to find the general
solution of the homogeneous equation. Now determine the constants by using the constraint that u 0 as
|x| , and by integrating the partial differential equation over a small ball around the origin (and using Gauss
theorem).
3. Now use similar ideas to re-derive the fundamental solution in two dimensions. Can we still say u 0 as
|x| ? Use instead the constraint that u = 0 when |x| = 1.
4. Finally derive the 2-D solution from the 3-D one using the method of descent. Consider Laplaces equation in
three dimensions with a line source at x = 0, y = 0, < z < ,
uxx + uyy + uzz = (x)(y).
p
Use the fundamental solution to find u(r) where r = x2 + y 2 , and without loss of generality we have taken
the plane at z = 0. Then evaluate this integral to find u. (Hint: first try to compute ur )

Exercise 45.8
Consider the heat equation on the bounded domain 0 < x < L with fixed temperature at each end. Use Laplace
transforms to determine the Green Function which satisfies
Gt Gxx = (x )(t),
G(0, t) = 0 G(L, t) = 0,
G(x, 0 ) = 0.
1. First show that ps  ps 
cosh
(L x> + x< ) cosh
(L x> x< )
L[G(x, t)] = ps 
2 s sinh
L

1963
2. Show that this can be re-written as

X 1 s 1 s
L[G(x, t)] = e |x2kL| e |x+2kL| .
k=
2 s 2 s

3. Use this to find G in terms of fundamental solutions


1 x2
f (x, t) = e 4t ,
2 t
and comment on how this Greens function corresponds to real and image sources. Additionally compare
this to the alternative expression,

2 X n222 t nx n
G(x, t) = e L sin sin ,
L n=1 L L

and comment on the convergence of the respective formulations for small and large time.

Exercise 45.9
Consider the Green function for the 1-D heat equation

Gt Gxx = (x )(t ),

on the semi-infinite domain with insulated end

Gx (0, t) = 0, G 0 as x ,

and subject to the initial condition


G(x, ) = 0.

1. Solve for G with the Fourier cosine transform.

1964
2. (15 points) Relate this to the fundamental solution on the infinite domain, and discuss in terms of responses to
real and image sources. Give the solution for x > 0 of

ut uxx = q(x, t),


ux (0, t) = 0, u 0 as x ,
u(x, 0) = f (x).

Exercise 45.10
Consider the heat equation
ut = uxx + (x )(t),
on the infinite domain < x < , where we assume u 0 as x and initially u(x, 0 ) = 0.

1. First convert this to a problem where there is no forcing, so that

ut = uxx

with an appropriately modified initial condition.


2. Now use Laplace tranforms to convert this to an ordinary differential equation in u(x, s), where u(x, s) =
L[u(x, t)]. Solve this ordinary differential equation and show that
1 s |x|
u(x, s) = e .
2 s
R
Recall f(s) = L[f (t)] = 0 est f (t) dt.
3. Finally use the Laplace inversion formula and Cauchys Theorem on an appropriate contour to compute u(x, t).
Recall Z
1 1
f (t) = L [F (s)] = F (s) est ds,
2
where is the Bromwich contour (s = a + t where t ( . . . ) and a is a non-negative constant such that
the contour lies to the right of all poles of f).

1965
Exercise 45.11
Derive the causal Green function for the one dimensional wave equation on (..). That is, solve

Gtt c2 Gxx = (x )(t ),


G(x, t; , ) = 0 for t < .

Use the Green function to find the solution of the following wave equation with a source term.

utt c2 uxx = q(x, t), u(x, 0) = ut (x, 0) = 0

Exercise 45.12
By reducing the problem to a series of one dimensional Green function problems, determine G(x, xi) if

2 G = (x xi)

(a) on the rectangle 0 < x < L, 0 < y < H and

G(0, y; , ) = Gx (L, y; , ) = Gy (x, 0; , ) = Gy (x, H; , ) = 0

(b) on the box 0 < x < L, 0 < y < H, 0 < z < W with G = 0 on the boundary.

(c) on the semi-circle 0 < r < a, 0 < < with G = 0 on the boundary.

(d) on the quarter-circle 0 < r < a, 0 < < /2 with G = 0 on the straight sides and Gr = 0 at r = a.

Exercise 45.13
Using the method of multi-dimensional eigenfunction expansions, determine G(x, x0 ) if

2 G = (x x0 )

and

1966
(a) on the rectangle (0 < x < L, 0 < y < H)

G
at x = 0, G=0 at y = 0, =0
y

G G
at x = L, =0 at y = H, =0
x y

(b) on the rectangular shaped box (0 < x < L, 0 < y < H, 0 < z < W ) with G = 0 on the six sides.

(c) on the semi-circle (0 < r < a, 0 < < ) with G = 0 on the entire boundary.

(d) on the quarter-circle (0 < r < a, 0 < < /2) with G = 0 on the straight sides and G/r = 0 at r = a.

Exercise 45.14
Using the method of images solve

2 G = (x x0 )

in the first quadrant (x 0 and y 0) with G = 0 at x = 0 and G/y = 0 at y = 0. Use the Green function to
solve in the first quadrant

2 u = 0
u(0, y) = g(y)
u
(x, 0) = h(x).
y

1967
Exercise 45.15
Consider the wave equation defined on the half-line x > 0:

2u 2
2 u
= c + Q(x, t),
t2 x2
u(x, 0) = f (x)
u
(x, 0) = g(x)
t
u(0, t) = h(t)

(a) Determine the appropriate Greens function using the method of images.

(b) Solve for u(x, t) if Q(x, t) = 0, f (x) = 0, and g(x) = 0.

(c) For what values of t does h(t) influence u(x1 , t1 ). Interpret this result physically.

Exercise 45.16
Derive the Green functions for the one dimensional wave equation on (..) for non-homogeneous initial conditions.
Solve the two problems

gtt c2 gxx = 0, g(x, 0; , ) = (x ), gt (x, 0; , ) = 0,


tt c2 xx = 0, (x, 0; , ) = 0, t (x, 0; , ) = (x ),

using the Fourier transform.


Exercise 45.17
Use the Green functions from Problem 45.11 and Problem 45.16 to solve

utt c2 uxx = f (x, t), x > 0, < t <


u(x, 0) = p(x), ut (x, 0) = q(x).

Use the solution to determine the domain of dependence of the solution.

1968
Exercise 45.18
Show that the Green function for the reduced wave equation, u k 2 u = 0 in the rectangle, 0 x a, 0 y b,
and vanishing on the sides is:
 
2 X sinh(n y< ) sinh(n (y> b))  nx  n
G(x, y; , ) = sin sin ,
a n=1 n sinh(n b) a a

where r
n2 2
n = k2 + .
a2
Exercise 45.19
Find the Green function for the reduced wave equation u k 2 u = 0, in the quarter plane: 0 < x < , 0 < y <
subject to the mixed boundary conditions:

u(x, 0) = 0, ux (0, y) = 0.

Find two distinct integral representations for G(x, y; , ).

Exercise 45.20
Show that in polar coordinates the Green function for u = 0 in the infinite sector, 0 < < , 0 < r < , and
vanishing on the sides is given by,
  
r
1 cosh
ln
cos
( )
G(r, , , ) = ln    .
4 cosh
ln r
cos ( + )

Use this to find the harmonic function u(r, ) in the given sector which takes on the boundary values:
(
0 for r < c
u(r, ) = u(r, ) =
1 for r > c.

1969
Exercise 45.21
The Green function for the initial value problem,

ut uxx = 0, u(x, 0) = f (x),

on < x < is
1 2
G(x, t; ) = e(x) /(4t) .
4t
Use the method of images to find the corresponding Green function for the mixed initial-boundary problems:
1. ut = uxx , u(x, 0) = f (x) for x > 0, u(0, t) = 0,
2. ut = uxx , u(x, 0) = f (x) for x > 0, ux (0, t) = 0.

Exercise 45.22
Find the Green function (expansion) for the one dimensional wave equation utt c2 uxx = 0 on the interval 0 < x < L,
subject to the boundary conditions:

a) u(0, t) = ux (L, t) = 0,
b) ux (0, t) = ux (L, t) = 0.

Write the final forms in terms showing the propagation properties of the wave equation, i.e., with arguments ((x )
(t )).

Exercise 45.23
Solve, using the above determined Green function,

utt c2 uxx = 0, 0 < x < 1, t > 0,


ux (0, t) = ux (1, t) = 0,
u(x, 0) = x2 (1 x)2 , ut (x, 0) = 1.

For c = 1, find u(x, t) at x = 3/4, t = 7/2.

1970
45.6 Hints
Hint 45.1

Hint 45.2

Hint 45.3

Hint 45.4

Hint 45.5

Hint 45.6

Hint 45.7

Hint 45.8

Hint 45.9

Hint 45.10

1971
Hint 45.11

Hint 45.12
Take a Fourier transform in x. This will give you an ordinary differential equation Green function problem for G. Find
the continuity and jump conditions at t = . After solving for G, do the inverse transform with the aid of a table.
Hint 45.13

Hint 45.14

Hint 45.15

Hint 45.16

Hint 45.17

Hint 45.18
Use Fourier sine and cosine transforms.

Hint 45.19
The the conformal mapping z = w/ to map the sector to the upper half plane. The new problem will be
Gxx + Gyy = (x )(y ), < x < , 0 < y < ,
G(x, 0, , ) = 0,
G(x, y, , ) 0 as x, y .
Solve this problem with the image method.

1972
Hint 45.20

Hint 45.21

Hint 45.22

1973
45.7 Solutions
Solution 45.1
The Green function problem is
Gt Gxx = (x )(t ), G(x, t|, ) = 0 for t < , G 0 as x
We take the Fourier transform of the differential equation.
Gt + 2 G = F[(x )](t ), G(, t|, ) = 0 for t <
Now we have an ordinary differential equation Green function problem for G. The homogeneous solution of the ordinary
differential equation is
2
e t
The jump condition is
G(, 0; , + ) = F[(x )].
We write the solution for G and invert using the convolution theorem.
2
G = F[(x )] e (t ) H(t )
r 
x2 /(4(t ))
G = F[(x )]F e H(t )
(t )
Z r
1 2
G= (x y ) ey /(4(t )) dyH(t )
2 (t )
1 2
G= p e(x) /(4(t )) H(t )
4(t )
We write the solution of the diffusion equation using the Green function.
Z Z Z
u= G(x, t|, )s(, ) d d + G(x, t|, 0)f () d
0
Z t Z Z
1 (x)2 /(4(t )) 1 2 /(4t)
u= p e s(, ) d d + e(x) f () d
0 4(t ) 4t

1974
Solution 45.2
1. We apply Fourier transforms in x and y to the Green function problem.

Gtt c2 (Gxx + Gyy ) = (t )(x )(y )


 1 1
Gtt + c2 2 + 2 G = (t ) e e
2 2


This gives us an ordinary differential equation Green function problem for G(, , t). We find the causal solution.

That is, the solution that satisfies G(, , t) = 0 for t < .

p 
sin 2 + 2 c(t ) 1
e(+) H(t )
G = p 2
2
c + 2 4

Now we take inverse Fourier transforms in and .


e((x)+(y))
Z Z p 
G= p sin 2 + 2 c(t ) d dH(t )
4 2 c 2 + 2

We make the change of variables = cos , = sin and do the integration in polar coordinates.

2
e((x) cos +(y) sin )
Z Z
1
G= 2 sin (c(t )) d dH(t )
4 c 0 0

1975
Next we introduce polar coordinates for x and y.

x = r cos , y = r sin
Z Z 2
1
G= er(cos cos +sin sin ) d sin (c(t )) dH(t )
4 2 c
0
Z0 Z 2
1
G= 2 er cos() d sin (c(t )) dH(t )
4 c 0
Z0
1
G= J0 (r) sin (c(t )) dH(t )
2c 0
1 1
G= p H(c(t ) r)H(t )
2c (c(t ))2 r2
H(c(t ) |x |)
G(x, t|, ) = p
2c (c(t ))2 |x |2

2. To find the 1D Green function, we consider a line source, (x)(t). Without loss of generality, we have taken the

1976
source to be at x = 0, t = 0. We use the 2D Green function and integrate over space and time.

gtt c2 g = (x)(t)
 p 
Z Z Z H c(t ) (x )2 + (y )2
g= p ()( ) d d d
2c (c(t ))2 (x )2 (y )2
 p 
Z H ct x2 + 2
1
g= p d
2c (ct)2 x2 2
Z (ct)2 x2
1 1
g= p dH (ct |x|)
2c (ct)2 x2 (ct)2 x2 2
1
g(x, t|0, 0) = H (ct |x|)
2c
1
g(x, t|, ) = H (c(t ) |x |)
2c

Solution 45.3
1.

Gtt = c2 Gxx , G(x, 0) = 0, Gt (x, 0) = (x )


2 2
Gtt = c G, G(, 0) = 0, Gt (, 0) = F[(x )]
1
G = F[(x )] sin(ct)
c

G = F[(x )]F[H(ct |x|)]
c Z

1
G(x, t) = (x )H(ct ||) d
c 2
1
G(x, t) = H(ct |x |)
2c

1977
2. We can write the solution of
utt = c2 uxx , u(x, 0) = 0, ut (x, 0) = f (x)

in terms of the Green function we found in the previous part.


Z
u= G(x, t|)f () d

We consider c = 1 with the initial condition f (x) = (1 |x|)H(1 |x|).


Z x+t
1
u(x, t) = (1 ||)H(1 ||) d
2 xt

First we consider the case t < 1/2. We will use fact that the solution is symmetric in x.


0, x + t < 1
1 x+t
R
2 R1 (1 ||) d, x t < 1 < x + t



x+t
u(x, t) = 12 xt (1 ||) d, 1 < x t, x + t < 1
1 1
R
(1 ||) d, xt<1<x+t




2 xt
0, 1<xt



0, x + t < 1
1 2
x t < 1 < x + t

4 (1 + t + x)



(1 + x)t 1 < x t, x + t < 0



u(x, t) = 12 (2t t2 x2 ) xt<0<x+t

(1 x)t 0 < x t, x + t < 1




1




4
(1 + t x)2 xt<1<x+t

0, 1<xt

1978
Next we consider the case 1/2 < t < 1.



0, x + t < 1
1 x+t
R
2 R1 (1 ||) d, x t < 1 < x + t



x+t
u(x, t) = 12 xt (1 ||) d, 1 < x t, x + t < 1
1 1
R
(1 ||) d, x t < 1 < x + t




2 xt
0, 1<xt



0, x + t < 1
1 2
1 < x + t < 0

4 (1 + t + x)



1 2
4 (1 t + 2t(1 x) + x(2 x)) x t < 1, 0 < x + t



u(x, t) = 12 (2t t2 x2 ) 1 < x t, x + t < 1
1 2
(1 t + 2t(1 + x) x(2 + x)) x t < 0, 1 < x + t



4
1




4
(1 + t x)2 0<xt<1

0, 1<xt

1979
Finally we consider the case 1 < t.


0, x + t < 1
x+t

1
R
2 R1 (1 ||) d, 1 < x + t < 1



1
u(x, t) = 21 1 (1 ||) d, x t < 1, 1 < x + t
1 1 (1 ||) d, 1 < x t < 1

R


2 xt
0, 1<xt



0, x + t < 1
1 2
1 < x + t < 0



4
(1 + t + x)

1
4 (1 (t + x 2)(t + x)) 0 < x + t < 1



u(x, t) = 21 x t < 1, 1 < x + t
1
(1 (t x 2)(t x)) 1 < x t < 0


4
1




4
(1 + t x)2 0<xt<1

0, 1<xt

Figure 45.1 shows the solution at t = 1/2 and t = 2.

0.5 0.5
0.4 0.4
0.3 0.3
0.2 0.2
0.1 0.1
-2 -1 1 2 -4 -2 2 4

Figure 45.1: The solution at t = 1/2 and t = 2.

Figure 45.2 shows the behavior of the solution in the phase plane. There are lines emanating form x = 1, 0, 1
showing the range of influence of these points.

1980
u=1

u=0 u=0
x

Figure 45.2: The behavior of the solution in the phase plane.

Solution 45.4
We define
L[u] 2 u + a(x) u + h(x)u.
We use the Divergence Theorem to derive a generalized Greens Theorem.
Z Z
uL[v] dx = u(2 v + a v + hv) dx
Z V Z V

uL[v] dx = (u2 v + (uva) v (au) + huv) dx


V V
Z Z Z
2
uL[v] dx = v( u (au) + hu) dx + (uv vu + uva) n dA
V Z V Z V

(uL[v] vL [u]) dx = (uv vu + uva) n dA
V V

We define the adjoint operator L .


L [u] = 2 u (au) + hu

1981
We substitute the solution u and the adjoint Green function G into the generalized Greens Theorem.

Z Z

(G L[u] uL [G ]) dx = (G u uG + vG a) n dA
V Z V

(G q uL [G ]) dx = 0

V

If the adjoint Green function satisfies L [G ] = (x) then we can write u as an integral of the adjoint Green function
and the inhomegeneity.
Z
u() = G (x|)q(x) dx
V

Thus we see that the adjoint Green function problem is the appropriate one to consider. For L[G] = (x ),

Z
u() 6= G(x|)q(x) dx
V

Solution 45.5
1.

2 u = C(x + ) C(x )
C C
u= +
4|x + | 4|x |

1982
2. We take c = D/ and consider the limit  0.
!
D 1 1
u = lim p p
0 4 (x /2)2 + y 2 + z 2 (x + /2)2 + y 2 + z 2
p p
D (x + /2)2 + y 2 + z 2 (x /2)2 + y 2 + z 2
u = lim p
0 4 ((x /2)2 + y 2 + z 2 )((x + /2)2 + y 2 + z 2 )
x x
 
D r + 2r + O(2 ) r 2r + O(2 )
u = lim
0 4 r2 + O()
D xr + O()
u = lim
0 4 r2 + O()
Dx
u=
4r3

3. Let = 0 a/2.

1
2 u = lim (x + ) (x )

0  
1 1 1 1
u = lim
4 0  |x ( 0 + a/2)| |x ( 0 a/2)|

We note that this is the definition of a directional derivative.


 
1 1
u(x) = ~a
4 |x | =0

Solution 45.6
The Green function is  
1 |x |
G(x|) = ln .
2 |||x |

1983
We write this in polar coordinates. Denote x = r e and = e . Let = be the difference in angle between
x and .
p !
1 r2 + 2 2r cos
G(x|) = ln p
2 r2 + 1/2 2(r/) cos
 2
r + 2 2r cos

1
G(x|) = ln 2 2
4 r + 1 2r cos
We solve Laplaces equation with the Green function.
I
u(x) = f () G(x|) n ds
Z 2
u(r, ) = f ()G (r, |1, ) d
0
1 r4 + r(r2 1)(2 + 1) cos
G =
2 (r2 + 2 2r cos )(r2 2 + 1 2r cos )
1 1 r2
G (r, |1, ) =
2 1 + r2 2r cos
2
1 r2
Z
f ()
u(r, ) = 2
d
2 0 1 + r 2r cos( )
Solution 45.7
1.
G = (x )(y )(z )
1 2G
   
1 G 1 G
r2 + 2 sin() + 2 = 3 (r)
r2 r r r sin r sin 2
2. Since the Green function has spherical symmetry, G = G = 0. This reduces the problem to an ordinary
differential equation.  
1 2 G
r = 3 (r)
r2 r r

1984
We find the homogeneous solutions.

2
urr + ur = 0
r
2 ln r
ur = c e = cr2
c1
u= + c2
r

We consider the solution that vanishes at infinity.

c
u=
r

Thus we see that G = c/r. We determine the constant by integrating G over a sphere about the origin, R.
ZZZ
G dx = 1
R
ZZ
G n ds = 1
ZR
Z
Gr ds = 1
R
Z Z 2
c 2
r sin() dd = 1
0 0 r2
4c = 1
1
c=
4
1
G=
4r

1985
3. We write the Laplacian in circular coordinates.

G = (x )(y )
1 2G
 
1 G
r + 2 2 = 2 (r)
r r r r

Since the Green function has circular symmetry, G = 0. This reduces the problem to an ordinary differential
equation.
 
1 G
r = 2 (r)
r r r

We find the homogeneous solutions.

1
urr + ur = 0
r
ur = c e ln r = cr1
u = c1 ln r + c2

There are no solutions that vanishes at infinity. Instead we take the solution that vanishes at r = 1.

u = c ln r

1986
Thus we see that G = c ln r. We determine the constant by integrating G over a ball about the origin, R.

ZZ
G dx = 1
R
Z
G n ds = 1
R
Z
Gr ds = 1
Z R
2
c
r d = 1
0 r
2c = 1
1
G= ln r
2

1987
4.
Z Z Z
1
u= ()() d d d
4(r )
Z Z Z
1 ()()
u= p d d d
4 (x )2 + (y )2 + (z )2

Z
1 1
u= p d
4 x2 + y 2 + (z )2
Z
1 1
u= p d
4 r2 + 2
Z
1 r
ur = d
4 (r + 2 )3/2
2

1 2
ur =
4 r
1
u= ln r
2
Solution 45.8
1. We take the Laplace transform of the differential equation and the boundary conditions in x.

Gt Gxx = (x )(t )
sG Gxx = (x )
s 1
Gxx G = (x ), G(0, t) = G(L, t) = 0

Now we have an ordinary differential equation Green function problem. We find homogeneous solutions which
respectively satisfy the left and right boundary conditions and compute their Wronskian.
r  r 
s s
y1 = sinh x , y2 = sinh (L x)

1988
ps  ps 
sinh

x sinh p
(L x)
W = p s p s
 ps s


cosh
x cosh
(L x)
r  r  r  r  r 
s s s s s
= 2 sinh x cosh (L x) + cosh x sinh (L x)

r r 
s s
= 2 sinh L

We write the Green function in terms of the homogeneous solutions of the Wronskian.
r  r 
1 1 s s
G = p s  sinh x< sinh (L x> )
2 s sinh
p

L
ps  ps 
sinh
x < sinh (L x > )
G = p s 
2 s sinh
L
ps  ps 
cosh
(L x> + x< ) cosh
(L x> x< )
G = ps 
2 s sinh
L

2. We expand 1/ sinh(x) in a series.

1 2
= x
sinh(x) e ex
2 ex
=
1 e2x

X
x
= 2e e2nx
n=0

X
=2 e(2n+1)x
n=0

1989
We use the expansion of the hyperbolic cosecant in our expression for the Green function.

e s/(Lx> +x< ) + e s/(Lx> +x< ) e s/(Lx> x< ) e s/(Lx> x< )
G = ps 
4 s sinh
L

1  /s(Lx> +x< )
G = e + e /s(Lx> +x< )
2 s

X
/s(Lx> x< ) /s(Lx> x< ) (2n+1) s/L
e e e
n=0


1 X X
G = e s/(x> +x< 2nL) + e s/(x> x< 2(n+1)L)
2 s n=0 n=0

!
X X
s/(x> x< 2nL)
e e s/(x> +x< 2(n+1)L)
n=0 n=0

1
1 X
s/(x> +x< 2nL)
X
G = e + e s/(x> x< +2nL)
2 s n=0 n=
1
!
X X
s/(x> x< 2nL)
e e s/(x> +x< +2nL)
n=0 n=


!
1 X X
G = e s/|x< x> 2nL|
e s/|x< +x> 2nL|
2 s n= n=

!
1 X X
G = e s/|x2nL|
e s/|x+2nL|
2 s n= n=

1990
3. We take the inverse Laplace transform to find the Green function for the diffusion equation.


!
1 X
(x2nL)2 /(4t)
X
(x+2nL)2 /(4t)
G= e e
2 t n= n=

X
X
G= f (x 2nL, t) f (x + 2nL, t)
n= n=

On the interval (L . . . L), there is a real source at x = and a negative image source at x = . This pattern
is repeated periodically.

The above formula is useful when approximating the solution for small time, t  1. For such small t, the terms
decay very quickly away from n = 0. A small number of terms could be used for an accurate approximation.

The alternate formula is useful when approximating the solution for large time, t  1. For such large t, the terms
in the sine series decay exponentially Again, a small number of terms could be used for an accurate approximation.

Solution 45.9
1. We take the Fourier cosine transform of the differential equation.

Gt Gxx = (x )(t )
 
2 1
Gt G Gx (0, t) = Fc [(x )](t )

Gt + 2 G = Fc [(x )](t )
2
G = Fc [(x )] e (t ) H(t )
r 
x2 /(4(t ))
G = Fc [(x )]Fc e H(t )
(t )

1991
We do the inversion with the convolution theorem.
Z r
1 
|x|2 /(4(t )) (x+)2 /(4(t ))

G= ( ) e +e dH(t )
2 0 (t )
1 
(x)2 /(4(t )) (x+)2 /(4(t ))

G(x, t; , ) = p e + e H(t )
4(t )

2. The fundamental solution on the infinite domain is


1 2 /(4(t ))
F (x, t; , ) = p e(x) H(t ).
4(t )
We see that the Green function on the semi-infinite domain that we found above is a sum of fundamental solutions.

G(x, t; , ) = F (x, t; , ) + F (x, t; , )

Now we solve the inhomogeneous problem.


Z tZ Z
u(x, t) = G(x, t; , )q(, ) d d + G(x, t; , 0)f () d
0 0 0

Z tZ
1 1  (x)2 /(4(t )) 2

u(x, t) = e + e(x+) /(4(t )) q(, ) d d
4 0 0 t
Z 
1 2 2

+ e(x) /(4t) + e(x+) /(4t) f () d
4t 0
Solution 45.10
1. We integrate the heat equation from t = 0 to t = 0+ to determine an initial condition.

ut = uxx + (x )(t)
u(x, 0+ ) u(x, 0 ) = (x )

1992
Now we have an initial value problem with no forcing.

ut = uxx , for t > 0, u(x, 0) = (x )

2. We take the Laplace transform of the initial value problem.


su u(x, 0) = uxx
s 1
uxx u = (x ), u(, s) = 0

The solutions that satisfy the left and right boundary conditions are, respectively,

s/x s/x
u1 = e , u2 = e
We compute the Wronskian of these solutions and then write the solution for u.

s/x
r
e s/x e s

W = p = 2


s/x
p s/x
s/ e s/ e

1 e s/x< e s/x>
u =
2 s
p

1
u = e s/|x|
2 s

3. In Exercise 31.16, we showed that


ea/t
r 
1 2as
L e = .
s t
We use this result to do the inverse Laplace transform.
1 2
u(x, t) = e(x) /(4t)
2 t

1993
Solution 45.11

Gtt c2 Gxx = (x )(t ),


G(x, t; , ) = 0 for t < .
We take the Fourier transform in x.
Gtt + c2 2 G = F[(x )](t ), G(, 0; , ) = Gt (, 0; , ) = 0
Now we have an ordinary differential equation Green function problem for G. We have written the causality condition,
the Green function is zero for t < , in terms of initial conditions. The homogeneous solutions of the ordinary differential
equation are
{cos(ct), sin(ct)}.
It will be handy to use the fundamental set of solutions at t = :
 
1
cos(c(t )), sin(c(t )) .
c
The continuity and jump conditions are
G(, 0; , + ) = 0, Gt (, 0; , + ) = F[(x )]
We write the solution for G and invert using the convolution theorem.
1
G = F[(x )]H(t ) sin(c(t ))
hc
i
G = H(t )F[(x )]F H(c(t ) |x|)
Z c
1
G = H(t ) (y )H(c(t ) |x y|) dy
c 2
1
G = H(t )H(c(t ) |x |)
2c
1
G = H(c(t ) |x |)
2c

1994
The Green function for = = 0 and c = 1 is plotted in Figure 45.3 on the domain x (1..1), t (0..1). The
1
Green function is a displacement of height 2c that propagates out from the point x = in both directions with speed c.
The Green function shows the range of influence of a disturbance at the point x = and time t = . The disturbance
influences the solution for all ct < x < + ct and t > .

1
0.8
0.6
t
0.4

0.2

0.4

0.2
-1 -0.5 0 0.5 0
1
x

Figure 45.3: Green function for the wave equation.

Now we solve the wave equation with a source.

utt c2 uxx = q(x, t), u(x, 0) = ut (x, 0) = 0


Z Z
u= G(x, t|, t)q(, ) d d
0
Z Z
1
u= H(c(t ) |x |)q(, ) d d
0 2c

1 t x+c(t )
Z Z
u= q(, ) d d
2c 0 xc(t )

1995
Solution 45.12
1. We expand the Green function in eigenfunctions in x.
 
X (2n 1)x
G(x; xi) = an (y) sin
n=1
2L

We substitute the expansion into the differential equation.


r  
2
X 2 (2n 1)x
an (y) sin = (x )(y )
n=1
L 2L

 2 !r  
X (2n 1) 2 (2n 1)x
a00n (y) an (y) sin
n=1
2L L 2L
r  r  
X 2 (2n 1) 2 (2n 1)x
= (y ) sin sin
n=1
L 2L L 2L
 2 r  
(2n 1) 2 (2n 1)
a00n (y) an (y) = sin (y )
2L L 2L
From the boundary conditions at y = 0 and y = H, we obtain boundary conditions for the an (y).
a0n (0) = a0n (H) = 0.
The solutions that satisfy the left and right boundary conditions are
   
(2n 1)y (2n 1)(H y)
an1 = cosh , an2 = cosh .
2L 2L
The Wronskian of these solutions is
 
(2n 1) (2n 1)
W = sinh .
2L 2

1996
Thus the solution for an (y) is
   
 cosh (2n1)y< (2n1)(Hy> )
cosh
r 
2 (2n 1) 2L 2L
an (y) = sin  
L 2L (2n1) sinh (2n1)
2L 2

   
2 2L (2n 1) (2n 1)y<
an (y) = csch cosh
(2n 1) 2 2L
   
(2n 1)(H y> ) (2n 1)
cosh sin .
2L 2L

This determines the Green function.


   
2 2L X 1 (2n 1) (2n 1)y<
G(x; xi) = csch cosh
n=1 2n 1 2 2L
     
(2n 1)(H y> ) (2n 1) (2n 1)x
cosh sin sin
2L 2L 2L

2. We seek a solution of the form



X 2  mx   ny 
G(x; xi) = amn (z) sin sin .
m=1 LH L H
n=1

We substitute this into the differential equation.



2
X 2  mx   ny 
amn (z) sin sin = (x )(y )(z )
m=1 LH L H
n=1

1997
   
X m 2  n 2 2  mx   ny 
a00mn (z) + amn (z) sin sin
m=1
L H LH L H
n=1
   
X 2 m n 2  mx   ny 
= (z ) sin sin sin sin
m=1 LH L H LH L H
n=1
     
00 m 2  n 2 2 m n
amn (z) + amn (z) = sin sin (z )
L H LH L H
From the boundary conditions on G, we obtain boundary conditions for the amn .

amn (0) = amn (W ) = 0

The solutions that satisfy the left and right boundary conditions are
r  ! r  !
m 2  n  2 m 2  n  2
amn1 = sinh + z , amn2 = sinh + (W z) .
L H L H

The Wronskian of these solutions is


r r !
m 2  n 2 m 2  n  2
W = + sinh + W .
L H L H

Thus the solution for amn (z) is


   
2 m n
amn (z) = sin sin
LH L H
q  q 
m 2 n 2 m 2 n 2
   
sinh +L
z< sinh
H
+ (W z> ) L H
q  q 
m 2 n 2 m 2 n 2
  
L
+ H sinh L
+ H W

1998
   
2 m n
amn (z) = csch (mn W ) sin sin
mn LH L H
sinh (mn z< ) sinh (mn (W z> )) ,
where r 
m 2  n 2
mn = + .
L H
This determines the Green function.
 
4 X 1 m  mx 
G(x; xi) = csch (mn W ) sin sin
LH m=1 mn L L
n=1
 
n  ny 
sin sin sinh (mn z< ) sinh (mn (W z> ))
H H
3. First we write the problem in circular coordinates.
2 G = (x xi)
1 1 1
Grr + Gr + 2 G = (r )( ),
r r r
G(r, 0; , ) = G(r, ; , ) = G(0, ; , ) = G(a, ; , ) = 0
Because the Green function vanishes at = 0 and = we expand it in a series of the form
X
G= gn (r) sin(n).
n=1

We substitute the series into the differential equation.



n2

X
00 1 0 1 X 2
gn (r) + gn (r) 2 gn (r) sin(n) = (r ) sin(n) sin(n)
n=1
r r r n=1

1 n2 2
gn00 (r) + gn0 (r) 2 gn (r) = sin(n)(r )
r r r

1999
From the boundary conditions on G, we obtain boundary conditions for the gn .
gn (0) = gn (a) = 0
The solutions that satisfy the left and right boundary conditions are
 r n  a n
n
gn1 = r , gn2 = .
a r
The Wronskian of these solutions is
2nan
W = .
r
Thus the solution for gn (r) is
  n 
r> n a
n

2 r < a
r>
gn (r) = sin(n) 2nan

   n 
1  r<  n r> n a
gn (r) = sin(n) .
n a a r>
This determines the solution.
  n 
X 1  r< n  r> n a
G= sin(n) sin(n)
n=1
n a a r >

4. First we write the problem in circular coordinates.


1 1 1
Grr + Gr + 2 G = (r )( ),
r r r
G(r, 0; , ) = G(r, /2; , ) = G(0, ; , ) = Gr (a, ; , ) = 0
Because the Green function vanishes at = 0 and = /2 we expand it in a series of the form

X
G= gn (r) sin(2n).
n=1

2000
We substitute the series into the differential equation.

4n2

X
00 1 0 1 X 4
gn (r) + gn (r) 2 gn (r) sin(2n) = (r ) sin(2n) sin(2n)
n=1
r r r n=1

1 0 4n2 4
gn00 (r) + gn (r) 2 gn (r) = sin(2n)(r )
r r r
From the boundary conditions on G, we obtain boundary conditions for the gn .

gn (0) = gn0 (a) = 0

The solutions that satisfy the left and right boundary conditions are
 r 2n  a 2n
2n
gn1 = r , gn2 = + .
a r
The Wronskian of these solutions is
4na2n
W = .
r
Thus the solution for gn (r) is
  2n 
r> 2n a
2n

r< a
+ r>
4
gn (r) = sin(2n) 2n
4na
 2n !
1  r 2n
<
 r 2n
> a
gn (r) = sin(2n) +
n a a r>

This determines the solution.


 2n !
X 1  r< 2n  r> 2n a
G= + sin(2n) sin(2n)
n=1
n a a r >

2001
Solution 45.13
1. The set
  
(2m 1)x
{Xn } = sin
2L m=1

are eigenfunctions of 2 and satisfy the boundary conditions Xn (0) = Xn0 (L) = 0. The set

n  ny o
{Yn } = cos
H n=0

are eigenfunctions of 2 and satisfy the boundary conditions Yn0 (0) = Yn0 (H) = 0. The set

 
(2m 1)x
  ny 
sin cos
2L H m=1,n=0

are eigenfunctions of 2 and satisfy the boundary conditions of this problem. We expand the Green function in
a series of these eigenfunctions.

r    
X 2 (2m 1)x X 2 (2m 1)x  ny 
G= gm0 sin + gmn sin cos
m=1
LH 2L m=1 LH 2L H
n=1

We substitute the series into the Green function differential equation.

G = (x )(y )

2002
 2 r  
X (2m 1) 2 (2m 1)x
gm0 sin
m=1
2L LH 2L
 2  !  
X (2m 1) ny 2 2 (2m 1)x  ny 
gmn + sin cos
m=1
2L H LH 2L H
n=1
r  r  
X 2 (2m 1) 2 (2m 1)x
= sin sin
m=1
LH 2L LH 2L
     
X 2 (2m 1) n 2 (2m 1)x  ny 
+ sin cos sin cos
m=1 LH 2L H LH 2L H
n=1

We equate terms and solve for the coefficients gmn .


r  2  
2 2L (2m 1)
gm0 = sin
LH (2m 1) 2L
   
2 1 (2m 1) n
gmn =    sin cos
LH 2 2m1 2 + n 2 2L H

2L H

This determines the Green function.

2. Note that (r   )
8 kx  my   nz 
sin , sin , sin : k, m, n Z+
LHW L H W
is orthonormal and complete on (0 . . . L) (0 . . . H) (0 . . . W ). The functions are eigenfunctions of 2 . We
expand the Green function in a series of these eigenfunctions.
r  
X 8 kx  my   nz 
G= gkmn sin sin sin
k,m,n=1
LHW L H W

2003
We substitute the series into the Green function differential equation.

G = (x )(y )(z )

 2 !r  
X k  m 2  n 2 8 kx  my   nz 
gkmn + + sin sin sin
k,m,n=1
L H W LHW L H W
r      
X 8 k m n
= sin sin sin
k,m,n=1
LHW L H W
r  
8 kx  my   nz 
sin sin sin
LHW L H W
We equate terms and solve for the coefficients gkmn .
q
8 k m n
  
LHW
sin L
sin H
sin W
gkmn =  
2 2 2

2 Lk + H m
+ Wn
 

This determines the Green function.


3. The Green function problem is
1 1 1
G Grr + Gr + 2 G = (r )( ).
r r r
We seek a set of functions {n ()Rnm (r)} which are orthogonal and complete on (0 . . . a) (0 . . . ) and which
2
are eigenfunctions of the laplacian. For the n we choose eigenfunctions of 2.

00 = 2 , (0) = () = 0
n = n, n = sin(n), n Z+

2004
Now we look for eigenfunctions of the laplacian.
1 1
(Rn )rr + (Rn )r + 2 (Rn ) = 2 Rn
r r
2
1 n
R00 n + R0 n 2 Rn = 2 Rn
 r 2
 r
1 n
R00 + R0 + 2 2 R = 0, R(0) = R(a) = 0
r r
The general solution for R is
R = c1 Jn (r) + c2 Yn (r).
the solution that satisfies the left boundary condition is R = cJn (r). We use the right boundary condition to
determine the eigenvalues.  
jn,m jn,m r
m = , Rnm = Jn , m, n Z+
a a
here jn,m is the mth root of Jn .
Note that    
jn,m r +
sin(n)Jn : m, n Z
a
is orthogonal and complete on (r, ) (0 . . . a) (0 . . . ). We use the identities
Z Z 1
2 1 2
sin (n) d = , rJn2 (jn,m r) dr = Jn+1 (jn,m )
0 2 0 2
to make the functions orthonormal.
   
2 jn,m r +
sin(n)Jn : m, n Z
a|Jn+1 (jn,m )| a
We expand the Green function in a series of these eigenfunctions.
 
X 2 jn,m r
G= gnm Jn sin(n)
n,m=1
a|J n+1 (j n,m )| a

2005
We substitute the series into the Green function differential equation.

1 1 1
Grr + Gr + 2 G = (r )( )
r r r

 2  
X jn,m 2 jn,m r
gnm Jn sin(n)
n,m=1
a a|J n+1 (j n,m )| a
   
X 2 jn,m 2 jn,m r
= Jn sin(n) Jn sin(n)
n,m=1
a|J n+1 (j n,m )| a a|J n+1 (j n,m )| a

We equate terms and solve for the coefficients gmn .


 2  
a 2 jn,m
gnm = Jn sin(n)
jn,m a|Jn+1 (jn,m )| a

This determines the green function.

4. The Green function problem is

1 1 1
G Grr + Gr + 2 G = (r )( ).
r r r

We seek a set of functions {n ()Rnm (r)} which are orthogonal and complete on (0 . . . a) (0 . . . /2) and
2
which are eigenfunctions of the laplacian. For the n we choose eigenfunctions of 2.

00 = 2 , (0) = (/2) = 0
n = 2n, n = sin(2n), n Z+

2006
Now we look for eigenfunctions of the laplacian.
1 1
(Rn )rr + (Rn )r + 2 (Rn ) = 2 Rn
r r
2
1 (2n)
R00 n + R0 n 2 Rn = 2 Rn
 r 2
r
1 (2n)
R00 + R0 + 2 2 R = 0, R(0) = R(a) = 0
r r
The general solution for R is
R = c1 J2n (r) + c2 Y2n (r).
the solution that satisfies the left boundary condition is R = cJ2n (r). We use the right boundary condition to
determine the eigenvalues.
j 0 2n,m
 0
j 2n,m r

m = , Rnm = J2n , m, n Z+
a a
here j 0 n,m is the mth root of J 0 n .
Note that   0
j 2n,m r
 
0 +
sin(2n)J 2n : m, n Z
a
is orthogonal and complete on (r, ) (0 . . . a) (0 . . . /2). We use the identities
Z

sin(m) sin(n) d = mn ,
0 2
02
j ,n 2
Z 1
0 0 0
2
rJ (j ,m r)J (j ,n r) dr = J (j ,n ) mn
0 2j 0 2,n
to make the functions orthonormal.

0  0
2j 2n,m j 2n,m r

+
sin(2n)J : m, n Z
a j 0 2
q 2n
2 0 a
2n,m 4n |J2n (j 2n,m )|

2007
We expand the Green function in a series of these eigenfunctions.

2j 0 2n,m j 0 2n,m r
X  
G= gnm q J2n sin(2n)
n,m=1 a j 0 22n,m 4n2 |J2n (j 0 2n,m )| a

We substitute the series into the Green function differential equation.


1 1 1
Grr + Gr + 2 G = (r )( )
r r r

 0
j 2n,m 2 2j 0 2n,m j 0 2n,m r
X   
gnm q J2n sin(2n)
a a j 02 2 0 a
n,m=1 2n,m 4n |J2n (j 2n,m )|

2j 0 2n,m j 0 2n,m
X  
= q J2n sin(2n)
n,m=1 a j 0 22n,m 4n2 |J2n (j 0 2n,m )| a

2j 0 2n,m j 0 2n,m r
 
q J2n sin(2n)
a j 0 22n,m 4n2 |J (j 0 a
2n 2n,m )|

We equate terms and solve for the coefficients gmn .


2
2j 0 2n,m
  0
j 2n,m

a
gnm = 0 q 02 J2n sin(2n)
j 2n,m 2 0
a j 2n,m 4n |J2n (j 2n,m )| a

This determines the green function.


Solution 45.14
We start with the equation
2 G = (x )(y ).

2008
We do an odd reflection across the y axis so that G(0, y; , ) = 0.

2 G = (x )(y ) (x + )(y )

Then we do an even reflection across the x axis so that Gy (x, 0; , ) = 0.

2 G = (x )(y ) (x + )(y ) + (x )(y + ) (x + )(y + )

We solve this problem using the infinite space Green function.

1 1
ln (x )2 + (y )2 ln (x + )2 + (y )2
 
G=
4 4
1 1
ln (x )2 + (y + )2 ln (x + )2 + (y + )2
 
+
4 4

((x )2 + (y )2 ) ((x )2 + (y + )2 )
 
1
G= ln
4 ((x + )2 + (y )2 ) ((x + )2 + (y + )2 )
Now we solve the boundary value problem.
Z   Z
G u(x, y)
u(, ) = u(x, y) G dS + Gu dV
S n n V
Z 0 Z
u(, ) = u(0, y)(Gx (0, y; , )) dy + G(x, 0; , )(uy (x, 0)) dx
0
Z Z
u(, ) = g(y)Gx (0, y; , ) dy + G(x, 0; , )h(x) dx
0 0

Z 
(x )2 + 2
Z   
1 1 1
u(, ) = + g(y) dy + ln h(x) dx
0 2 + (y )2 2 + (y + )2 2 0 (x + )2 + 2
x
Z 
(x )2 + y 2
Z   
1 1 1
u(x, y) = + g() d + ln h() d
0 x2 + (y )2 x2 + (y + )2 2 0 (x + )2 + y 2

2009
Solution 45.15
First we find the infinite space Green function.

Gtt c2 Gxx = (x )(t ), G = Gt = 0 for t <

We solve this problem with the Fourier transform.

Gtt + c2 2 G = F[(x )](t )


1
G = F[(x )]H(t ) sin(c(t ))
hc
i
G = H(t )F[(x )]F H(c(t ) |x|)
Z c
1
G = H(t ) (y )H(c(t ) |x y|) dy
c 2
1
G = H(t )H(c(t ) |x |)
2c
1
G = H(c(t ) |x |)
2c

1. So that the Green function vanishes at x = 0 we do an odd reflection about that point.

Gtt c2 Gxx = (x )(t ) (x + )(t )


1 1
G = H(c(t ) |x |) H(c(t ) |x + |)
2c 2c

2. Note that the Green function satisfies the symmetry relation

G(x, t; , ) = G(, ; x, t).

This implies that


Gxx = G , Gtt = G .

2010
We write the Green function problem and the inhomogeneous differential equation for u in terms of and .

G c2 G = (x )(t ) (45.4)
u c2 u = Q(, ) (45.5)

We take the difference of u times Equation 45.4 and G times Equation 45.5 and integrate this over the domain
(0, ) (0, t+ ).
Z t+ Z Z t+ Z
uG u G c2 (uG u G) d d

(u(x )(t ) GQ) d d =
0 0 0 0
Z t+ Z Z t+ Z  
2
u(x, t) = GQ d d + (uG u G) c (uG u G) d d
0 0 0 0
Z t+ Z Z Z t+
u(x, t) = GQ d d + t+
[uG u G]0 d c 2
[uG u G]
0 d
0 0 0 0
Z t+ Z Z Z t+
2
u(x, t) = GQ d d [uG u G] =0 d + c [uG ]=0 d
0 0 0 0

We consider the case Q(x, t) = f (x) = g(x) = 0.


Z t+
2
u(x, t) = c h( )G (x, t; 0, ) d
0

We calculate G .
1
G= (H(c(t ) |x |) H(c(t ) |x + |))
2c
1
G = ((c(t ) |x |)(1) sign(x )(1) (c(t ) |x + |)(1) sign(x + ))
2c
1
G (x, t; 0, ) = (c(t ) |x|) sign(x)
c

2011
We are interested in x > 0.

1
G (x, t; 0, ) = (c(t ) x)
c

Now we can calculate the solution u.


Z t+
2 1
u(x, t) = c h( ) (c(t ) x) d
0 c
Z t+  x
u(x, t) = h( ) (t ) d
0 c
 x
u(x, t) = h t
c

3. The boundary condition influences the solution u(x1 , t1 ) only at the point t = t1 x1 /c. The contribution from
the boundary condition u(0, t) = h(t) is a wave moving to the right with speed c.

Solution 45.16

gtt c2 gxx = 0, g(x, 0; , ) = (x ), gt (x, 0; , ) = 0


gtt + c2 2 gxx = 0, g(x, 0; , ) = F[(x )], gt (x, 0; , ) = 0
g = F[(x )] cos(ct)
g = F[(x )]F[((x + ct) + (x ct))]
Z
1
g= ( )((x + ct) + (x ct)) d
2
1
g(x, t; ) = ((x + ct) + (x ct))
2

2012
tt c2 xx = 0, (x, 0; , ) = 0, t (x, 0; , ) = (x )
tt + c2 2 xx = 0, (x, 0; , ) = 0, t (x, 0; , ) = F[(x )]
1
= F[(x )] sin(ct)
h c i
= F[(x )]F (H(x + ct) + H(x ct))
Z c
1
= ( ) (H(x + ct) + H(x ct)) d
2 c
1
(x, t; ) = (H(x + ct) + H(x ct))
2c
Solution 45.17
Z Z Z Z
u(x, t) = G(x, t; , )f (, ) d d + g(x, t; )p() d + (x, t; )q() d
0

Z Z
1
u(x, t) = H(t )(H(x + c(t )) H(x c(t )))f (, ) d d
2c 0
Z
1
Z
1
+ ((x + ct) + (x ct))p() d + (H(x + ct) + H(x ct))q() d
2 2c

Z tZ
1
u(x, t) = (H(x + c(t )) H(x c(t )))f (, ) d d
2c 0
Z x+ct
1 1
+ (p(x + ct) + p(x ct)) + q() d
2 2c xct

Z tZ x+c(t ) Z x+ct
1 1 1
u(x, t) = f (, ) d d + (p(x + ct) + p(x ct)) + q() d
2c 0 xc(t ) 2 2c xct

2013
This solution demonstrates the domain of dependence of the solution. The first term is an integral over the triangle
domain {(, ) : 0 < < t, x c < < x + c }. The second term involves only the points (x ct, 0). The third
term is an integral on the line segment {(, 0) : x ct < < x + ct}. In totallity, this is just the triangle domain. This
is shown graphically in Figure 45.4.

x,t

Domain of
Dependence

x-ct x+ct

Figure 45.4: Domain of dependence for the wave equation.

Solution 45.18
Single Sum Representation. First we find the eigenfunctions of the homogeneous problem u k 2 u = 0. We
substitute the separation of variables, u(x, y) = X(x)Y (y) into the partial differential equation.

X 00 Y + XY 00 k 2 XY = 0
X 00 2 Y 00
=k = 2
X Y
We have the regular Sturm-Liouville eigenvalue problem,

X 00 = 2 X, X(0) = X(a) = 0,

2014
which has the solutions,
n  nx 
n = , Xn = sin , n N.
a a
We expand the solution u in a series of these eigenfunctions.

X  nx 
G(x, y; , ) = cn (y) sin
n=1
a

We substitute this series into the partial differential equation to find equations for the cn (y).
  
X n 2 00 2
 nx 
cn (y) + cn (y) k cn (y) sin = (x )(y )
n=1
a a

The series expansion of the right side is,



X  nx 
(x )(y ) = dn (y) sin
n=1
a
Z a
2  nx 
dn (y) = (x )(y ) sin dx
a 0 a
 
2 n
dn (y) = sin (y ).
a a

The the equations for the cn (y) are


  n 2   
2 n
c00n (y) k +2
cn (y) = sin (y ), cn (0) = cn (b) = 0.
a a a
p
The homogeneous solutions are {cosh(n y), sinh(n y)}, where n = k 2 (n/a)2 . The solutions that satisfy the
boundary conditions at y = 0 and y = b are, sinh(n y) and sinh(n (y b)), respectively. The Wronskian of these

2015
solutions is,

sinh(n y) sinh(n (y b))
W (y) =

n cosh(n y) n cosh(n (y b))
= n (sinh(n y) cosh(n (y b)) sinh(n (y b)) cosh(n y))
= n sinh(n b).
The solution for cn (y) is  
2 n sinh(n y< ) sinh(n (y> b))
cn (y) = sin .
a a n sinh(n b)
The Green function for the partial differential equation is
 
2 X sinh(n y< ) sinh(n (y> b))  nx  n
G(x, y; , ) = sin sin .
a n=1 n sinh(n b) a a

Solution 45.19
We take the Fourier cosine transform in x of the partial differential equation and the boundary condition along y = 0.
Gxx + Gyy k 2 G = (x )(y )
1 1
2 G(, y) Gx (0, y) + Gyy (, y) k 2 G(, y) = cos()(y )

1
Gyy (, y) (k 2 + 2 )G(, y) == cos()(y ), G(, 0) = 0

Then we take the Fourier sine transform in y.


1
2 G(, ) + G(, 0) (k 2 + 2 )G(, ) = 2 cos() sin()

cos() sin()
G = 2 2
(k + 2 + 2 )

2016
We take two inverse transforms to find the solution. For one integral representation of the Green function we take the
inverse sine transform followed by the inverse cosine transform.

sin() 1
G = cos()
(k + 2 + 2 )
2


 
1 k2 +2 y
G = cos()Fs [(y )]Fc e
k 2 + 2
1
Z
1     
G(, y) = cos() (z ) exp k 2 + 2 |y z| exp k 2 + 2 (y + z) dz
2 0 k 2 + 2
cos()     
G(, y) = exp k 2 + 2 |y | exp k 2 + 2 (y + )
2 k 2 + 2
1 cos() 
Z    
G(x, y; , ) = 2 2
exp k + |y | exp k + (y + )2 2 d
0 k 2 + 2

For another integral representation of the Green function, we take the inverse cosine transform followed by the inverse
sine transform.

cos() 1
G(, ) = sin()
(k + 2 + 2 )
2


" #
1 2 2
G(, ) = sin()Fc [(x )]Fc p e k + x
2
k + 2

1
Z
1  
2 2 2 2
G(x, ) = sin() (z ) p e k + |xz| + e k + (x+z) dz
2 0 k2 + 2
1 1  
2 2 2 2
G(x, ) = sin() p e k + |x| + e k + (x+)
2 k 2 + 2
1 sin(y) sin()  k2 + 2 |x|
Z 
k2 + 2 (x+)
G(x, y; , ) = p e +e d
0 k2 + 2

2017
Solution 45.20
The problem is:

1 1 (r )( )
Grr + Gr + 2 G = , 0 < r < , 0 < < ,
r r r
G(r, 0, , ) = G(r, , , ) = 0,
G(0, , , ) = 0
G(r, , , ) 0 as r .

Let w = r ei and z = x + iy. We use the conformal mapping, z = w/ to map the sector to the upper half z plane.
The problem is (x, y) space is

Gxx + Gyy = (x )(y ), < x < , 0 < y < ,


G(x, 0, , ) = 0,
G(x, y, , ) 0 as x, y .

We will solve this problem with the method of images. Note that the solution of,

Gxx + Gyy = (x )(y ) (x )(y + ), < x < , < y < ,


G(x, y, , ) 0 as x, y ,

satisfies the condition, G(x, 0, , ) = 0. Since the infinite space Green function for the Laplacian in two dimensions is
1
ln (x )2 + (y )2 ,

4
the solution of this problem is,
1 1
ln (x )2 + (y )2 ln (x )2 + (y + )2
 
G(x, y, , ) =
4   4
1 (x )2 + (y )2
= ln .
4 (x )2 + (y + )2

2018
Now we solve for x and y in the conformal mapping.

z = w/ = (r ei )/
x + iy = r/ (cos(/) + i sin(/))
x = r/ cos(/), y = r/ sin(/)

We substitute these expressions into G(x, y, , ) to obtain G(r, , , ).

(r/ cos(/) / cos(/))2 + (r/ sin(/) / sin(/))2


 
1
G(r, , , ) = ln
4 (r/ cos(/) / cos(/))2 + (r/ sin(/) + / sin(/))2
 2/
+ 2/ 2r/ / cos(( )/)

1 r
= ln 2/
4 r + 2/ 2r/ / cos(( + )/)
(r/)/ /2 + (/r)/ /2 cos(( )/)
 
1
= ln
4 (r/)/ /2 + (/r)/ /2 cos(( + )/)
 ln(r/)/
/2 + e ln(/r)/ /2 cos(( )/)

1 e
= ln ln(r/)/
4 e /2 + e ln(/r)/ /2 cos(( + )/)
 
/ r
1 cosh ln
cos(( )/)
G(r, , , ) = ln  
4 cosh / r
cos(( + )/)
ln

Now recall that the solution of


u = f (x),
subject to the boundary condition,
u(x) = g(x),
is Z Z I
u(x) = f (xi)G(x; xi) dA + g(xi) G(x; xi) n ds .

2019
The normal directions along the lower and upper edges of the sector are and , respectively. The gradient in polar
coordinates is


= + .

We only need to compute the component of the gradient of G. This is

1 sin(( )/) sin(( )/)


G=    +    
4 cosh ln r cos(( )/) 4 cosh ln r cos(( + )/)

Along = 0, this is

1 sin(/)
G (r, , , 0) =    .
2 cosh ln r cos(/)

Along = , this is

1 sin(/)
G (r, , , ) =    .
2 cosh
ln r
+ cos(/)

2020
The solution of our problem is
Z c Z
sin(/) sin(/)
u(r, ) =     d +     d
2 cosh ln r + cos(/) c 2 cosh ln r cos(/)
Z
sin(/) sin(/)
u(r, ) =    +     d
c 2 cosh ln r cos(/) 2 cosh ln r + cos(/)
   Z
1 1
u(r, ) = sin cos     d
c 2 r
cosh ln cos 2

   Z
1 1
u(r, ) = sin cos 2 x
 dx
cos2

ln(c/r) cosh
   Z
2 1
u(r, ) = sin cos 2x
 2
 dx
ln(c/r) cosh
cos

Solution 45.21
First consider the Green function for
ut uxx = 0, u(x, 0) = f (x).
The differential equation and initial condition is

Gt = Gxx , G(x, 0; ) = (x ).

The Green function is a solution of the homogeneous heat equation for the initial condition of a unit amount of heat
concentrated at the point x = . You can verify that the Green function is a solution of the heat equation for t > 0
and that it has the property: Z
G(x, t; ) dx = 1, for t > 0.

This property demonstrates that the total amount of heat is the constant 1. At time t = 0 the heat is concentrated at
the point x = . As time increases, the heat diffuses out from this point.

2021
The solution for u(x, t) is the linear combination of the Green functions that satisfies the initial condition u(x, 0) =
f (x). This linear combination is
Z
u(x, t) = G(x, t; )f () d.

G(x, t; 1) and G(x, t; 1) are plotted in Figure 45.5 for the domain t [1/100..1/4], x [2..2] and = 1.

2 2
1
1
0
0

0.1
-1

0.2
-2

Figure 45.5: G(x, t; 1) and G(x, t; 1)

Now we consider the problem

ut = uxx , u(x, 0) = f (x) for x > 0, u(0, t) = 0.

2022
Note that the solution of

Gt = Gxx , x > 0, t > 0,


G(x, 0; ) = (x ) (x + ),

satisfies the boundary condition G(0, t; ) = 0. We write the solution as the difference of infinite space Green functions.
1 2 1 2
G(x, t; ) = e(x) /(4t) e(x+) /(4t)
4t 4t
1  (x)2 /(4t) 2

= e e(x+) /(4t)
4t
 
1 (x2 + 2 )/(4t) x
G(x, t; ) = e sinh
4t 2t
Next we consider the problem

ut = uxx , u(x, 0) = f (x) for x > 0, ux (0, t) = 0.

Note that the solution of

Gt = Gxx , x > 0, t > 0,


G(x, 0; ) = (x ) + (x + ),

satisfies the boundary condition Gx (0, t; ) = 0. We write the solution as the sum of infinite space Green functions.
1 2 1 2
G(x, t; ) = e(x) /(4t) + e(x+) /(4t)
4t 4t
 
1 (x2 + 2 )/(4t) x
G(x, t; ) = e cosh
4t 2t

The Green functions for the two boundary conditions are shown in Figure 45.6.

2023
2 1 2 1
1 0.8 1 0.8
0 0.6 0 0.6
0.05 0.4 0.05 0.4
0.1 0.2 0.1 0.2
0.15 0.15
0.2 0 0.2 0
0.25 0.25

Figure 45.6: Green functions for the boundary conditions u(0, t) = 0 and ux (0, t) = 0.

Solution 45.22

a) The Green function problem is

Gtt c2 Gxx = (t )(x ), 0 < x < L, t > 0,


G(0, t; , ) = Gx (L, t; , ) = 0,
G(x, t; , ) = 0 for t < .

The condition that G is zero for t < makes this a causal Green function. We solve this problem by expanding G in
a series of eigenfunctions of the x variable. The coefficients in the expansion will be functions of t. First we find the
eigenfunctions of x in the homogeneous problem. We substitute the separation of variables u = X(x)T (t) into the

2024
homogeneous partial differential equation.
XT 00 = c2 X 00 T
T 00 X 00
= = 2
c2 T X
The eigenvalue problem is
X 00 = 2 X, X(0) = X 0 (L) = 0,
which has the solutions,  
(2n 1) (2n 1)x
n = , Xn = sin , n N.
2L 2L
The series expansion of the Green function has the form,
 
X (2n 1)x
G(x, t; , ) = gn (t) sin .
n=1
2L

We determine the coefficients by substituting the expansion into the Green function differential equation.
Gtt c2 Gxx = (x )(t )
 2 !  
X
00 (2n 1)c (2n 1)x
gn (t) + gn (t) sin = (x )(t )
n=1
2L 2L

We need to expand the right side of the equation in the sine series
 
X (2n 1)x
(x )(t ) = dn (t) sin
n=1
2L
Z L  
2 (2n 1)x
dn (t) = (x )(t ) sin dx
L 0 2L
 
2 (2n 1)
dn (t) = sin (t )
L 2L

2025
By equating coefficients in the sine series, we obtain ordinary differential equation Green function problems for the gn s.
 2  
00 (2n 1)c 2 (2n 1)
gn (t; ) + gn (t; ) = sin (t )
2L L 2L
From the causality condition for G, we have the causality conditions for the gn s,
gn (t; ) = gn0 (t; ) = 0 for t < .
The continuity and jump conditions for the gn are
 
2 (2n 1)
+
gn ( ; ) = 0, gn0 ( + ; ) = sin .
L 2L
A set of homogeneous solutions of the ordinary differential equation are
    
(2n 1)ct (2n 1)ct
cos , sin
2L 2L
Since the continuity and jump conditions are given at the point t = , a handy set of solutions to use for this problem
is the fundamental set of solutions at that point:
    
(2n 1)c(t ) 2L (2n 1)c(t )
cos , sin
2L (2n 1)c 2L
The solution that satisfies the causality condition and the continuity and jump conditions is,
   
4 (2n 1) (2n 1)c(t )
gn (t; ) = sin sin H(t ).
(2n 1)c 2L 2L
Substituting this into the sum yields,
     
4 X 1 (2n 1) (2n 1)c(t ) (2n 1)x
G(x, t; , ) = H(t ) sin sin sin .
c n=1
2n 1 2L 2L 2L

We use trigonometric identities to write this in terms of traveling waves.

2026
 
1 X 1 (2n 1)((x ) c(t ))
G(x, t; , ) = H(t ) sin
c n=1
2n 1 2L
   
(2n 1)((x ) + c(t )) (2n 1)((x + ) c(t ))
+ sin sin
2L 2L
 !
(2n 1)((x + ) + c(t ))
sin
2L

b) Now we consider the Green function with the boundary conditions,

ux (0, t) = ux (L, t) = 0.

First we find the eigenfunctions in x of the homogeneous problem. The eigenvalue problem is

X 00 = 2 X, X 0 (0) = X 0 (L) = 0,

which has the solutions,

0 = 0, X0 = 1,
n  nx 
n = , Xn = cos , n = 1, 2, . . . .
L L
The series expansion of the Green function for t > has the form,

1 X  nx 
G(x, t; , ) = g0 (t) + gn (t) cos .
2 n=1
L

(Note the factor of 1/2 in front of g0 (t). With this, the integral formulas for all the coefficients are the same.) We

2027
determine the coefficients by substituting the expansion into the partial differential equation.
Gtt c2 Gxx = (x )(t )
 
1 00 X
00
 nc 2  nx 
g (t) + gn (t) + gn (t) cos = (x )(t )
2 0 n=1
L L

We expand the right side of the equation in the cosine series.



1 X  nx 
(x )(t ) = d0 (t) + dn (t) cos
2 n=1
L
2 L
Z  nx 
dn (t) = (x )(t ) cos dx
L 0 L
 
2 n
dn (t) = cos (t )
L L
By equating coefficients in the cosine series, we obtain ordinary differential equations for the gn .
 nc 2  
00 2 n
gn (t; ) + gn (t; ) = cos (t ), n = 0, 1, 2, . . .
L L L
From the causality condition for G, we have the causality condiions for the gn ,
gn (t; ) = gn0 (t; ) = 0 for t < .
The continuity and jump conditions for the gn are
 
2 n
+
gn ( ; ) = 0, gn0 ( + ; ) = cos .
L L
The homogeneous solutions of the ordinary differential equation for n = 0 and n > 0 are respectively,
    
nct nct
{1, t}, cos , sin .
L L

2028
Since the continuity and jump conditions are given at the point t = , a handy set of solutions to use for this problem
is the fundamental set of solutions at that point:
    
nc(t ) L nc(t )
{1, t }, cos , sin .
L nc L

The solutions that satisfy the causality condition and the continuity and jump conditions are,

2
g0 (t) = (t )H(t ),
 L  
2 n nc(t )
gn (t) = cos sin H(t ).
nc L L

Substituting this into the sum yields,


    !
t 2 X1 n nc(t )  nx 
G(x, t; , ) = H(t ) + cos sin cos .
L c n=1 n L L L

We can write this as the sum of traveling waves.

 
t 1 X 1 n((x ) c(t ))
G(x, t; , ) = H(t ) + H(t ) sin
L 2c n 2L
  n=1 
n((x ) + c(t )) n((x + ) c(t ))
+ sin sin
2L 2L
 !
n((x + ) + c(t ))
+ sin
2L

2029
Solution 45.23
First we derive Greens identity for this problem. We consider the integral of uL[v] L[u]v on the domain 0 < x < 1,
0 < t < T.
Z TZ 1
(uL[v] L[u]v) dx dt
0 0
Z TZ 1
u(vtt c2 vxx (utt c2 uxx )v dx dt

0 0
Z T Z 1   
2

, c (uvx ux v), uvt ut v dx dt
0 0 x t
Now we can use the divergence theorem to write this as an integral along the boundary of the domain.
I
c2 (uvx ux v), uvt ut v n ds


The domain and the outward normal vectors are shown in Figure 45.7.
Writing out the boundary integrals, Greens identity for this problem is,
Z TZ 1 Z 1
2 2

u(vtt c vxx ) (utt c uxx )v dx dt = (uvt ut v)t=0 dx
0 0 0
Z 0 Z T Z 1
2 2
+ (uvt ut v)t=T dx c (uvx ux v)x=1 dt + c (uvx ux v)x=0 dt
1 0 T

The Green function problem is


Gtt c2 Gxx = (x )(t ), 0 < x, < 1, t, > 0,
Gx (0, t; , ) = Gx (1, t; , ) = 0, t > 0, G(x, t; , ) = 0 for t < .
If we consider G as a function of (, ) with (x, t) as parameters, then it satisfies:
G c2 G = (x )(t ),
G (x, t; 0, ) = G (x, t; 1, ) = 0, > 0, G(x, t; , ) = 0 for > t.

2030
n=(0,1)

t=T

n=(-1,0) n=(1,0)

t=0
x=0 x=1
n=(0,-1)

Figure 45.7: Outward normal vectors of the domain.

Now we apply Greens identity for u = u(, ), (the solution of the wave equation), and v = G(x, t; , ), (the Green
function), and integrate in the (, ) variables. The left side of Greens identity becomes:
Z TZ 1
u(G c2 G ) (u c2 u )G d d

0 0
Z TZ 1
(u((x )(t )) (0)G) d d
0 0
u(x, t).
Since the normal derivative of u and G vanish on the sides of the domain, the integrals along = 0 and = 1 in
Greens identity vanish. If we take T > t, then G is zero for = T and the integral along = T vanishes. The one
remaining integral is Z 1
(u(, 0)G (x, t; , 0) u (, 0)G(x, t; , 0) d.
0

2031
Thus Greens identity allows us to write the solution of the inhomogeneous problem.
Z 1
u(x, t) = (u (, 0)G(x, t; , 0) u(, 0)G (x, t; , 0)) d.
0

With the specified initial conditions this becomes


Z 1
u(x, t) = (G(x, t; , 0) 2 (1 )2 G (x, t; , 0)) d.
0

Now we substitute in the Green function that we found in the previous exercise. The Green function and its derivative
are,

X 2
G(x, t; , 0) = t + cos(n) sin(nct) cos(nx),
n=1
nc

X
G (x, t; , 0) = 1 2 cos(n) cos(nct) cos(nx).
n=1

The integral of the first term is,



!
Z 1 X 2
t+ cos(n) sin(nct) cos(nx) d = t.
0 n=1
nc

The integral of the second term is



Z 1 !
X 1 X 1
2 (1 )2 1 + 2 cos(n) cos(nct) cos(nx) d = 3 44
cos(2nx) cos(2nct).
0 n=1
30 n=1
n

Thus the solution is



1 X 1
u(x, t) = +t3 cos(2nx) cos(2nct).
30 n=1
n 4
4

2032
For c = 1, the solution at x = 3/4, t = 7/2 is,

1 7 X 1
u(3/4, 7/2) = + 3 cos(3n/2) cos(7n).
30 2 n=1
n 4
4

Note that the summand is nonzero only for even terms.



53 3 X 1
u(3/4, 7/2) = cos(3n) cos(14n)
15 16 4 n=1 n4

53 3 X (1)n
=
15 16 4 n=1 n4
53 3 7 4
=
15 16 4 720

12727
u(3/4, 7/2) =
3840

2033
Chapter 46

Conformal Mapping

2034
46.1 Exercises
Exercise 46.1
Use an appropriate conformal map to find a non-trivial solution to Laplaces equation

uxx + uyy = 0,

on the wedge bounded by the x-axis and the line y = x with boundary conditions:

1. u = 0 on both sides.
du
2. = 0 on both sides (where n is the inward normal to the boundary).
dn
Exercise 46.2
Consider
uxx + uyy = (x )(y ),
on the quarter plane x, y > 0 with u(x, 0) = u(0, y) = 0 (and , > 0).

1. Use image sources to find u(x, y; , ).

2. Compare this to the solution which would be obtained using conformal maps and the Green function for the upper
half plane.

3. Finally use this idea and conformal mapping to discover how image sources are arrayed when the domain is now
the wedge bounded by the x-axis and the line y = x (with u = 0 on both sides).

Exercise 46.3
= + is an analytic function of z, = (z). We assume that 0 (z) is nonzero on the domain of interest. u(x, y)
is an arbitrary smooth function of x and y. When expressed in terms of and , u(x, y) = (, ). In Exercise 8.15
we showed that 2  2
2 2 d u 2u

+ 2 = + .
2 dz x2 y 2

2035
1. Show that if u satisfies Laplaces equation in the z-plane,

uxx + uyy = 0,

then satisfies Laplaces equation in the -plane,

+ = 0,

2. Show that if u satisfies Helmholtzs equation in the z-plane,

uxx + uyy = u,

then in the -plane satisfies 2


dz
+ = .
d
3. Show that if u satisfies Poissons equation in the z-plane,

uxx + uyy = f (x, y),

then satisfies Poissons equation in the -plane,


2
dz
+ = (, ),
d
where (, ) = f (x, y).
4. Show that if in the z-plane, u satisfies the Green function problem,

uxx + uyy = (x x0 )(y y0 ),

then in the -plane, satisfies the Green function problem,

+ = ( 0 )( 0 ).

2036
Exercise 46.4
A semi-circular rod of infinite extent is maintained at temperature T = 0 on the flat side and at T = 1 on the curved
surface:
x2 + y 2 = 1, y > 0.
Use the conformal mapping
1+z
w = + = , z = x + y,
1z
to formulate the problem in terms of and . Solve the problem in terms of these variables. This problem is solved
with an eigenfunction expansion in Exercise 37.24. Verify that the two solutions agree.
Exercise 46.5
Consider Laplaces equation on the domain < x < , 0 < y < , subject to the mixed boundary conditions,

u = 1 on y = 0, x > 0,
u = 0 on y = , x > 0,
uy = 0 on y = 0, y = , x < 0.

Because of the mixed boundary conditions, (u and uy are given on separate parts of the same boundary), this problem
cannot be solved with separation of variables. Verify that the conformal map,

= cosh1 (ez ),

with z = x + y, = + maps the infinite interval into the semi-infinite interval, > 0, 0 < < . Solve Laplaces
equation with the appropriate boundary conditions in the plane by inspection. Write the solution u in terms of x and
y.

2037
46.2 Hints
Hint 46.1

Hint 46.2

Hint 46.3

Hint 46.4
Show that w = (1 + z)/(1 z) maps the semi-disc, 0 < r < 1, 0 < < to the first quadrant of the w plane. Solve
the problem for v(, ) by taking Fourier sine transforms in and .
To show that the solution for v(, ) is equivalent to the series expression for u(r, ), first find an analytic function
g(w) of which v(, ) is the imaginary part. Change variables to z to obtain the analytic function f (z) = g(w). Expand
f (z) in a Taylor series and take the imaginary part to show the equivalence of the solutions.

Hint 46.5
To see how the boundary is mapped, consider the map,

z = log(cosh ).

The problem in the plane is,

v + v = 0, > 0, 0 < < ,


v (0, ) = 0, v(, 0) = 1, v(, ) = 0.

To solve this, find a plane that satisfies the boundary conditions.

2038
46.3 Solutions
Solution 46.1
We map the wedge to the upper half plane with the conformal transformation = z 4 .
1. We map the wedge to the upper half plane with the conformal transformation = z 4 . The new problem is

u + u = 0, u(, 0) = 0.

This has the solution u = . We transform this problem back to the wedge.

u(x, y) = = z 4


u(x, y) = = x4 + 4x3 y 6x2 y 2 4xy 3 + y 4




u(x, y) = 4x3 y 4xy 3


u(x, y) = 4xy x2 y 2


2. We dont need to use conformal mapping to solve the problem with Neumman boundary conditions. u = c is a
solution to
du
uxx + uyy = 0, =0
dn
on any domain.
Solution 46.2
1. We add image sources to satisfy the boundary conditions.

uxx + uyy = (x )(y ) (x + )(y ) (x )(y + ) + (x + )(y + )

1  p 2 2
 p
2 2

u= ln (x ) + (y ) ln (x + ) + (y )
2 p  p 
ln (x )2 + (y + )2 + ln (x + )2 + (y + )2

2039
((x )2 + (y )2 ) ((x + )2 + (y + )2 )
 
1
u= ln
4 ((x + )2 + (y )2 ) ((x )2 + (y + )2 )
2. The Green function for the upper half plane is
((x )2 + (y )2 )
 
1
G= ln
4 ((x )2 + (y + )2 )
We use the conformal map,
c = z 2 , c = a + b.
a = x2 y 2 , b = 2xy
We compute the Jacobian of the mapping.

ax ay 2x 2y
= 4 x2 + y 2

J = =
bx by 2y 2x
We transform the problem to the upper half plane, solve the problem there, and then transform back to the first
quadrant.
uxx + uyy = (x )(y )
2
dc
(uaa + ubb ) = 4 x2 + y 2 (a )(b )

dz
(uaa + ubb ) |2z|2 = 4 x2 + y 2 (a )(b )


uaa + ubb = (a )(b )


((a )2 + (b )2 )
 
1
u= ln
4 ((a )2 + (b + )2 )
 2
((x y 2 2 + 2 )2 + (2xy 2)2 )

1
u= ln
4 ((x2 y 2 2 + 2 )2 + (2xy + 2)2 )
((x )2 + (y )2 ) ((x + )2 + (y + )2 )
 
1
u= ln
4 ((x + )2 + (y )2 ) ((x )2 + (y + )2 )

2040
We obtain the some solution as before.
3. First consider
u = (x )(y ), u(x, 0) = u(x, x) = 0.
Enforcing the boundary conditions will require 7 image sources obtained from 4 odd reflections. Refer to Fig-
ure 46.1 to see the reflections pictorially. First we do a negative reflection across the line y = x, which adds a
negative image source at the point (, ) This enforces the boundary condition along y = x.
u = (x )(y ) (x )(y ), u(x, 0) = u(x, x) = 0
Now we take the negative image of the reflection of these two sources across the line y = 0 to enforce the
boundary condition there.
u = (x )(y ) (x )(y ) (x )(y + ) + (x )(y + )
The point sources are no longer odd symmetric about y = x. We add two more image sources to enforce that
boundary condition.

u = (x )(y ) (x )(y ) (x )(y + ) + (x )(y + )


+ (x + )(y ) (x + )(y )
Now sources are no longer odd symmetric about y = 0. Finally we add two more image sources to enforce that
boundary condition. Now the sources are odd symmetric about both y = x and y = 0.

u = (x )(y ) (x )(y ) (x )(y + ) + (x )(y + )


+ (x + )(y ) (x + )(y ) + (x + )(y + ) (x + )(y + )

Solution 46.3
2  2
2 2 d u 2u

+ 2 = + .
2 dz x2 y 2

2041
Figure 46.1: Odd reflections to enforce the boundary conditions.

1.

uxx + uyy = 0
2
d
( + ) = 0
dz
+ = 0

2042
2.

uxx + uyy = u
2
d
( + ) =
dz
2
dz
+ =
d

3.

uxx + uyy = f (x, y)


2
d
( + ) = (, )
dz
2
dz
+ = (, )
d

4. The Jacobian of the mapping is



x y
J = = x y x y = x2 + y2 .
x y

Thus the Dirac delta function on the right side gets mapped to
1
( 0 )( 0 ).
x2 + y2

Next we show that |dz/d|2 has the same value as the Jacobian.
2
dz
= (x + y )(x y ) = x2 + y2
d

2043
Now we transform the Green function problem.

uxx + uyy = (x x0 )(y y0 )


2
d 1
( + ) = ( 0 )( 0 )
dz x + y2
2

+ = ( 0 )( 0 )

Solution 46.4
The mapping,
1+z
w= ,
1z
maps the unit semi-disc to the first quadrant of the complex plane.
We write the mapping in terms of r and .

1 + r e 1 r2 + 2r sin
+ = =
1 r e 1 + r2 2r cos

1 r2
=
1 + r2 2r cos
2r sin
=
1 + r2 2r cos
2r
Consider a semi-circle of radius r. The image of this under the conformal mapping is a semi-circle of radius 1r 2 and
1+r 2 1r 1+r
center 1r2 in the first quadrant of the w plane. This semi-circle intersects the axis at 1+r and 1r . As r ranges
from zero to one, these semi-circles cover the first quadrant of the w plane. (See Figure 46.2.)
We also note how the boundary of the semi-disc is mapped to the boundary of the first quadrant of the w plane.
The line segment = 0 is mapped to the real axis > 1. The line segment = is mapped to the real axis 0 < < 1.
Finally, the semi-circle r = 1 is mapped to the positive imaginary axis.

2044
1 5
4
3
2
1

-1 1 1 2 3 4 5

1+z
Figure 46.2: The conformal map, w = 1z
.

The problem for v(, ) is,


v + v = 0, > 0, > 0,
v(, 0) = 0, v(0, ) = 1.
We will solve this problem with the Fourier sine transform. We take the Fourier sine transform of the partial differential
equation, first in and then in .

2 v(, ) + v(0, ) + v(, ) = 0, v(, 0) = 0

2
v(, ) + + v(, ) = 0, v(, 0) = 0

) + 2 v(,
2 v(, ) + v(, 0) = 0
2

) =
v(,
2 (2 + 2 )
Now we utilize the Fourier sine transform pair,
/
Fs ecx =
 
,
2 + c2

2045
to take the inverse sine transform in .
1
v(, ) = e

With the Fourier sine transform pair,
h  x i 1 c
Fs 2 arctan = e ,
c
we take the inverse sine transform in to obtain the solution.
 
2
v(, ) = arctan

Since v is harmonic, it is the imaginary part of an analytic function g(w). By inspection, we see that this function is

2
g(w) = log(w).

We change variables to z, f (z) = g(w).
 
2 1+z
f (z) = log
1z
We expand f (z) in a Taylor series about z = 0,

4 X zn
f (z) = ,
n=1 n
oddn

and write the result in terms of r and , z = r e .



4 X rn e
f (z) =
n=1 n
oddn

2046
u(r, ) is the imaginary part of f (z).

4X1 n
u(r, ) = r sin(n)
n=1 n
oddn

This demonstrates that the solutions obtained with conformal mapping and with an eigenfunction expansion in Exer-
cise 37.24 agree.
Solution 46.5
Instead of working with the conformal map from the z plane to the plane,

= cosh1 (ez ),

it will be more convenient to work with the inverse map,

z = log(cosh ),

which maps the semi-infinite strip to the infinite one. We determine how the boundary of the domain is mapped so
that we know the appropriate boundary conditions for the semi-infinite strip domain.

A { : > 0, = 0} 7 {log(cosh()) : > 0} = {z : x > 0, y = 0}


B { : > 0, = } 7 {log( cosh()) : > 0} = {z : x > 0, y = }
C { : = 0, 0 < < /2} 7 {log(cos()) : 0 < < /2} = {z : x < 0, y = 0}
D { : = 0, /2 < < } 7 {log(cos()) : /2 < < } = {z : x < 0, y = }

From the mapping of the boundary, we see that the solution v(, ) = u(x, y), is 1 on the bottom of the semi-infinite
strip, 0 on the top. The normal derivative of v vanishes on the vertical boundary. See Figure 46.3.
In the plane, the problem is,

v + v = 0, > 0, 0 < < ,


v (0, ) = 0, v(, 0) = 1, v(, ) = 0.

2047
y

B D B
D z=log(cosh( ))

C x
A C A

y

v=0 uy=0 u=0

v=0
x
v=1 uy=0 u=1

Figure 46.3: The mapping of the boundary conditions.

By inspection, we see that the solution of this problem is,


v(, ) = 1 .

The solution in the z plane is


1
= cosh1 (ez ) ,

u(x, y) = 1

where z = x + y. We will find the imaginary part of cosh1 (ez ) in order to write this explicitly in terms of x and y.

2048
Recall that we can write the cosh1 in terms of the logarithm.
 
cosh1 (w) = log w + w2 1
 
cosh1 (ez ) = log ez + e2z 1
 
z

= log e 1+ 1 e 2z
 
= z + log 1 + 1 e2z
Now we need to find the imaginary part. Well work from the inside out. First recall,
r
p   y  p   y 
x + y = x2 + y 2 exp tan1 = 4 x2 + y 2 exp tan1 ,
x 2 x
so that we can write the innermost factor as,
p
1 e2z = 1 e2x cos(2y) + e2x sin(2y)
  2x 
p
4 2x 2 2x 2
1 e sin(2y)
= (1 e cos(2y)) + (e sin(2y)) exp tan
2 1 e2x cos(2y)
  
p
4 2x 4x
1 sin(2y)
= 1 2e cos(2y) + e exp tan
2 e2x cos(2y)
We substitute this into the logarithm.

   

2z
 p
4 2x 4x
1 sin(2y)
log 1 + 1 e = log 1 + 1 2 e cos(2y) + e exp tan
2 e2x cos(2y)
Now we can write .
  
= = z + log 1 + 1 e2z
p   
4
1 2 e2x cos(2y) + e4x sin 12 tan1 e2xsin(2y)
cos(2y)
= y + tan1   
sin(2y)
p 1
2x 4x 1
1 + 1 2e
4
cos(2y) + e cos 2 tan e2x cos(2y)

2049
Finally we have the solution, u(x, y).
  
sin(2y)
p
1 24
e 2x cos(2y) + e4x sin 1 tan1
y 1 2 e2x cos(2y)
u(x, y) = 1 tan1   
1 + 4 1 2 e2x cos(2y) + e4x cos 12 tan1 e2xsin(2y)
p
cos(2y)

2050
Chapter 47

Non-Cartesian Coordinates

47.1 Spherical Coordinates

Writing rectangular coordinates in terms of spherical coordinates,

x = r cos sin
y = r sin sin
z = r cos .

2051
The Jacobian is

cos sin r sin sin r cos cos

sin sin r cos sin r sin cos

cos 0 r sin

cos sin sin cos cos

= r2 sin sin sin cos sin cos
cos 0 sin
2
= r sin ( cos2 sin2 sin2 cos2 cos2 cos2 sin2 sin2 )
= r2 sin (sin2 + cos2 )
= r2 sin .

Thus we have that ZZZ ZZZ


f (x, y, z) dx dy dz = f (r, , )r2 sin dr d d.
V V

47.2 Laplaces Equation in a Disk


Consider Laplaces equation in polar coordinates

1 2u
 
1 u
r + = 0, 0r1
r r r r2 2

subject to the the boundary conditions

1. u(1, ) = f ()

2. ur (1, ) = g().

2052
We separate variables with u(r, ) = R(r)T ().

1 0 1
(R T + rR00 T ) + 2 RT 00 = 0
r r
00 0
2 R R T 00
r +r = =
R R T
Thus we have the two ordinary differential equations

T 00 + T = 0, T (0) = T (2), T 0 (0) = T 0 (2)


r2 R00 + rR0 R = 0, R(0) < .

The eigenvalues and eigenfunctions for the equation in T are

1
0 = 0, T0 =
2
n = n 2 , Tn(1) = cos(n), Tn(2) = sin(n)

(I chose T0 = 1/2 so that all the eigenfunctions have the same norm.)

For = 0 the general solution for R is


R = c1 + c2 log r.
Requiring that the solution be bounded gives us
R0 = 1.
For = n2 > 0 the general solution for R is
R = c1 rn + c2 rn .
Requiring that the solution be bounded gives us
Rn = r n .

2053
Thus the general solution for u is


a0 X n
u(r, ) = + r [an cos(n) + bn sin(n)] .
2 n=1

For the boundary condition u(1, ) = f () we have the equation



a0 X
f () = + [an cos(n) + bn sin(n)] .
2 n=1

If f () has a Fourier series then the coefficients are

1 2
Z
a0 = f () d
0
1 2
Z
an = f () cos(n) d
0
1 2
Z
bn = f () sin(n) d.
0

For the boundary condition ur (1, ) = g() we have the equation



X
g() = n [an cos(n) + bn sin(n)] .
n=1

g() has a series of this form only if


Z 2
g() d = 0.
0

2054
The coefficients are
Z 2
1
an = g() cos(n) d
n 0
Z 2
1
bn = g() sin(n) d.
n 0

47.3 Laplaces Equation in an Annulus


Consider the problem

1 2u
 
2 1 u
u= r + = 0, 0 r < a, < ,
r r r r2 2

with the boundary condition


u(a, ) = 2 .

So far this problem only has one boundary condition. By requiring that the solution be finite, we get the boundary
condition
|u(0, )| < .

By specifying that the solution be C 1 , (continuous and continuous first derivative) we obtain

u u
u(r, ) = u(r, ) and (r, ) = (r, ).

We will use the method of separation of variables. We seek solutions of the form

u(r, ) = R(r)().

2055
Substituting into the partial differential equation,
2 u 1 u 1 2u
+ + =0
r2 r r r2 2
1 1
R00 + R0 = 2 R00
r r
r2 R00 rR0 00
+ = =
R R
Now we have the boundary value problem for ,

00 () + () = 0, < ,

subject to

() = () and 0 () = 0 ()

We consider the following three cases for the eigenvalue, ,



< 0. No linear combination of the solutions, = exp( ), exp( ), can satisfy the boundary conditions.
Thus there are no negative eigenvalues.

= 0. The general solution solution is = a + b. By applying the boundary conditions, we get = a. Thus we
have the eigenvalue and eigenfunction,
0 = 0, A0 = 1.

> 0. The general solution is = a cos( )+b sin( ). Applying the boundary conditions yields the eigenvalues

n = n 2 , n = 1, 2, 3, . . .

with the associated eigenfunctions

An = cos(n) and Bn = sin(n).

2056
The equation for R is
r2 R00 + rR0 n R = 0.
In the case 0 = 0, this becomes
1
R00 = R0
r
0 a
R =
r
R = a log r + b

Requiring that the solution be bounded at r = 0 yields (to within a constant multiple)

R0 = 1.

For n = n2 , n 1, we have

r2 R00 + rR0 n2 R = 0

Recognizing that this is an Euler equation and making the substitution R = r ,

( 1) + n2 = 0
= n
R = arn + brn .

requiring that the solution be bounded at r = 0 we obtain (to within a constant multiple)

Rn = r n

The general solution to the partial differential equation is a linear combination of the eigenfunctions

X
u(r, ) = c0 + [cn rn cos n + dn rn sin n] .
n=1

2057
We determine the coefficients of the expansion with the boundary condition

X
2
u(a, ) = = c0 + [cn an cos n + dn an sin n] .
n=1

We note that the eigenfunctions 1, cos n, and sin n are orthogonal on . Integrating the boundary
condition from to yields
Z Z
2
d = c0 d

2

c0 = .
3
Multiplying the boundary condition by cos m and integrating gives
Z Z
2 m
cos m d = cm a cos2 m d

(1)m 8
cm = .
m 2 am
We multiply by sin m and integrate to get
Z Z
2 m
sin m d = dm a sin2 m d

dm = 0

Thus the solution is



2 X (1)n 8 n
u(r, ) = + 2 an
r cos n.
3 n=1
n

2058
Part VI

Calculus of Variations

2059
Chapter 48

Calculus of Variations

2060
48.1 Exercises
Exercise 48.1 R
Discuss the problem of minimizing 0 ((y 0 )4 6(y 0 )2 ) dx, y(0) = 0, y() = . Consider both C 1 [0, ] and Cp1 [0, ],
and comment (with reasons) on whether your answers are weak or strong minima.

Exercise 48.2
Consider
Rx
1. x01 (a(y 0 )2 + byy 0 + cy 2 ) dx, y(x0 ) = y0 , y(x1 ) = y1 , a 6= 0,
Rx
2. x01 (y 0 )3 dx, y(x0 ) = y0 , y(x1 ) = y1 .
Can these functionals have broken extremals, and if so, find them.
Exercise 48.3
Discuss finding a weak extremum for the following:
R1
1. 0 ((y 00 )2 2xy) dx, y(0) = y 0 (0) = 0, y(1) = 1
120
R1
2. 0 21 (y 0 )2 + yy 0 + y 0 + y dx


Rb
3. a (y 2 + 2xyy 0 ) dx, y(a) = A, y(b) = B
R1
4. 0 (xy + y 2 2y 2 y 0 ) dx, y(0) = 1, y(1) = 2

Exercise 48.4
Find the natural boundary conditions associated with the following functionals:
RR
1. D F (x, y, u, ux , uy ) dx dy
RR  R
2. D p(x, y)(u2x + u2y ) q(x, y)u2 dx dy + (x, y)u2 ds
Here D represents a closed boundary domain with boundary , and ds is the arc-length differential. p and q are known
in D, and is known on .

2061
Exercise 48.5
The equations for water waves with free surface y = h(x, t) and bottom y = 0 are

xx + yy = 0 0 < y < h(x, t),


1 1
t + 2x + 2y + gy = 0 on y = h(x, t),
2 2
ht + x hx y = 0, on y = h(x, t),
y = 0 on y = 0,

where the fluid motion is described by (x, y, t) and g is the acceleration of gravity. Show that all these equations may
be obtained by varying the functions (x, y, t) and h(x, t) in the variational principle
!
ZZ Z h(x,t)  
1 1
t + 2x + 2y + gy dy dx dt = 0,
R 0 2 2

where R is an arbitrary region in the (x, t) plane.

Exercise 48.6 Rb
Extremize the functional a F (x, y, y 0 ) dx, y(a) = A, y(b) = B given that the admissible curves can not penetrate
R 10
the interior of a given region R in the (x, y) plane. Apply your results to find the curves which extremize 0 (y 0 )3 dx,
y(0) = 0, y(10) = 0 given that the admissible curves can not penetrate the interior of the circle (x 5)2 + y 2 = 9.

Exercise 48.7 R p
Consider the functional y ds where ds is the arc-length differential (ds = (dx)2 + (dy)2 ). Find the curve or
curves from a given vertical line to a given fixed point B = (x1 , y1 ) which minimize this functional. Consider both the
classes C 1 and Cp1 .

Exercise 48.8
A perfectly flexible uniform rope of length L hangs in equilibrium with one end fixed at (x1 , y1 ) so that it passes over
a frictionless pin at (x2 , y2 ). What is the position of the free end of the rope?

2062
Exercise 48.9
The drag on a supersonic airfoil of chord c and shape y = y(x) is proportional to
Z c  2
dy
D= dx.
0 dx
Find the shape for minimum drag if the moment of inertia of the contour with respect to the x-axis is specified; that
is, find the shape for minimum drag if
Z c
y 2 dx = A, y(0) = y(c) = 0, (c, A given).
0

Exercise 48.10
The deflection y of a beam executing free (small) vibrations of frequency satisfies the differential equation
d2
 
dy
2
EI 2 y = 0,
dx dx
where EI is the flexural rigidity and is the linear mass density. Show that the deflection modes are extremals of the
problem RL !
00 2
EI(y ) dx
2 0
RL = 0, (L = length of beam)
y 2 dx
0
when appropriate homogeneous end conditions are prescribed. Show that stationary values of the ratio are the squares
of the natural frequencies.
Exercise 48.11
A boatman wishes to steer his boat so as to minimize the transit time required to cross a river of width l. The path of
the boat is given parametrically by
x = X(t), y = Y (t),
for 0 t T . The river has no cross currents, so the current velocity is directed downstream in the y-direction. v0 is
the constant boat speed relative to the surrounding water, and w = w(x, y, t) denotes the downstream river current at
point (x, y) at time t. Then,
X(t) = v0 cos (t), Y (t) = v0 sin (t) + w,

2063
where (t) is the steering angle of the boat at time t. Find the steering control function (t) and the final time T that
will transfer the boat from the initial state (X(0), Y (0)) = (0, 0) to the final state at X(t) = l in such a way as to
minimize T .
Exercise 48.12
Two particles of equal mass m are connected by an inextensible string which passes through a hole in a smooth
p
horizontal table. The first particle is on the table moving with angular velocity = g/ in a circular path, of radius
, around the hole. The second particle is suspended vertically and is in equilibrium. At time t = 0, the suspended
mass is pulled downward a short distance and released while the first mass continues to rotate.
1. If x represents the distance of the second mass below its equilibrium at time t and represents the angular
position of the first particle at time t, show that the Lagrangian is given by
 
2 1 2 2
L = m x + ( x) + gx
2
and obtain the equations of motion.
2. In the case where the displacement of the suspended mass from equilibrium is small, show that the suspended
mass performs small vertical oscillations and find the period of these oscillations.
Exercise 48.13
A rocket is propelled vertically upward so as to reach a prescribed height h in minimum time while using a given fixed
quantity of fuel. The vertical distance x(t) above the surface satisfies,
mx = mg + mU (t), x(0) = 0,
(x)(0) = 0,
where U (t) is the acceleration provided by engine thrust. We impose the terminal constraint x(T ) = h, and we wish
to find the particular thrust function U (t) which will minimize T assuming that the total thrust of the rocket engine
over the entire thrust time is limited by the condition,
Z T
U 2 (t) dt = k 2 .
0

Here k is a given positive constant which measures the total amount of fuel available.

2064
Exercise 48.14
A space vehicle moves along a straight path in free space. x(t) is the distance to its docking pad, and a, b are its
position and speed at time t = 0. The equation of motion is
x = M sin V, x(0) = a, x(0) = b,
where the control function V (t) is related to the rocket acceleration U (t) by U = M sin V , M = const. We wish
to dock the vehicle in minimum time; that is, we seek a thrust function U (t) which will minimize the final time T
while bringing the vehicle to rest at the origin with x(T ) = 0, x(T ) = 0. Find U (t), and in the (x, x)-plane plot the
corresponding trajectory which transfers the state of the system from (a, b) to (0, 0). Account for all values of a and b.

Exercise 48.15 Rm p
Find a minimum for the functional I(y) = 0 y + h 1 + (y 0 )2 dx in which h > 0, y(0) = 0, y(m) = M > h.
Discuss the nature of the minimum, (i.e., weak, strong, . . . ).

Exercise 48.16 R p
Show that for the functional n(x, y) 1 + (y 0 )2 dx, where n(x, y) 0 in some domain D, the Weierstrass E function
E(x, y, q, y 0 ) is non-negative for arbitrary finite p and y 0 at any point of D. What is the implication of this for Fermats
Principle?
Exercise 48.17
2
Consider the integral 1+y
R
0
(y ) 2 dx between fixed limits. Find the extremals, (hyperbolic sines), and discuss the Jacobi,
Legendre, and Weierstrass conditions and their implications regarding weak and strong extrema. Also consider the value
of the integral on any extremal compared with its value on the illustrated strong variation. Comment!
Pi Qi are vertical segments, and the lines Qi Pi+1 are tangent to the extremal at Pi+1 .
Exercise 48.18
Rx
Consider I = x01 y 0 (1 + x2 y 0 ) dx, y(x0 ) = y0 , y(x1 ) = y1 . Can you find continuous curves which will minimize I if

(i) x0 = 1, y0 = 1, x1 = 2, y1 = 4,
(ii) x0 = 1, y0 = 3, x1 = 2, y1 = 5,
(iii) x0 = 1, y0 = 1, x1 = 2, y1 = 1.

2065
Exercise 48.19
Starting from ZZ Z
(Qx Py ) dx dy = (P dx + Q dy)
D
prove that
ZZ ZZ Z
(a) xx dx dy = xx dx dy + (x x ) dy,
Z ZD Z ZD Z
(b) yy dx dy = yy dx dy (y y ) dx,
Z ZD Z ZD Z
Z
1 1
(c) xy dx dy = xy dx dy (x x ) dx + (y y ) dy.
D D 2 2
Then, consider Z t1 ZZ
(uxx + uyy )2 + 2(1 )(uxx uyy u2xy ) dx dy dt.

I(u) =
t0 D
Show that Z t1 ZZ Z t1 Z  
4 (u)
I = ( u)u dx dy dt + P (u)u + M (u) ds dt,
t0 D t0 n
where P and M are the expressions we derived in class for the problem of the vibrating plate.
Exercise 48.20
For the following functionals use the Rayleigh-Ritz method to find an approximate solution of the problem of minimizing
the functionals and compare your answers with the exact solutions.
Z 1
(y 0 )2 y 2 2xy dx,

y(0) = 0 = y(1).
0

For this problem take an approximate solution of the form

y = x(1 x) (a0 + a1 x + + an xn ) ,

2066
and carry out the solutions for n = 0 and n = 1.

Z 2
(y 0 )2 + y 2 + 2xy dx,

y(0) = 0 = y(2).
0


2
x2 1 2
Z  
0 2
x(y ) y 2x2 y dx, y(1) = 0 = y(2)
1 x

Exercise 48.21
Let K(x) belong to L1 (, ) and define the operator T on L2 (, ) by
Z
T f (x) = K(x y)f (y) dy.

1. Show that the spectrum of T consists of the range of the Fourier transform K of K, (that is, the set of all values
K(y) with < y < ), plus 0 if this is not already in the range. (Note: From the assumption on K it follows
that K is continuous and approaches zero at .)

2. For in the spectrum of T , show that is an eigenvalue if and only if K takes on the value on at least some
interval of positive length and that every other in the spectrum belongs to the continuous spectrum.

3. Find an explicit representation for (T I)1 f for not in the spectrum, and verify directly that this result
agrees with that givenby the Neumann series if is large enough.

Exercise 48.22
Let U be the space of twice continuously differentiable functions f on [1, 1] satisfying f (1) = f (1) = 0, and
d2
W = C[1, 1]. Let L : U 7 W be the operator dx 2 . Call in the spectrum of L if the following does not occur:

There is a bounded linear transformation T : W 7 U such that (L I)T f = f for all f W and T (L I)f = f
for all f U . Determine the spectrum of L.

2067
Exercise 48.23
Solve the integral equations
Z 1
x2 y y 2 (y) dy

1. (x) = x +
0
Z x
2. (x) = x + K(x, y)(y) dy
0

where (
sin(xy) for x 1 and y 1,
K(x, y) =
0 otherwise
In both cases state for which values of the solution obtained is valid.
Exercise 48.24
1. Suppose that K = L1 L2 , where L1 L2 L2 L1 = I. Show that if x is an eigenvector of K corresponding to the
eigenvalue , then L1 x is an eigenvector of K corresponding to the eigenvalue 1, and L2 x is an eigenvector
corresponding to the eigenvalue + 1.
d 2
2. Find the eigenvalues and eigenfunctions of the operator K dt + t4 in the space of functions u L2 (, ).
2
(Hint: L1 = 2t + dt
d
, L2 = 2t dt
d
. et /4 is the eigenfunction corresponding to the eigenvalue 1/2.)

Exercise 48.25
Prove that if the value of = 1 is in the residual spectrum of T , then 1 is in the discrete spectrum of T .

Exercise 48.26
Solve

1. Z 1
00
u (t) + sin(k(s t))u(s) ds = f (t), u(0) = u0 (0) = 0.
0

2068
2. Z
u(x) = K(x, s)u(s) ds
0
where 
sin x+s
1 2 
X sin nx sin ns
K(x, s) = log =

xs
2 sin 2 n=1
n

3.
2
1 h2
Z
1
(s) = (t) dt, |h| < 1
0 2 1 2h cos(s t) + h2
4. Z
(x) = cosn (x )() d

Exercise 48.27
Let K(x, s) = 2 2 6|x s| + 3(x s)2 .
1. Find the eigenvalues and eigenfunctions of
Z 2
(x) = K(x, s)(s) ds.
0

(Hint: Try to find an expansion of the form



X
K(x, s) = cn en(xs) .)
n=

2. Do the eigenfunctions form a complete set? If not, show that a complete set may be obtained by adding a suitable
set of solutions of Z 2
K(x, s)(s) ds = 0.
0

2069
3. Find the resolvent kernel (x, s, ).

Exercise 48.28
Let K(x, s) be a bounded self-adjoint kernel on the finite interval (a, b), and let T be the integral operator on L2 (a, b)
with kernel K(x, s). For a polynomial p(t) = a0 +a1 t+ +an tn we define the operator p(T ) = a0 I +a1 T + +an T n .
Prove that the eigenvalues of p(T ) are exactly the numbers p() with an eigenvalue of T .

Exercise 48.29
Show that if f (x) is continuous, the solution of
Z
(x) = f (x) + cos(2xs)(s) ds
0

is R
f (x) + f (s) cos(2xs) ds
0
(x) = .
1 2 /4

Exercise 48.30
Consider
Lu = 0 in D, u = f on C,

where
Lu uxx + uyy + aux + buy + cu.

Here a, b and c are continuous functions of (x, y) on D + C. Show that the adjoint L is given by

L v = vxx + vyy avx bvy + (c ax by )v

and that Z Z

(vLu uL v) = H(u, v), (48.1)
D C

2070
where

H(u, v) (vux uvx + auv) dy (vuy uvy + buv) dx


 
u v x y
= v u + auv + buv ds.
n n n n
Take v in (48.1) to be the harmonic Green function G given by
!
1 1
G(x, y; , ) = log p + ,
2 (x )2 + (y )2

and show formally, (use Delta functions), that (48.1) becomes


Z Z

u(, ) u(L )G dx dy = H(u, G) (48.2)
D C

where u satisfies Lu = 0, (G = in D, G = 0 on C). Show that (48.2) can be put into the forms
Z

u+ (c ax by )G aGx bGy u dx dy = U (48.3)
D

and Z
u+ (aux + buy + cu)G dx dy = U, (48.4)
D
where U is the known harmonic function in D with assumes the boundary values prescribed for u. Finally, rigorously
show that the integrodifferential equation (48.4) can be solved by successive approximations when the domain D is
small enough.
Exercise 48.31
Find the eigenvalues and eigenfunctions of the following kernels on the interval [0, 1].
1.
K(x, s) = min(x, s)

2071
2.
K(x, s) = emin(x,s)
(Hint: 00 + 0 + ex = 0 can be solved in terms of Bessel functions.)

Exercise 48.32
Use Hilbert transforms to evaluate
Z
sin(kx) sin(lx)
1. dx
x2 z 2
Z
cos(px) cos(qx)
2. dx
x2
Z
(x2 ab) sin x + (a + b)x cos x
3. dx
x(x2 + a2 )(x2 + b2 )

Exercise 48.33
Show that
Z
(1 t2 )1/2 log(1 + t)   
dt = x log 2 1 + (1 x2 )1/2 arcsin(x) .
tx 2

Exercise 48.34
Let C be a simple closed contour. Let g(t) be a given function and consider
Z
1 f (t) dt
= g(t0 ) (48.5)
C t t0
Note that the left side can be written as F + (t0 ) + F (t0 ). Define a function W (z) such that W (z) = F (z) for z inside
C and W (z) = F (z) for z outside C. Proceeding in this way, show that the solution of (48.5) is given by
Z
1 g(t) dt
f (t0 ) = .
C t t0

2072
Exercise 48.35
If C is an arc with endpoints and , evaluate
Z
1 1
(i) d, where 0 < < 1
C ( )1 ( ) ( )

n
Z 
1
(ii) d, where 0 < < 1, integer n 0.
C
Exercise 48.36
Solve
Z 1
(y)
dy = f (x).
1 y2 x2

Exercise 48.37
Solve
1 1 f (t)
Z
dt = f (x), where 1 < < 1.
0 t x
Are there any solutions for > 1? (The operator on the left is self-adjoint. Its spectrum is 1 1.)

Exercise 48.38
Show that the general solution of
tan(x) 1 f (t)
Z
dt = f (x)
0 tx
is
k sin(x)
f (x) = .
(1 x)1x/ xx/

Exercise 48.39
Show that the general solution of
Z
0 f (t)
f (x) + dt = 1
C tx

2073
is given by
1
f (x) = + k ex ,

(k is a constant). Here C is a simple closed contour, a constant and f (x) a differentiable function on C. Generalize
the result to the case of an arbitrary function g(x) on the right side, where g(x) is analytic inside C.

Exercise 48.40
Show that the solution of
Z  
1
+ P (t x) f (t) dt = g(x)
C tx
is given by Z Z
1 g( ) 1
f (t) = 2 d 2 g( )P ( t) d.
C t C
Here C is a simple closed curve, and P (t) is a given entire function of t.

Exercise 48.41
Solve
Z 1 Z 3
f (t) f (t)
dt + dt = x
0 tx 2 tx

where this equation is to hold for x in either (0, 1) or (2, 3).

Exercise 48.42
Solve
Z x Z 1
f (t) f (t)
dt + A dt = 1
0 xt x tx
where A is a real positive constant. Outline briefly the appropriate method of A is a function of x.

2074
48.2 Hints
Hint 48.1

Hint 48.2

Hint 48.3

Hint 48.4

Hint 48.5

Hint 48.6

Hint 48.7

Hint 48.8

Hint 48.9

Hint 48.10

2075
Hint 48.11

Hint 48.12

Hint 48.13

Hint 48.14

Hint 48.15

Hint 48.16

Hint 48.17

Hint 48.18

Hint 48.19

Hint 48.20

Hint 48.21

2076
Hint 48.22

Hint 48.23

Hint 48.24

Hint 48.25

Hint 48.26

Hint 48.27

Hint 48.28

Hint 48.29

Hint 48.30

Hint 48.31

Hint 48.32

2077
Hint 48.33

Hint 48.34

Hint 48.35

Hint 48.36

Hint 48.37

Hint 48.38

Hint 48.39

Hint 48.40

Hint 48.41

Hint 48.42

2078
48.3 Solutions
Solution 48.1
C 1 [0, ] Extremals
Admissible Extremal. First we consider continuously differentiable extremals. Because the Lagrangian is a
function of y 0 alone, we know that the extremals are straight lines. Thus the admissible extremal is

y = x.

Legendre Condition.

Fy0 y0 = 12(y 0 )2 12
 2 !

= 12 1


< 0 for |/| < 1

= 0 for |/| = 1

> 0 for |/| > 1

Thus we see that x may be a minimum for |/| 1 and may be a maximum for |/| 1.
Jacobi Condition. Jacobis accessory equation for this problem is

(F,y0 y0 h0 )0 = 0
 2 ! !0

12 1 h0 = 0

h00 = 0
The problem h00 = 0, h(0) = 0, h(c) = 0 has only the trivial solution for c > 0. Thus we see that there are no
conjugate points and the admissible extremal satisfies the strengthened Legendre condition.

2079
A Weak Minimum. For |/| > 1 the admissible extremal x is a solution of
the Euler equation, and satisfies the strengthened Jacobi and Legendre conditions.
Thus it is a weak minima. (For |/| < 1 it is a weak maxima for the same
reasons.)

Weierstrass Excess Function. The Weierstrass excess function is


E(x, y, y 0 , w) = F (w) F (y 0 ) (w y 0 )F,y0 (y 0 )
= w4 6w2 (y 0 )4 + 6(y 0 )2 (w y 0 )(4(y 0 )3 12y 0 )
 4  2  3
4 2
= w 6w +6 (w )(4 12 )

 2 !  4  2

= w4 6w2 w 4 3 +3 6

We can find the stationary points of the excess function by examining its derivative. (Let = /.)
E 0 (w) = 4w3 12w + 4 ()2 3 = 0


1  1 
w1 = , w2 = 4 2 w3 = + 4 2
2 2
The excess function evaluated at these points is
E(w1 ) = 0,
3 4 2
2 3/2

E(w2 ) = 3 6 6 3(4 ) ,
2
3  
E(w3 ) = 34 62 6 + 3(4 2 )3/2 .
2

E(w2 ) is negative for 1 < < 3 and E(w3 ) is negative for 3 < < 1. This implies that the weak
minimum y = x/ is not a strong local minimum for || < 3|. Since E(w1 ) = 0,we cannot use the
Weierstrass excess function to determine if y = x/ is a strong local minima for |/| > 3.

2080
Cp1 [0, ] Extremals
Erdmanns Corner Conditions. Erdmanns corner conditions require that
F,y0 = 4(y 0 )3 12y 0
and
F y 0 F,y0 = (y 0 )4 6(y 0 )2 y 0 (4(y 0 )3 12y 0 )
are continuous at corners. Thus the quantities
(y 0 )3 3y 0 and (y 0 )4 2(y 0 )2
0 0
are continuous. Denoting p = y and q = y+ , the first condition has the solutions
1 p 
p = q, p = q 3 4 q 2 .
2
The second condition has the solutions,
p
p = q, p = 2 q 2
Combining these, we have
p = q, 3, q = 3, p = 3, q = 3.
p=
0
0

Thus we see that there can be a corner only when y = 3 and y+ = 3.
0
1, = 3. Notice the the Lagrangian is minimized point-wise if y =
Case
3. For this case the unique, strong global minimum is

y = 3 sign()x.

Case 2, || < 3||. For this case there are an infinite number of strong

0 0
minima. Any piecewise linear curve satisfying y (x) = 3 and y+ (x) = 3
and y(0) = 0, y()
= is a strong minima.
Case 3, || > 3||. First note that the extremal cannot have corners. Thus the
unique extremal is y = x. We know that this extremal is a weak local minima.

2081
Solution 48.2
1. Z x1
(a(y 0 )2 + byy 0 + cy 2 ) dx, y(x0 ) = y0 , y(x1 ) = y1 , a 6= 0
x0

Erdmanns First Corner Condition. Fy0 = 2ay 0 + by must be continuous at a corner. This implies that y must
be continuous, i.e., there are no corners.

The functional cannot have broken extremals.

2. Z x1
(y 0 )3 dx, y(x0 ) = y0 , y(x1 ) = y1
x0

Erdmanns First Corner Condition. Fy0 = 3(y 0 )2 must be continuous at a corner. This implies that y
0 0
= y+ .
Erdmanns Second Corner Condition. F y 0 Fy0 = (y 0 )3 y 0 3(y 0 )2 = 2(y 0 )3 must be continuous at a corner.
This implies that y is continuous at a corner, i.e. there are no corners.

The functional cannot have broken extremals.

Solution 48.3
1.
Z 1
1
(y 00 )2 2xy dx, y(0) = y 0 (0) = 0,

y(1) =
0 120
Eulers Differential Equation. We will consider C 4 extremals which satisfy Eulers DE,

(F,y00 )00 (F,y0 )0 + F,y = 0.

For the given Lagrangian, this is,


(2y 00 )00 2x = 0.

2082
Natural Boundary Condition. The first variation of the performance index is
Z 1
J = (F,y y + F,y0 y 0 + Fy00 y 00 ) dx.
0

From the given boundary conditions we have y(0) = y 0 (0) = y(1) = 0. Using Eulers DE, we have,
Z 1
J = ((Fy0 (F,y00 )0 )0 y + F,y0 y 0 + Fy00 y 00 ) dx.
0

Now we apply integration by parts.


h i1 Z 1
0
J = (Fy0 (F,y00 ) )y + ((Fy0 (F,y00 )0 )y 0 + F,y0 y 0 + Fy00 y 00 ) dx
0 0
Z 1
= ((F,y00 )0 y 0 + Fy00 y 00 ) dx
0
h i1
= F,y00 y 0
0
0
= F,y00 (1)y (1)

In order that the first variation vanish, we need the natural boundary condition F,y00 (1) = 0. For the given
Lagrangian, this condition is
y 00 (1) = 0.

The Extremal BVP. The extremal boundary value problem is


1
y 0000 = x, y(0) = y 0 (0) = y 00 (1) = 0, y(1) = .
120
The general solution of the differential equation is
1 5
y = c0 + c1 x + c2 x2 + c3 x3 + x.
120

2083
Applying the boundary conditions, we see that the unique admissible extremal is
x2 3
y = (x 5x + 5).
120
This may be a weak extremum for the problem.
Legendres Condition. Since
F,y00 y00 = 2 > 0,
the strengthened Legendre condition is satisfied.
Jacobis Condition. The second variation for F (x, y, y 00 ) is
Z b
d2 J

00 2 00 2
= F 00
,y y 00 (h ) + 2 F,yy 00 hh + F,yy h dx
d2 =0 a

Jacobis accessory equation is,

(2F,y00 y00 h00 + 2F,yy00 h)00 + 2F,yy00 h00 + 2F,yy h = 0,

(h00 )00 = 0
Since the boundary value problem,

h0000 = 0, h(0) = h0 (0) = h(c) = h00 (c) = 0,

has only the trivial solution for all c > 0 the strengthened Jacobi condition is satisfied.

A Weak Minimum. Since the admissible extremal,

x2 3
y = (x 5x + 5),
120
satisfies the strengthened Legendre and Jacobi conditions, we conclude that it is
a weak minimum.

2084
2. Z 1  
1 0 2
(y ) + yy 0 + y 0 + y dx
0 2

Boundary Conditions. Since no boundary conditions are specified, we have the Euler boundary conditions,

F,y0 (0) = 0, F,y0 (1) = 0.

The derivatives of the integrand are,

F,y = y 0 + 1, F,y0 = y 0 + y + 1.

The Euler boundary conditions are then

y 0 (0) + y(0) + 1 = 0, y 0 (1) + y(1) + 1 = 0.

Erdmanns Corner Conditions. Erdmanns first corner condition specifies that

Fy0 (x) = y 0 (x) + y(x) + 1

must be continuous at a corner. This implies that y 0 (x) is continuous at corners, which means that there are no
corners.
Eulers Differential Equation. Eulers DE is

(F,y0 )0 = Fy ,

y 00 + y 0 = y 0 + 1,
y 00 = 1.
The general solution is
1
y = c0 + c1 x + x2 .
2

2085
The boundary conditions give us the constraints,

c0 + c1 + 1 = 0,
5
c0 + 2c1 + = 0.
2
The extremal that satisfies the Euler DE and the Euler BCs is
1 2 
y = x 3x + 1 .
2

Legendres Condition. Since the strengthened Legendre condition is satisfied,

F,y0 y0 (x) = 1 > 0,

we conclude that the extremal is a weak local minimum of the problem.


Jacobis Condition. Jacobis accessory equation for this problem is,
 0  
F,y0 y0 h0 F,yy (F,yy0 )0 h = 0, h(0) = h(c) = 0,
0
(h0 ) ((1)0 ) h = 0, h(0) = h(c) = 0,
00
h = 0, h(0) = h(c) = 0,
Since this has only trivial solutions for c > 0 we conclude that there are no conjugate points. The extremal
satisfies the strengthened Jacobi condition.

The only admissible extremal,


1 2 
y = x 3x + 1 ,
2
satisfies the strengthened Legendre and Jacobi conditions and is thus a weak
extremum.

2086
3. Z b
(y 2 + 2xyy 0 ) dx, y(a) = A, y(b) = B
a

Eulers Differential Equation. Eulers differential equation,

(F,y0 )0 = Fy ,

(2xy)0 = 2y + 2xy 0 ,
2y + 2xy 0 = 2y + 2xy 0 ,
is trivial. Every C 1 function satisfies the Euler DE.
Erdmanns Corner Conditions. The expressions,

F,y0 = 2xy, F y 0 F,y0 = y 2 + 2xy y 0 y 0 (2xh) = y 2

are continuous at a corner. The conditions are trivial and do not restrict corners in the extremal.
Extremal. Any piecewise smooth function that satisfies the boundary conditions y(a) = A, y(b) = B is an
admissible extremal.
An Exact Derivative. At this point we note that
Z b Z b
2 0 d
(y + 2xyy ) dx = (xy 2 ) dx
a a dx
 2 b
= xy a
= bB 2 aA2 .

The integral has the same value for all piecewise smooth functions y that satisfy the boundary conditions.

Since the integral has the same value for all piecewise smooth functions that satisfy
the boundary conditions, all such functions are weak extrema.

2087
4. Z 1
(xy + y 2 2y 2 y 0 ) dx, y(0) = 1, y(1) = 2
0

Erdmanns Corner Conditions. Erdmanns first corner condition requires F,y0 = 2y 2 to be continuous, which
is trivial. Erdmanns second corner condition requires that

F y 0 F,y0 = xy + y 2 2y 2 y 0 y 0 (2y 2 ) = xy + y 2

is continuous. This condition is also trivial. Thus the extremal may have corners at any point.
Eulers Differential Equation. Eulers DE is

(F,y0 )0 = F,y ,

(2y 2 )0 = x + 2y 4yy 0
x
y=
2

Extremal. There is no piecewise smooth function that satisfies Eulers differential


equation on its smooth segments and satisfies the boundary conditions y(0) = 1,
y(1) = 2. We conclude that there is no weak extremum.

Solution 48.4
1. We require that the first variation vanishes
ZZ

Fu h + Fux hx + Fuy hy dx dy = 0.
D

We rewrite the integrand as


ZZ

Fu h + (Fux h)x + (Fuy h)y (Fux )x h (Fuy )y h dx dy = 0,
D

2088
ZZ ZZ
 
Fu (Fux )x (Fuy )y h dx dy + (Fux h)x + (Fuy h)y dx dy = 0.
D D
Using the Divergence theorem, we obtain,
ZZ Z

Fu (Fux )x (Fuy )y h dx dy + (Fux , Fuy ) n h ds = 0.
D

In order that the line integral vanish we have the natural boundary condition,

(Fux , Fuy ) n = 0 for (x, y) .

We can also write this as


dy dx
Fux Fuy = 0 for (x, y) .
ds ds
The Euler differential equation for this problem is

Fu (Fux )x (Fuy )y = 0.

2. We consider the natural boundary conditions for


ZZ Z
F (x, y, u, ux , uy ) dx dy + G(x, y, u) ds.
D

We require that the first variation vanishes.


ZZ Z Z

Fu (Fux )x (Fuy )y h dx dy + (Fux , Fuy ) n h ds + Gu h ds = 0,
D
ZZ Z
 
Fu (Fux )x (Fuy )y h dx dy + (Fux , Fuy ) n + Gu h ds = 0,
D
In order that the line integral vanishes, we have the natural boundary conditions,

(Fux , Fuy ) n + Gu = 0 for (x, y) .

2089
For the given integrand this is,

(2pux , 2puy ) n + 2u = 0 for (x, y) ,

pu n + u = 0 for (x, y) .
We can also denote this as
u
p + u = 0 for (x, y) .
n
Solution 48.5
First we vary .
!
ZZ Z h(x,t)  
1 1
() = t + t + (x + x )2 + (y + y )2 + gy dy dx dt
R 0 2 2
!
ZZ Z h(x,t)
0 (0) = (t + x x + y y ) dy dx dt = 0
R 0

ZZ  Z h(x,t) Z h(x,t) Z h(x,t)


0
(0) = dy [ht ]y=h(x,t) + x dy [x hx ]y=h(x,t) xx dy
R t 0 x 0 0
Z h(x,t) 
h(x,t)
+ [y ]0 yy dy dx dt = 0
0

Since vanishes on the boundary of R, we have


ZZ  Z h(x,t) 
0
(0) = [(ht x hx y )]y=h(x,t) [y ]y=0 (xx + yy ) dy dx dt = 0.
R 0

From the variations which vanish on y = 0, h(x, t) we have

2 = 0.

2090
This leaves us with ZZ  
0
(0) = [(ht x hx y )]y=h(x,t) [y ]y=0 dx dt = 0.
R
By considering variations which vanish on y = 0 we obtain,
ht x hx y = 0 on y = h(x, t).
Finally we have
y = 0 on y = 0.
Next we vary h(x, t).
ZZ Z h(x,t)+(x,t)  
1 2 1 2
() = t + x + y + gy dx dt
R 0 2 2
ZZ  
1 1
0 () = t + 2x + 2y + gy dx dt = 0
R 2 2 y=h(x,t)
This gives us the boundary condition,
1 1
t + 2x + 2y + gy = 0 on y = h(x, t).
2 2
Solution 48.6
The parts of the extremizing curve which lie outside the boundary of the region R must be extremals, (i.e., solutions of
Eulers equation) since if we restrict our variations to admissible curves outside of R and its boundary, we immediately
obtain Eulers equation. Therefore an extremum can be reached only on curves consisting of arcs of extremals and
parts of the boundary of region R.
Thus, our problem is to find the points of transition of the extremal to the boundary of R. Let the boundary of R
be given by (x). Consider an extremum that starts at the point (a, A), follows an extremal to the point (x0 , (x0 )),
follows the R to (x1 , (x1 )) then follows an extremal to the point (b, B). We seek transversality conditions for the
points x0 and x1 . We will extremize the expression,
Z x0 Z x1 Z b
0 0
I(y) = F (x, y, y ) dx + F (x, , ) dx + F (x, y, y 0 ) dx.
a x0 x1

2091
Let c be any point between x0 and x1 . Then extremizing I(y) is equivalent to extremizing the two functionals,
Z x0 Z c
0
I1 (y) = F (x, y, y ) dx + F (x, , 0 ) dx,
a x0

Z x1 Z b
0
I2 (y) = F (x, , ) dx + F (x, y, y 0 ) dx,
c x1

I = 0 I1 = I2 = 0.
We will extremize I1 (y) and then use the derived transversality condition on all points where the extremals meet R.
The general variation of I1 is,
Z x0  
d x
I1 (y) = Fy Fy0 dx + [Fy0 y]xa0 + [(F y 0 Fy0 )x]a0
a dx
c
+ [F0 (x)]cx0 + [(F 0 F0 )x]x0 = 0

Note that x = y = 0 at x = a, c. That is, x = x0 is the only point that varies. Also note that (x) is not
independent of x. (x) 0 (x)x. At the point x0 we have y 0 (x)x.
Z x0  
d 0
0

I1 (y) = Fy Fy0 dx + (Fy0 x) + ((F y Fy0 )x)
a dx
x0 x0

(F0 0 x) ((F 0 F0 )x) = 0



x0 x0

x0
Z  
d 0 0 0
0
I1 (y) = Fy Fy0 dx + ((F (x, y, y ) F (x, , ) + ( y )Fy0 )x) =0
a dx x0

Since I1 vanishes for those variations satisfying x0 = 0 we obtain the Euler differential equation,

d
Fy Fy0 = 0.
dx

2092
Then we have
((F (x, y, y 0 ) F (x, , 0 ) + (0 y 0 )Fy0 )x)

=0
x0

for all variations x0 . This implies that



0 0 0 0

(F (x, y, y ) F (x, , ) + ( y )Fy0 ) = 0.
x0

Two solutions of this equation are


y 0 (x0 ) = 0 (x0 ) and Fy0 = 0.

Transversality condition. If Fy0 is not identically zero, the extremal must be


tangent to R at the points of contact.
R 10
Now we apply this result to to find the curves which extremize 0 (y 0 )3 dx, y(0) = 0, y(10) = 0 given that the
admissible curves can not penetrate the interior of the circle (x 5)2 + y 2 = 9. Since the Lagrangian is a function of
y 0 alone, the extremals are straight lines.
The Erdmann corner conditions require that

Fy0 = 3(y 0 )2 and F y 0 Fy0 = (y 0 )3 y 0 3(y 0 )2 = 2(y 0 )3

are continuous at corners. This implies that y 0 is continuous. There are no corners.
We see that the extrema are

3
p
4
x, for 0 x 16
5
,
16 34
y(x) = 9 (x 5)2 , for 5 x 5 ,

3
4 x, for 34
5
x 10.

Note that the extremizing curves neither minimize nor maximize the integral.

2093
Solution 48.7
C1 Extremals. Without loss of generality, we take the vertical line to be the y axis. We will consider x1 , y1 > 1. With
p
ds = 1 + (y 0 )2 dx we extremize the integral,
Z x1
p
y 1 + (y 0 )2 dx.
0

Since the Lagrangian is independent of x, we know that the Euler differential equation has a first integral.
d
Fy0 Fy = 0
dx
y 0 Fy0 y + y 00 Fy0 y0 Fy = 0
d 0
(y Fy0 F ) = 0
dx
y 0 Fy0 F = const
For the given Lagrangian, this is
y0 p
y0 y p y 1 + (y 0 )2 = const,
1 + (y 0 )2
p
(y 0 )2 y y(1 + (y 0 )2 ) = const 1 + (y 0 )2 ,
p
y = const 1 + (y 0 )2
y = const is one solution. To find the others we solve for y 0 and then solve the differential equation.

y = a(1 + (y 0 )2 )
r
ya
y0 =
a
r
a
dx = dy
ya

2094
p
x + b = 2 a(y a)
x2 bx b2
y= + +a
4a 2a 4a
The natural boundary condition is
0
yy
Fy0 x=0 = p
= 0,
1 + (y 0 )2 x=0

y 0 (0) = 0
The extremal that satisfies this boundary condition is

x2
y= + a.
4a
Now we apply y(x1 ) = y1 to obtain
 
1
q
a= y1 y12 x21
2
for y1 x1 . The value of the integral is
Z s x1
x1 (x21 + 12a2 )
  x 2 
x2
+a 1+ dx = .
0 4a 2a 12a3/2

By denoting y1 = cx1 , c 1 we have


1 
a= cx1 x1 c2 1
2
The values of the integral for these two values of a are

2 2
3/2 1 + 3c 3c c 1
2(x1 ) .
3(c c2 1)3/2

2095

The values are equal only when c = 1. These values, (divided by x1 ), are plotted in Figure 48.1 as a function of c.
The former and latter are fine and coarse dashed lines, respectively. The extremal with
 
1
q
2 2
a= y1 + y1 x1
2

has the smaller performance index. The value of the integral is


p
x1 (x21 + 3(y1 + y12 x21 )2
p .
3 2(y1 + y12 x21 )3

The function y = y1 is an admissible extremal for all x1 . The value of the integral for this extremal is x1 y1 which
is larger than the integral of the quadratic we analyzed before for y1 > x1 .

3.5

2.5

1.2 1.4 1.6 1.8 2

Figure 48.1:

Thus we see that


x2
 
1
q
2 2
y = + a, a= y1 + y1 x1
4a 2

2096
is the extremal with the smaller integral and is the minimizing curve in C 1 for y1 x1 . For y1 < x1 the C 1 extremum
is,
y = y1 .
C1p Extremals. Consider the parametric form of the Lagrangian.
Z t1 p p
y(t) (x0 (t))2 + (y 0 (t))2 dt
t0

The Euler differential equations are


d d
fx0 fx = 0 and fy0 fy = 0.
dt dt
If one of the equations is satisfied, then the other is automatically satisfied, (or the extremal is straight). With either
of these equations we could derive the quadratic extremal and the y = const extremal that we found previously. We
will find one more extremal by considering the first parametric Euler differential equation.
d
fx0 fx = 0
dt
p !
d y(t)x0 (t)
p =0
dt (x0 (t))2 + (y 0 (t))2
p
y(t)x0 (t)
p = const
(x0 (t))2 + (y 0 (t))2
Note that x(t) = const is a solution. Thus the extremals are of the three forms,

x = const,
y = const,
x2 bx b2
y= + + + a.
4a 2a 4a

2097
The Erdmann corner conditions require that

yy 0
Fy0 = p ,
1 + (y 0 )2
0 2
0 p 0 2
y(y ) y
F y Fy = y 1 + (y )
0 p = p
1 + (y 0 )2 1 + (y 0 )2
are continuous at corners. There can be corners only if y = 0.
Now we piece the three forms together to obtain Cp1 extremals that satisfy the Erdmann corner conditions. The
only possibility that is not C 1 is the extremal that is a horizontal line from (0, 0) to (x1 , 0) and then a vertical line from
(x1 , y1 ). The value of the integral for this extremal is
Z y1
2
t dt = (y1 )3/2 .
0 3
Equating the performance indices of the quadratic extremum and the piecewise smooth extremum,
p
x1 (x21 + 3(y1 + y12 x21 )2 2
p = (y1 )3/2 ,
2
3 2(y1 + y1 x1 ) 2 3 3
p
32 3
y1 = x1 .
3
The only real positive solution is p
3+2 3
y1 = x1 1.46789 x1 .
3
The piecewise smooth extremal has the smaller performance index for y1 smaller than this value and the quadratic
extremal has the smaller performance index for y1 greater than this value.
p
The Cp1 extremum is the piecewise smooth extremal for y1 x1 3 + 2 3/ 3
p
and is the quadratic extremal for y1 x1 3 + 2 3/ 3.

2098
Solution 48.8
The shape of the rope will be a catenary between x1 and x2 and be a vertically hanging segment after that. Let the
length of the vertical segment be z. Without loss of generality we take x1 = y2 = 0. The potential energy, (relative to
y = 0), of a length of rope ds in 0 x x2 is mgy = gy ds. The total potential energy of the vertically hanging
rope is m(center of mass)g = z(z/2)g. Thus we seek to minimize,
Z x2
1
g y ds gz 2 , y(0) = y1 , y(x2 ) = 0,
0 2

subject to the isoperimetric constraint, Z x2


ds z = L.
0
p
Writing the arc-length differential as ds = 1 + (y 0 )2 dx we minimize
Z x2 p 1
g y 1 + (y 0 )2 ds gz 2 , y(0) = y1 , y(x2 ) = 0,
0 2

subject to, Z x2 p
1 + (y 0 )2 dx z = L.
0
Rb
Consider the more general problem of finding functions y(x) and numbers z which extremize I a F (x, y, y 0 ) dx+
Rb
f (z) subject to J a G(x, y, y 0 ) dx + g(z) = L.
Suppose y(x) and z are the desired solutions and form the comparison families, y(x) + 1 1 (x) + 2 2 (x), z + 1 1 +
2 2 . Then, there exists a constant such that


(I + J) 1 ,2 =0 = 0
1

(I + J) 1 ,2 =0 = 0.
2

2099
These equations are Z b 
d
H,y0 Hy 1 dx + h0 (z)1 = 0,
a dx
and Z b 
d
H,y0 Hy 2 dx + h0 (z)2 = 0,
a dx
where H = F + G and h = f + g. From this we conclude that
d
H,y0 Hy = 0, h0 (z) = 0
dx
with determined by Z b
J= G(x, y, y 0 ) dx + g(z) = L.
a

Now we apply these results to our problem. Since f (z) = 21 gz 2 and g(z) = z we have
gz = 0,


z= .
g
It was shown in class that the solution of the Euler differential equation is a family of catenaries,
 
x c2
y = + c1 cosh .
g c1
One can find c1 and c2 in terms of by applying the end conditions y(0) = y1 and y(x2 ) = 0. Then the expression for
y(x) and z = /g are substituted into the isoperimetric constraint to determine .
Consider the special case that (x1 , y1 ) = (0, 0) and (x2 , y2 ) = (1, 0). In this case we can use the fact that
y(0) = y(1) to solve for c2 and write y in the form
 
x 1/2
y = + c1 cosh .
g c1

2100
Applying the condition y(0) = 0 would give us the algebraic-transcendental equation,
 
1
y(0) = + c1 cosh = 0,
g 2c1

which we cant solve in closed form. Since we ran into a dead end in applying the boundary condition, we turn to the
isoperimetric constraint. Z 1p
1 + (y 0 )2 dx z = L
0
1  
x 1/2
Z
cosh dx z = L
0 c1
 
1
2c1 sinh z =L
2c1
With the isoperimetric constraint, the algebraic-transcendental equation and z = /g we now have
 
1
z = c1 cosh ,
2c1
 
1
z = 2c1 sinh L.
2c1

For any fixed L, we can numerically solve for c1 and thus obtain z. You can derive that there are no solutions unless
L is greater than about 1.9366. If L is smaller than this, the rope would slip off the pin. For L = 2, c1 has the values
0.4265 and 0.7524. The larger value of c1 gives the smaller potential energy. The position of the end of the rope is
z = 0.9248.
Solution 48.9 Rc
Using the method of Lagrange multipliers, we look for stationary values of 0 ((y 0 )2 + y 2 ) dx,
Z c
((y 0 )2 + y 2 ) dx = 0.
0

2101
The Euler differential equation is
d
F( , y 0 ) F,y = 0,
dx
d
(2y 0 ) 2y = 0.
dx
Together with the homogeneous boundary conditions, we have the problem

y 00 y = 0, y(0) = y(c) = 0,

which has the solutions,


 n 2  nx 
n = , ,
yn = an sin n Z+ .
c c
Now we determine the constants an with the moment of inertia constraint.
Z c  nx  ca2
a2n sin2 dx = n = A
0 c 2

Thus we have the extremals, r


2A  nx 
yn = sin , n Z+ .
c c
The drag for these extremals is
Z c  2
2A n 2 nx
  An2 2
D= cos dx = .
c 0 c c c2

We see that the drag is minimum for n = 1. The shape for minimum drag is
r
2A  nx 
y = sin .
c c

2102
Solution 48.10
Consider the general problem of determining the stationary values of the quantity 2 given by
Rb
2 a
F (x, y, y 0 , y 00 ) dx I
= Rb .
0 00
G(x, y, y , y ) dx J
a

The variation of 2 is
JI IJ
2 = 2
J 
1 I
= I J
J J
1
I 2 J .

=
J
The the values of y and y 0 are specified on the boundary, then the variations of I and J are
Z b 2  Z b 2 
d d d d
I = F,y00 F,y0 + F,y y dx, J = G,y00 G,y0 + G,y y dx
a dx2 dx a dx2 dx
Thus 2 = 0 becomes R b  d2 
d
a dx2
H,y00 H 0
dx ,y
+ H,y y dx
Rb = 0,
a
G dx
where H = F 2 G. A necessary condition for an extremum is

d2 d
2
H,y00 H,y0 + H,y = 0 where H F 2 G.
dx dx
For our problem we have F = EI(y 00 )2 and G = y so that the extremals are solutions of
d2
 
dy
2
EI 2 y = 0,
dx dx

2103
With homogeneous boundary conditions we have an eigenvalue problem with deflections modes yn (x) and corresponding
natural frequencies n .

Solution 48.11
We assume that v0 > w(x, y, t) so that the problem has a solution for any end point. The crossing time is
Z l
1 l
1 Z
T = X(t) dx = sec (t) dx.
0 v0 0

Note that
dy w + v0 sin
=
dx v0 cos
w
= sec + tan
v0
w
= sec + sec2 1.
v0

We solve this relation for sec .  2


0w
y sec = sec2 1
v0

w 0 w2
(y 0 )2 2 y sec + 2 sec2 = sec2 1
v0 v0

(v02 w2 ) sec2 + 2v0 wy 0 sec v02 ((y 0 )2 + 1) = 0


p
2v0 wy 0 4v02 w2 (y 0 )2 + 4(v02 w2 )v02 ((y 0 )2 + 1)
sec =
2(v02 w2 )
p
wy 0 v02 ((y 0 )2 + 1) w2
sec = v0
(v02 w2 )

2104
Since the steering angle satisfies /2 /2 only the positive solution is relevant.
p
wy 0 + v02 ((y 0 )2 + 1) w2
sec = v0
(v02 w2 )
Time Independent Current. If we make the assumption that w = w(x, y) then we can write the crossing time
as an integral of a function of x and y.
Z l p
wy 0 + v02 ((y 0 )2 + 1) w2
T (y) = dx
0 (v02 w2 )
A necessary condition for a minimum is T = 0. The Euler differential equation for this problem is
d
F,y0 F,y = 0
dx
!! !
d 1 v02 y 0 wy w(v 2 (1 + 2(y 0 )2 ) w2 )
2
w + p 2 2 p y 0 (v02 + w2 )
dx v0 w 2 v0 ((y 0 )2 + 1) w2 (v0 w2 )2 2 0 2
v0 ((y ) + 1) w 2

By solving this second order differential equation subject to the boundary conditions y(0) = 0, y(l) = y1 we obtain the
path of minimum crossing time.
Current w = w(x). If the current is only a function of x, then the Euler differential equation can be integrated to
obtain, !
1 v02 y 0
w + p 2 = c0 .
v02 w2 v0 ((y 0 )2 + 1) w2
Solving for y 0 ,
w + c0 (v02 w2 )
y0 = p .
v0 1 2c0 w c20 (v02 w2 )
Since y(0) = 0, we have
x
w() + c0 (v02 (w())2 )
Z
y(x) = p .
0 v0 1 2c0 w() c20 (v02 (w())2 )

2105
For any given w(x) we can use the condition y(l) = y1 to solve for the constant c0 .
Constant Current. If the current is constant then the Lagrangian is a function of y 0 alone. The admissible
extremals are straight lines. The solution is then
y1 x
y(x) = .
l
Solution 48.12
1. The kinetic energy of the first particle is 21 m(( x))2 . Its potential energy, relative to the table top, is zero.
The kinetic energy of the second particle is 21 mx2 . Its potential energy, relative to its equilibrium position is
mgx. The Lagrangian is the difference of kinetic and potential energy.
 
2 1 2 2
L = m x + ( x) + gx
2

The Euler differential equations are the equations of motion.


d d
L,x Lx = 0,L L = 0
dt dt ,
d 2 d  2 2

(2mx) + m( x) mg = 0, m( x) = 0
dt dt
2x + ( x)2 g = 0, ( x)2 2 = const
p
When x = 0, = = g/. This determines the constant in the equation of motion for .

g
=
( x)2

Now we substitute the expression for into the equation of motion for x.
3 g
2x + ( x) g =0
( x)4

2106
3
 
2x + 1 g =0
( x)3
 
1
2x + 1 g =0
(1 x/)3

2. For small oscillations, x  1. Recall the binomial expansion,
 
a
X a
(1 + z) = zn, for |z| < 1,
n=0
n

(1 + z)a 1 + az, for |z|  1.


We make the approximation,
1 x
1 + 3 ,
(1 x/)3
to obtain the linearized equation of motion,
3g
2x +
x = 0.

This is the equation of a harmonic oscillator with solution
p 
x = a sin 3g2(t b) .

The period of oscillation is,



T = 2 23g.

Solution 48.13
We write the equation of motion and boundary conditions,

x = U (t) g, x(0) = x(0) = 0, x(T ) = h,

2107
as the first order system,

x = 0, x(0) = 0, x(T ) = h,
y = U (t) g, y(0) = 0.

We seek to minimize, Z T
T = dt,
0

subject to the constraints,

x y = 0,
y U (t) + g = 0,
Z T
U 2 (t) dt = k 2 .
0

Thus we seek extrema of


Z T Z T
1 + (t)(x y) + (t)(y U (t) + g) + U 2 (t) dt.

H dt
0 0

Since y is not specified at t = T , we have the natural boundary condition,



H,y t=T = 0,

(T ) = 0.
The first Euler differential equation is
d
H,x H,x = 0,
dt
d
(t) = 0.
dt

2108
We see that (t) = is constant. The next Euler DE is

d
H,y H,y = 0,
dt
d
(t) + = 0.
dt
(t) = t + const
With the natural boundary condition, (T ) = 0, we have

(t) = (T t).

The final Euler DE is,


d
H H,U = 0,
dt ,U
(t) 2U (t) = 0.
Thus we have
(T t)
U (t) = .
2
This is the required thrust function. We use the constraints to find , and
R T T.
Substituting U (t) = (T t)/(2) into the isoperimetric constraint, 0 U 2 (t) dt = k 2 yields

2 T 3
= k2,
12 2

3k
U (t) = 3/2 (T t).
T
The equation of motion for x is
3k
x = U (t) g = (T t).
T 3/2

2109
Integrating and applying the initial conditions x(0) = x(0) = 0 yields,
kt2 (3T t) 1 2
x(t) = gt .
2 3T 3/2 2
Applying the condition x(T ) = h gives us,
k 1
T 3/2 gT 2 = h,
3 2
1 2 4 k 3
g T T + ghT 2 + h2 = 0.
4 3
p
If k 4 2/3g 3/2 h then this fourth degree polynomial has positive, realp solutions
for T . With strict inequality, the
3/2
minimum time is the smaller of the two positive, real solutions. If k < 4 2/3g h then there is not enough fuel to
reach the target height.
Solution 48.14
We have x = U (t) where U (t) is the acceleration furnished by the thrust of the vehicles engine. In practice, the engine
will be designed to operate within certain bounds, say M U (t) M , where M is the maximum forward/backward
acceleration. To account for the inequality constraint we write U = M sin V (t) for some suitable V (t). More generally,
if we had (t) U (t) (t), we could write this as U (t) = +2
+
2
sin V (t).
We write the equation of motion as a first order system,
x = y, x(0) = a, x(T ) = 0,
y = M sin V, y(0) = b, y(T ) = 0.
Thus we minimize Z T
T = dt
0
subject to the constraints,
x y = 0
y M sin V = 0.

2110
Consider
H = 1 + (t)(x y) + (t)(y M sin V ).
The Euler differential equations are

d d
H,x H,x = 0 (t) = 0 (t) = const
dt dt
d d
H,y H,y = 0 (t) + = 0 (t) = t + const
dt dt
d
H,V H,V = 0 (t)M cos V (t) = 0 V (t) = + n.
dt 2
Thus we see that  
U (t) = M sin + n = M.
2
Therefore, if the rocket is to be transferred from its initial state to is specified final state in minimum time with
a limited source of thrust, (|U | M ), then the engine should operate at full power at all times except possibly for a
finite number of switching times. (Indeed, if some power were not being used, we would expect the transfer would be
speeded up by using the additional power suitably.)
To see how this bang-bang process works, well look at the phase plane. The problem

x = y, x(0) = c,
y = M, y(0) = d,

has the solution


t2
x(t) = c + dt M , y(t) = d M t.
2
We can eliminate t to get
y2 d2
x= +c .
2M 2M
These curves are plotted in Figure 48.2.

2111
Figure 48.2:

There is only curve in each case which transfers the initial state to the origin. We will denote these curves and ,
respectively. Only if the initial point (a, b) lies on one of these two curves can we transfer the state of the system to the
b2 b2
origin along an extremal without switching. If a = 2M and b < 0 then this is possible using U (t) = M . If a = 2M
and b > 0 then this is possible using U (t) = M . Otherwise we follow an extremal that intersects the initial position
until this curve intersects or . We then follow or to the origin.
Solution 48.15
Since the integrand does not explicitly depend on x, the Euler differential equation has the first integral,
F y 0 Fy0 = const.
0

0 y y+h
p p
0 2
y + h 1 + (y ) y p = const
1 + (y 0 )2

y+h
p = const
1 + (y 0 )2
y + h = c21 (1 + (y 0 )2 )

2112
q
y + h c21 = c1 y 0
c1 dy
p = dx
y + h c21
q
2c1 y + h c21 = x c2
4c21 (y + h c21 ) = (x c2 )2
Since the extremal passes through the origin, we have

4c21 (h c21 ) = c22 .

4c21 y = x2 2c2 x (48.6)


Introduce as a parameter the slope of the extremal at the origin; that is, y 0 (0) = . Then differentiating (48.6) at
h 2h
x = 0 yields 4c21 = 2c2 . Together with c22 = 4c21 (h c21 ) we obtain c21 = 1+ 2 and c2 = 1+2 . Thus the equation

of the pencil (48.6) will have the form


1 + 2 2
y = x + x. (48.7)
4h
2
To find the envelope of this family we differentiate ( 48.7) with respect to to obtain 0 = x + 2h x and eliminate
between this and ( 48.7) to obtain
x2
y = h + .
4h
See Figure 48.3 for a plot of some extremals and the envelope.
All extremals (48.7) lie above the envelope which in ballistics is called the parabola of safety. If (m, M ) lies outside
2
the parabola, M < h + m 4h
, then it cannot be joined to (0, 0) by an extremal. If (m, M ) is above the envelope
then there are two candidates. Clearly we rule out the one that touches the envelope because of the occurrence of
conjugate points. For the other extremal, problem 2 shows that E 0 for all y 0 . Clearly we can embed this extremal
in an extremal pencil, so Jacobis test is satisfied. Therefore the parabola that does not touch the envelope is a strong
minimum.

2113
y

x
h 2h

-h

Figure 48.3: Some Extremals and the Envelope.

Solution 48.16

E = F (x, y, y 0 ) F (x, y, p) (y 0 p)Fy0 (x, y, p)


p p np
= n 1 + (y 0 )2 n 1 + p2 (y 0 p) p
1 + p2
n  p p 
=p 1 + (y 0 )2 1 + p2 (1 + p2 ) (y 0 p)p
1 + p2
n p 
=p 1 + (y 0 )2 + p2 + (y 0 )2 p2 2y 0 p + 2y 0 p (1 + py 0 )
1 + p2
n p 
=p (1 + py 0 )2 + (y 0 p)2 (1 + py 0 )
1 + p2
0

2114
ds 1
The speed of light in an inhomogeneous medium is dt
= n(x,y
. The time of transit is then
Z (b,B) Z b
dt p
T = ds = n(x, y) 1 + (y 0 )2 dx.
(a,A) ds a

Since E 0, light traveling on extremals follow the time optimal path as long as the extremals do not intersect.
Solution 48.17
Extremals. Since the integrand does not depend explicitly on x, the Euler differential equation has the first integral,

F y 0 F,y0 = const.

1 + y2 0 2(1 + y )
2
y = const
(y 0 )2 (y 0 )3
dy
p = const dx
1 + (y 0 )2
arcsinh(y) = c1 x + c2
y = sinh(c1 x + c2 )
Jacobi Test. We can see by inspection that no conjugate points exist. Consider the central field through (0, 0),
sinh(cx), (See Figure 48.4).
We can also easily arrive at this conclusion analytically as follows: Solutions u1 and u2 of the Jacobi equation are
given by
y
u1 = = cosh(c1 x + c2 ),
c2
y
u2 = = x cosh(c1 x + c2 ).
c1
Since u2 /u1 = x is monotone for all x there are no conjugate points.

2115
3

-3 -2 -1 1 2 3

-1

-2

-3

Figure 48.4: sinh(cx)

Weierstrass Test.

E = F (x, y, y 0 ) F (x, y, p) (y 0 p)F,y0 (x, y, p)


1 + y2 1 + y2 0 2(1 + y 2 )
= (y p)
(y 0 )2 p2 p3
1 + y 2 p3 p(y 0 )2 + 2(y 0 )3 2p(y 0 )2
 
= 0 2 2
(y ) p p
2
(p y ) (p + 2y 0 )
0 2
 
1+y
= 0 2 2
(y ) p p

For p = p(x, y) bounded away from zero, E is one-signed for values of y 0 close to p. However, since the factor (p + 2y 0 )
can have any sign for arbitrary values of y 0 , the conditions for a strong minimum are not satisfied.
Furthermore, since the extremals are y = sinh(c1 x + c2 ), the slope function p(x, y) will be of one sign only if the
range of integration is such that we are on a monotonic piece of the sinh. If we span both an increasing and decreasing
section, E changes sign even for weak variations.

2116
Legendre Condition.
6(1 + y 2 )
F,y0 y0 = >0
(y 0 )4
Note that F cannot be represented in a Taylor series for arbitrary values of y 0 due to the presence of a discontinuity in
F when y 0 = 0. However, F,y0 y0 > 0 on an extremal implies a weak minimum is provided by the extremal.
2
Strong Variations. Consider 1+y
R
(y 0 )2
dx on both an extremal and on the special piecewise continuous variation in
2
the figure. On P Q we have y 0 = with implies that 1+y (y 0 )2
= 0 so that there is no contribution to the integral from
P Q.
On QR the value of y 0 is greater than its value along the extremal P R while the value of y on QR is less than the
2
value of y along P R. Thus on QR the quantity 1+y (y 0 )2
is less than it is on the extremal P R.

1 + y2 1 + y2
Z Z
dx < dx
QR (y 0 )2 PR (y 0 )2

Thus the weak minimum along the extremal can be weakened by a strong variation.
Solution 48.18
The Euler differential equation is
d
F,y0 F,y = 0.
dx
d
(1 + 2x2 y 0 ) = 0
dx
1 + 2x2 y 0 = const
1
y 0 = const 2
x
c1
y= + c2
x
(i) No continuous extremal exists in 1 x 2 that satisfies y(1) = 1 and y(2) = 4.

2117
(ii) The continuous extremal that satisfies the boundary conditions is y = 7 x4 . Since F,y0 y0 = 2x2 0 has a Taylor
series representation for all y 0 , this extremal provides a strong minimum.

(iii) The continuous extremal that satisfies the boundary conditions is y = 1. This is a strong minimum.

Solution 48.19
For identity (a) we take P = 0 and Q = x x . For identity (b) we take P = y y and Q = 0. For identity
(c) we take P = 12 (x x ) and Q = 21 (y y ).
ZZ     Z  
1 1 1 1
(y y )x (x x )y dx dy = (x x ) dx + (y y ) dy
D 2 2 2 2

ZZ  
1 1
(x y + xy x y xy ) + (y x xy y x xy ) dx dy
D 2 2
Z Z
1 1
= (x x ) dx + (y y ) dy
2 2
ZZ ZZ Z Z
1 1
xy dx dy = xy dx dy (x x ) dx + (y y ) dy
D D 2 2
The variation of I is
Z t1 Z Z
I = (2(uxx + uyy )(uxx + uyy ) + 2(1 )(uxx uyy + uyy uxx 2uxy uxy )) dx dy dt.
t0 D

From (a) we have


ZZ ZZ
2(uxx + uyy )uxx dx dy = 2(uxx + uyy )xx u dx dy
D D Z
+ 2((uxx + uyy )ux (uxx + uyy )x u) dy.

2118
From (b) we have
ZZ ZZ
2(uxx + uyy )uyy dx dy = 2(uxx + uyy )yy u dx dy
D D Z
2((uxx + uyy )uy (uxx + uyy )y u) dy.

From (a) and (b) we get
ZZ
2(1 )(uxx uyy + uyy uxx ) dx dy
D
ZZ
= 2(1 )(uxxyy + uyyxx )u dx dy
DZ

+ 2(1 )((uxx uy uxxy u) dx + (uyy ux uyyx u) dy).



Using c gives us
ZZ ZZ
2(1 )(2uxy uxy ) dx dy = 2(1 )(2uxyxy u) dx dy
D DZ

+ 2(1 )(uxy ux uxyx u) dx


Z
2(1 )(uxy uy uxyy u) dy.

Note that
u
ds = ux dy uy dx.
n
Using the above results, we obtain
Z t1 Z Z Z t1 Z 
(2 u)

4 2 (u)
I = 2 ( u)u dx dy dt + 2 u + ( u) ds dt
t0 D t0 n n
Z t1 Z 
+ 2(1 ) (uyy ux uxy uy ) dy + (uxy ux uxx uy ) dx dt.
t0

2119
Solution 48.20
1. Exact Solution. The Euler differential equation is
d
F,y0 = F,y
dx
d
[2y 0 ] = 2y 2x
dx
y 00 + y = x.

The general solution is

y = c1 cos x + c2 sin x x.

Applying the boundary conditions we obtain,

sin x
y= x.
sin 1
The value of the integral for this extremal is
 
sin x 2
J x = cot(1) 0.0245741.
sin 1 3

n = 0. We consider an approximate solution of the form y(x) = ax(1x). We substitute this into the functional.
Z 1
3 1
(y 0 )2 y 2 2xy dx = a2 a

J(a) =
0 10 6
The only stationary point is
3 1
J 0 (a) = a = 0
5 6
5
a= .
18

2120
Since  
00 5 3
J = > 0,
18 5
we see that this point is a minimum. The approximate solution is

5
y(x) = x(1 x).
18
This one term approximation and the exact solution are plotted in Figure 48.5. The value of the functional is

5
J = 0.0231481.
216

0.07

0.06

0.05

0.04

0.03

0.02

0.01

0.2 0.4 0.6 0.8 1

Figure 48.5: One Term Approximation and Exact Solution.

n = 1. We consider an approximate solution of the form y(x) = x(1 x)(a + bx). We substitute this into the
functional. Z 1
1
(y 0 )2 y 2 2xy dx = 63a2 + 63ab + 26b2 35a 21b
 
J(a, b) =
0 210

2121
We find the stationary points.
1
Ja = (18a + 9b 5) = 0
30
1
Jb = (63a + 52b 21) = 0
210
71 7
a= , b=
369 41
Since the Hessian matrix  3 3
 
Jaa Jab 5 10
H= = 3 26 ,
Jba Jbb 10 105
is positive definite,
3 41
> 0, det(H) = ,
5 700
we see that this point is a minimum. The approximate solution is
 
71 7
y(x) = x(1 x) + x .
369 41

This two term approximation and the exact solution are plotted in Figure 48.6. The value of the functional is
136
J = 0.0245709.
5535

2. Exact Solution. The Euler differential equation is


d
F,y0 = F,y
dx
d
[2y 0 ] = 2y + 2x
dx
y 00 y = x.

2122
0.07

0.06

0.05

0.04

0.03

0.02

0.01

0.2 0.4 0.6 0.8 1

Figure 48.6: Two Term Approximation and Exact Solution.

The general solution is

y = c1 cosh x + c2 sinh x x.

Applying the boundary conditions, we obtain,

2 sinh x
y= x.
sinh 2
The value of the integral for this extremal is

2(e4 13)
J = 0.517408.
3(e4 1)

Polynomial Approximation. Consider an approximate solution of the form

y(x) = x(2 x)(a0 + a1 x + an xn ).

2123
The one term approximate solution is
5
y(x) = x(2 x).
14
This one term approximation and the exact solution are plotted in Figure 48.7. The value of the functional is

10
J = 0.47619.
21

0.5 1 1.5 2
-0.05

-0.1

-0.15

-0.2

-0.25

-0.3

-0.35

Figure 48.7: One Term Approximation and Exact Solution.

The two term approximate solution is


 
33 7
y(x) = x(2 x) x .
161 46
This two term approximation and the exact solution are plotted in Figure 48.8. The value of the functional is
416
J = 0.51677.
805

2124
0.5 1 1.5 2
-0.05

-0.1

-0.15

-0.2

-0.25

-0.3

-0.35

Figure 48.8: Two Term Approximation and Exact Solution.

Sine Series Approximation. Consider an approximate solution of the form


 x   x 
y(x) = a1 sin + a2 sin (x) + + an sin n .
2 2
The one term approximate solution is
16  x 
y(x) = sin .
( 2 + 4) 2
This one term approximation and the exact solution are plotted in Figure 48.9. The value of the functional is
64
J = 0.467537.
2 ( 2
+ 4)

The two term approximate solution is


16  x  2
y(x) = 2
sin + 2
sin(x).
( + 4) 2 ( + 1)

2125
0.5 1 1.5 2
-0.05

-0.1

-0.15

-0.2

-0.25

-0.3

-0.35

Figure 48.9: One Term Sine Series Approximation and Exact Solution.

This two term approximation and the exact solution are plotted in Figure 48.10. The value of the functional is
4(17 2 + 20)
J = 0.504823.
2 ( 4 + 5 2 + 4)

3. Exact Solution. The Euler differential equation is


d
F,y0 = F,y
dx
d x2 1
[2xy 0 ] = 2 y 2x2
dx  x 
00 1 0 1
y + y + 1 2 y = x
x x

The general solution is

y = c1 J1 (x) + c2 Y1 (x) x

2126
0.5 1 1.5 2

-0.1

-0.2

-0.3

Figure 48.10: Two Term Sine Series Approximation and Exact Solution.

Applying the boundary conditions we obtain,

(Y1 (2) 2Y1 (1))J1 (x) + (2J1 (1) J1 (2))Y1 (x)


y= x
J1 (1)Y1 (2) Y1 (1)J1 (2)

The value of the integral for this extremal is

J 0.310947

Polynomial Approximation. Consider an approximate solution of the form

y(x) = (x 1)(2 x)(a0 + a1 x + an xn ).

The one term approximate solution is

23
y(x) = (x 1)(2 x)
6(40 log 2 23)

2127
This one term approximation and the exact solution are plotted in Figure 48.11. The one term approximation is
a surprisingly close to the exact solution. The value of the functional is

529
J = 0.310935.
360(40 log 2 23)

0.2

0.15

0.1

0.05

1.2 1.4 1.6 1.8 2

Figure 48.11: One Term Polynomial Approximation and Exact Solution.

Solution 48.21
1. The spectrum of T is the set,

{ : (T I) is not invertible.}

2128
(T I)f = g
Z
K(x y)f (y) dy f (x) = g

K()f() f() = g()


 
K() f() = g()

We may not be able to solve for f(), (and hence invert T I), if = K(). Thus all values of K() are in
the spectrum. If K() is everywhere nonzero we consider the case = 0. We have the equation,
Z
K(x y)f (y) dy = 0

Since there are an infinite number of L2 (, ) functions which satisfy this, (those which are nonzero on a set
of measure zero), we cannot invert the equation. Thus = 0 is in the spectrum. The spectrum of T is the range
of K() plus zero.

2. Let be a nonzero eigenvalue with eigenfunction .

(T I) = 0, x
Z
K(x y)(y) dy (x) = 0, x

Since K is continuous, T is continuous. This implies that the eigenfunction is continuous. We take the
Fourier transform of the above equation.

K()() () = 0,
 
K() () = 0,

2129
is not identically
If (x) is absolutely integrable, then () is continous. Since (x) is not identically zero, ()
is nonzero on some interval of positive length, (a, b). From the above equation
zero. Continuity implies that ()
we see that K() = for (a, b).
Now assume that K() = in some interval (a, b). Any function () that is nonzero only for (a, b) satisfies
 
K() () = 0, .

By taking the inverse Fourier transform we obtain an eigenfunction (x) of the eigenvalue .

3. First we use the Fourier transform to find an explicit representation of u = (T I)1 f .

u = (T I)1 f (T I)u = f
Z
K(x y)u(y) dy u = f

2 K u u = f
f
u =
2 K
1 f
u =
1 2 K/

For || > |2 K| we can expand the denominator in a geometric series.


!n
1 X 2 K
u = f
n=0
Z
1X 1
u= Kn (x y)f (y) dy
n=0 n

2130
Here Kn is the nth iterated kernel. Now we form the Neumann series expansion.

u = (T I)1 f
 1
1 1
= I T f


1X 1 n
= T f
n=0 n

1X 1 n
= T f
n=0 n
Z
1X 1
= Kn (x y)f (y) dy
n=0 n

The Neumann series is the same as the series we derived with the Fourier transform.
Solution 48.22
We seek a transformation T such that
(L I)T f = f.
We denote u = T f to obtain a boundary value problem,

u00 u = f, u(1) = u(1) = 0.

This problem has a unique solution if and only if the homogeneous adjoint problem has only the trivial solution.

u00 u = 0, u(1) = u(1) = 0.

This homogeneous problem has the eigenvalues and eigenfunctions,


 n 2  n 
n = , un = sin (x + 1) , n N.
2 2

2131
The inhomogeneous problem has the unique solution

Z 1
u(x) = G(x, ; )f () d
1

where

sin( (x< +1)) sin( (1x> ))




sin(2 )
, < 0,

G(x, ; ) = 12 (x< + 1)(1 x> ), = 0,

( ) ( > )
sinh (x +1) sinh (1x )
<

, > 0,

sinh(2 )

for 6= (n/2)2 , n N. We set


Z 1
Tf = G(x, ; )f () d
1

and note that since the kernel is continuous this is a bounded linear transformation. If f W , then

Z 1
(L I)T f = (L I) G(x, ; )f () d
1
Z 1
= (L I)[G(x, ; )]f () d
1
Z 1
= (x )f () d
1
= f (x).

2132
If f U then
Z 1
G(x, ; ) f 00 () f () d

T (L I)f =
1
Z 1 Z 1
0 1 0 0
= [G(x, ; )f ()]1 G (x, ; )f () d G(x, ; )f () d
1 1
Z 1 Z 1
0 1 00
= [G (x, ; )f ()]1 + G (x, ; )f () d G(x, ; )f () d
1 1
Z 1
G00 (x, ; ) G(x, ; ) f () d

=
1
Z 1
= (x )f () d
1
= f (x).
L has the point spectrum n = (n/2)2 , n N.

Solution 48.23
1. We see that the solution is of the form (x) = a + x + bx2 for some constants a and b. We substitute this into
the integral equation.
Z 1
x2 y y 2 (y) dy

(x) = x +
Z 01
a + x + bx2 = x + x2 y y 2 (a + x + bx2 ) dy

0

a + bx2 = (15 + 20a + 12b) + (20 + 30a + 15b)x2

60
By equating the coefficients of x0 and x2 we solve for a and b.
( + 60) 5( 60)
a= , b=
4(2 + 5 + 60) 6(2 + 5 + 60)

2133
Thus the solution of the integral equation is
 
5( 24) 2 + 60
(x) = x 2 x + .
+ 5 + 60 6 4

2. For x < 1 the integral equation reduces to


(x) = x.

For x 1 the integral equation becomes,


Z 1
(x) = x + sin(xy)(y) dy.
0

We could solve this problem by writing down the Neumann series. Instead we will use an eigenfunction expansion.
Let {n } and {n } be the eigenvalues and orthonormal eigenfunctions of
Z 1
(x) = sin(xy)(y) dy.
0

We expand (x) and x in terms of the eigenfunctions.



X
(x) = an n (x)
n=1
X
x= bn n (x), bn = hx, n (x)i
n=1

We determine the coefficients an by substituting the series expansions into the Fredholm equation and equating

2134
coefficients of the eigenfunctions.
Z 1
(x) = x + sin(xy)(y) dy
0

X
X Z 1
X
an n (x) = bn n (x) + sin(xy) an n (y) dy
n=1 n=1 0 n=1

X X X 1
an n (x) = bn n (x) + an n (x)
n=1 n=1 n=1
n
 

an 1 = bn
n
If is not an eigenvalue then we can solve for the an to obtain the unique solution.
bn n b n bn
an = = = bn +
1 /n n n

X bn
(x) = x + n (x), for x 1.

n=1 n

If = m , and hx, m i = 0 then there is the one parameter family of solutions,



X bn
(x) = x + cm (x) + n (x), for x 1.
n=1
n
n6=m

If = m , and hx, m i =
6 0 then there is no solution.

Solution 48.24
1.
Kx = L1 L2 x = x

2135
L1 L2 (L1 x) = L1 (L1 l2 I)x
= L1 (x x)
= ( 1)(L1 x)

L1 L2 (L2 x) = (L2 L1 + I)L2 x


= L2 x + L2 x
= ( + 1)(L2 x)

2.
     
d t d t d t d t
L1 L2 L 2 L1 = + + + +
dt 2 dt 2 dt 2 dt 2
t2 t2
 
d t d 1 t d d t d 1 t d
= + + I + I I+ + I
dt 2 dt 2 2 dt 4 dt 2 dt 2 2 dt 4
=I
d 1 t2 1
L 1 L2 = + I + I = K + I
dt 2 4 2
t2 /4 2
We note that e is an eigenfunction corresponding to the eigenvalue = 1/2. Since L1 et /4 = 0 the result
2 2
of this problem does not produce any negative eigenvalues. However, Ln2 et /4 is the product of et /4 and a
polynomial of degree n in t. Since this function is square integrable it is and eigenfunction. Thus we have the
eigenvalues and eigenfunctions,
 n1
1 t d 2
n = n , n = et /4 , for n N.
2 2 dt

Solution 48.25
Since 1 is in the residual spectrum of T , there exists a nonzero y such that
h(T 1 I)x, yi = 0

2136
for all x. Now we apply the definition of the adjoint.
hx, (T 1 I) yi = 0, x

hx, (T 1 I)yi = 0, x

(T 1 I)y = 0

y is an eigenfunction of T corresponding to the eigenvalue 1 .


Solution 48.26
1.
Z 1
00
u (t) + sin(k(s t))u(s) ds = f (t), u(0) = u0 (0) = 0
0
Z 1 Z 1
00
u (t) + cos(kt) sin(ks)u(s) ds sin(kt) cos(ks)u(s) ds = f (t)
0 0
u00 (t) + c1 cos(kt) c2 sin(kt) = f (t)
u00 (t) = f (t) c1 cos(kt) + c2 sin(kt)
The solution of
u00 (t) = g(t), u(0) = u0 (0) = 0
using Green functions is Z t
u(t) = (t )g( ) d.
0
Thus the solution of our problem has the form,
Z t Z t Z t
u(t) = (t )f ( ) d c1 (t ) cos(k ) d + c2 (t ) sin(k ) d
0 0 0
t
1 cos(kt) kt sin(kt)
Z
u(t) = (t )f ( ) d c1 + c 2
0 k2 k2

2137
We could determine the constants by multiplying in turn by cos(kt) and sin(kt) and integrating from 0 to 1. This
would yields a set of two linear equations for c1 and c2 .

2. Z
X
sin nx sin ns
u(x) = u(s) ds
0 n=1
n

We expand u(x) in a sine series.



!
!
Z
X X sin nx sin ns X
an sin nx = am sin ms ds
n=1 0 n=1
n m=1
Z
X X sin nx X
an sin nx = am sin ns sin ms ds
n=1 n=1
n m=1 0

X X sin nx X
an sin nx = am mn
n=1 n=1
n m=1 2

X X sin nx
an sin nx = an
n=1
2 n=1 n

The eigenvalues and eigenfunctions are

2n
n = , un = sin nx, n N.

3.
2
1 r2
Z
1
() = (t) dt, |r| < 1
0 2 1 2r cos( t) + r2
We use Poissons formula.
() = u(r, ),

2138
where u(r, ) is harmonic in the unit disk and satisfies, u(1, ) = (). For a solution we need = 1 and that
u(r, ) is independent of r. In this case u() satisfies

u00 () = 0, u() = ().

The solution is () = c1 + c2 . There is only one eigenvalue and corresponding eigenfunction,

= 1, = c1 + c2 .

4.
Z
(x) = cosn (x )() d

We expand the kernel in a Fourier series. We could find the expansion by integrating to find the Fourier coefficients,
but it is easier to expand cosn (x) directly.

 n
n 1 x x
cos (x) = (e + e )
2
        
1 n nx n (n2)x n (n2)x n nx
= n e + e + + e + e
2 0 1 n1 n

2139
If n is odd,

"   
1 n nx n
cosn (x) = n nx
(e + e )+ (e(n2)x + e(n2)x ) +
2 0 1
  #
n
+ (ex + ex )
(n 1)/2
      
1 n n n
= n 2 cos(nx) + 2 cos((n 2)x) + + 2 cos(x)
2 0 1 (n 1)/2
(n1)/2  
1 X n
= n1 cos((n 2m)x)
2 m=0
m
n  
1 X n
= n1 cos(kx).
2 k=1
(n k)/2
odd k

2140
If n is even,
"   
1 n nx n
cosn (x) = n nx
(e + e )+ (e(n2)x + e(n2)x ) +
2 0 1
   #
n i2x i2x n
+ (e + e )+
n/2 1 n/2
       
1 n n n n
= n 2 cos(nx) + 2 cos((n 2)x) + + 2 cos(2x) +
2 0 1 n/2 1 n/2
  (n2)/2  
1 n 1 X n
= n + n1 cos((n 2m)x)
2 n/2 2 m=0
m
  n  
1 n 1 X n
= n + n1 cos(kx).
2 n/2 2 k=2
(n k)/2
even k

We will denote,
n
a0 X
cosn (x ) = ak cos(k(x )),
2 k=1
where
1 + (1)nk 1
 
n
ak = .
2 2n1 (n k)/2
We substitute this into the integral equation.
n
!
Z
a0 X
(x) = ak cos(k(x )) () d
2 k=1
Z n  Z Z 
a0 X
(x) = () d + ak cos(kx) cos(k)() d + sin(kx) sin(k)() d
2 k=1

2141
For even n, substituting (x) = 1 yields = a1 0 . For n and m both even or odd, substituting (x) = cos(mx)
or (x) = sin(mx) yields = a1m . For even n we have the eigenvalues and eigenvectors,

1
0 = , 0 = 1,
a0
1
m = , (1)
m = cos(2mx), (2)
m = sin(2mx), m = 1, 2, . . . , n/2.
a2m
For odd n we have the eigenvalues and eigenvectors,
1
m = , (1)
m = cos((2m 1)x), (2)
m = sin((2m 1)x), m = 1, 2, . . . , (n + 1)/2.
a2m1
Solution 48.27
1. First we shift the range of integration to rewrite the kernel.
Z 2
2 2 6|x s| + 3(x s)2 (s) ds

(x) =
Z0 x+2
2 2 6|y| + 3y 2 (x + y) dy

(x) =
x

We expand the kernel in a Fourier series.



X
K(y) = 2 2 6|y| + 3y 2 = cn eny
n=
(
x+2 6
n 6= 0,
Z
1 ,
cn = K(y) eny dy = n2
2 x 0, n=0

X 6 ny X 12
K(y) = e = cos(ny)
n=
n2 n=1
n2
n6=0

2142

X 12 X 12 
K(x, s) = cos(n(x s)) = cos(nx) cos(nx) + sin(nx) sin(ns)
n=1
n2 n=1
n2
Now we substitute the Fourier series expression for the kernel into the eigenvalue problem.

Z 2 X !
1 
(x) = 12 2
cos(nx) cos(ns) + sin(nx) sin(ns) (s) ds
0 n=1
n

From this we obtain the eigenvalues and eigenfunctions,

n2 1 1
n = , (1)
n = cos(nx), (2)
n = sin(nx), n N.
12

2. The set of eigenfunctions do not form a complete set. Only those functions with a vanishing integral on [0, 2]
can be represented. We consider the equation
Z 2
K(x, s)(s) ds = 0
0

Z 2 X !
12 
cos(nx) cos(ns) + sin(nx) sin(ns) (s) ds = 0
0 n=1
n2

This has the solutions = const. The set of eigenfunctions

1 1 1
0 = , (1)
n = cos(nx), (2)
n = sin(nx), n N,
2

is a complete set. We can also write the eigenfunctions as

1
n = enx , n Z.
2

2143
3. We consider the problem
u T u = f.
For 6= , ( not an eigenvalue), we can obtain a unique solution for u.
Z 2
u(x) = f (x) + (x, s, )f (s) ds
0
Since K(x, s) is self-adjoint and L2 (0, 2), we have

X n (x)n (s)
(x, s, ) =
n=
n
n6=0

X 1
2
enx ens
= n2
n= 12

n6=0

X en(xs)
= 6
n=
n2 12
n6=0


X cos(n(x s))
(x, s, ) = 12
n=1
n2 12

Solution 48.28
First assume that is an eigenvalue of T , T = .
n
X
p(T ) = an T n
k=0
Xn
= an n
k=0
= p()

2144
p() is an eigenvalue of p(T ).
Now assume that is an eigenvalues of p(T ), p(T ) = . We assume that T has a complete, orthonormal set of
eigenfunctions, {n } corresponding to the set of eigenvalues {n }. We expand in these eigenfunctions.

p(T ) =
X X
p(T ) cn n = cn n
X X
cn p(n )n = cn n
p(n ) = , n such that cn 6= 0

Thus all eigenvalues of p(T ) are of the form p() with an eigenvalue of T .

Solution 48.29
The Fourier cosine transform is defined,

Z
1
f() = f (x) cos(x) dx,
0
Z
f (x) = 2 f() cos(x) d.
0

We can write the integral equation in terms of the Fourier cosine transform.

Z
(x) = f (x) + cos(2xs)(s) ds
0

(x) = f (x) + (2x) (48.8)

2145
We multiply the integral equation by 1 cos(2xs) and integrate.
1 1
Z Z Z
cos(2xs)(x) dx = cos(2xs)f (x) dx + cos(2xs)(2x) dx
0 0 0

Z

(2s) = f (2s) + cos(xs)(x) dx
2 0

(2s) = f(2s) + (s)
4

4 4
(x) = f(2x) + (2x) (48.9)

We eliminate between (48.8) and (48.9).
2
 
1 (x) = f (x) + f(2x)
4
R
f (x) + 0 f (s) cos(2xs) ds
(x) =
1 2 /4
Solution 48.30

Z Z
vLu dx dy = v(uxx + uyy + aux + buy + cu) dx dy
D ZD
= (v2 u + avux + bvuy + cuv) dx dy
ZD Z
2
= (vu uv) n ds
(u v + avux + bvuy + cuv) dx dy +
D C
Z Z   Z  
2 x y u v
= (u v auvx buvy uvax uvby + cuv) dx dy + auv + buv ds + v u
D C n n C n n

2146
Thus we see that
Z Z

(vLu uL v) dx dy = H(u, v) ds,
D C

where
L v = vxx + vyy avx bvy + (c ax by )v

and
 
u v x y
H(u, v) = v u + auv + buv .
n n n n

Let G be the harmonic Green function, which satisfies,

G = in D, G = 0 on C.

Let u satisfy Lu = 0.
Z Z

(GLu uL G) dx dy = H(u, G) ds
D Z Z C

uL G dx dy = H(u, G) ds
D C
Z Z Z

uG dx dy u(L )G dx dy = H(u, G) ds
D D C
Z Z Z

u(x )(y ) dx dy u(L )G dx dy = H(u, G) ds
D D C
Z Z
u(, ) u(L )G dx dy = H(u, G) ds
D C

2147
We expand the operators to obtain the first form.
Z Z  
u G x y
u+ u(aGx bGy + (c ax by )G) dx dy = G u + auG + buG ds
D C n n n n
Z Z
G
u + ((c ax by )G aGx bGy )u dx dy = u ds
D C n
Z
u + ((c ax by )G aGx bGy )u dx dy = U
D

Here U is the harmonic function that satisfies U = f on C.


We use integration by parts to obtain the second form.
Z
u + (cuG ax uG by uG auGx buGy ) dx dy = U
D
Z Z  
y x
u + (cuG ax uG by uG + (au)x G + (bu)y G) dx dy auG + buG ds = U
D C n n
Z
u + (cuG ax uG by uG + ax uG + aux G + by uG + buy G) dx dy = U
D
Z
u+ (aux + buy + cu)G dx dy = U
D

Solution 48.31
1. First we differentiate to obtain a differential equation.
Z 1 Z x Z 1 
s x
(x) = min(x, s)(s) ds = e (s) ds + e (s) ds
0 0 x
 Z 1  Z 1
0
(x) = x(x) + (s) ds x(x) = (s) ds
x x
00
(x) = (x)

2148
We note that that (x) satisfies the constraints,

Z 1
(0) = 0 (s) ds = 0,
0
Z 1
0
(1) = (s) ds = 0.
1

Thus we have the problem,


00 + = 0, (0) = 0 (1) = 0.

The general solution of the differential equation is



a + bx for = 0


  
(x) = a cos x + b sin x for > 0
a cosh x + b sinh x for < 0

We see that for = 0 and < 0 only the trivial solution satisfies the homogeneous boundary conditions. For
positive the left boundary condition demands that a = 0. The right boundary condition is then

 
b cos =0

The eigenvalues and eigenfunctions are

 2  
(2n 1) (2n 1)
n = , n (x) = sin x , nN
2 2

2149
2. First we differentiate the integral equation.
Z x Z 1 
s x
(x) = e (s) ds + e (s) ds
0 x
 Z 1 
0 x x x
(x) = e (x) + e (s) ds e (x)
x
Z 1
x
= e (s) ds
x
 Z 1 
00 x x
(x) = e (s) ds e (x)
x

(x) satisfies the differential equation


00 0 + ex = 0.
We note the boundary conditions,
(0) 0 (0) = 0, 0 (1) = 0.
In self-adjoint form, the problem is
0
ex 0 + = 0, (0) 0 (0) = 0, 0 (1) = 0.

The Rayleigh quotient is


1 R1
[ ex 0 ]0 + 0 ex (0 )2 dx
= R1
0
2 dx
1
(0)0 (0) + 0 ex (0 )2 dx
R
= R1
0
2 dx
R1
((0))2 + 0 ex (0 )2 dx
= R1
0
2 dx

2150
Thus we see that there are only positive eigenvalues. The differential equation has the general solution
    
(x) = ex/2 aJ1 2 ex/2 + bY1 2 ex/2

We define the functions,


   
u(x; ) = ex/2 J1 2 ex/2 , v(x; ) = ex/2 Y1 2 ex/2 .

We write the solution to automatically satisfy the right boundary condition, 0 (1) = 0,
(x) = v 0 (1; )u(x; ) u0 (1; )v(x; ).
We determine the eigenvalues from the left boundary condition, (0) 0 (0) = 0. The first few are
1 0.678298
2 7.27931
3 24.9302
4 54.2593
5 95.3057
The eigenfunctions are,
n (x) = v 0 (1; n )u(x; n ) u0 (1; n )v(x; n ).

Solution 48.32
1. First note that
sin(kx) sin(lx) = sign(kl) sin(ax) sin(bx)
where
a = max(|k|, |l|), b = min(|k|, |l|).
Consider the analytic function,
e(ab)x e(a+b)
= sin(ax) sin(bx) cos(ax) sin(bx).
2

2151
Z Z
sin(kx) sin(lx) sin(ax) sin(bx)
2 2
dx = sign(kl) dx
x z x2 z 2
Z  
1 sin(ax) sin(bx) sin(ax) sin(bx)
= sign(kl) dx
2z xz x+z
1
= sign(kl) ( cos(az) sin(bz) + cos(az) sin(bz))
2z
Z
sin(kx) sin(lx)
dx = sign(kl) cos(az) sin(bz)
x2 z 2 z

2. Consider the analytic function,


e|p|x e|q|x cos(|p|x) cos(|q|x) + (sin(|p|x) sin(|q|x))
= .
x x
Z Z
cos(px) cos(qx) cos(|p|x) cos(|q|x)
2
dx = dx
x x2
sin(|p|x) sin(|q|x)
= lim
x0 x
Z
cos(px) cos(qx)
dx = (|q| |p|)
x2

3. We use the analytic function,


(x a)(x b) ex (x2 ab) sin x + (a + b)x cos x + ((x2 ab) cos x + (a + b)x sin x)
=
(x2 + a2 )(x2 + b2 ) (x2 + a2 )(x2 + b2 )
Z
(x2 ab) sin x + (a + b)x cos x (x2 ab) cos x + (a + b)x sin x
= lim
x(x2 + a2 )(x2 + b2 ) x0 (x2 + a2 )(x2 + b2 )
ab
= 2 2
ab

2152
Z
(x2 ab) sin x + (a + b)x cos x
2 2 2 2
=
(x + a )(x + b ) ab

Solution 48.33
We consider the function
G(z) = (1 z 2 )1/2 + z log(1 + z).


For (1 z 2 )1/2 = (1 z)1/2 (1 + z)1/2 we choose the angles,

< arg(1 z) < , 0 < arg(1 + z) < 2,

so that there is a branch cut on the interval (1, 1). With this choice of branch, G(z) vanishes at infinity. For the
logarithm we choose the principal branch,
< arg(1 + z) < .
For t (1, 1),
 
G+ (t) = 1 t2 + t log(1 + t),
 
G (t) = 1 t2 + t log(1 + t),


G+ (t) G (t) = 2 1 t2 log(1 + t),
1 +
G (t) + G (t) = t log(1 + t).

2
For t (, 1),
 
+
G (t) = t2
1 + t (log(t 1) + ) ,
 
G (t) = 1 t2 + t (log(t 1) ) ,

2153
 
G+ (t) G (t) = 2 t2 1 + t .
For x (1, 1) we have
1 +
G (x) + G (x)

G(x) =
2
= x log(1 + x)
Z 1
2 2 1 t2 log(1 + t)
1 2( t 1 + t)
Z
1 1
= dt + dt
2 tx 2 1 tx
From this we have
Z 1
1 t2 log(1 + t)
dt
1 tx


t2 1
t
Z
= x log(1 + x) + dt
1 t+x
 
= x log(1 + x) 1 + 1 x2 1 x2 arcsin(x) + x log(2) + x log(1 + x)
2

Z 1
1 t2 log(1 + t)   
dt = x log x 1 + 1 x2 arcsin(x)
1 tx 2

Solution 48.34
Let F (z) denote the value of the integral.
Z
1 f (t) dt
F (z) =
C t z
From the Plemelj formula we have,
Z
+ 1 f (t)
F (t0 ) + F (t0 ) = dt,
C t t0
f (t0 ) = F + (t0 ) F (t0 ).

2154
With W (z) defined as above, we have

W + (t0 ) + W (t0 ) = F + (t0 ) F (t0 ) = f (t0 ),

and also

W + (t) W (t)
Z
+ 1
W (t0 ) + W (t0 ) = dt
t t0
ZC +
1 F (t) + F (t)
= dt
t t0
ZC
1 g(t)
= dt.
C t t0

Thus the solution of the integral equation is

Z
1 g(t)
f (t0 ) = dt.
C t t0

2155
Solution 48.35
(i)

 
1
G( ) = ( )

 
+ 1
G () = ( )

G () = e2 G+ ()
 
+ 2 1
G () G () = (1 e )( )

 
+ 2 1
G () + G () = (1 + e )( )

2
(1 e
Z
1 ) d
G+ () + G () =
C ( ) ( ) ( )
1

( )1
Z
1 d
= cot()
C ( )1 ( ) ( ) ( )

(ii) Consider the branch of

 
z
z

2156
that tends to unity as z . We find a series expansion of this function about infinity.

    
z 
= 1 1
z z z
   j ! X    !
X k
= (1)j (1)k
j=0
j z k=0
k z
j    !
X X
= (1)j jk k z j
j=0 k=0
j k k

Define the polynomial

j
n    !
X X jk k
Q(z) = (1)j z nj .
j=0 k=0
jk k

Then the function

 
z
G(z) = z n Q(z)
z

2157
vanishes at infinity.
 
+
G () = n Q()

 
2
G () = e n Q()

 

+
n 1 e2

G () G () =

 

+
n 1 + e2 2Q()

G () + G () =

Z    
1 2 1
n
n 1 + e2 2Q()
 
1e d =
i C
Z   n
 
1
d = cot() n (1 cot())Q()
i C
 
n
Z   
1 n
d = cot() Q() Q()
i C

Solution 48.36

Z 1 Z 1 Z 1
(y) 1 (y) 1 (y)
dy = dy dy
1 y 2 x2 2x 1 yx 2x 1 y + x
Z 1 Z 1
1 (y) 1 (y)
= dy + dy
2x 1 yx 2x 1 y x
Z 1
1 (y) + (y)
= dy
2x 1 yx

2158
Z 1
1 (y) + (y)
dy = f (x)
2x 1 yx
1 1 (y) + (y)
Z
2x
dy = f (x)
1 yx
Z 1
1 2y p 1 k
(x) + (x) = f (y) 1 y 2 dy +
1 x2 1 yx 1 x2
Z 1 p
1 2yf (y) 1 y 2 k
(x) + (x) = dy +
2 1 x2 1 yx 1 x2
Z 1 p
1 yf (y) 1 y 2 k
(x) = dy + + g(x)
2
1 x 1 2 yx 1 x2
Here k is an arbitrary constant and g(x) is an arbitrary odd function.

Solution 48.37
We define
Z 1
1 f (t)
F (z) = dt.
2 0 t z
The Plemelj formulas and the integral equation give us,
F + (x) F (x) = f (x)
F + (x) + F (x) = f (x).
We solve for F + and F .
F + (x) = ( + 1)f (x)
F (x) = ( 1)f (x)
By writing
F + (x) +1

=
F (x) 1

2159
we seek to determine F to within a multiplicative constant.


+ +1
log F (x) log F (x) = log
1
 
+ 1+
log F (x) log F (x) = log +
1
log F + (x) log F (x) = +

We have left off the additive term of 2n in the above equation, which will introduce factors of z k and (z 1)m in
F (z). We will choose these factors so that F (z) has integrable algebraic singularites and vanishes at infinity. Note that
we have defined to be the real parameter,

 
1+
= log .
1

By the discontinuity theorem,

Z 1
1 +
log F (z) = dz
2 0 t z
   
1 1z
= log
2 2 z
 1/2/(2) !
z1
= log
z

2160
 1/2/(2)
z1
F (z) = z k (z 1)m
z
/(2)

1 z1
F (z) = p
z(z 1) z
e(/(2)) 1 x /(2)
 

F (x) = p
x(1 x) x
/(2)
e/2

1x
F (x) = p
x(1 x) x

Define  /(2)
1 1x
f (x) = p .
x(1 x) x
We apply the Plemelj formulas.

1 1 /2
Z
 f (t)
e e/2 dt = e/2 + e/2 f (x)

0 tx
Z 1
1 f (t)  
dt = tanh f (x)
0 t x 2

Thus we see that the eigenfunctions are


  tanh1 ()/
1 1x
(x) = p
x(1 x) x

for 1 < < 1.


The method used in this problem cannot be used to construct eigenfunctions for > 1. For this case we cannot
find an F (z) that has integrable algebraic singularities and vanishes at infinity.

2161
Solution 48.38

1 1 f (t)
Z

dt = f (x)
0 t x tan(x)
We define the function, Z 1
1 f (t)
F (z) = dt.
2 0 t z
The Plemelj formula are,

F + (x) F (x) = f (x)



F + (x) + F (x) = f (x).
tan(x)
We solve for F + and F .  
1
F (x) = 1 f (x)
2 tan(x)
From this we see
F + (x) 1 / tan(x)

= = e2x .
F (x) 1 / tan(x)
We seek to determine F (z) up to a multiplicative constant. Taking the logarithm of this equation yields

log F + (x) log F (x) = 2x + 2n.

The 2n term will give us the factors (z 1)k and z m in the solution for F (z). We will choose the integers k and m
so that F (z) has only algebraic singularities and vanishes at infinity. We drop the 2n term for now.
Z 1
1 2t
log F (z) = dt
2 0 t z
   z/
1 z 1z 1/ z1
log F (z) = + log F (z) = e
z z

2162
We replace e1/ by a multiplicative constant and multiply by (z 1)1 to give F (z) the desired properties.
c
F (z) =
(z 1)1z/ z z/

We evaluate F (z) above and below the branch cut.

c c ex
F (x) = =
e(x) (1 x)1x/ xx/ (1 x)1x/ xx/

Finally we use the Plemelj formulas to determine f (x).

k sin(x)
f (x) = F + (x) F (x) =
(1 x)1x/ xx/

Solution 48.39
Consider the equation,
Z
0 f (t)
f (z) + dt = 1.
C tz
Since the integral is an analytic function of z off C we know that f (z) is analytic off C. We use Cauchys theorem to
evaluate the integral and obtain a differential equation for f (x).
Z
0 f (t)
f (x) + dt = 1
C tx
f 0 (x) + f (x) = 1
1
f (x) = + c ex

Consider the equation, Z


0 f (t)
f (z) + dt = g(z).
C tz

2163
Since the integral and g(z) are analytic functions inside C we know that f (z) is analytic inside C. We use Cauchys
theorem to evaluate the integral and obtain a differential equation for f (x).

Z
0 f (t)
f (x) + dt = g(x)
C tx
f 0 (x) + f (x) = g(x)
Z x
f (x) = e(x) g() d + c ex
z0

Here z0 is any point inside C.

Solution 48.40

Z  
1
+ P (t x) f (t) dt = g(x)
C tx
Z Z
1 f (t) 1 1
dt = g(x) P (t x)f (t) dt
C t x C

We know that if
Z
1 f ( )
d = g()
C

then
Z
1 g( )
f () = d.
C

2164
We apply this theorem to the integral equation.
Z Z Z 
1 g(t) 1 1
f (x) = 2 dt + 2
P ( t)f ( ) d dt
C tx C C tx
Z Z 
P ( t)
Z
1 g(t) 1
= 2 dt + dt f ( ) d
C tx 2 C C t x
Z Z
1 g(t) 1
= 2 dt P (t x)f (t) dt
C tx C

Now we substitute the non-analytic part of f (t) into the integral. (The analytic part integrates to zero.)
Z Z  Z 
1 g(t) 1 1 g( )
= 2 dt P (t x) 2 d dt
tx C C t
ZC Z  
P (t x)
Z
1 g(t) 1 1
= 2 dt dt g( ) d
tx 2 C C t
ZC Z
1 g(t) 1
= 2 dt P ( x)g( ) d
C tx 2 C
Z Z
1 g(t) 1
f (x) = 2 dt 2 P (t x)g(t) dt
C tx C

Solution 48.41

Solution 48.42

2165
Part VII

Nonlinear Differential Equations

2166
Chapter 49

Nonlinear Ordinary Differential Equations

2167
49.1 Exercises
Exercise 49.1
A model set of equations to describe an epidemic, in which x(t) is the number infected, y(t) is the number susceptible,
is
dx dy
= rxy x, = rxy + ,
dt dt
where r > 0, 0, 0. Initially x = x0 , y = y0 at t = 0. Directly from the equations, without using the phase
plane:

1. Find the solution, x(t), y(t), in the case = = 0.

2. Show for the case = 0, 6= 0 that x(t) first decreases or increases according as ry0 < or ry0 > . Show
that x(t) 0 as t in both cases. Find x as a function of y.

3. In the phase plane: Find the position of the singular point and its type when > 0, > 0.

Exercise 49.2
Find the singular points and their types for the system

du
= ru + v(1 v)(p v), r > 0, 0 < p < 1,
dx
dv
= u,
dx

which comes from one of our nonlinear diffusion problems. Note that there is a solution with

u = (1 v)

for special values of and r. Find v(x) for this special case.

2168
Exercise 49.3
Check that r = 1 is a limit cycle for
dx
= y + x(1 r2 )
dt
dy
= x + y(1 r2 )
dt
(r = x2 + y 2 ), and that all solution curves spiral into it.

Exercise 49.4
Consider
y = f (y) x
x = y
Introduce new coordinates, R, given by
x = R cos
1
y = R sin

and obtain the exact differential equations for R(t), (t). Show that R(t) continually increases with t when R 6= 0.
Show that (t) continually decreases when R > 1.

Exercise 49.5
One choice of the Lorenz equations is
x = 10x + 10y
y = Rx y xz
8
z = z + xy
3
Where R is a positive parameter.

2169
1. Invistigate the nature of the sigular point at (0, 0, 0) by finding the eigenvalues and their behavior for all 0 < R <
.
2. Find the other singular points when R > 1.
3. Show that the appropriate eigenvalues for these other singular points satisfy the cubic
33 + 412 + 8(10 + R) + 160(R 1) = 0.

4. There is a special value of R, call it Rc , for which the cubic has two pure imaginary roots, say. Find Rc and
; then find the third root.
Exercise 49.6
In polar coordinates (r, ), Einsteins equations lead to the equation
d2 v 1
+ v = 1 + v 2 , v= ,
d2 r
for planetary orbits. For Mercury,  = 8 108 . When  = 0 (Newtonian theory) the orbit is given by
v = 1 + A cos , period 2.
Introduce = and use perturbation expansions for v() and in powers of  to find the corrections proportional to
.
[A is not small;  is the small parameter].

Exercise 49.7
Consider the problem
x + 02 x + x2 = 0, x = a, x = 0 at t = 0
Use expansions
x = a cos + a2 x2 () + a3 x3 () + , = t
= 0 + a2 2 + ,

2170
to find a periodic solution and its natural frequency .
Note that, with the expansions given, there are no secular term troubles in the determination of x2 (), but x2 ()
is needed in the subsequent determination of x3 () and .
Show that a term a1 in the expansion for would have caused trouble, so 1 would have to be taken equal to
zero.
Exercise 49.8
Consider the linearized traffic problem
dpn (t)
= [pn1 (t) pn (t)] , n 1,
dt
pn (0) = 0, n 1,
p0 (t) = aet , t > 0.
(We take the imaginary part of pn (t) in the final answers.)
1. Find p1 (t) directly from the equation for n = 1 and note the behavior as t .
2. Find the generating function

X
G(s, t) = pn (t)sn .
n=1

3. Deduce that
pn (t) An et , as t ,
and find the expression for An . Find the imaginary part of this pn (t).

Exercise 49.9
1. For the equation modified with a reaction time, namely
d
pn (t + ) = [pn1 (t) pn (t)] n 1,
dt
find a solution of the form in 1(c) by direct substitution in the equation. Again take its imaginary part.

2171
2. Find a condition that the disturbance is stable, i.e. pn (t) remains bounded as n .

3. In the stable case show that the disturbance is wave-like and find the wave velocity.

2172
49.2 Hints
Hint 49.1

Hint 49.2

Hint 49.3

Hint 49.4

Hint 49.5

Hint 49.6

Hint 49.7

Hint 49.8

Hint 49.9

2173
49.3 Solutions
Solution 49.1
1. When = = 0 the equations are

dx dy
= rxy, = rxy.
dt dt

Adding these two equations we see that

dx dy
= .
dt dt

Integrating and applying the initial conditions x(0) = x0 and y(0) = y0 we obtain

x = x0 + y0 y

Substituting this into the differential equation for y,

dy
= r(x0 + y0 y)y
dt
dy
= r(x0 + y0 )y + ry 2 .
dt

2174
We recognize this as a Bernoulli equation and make the substitution u = y 1 .
dy
y 2 = r(x0 + y0 )y 1 r
dt
du
= r(x0 + y0 )u r
dt
d r(x0 +y0 )t 
e u = rer(x0 +y0 )t
dt Z t
u=er(x0 +y0 )t
rer(x0 +y0 )t dt + cer(x0 +y0 )t
1
u= + cer(x0 +y0 )t
x0 + y0
 1
1 r(x0 +y0 )t
y= + ce
x0 + y0
Applying the initial condition for y,
 1
1
+c = y0
x0 + y0
1 1
c= .
y0 x0 + y0
The solution for y is then
   1
1 1 1 r(x0 +y0 )t
y= + e
x0 + y0 y0 x0 + y0
Since x = x0 + y0 y, the solution to the system of differential equations is

 1  1
   
1 1 r(x0 +y0 )t 1 1 r(x0 +y0 )t
x = x0 + y0 + 1e , y= + 1e .
y0 x0 + y0 y0 x0 + y0

2175
2. For = 0, 6= 0, the equation for x is
x = rxy x.
At t = 0,
x(0) = x0 (ry0 ).
Thus we see that if ry0 < , x is initially decreasing. If ry0 > , x is initially increasing.
Now to show that x(t) 0 as t . First note that if the initial conditions satisfy x0 , y0 > 0 then x(t), y(t) > 0
for all t 0 because the axes are a seqaratrix. y(t) is is a strictly decreasing function of time. Thus we see
that at some time the quantity x(ry ) will become negative. Since y is decreasing, this quantity will remain
negative. Thus after some time, x will become a strictly decreasing quantity. Finally we see that regardless of
the initial conditions, (as long as they are positive), x(t) 0 as t .
Taking the ratio of the two differential equations,
dx
= 1 + .
dy ry

x = y + ln y + c
r
Applying the intial condition,

x0 = y0 + ln y0 + c
r

c = x0 + y0 ln y0 .
r
Thus the solution for x is  
y
x = x0 + (y0 y) + ln .
r y0

3. When > 0 and > 0 the system of equations is


x = rxy x
y = rxy + .

2176
The equilibrium solutions occur when
x(ry ) = 0
rxy = 0.
Thus the singular point is

x= , y= .
r
Now to classify the point. We make the substitution u = (x ), v = (y r ).
   
 
u = r u + v+ u+
r
 

v = r u + v+ +
r

r
u = v + ruv

r
v = u v ruv

The linearized system is
r
u = v

r
v = u v

Finding the eigenvalues of the linearized system,

r r
2
r = + + r = 0

+

2177
q
r

( r

)2 4r
=
2
Since both eigenvalues have negative real part, we see that the singular point is asymptotically stable. A plot
of the vector field for r = = = 1 is attached. We note that there appears to be a stable singular point at
x = y = 1 which corroborates the previous results.
Solution 49.2
The singular points are
u = 0, v = 0, u = 0, v = 1, u = 0, v = p.
The point u = 0, v = 0. The linearized system about u = 0, v = 0 is
du
= ru
dx
dv
= u.
dx
The eigenvalues are
r 0 2
1 = r = 0.

= 0, r.
Since there are positive eigenvalues, this point is a source. The critical point is unstable.
The point u = 0, v = 1. Linearizing the system about u = 0, v = 1, we make the substitution w = v 1.
du
= ru + (w + 1)(w)(p 1 w)
dx
dw
=u
dx
du
= ru + (1 p)w
dx
dw
=u
dx

2178

r (p 1)
= 2 r + p 1 = 0
1
p
r r2 4(p 1)
=
2
Thus we see that this point is a saddle point. The critical point is unstable.
The point u = 0, v = p. Linearizing the system about u = 0, v = p, we make the substitution w = v p.

du
= ru + (w + p)(1 p w)(w)
dx
dw
=u
dx

du
= ru + p(p 1)w
dx
dw
=u
dx

r p(1 p)
= 2 r + p(1 p) = 0
1
p
r r2 4p(1 p)
=
2
Thus we see that this point is a source. The critical point is unstable.
The solution of for special values of and r. Differentiating u = v(1 v),

du
= 2v.
dv

2179
Taking the ratio of the two differential equations,

du v(1 v)(p v)
=r+
dv u
v(1 v)(p v)
=r+
v(1 v)
(p v)
=r+

Equating these two expressions,


p v
2v = r + .

Equating coefficients of v, we see that = 1 .


2

1
= r + 2p
2


Thus we have the solution u = 1 v(1 v) when r = 1 2p. In this case, the differential equation for v is
2 2

dv 1
= v(1 v)
dx 2
dv 1 1
v 2 = v 1 +
dx 2 2

2180
We make the change of variablles y = v 1 .

dy 1 1
= y +
dx 2 2

x/ 2
d   e
ex/ 2 y =
dx 2


Z x x/ 2
e
y = ex/ 2 dx + cex/ 2
2

y = 1 + cex/ 2

The solution for v is

1
v(x) = .
1 + cex/ 2
Solution 49.3
We make the change of variables

x = r cos
y = r sin .

Differentiating these expressions with respect to time,

x = r cos r sin
y = r sin + r cos .

Substituting the new variables into the pair of differential equations,

r cos r sin = r sin + r cos (1 r2 )


r sin + r cos = r cos + r sin (1 r2 ).

2181
Multiplying the equations by cos and sin and taking their sum and difference yields

r = r(1 r2 )
r = r.

We can integrate the second equation.

r = r(1 r2 )
= t + 0

At this point we could note that r > 0 in (0, 1) and r < 0 in (1, ). Thus if r is not initially zero, then the solution
tends to r = 1.
Alternatively, we can solve the equation for r exactly.

r = r r3
r 1
3
= 2 1
r r

We make the change of variables u = 1/r2 .

1
u = u 1
2
u + 2u = 2
Z t
2t
u=e 2e2t dt + ce2t

u = 1 + ce2t
1
r=
1 + ce2t
Thus we see that if r is initiall nonzero, the solution tends to 1 as t .

2182
Solution 49.4
The set of differential equations is

y = f (y) x
x = y.

We make the change of variables

x = R cos
1
y = R sin


Differentiating x and y,

x = R cos R sin
1 1
y = R sin + R cos .
 

The pair of differential equations become


 
1
R sin + R cos = f R sin R cos

1
R cos R sin = R sin .


 
1 1 1
R sin + R cos = R cos f R sin
  
1
R cos R sin = R sin .


2183
Multiplying by cos and sin and taking the sum and difference of these differential equations yields
 
1 1
R = sin f R sin
 
 
1 1 1
R = R + cos f R sin .
  

Dividing by R in the second equation,


 
1 1
R = sin f R sin
 
 
1 1 cos 1
= + f R sin .
  R 

We make the assumptions that 0 <  < 1 and that f (y) is an odd function that is nonnegative for positive y and
satisfies |f (y)| 1 for all y.
Since sin is odd,  
1
sin f R sin

is nonnegative. Thus R(t) continually increases with t when R 6= 0.
If R > 1 then
   
cos 1 1

R f R sin f R sin
 
1.

Thus the value of ,  


1 1 cos 1
+ f R sin ,
  R 
is always nonpositive. Thus (t) continually decreases with t.

2184
Solution 49.5
1. Linearizing the Lorentz equations about (0, 0, 0) yields

x 10 10 0 x
y = R 1 0 y
z 0 0 8/3 z

The eigenvalues of the matrix are

8
1 = ,
3
11 81 + 40R
2 =
2
11 + 81 + 40R
3 = .
2

There are three cases for the eigenvalues of the linearized system.

R < 1. There are three negative, real eigenvalues. In the linearized and also the nonlinear system, the origin is
a stable, sink.
R = 1. There are two negative, real eigenvalues and one zero eigenvalue. In the linearized system the origin
is stable and has a center manifold plane. The linearized system does not tell us if the nonlinear system is
stable or unstable.
R > 1. There are two negative, real eigenvalues, and one positive, real eigenvalue. The origin is a saddle point.

2. The other singular points when R > 1 are


r r !
8 8
(R 1), (R 1), R 1 .
3 3

2185
3. Linearizing about the point r r !
8 8
(R 1), (R 1), R 1
3 3
yields
10 10 0
X q X
1 1 83 (R 1) Y

Y =
q q
Z 8
(R 1) 8
(R 1) 8 Z
3 3 3

The characteristic polynomial of the matrix is

41 2 8(10 + R) 160
3 + + + (R 1).
3 3 3
Thus the eigenvalues of the matrix satisfy the polynomial,

33 + 412 + 8(10 + R) + 160(R 1) = 0.

Linearizing about the point r r !


8 8
(R 1), (R 1), R 1
3 3
yields
10 10 0
X q X
8
1 1 (R 1)

Y
Y = 3
q q
Z 83 (R 1) 83 (R 1) 38 Z

The characteristic polynomial of the matrix is

41 2 8(10 + R) 160
3 + + + (R 1).
3 3 3

2186
Thus the eigenvalues of the matrix satisfy the polynomial,
33 + 412 + 8(10 + R) + 160(R 1) = 0.

4. If the characteristic polynomial has two pure imaginary roots and one real root, then it has the form
( r)(2 + 2 ) = 3 r2 + 2 r2 .
Equating the 2 and the term with the characteristic polynomial yields
r
41 8
r= , = (10 + R).
3 3
Equating the constant term gives us the equation
41 8 160
(10 + Rc ) = (Rc 1)
3 3 3
which has the solution
470
Rc = .
19
For this critical value of R the characteristic polynomial has the roots
41
1 =
3
4
2 = 2090
19
4
3 = 2090.
19
Solution 49.6
The form of the perturbation expansion is
v() = 1 + A cos + u() + O(2 )
= (1 + 1 + O(2 )).

2187
Writing the derivatives in terms of ,

d d
= (1 + 1 + )
d d
2
d d2
= (1 + 2 1 + ) .
d2 d2

Substituting these expressions into the differential equation for v(),

1 + 21 + O(2 ) A cos + u00 + O(2 ) + 1 + A cos + u() + O(2 )


  

= 1 +  1 + 2A cos + A2 cos2 + O()


 

u00 + u 21 A cos =  + 2A cos + A2 cos2 + O(2 ).

Equating the coefficient of ,

1
u00 + u = 1 + 2(1 + 1 )A cos + A2 (cos 2 + 1)
2
1 1
u00 + u = (1 + A2 ) + 2(1 + 1 )A cos + A2 cos 2.
2 2
To avoid secular terms, we must have 1 = 1. A particular solution for u is

1 1
u = 1 + A2 A2 cos 2.
2 6
The the solution for v is
 
1 2 1 2
v() = 1 + A cos((1 )) +  1 + A A cos(2(1 )) + O(2 ).
2 6

2188
Solution 49.7
Substituting the expressions for x and into the differential equations yields
  2     2  
2 2 d x2 2 3 2 d x3
a 0 + x2 + cos + a 0 + x3 20 2 cos + 2x2 cos + O(a4 ) = 0
d2 d2

Equating the coefficient of a2 gives us the differential equation


d2 x2
+ x2 = (1 + cos 2).
d2 202

The solution subject to the initial conditions x2 (0) = x02 (0) = 0 is



x2 = (3 + 2 cos + cos 2).
602

Equating the coefficent of a3 gives us the differential equation


 2
2 52 2 2
  
2 d x3
0 + x3 + 2 20 2 + 2 cos + 2 cos 2 + 2 cos 3 = 0.
d2 30 60 30 60
To avoid secular terms we must have
52
2 = .
120
Solving the differential equation for x3 subject to the intial conditions x3 (0) = x03 (0) = 0,

2
x3 = (48 + 29 cos + 16 cos 2 + 3 cos 3).
14404

Thus our solution for x(t) is


   2 
2 3
x(t) = a cos + a (3 + 2 cos + cos 2) + a (48 + 29 cos + 16 cos 2 + 3 cos 3) + O(a4 )
602 14404

2189
 
52
where = 0 a2 120
t.
Now to see why we didnt need an a1 term. Assume that

x = a cos + a2 x2 () + O(a3 ); = t
= 0 + a1 + O(a2 ).

Substituting these expressions into the differential equation for x yields

a2 02 (x002 + x2 ) 20 1 cos + cos2 = O(a3 )


 

1
x002 + x2 = 2 cos 2 (1 + cos 2).
0 20
In order to eliminate secular terms, we need 1 = 0.
Solution 49.8
1. The equation for p1 (t) is

dp1 (t)
= [p0 (t) p1 (t)].
dt
dp1 (t)
= [aet p1 (t)]
dt
d t
e p1 (t) = aet et

dt
a t
p1 (t) = e + cet
+

Applying the initial condition, p1 (0) = 0,

a
et et

p1 (t) =
+

2190
2. We start with the differential equation for pn (t).

dpn (t)
= [pn1 (t) pn (t)]
dt
Multiply by sn and sum from n = 1 to .

X
X
p0n (t)sn = [pn1 (t) pn (t)]sn
n=1 n=1

G(s, t) X
= pn sn+1 G(s, t)
t n=0

G(s, t) X
= sp0 + pn sn+1 G(s, t)
t n=1
G(s, t)
= aset + sG(s, t) G(s, t)
t
G(s, t)
= aset + (s 1)G(s, t)
t
(1s)t
G(s, t) = ase(1s)t et

e
t
as
G(s, t) = et + C(s)e(s1)t
(1 s) +
The initial condition is
X
G(s, 0) = pn (0)sn = 0.
n=1

The generating function is then


as
et e(s1)t .

G(s, t) =
(1 s) +

2191
3. Assume that |s| < 1. In the limit t we have
as
G(s, t) et
(1 s) +
as
G(s, t) et
1 + / s
as/(1 + /) t
G(s, t) e
1 s/(1 + /)
 n
aset X s
G(s, t)
1 + / n=0 1 + /

X sn
G(s, t) aet
n=1
(1 + /)n

Thus we have
a
pn (t) n
et as t .
(1 + /)

 
a t
=(pn (t)) = e
(1 + /)n
 n
1 /
=a [cos(t) + sin(t)]
1 + (/)2
a
= [cos(t)=[(1 /)n ] + sin(t)<[(1 /)n ]]
(1 + (/)2 )n

n n
a X
(j+1)/2
 j X
j/2
 j
= cos(t) (1) + sin(t) (1)
(1 + (/)2 )n
j=1
j=0

odd j even j

2192
Solution 49.9
1. Substituting pn = An et into the differential equation yields

An e(t+ ) = [An1 et An et ]
An ( + e ) = An1

We make the substitution An = rn .

rn ( + e ) = rn1

r=
+ e

Thus we have

 n
1
pn (t) = et .
1 + e /

2193
Taking the imaginary part,

 n 
1 t
=(pn (t)) = = e
1 + e
n
1 e
 

== cos(t) + sin(t)
1 + (e e ) + ( )2
1 sin( ) cos( ) n
  

== cos(t) + sin(t)
1 2 sin( ) + ( )2
 n h
1 h n i
= cos(t)= 1 sin( ) cos( )
1 2 sin( ) + ( )2
h n i i
+ sin(t)< 1 sin( ) cos( )

 n
1
=
1 2 sin( ) + ( )2

n
(j+1)/2
h X h ij h inj
cos(t) (1) cos( ) 1 sin( )
j=1

odd j
n h
X ij h inj i
+ sin(t) (1)j/2 cos( ) 1 sin( )
j=0

even j

2194
2. pn (t) will remain bounded in time as n if

1
1 + e 1


2
1 + e 1


 2
1 2 sin( ) + 1


2 sin( )

3.

2195
Chapter 50

Nonlinear Partial Differential Equations

2196
50.1 Exercises
Exercise 50.1
Consider the nonlinear PDE
ut + uux = 0.
The solution u is constant along lines (characteristics) such that x ut = k for any constant k. Thus the slope of
these lines will depend on the initial data u(x, 0) = f (x).

1. In terms of this initial data, write down the equation for the characteristic in the x, t plane which goes through
the point (x, t) = (, 0).

2. State a criteria on f such that two characteristics will intersect at some positive time t. Assuming intersections
do occur, what is the time of the first intersection? You may assume that f is everywhere continuous and
differentiable.
2
3. Apply this to the case where f (x) = 1 ex to indicate where and when a shock will form and sketch (roughly)
the solution both before and after this time.
Exercise 50.2
Solve the equation
t + (1 + x)x + = 0 in < x < , t > 0,
with initial condition (x, 0) = f (x).

Exercise 50.3
Solve the equation

t + x + =0
1+x
in the region 0 < x < , t > 0 with initial condition (x, 0) = 0, and boundary condition (0, t) = g(t). [Here is a
positive constant.]

2197
Exercise 50.4
Solve the equation
t + x + 2 = 0
in < x < , t > 0 with initial condition (x, 0) = f (x). Note that the solution could become infinite in finite
time.
Exercise 50.5
Consider
ct + ccx + c = 0, < x < , t > 0.
1. Use the method of characteristics to solve the problem with

c = F (x) at t = 0.

( is a positive constant.)
2. Find equations for the envelope of characteristics in the case F 0 (x) < 0.
3. Deduce an inequality relating max |F 0 (x)| and which decides whether breaking does or does not occur.

Exercise 50.6
For water waves in a channel the so-called shallow water equations are

ht + (hv)x = 0 (50.1)
 
2 1 2
(hv)t + hv + gh = 0, g = constant. (50.2)
2 x

Investigate whether there are solutions with v = V (h), where V (h) is not posed in advance but is obtained from
requiring consistency between the h equation obtained from (1) and the h equation obtained from (2).
There will be two possible choices for V (h) depending on a choice of sign. Consider each case separately. In
each case fix the arbitrary constant that arises in V (h) by stipulating that before the waves arrive, h is equal to the
undisturbed depth h0 and V (h0 ) = 0.
Find the h equation and the wave speed c(h) in each case.

2198
Exercise 50.7
After a change of variables, the chemical exchange equations can be put in the form


+ =0 (50.3)
t x

= ; , , = positive constants. (50.4)
t
1. Investigate wave solutions in which = (X), = (X), X = x U t, U = constant, and show that (X)
must satisfy an ordinary differential equation of the form
d
= quadratic in .
dX

2. Discuss ths smooth shock solution as we did for a different example in class. In particular find the expression
for U in terms of the values of as X , and find the sign of d/dX. Check that
2 1
U=
2 1
in agreement with the discontinuous theory.
Exercise 50.8
Find solitary wave solutions for the following equations:

1. t + x + 6x xxt = 0. (Regularized long wave or B.B.M. equation)

2. utt uxx 23 u2 xx uxxxx = 0. (Boussinesq)




3. tt xx + 2x xt + xx t xxxx = 0. (The solitary wave form is for u = x )

4. ut + 30u2 u1 + 20u1 u2 + 10uu3 + u5 = 0. (Here the subscripts denote x derivatives.)

2199
50.2 Hints
Hint 50.1

Hint 50.2

Hint 50.3

Hint 50.4

Hint 50.5

Hint 50.6

Hint 50.7

Hint 50.8

2200
50.3 Solutions
Solution 50.1
1.
x = + u(, 0)t
x = + f ()t

2. Consider two points 1 and 2 where 1 < 2 . Suppose that f (1 ) > f (2 ). Then the two characteristics passing
through the points (1 , 0) and (2 , 0) will intersect.
1 + f (1 )t = 2 + f (2 )t
2 1
t=
f (1 ) f (2 )
We see that the two characteristics intersect at the point
 
2 1 2 1
(x, t) = 1 + f (1 ) , .
f (1 ) f (2 ) f (1 ) f (2 )
We see that if f (x) is not a non-decreasing function, then there will be a positive time when characteristics
intersect.
Assume that f (x) is continuously differentiable and is not a non-decreasing function. That is, there are points
where f 0 (x) is negative. We seek the time T of the first intersection of characteristics.
2 1
T = min
1 <2 f (1 ) f (2 )
f (1 )>f (2 )

(f (2 )f (1 ))/(2 1 ) is the slope of the secant line on f (x) that passes through the points 1 and 2 . Thus we
seek the secant line on f (x) with the minimum slope. This occurs for the tangent line where f 0 (x) is minimum.
1
T =
min f 0 ()

2201
3. First we find the time when the characteristics first intersect. We find the minima of f 0 (x) with the derivative
test.
2
f (x) = 1 ex
2
f 0 (x) = 2x ex
2
f 00 (x) = 2 4x2 ex = 0


1
x =
2

The minimum slope occurs at x = 1/ 2.
1 e1/2
T = = 1.16582
2 e1/2 / 2 2
Figure 50.1 shows the solution at various times up to the first collision of characteristics, when a shock forms.
After this time, the shock wave moves to the right.
Solution 50.2
The method of characteristics gives us the differential equations
x0 (t) = (1 + x) x(0) =
d
= (, 0) = f ()
dt
Solving the first differential equation,
x(t) = cet 1, x(0) =
t
x(t) = ( + 1)e 1
The second differential equation then becomes
(x(t), t) = cet , (, 0) = f (), = (x + 1)et 1
(x, t) = f ((x + 1)et 1)et

2202
1 1
0.8 0.8
0.6 0.6
0.4 0.4
0.2 0.2
-3 -2 -1 1 2 3 -3 -2 -1 1 2 3
1 1
0.8 0.8
0.6 0.6
0.4 0.4
0.2 0.2
-3 -2 -1 1 2 3 -3 -2 -1 1 2 3

Figure 50.1: The solution at t = 0, 1/2, 1, 1.16582.

Thus the solution to the partial differential equation is

(x, t) = f ((x + 1)et 1)et .

Solution 50.3

d
= t + x0 (t)x =
dt 1+x
The characteristic curves x(t) satisfy x0 (t) = 1, so x(t) = t + c. The characteristic curve that separates the region
with domain of dependence on the x axis and domain of dependence on the t axis is x(t) = t. Thus we consider the
two cases x > t and x < t.

x > t. x(t) = t + .

2203
x < t. x(t) = t .

Now we solve the differential equation for in the two domains.

x > t.

d
= , (, 0) = 0, =xt
dt 1+x
d
=
dt 1+t+
 Z t 
1
= c exp dt
t++1
= cexp ( log(t + + 1))
= c(t + + 1)

applying the initial condition, we see that

=0

x < t.

d
= , (0, ) = g( ), =tx
dt 1+x
d
=
dt 1+t
= c(t + 1 )
= g( )(t + 1 )
= g(t x)(x + 1)

2204
Thus the solution to the partial differential equation is
(
0 for x > t
(x, t) =
g(t x)(x + 1) for x < t.

Solution 50.4
The method of characteristics gives us the differential equations
x0 (t) = 1 x(0) =
d
= 2 (, 0) = f ()
dt
Solving the first differential equation,
x(t) = t + .
The second differential equation is then
d
= 2 , (, 0) = f (), =xt
dt
2 d = dt
1 = t + c
1
=
tc
1
=
t + 1/f ()
1
= .
t + 1/f (x t)
Solution 50.5
1. Taking the total derivative of c with respect to t,
dc dx
= ct + cx .
dt dt

2205
Equating terms with the partial differential equation, we have the system of differential equations
dx
=c
dt
dc
= c.
dt
subject to the initial conditions
x(0) = , c(, 0) = F ().
We can solve the second ODE directly.

c(, t) = c1 et
c(, t) = F ()et

Substituting this result and solving the first ODE,


dx
= F ()et
dt
F () t
x(t) = e + c2

F ()
x(t) = (1 et ) + .

The solution to the problem at the point (x, t) is found by first solving

F ()
x= (1 et ) +

for and then using this value to compute

c(x, t) = F ()et .

2206
2. The characteristic lines are given by the equation

F ()
x(t) = (1 et ) + .

The points on the envelope of characteristics also satisfy

x(t)
= 0.

Thus the points on the envelope satisfy the system of equations

F ()
x= (1 et ) +

F 0 ()
0= (1 et ) + 1.

By substituting

1 et =
F 0 ()
into the first equation we can eliminate its t dependence.

F ()
x= +
F 0 ()

Now we can solve the second equation in the system for t.



et = 1 + 0
F ()
 
1
t = log 1 + 0
F ()

2207
Thus the equations that describe the envelope are
F ()
x= +
F 0 ()
 
1
t = log 1 + 0 .
F ()
3. The second equation for the envelope has a solution for positive t if there is some x that satisfies

1 < 0 < 0.
F (x)
This is equivalent to
< F 0 (x) < .
So in the case that F 0 (x) < 0, there will be breaking iff
max |F 0 (x)| > .
Solution 50.6
With the substitution v = V (h), the two equations become
ht + (V + hV 0 )hx = 0
(V + hV 0 )ht + (V 2 + 2hV V 0 + gh)hx = 0.
We can rewrite the second equation as
V 2 + 2hV V 0 + gh
ht + hx = 0.
V + hV 0
Requiring that the two equations be consistent gives us a differential equation for V .
V 2 + 2hV V 0 + gh
V + hV 0 =
V + hV 0
V + 2hV V + h (V ) = V 2 + 2hV V 0 + gh
2 0 2 0 2

g
(V 0 )2 = .
h

2208
There are two choices depending on which sign we choose when taking the square root of the above equation.
Positive V0 .
r
0 g
V =
h
p
V = 2 gh + const

We apply the initial condition V (h0 ) = 0.

p
V = 2 g( h h0 )

The partial differential equation for h is then


p
ht + (2 g( h h0 )h)x = 0
p
ht + g(3 h 2 h0 )hx = 0

The wave speed is


p
c(h) = g(3 h 2 h0 ).
Negative V0 .
r
0 g
V =
h
p
V = 2 gh + const

We apply the initial condition V (h0 ) = 0.

p
V = 2 g( h0 h)

2209
The partial differential equation for h is then

p
ht + g(2 h0 3 h)hx = 0.

The wave speed is


p
c(h) = g(2 h0 3 h).

Solution 50.7
1. Making the substitutions, = (X), = (X), X = xU t, the system of partial differential equations becomes

U 0 + 0 = 0
U 0 = .

Integrating the first equation yields

U + = c
= c + U .

Now we substitute the expression for into the second partial differential equation.

U 0 = (c + U ) (c + U )
 c  c
0 = + + + +
U U U

Thus (X) satisfies the ordinary differential equation

 
0 2 c c
= + + .
U U U

2210
2. Assume that
(X) 1 as X +
(X) 2 as X
0 (X) 0 as X .
Integrating the ordinary differential equation for ,
Z
d
X= c
 c
.
2 + U
+ U
U

We see that the roots of the denominator of the integrand must be 1 and 2 . Thus we can write the ordinary
differential equation for (X) as
0 (X) = ( 1 )( 2 ) = 2 (1 + 2 ) + 1 2 .
Equating coefficients in the polynomial with the differential equation for part 1, we obtain the two equations
c c
= 1 2 , + = (1 + 2 ).
U U U
Solving the first equation for c,
U 1 2
c= .

Now we substitute the expression for c into the second equation.
U 1 2
+ = (1 + 2 )
U U
2 1 2
=+ (1 + 2 )
U
Thus we see that U is

U= .
2 + 2 1 2 (1 + 2 )

2211
Since the quadratic polynomial in the ordinary differential equation for (X) is convex, it is negative valued
between its two roots. Thus we see that
d
< 0.
dX

Using the expression for that we obtained in part 1,

2 1 c + U 2 (c + U 1 )
=
2 1 2 1
2 1
=U
2 1
= U.

Now lets return to the ordinary differential equation for (X)

0 (X) = ( 1 )( 2 )
Z
d
X=
( 1 )( 2 )
Z  
1 1 1
X= + d
(2 1 ) 1 2
 
1 1
X X0 = ln
(2 1 )
 2 
1
(2 1 )(X X0 ) = ln
2
1
= exp ((2 1 )(X X0 ))
2
1 = (2 ) exp ((2 1 )(X X0 ))
[1 + exp ((2 1 )(X X0 ))] = 1 + 2 exp ((2 1 )(X X0 ))

2212
Thus we obtain a closed form solution for

1 + 2 exp ((2 1 )(X X0 ))


=
1 + exp ((2 1 )(X X0 ))
Solution 50.8
1.
t + x + 6x xxt = 0
We make the substitution
(x, t) = z(X), X = x U t.

(1 U )z 0 + 6zz 0 + U z 000 = 0
(1 U )z + 3z 2 + U z 00 = 0
1 1
(1 U )z 2 + z 3 + U (z 0 )2 = 0
2 2
U 1 2
(z 0 )2 = z2 z3
U rU !
U 1 1 U 1
z(X) = sech2 X
2 2 U
r !!
U 1 1 U 1 p
(x, t) = sech2 x (U 1)U t
2 2 U

The linearized equation is


t + x xxt = 0.
Substituting = ex+t into this equation yields
2 = 0

= .
1 2

2213
We set
U 1
2 = .
U
is then


=
1p 2
(U 1)/U
=
1 (U 1)/U )
p
(U 1)U
=
U (U 1)
p
= (U 1)U .

The solution for becomes


 
x t
sech2
2 2

where

= .
1 2

2.
 
3 2
utt uxx u uxxxx = 0
2 xx

We make the substitution


u(x, t) = z(X), X = x U t.

2214
 00
00 3 2
2
(U 1)z z z 0000 = 0
2
 0
0 3 2
2
(U 1)z z z 000 = 0
2
3
(U 2 1)z z 2 z 00 = 0
2

We multiply by z 0 and integrate.

1 2 1 1
(U 1)z 2 z 3 (z 0 )2 = 0
2 2 2
(z 0 )2 = (U 2 1)z 2 z 3
1 2
 
2 2
z = (U 1) sech U 1X
2
1 2
  
2 2 2
u(x, t) = (U 1) sech U 1x U U 1t
2

The linearized equation is


utt uxx uxxxx = 0.

Substituting u = ex+t into this equation yields

2 2 4 = 0
2 = 2 (2 + 1).

We set

= U 2 1.

2215
is then
2 = 2 (2 + 1)
= (U 2 1)U 2

= U U 2 1.

The solution for u becomes  


2 2 x t
u(x, t) = sech
2
where
2 = 2 (2 + 1).
3.
tt xx + 2x xt + xx t xxxx
We make the substitution
(x, t) = z(X), X = x U t.

(U 2 1)z 00 2U z 0 z 00 U z 00 z 0 z 0000 = 0
(U 2 1)z 00 3U z 0 z 00 z 0000 = 0
3
(U 2 1)z 0 (z 0 )2 z 000 = 0
2
Multiply by z 00 and integrate.
1 2 1 1
(U 1)(z 0 )2 (z 0 )3 (z 00 )2 = 0
2 2 2
(z ) = (U 1)(z ) (z 0 )3
00 2 2 0 2

1 2
 
0 2 2
z = (U 1) sech U 1X
2
1 2
  
2 2 2
x (x, t) = (U 1) sech U 1x U U 1t .
2

2216
The linearized equation is
tt xx xxxx
Substituting = ex+t into this equation yields

2 = 2 (2 + 1).

The solution for x becomes  


2 2 x t
x = sech
2
where
2 = 2 (2 + 1).

4.
ut + 30u2 u1 + 20u1 u2 + 10uu3 + u5 = 0
We make the substitution
u(x, t) = z(X), X = x U t.

U z 0 + 30z 2 z 0 + 20z 0 z 00 + 10zz 000 + z (5) = 0

Note that (zz 00 )0 = z 0 z 00 + zz 000 .

U z 0 + 30z 2 z 0 + 10z 0 z 00 + 10(zz 00 )0 + z (5) = 0


U z + 10z 3 + 5(z 0 )2 + 10zz 00 + z (4) = 0

Multiply by z 0 and integrate.

1 5 1
U z 2 + z 4 + 5z(z 0 )2 (z 00 )2 + z 0 z 000 = 0
2 2 2

2217
Assume that
(z 0 )2 = P (z).
Differentiating this relation,

2z 0 z 00 = P 0 (z)z 0
1
z 00 = P 0 (z)
2
1
z 000 = P 00 (z)z 0
2
1
z 000 z 0 = P 00 (z)P (z).
2

Substituting this expressions into the differential equation for z,


1 5 11 0 1
U z 2 + z 4 + 5zP (z) (P (z))2 + P 00 (z)P (z) = 0
2 2 24 2
4U z 2 + 20z 4 + 40zP (z) (P 0 (z))2 + 4P 00 (z)P (z) = 0

Substituting P (z) = az 3 + bz 2 yields

(20 + 40a + 15a2 )z 4 + (40b + 20ab)z 3 + (4b2 + 4U )z 2 = 0

This equation is satisfied by b2 = U , a = 2. Thus we have



(z 0 )2 = U z 2 2z 3
 
U 2 1 1/4
z= sech U X
2 2
 
U 2 1 1/4 5/4
u(x, t) = sech (U x U t)
2 2

2218
The linearized equation is
ut + u5 = 0.
Substituting u = ex+t into this equation yields

5 = 0.

We set
= U 1/4 .
The solution for u(x, t) becomes
2
 
x t
sech2
2 2
where
= 5 .

2219
Part VIII

Appendices

2220
Appendix A

Greek Letters

The following table shows the greek letters, (some of them have two typeset variants), and their corresponding
Roman letters.
Name Roman Lower Upper
alpha a
beta b
chi c
delta d
epsilon e 
epsilon (variant) e
phi f
phi (variant) f
gamma g
eta h
iota i
kappa k
lambda l
mu m

2221
nu n
omicron o o
pi p
pi (variant) p $
theta q
theta (variant) q
rho r
rho (variant) r %
sigma s
sigma (variant) s
tau t
upsilon u
omega w
xi x
psi y
zeta z

2222
Appendix B

Notation

C class of continuous functions


Cn class of n-times continuously differentiable functions
C set of complex numbers
(x) Dirac delta function
F[] Fourier transform
Fc [] Fourier cosine transform
Fs [] Fourier sine transform R

Eulers constant, = 0 ex Log x dx
() Gamma function
H(x) Heaviside function
(1)
H (x) Hankel function of the first kind and order
(2)
H (x) Hankel
function of the second kind and order
1
J (x) Bessel function of the first kind and order
K (x) Modified Bessel function of the first kind and order
L[] Laplace transform

2223
N set of natural numbers, (positive integers)
N (x) Modified Bessel function of the second kind and order
R set of real numbers
R+ set of positive real numbers
R set of negative real numbers
o(z) terms smaller than z
O(z)
R terms no bigger than z
principal value of the integral
d
() digamma function, () = d log ()
dn
(n) () (n)
polygamma function, () = d n ()
nu
u(n) (x) xn
n+m u
u(n,m) (x, y) xn y m
Y (x) Bessel function of the second kind and order , Neumann function
Z set of integers
Z+ set of positive integers

2224
Appendix C

Formulas from Complex Variables

Analytic Functions. A function f (z) is analytic in a domain if the derivative f 0 (z) exists in that domain.
If f (z) = u(x, y) + v(x, y) is defined in some neighborhood of z0 = x0 + y0 and the partial derivatives of u and v
are continuous and satisfy the Cauchy-Riemann equations

ux = v y , uy = vx ,

then f 0 (z0 ) exists.

Residues. If f (z) has the Laurent expansion


X
f (z) = an z n ,
n=

then the residue of f (z) at z = z0 is


Res(f (z), z0 ) = a1 .

2225
Residue Theorem. Let C be a positively oriented, simple, closed contour. If f (z) is analytic in and on C except
for isolated singularities at z1 , z2 , . . . , zN inside C then
I N
X
f (z) dz = 2 Res(f (z), zn ).
C n=1

If in addition f (z) is analytic outside C in the finite complex plane then


I    
1 1
f (z) dz = 2 Res 2
f ,0 .
C z z

Residues of a pole of order n. If f (z) has a pole of order n at z = z0 then


dn1
 
1 n
Res(f (z), z0 ) = lim [(z z0 ) f (z)] .
zz0 (n 1)! dz n1

Jordans Lemma. Z

eR sin d <
.
0 R
Let a be a positive constant. If f (z) vanishes as |z| then the integral
Z
f (z) eaz dz
C

along the semi-circle of radius R in the upper half plane vanishes as R .

Taylor Series. Let f (z) be a function that is analytic and single valued in the disk |z z0 | < R.

X f (n) (z0 )
f (z) = (z z0 )n
n=0
n!

The series converges for |z z0 | < R.

2226
Laurent Series. Let f (z) be a function that is analytic and single valued in the annulus r < |z z0 | < R. In this
annulus f (z) has the convergent series,
X
f (z) = cn (z z0 )n ,
n=

where I
1 f (z)
cn = dz
2 (z z0 )n+1
and the path of integration is any simple, closed, positive contour around z0 and lying in the annulus. The path of
integration is shown in Figure C.1.

Im(z)

r
Re(z)

Figure C.1: The Path of Integration.

2227
Appendix D

Table of Derivatives

Note: c denotes a constant and 0 denotes differentiation.


d df dg
(f g) = g+f
dx dx dx
d f f 0g f g0
=
dx g g2

d c
f = cf c1 f 0
dx
d
f (g) = f 0 (g)g 0
dx

d2
f (g) = f 00 (g)(g 0 )2 + f 0 g 00
dx2

dn
  n   n1   n2 2   n
n d f n d f dg n d fdg n d g
n
(f g) = n
g+ n1
+ n2 2
+ + f
dx 0 dx 1 dx dx 2 dx dx n dxn

2228
d 1
ln x =
dx |x|

d x
c = cx ln c
dx
d g df dg
f = gf g1 + f g ln f
dx dx dx
d
sin x = cos x
dx
d
cos x = sin x
dx
d
tan x = sec2 x
dx
d
csc x = csc x cot x
dx
d
sec x = sec x tan x
dx
d
cot x = csc2 x
dx
d 1
arcsin x = , arcsin x
dx 1 x2 2 2

2229
d 1
arccos x = , 0 arccos x
dx 1 x2

d 1
arctan x = , arctan x
dx 1 + x2 2 2
d
sinh x = cosh x
dx
d
cosh x = sinh x
dx
d
tanh x = sech2 x
dx
d
csch x = csch x coth x
dx
d
sech x = sech x tanh x
dx
d
coth x = csch2 x
dx
d 1
arcsinh x =
dx x2 + 1

d 1
arccosh x = , x > 1, arccosh x > 0
dx 2
x 1

d 1
arctanh x = , x2 < 1
dx 1 x2

2230
Z x
d
f () d = f (x)
dx c
Z c
d
f () d = f (x)
dx x

Z h Z h
d f (, x)
f (, x) d = d + f (h, x)h0 f (g, x)g 0
dx g g x

2231
Appendix E

Table of Integrals

Z Z
dv du
u dx = uv v dx
dx dx

f 0 (x)
Z
dx = log f (x)
f (x)

f 0 (x)
Z p
p dx = f (x)
2 f (x)

x+1
Z
x dx = for 6== 1
+1
Z
1
dx = log x
x

eax
Z
eax dx =
a

2232
abx
Z
bx
a dx = for a > 0
b log a
Z
log x dx = x log x x

Z
1 1 x
dx = arctan
x2+a 2 a a
(
1
log ax for x2 < a2
Z
1 2a a+x
dx = 1 xa
x 2 a2 2a
log x+a
for x2 > a2
Z
1 x x
dx = arcsin = arccos for x2 < a2
a2 x 2 |a| |a|
Z
1
dx = log(x + x2 a2 )
x 2 a2
Z
1 1 x
dx = sec1
x x2 a2 |a| a
 
a2 x 2
Z
1 1 a+
dx = log
x a2 x 2 a x
Z
1
sin(ax) dx = cos(ax)
a
Z
1
cos(ax) dx = sin(ax)
a

2233
Z
1
tan(ax) dx = log cos(ax)
a
Z
1 ax
csc(ax) dx = log tan
a 2
Z
1  ax 
sec(ax) dx = log tan +
a 4 2
Z
1
cot(ax) dx = log sin(ax)
a
Z
1
sinh(ax) dx = cosh(ax)
a
Z
1
cosh(ax) dx = sinh(ax)
a
Z
1
tanh(ax) dx = log cosh(ax)
a
Z
1 ax
csch(ax) dx =log tanh
a 2
Z  
i i ax
sech(ax) dx = log tanh +
a 4 2
Z
1
coth(ax) dx = log sinh(ax)
a

2234
Z
1 x
x sin ax dx = 2
sin ax cos ax
a a

a2 x 2 2
Z
2x
x2 sin ax dx = sin ax cos ax
a2 a3
Z
1 x
x cos ax dx = 2
cos ax + sin ax
a a

2x cos ax a2 x2 2
Z
x2 cos ax dx = + sin ax
a2 a3

2235
Appendix F

Definite Integrals

Integrals from to . Let f (z) be analytic except for isolated singularities, none of which lie on the real
axis. Let a1 , . . . , am be the singularities of f (z) in the upper half plane; and CR be the semi-circle from R to R in
the upper half plane. If  
lim R max |f (z)| = 0
R zCR

then Z m
X
f (x) dx = 2 Res (f (z), aj ) .
j=1

Let b1 , . . . , bn be the singularities of f (z) in the lower half plane. Let CR be the semi-circle from R to R in the lower
half plane. If  
lim R max |f (z)| = 0
R zCR

then Z n
X
f (x) dx = 2 Res (f (z), bj ) .
j=1

2236
Integrals from 0 to . Let f (z) be analytic except for isolated singularities, none of which lie on the positive real
axis, [0, ). Let z1 , . . . , zn be the singularities of f (z). If f (z)  z as z 0 for some > 1 and f (z)  z as
z for some < 1 then
Z Xn
f (x) dx = Res (f (z) log z, zk ) .
0 k=1
Z n n
1X 2
 X
f (x) log dx = Res f (z) log z, zk + Res (f (z) log z, zk )
0 2 k=1 k=1

Assume that a is not an integer. If z a f (z)  z as z 0 for some > 1 and z a f (z)  z as z for some
< 1 then
Z n
a 2 X
x f (x) dx = 2a
Res (z a f (z), zk ) .
0 1 e
k=1
n n
2a X
Z
a 2 X
a
x f (x) log x dx = Res (z f (z) log z, zk ) , + 2 Res (z a f (z), zk )
0 1 e2a k=1
sin (a) k=1

Fourier Integrals. Let f (z) be analytic except for isolated singularities, none of which lie on the real axis. Suppose
that f (z) vanishes as |z| . If is a positive real number then
Z n
X
x
f (x) e dx = 2 Res(f (z) ez , zk ),
k=1

where z1 , . . . , zn are the singularities of f (z) in the upper half plane. If is a negative real number then
Z n
X
x
f (x) e dx = 2 Res(f (z) ez , zk ),
k=1

where z1 , . . . , zn are the singularities of f (z) in the lower half plane.

2237
Appendix G

Table of Sums


X r
rn = , for |r| < 1
n=1
1r

N
X r rN +1
rn =
n=1
1r

b
X (a + b)(b + 1 a)
n=
n=a
2

N
X N (N + 1)
n=
n=1
2

b
X b(b + 1)(2b + 1) a(a 1)(2a 1)
n2 =
n=a
6

2238
N
X N (N + 1)(2N + 1)
n2 =
n=1
6

X (1)n+1
= log(2)
n=1
n


X 1 2
=
n=1
n2 6

X (1)n+1 2
=
n=1
n2 12


X 1
3
= (3)
n=1
n

X (1)n+1 3(3)
=
n=1
n3 4


X 1 4
=
n=1
n4 90

X (1)n+1 7 4
=
n=1
n4 720

2239

X 1
= (5)
n=1
n5

X (1)n+1 15(5)
=
n=1
n5 16


X 1 6
=
n=1
n6 945

X (1)n+1 31 6
=
n=1
n6 30240

2240
Appendix H

Table of Taylor Series


X
(1 z)1 = zn |z| < 1
n=0


X
2
(1 z) = (n + 1)z n |z| < 1
n=0

 

X
(1 + z) = zn |z| < 1
n=0
n


z
X zn
e = |z| <
n=0
n!


X zn
log(1 z) = |z| < 1
n=1
n

2241

z 2n1
 
1+z X
log =2 |z| < 1
1z n=1
2n 1


X (1)n z 2n
cos z = |z| <
n=0
(2n)!


X (1)n z 2n+1
sin z = |z| <
n=0
(2n + 1)!

z 3 2z 5 17z 7
tan z = z + + + + |z| < 2
3 15 315

z3 1 3z 5 1 3 5z 7
 
1
cos z = z + + + + |z| < 1
2 23 245 2467

z3 1 3z 5 1 3 5z 7
sin1 z = z + + + + |z| < 1
23 245 2467

1
X (1)n+1 z 2n1
tan z= |z| < 1
n=1
2n 1


X z 2n
cosh z = |z| <
n=0
(2n)!


X z 2n+1
sinh z = |z| <
n=0
(2n + 1)!

2242
z 3 2z 5 17z 7
tanh z = z + + |z| < 2
3 15 315

X (1)n  z +2n
J (z) = |z| <
n=0
n!( + n + 1) 2


X 1  z +2n
I (z) = |z| <
n=0
n!( + n + 1) 2

2243
Appendix I

Continuous Transforms

I.1 Properties of Laplace Transforms


Let f (t) be piecewise continuous and of exponential order . Unless otherwise noted, the transform is defined for s > 0.
To reduce clutter, it is understood that the Heaviside function H(t) multiplies the original function in the following two
tables.
Z
f (t) est f (t) dt
0

Z c+
1
ets f(s) ds f(s)
2 c

af (t) + bg(t) af(s) + bg(s)

d
f (t) sf(s) f (0)
dt

2244
d2
f (t) s2 f(s) sf (0) f 0 (0)
dt2
dn
f (t) sn f(s) sn1 f (0)
dtn
sn2 f 0 (0) f (n1) (0)

t
f(s)
Z
f ( ) d
0 s

f(s)
Z tZ
f (s) ds d
0 0 s2

ect f (t) f(s c) s>c+


 
1 t
f , c>0 f(cs)
c c
 
1 (b/c)t t
e f , c>0 f(cs b)
c c

f (t c)H(t c), c>0 ecs f(s)

d
tf (t) f (s)
ds
dn
tn f (t) (1)n f (s)
dsn
Z 1 Z
f (t) f (t)
, dt exists f(t) dt
t 0 t s

2245
Z t
f ( )g(t ) d, f, g C 0 f(s)g(s)
0

RT
0
est f (t) dt
f (t), f (t + T ) = f (t)
1 esT
RT
0
est f (t) dt
f (t), f (t + T ) = f (t)
1 + esT

2246
I.2 Table of Laplace Transforms
Z
f (t) est f (t) dt
0

Z c+
1
ets f(s) ds f(s)
2 c

1
1
s
1
t
s2
n!
tn , for n = 0, 1, 2, . . .
sn+1

1/2 3/2
t s
2
1/2
t1/2 s

n1/2 + (1)(3)(5) (2n 1) n1/2
t , nZ s
2n
( + 1)
t , <() > 1
s+1
Log s
Log t
s

2247
( + 1)
t Log t, <() > 1 (( + 1) Log s)
sn+1

(t) 1 s>0

(n) (t), n Z0+ sn s>0

1
ect s>c
sc
1
t ect s>c
(s c)2

tn1 ect 1
, n Z+ s>c
(n 1)! (s c)n
c
sin(ct)
s2 + c2
s
cos(ct)
s2 + c 2
c
sinh(ct) s > |c|
s2 c2
s
cosh(ct) s > |c|
s2 c2
2cs
t sin(ct)
(s2 + c2 )2

s2 c 2
t cos(ct)
(s2 + c2 )2

2248
n!
tn ect , n Z+
(s c)n+1
c
edt sin(ct) s>d
(s d)2 + c2

sd
edt cos(ct) s>d
(s d)2 + c2
(
0 for c < 0
(t c) sc
e for c > 0
(
0 for t < c 1 cs
H(t c) = e
1 for t > c s

cn
J (ct)  > 1
s2 + c 2 s + s2 + c 2

cn
I (ct)  <(s) > c, > 1
s2 c 2 s s2 + c 2

2249
I.3 Table of Fourier Transforms
Z
1
f (x) f (x) ex dx
2
Z
F() ex d F()

af (x) + bg(x) aF () + bG()

f (n) (x) ()n F ()

xn f (x) n F (n) ()

f (x + c) ec F ()

ecx f (x) F ( + c)

f (cx) |c|1 F (/c)


Z
f (x)g(x) F G() = F ()G( ) d

Z
1 1
f g(x) = f ()g(x ) d F ()G()
2 2

2 1 2
ecx , c>0 e /4c
4c

2250
c/
ec|x| , c>0
2 + c2
2c
, c>0 ec||
x2 + c2
(
1 0 for > 0
, >0
x e for < 0
(
1 e for > 0
, <0
x 0 for < 0

1
sign()
x 2
(
0 for x < c 1 c
H(x c) = e
1 for x > c 2

1
ecx H(x), <(c) > 0
2(c + )

1
ecx H(x), <(c) > 0
2(c )

1 ()

1
(x ) e
2

((x + ) + (x )) cos()

2251
((x + ) (x )) sin()
(
1 for |x| < c sin(c)
H(c |x|) = ,c>0
0 for |x| > c

2252
I.4 Table of Fourier Transforms in n Dimensions
Z
1
f (x) f (x) ex dx
(2)n Rn
Z
F() ex d F()
Rn

af (x) + bg(x) aF () + bG()


 n/2 2 /4c 2
enx ec
c

2253
I.5 Table of Fourier Cosine Transforms
Z
1
f (x) f (x) cos (x) dx
0
Z
2 C() cos (x) d C()
0

1
f 0 (x) S() f (0)

1 0
f 00 (x) 2 C() f (0)


xf (x) Fs [f (x)]

1  
f (cx), c>0 C
c c
2c
ec
x2 + c2

c/
ecx
2 + c2
2 1 2
ecx e /(4c)
4c
r
x2 /(4c) 2
e ec
c

2254
I.6 Table of Fourier Sine Transforms
Z
1
f (x) f (x) sin (x) dx
0
Z
2 S() sin (x) d S()
0

f 0 (x) C()

1
f 00 (x) 2 S() + f (0)


xf (x) Fc [f (x)]

1  
f (cx), c>0 S
c c
2x
ec
x2 + c2

/
ecx
2 + c2
x 1 c
2 arctan e
c
1 cx 1  
e arctan
x c

2255
1
1

2
1
x
2 2 /(4c)
x ecx e
4c3/2

x x2 /(4c) 2
e ec
2c3/2

2256
Appendix J

Table of Wronskians

W [x a, x b] ba

W eax , ebx (b a) e(a+b)x


 

W [cos(ax), sin(ax)] a

W [cosh(ax), sinh(ax)] a

W [eax cos(bx), eax sin(bx)] b e2ax

W [eax cosh(bx), eax sinh(bx)] b e2ax

W [sin(c(x a)), sin(c(x b))] c sin(c(b a))

W [cos(c(x a)), cos(c(x b))] c sin(c(b a))

W [sin(c(x a)), cos(c(x b))] c cos(c(b a))

2257
W [sinh(c(x a)), sinh(c(x b))] c sinh(c(b a))

W [cosh(c(x a)), cosh(c(x b))] c cosh(c(b a))

W [sinh(c(x a)), cosh(c(x b))] c cosh(c(b a))

W edx sin(c(x a)), edx sin(c(x b)) c e2dx sin(c(b a))


 

W edx cos(c(x a)), edx cos(c(x b)) c e2dx sin(c(b a))


 

W edx sin(c(x a)), edx cos(c(x b)) c e2dx cos(c(b a))


 

W edx sinh(c(x a)), edx sinh(c(x b)) c e2dx sinh(c(b a))


 

W edx cosh(c(x a)), edx cosh(c(x b)) c e2dx sinh(c(b a))


 

W edx sinh(c(x a)), edx cosh(c(x b)) c e2dx cosh(c(b a))


 

W [(x a) ecx , (x b) ecx ] (b a) e2cx

2258
Appendix K

Sturm-Liouville Eigenvalue Problems

y 00 + 2 y = 0, y(a) = y(b) = 0
 
n n(x a)
n = , yn = sin , nN
ba ba
ba
hyn , yn i =
2
y 00 + 2 y = 0, y(a) = y 0 (b) = 0
 
(2n 1) (2n 1)(x a)
n = , yn = sin , nN
2(b a) 2(b a)
ba
hyn , yn i =
2
y 00 + 2 y = 0, y 0 (a) = y(b) = 0
 
(2n 1) (2n 1)(x a)
n = , yn = cos , nN
2(b a) 2(b a)

2259
ba
hyn , yn i =
2
y 00 + 2 y = 0, y 0 (a) = y 0 (b) = 0
 
n n(x a)
n = , yn = cos , n = 0, 1, 2, . . .
ba ba

ba
hy0 , y0 i = b a, hyn , yn i = for n N
2

2260
Appendix L

Green Functions for Ordinary Differential


Equations

G0 + p(x)G = (x ), G( : ) = 0
 Z x 
G(x|) = exp p(t) dt H(x )

y 00 = 0, y(a) = y(b) = 0
(x< a)(x> b)
G(x|) =
ba
y 00 = 0, y(a) = y 0 (b) = 0
G(x|) = a x<

y 00 = 0, y 0 (a) = y(b) = 0
G(x|) = x> b

2261
y 00 c2 y = 0, y(a) = y(b) = 0

sinh(c(x< a)) sinh(c(x> b))


G(x|) =
c sinh(c(b a))

y 00 c2 y = 0, y(a) = y 0 (b) = 0

sinh(c(x< a)) cosh(c(x> b))


G(x|) =
c cosh(c(b a))

y 00 c2 y = 0, y 0 (a) = y(b) = 0

cosh(c(x< a)) sinh(c(x> b))


G(x|) =
c cosh(c(b a))

npi
y 00 + c2 y = 0, y(a) = y(b) = 0, c 6= ba
, nN

sin(c(x< a)) sin(c(x> b))


G(x|) =
c sin(c(b a))

(2n1)pi
y 00 + c2 y = 0, y(a) = y 0 (b) = 0, c 6= 2(ba)
, nN

sin(c(x< a)) cos(c(x> b))


G(x|) =
c cos(c(b a))

(2n1)pi
y 00 + c2 y = 0, y 0 (a) = y(b) = 0, c 6= 2(ba)
, nN

cos(c(x< a)) sin(c(x> b))


G(x|) =
c cos(c(b a))

2262
y 00 + 2cy 0 + dy = 0, y(a) = y(b) = 0, c2 > d

ecx< sinh( c2 d(x< a)) ecx< sinh( c2 d(x> b))
G(x|) =
c2 d e2c sinh( c2 d(b a))

y 00 + 2cy 0 + dy = 0, y(a) = y(b) = 0, c2 < d, d c2 6= n
ba
, nN

ecx< sin( d c2 (x< a)) ecx< sin( d c2 (x> b))
G(x|) =
d c2 e2c sin( d c2 (b a))

y 00 + 2cy 0 + dy = 0, y(a) = y(b) = 0, c2 = d

(x< a) ecx< (x> b) ecx<


G(x|) =
(b a) e2c

2263
Appendix M

Trigonometric Identities

M.1 Circular Functions


Pythagorean Identities

sin2 x + cos2 x = 1, 1 + tan2 x = sec2 x, 1 + cot2 x = csc2 x

Angle Sum and Difference Identities

sin(x + y) = sin x cos y + cos x sin y


sin(x y) = sin x cos y cos x sin y
cos(x + y) = cos x cos y sin x sin y
cos(x y) = cos x cos y + sin x sin y

2264
Function Sum and Difference Identities
1 1
sin x + sin y = 2 sin (x + y) cos (x y)
2 2
1 1
sin x sin y = 2 cos (x + y) sin (x y)
2 2
1 1
cos x + cos y = 2 cos (x + y) cos (x y)
2 2
1 1
cos x cos y = 2 sin (x + y) sin (x y)
2 2

Double Angle Identities


sin 2x = 2 sin x cos x, cos 2x = cos2 x sin2 x

Half Angle Identities


x 1 cos x x 1 + cos x
sin2 = , cos2 =
2 2 2 2

Function Product Identities


1 1
sin x sin y = cos(x y) cos(x + y)
2 2
1 1
cos x cos y = cos(x y) + cos(x + y)
2 2
1 1
sin x cos y = sin(x + y) + sin(x y)
2 2
1 1
cos x sin y = sin(x + y) sin(x y)
2 2

Exponential Identities
ex ex ex + ex
ex = cos x + sin x, sin x = , cos x =
2 2

2265
M.2 Hyperbolic Functions
Exponential Identities
ex ex ex + ex
sinh x = , cosh x =
2 2
x
e e x
sinh x
tanh x = = x
cosh x e + ex

Reciprocal Identities
1 1 1
csch x = , sech x = , coth x =
sinh x cosh x tanh x

Pythagorean Identities
cosh2 x sinh2 x = 1, tanh2 x + sech2 x = 1

Relation to Circular Functions


sinh(x) = sin x sinh x = sin(x)
cosh(x) = cos x cosh x = cos(x)
tanh(x) = tan x tanh x = tan(x)

Angle Sum and Difference Identities


sinh(x y) = sinh x cosh y cosh x sinh y
cosh(x y) = cosh x cosh y sinh x sinh y
tanh x tanh y sinh 2x sinh 2y
tanh(x y) = =
1 tanh x tanh y cosh 2x cosh 2y
1 coth x coth y sinh 2x sinh 2y
coth(x y) = =
coth x coth y cosh 2x cosh 2y

2266
Function Sum and Difference Identities
1 1
sinh x sinh y = 2 sinh (x y) cosh (x y)
2 2
1 1
cosh x + cosh y = 2 cosh (x + y) cosh (x y)
2 2
1 1
cosh x cosh y = 2 sinh (x + y) sinh (x y)
2 2
sinh(x y)
tanh x tanh y =
cosh x cosh y
sinh(x y)
coth x coth y =
sinh x sinh y

Double Angle Identities


sinh 2x = 2 sinh x cosh x, cosh 2x = cosh2 x + sinh2 x

Half Angle Identities


x cosh x 1 x cosh x + 1
sinh2 = , cosh2 =
2 2 2 2

Function Product Identities


1 1
sinh x sinh y = cosh(x + y) cosh(x y)
2 2
1 1
cosh x cosh y = cosh(x + y) + cosh(x y)
2 2
1 1
sinh x cosh y = sinh(x + y) + sinh(x y)
2 2

See Figure M.1 for plots of the hyperbolic circular functions.

2267
1
3
2 0.5
1
-2 -1 1 2 -2 -1 1 2
-1
-2 -0.5
-3 -1

Figure M.1: cosh x, sinh x and then tanh x

2268
Appendix N

Bessel Functions

N.1 Definite Integrals


Let > 1.
Z 1
1 0 2
rJ (j,m r)J (j,n r) dr = (J (j,n )) mn
0 2
02
j ,n 2
Z 1
2
rJ (j 0 ,m r)J (j 0 ,n r) dr = 0 2 J (j 0 ,n ) mn
0 2j ,n
Z 1  2 
1 a
rJ (m r)J (n r) dr = 2 2
+ n (J (n ))2 mn
2 2
0 2 n b

Here n is the nth positive root of aJ (r) + brJ 0 (r), where a, b R.

2269
Appendix O

Formulas from Linear Algebra

Kramers Rule. Consider the matrix equation


A~x = ~b.
This equation has a unique solution if and only if det(A) 6= 0. If the determinant vanishes then there are either no
solutions or an infinite number of solutions. If the determinant is nonzero, the solution for each xj can be written
det Aj
xj =
det A
where Aj is the matrix formed by replacing the j th column of A with b.

Example O.0.1 The matrix equation     


1 2 x1 5
= ,
3 4 x2 6
has the solution
5 2 1 5

6 4 8 3 6 9 9
x1 = = = 4, x2 = = = .
1 2 2 1 2 2 2
3 4 3 4

2270
Appendix P

Vector Analysis

Rectangular Coordinates
f = f (x, y, z), ~g = gx i + gy j + gz k

f f f
f = i+ j+ k
x y z

gx gy gz
~g = + +
x y z

i
j k

~g = x y z
gx gy gz

2f 2f 2f
f = 2 f = + +
x2 y 2 z 2

2271
Spherical Coordinates
x = r cos sin , y = r sin sin , z = r cos

f = f (r, , ), ~g = gr r + g + g

Divergence Theorem. ZZ I
u dx dy = u n ds

Stokes Theorem. ZZ I
( u) ds = u dr

2272
Appendix Q

Partial Fractions

A proper rational function


p(x) p(x)
=
q(x) (x a)n r(x)
Can be written in the form
 
p(x) a0 a1 an1
= + + + + ( )
(x )n r(x) (x )n (x )n1 x
where the ak s are constants and the last ellipses represents the partial fractions expansion of the roots of r(x). The
coefficients are
1 dk p(x)
 
ak = .
k! dxk r(x) x=

Example Q.0.2 Consider the partial fraction expansion of


1 + x + x2
.
(x 1)3
The expansion has the form
a0 a1 a2
3
+ 2
+ .
(x 1) (x 1) x1

2273
The coefficients are
1
a0 = (1 + x + x2 )|x=1 = 3,
0!
1 d
a1 = (1 + x + x2 )|x=1 = (1 + 2x)|x=1 = 3,
1! dx
1 d2 1
a2 = 2
(1 + x + x2 )|x=1 = (2)|x=1 = 1.
2! dx 2
Thus we have
1 + x + x2 3 3 1
3
= 3
+ 2
+ .
(x 1) (x 1) (x 1) x1

Example Q.0.3 Consider the partial fraction expansion of

1 + x + x2
.
x2 (x 1)2
The expansion has the form
a0 a1 b0 b1
2
+ + 2
+ .
x x (x 1) x1
The coefficients are
1 1 + x + x2
 
a0 = = 1,
0! (x 1)2
x=0
1 d 1 + x + x2 2(1 + x + x2 )
   
1 + 2x
a1 = = = 3,
1! dx (x 1)2
x=0 (x 1)2 (x 1)3
x=0
1 1 + x + x2
 
b0 = = 3,
0! x2
x=1
1 d 1 + x + x2 1 + 2x 2(1 + x + x2 )
   
b1 = = = 3,
1! dx x2
x=1 x2 x3
x=1

2274
Thus we have
1 + x + x2 1 3 3 3
2 2
= 2+ + 2
.
x (x 1) x x (x 1) x1

If the rational function has real coefficients and the denominator has complex roots, then you can reduce the work
in finding the partial fraction expansion with the following trick: Let and be complex conjugate pairs of roots of
the denominator.
 
p(x) a0 a1 an1
= + + +
(x )n (x )n r(x) (x )n (x )n1 x
 
a0 a1 an1
+ + + + + ( )
(x )n (x )n1 x
Thus we dont have to calculate the coefficients for the root at . We just take the complex conjugate of the coefficients
for .

Example Q.0.4 Consider the partial fraction expansion of


1+x
.
x2 + 1
The expansion has the form
a0 a0
+
xi x+i
The coefficients are
 
1 1 + x 1
a0 = = (1 i),
0! x + i x=i 2
1 1
a0 = (1 i) = (1 + i)
2 2
Thus we have
1+x 1i 1+i
2
= + .
x +1 2(x i) 2(x + i)

2275
Appendix R

Finite Math

Newtons Binomial Formula.


n  
n
X k
(a + b) = ank bk
k=0
n
n(n 1) n2 2
= an + nan1 b + a b + + nabn1 + bn ,
2
The binomial coefficients are,  
k n!
= .
n k!(n k)!

2276
Appendix S

Physics

In order to reduce processing costs, a chicken farmer wished to acquire a plucking machine. Since there was no such
machine on the market, he hired a mechanical engineer to design one. After extensive research and testing, the professor
concluded that it was impossible to build such a machine with current technology. The farmer was disappointed, but
not wanting to abandon his dream of an automatic plucker, he consulted a physicist. After a single afternoon of work,
the physicist reported that not only could a plucking machine be built, but that the design was simple. The elated
farmer asked him to describe his method. The physicist replied, First, assume a spherical chicken . . . .
The problems in this text will implicitly make certain simplifying assumptions about chickens. For example, a
problem might assume a perfectly elastic, frictionless, spherical chicken. In two-dimensional problems, we will assume
that chickens are circular.

2277
Appendix T

Probability

T.1 Independent Events


Once upon a time I was talking with the father of one of my colleagues at Caltech. He was an educated man. I think
that he had studied Russian literature and language back when he was in college. We were discussing gambling. He
told me that he had a scheme for winning money at the game of 21. I was familiar with counting cards. Being a
mathematician, I was not interested in hearing about conditional probability from a literature major, but I said nothing
and prepared to hear about his particular technique. I was quite surprised with his method: He said that when he
was on a winning streak he would bet more and when he was on a losing streak he would bet less. He conceded that
he lost more hands than he won, but since he bet more when he was winning, he made money in the end.
I respectfully and thoroughly explained to him the concept of an independent event. Also, if one is not counting
cards then each hand in 21 is essentially an independent event. The outcome of the previous hand has no bearing on
the current. Throughout the explanation he nodded his head and agreed with my reasoning. When I was finished he
replied, Yes, thats true. But you see, I have a method. When Im on my winning streak I bet more and when Im on
my losing streak I bet less.
I pretended that I understood. I didnt want to be rude. After all, he had taken the time to explain the concept
of a winning streak to me. And everyone knows that mathematicians often do not easily understand practical matters,

2278
particularly games of chance.
Never explain mathematics to the layperson.

T.2 Playing the Odds


Years ago in a classroom not so far away, your author was being subjected to a presentation of a lengthy proof. About
five minutes into the lecture, the entire class was hopelessly lost. At the forty-five minute mark the professor had a
combinatorial expression that covered most of a chalk board. From his previous queries the professor knew that none
of the students had a clue what was going on. This pleased him and he had became more animated as the lecture had
progressed. He gestured to the board with a smirk and asked for the value of the expression. Without a moments
hesitation, I nonchalantly replied, zero. The professor was taken aback. He was clearly impressed that I was able to
evaluate the expression, especially because I had done it in my head and so quickly. He enquired as to my method.
Probability, I replied. Professors often present difficult problems that have simple, elegant solutions. Zero is the
most elegant of numerical answers and thus most likely to be the correct answer. My second guess would have been
one. The professor was not amused.
Whenever a professor asks the class a question which has a numeric answer, immediately respond, zero. If you are
asked about your method, casually say something vague about symmetry. Speak with confidence and give non-verbal
cues that you consider the problem to be elementary. This tactic will usually suffice. Its quite likely that some kind of
symmetry is involved. And if it isnt your response will puzzle the professor. They may continue with the next topic,
not wanting to admit that they dont see the symmetry in such an elementary problem. If they press further, start
mumbling to yourself. Pretend that you are lost in thought, perhaps considering some generalization of the result. They
may be a little irked that you are ignoring them, but its better than divulging your true method.

2279
Appendix U

Economics

There are two important concepts in economics. The first is Buy low, sell high, which is self-explanitory. The
second is opportunity cost, the highest valued alternative that must be sacrificed to attain something or otherwise
satisfy a want. I discovered this concept as an undergraduate at Caltech. I was never very interested in computer
games, but one day I found myself randomly playing tetris. Out of the blue I was struck by a revelation: I could be
having sex right now. I havent played a computer game since.

2280
Appendix V

Glossary

Phrases often have different meanings in mathematics than in everyday usage. Here I have collected definitions of
some mathematical terms which might confuse the novice.

beyond the scope of this text: Beyond the comprehension of the author.
difficult: Essentially impossible. Note that mathematicians never refer to problems they have solved as being difficult.
This would either be boastful, (claiming that you can solve difficult problems), or self-deprecating, (admitting
that you found the problem to be difficult).
interesting: This word is grossly overused in math and science. It is often used to describe any work that the author
has done, regardless of the works significance or novelty. It may also be used as a synonym for difficult. It has a
completely different meaning when used by the non-mathematician. When I tell people that I am a mathematician
they typically respond with, That must be interesting., which means, I dont know anything about math or
what mathematicians do. I typically answer, No. Not really.
non-obvious or non-trivial: Real fuckin hard.
one can prove that . . . : The one that proved it was a genius like Gauss. The phrase literally means you havent
got a chance in hell of proving that . . .

2281
simple: Mathematicians communicate their prowess to colleagues and students by referring to all problems as simple
or trivial. If you ever become a math professor, introduce every example as being really quite trivial. 1

Here are some less interesting words and phrases that you are probably already familiar with.

corollary: a proposition inferred immediately from a proved proposition with little or no additional proof

lemma: an auxiliary proposition used in the demonstration of another proposition

theorem: a formula, proposition, or statement in mathematics or logic deduced or to be deduced from other formulas
or propositions

1
For even more fun say it in your best Elmer Fudd accent. This next pwobwem is weawy quite twiviaw.

2282
Appendix W

whoami

2283
Figure W.1: Graduation, June 13, 2003.

2284
Index

a + i b form, 184 Bessel functions, 1622


Abels formula, 910 generating function, 1629
absolute convergence, 526 of the first kind, 1628
adjoint second kind, 1644
of a differential operator, 915 Bessels equation, 1622
of operators, 1314 Bessels Inequality, 1296
analytic, 361 Bessels inequality, 1340
Analytic continuation bilinear concomitant, 917
Fourier integrals, 1550 binomial coefficients, 2276
analytic continuation, 437 binomial formula, 2276
analytic functions, 2225 boundary value problems, 1109
anti-derivative, 473 branch
Argand diagram, 184 principal, 6
argument branch point, 270
of a complex number, 186 branches, 6
argument theorem, 501
asymptotic expansions, 1251 calculus of variations, 2060
integration by parts, 1263 canonical forms
asymptotic relations, 1251 constant coefficient equation, 1018
autonomous D.E., 992 of differential equations, 1018
average value theorem, 499 cardinality
of a set, 3
Bernoulli equations, 984 Cartesian form, 184

2285
Cartesian product computer games, 2280
of sets, 3 connected region, 240
Cauchy convergence, 526 constant coefficient differential equations, 930
Cauchy principal value, 634, 1548 continuity, 53
Cauchys inequality, 497 uniform, 55
Cauchy-Riemann equations, 367, 2225 continuous
chicken piecewise, 55
spherical, 2277 continuous functions, 53, 536, 539
clockwise, 241 contour, 240
closed interval, 3 traversal of, 241
closure relation contour integral, 465
and Fourier transform, 1552 convergence
discrete sets of functions, 1297 absolute, 526
codomain, 4 Cauchy, 526
comparison test, 529 comparison test, 529
completeness Gauss test, 536
of sets of functions, 1297 in the mean, 1296
sets of vectors, 34 integral test, 530
complex conjugate, 182, 184 of integrals, 1470
complex derivative, 360, 361 Raabes test, 535
complex infinity, 242 ratio test, 531
complex number, 182 root test, 533
magnitude, 185 sequences, 525
modulus, 185 series, 526
complex numbers, 180 uniform, 536
arithmetic, 193 convolution theorem
set of, 3 and Fourier transform, 1554
vectors, 193 for Laplace transforms, 1490
complex plane, 184 convolutions, 1490
first order differential equations, 803 counter-clockwise, 241

2286
curve, 240 equidimensional-in-x, 995
closed, 240 equidimensional-in-y, 997
continuous, 240 Euler, 940
Jordan, 240 exact, 782, 945
piecewise smooth, 240 first order, 773, 791
simple, 240 homogeneous, 774
smooth, 240 homogeneous coefficient, 786
inhomogeneous, 774
definite integral, 122 linear, 774
degree order, 773
of a differential equation, 774 ordinary, 773
del, 157 scale-invariant, 1000
delta function separable, 780
Kronecker, 34 without explicit dep. on y, 946
derivative differential operator
complex, 361 linear, 902
determinant Dirac delta function, 1041, 1298
derivative of, 905 direction
difference negative, 241
of sets, 4 positive, 241
difference equations directional derivative, 157
constant coefficient equations, 1174 discontinuous functions, 54, 1337
exact equations, 1168 discrete derivative, 1167
first order homogeneous, 1169 discrete integral, 1167
first order inhomogeneous, 1171 disjoint sets, 4
differential calculus, 48 domain, 4
differential equations
autonomous, 992 economics, 2280
constant coefficient, 930 eigenfunctions, 1330
degree, 774 eigenvalue problems, 1330

2287
eigenvalues, 1330 table of, 2254
elements Fourier series, 1330
of a set, 2 and Fourier transform, 1539
empty set, 2 uniform convergence, 1353
entire, 361 Fourier Sine series, 1345
equidimensional differential equations, 940 Fourier sine series, 1429
equidimensional-in-x D.E., 995 Fourier sine transform, 1563
equidimensional-in-y D.E., 997 of derivatives, 1564
Euler differential equations, 940 table of, 2255
Eulers formula, 189 Fourier transform
Eulers notation alternate definitions, 1544
i, 182 closure relation, 1552
Eulers theorem, 786 convolution theorem, 1554
Euler-Mascheroni constant, 1611 of a derivative, 1553
exact differential equations, 945 Parsevals theorem, 1557
exact equations, 782 shift property, 1559
exchanging dep. and indep. var., 990 table of, 2250, 2253
extended complex plane, 242 Fredholm alternative theorem, 1109
extremum modulus theorem, 500 Fredholm equations, 1027
Frobenius series
Fibonacci sequence, 1179 first order differential equation, 808
fluid flow function
ideal, 383 bijective, 5
formally self-adjoint operators, 1315 injective, 5
Fourier coefficients, 1291, 1335 inverse of, 6
behavior of, 1349 multi-valued, 6
Fourier convolution theorem, 1554 single-valued, 4
Fourier cosine series, 1344 surjective, 5
Fourier cosine transform, 1562 function elements, 437
of derivatives, 1564 functional equation, 389

2288
fundamental set of solutions homogeneous solutions
of a differential equation, 913 of differential equations, 902
fundamental theorem of algebra, 498
fundamental theorem of calculus, 125 i
Eulers notation, 182
gamblers ruin problem, 1166, 1175 ideal fluid flow, 383
Gamma function, 1605 identity map, 5
difference equation, 1605 ill-posed problems, 801
Eulers formula, 1605 linear differential equations, 911
Gauss formula, 1609 image
Hankels formula, 1607 of a mapping, 5
Weierstrass formula, 1611 imaginary number, 182
Gauss test, 536 imaginary part, 182
generating function improper integrals, 130
for Bessel functions, 1628 indefinite integral, 116, 473
geometric series, 527 indicial equation, 1201
Gibbs phenomenon, 1358 infinity
gradient, 157 complex, 242
Gramm-Schmidt orthogonalization, 1284 first order differential equation, 814
greatest integer function, 5 point at, 242
Greens formula, 917, 1315 inhomogeneous differential equations, 774
initial conditions, 796
harmonic conjugate, 372 inner product
harmonic series, 528, 564 of functions, 1288
Heaviside function, 797, 1041 integers
holomorphic, 361 set of, 2
homogeneous coefficient equations, 786 integral bound
homogeneous differential equations, 774 maximum modulus, 467
homogeneous functions, 786 integral calculus, 116
homogeneous solution, 793 integral equations, 1027

2289
boundary value problems, 1027 convolution theorem, 1490
initial value problems, 1027 of derivatives, 1490
integrals Laurent expansions, 627, 2225
improper, 130 Laurent series, 555, 2227
integrating factor, 792 first order differential equation, 808
integration leading order behavior
techniques of, 127 for differential equations, 1255
intermediate value theorem, 55 least integer function, 5
intersection least squares fit
of sets, 4 Fourier series, 1337
interval Legendre polynomials, 1285
closed, 3 limit
open, 3 left and right, 50
inverse function, 6 limits of functions, 48
inverse image, 5 line integral, 463
irregular singular points, 1216 complex, 465
first order differential equations, 812 linear differential equations, 774
linear differential operator, 902
j electrical engineering, 182 linear space, 1278
Jordan curve, 240 Liouvilles theorem, 497
Jordans lemma, 2226
magnitude, 185
Kramers rule, 2270 maximum modulus integral bound, 467
Kronecker delta function, 34 maximum modulus theorem, 500
LHospitals rule, 75 Mellin inversion formula, 1478
Lagranges identity, 917, 949, 1314 minimum modulus theorem, 500
Laplace transform modulus, 185
inverse, 1477 multi-valued function, 6
Laplace transform pairs, 1479 nabla, 157
Laplace transforms, 1475

2290
natural boundary, 437 of differential equations, 903
Newtons binomial formula, 2276 periodic extension, 1336
norm piecewise continuous, 55
of functions, 1288 point at infinity, 242
normal form differential equations, 1216
of differential equations, 1021 polar form, 188
null vector, 24 potential flow, 383
power series
one-to-one mapping, 5 definition of, 539
open interval, 3 differentiation of, 547
opportunity cost, 2280 integration of, 547
optimal asymptotic approximations, 1268 radius of convergence, 541
order uniformly convergent, 539
of a differential equation, 773 principal argument, 186
of a set, 3 principal branch, 6
ordinary points principal root, 199
first order differential equations, 803 principal value, 634, 1548
of linear differential equations, 1184 pure imaginary number, 182
orthogonal series, 1291
orthogonality Raabes test, 535
weighting functions, 1290 range
orthonormal, 1288 of a mapping, 5
ratio test, 531
Parsevals equality, 1340 rational numbers
Parsevals theorem set of, 3
for Fourier transform, 1557 Rayleighs quotient, 1426
partial derivative, 155 minimum property, 1426
particular solution, 793 real numbers
of an ODE, 1059 set of, 3
particular solutions real part, 182

2291
rectangular unit vectors, 24 series, 525
reduction of order, 947 comparison test, 529
and the adjoint equation, 948 convergence of, 525, 526
difference equations, 1177 Gauss test, 536
region geometric, 527
connected, 240 integral test, 530
multiply-connected, 240 Raabes test, 535
simply-connected, 240 ratio test, 531
regular, 361 residuals, 527
regular singular points root test, 533
first order differential equations, 806 tail of, 526
regular Sturm-Liouville problems, 1420 set, 2
properties of, 1428 similarity transformation, 1888
residuals single-valued function, 4
of series, 527 singularity, 377
residue theorem, 631, 2226 branch point, 377
principal values, 643 spherical chicken, 2277
residues, 627, 2225 stereographic projection, 243
of a pole of order n, 627, 2226 Stirlings approximation, 1613
Riccati equations, 986 subset, 3
Riemann zeta function, 528 proper, 3
Riemann-Lebesgue lemma, 1471
root test, 533 Taylor series, 550, 2226
Rouches theorem, 502 first order differential equations, 805
table of, 2241
scalar field, 155 transformations
scale-invariant D.E., 1000 of differential equations, 1018
separable equations, 780 of independent variable, 1024
sequences to constant coefficient equation, 1025
convergence of, 525 to integral equations, 1027

2292
trigonometric identities, 2264

uniform continuity, 55
uniform convergence, 536
of Fourier series, 1353
of integrals, 1470
union
of sets, 4

variation of parameters
first order equation, 795
vector
components of, 25
rectangular unit, 24
vector calculus, 154
vector field, 155
vector-valued functions, 154
Volterra equations, 1027

wave equation
DAlemberts solution, 1933
Fourier transform solution, 1933
Laplace transform solution, 1934
Webers function, 1644
Weierstrass M-test, 537
well-posed problems, 801
linear differential equations, 911
Wronskian, 906, 907

zero vector, 24

2293

You might also like